You are on page 1of 735

Yasser M.

Awaad

Absolute Pediatric
Neurology

Essential Questions
and Answers

123
Absolute Pediatric Neurology
Yasser M. Awaad

Absolute Pediatric
Neurology
Essential Questions and Answers
Yasser M. Awaad, M.D., M.Sc., FAAP, FAAN
Department of Pediatrics
University of Michigan
Ann Arbor, MI, USA

ISBN 978-3-319-78800-5    ISBN 978-3-319-78801-2 (eBook)


https://doi.org/10.1007/978-3-319-78801-2

Library of Congress Control Number: 2018944124

© Springer International Publishing AG, part of Springer Nature 2018


This work is subject to copyright. All rights are reserved by the Publisher, whether the whole or part of
the material is concerned, specifically the rights of translation, reprinting, reuse of illustrations, recita-
tion, broadcasting, reproduction on microfilms or in any other physical way, and transmission or infor-
mation storage and retrieval, electronic adaptation, computer software, or by similar or dissimilar
methodology now known or hereafter developed.
The use of general descriptive names, registered names, trademarks, service marks, etc. in this publica-
tion does not imply, even in the absence of a specific statement, that such names are exempt from the
relevant protective laws and regulations and therefore free for general use.
The publisher, the authors and the editors are safe to assume that the advice and information in this
book are believed to be true and accurate at the date of publication. Neither the publisher nor the
authors or the editors give a warranty, express or implied, with respect to the material contained herein
or for any errors or omissions that may have been made. The publisher remains neutral with regard to
jurisdictional claims in published maps and institutional affiliations.

Printed on acid-free paper

This Springer imprint is published by the registered company Springer International Publishing AG
part of Springer Nature
The registered company address is: Gewerbestrasse 11, 6330 Cham, Switzerland
I am dedicating this work to my beloved family, especially my father, Mahmoud Awaad,
mother, Galila Jad and children, Rania, Ammar and Emmad Awaad.

A very special dedication to my wife, Ola Abdellatif, who blessed me with unwavering
support throughout my life journey.

Yasser M. Awaad
VII

Contents

1 Autism����������������������������������������������������������������������������������������������������������������������������������������������������    1

2 Behavior������������������������������������������������������������������������������������������������������������������������������������������������  17

3 Genetic Disorders�����������������������������������������������������������������������������������������������������������������������������  29

4 Congenital�������������������������������������������������������������������������������������������������������������������������������������������� 117

5 Demyelinating Disorders�������������������������������������������������������������������������������������������������������������� 145

6 Developmental Disorders������������������������������������������������������������������������������������������������������������ 171

7 Eating Disorders�������������������������������������������������������������������������������������������������������������������������������� 185

8 Epilepsy 1��������������������������������������������������������������������������������������������������������������������������������������������� 187

9 Epilepsy 2��������������������������������������������������������������������������������������������������������������������������������������������� 237

10 Ethics and Professionalism���������������������������������������������������������������������������������������������������������� 333

11 Headache���������������������������������������������������������������������������������������������������������������������������������������������� 341

12 Infections���������������������������������������������������������������������������������������������������������������������������������������������� 377

13 Mental Retardation/Cognitive Disorders����������������������������������������������������������������������������� 409

14 Metabolic���������������������������������������������������������������������������������������������������������������������������������������������� 417

15 Movement Disorders���������������������������������������������������������������������������������������������������������������������� 443

16 Neuro-cutaneous ­Syndromes���������������������������������������������������������������������������������������������������� 467

17 Neurologic Complications of Systemic Disease���������������������������������������������������������������� 489

18 Neuromuscular���������������������������������������������������������������������������������������������������������������������������������� 503

19 Neuro-Oncology������������������������������������������������������������������������������������������������������������������������������� 553

20 Neuro-Ophthalmology������������������������������������������������������������������������������������������������������������������ 585

21 ­Neurophysiology/Localization�������������������������������������������������������������������������������������������������� 591
VIII Contents

22 Short Q/A���������������������������������������������������������������������������������������������������������������������������������������������� 611

23 Sleep Disorders���������������������������������������������������������������������������������������������������������������������������������� 623

24 Spinal Cord������������������������������������������������������������������������������������������������������������������������������������������ 643

25 Static Encephalopathy/Cerebral Palsy/Rehabilitation��������������������������������������������������� 659

26 Toxic Encephalopathy�������������������������������������������������������������������������������������������������������������������� 673

27 Traumatic Brain Injury�������������������������������������������������������������������������������������������������������������������� 687

28 Vascular������������������������������������������������������������������������������������������������������������������������������������������������� 705

Supplementary Information
Index�������������������������������������������������������������������������������������������������������������������������������������������������������� 731
1 1

Autism

© Springer International Publishing AG, part of Springer Nature 2018


Y. M. Awaad, Absolute Pediatric Neurology, https://doi.org/10.1007/978-3-319-78801-2_1
2 Chapter 1 · Autism

?? 1. A 24 months old boy is referred for mental disorders, 4th ed. Washington DC:
1 abnormal development and behavior. American Psychiatric Association; 1994.
He has no use of speech to communi- 55 Filipek PA, Accardo PJ, Ashwal G et al.
cate his needs, has a limited repertoire International classification of diseases:
of activities, and poor eye contact. He diagnostic criteria for research, 10th ed.
was diagnosed as autistic spectrum the World Health Organization. Practice
disorder by his primary care physician, parameter: screening and diagnosis of
but his parents are interested in find- autism: report of the Quality Standards
ing out his specific diagnosis. Subcommittee of the American Academy
Which of the following tests can be of Neurology and the Child Neurology
used to best confirm his diagnosis Society. Neurology. 2000;55:468–479.
and clarifying its type?
A. Blood lead level and serum IgG for ?? 2. A 19 months old boy is referred for
gluten sensitivity developmental delay evaluation.
B. Chromosomal testing for 16p11.2 He was born at 37 weeks gestation
deletion/duplication as repeat cesarean section, birth
C. Checklist for autism in toddlers weight of 3.0 kilograms. He rolled at
(CHAT) or modified checklist 6 months, walked at 11 months, and
(m-CHAT) said 3 words at 11 months. Currently
D. Denver developmental screening he does not speaks any words, does
test – revised (Denver II) not know body parts and will not
E. PDD assessment scale/ screening follow one-step commands. Sleep
questionnaire habits are very poor, with frequent
night awakenings. He has episodes of
vv Correct Answer is: C flapping movements of his hands and
Environmental factors might contribute arms, last 20–30 min, but disappear
in many children with autistic spectrum if he was picked up, and reappear if
disorders, such as lead exposure or celiac he was put down. He will not point to
disease, but testing for those factors do objects and has variable eye contact.
not diagnose ASD. It is not the first test Which of the following is the most
to do, 16p11.2 deletion/duplication is likely diagnosis?
associated with autism, and is not a test to A. Hearing loss
diagnose autism. The m-CHAT is a screen- B. Acquired auditory aphasia (Lan-
ing testing of both parental information dau Kleffner syndrome)
and examiner observations, and it is for C. Mental retardation
16–30 month toddlers. The Denver II is a D. Autistic disorder
screening device, and is no longer recom- E. Childhood ­schizophrenia
mended when screening for autism. The
PDD Assessment scale/ screening ques- vv Correct Answer is: D
tionnaire is not designed as a health care Diagnosis of autistic spectrum disorders
professional test. It is given to parents/ before the age of 2 years can be challeng-
caretakers for reaching the diagnosing of ing, and other conditions should to be
their children. considered. Hearing loss is very important
to rule out with formal hearing testing.
zz Suggested Reading This patient has a number of features that
55 Article on PDD scale on 7 Childbrain.com assist with making the diagnosis of autis-
(7 http://childbrain.com/pddq6.shtml). tic disorder, including restricted language
Diagnostic and statistical manual of and social skills and motor stereotypies.
Autism
3 1
zz Suggested Reading social interaction and the presence of
55 Zwaigenbaum L, Bryson S, Lord C, Rogers restricted, stereotyped patterns of behav-
S, Carter A, Carver L, Chawarska K, ior and interests. Asperger syndrome
Constantino J, Dawson G, Dobkins K, Fein patients often exhibit stereotyped behav-
D, Iverson J, Klin A, Landa R, Messinger iors and an intense preoccupation with
D, Ozonoff S, Sigman M, Stone W, a narrow activity or interest which is not
Tager-Flusberg H, Yirmiya N. Clinical required for diagnosis. In general, there is
assessment and management of toddlers no delay in cognitive or language devel-
with suspected autism spectrum disorder: opment, self-help or adaptive skills, or
insights from studies of high-risk infants. interest in the environment. Differential
Pediatrics. 2009;123(5):1383–91. diagnosis includes autism, pervasive
55 Wong V, Hui LH, Lee WC, Leung LS, Ho developmental disorder (not otherwise
PK, Lau WL, Fung CW, Chung specified), attention deficit hyperactivity
BA. Modified screening tool for autism disorder, obsessive compulsive disorder,
(Checklist for Autism in Toddlers [CHAT- selective mutism, childhood disintegra-
23]) for Chinese children. Pediatrics. tive disorder.
2004;114(2):e166–76.
zz Suggested Reading
?? 3. An 8-year-old boy with “behavioral 55 American Psychiatric Association.
problems” and an intense preoccupa- Diagnostic and statistical manual of
tion with trains that started at 3 years mental disorders DSM-IV-TR. 4th ed.
of age. There were no perinatal com- Washington DC: American Psychiatric
plications and normal early develop- Association; 2000.
mental milestones. He has no close 55 Khouzam HR, El-Gabalawi F, Pirwani N,
friends, and difficulties in social set- Priest F. Asperger’s disorder: a review of its
tings at school. He has no problem in diagnosis and treatment. Compr
speech and language development, Psychiatry. 2004;45(3):184–91.
but he is unusually verbose, tangen- 55 McPartland J, Klin A. Asperger’s syndrome.
tial or circumstantial, and at times Adolesc Med Clin. 2006;17(3):771–88.
exhibits inappropriate humor. He is
extremely sensitive to stimulation, ?? 4. The choices are genetic or environ-
and has had difficulty falling asleep mental risk factors for developing
with intermittent awakenings during autism spectrum disorders and/or
the night. He was inattentive during intellectual disabilities EXCEPT:
the evaluation, but his neurological A. More than 200 CGG trinucleotide
examination was normal. repeats
Which of the following is the most B. De novo mutations in the SCN2A
likely diagnosis? (sodium channel, voltage-­gated,
A. Autism type II, alpha), STXBP1 (syn-
B. Conduct disorder taxin binding protein 1), SCN1A
C. Asperger’s syndrome (sodium channel, voltage-gated,
D. Attention deficit hyperactivity dis- type I, alpha), or SYNE1 (synap-
order tic nuclear envelope protein 1)
E. Depression genes
C. TSC-1 (Tuberous Sclerosis Com-
vv Correct Answer is: C plex) gene mutations
Asperger syndrome is an autistic spec- D. Deletions within the MECP2 gene
trum disorder characterized by poor E. Older parental age
4 Chapter 1 · Autism

vv Correct Answer is: E E. The risk of epilepsy in children


1 Older paternal age at the time of birth has and adolescents with autism
been considered as a risk factor for devel- spectrum disorders ranges from
oping autism spectrum disorders (ASD), 6–26%, and it is higher in patients
no sufficient evidence to validate this with cerebral palsy or intellectual
concern. Greater than 200 CGG trinucleo- disabilities
tide repeats is diagnostic of Fragile X syn-
drome, it might have ASD manifestations vv Correct Answer is: E
and/or intellectual disability. TSC-1 muta- The incidence of epilepsy in children and
tions can cause Tuberous Sclerosis, which adolescents with autism spectrum disor-
often has concomitant diagnoses of ASD ders is higher in children with intellectual
and epilepsy. Deletions within the MECP2 disabilities and cerebral palsy. Mortality
gene are confirmatory of Rett Syndrome, rates increase in patients having autism
which can manifest with ASD, intellectual associated with epilepsy. The growing
disability, and epilepsy. evidence that epilepsies and ASD shares
common biological etiologies and mecha-
zz Suggested Reading nisms, increase interest and research in
55 Gardener H, Spiegelman D, Buka devising interventions that can change
SL. Prenatal risk factors for autism: the course of both diseases.
comprehensive meta-analysis. Pediatrics. It is difficult to detect seizures in
2011;128:344–55. patients with autism spectrum disorders,
55 Iossifov I, Ronemus M, Levy D, et al. De and acquiring electroencephalograms can
novo gene disruptions in children on be challenging in individuals with
the autistic spectrum. Neuron. 2012;74: behavioral challenges. With such individu-
285–99. als, utilization of behavioral interventions
55 Sebat J, Lakshmi B, Malhotra D, et al. can be helpful, and technologies utilizing
Strong association of de novo copy electrodes not requiring scalp abrasion,
number mutations with autism. Science. glues or pastes, allow for successful EEG
2007;316:445–9. acquisition, which can yield important
information about clinical and subclinical
?? 5. Which statement about autism spec- epilepsies.
trum disorders and epilepsy is true?
A. The incidence of epilepsy autism zz Suggested Reading
spectrum disorders is similar for 55 Amiet C, Gourfinkel-­AnI, Bouzamondo A,
those with mild, moderate or et al. Epilepsy in autism is associated with
severe manifestations intellectual disability and gender: evidence
B. The incidence and prevalence from a meta-analysis. Biol Psychiatry.
of epilepsy in autism spectrum 2008;64:577–82.
disorders is independent of con- 55 Bolton PF, Carcani-Rathwell I, Hutton J,
comitant, co-morbid neurological et al. Epilepsy in autism: features and
disorders/diagnoses correlates. Br J Psychiatry. 2011;198:289–94.
C. The biological and molecular mech- 55 McVicar KA, Ballaban-Gil K, Rapin I.
anisms causing or leading to epilep- et al. Epileptiform EEG abnormalities in
sies or autism spectrum disorders children with language regression.
are not similar and dependent Neurology. 2005;65:129–31.
D. EEG in autism spectrum disorders 55 Pickett J. et al. Mortality in individuals
is not a useful test because of with autism, with and without epilepsy. J
behavioral non-compliance Child Neurol. 2011;26:932–9.
Autism
5 1
?? 6. A 4 year old boy is here for a neuro- took VPA during pregnancy, autism risk
logical evaluation. You noted that he increased 3-fold and the risk of autism
does not have joint attention, makes spectrum disorder (ASD) increased 5-fold
minimal eye contact, has echolalia over the general population risk. The
and repetitively spins instead of absolute risk to VPA-exposed children
playing with the toys in the room. was less than 5%, although the upper
You consider the diagnosis of Autism 95% confidence interval (CI) limit for ASD
Spectrum Disorder. In taking the was 7.49%. There was no contribution of
history, the mother states that she maternal epilepsy to the risk of VPA use.
had taken valproic acid and topira- Hypospadias and spina bifida and occurs
mate for severe migraines during with first trimester exposure, but the
the pregnancy. The valproic acid was autism risk was present in this study with
added after the first trimester. She VPA exposure starting at any trimester
had also taken folic acid. She asked during pregnancy.”
you if any of the medications could In Thalidomide exposure, there was a
have contributed to the child’s con- 50 fold higher rate of Autism in those with
dition. thalidomide embryopathy than in the
Your response would be: general population.
A. Taking Valproic acid only increases The risk of oral clefts in infants
the risk of spinal tube defects exposed to topiramate during the first
B. Only thalidomide has been shown trimester of pregnancy has been increases
to increase the risk of Autism according the data from the North
C. Taking topiramate does not cause American Antiepileptic Drug (AED)
any birth defects Pregnancy Registry. Infants exposed to
D. The risk of autism increased 3-fold topiramate as a single therapy experi-
for children born to mothers who enced a 1.4% prevalence of oral clefts,
took VPA during pregnancy compared with a prevalence of 0.38–
E. The risk of autism is similar to 0.55% in infants exposed to other
spina bifida; it only increases dur- antiepileptic drugs.
ing first trimester exposure
zz Suggested Reading
vv Correct Answer is: D 55 Harden, C. In utero valproate exposure
Thalidomide and Valproic Acid have and autism: long suspected, finally proven.
been shown to increase the risk of Epilepsy Curr. 2013;13(6):282–4.
Autism in utero exposures. Valproic Acid 55 Christensen J, Grønborg TK, Sørensen MJ,
exposure can increase the risk of neural Schendel D, Parner ET, Pedersen LH,
tube defects is a well-known fact. The Vestergaard M. Prenatal valproate expo-
study by Christensen et al. and the com- sure and risk of autism spectrum disorders
mentary by Cynthia Harden revealed and childhood autism. JAMA.
there “were an association of in utero 2013;309(16):1696–703.
VPA exposure and the risk of autism 55 Strömland K, Nordin V, Miller M,
determined from a population-based Akerström B, Gillberg C. Autism in
study utilizing Danish national medical thalidomideembryopathy: a population
databases. Children born between 1996 study. Dev Med Child Neurol.
and 2006 were followed for the docu- 1994;36(4):351–6.
mented occurrence of autism up to 14 55 Risk of oral birth defects in children born
years of age, with a mean follow-up of to mothers taking topiramate. FDA NEWS
9 years. Children born to mothers who RELEASE, March 4, 2011.
6 Chapter 1 · Autism

?? 7. An 11-year-old boy came for a neu- 55 Sugden SG, Corbett BA. Autism-­
1 rologic evaluation because of “odd presentation, diagnosis and management.
behavior”. He and his family recently Continuum Lifelong Learning Neurol.
relocated from another state. He does 2006;12(5):47–59.
not maintain good eye contact when
you tried to engage him. His parents ?? 8. A 4-year-old boy presents with a
mentioned that he had no friends and speech delay. He was a full term,
is not very affectionate with family. He with no prenatal or perinatal
is methodical and rigid in his patterns, complications. He began walk-
getting upset if there is a change in ing at 13 months of age, is able to
his routine. Also they mentioned that feed himself with utensils. He says
he is fixated on following the statistics many different words during the
of his favorite football team. He does day; however most of this speech
very well in school. There is no history is repetition of things he hears on
of language or cognitive impairment. the television or from his family. He
Which of the following is the most has no interactive play and does
correct diagnosis? not identify body parts. He has no
A. attention deficit hyperactivity dis- chronic medical illnesses and takes
order no medication. No family history of
B. autism spectrum disorder developmental delays. There have
C. Landau-Kleffner syndrome been no difficulties with feeding
D. Rett disorder or swallowing. He does not have
E. Tourette syndrome eye contact and does not go to his
parents for reassurance. He makes
vv Correct Answer: B several grunting sounds; a brief
This patient has the main symptoms of echolalic speech is produced. When
autism spectrum disorder, including lack given blocks to stack, He arranges
of social interaction, as well as restrictive blocks in a straight line. The rest of
and repetitive behaviors. The new DSM his neurological exam is normal for
V eliminates the diagnosis of Asperger age. Hearing testing was normal one
syndrome (which fits this patient best month ago.
because of intact language function, Which of the following diagnoses is
which is characteristically impaired in most likely for the child’s language
DSM IV autism symptom complex) in favor delay?
of including these patients in the broader A. autism spectrum disorder
category of “autism spectrum disorder”. B. cerebral palsy
This patient’s presentation is not sugges- C. congenital perisylvian syndrome
tive of ADHD. Rett syndrome is associated D. Landau-Kleffner syndrome
with regression of language abilities and E. Menkes disease
hand stereotypies, typically seen in girls
(X-linked dominant). Tourette syndrome vv Correct Answer is: A
is a tic disorder, commonly with comorbid Delays in acquisition of language are com-
ADHD and OCD. mon in the pediatric population. Although
isolated expressive or mixed expressive
zz Suggested Reading and receptive language delays are two
55 Pellegrino L, Liptak GS. Consultation with prominent developmental disabilities
the specialist: asperger syndrome. Pediatr of childhood, often there are clues from
Rev. 2011;32(11):481–9. the history or physical exam that suggest
Autism
7 1
another possible diagnosis. You cannot in neuronal migration that results in a
rely on hearing test to be performed to bilateral thickened cortex (pachygyria)
exclude congenital or acquired hearing in the sylvian and Rolandic regions. All
loss, even if the newborn hearing screen affected children have not only a delay in
was normal. If, in addition to the language language (presumably from the disorga-
delay, there is an impairment of social nized cortex in the Broca region) but also
interaction (in this case suggested by a prominent pseudobulbar palsy causing
poor eye contact, lack of sharing atten- prominent dysphagia and feeding difficul-
tion with caregiver (interactive play), an ties. Epileptic encephalopathies such as
autism spectrum disorder is likely. Milder Landau-Kleffner syndrome (LKS) are quite
subtypes of the spectrum include perva- rare, especially in comparison to the 1:150
sive developmental disorder (PDD) and incidence of primary autism disorders,
Asperger syndrome. Patients with PDD though should be considered in a child
may have normal nonverbal intelligence, who develops typical language and social
and may have some segments of cognitive skills prior to age 2, and then undergoes a
ability that are quite advanced. Patients period of regression between 2–4 years of
with Asperger syndrome may have typi- age, often associated with development of
cally developed language skills, but are clinical seizures. Occasionally the regres-
quite impaired in the social arena. There sion will ensue prior to the appearance of
are multiple etiologies that may lead to an the ictal events. Electroencephalography
autistic phenotype, though in most cases in LKS demonstrates electrical status
a formal etiology is not discovered. When epilepticus of sleep (ESES). Treatment may
there is an associated severe intellectual be resistant to traditional anticonvulsants
disability present, testing to include chro- (such as valproic acid or levetiracetam)
mosome microarray, fragile X evaluation and may require high dose benzodi-
and MRI of the brain may be considered. azepines nightly and/or corticosteroid
The incidence of autism clearly is increas- treatment. Response of the electrical
ing with time, likely due to genetic fac- features of the syndrome usually leads to
tors, though other environmental factors improvement in the encephalopathy, but
have additionally been proposed. Large early recognition and aggressive manage-
meta-analytic studies have not shown ment are essential. Menke’s syndrome is
vaccination to be a causative factor. Early a disorder of copper metabolism and is
identification of the child with autism is apparent at birth with severe hypotonia
important, as the only clearly beneficial and sparse, brittle hair and early onset of
treatment is early institution of autism epilepsy. Seizures are also a commonly
specific therapy such as applied behav- associated problem. Although it is impor-
ioral analysis (ABA therapy). Even with tant to expand the diagnostic evaluation
early therapy, failure to develop language of the child with autistic-­like behavior
before the age of 5 is an extremely poor associated with significant loss of previ-
prognostic sign. Cerebral palsy is defined ously acquired skills (regression), many
specifically by static delays in motor con- of these patients will not have a specific
trol due to an early insult or malformation etiology discovered and will be diagnosed
of the brain. Though often associated with simply with a regressive form of autism.
language and cognitive delay, this diagno- Up to one-third of children with autism
sis strictly pertains to a motor disturbance experience a regression in skills (typically
(most commonly muscle spasticity). restricted to language and social skills and
Congenital bilateral perisylvian syndrome sparing motor function) between the ages
is a rare, usually sporadic, disturbance of 1 and 3 for reasons as of yet unknown.
8 Chapter 1 · Autism

zz Suggested Reading diagnostic criterion but may be an associ-


1 55 Fenichel GM. Clinical pediatric neurology: ated feature, although supernormal IQs may
a signs and symptoms approach. 6th ed. occur in some. Some improvements occur
Philadelphia: Saunders Elsevier; 2005. in adolescence, although seizures, mood
p.119–21. disorders, or other comorbidities may also
emerge. A genetic basis has been consid-
?? 9. A 4-year-old boy was seen by his ered based on the high concordance rates
primary care physician. The family among monozygotic twins and increased
mentioned that he is “emotionless” as incidence in families with another child with
compared with his affectionate siblings. autism. Neuropathology seen in patients
He never had eye contact. He does not with autism include under-­development
like to be hugged or kissed and does of limbic structures and reduced cerebellar
not play with others. He prefers to sit Purkinje cells. Other genetic and metabolic
alone in his room, playing with the rac- disorders have been associated with autism,
ing cars, although he has several other including neurofibromatosis, Down syn-
toys. He often flaps his arms at his sides drome, fetal alcohol syndrome, and peroxi-
and sometimes when upset will repeat- somal disorders. A diagnostic workup might
edly bang his forehead on the nearest be indicated for these disorders. Personality
piece of furniture or on a wall. He has disorder cannot be made at this time,
few words in his vocabulary, but not as although some of the personality disorders
many as his siblings did at his age. include rigid behavior.
What is your most likely diagnosis? This patient does not meet diagnostic
A. Autism criteria for depression and schizophrenia.
B. Avoidant personality disorder
C. Depression zz Suggested Reading
D. Schizophrenia 55 Comprehensive review in clinical neurol-
E. Schizoid personality disorder ogy: a multiple choice question book for
the wards and boards ©2011 Wolters
vv Correct Answer: A Kluwer Health Lippincott Williams &
The most likely diagnosis is autism. Autistic Wilkins. All rights reserved.
spectrum disorder is characterized by the
triad of impaired social skills, impaired com- ?? 10. A 20-month-old boy has been evaluated
munication skills, and restricted repertoire for evaluation severe expressive and
of activities and interests. Impaired com- receptive language delay. He cannot
munication skills include both verbal and point and pulls his parents around the
nonverbal skills. Social skill abnormalities house to indicate his needs. He has eye
include lack of eye contact or atypical eye contact occasionally. He lines up his toys
contact, failure to develop peer relation- and gets upset when they are taken out
ships, and lack of emotional reciprocity. of order. He flaps his hands when he is
Motor stereotypies, repetitive voluntary excited. On physical exam, he has two
behaviors such as those described here, hypo-pigmented macules on his back.
lack of flexibility, preoccupation with What is your next step in his evalua-
specific objects or part of an object, and tion?
ritualistic patterns of behavior occur. This A. Fragile X testing
spectrum is broad, ranging from high- B. High resolution chromosomes
functioning individuals as seen in Asperger C. EEG
disorder to nonverbal children with little D. Modified Checklist for Autism in
interaction and sometimes no language Toddlers
skills. Cognitive delay is not a part of the E. Skull X-Rays
Autism
9 1
vv Correct Answer is: D What testing would you do to help
The child clinical presentation is classical counsel the family?
for autism. Autism is a neurodevelopmen- A. Chromosomal microarray analysis
tal disorder characterized by impaired B. Mitochondrial point mutation
social communication, delayed expressive testing
and receptive language and stereotypic C. Head MRI
behavior. The language disorder is a delay D. EEG
in the development of spoken language or E. Serum amino acids
difficulties sustaining speech. This is also a
social disorder with marked difficulty in the vv Correct Answer is: A
use of nonverbal behaviors like gestures. Genetic studies in families indicate a 5%
There are restricted patterns of behavior recurrence rate. Although the cause of
like hand flapping, twisting or complex autism is not clear, most studies showed
body movements. There may be absence of that the cause is most likely genetic.
imaginative play. The cause for most cases Comparative genomic hybridization
of autism is unknown. Genetic disorders (microarray or oligo array) offers high
may be associated with autism spectrum resolution genome wide screening of seg-
disorders, Rett’s syndrome, Fragile X mental genomic copy number variations.
and tuberous sclerosis complex. Early Microdeletions and duplications can
identification of Autism will allow specific be detected by this technique. De novo
interventions to improve the outcome. The deletions and duplications offer insights
Modified checklist for autism in toddlers to this well-recognized disorder. Due to
is an easy tool to use for the pediatrician its increasing widespread availability
or primary provider. It is highly recom- it has offered insight to the etiology of
mended that a multidisciplinary team that autism and autistic spectrum disorders.
specializes in children with these disorders The most common abnormalities include
do the evaluation. The most common mea- maternally inherited duplication of
sures used include the Childhood Autism Chromosome 15, de novo or inherited
rating scales (trained observer), Gilliam duplications of 16p 11.2.
Autism rating scale (parents report). The
gold standard is the Autism Diagnostic zz Suggested Reading
Observation Schedule (ADDOS) or the 55 Shen Y, Dries KA, Holm IA et al. Clinical
Autism Diagnostic Interview-Revised (AID-­ genetic testing for patients with autistic
R). The hypo-pigmented spots raise con- spectrum disorders. Pediatrics. 2010.
cern of tuberous sclerosis, so an MRI might 125:e727–35.
be indicated.
?? 12. A 12-year-old boy with normal early
zz Suggested Reading developmental milestones comes to
55 De los Reyes EC, Roach ES. In: Cecil’s you for behavioral difficulties. He has
essential of medicine. Andreoli long history of difficulties interact-
T. Benjamin I, Griggs R, Wing E, editors. ing with peers. He prefers adults and
Philadelphia: Saunder Elsevier; 2010 younger children. Lately, he became
p. 1192–22. aggressive towards his classmates
and even hit another child who was
?? 11. The mother of a 5–year-old autistic making fun of him. He has excellent
boy is planning to have another child. grades in math but is failing in classes
She is asking you if there is a possibil- like English and reading. His favorite is
ity that her next child will be affected Robotics. He has normal exam except
with the same disorder. monotonous, dysprosodic speech.
10 Chapter 1 · Autism

What is your diagnosis? vv Correct Answer is: A


1 A. Asperger’s syndrome Early diagnosis is recommended in chil-
B. Sensory processing disorder dren with autistic spectrum disorders. The
C. Pervasive developmental disorder management of autism is based on the prin-
D. Mental retardation ciples of behavioral and educational inter-
E. Verbal apraxia vention. Applied Behavioral Analysis is the
process of applying interventions that are
vv Correct Answer is: A based on the principles of learning, derived
Asperger’s syndrome is a neurodevelop- from experimental psychology research,
mental disorder characterized by deficits to systematically change behavior and to
in social interactions, pragmatic language demonstrate that the interventions used
and restricted interests. Usually children are responsible for the observable improve-
with the syndrome have normal acquisition ment in behavior. ABA methods are used to
of early developmental language skills with increase and maintain desirable adaptive
average cognitive skills. Diagnosis is usu- behaviors, reduce interfering maladaptive
ally around 6 years of age since the increas- behaviors (or narrow the conditions under
ing social demands of school may lead which they occur), teach new skills, and
to evaluation of the atypical behaviors. generalize behaviors to new environments
Language is characterized as “pedantic”. or situations. Alternating program that is
Although verbal in specific topics of their well accepted is structured teaching or the
interest, Asperger patients may have dif- TEACCH program. The bases of the program,
ficulty expressing their feelings or relating emphasize structure and organization.
to others, especially with their peers. Other interventions include speech/lan-
guage therapy and occupational therapy.
zz Suggested Reading However, these are not case controlled
55 Levy SE, Hyma SL, Pinto-Martin J. Autism studies showed improvements on gluten
spectrum disorders. In: Accardo PJ, ed. and casein free diet. Media and certain
Neurodevelopmental disabilities in infancy parent groups have advocated the manage-
and childhood. Vol. 2, 3rd. Baltimore: ment of autism with multiple vitamins and
Brooke’s Publishing; 2008. p. 497. supplements, however, there are no case
55 Place-Johnson C, Myers SM, Council on controlled studies to determine efficacy.
children with disabilities. identification
and evaluation of children with autism zz Suggested Reading
spectrum disorders. Pediatrics. 2007. 55 Myers S, Place-Johnson C, Council on
p. 1182–1215. children with disabilities. Management of
children with autism spectrum disorders.
?? 13. The family of a 6-year-­old boy diag- Pediatrics. 2007. p. 1162–82.
nosed with autism would like to know 55 Filipek PA, Accardo PJ, Ashwal G et al.
what the best treatment interven- Practice parameter: screening and diagnosis
tions are for their son. Physical exam of autism: report of the Quality Standards
is normal. Subcommittee of the American Academy of
What is your advice to the family? Neurology and the Child Neurology
A. Behavioral intervention with Society. Neurology. 2000;55:468–79.
Applied Behavioral Analysis
B. Hyperbaric oxygen treatments ?? 14. You evaluate A 9 years-old boy with
C. Gluten and casein free diet a longstanding diagnosis of autism
D. Chelation therapy spectrum disorder-­moderate/severe.
E. Vitamin and mineral supplements His family describes occasional
Autism
11 1
“freeze episodes”, characterized by zz Suggested Reading
a general cessation of movement in 55 Ho A, Michelson D, Aaen G, Ashwal
a fixed posture, during which time S. Cerebral folate deficiency presenting as
he is awake, alert but is not interac- adolescent catatonic schizophrenia: a case
tive or conversant. He has a history report. J Child Neurol. 2010;25:898–900.
of seizures, happens approximately 55 Hyland K, Shoffner J, Heales SJ. Cerebral
once a year while maintained on folate deficiency. J Inherit Metab Dis.
lamotrigine. 2010;33:563–70.
All the mentioned below differential
diagnostic entities warrant consider- ?? 15. A 5-month-old baby has refractory
ation, which is most consistent with infantile spasms. A cardiac rhabdo-
the boy’s history: myoma was diagnosed prenatally.
A. GLUT-1 deficiency Her skin has multiple hypomelanotic
B. Dravet’s syndrome macules. Her Head MRI shows frontal
C. Landau-Kleffner syndrome cortical tubers. The EEG reveals multi-
D. Disorder of Cerebral Folate Metab- focal temporal regions discharges.
olism What symptom in her history puts
E. Fragile X syndrome her at risk for developing autism?
A. Temporal lobe focus
vv Correct Answer is: D B. Hypomelanotic macules
The history is consistent with recurrent C. Cardiac rhabdomyoma
episodes of catatonia-like episodes. D. Age of developing infantile spasms
Catatonia in individuals with autism E. Frontal cortical tubers
spectrum disorders has been reported to
be associated with Disorders of Cerebral vv Correct Answer is: A
Folate Metabolism, and in some cases Current evidence indicates that the likeli-
can be improved with the administration hood of a child with tuberous sclerosis
of folinic acid. GLUT-1 deficiency can developing an autism spectrum disorder
be associated with epilepsy and autism is greater if the child has a mutation in the
spectrum disorders, but usually intrac- TSC2 gene, although autism can and does
table epilepsies relatively uncontrolled develop in children with TSC1 mutations.
by antiepileptic drugs, but responsive to The likelihood is also greater if the child has
the ketogenic diet. Dravet’s Syndrome early-­onset infantile spasms that are dif-
is associated with genetic mutations of ficult to control, especially if there is an epi-
the SCN1A gene, and lamotrigine might leptiform focus in the temporal lobes. The
worsen the seizures. Landau-Kleffner emerging evidence is consistent with the
Syndrome is associated with language notion that early onset electrophysiological
regression and autism spectrum disor- disturbances within the temporal lobes has
ders, with epileptiform activity seeing a deleterious effect on the development
during sleep, and classically without and establishment of key social cognitive
observable seizure events. Fragile X representations concerned with processing
syndrome is associated with intellectual social information, especially from faces.
disability, and can have associated with
epilepsy or autism spectrum disor- zz Suggested Reading
ders. Other than Disorders of Cerebral 55 Bolton, PF. Neuroepileptic correlates of
Folate Metabolism, the other entities autistic symptomatology in tuberous
listed are not generally associated with sclerosis. Ment Retard Dev Disabil Res
catatonia. Rev. 2004;10(2):126–31.
12 Chapter 1 · Autism

?? 16. A 6-year-old boy is referred by his abnormal social approach and


1 school teacher because he is having failure of normal back-and-forth
difficulty with his peers. His mom conversation; to reduced sharing of
reported that he does not interact interests, emotions, or affect; to
with others. He only likes to talk failure to initiate or respond to social
about and play with the video game interactions.
Minecraft. He does not understand 2. Deficits in nonverbal communicative
peer teasing. He does not under- behaviors used for social interaction,
stand emotional cues. His mom ranging, for example, from poorly
states that he has a great vocabulary integrated verbal and nonverbal
and is doing well academically. On communication; to abnormalities in
examine, he has no eye contact, has eye contact and body language or
limited facial expression and does deficits in understanding and use of
not have joint attention. He cannot gestures; to a total lack of facial
engage in a back and forth conversa- expressions and nonverbal commu-
tion but tells you about Minecraft. nication.
You consider whether he fulfills the 3. Deficits in developing, maintaining,
criteria for an Autism Spectrum Dis- and understanding relationships,
order. ranging, for example, from difficul-
Which below choice is not included ties adjusting behavior to suit
in the DSM V Criteria for Autism various social contexts; to difficulties
Spectrum Disorder? in sharing imaginative play or in
A. Deficits in social-­emotional reci- making friends; to absence of
procity interest in peers.
B. Deficits in nonverbal communi- Specify current severity:
cative behaviors used for social Severity is based on social communica-
interaction tion impairments and restricted repetitive
C. Deficits in developing and main- patterns of behavior.
taining relationships, appropriate 2. Restricted, repetitive patterns of
to developmental level behavior, interests, or activities, as
D. Stereotyped repetitive movement, manifested by at least two of the
speech, or use of objects following, currently or by history
E. Delays in language (examples are illustrative, not exhaus-
tive; see text):
vv Correct Answer is: E 1. Stereotyped or repetitive motor
The DSM V that was published in 2013 movements, use of objects, or
revealed changes in the criteria for speech (e.g., simple motor stereoty-
ASD. The criteria no longer include lan- pies, lining up toys or flipping
guage delays. objects, echolalia, idiosyncratic
Diagnostic Criteria include: phrases).
1. Persistent deficits in social communica- 2. Insistence on sameness, inflexible
tion and social interaction across adherence to routines, or ritualized
multiple contexts, as manifested by the patterns or verbal nonverbal
following, currently or by history behavior (e.g., extreme distress at
(examples are illustrative, not exhaus- small changes, difficulties with
tive, see text): transitions, rigid thinking patterns,
1. Deficits in social-­emotional reciproc- greeting rituals, need to take same
ity, ranging, for example, from route or eat food every day).
Autism
13 1
3. Highly restricted, fixated interests meet criteria for autism spectrum disor-
that are abnormal in intensity or der, should be evaluated for social (prag-
focus (e.g., strong attachment to or matic) communication disorder.
preoccupation with unusual objects, Specify if:
excessively circumscribed or 55 With or without accompanying intellectual
perseverative interest). impairment.
4. Hyper- or hypo reactivity to sensory 55 With or without accompanying language
input or unusual interests in sensory impairment.
aspects of the environment (e.g., 55 Associated with a known medical or
apparent indifference to pain/ genetic condition or environmental factor.
temperature, adverse response to 55 Associated with another neurodevelop-
specific sounds or textures, excessive mental, mental, or behavioral disorder.
smelling or touching of objects, 55 With Catatonia.
visual fascination with lights or
movement).
Specify current severity: zz Suggested Reading
Severity is based on social communica- 55 Diagnostic and statistical manual of
tion impairments and restricted, repetitive mental disorders: DSM-5
patterns of behavior.
3. Symptoms must be present in the early ?? 17. A 4-month-old baby girl has refrac-
developmental period (but may not tory infantile spasms. Prenatally she
become fully manifest until social was found to have a cardiac rhabdo-
demands exceed limited capacities, or myoma. She has multiple hypomela-
may be masked by learned strategies in notic macules on her skin. Brain MRI
later life). reveals frontal cortical tubers. The EEG
4. Symptoms cause clinically significant reveals multifocal discharges; pre-
impairment in social, occupational, or dominantly in the frontal regions.
other important areas of current What factor puts her at risk for
functioning. developing autism?
5. These disturbances are not better A. Having refractory infantile spasms
explained by intellectual disability B. Having hypomelanotic macules
(intellectual developmental disorder) or C. Having a cardiac rhabdomyoma
global developmental delay. D. Having frontal cortical tubers
Intellectual disability and autism E. Having bifrontal discharges
spectrum disorder frequently co-occur;
to make comorbid diagnoses of autism vv Correct Answer is: A
spectrum disorder and intellectual Recent studies showed that the likeli-
disability, social communication should hood of a child with tuberous sclerosis
be below that expected for general developing an autism spectrum disorder
developmental level. is greater if the child has a mutation in the
Note: Individuals with a well-established TSC2 gene, although autism can and does
DSM-IV diagnosis of autistic disorder, develop in children with TSC1 mutations.
Asperger’s disorder, or pervasive develop- The likelihood will increase if the child has
mental disorder not otherwise specified early-onset infantile spasms that are dif-
should be given the diagnosis of autism ficult to control, especially if there is an epi-
spectrum disorder. Individuals who have leptiform focus in the temporal lobes. The
marked deficits in social communication, emerging evidence is consistent with the
but whose symptoms do not otherwise notion that early onset electrophysiological
14 Chapter 1 · Autism

disturbances within the temporal lobes has vv Correct Answer is: A


1 a deleterious effect on the development Language acquisition delays in the pedi-
and establishment of key social cognitive atric age group population is common.
representations concerned with processing Although isolated expressive or mixed
social information, perhaps especially from expressive and receptive language delays
faces. are two common developmental dis-
abilities of childhood, usually there are
zz Suggested Reading clues from the history or physical exam
55 Bolton, PF. Neuroepileptic correlates of that suggest another possible diagnosis.
autistic symptomatology in tuberous It is important for a hearing test to be per-
sclerosis. Ment Retard Dev Disabil Res formed to exclude congenital or acquired
Rev. 2004;10(2):126–31. hearing loss, even if the newborn hearing
screen was normal. If, in addition to the
?? 18. A 4-year-old boy with speech delay language delay, there is an impairment of
came to your office for an evaluation. social interaction (in this case suggested
He was born at full term, with no pre- by poor eye contact, lack of sharing atten-
natal or perinatal complications. He tion with caregiver (interactive play), an
walked at 14 months of age, is able autism spectrum disorder is likely. Milder
to feed himself with utensils. He uses subtypes of the spectrum include perva-
several words during the day; most of sive developmental disorder (PDD) and
them is repetition of things he hears Asperger syndrome. Patients with PDD
on the television or from his family. may have normal nonverbal intelligence,
He has no interactive play and does and may have some segments of cognitive
not identify body parts. He has no his- ability that are quite advanced. Patients
tory of chronic medical illnesses and with Asperger syndrome may have typi-
takes no medication. He has no family cally developed language skills, but are
history for any developmental delays. quite impaired in the social arena. There
There have been no difficulties with are multiple etiologies that may lead to an
feeding or swallowing. During his autistic phenotype, though in most cases
examination he does not make sus- a formal etiology is not discovered. When
tained eye contact and does not go to there is an associated severe intellectual
his parents for reassurance. He makes disability present, testing to include chro-
several grunting sounds; a brief echo- mosome microarray, fragile X evaluation
lalic speech is produced. When given and brain MRI may be considered. The
blocks to stack, he puts them in a incidence of autism is on the raise, likely
straight line. His neurological exam due to genetic factors, though other envi-
is normal for age. His hearing is nor- ronmental factors have additionally been
mal and confirmed by an audiology proposed. Vaccination is not a causative
examination one month ago. factor based on a large meta-analytic stud-
What is most likely diagnose for the ies. Early autism diagnosis is important,
boy’s language delay? as the only clearly beneficial treatment is
A. autism spectrum disorder early institution of autism specific therapy
B. Landau-Kleffner syndrome such as applied behavioral analysis (ABA
C. cerebral palsy therapy). Even with early treatment,
D. congenital perisylvian syndrome failure to develop language before the
E. Menke’s disease age of 5 is an extremely poor prognostic
Autism
15 1
sign. Epileptic encephalopathies such as that results in a bilateral thickened cortex
Landau-Kleffner syndrome (LKS) are quite (pachygyria) in the sylvian and Rolandic
rare, especially in comparison to the 1:150 regions. All affected children have not only
incidence of primary autism disorders, a delay in language (presumably from the
though should be considered in a child disorganized cortex in the Broca region)
who develops typical language and social but also a prominent pseudobulbar palsy
skills prior to age 2, and then undergoes a causing prominent dysphagia and feeding
period of regression between 2–4 years of difficulties. Seizures are also a commonly
age, often associated with development of associated problem. Menke’s syndrome
clinical seizures. Occasionally the regres- is a disorder of copper metabolism and is
sion will occur before the appearance of apparent at birth with severe hypotonia
the ictal events. EEG in LKS demonstrates and sparse, brittle hair and early onset
electrical status epilepticus of sleep of epilepsy. Although it is important to
(ESES). Treatment may be resistant to tra- expand the diagnostic evaluation of the
ditional anticonvulsants (such as valproic child with autistic-like behavior associ-
acid or levetiracetam) and may require ated with significant loss of previously
high dose benzodiazepines nightly and/or acquired skills (regression), many of these
corticosteroid treatment. Response of the patients will not have a specific etiology
electrical features of the syndrome usually discovered and will be diagnosed simply
leads to improvement in the encephalopa- with a regressive form of autism. Up to
thy, but early recognition and aggressive 1/3 of children with autism experience
management are essential. Cerebral palsy a regression in skills (typically restricted
is defined specifically by static delays in to language and social skills and sparing
motor control due to an early insult or motor function) between the ages of 1 and
malformation of the brain. Though often 3 for reasons as of yet unknown.
associated with language and cognitive
delay, this diagnosis strictly pertains to zz Suggested Reading
a motor disturbance (most commonly 55 Fenichel GM. Clinical pediatric neurology:
muscle spasticity). Congenital bilateral a signs and symptoms approach. 6th ed.
perisylvian syndrome is a rare, usually spo- Philadelphia: Saunders Elsevier; 2005.
radic, disturbance in neuronal migration p. 119–21.
17 2

Behavior

© Springer International Publishing AG, part of Springer Nature 2018


Y. M. Awaad, Absolute Pediatric Neurology, https://doi.org/10.1007/978-3-319-78801-2_2
18 Chapter 2 · Behavior

?? 1. A 7-year-old and 9-year-old brothers, to pay attention to details in schoolwork


are being seen for their annual checkup or other activities, difficulty sustaining
by their PCP. The mother looks worn attention on a task or activity, not listen-
2 out and mentioned that her sons have ing when spoken to and not following
instructions, difficulty with organiza-
always kept her busy. The younger
brother has a lot of trouble with his tion, avoidance or dislike of tasks that
school work. The mother said that he is require sustained mental activity, loss of
smart, but he is not paying attention to objects necessary for tasks or activities,
anything, cannot concentrate for more easy distractibility, and forgetfulness in
than few minutes on his homework, daily activities. The diagnostic criteria for
is easily distractible, and never does the hyperactive type include six or more
what he is told, is constantly misplac- symptoms of hyperactivity and impulsiv-
ing things, and even forgets to brush ity that have been present for the prior
his teeth and shower sometimes if she 6 months with at least some symptoms
does not remind him. The older brother occurring before age 7 years that are
also giving her trouble with his school leading to functional impairment and are
work, he “just can’t seem to sit still.” At not consistent with the developmental
home, he is fidgety at the dinner table level and include frequent fidgeting or
and while doing his homework. She was squirming, leaving a seat in situations
called several times to school because when remaining seated is expected, run-
he leaves his seat and sometimes the ning about or climbing excessively in
classroom. He talks constantly, jump- inappropriate situations, or feelings of
ing from one topic to another, he does restlessness, difficulty engaging quietly
not wait his turn to answer, and is con- in leisurely activities, talking excessively,
stantly butting into conversations. blurting out of answers, difficulty await-
What is your most likely diagnosis? ing a turn, and interruption of others.
A. This is a normal age appropriate Although the etiology of ADHD is not
behavior. clear, dysfunction in frontal-subcortical
B. Generalized anxiety disorder circuits has been implicated. Genetic
C. Oppositional defiant disorder studies have suggested involvement of
D. Attention-deficit/hyperactivity genes involved in dopamine action or
disorder metabolism, though environmental fac-
E. Conduct disorder tors play a role as well. Children of par-
ents with ADHD and siblings of children
vv Correct Answer is: D with ADHD are more likely to be affected
They have the classic symptomatol- with ADHD than the general population.
ogy of attention-deficit/ hyperactiv- Oppositional defiant disorder is a
ity disorder (ADHD). ADHD includes a distinct entity from ADHD, but the two
predominantly inattentive type, as in are often comorbid. Similarly, general-
the younger brother; a predominantly ized anxiety disorder is also comorbid
hyperactive type, as in the older brother with ADHD, as are the disruptive behav-
a combined type. The diagnostic criteria ior disorders. In childhood, academic
for the inattentive type include six or failure and peer rejection are the major
more symptoms of inattention that have consequences of ADHD, whereas in ado-
been present for the prior 6 months with lescence, there is a threefold increase in
at least some symptoms occurring before substance use and abuse. Approximately
age 7 years that are leading to functional 60% of patients with ADHD in child-
impairment and are not consistent with hood continue to be impaired in adult
developmental level and include failure life; ADHD may also not be recognized
Behavior
19 2
until adulthood. Adults with ADHD typi- and characteristic facial features—­
cally show instability with employment specifically a high anterior hair line;
and relationships. The first line of treat- frontotemporal hair sparsity, a long,
ment for ADHD are psychostimulants, thin face; frontal bossing; down slant-
including amphetamines and methyl- ing palpebral fissures; and a prominent
phenidates, of which there are various mandible. Advanced bone age is com-
oral preparations. Common side effects mon but is not a universal finding.
include reduced appetite, weight loss, Hypotonia and global developmental
insomnia, and headaches. An electrocar- delay is a neurologic hallmarks, however
diogram prior to initiation of stimulant mild intellectual disability is present
medications is recommended to exclude in the affected adults exhibiting. Sotos
underlying structural or conduction syndrome can be caused by either intra-
problems. genic truncating mutations or microde-
letions that result in haplo insufficiency
?? 2. An 8-year-old white boy came to your of the nuclear receptor binding SET
office for evaluation of developmental domain protein 1 (NSD1), located on
delay. Pregnancy was unremarkable chromosome 5q35.3. More than 80%
and he was a full-­term at birth. He accounts for intragenic mutations in
was at the regular nursery for 2 days European-­origin white patients, on the
because he had a feeding problem other hand microdeletions has a higher
and jaundice. He was 7 lbs. 8 oz. incidence in the Japanese population.
(25–50%ile) and length was 22 in. The majority of cases are sporadic,
(>95%ile) at birth. On your examina- though autosomal dominant pedigrees
tion, he is tall and macro-­cephalic have been reported.
(both > 97th percentile) with mildly
dysmorphic facial features, including zz Suggested Reading
a high, broad forehead, down-slant- 55 Leventopoulos G, et al. A clinical study of
ing palpebral fissures, and a pointed Sotos syndrome patients with review of the
chin. His hands and feet are enlarged. literature. Pediatr Neurol. 2009;40:357–64.
The rest of his neurologic examina-
tion is normal. No other similarly ?? 3. A 7-year-old girl with a history of
affected individuals in the family. language difficulties. She was born
Which genetic test would you like to full-term by normal spontaneous
order? vaginal delivery following an uncom-
A. Chromosome microarray plicated pregnancy. Her milestones,
B. FISH for chromosome 22q11 dele- walked at 10 months, said single
tion words at 12 months, and combined
C. Karyotype words at 22 months. At age 5, her
D. Methylation analysis of chromo- speech became progressively more
some 15q telegraphic, and she started pointing
E. Nuclear receptor SET domain con- or using sign language to indicate
taining protein-1 (NSD1 testing) her needs. On your neurologic exami-
nation, she is able to follow simple
vv Correct Answer is: D commands. She has word-finding
Sotos syndrome (cerebral gigantism), difficulties in spontaneous speech
is a genetic condition affecting 1: and on naming tasks and is unable to
10,000 to 1: 50,000 individuals. It is repeat short phrases. The rest of her
characterized by prenatal and post- neurologic examination is normal.
natal overgrowth, macrocephaly, Formal audiometric testing is normal.
20 Chapter 2 · Behavior

Her family states that she occasionally and production, eventually leading
stares off into space for 20 s to 1 min. to failure to respond to non-verbal
No convulsive seizure was reported. sounds and mutism 20–30% of patients
2 She has no family history of epilepsy.   do not have any clinical seizure activ-
ity at all. There is no clear relationship
Most likely her EEG is going to show?
A. Hypsarrhythmia with periods of between the severity of the seizures
electro decrement and the severity of the language defi-
B. Continuous, diffuse, polymorphic cits. EEG shows mainly bilateral pos-
theta and delta slowing terior temporal spikes or spike-wave
C. Bilateral periodic epileptiform dis- discharges that are activated by sleep.
charges (BiPEDs) The clinical seizures are usually easy
D. Electrical status epilepticus in to control, with spontaneous resolu-
slow wave sleep tion occurring in mid-adolescence.
E. Normal waking and sleep Treatment options include corticoste-
roids, levetiracetam, benzodiazepines,
vv Correct Answer is: D and valproic acid. Language function
In 1957, William L. Landau and Frank improves when the active phase of
R. Kleffner reported a “syndrome of spike-­wave discharges resolves; how-
acquired aphasia with convulsive dis- ever, if the aphasia begins in early
order in children” and postulated that childhood and/or persists for more
persistent convulsive discharges were than 1–2 years, long-term language
causing a functional ablation of brain deficits are almost universal.
areas concerned with language. Since
then, many childhood epileptic encepha- zz Suggested Reading
lopathy syndromes it has become 55 Smith MC, Hoeppner TJ. Epileptic
characterized by deterioration of one or encephalopathy of late childhood. J Clin
more cognitive functions with or without Neurophysiol. 2003;20(6):462–72.
motor, behavioral, and/or psychomo-
tor decline in association with an EEG ?? 4. A local school send a particularly
that shows strong activation of epileptic difficult student for consultation. A
activity during sleep. At this point, elec- 9 years old boy who has always been
trical status epilepticus of slow wave “hard-to-handle”. His teachers have
sleep (ESES) and continuous spikes and always assumed that his parents are
waves during slow wave sleep (CSWS) excessively lenient with him, par-
are considered synonymous terms, and ticularly because they lost his older
Landau-Kleffner syndrome (LKS) a dis- brother when this boy was 6 years
tinct presentation of ESES/CSWS in which old. His classroom teacher reported
acquired aphasia is the core symptom. that he refuses to do anything she
LKS classically presents with lan- asks and argues indiscriminately with
guage disturbance in normally devel- other children in the class. He is very
oping children aged 3 and 8 years. disruptive, throwing balled-up paper
Some have proposed that the language around the room while the children
disturbance is in fact a verbal audi- are supposed to be working on a sci-
tory agnosia (i.e. inability to compre- ence or math problem. She also men-
hend spoken language in the absence tioned that he seems quite bright, as
of hearing difficulties). Over time, he has an advanced vocabulary for
affected children experience a gradual age, including some words she would
deterioration in verbal comprehension rather never hear in the classroom.  
Behavior
21 2
You are going to evaluate him for: vv Correct Answer is: C
A. Attention deficit disorder Clinical history of maternal exposure
B. Oppositional-defiant disorder to alcohol and the physical appearance
C. Attention deficit disorder of the child can lead to the diagnosis of
D. Schizoaffective disorder fetal alcohol syndrome. The diagnostic
E. Autistic spectrum disorder criteria recommended by the Research
F. Obsessive-compulsive disorder Society of Alcoholism Fetal Alcohol
study group require: (1) Prenatal or
vv Correct Answer is: B postnatal growth retardation (2) CNS
Children with disruptive, antisocial dysfunction, developmental delay or
behavior should be evaluated for intellectual impairment; and (3) at least
oppositional-­defiant disorder. He might two of the following: m­ icrocephaly,
be showing difficulty paying attention microphthalmia and short palpebral fis-
or staying on task, but this is because sures, or all 3 and hypoplastic philtrum
of the intrusions of his need to “act out” with thin upper lip.
and oppose requests or needs of others.
The child in the case does not have the zz Suggested Reading
stero-typies or lack of social interac- 55 Menke J. Toxic and nutritional disorders.
tion of a child with an autism spectrum In: Menke J, Sarnat H, Maria BL, editors.
disorder. There is no description in this Child Neurology. 7th ed. Philadelphia:
case of a thought disorder or persevera- Lippincott Williams and Wilkins; 2006.
tive compulsions. p. 703–5.

zz Suggested Reading ?? 6. A 6-year-old boy came to your office


55 Althoff RR, Kuny-Slock AV, Verhulst FC, with his mother, who wants you to
Hudziak JJ, van der Ende J. Classes of start him on methylphenidate for
oppositional-defiant behavior: concurrent ADHD treatment. His medical his-
and predictive validity. J Child Psychol tory is remarkable for a speech delay,
Psychiatry. 2014;55(10):1162–71. impaired social functioning with
his school peers, restricted interests
?? 5. A 4–year-old girl brought to your office with a fascination for dinosaurs, and
for an evaluation for her developmen- hand flapping behaviors when he is
tal delay. She came with her maternal excited. In addition, his mother and
grandmother, who has custody of her teachers report that he has a very
because of parent neglect. Her physi- short attention span, poor impulse
cal exam showed, mild dysmorphic control, and hyperactive behavior.
features with a head circumference at Another brother has been diagnosed
the 5th percentile and height/weight with ADHD and has shown a dra-
at 10th percentile. She has moderate matic benefit from methylphenidate
to severe language delay. She has a treatment, with minimal adverse
thin upper lip with smooth philtrum.   effects, but he does not have the
Which of the following is the most same language and social difficulties
likely diagnosis? as this boy.  
A. Down’s syndrome Which of the following statements
B. Autism most accurately reflects the likeli-
C. Fetal alcohol syndrome hood of positive and negative medi-
D. Rett’s syndrome cation effects compared with his
E. Trisomy 18 brother’s response?
22 Chapter 2 · Behavior

A. Equally effective with equal side compared to her premorbid, devel-


effects opmentally age-appropriate state.
B. More effective but greater side During your evaluation she describes
2 effects episodes of non-command audi-
C. Less effective and greater side tory hallucinations. There is no past
effects medical history of head trauma or
D. Less effective but fewer side significant infectious illness. She is not
effects taking any medications. Family history
E. More effective with fewer side is non-contributory. She has a non-
effects focal neurologic examination, but you
noted occasional orofacial dyskinetic
vv Correct Answer is: C movements. There is a rise of 30 beats
Autism spectrum disorder (ASD) is per minute of her heart rate when
characterized by impairments in social going from lying to standing.  
functioning and communication, and Which of the following tests is most
restricted, repetitive behaviors and inter- likely to be diagnostic?
ests. On the other hand, ADHD is char- A. Anti-DNase type B antibody and
acterized by difficulties in maintaining ASO titers
attention and patterns of impulsive and/ B. Magnetic resonance angiography
or hyperactive behavior. The DSM-IV-TR of cerebral vessels
does not allow a comorbid diagnosis of C. Pelvic ultrasound
ASD and ADHD, because ASD is the more D. Thyroid function testing (T4, T3
severe disorder within the diagnostic and TSH) with thyroid-­peroxidase
hierarchy, so any inattention, hyperac- antibody (TPO) and anti-­
tivity, or impulsivity observed can be thyrotropin antibodies
thought of as originating from the pri- E. Fasting lactate and pyruvate
mary diagnosis of autism. This exclusion
fails to account for the many individu- vv Correct Answer is: C
als with ASD who also meet full criteria Onset of childhood acute or subacute
for ADHD; these individuals typically neuropsychiatric syndrome is indicative
have more severe overall dysfunctions. of Childhood Acute Neuropsychiatric
Studies have shown that patients with Syndrome (CANS, also known as PANS
both ASD and ADHD have a lower likeli- or PANDAS). In females, the clini-
hood of favorable response with stimu- cal findings of disinhibited behavior
lant treatment (25–50%) when compared patterns, agitation and psychosis,
to patients with isolated ADHD; the inci- associated with orofacial dyskinesias
dence of adverse effects in this popula- and autonomic dysfunction is indica-
tion is up to 60%, mainly agitation. tive of anti-NMDA-receptor antibody
encephalitis, a condition that can be
zz Suggested Reading associated with ovarian teratoma. This
55 Murray MJ. Attention-­deficit/hyperactivity entity can also present with altered
disorder in the context of autism spectrum level of consciousness or seizures. For
disorders. Curr Psychiatry Rep. NMDA-receptor antibody encephalitis
2010;12:382–8. associated with ovarian teratoma, sur-
gical resection of the tumor is usually
?? 7. 9-year-old female has a 6 month curative. Streptococcal-associated CANS
history of deteriorating school per- presents as Syndeham’s Chorea (a major
formance, disinhibited behavior, criterion of Rheumatic Fever, present-
and agitation, and out of character ing with acute-onset of chorea and
Behavior
23 2
obsessive-­compulsive disorder) or pos- zz Suggested Reading
sibly with “idiopathic” CANS. This latter 55 Jerrell JM, Hwang T-L, Livingston
category, initially reported as PANDAS, TS. Neurological adverse events associate
is described as presenting with acute with antipsychotic treatment in children
onset of obsessive-­compulsive symp- and adolescents. J Child Neurol.
toms as a primary criterion, although 2008;23:1392–9.
there may be an array of concomitant
neuropsychiatric symptoms, and tics as ?? 9. You evaluate an 8-year-­old fourth grade
a secondary criterion. Treatments can boy for academic problems. He has a
include antimicrobials directed at iden- normal prenatal and perinatal course,
tifiable infectious agents, and immuno-­ and early neurodevelopmental mile-
modulators. Hashimoto’s thyroiditis stones were achieved at an age appro-
more commonly presents in adoles- priate time. Adequate grades were
cence, and is commonly associated with obtained through 3rd grade. In 4th
more movement disorder phenomena grade, the mother and teachers have
(tremor, chorea, myoclonus, and ataxia). been complaining that he was inatten-
Identification of a non-­infectious or tive and poorly organized. His teacher
para-infectious encephalitis/encepha- reported that he has difficulty with
lopathy is important since treatment science and word problems, but good
interventions can successful, albeit with at reciting historical dates, and has
variable success rates across cohorts. trouble with math and reading compre-
hension. His family report that he can
zz Suggested Reading be awkward and has poor social skills,
55 Kaur S, Juneja M, Mishra D, Jain S. Anti- but is verbally expressive and verbose.  
N-methyl-D-aspartate receptor encephali- What is your next step in his care
tis: a case report and review of the and management?
literature. J Pediatr Neurosci. 2014;9:145–7. A. Head MRI
55 Singer HS, Gilbert DL, Wolf DS, et al. B. Trial of atomoxetine
Moving from PANDAS to CANS. J Pediatr. C. Trial of mixed amphetamine salts
2012;160:725–31. D. Psychiatric Consultation
E. Neuropsychological Evaluation

?? 8. One of your patients on risperidone vv Correct Answer is: E


(Risperdal).   This is the clinical presentation of a
Periodically you should monitor him non-­verbal learning disorder (NVLD),
for: which is characterized by discrepancies
A. Hemoglobin A-1-C between verbal and non-verbal IQ, dif-
B. Lipid panel (fasting) ficulties with conceptual learning and
C. Prolactin level (AM) abstract thinking, poor social skills/
D. Fasting blood glucose development, visual-spatial deficits,
E. All of the above and dyspraxia/incoordination. The
strength areas, good rote memory
vv Correct Answer is: F skills, strong auditory learning skills,
Neuroleptics side effects include the and eloquent expressive verbal abili-
induction of tardive dyskinesias and ties. Neuropsychological testing and
other extrapyramidal dysfunction, evaluation will provide additional
hyperprolactinemia, hyperlipidemia, information concerning this patient’s
and type-II diabetes. All above choices intellectual, cognitive and learning
are appropriate to monitor. profile, and will have a higher yield than
24 Chapter 2 · Behavior

psychiatric consultation in identify- with attention deficit disorder and he


ing a cognitive or learning disorder. was started on methyphenidate. The
Neuropsychological testing can empiri- dose was slowly titrated with a good
2 cally identify issues with cognitive pro- response in his inattentive symptoms.
cessing, executive function, memory, However, recently, the mother has
and emotional contributors that might noticed that he has seems more fidg-
be causing or contributing to this ety with repetitive eye blinking, shoul-
patient’s poor academic productivity. der shrugging, foot tapping and throat
In a struggling academic children, clearing. It is disruptive to his teachers.
there is a tendency to diagnosis ADHD In the office, he makes these move-
without a complete analysis of neu- ments and noises almost continuously,
ropsychological functioning to deter- but stops when he is being examined.  
mine if there are other reasons for a What is your next step in manage-
child’s poor academic function, and/ ment?
or to confirm a clinical impression of A. Discontinue methylphenidate
ADHD. Over diagnosing ADHD in the B. Brain MRI
United States has contributed to over- C. Obtain EEG
prescribing of psychostimulant drugs. D. Initiate fluoxetine
Although psychostimulants and non- E. Send antistreptolysin O titer
stimulants such as atomoxetine can be
useful for selected cases of individuals vv Correct Answer is: A
with ADHD, such a diagnosis should be Stimulants, specifically, methylphenidate
established utilizing objective, empiri- or amphetamine are the most commonly
cal data rather than strictly “clinical recommended medications for the
impressions”. Making or confirming a treatment of attention deficit disorder.
diagnosis of ADHD does not relay on the Behavioral therapy is also recommended.
positive responses to psychostimulant Approximately 15–30% of children expe-
medications. Since psychostimulant rience tics which are often transient.
medications can enhance neurologi- However some of these movements may
cal functioning in all people, with or be persistent and bothersome to the
without ADHD, overuse can occur. patient and families. In addition, 50% of
Medication use in those with ADHD patients with Tourette’s syndrome also
and associated academic productiv- have symptoms of attention deficit dis-
ity should be kept for selected cases. order.
Atomoxetine is also useful for selected
cases of ADHD, particularly in those zz Suggested Reading
with concomitant tics or anxiety. 55 American Academy of Pediatrics. Sub-
committee on attention deficit hyperactiv-
zz Suggested Reading ity disorder. Committee on Quality
55 Graf WD, Miller G, Nagel SK. Addressing Improvement.
the problem of ADHD medications as 55 Treatment of the School-age children with
neuroenhancements. Expert Rev Attention deficit hyperactivity disorder.
Neurother. 2014;14:569–81. Pediatrics. 2001;108(4):1033–44.

?? 10. The mother of an 8–year-old boy ?? 11. A10-year-old boy came to your office
comes to your office for evaluation of with his family for evaluation of
possible seizures. Four months ago frequent daydreaming. The family
he was diagnosed by his pediatrician mentioned that he is disorganized
Behavior
25 2
and tends to forget his homework. He ?? 12. A 4-year-old boy has language and
has trouble at school. His primary care social development delay, has a his-
physician requested an electroen- tory of stereotypical self-stimulating
cephalogram which was normal in the behaviors like rocking his body and
awake and asleep state. His physical flapping his hands. No family history
exam is unremarkable.   is available because he is adopted
What is the most appropriate next from Asia. He has microcephaly and
step in his evaluation? blonde hair.  
A. Connor’s questionnaire for par- Which of the following metabolic
ents and teachers conditions could be an explanation
B. High resolution chromosomes of his condition?
C. Head MRI with and without gado- A. Glutaric Acidemia type 1
linium B. Maple syrup urine disorder
D. Serum lactate C. Phenylketonuria
D. Zellweger syndrome (cerebro-
vv Correct Answer is: A hepatorenal syndrome)
Attention deficit disorder is an everyday E. Medium-chain acyl-­coenzyme A
problem seen by pediatricians, neurolo- dehydrogenase deficiency (MCAD)
gist and psychiatrist. It may present at
any period of a child’s life. Also there vv Correct Answer is: C
is an increasing awareness of this con- Autism is highly associated with
dition in the young adult. The main untreated phenylketonuria, and has the
symptoms are deficits in attention and greatest body of literature and evidence
vigilance as well as hyperactivity. The to support this notion. Many of the
DSM-4 diagnosis requires the presence others have early morbidity, or cause
of six out of the nine symptoms of inat- cognitive impairment without strong
tention and hyperactivity. There inat- association with autism.
tentive symptoms are inattention, poor
organization, forgetfulness, failing to zz Suggested Reading
give attention to details. The hyperac- 55 Muhle R, Trentacoste S, Rapin I. The
tive symptoms include talking exces- genetics of autism. Pediatrics.
sively, blurting out answers, frequent 2004;113(5):e472–86.
interruption and intrusion. No labora- 55 Chen CH, Hsiao KJ. A Chinese classic
tory or neurologic testing is indicated phenylketonuria manifested as autism. Br J
to diagnose ADHD unless there is an Psychiatry.1989;155:251–25.
abnormal exam. 55 Lowe TL, Tanaka K, Seashore MR, Young
EEG was normal in this case. JG, Cohen DJ. Detection of phenylketon-
Absence seizure is a differential diag- uria in autistic and psychotic children.
nosis for individuals who are noted to JAMA. 1980;243:126–8.
daydream. 55 Baieli S, Pavone L, Meli C, Fiumara A,
Coleman M. Autism and phenylketonuria.
zz Suggested Reading J Autism Dev Disord. 2003;33:201–4.
55 Pliska S, AACAP Work Group on Quality
issues. Practice parameter for the assess- ?? 13. A 15-year-old girl with primary gen-
ment and treatment of children and eralized epilepsy well controlled on
adolescents with attention- deficit/ lamotrigine has a sudden onset of
hyperactivity disorder. In J Am Acad Child multiple paroxysmal episodes unlike
Adolesc Pyschiatry. 2007;46(7) 894–921. her regular events. The mother
26 Chapter 2 · Behavior

described a sudden startled look, is not followed by post-ictal confusion


followed by whole-body stiffening or lethargyPresence of crying or shout-
and arching in conjunction with ing of obscenities.Eyes will be closed,
2 “swimming” movements of the and patients will tend to resist having
their eyes forced open.The diagnosis
upper extremities and thrashing
of the body. The events last 20 s to of psychogenic seizures made primar-
3 min in duration and are followed ily on video-­EEG documentation of
by somnolence and she cannot the non-epileptic nature of the clinical
remember the event. By history, episodes of concern. While considered
there are no trigger factors, and the definitive diagnostic test, video-EEG
lamotrigine serum level was normal. testing does not have perfect sensitiv-
A prolonged video-EEG is obtained, ity and specificity. Some non-lesional
showed occasional generalized focal epilepsies may be missed. 30–40%
polyphasic spike/wave discharges. of patients having psychogenic non-
One episode was captured during epileptic seizures a have true epileptic
the recording and is associated with seizures, making it extremely difficult to
muscle artifact, with resumption of ascertain what are or aren’t true epilep-
normal waking patterns after the tic events in the usual clinical settings,
event ends.   and even in the context of an emer-
What is the most appropriate next gency department visit. The presenta-
step in management? tion of the diagnosis of psychogenic
A. Switch to Valproic acid non-­epileptic seizures must be made
B. Obtain psychiatry consultation with great care and sensitivity. The pos-
C. Decrease dose of ­lamotrigine sibility of sexual abuse should always be
D. Start diphenhydramine considered.
E. Head MRI
zz Suggested Reading
vv Correct Answer is: B 55 Bodde NM, Brooks JL, Baker GA, Boon
Clinical presentation strongly suggests PA, Hendriksen JG, Aldenkamp
psychogenic non-epileptic seizures, AP. Psychogenic non-epileptic
which is a form of conversion disorder. seizures–diagnostic issues: a critical
These include the following:Episodes review. Clin Neurol Neurosurg.
are induced by stress, in certain settings, 2009;111(1):1–9.
or by specific individuals.Seizures recur 55 LaFrance WC Jr. Psychogenic nonepileptic
despite therapeutic antiepileptic drug seizures. Curr Opin Neurol.
levels. The ictus is of gradual onset and 2008;21(2):195–201.
of unusually prolonged duration. There
are thrashing/writhing movements, cry- ?? 14. A 10-year-old boy who is followed
ing and/or screaming, pelvic thrusting, regularly for ADHD has developed
and rolling side-to-side.Movements are frequent episodes of forced eye
intermittent, arrhythmic, out-of-­phase blinking and facial grimacing after
jerking, or with multiple periods of start- being started on a stimulant medi-
ing and stopping. cation. Although his parents report
There is no loss of consciousness that it is very helpful for improving
despite bilateral movements, and a his attention and reducing hyper-
lack of incontinence or cyanosis. This activity, they are worried about
Behavior
27 2
this eye blinking and the facial gri- taking her history, you find out that
macing and asking if he can be on her father was killed in a car accident
another medication. He is started on six months ago and she broke up with
atomoxetine.   her boyfriend after many verbal fights
What do you have to monitor during few months ago. She has been afraid
treatment? to drive a car and has had nightmares
A. Complete blood count about her father’s death. Her head-
B. Chest X ray aches are “all over my head” and con-
C. Suicidal ideation stant in nature. Their intensity does not
D. Serum sodium vary over the course of the day or from
E. Electrocardiogram day to day. Her neurological and gen-
eral physical examination are normal.  
vv Correct Answer is: C What is your diagnosis?
The boy developed tics with a trial A. Depression
of stimulants. Hence, the family has B. Post-traumatic stress disorder
requested another class of medica- C. Bipolar disorder
tions. Atomexetine, is a norepineph- D. Insomnia
rine reuptake inhibitor. Its side effects E. Learning disability
include weight loss, anorexia and
sleepiness. Acute liver failure has been vv Correct Answer is: B
reported as a rare complication. As Stressful events that lead to both emo-
with many neuroactive agents, there tional and physical consequences can be
is a black-­box warning about suicidal experienced by both children and adults.
ideation, which should be monitored While reactions to stressful events are usu-
throughout therapy. ally brief and associated with full recovery,
exposure to catastrophic or repeated
zz Suggested Reading traumatic events can lead to unconscious
55 Kinsbourne M, Wood F. Disorders of mental dissociation, an emotional numb-
mental development. In: Menkes JH, ing to suppress the pain and trauma. It is
Sarnat H, Maria B, editors. Child neurol- important to realize that, since younger
ogy. Philadelphia: Lippincott Williams and children have limited verbal skills and
Wilkins. 7th ed. p. 1133–1140. non-specific ways to react to stress, they
may not always meet DSM criteria for
?? 15. A 17-year-old girl is complaining of post-traumatic stress disorder. Children
chronic headaches, problems falling generally exhibit more emotional lability
asleep, upsetting dreams, and trouble after severe stress than do adults.
at school, all of which have been wors-
ening over the past several months. zz Suggested Reading
Her mother mentioned to you that 55 Carrion VG, Weems CF, Ray R, Reiss
her daughter has been losing interest AL. Toward an empirical definition
in school activities and in socializing of pediatric PTSD: the phenomenology
with her friends, and has been increas- of PTSD symptoms in youth. J Am
ingly irritable, moody, and angry. By Acad Child Adol Psych. 2002;41:166–73.
29 3

Genetic Disorders

© Springer International Publishing AG, part of Springer Nature 2018


Y. M. Awaad, Absolute Pediatric Neurology, https://doi.org/10.1007/978-3-319-78801-2_3
30 Chapter 3 · Genetic Disorders

?? 1. You are evaluating a 4-year-old boy C. The child has a congenital disor-
with a diagnosis of cerebral palsy. His der but exact determination of
mother reports that he has been slow risk of recurrence requires specific
in walking and talking and has abnor- genetic testing.
mal vision. His mother is 18-years-old D. The child has a congenital dis-
with a history of substance abuse. order with the risk of recurrence
3 There is no specific etiology for his being 25%.
CP. On examination, he has severe E. The child has a congenital disor-
hypotonia, ataxia and dysmetria. He der that is sporadic and the risk of
has nystagmus and frequently jerks recurrence is minimal.
his head to re-fixate. He does not talk
but frequently sighs and then takes vv Correct Answer is: D
deep breaths. His brain MRI is shown A child neurologist has to be fully
and read as normal. He has no retinal aware of brain imaging as well as the
response on electroretinogram. features both clinically and radiologi-
cally of a genetically based disorder
like Joubert Syndrom. The molar tooth
sign is a diagnostic feature, that is fre-
quently quite obvious, but not above
being overlooked by the inexperienced
radiologist or those not thinking about
the clinical issues. Gene definition
may be helpful particularly in confirm-
ing the diagnosis or recognizing the
subtypes. The recent identification of
retinal eletrophysiologic perturbation
adds another feature to the complexity
of Joubert.

zz Suggested Reading
55 Khan AO, et al. Ophthalmic features of
Joubert Syndrome. Ophthalmology.
2008;115:2286–9.
55 Valente EM, et al.

?? 2. You are seeing a 5-year-old girl with


a history of mental retardation and
..      Fig. 3.1 Axial T2-weighted MRI. (Courtesy of
Dr. Manikum Moodley and Dr. Gary Hsieh. Wikipedia)
lifelong seizures. Her seizures are
recurrent generalized and focal sei-
zures. She has poor seizure control in
What is the most accurate spite of using multiple anticonvulsant
­statement? agents. On examination, she has a
A. The problem is likely related to skull asymmetry with the right side
substance abuse, recurrence risk is being more prominent than the left.
low. OFC is 54 cm. Her face is also large.
B. The child has a disorder that will Some hyperpigmentation is noted on
lead to seizures and death with a the face and arm. She has hemiparesis
25% recurrence risk. on the left. EEG reveals prominent
Genetic Disorders
31 3
persistent seizure discharge on the been a matter of debate. Early seems
right and generalized slowing. Her better than later. Focality on EEG and
MRI is shown. consonant MRI abnormality, in the
uncontrolled epileptic should be suf-
ficient impetus for serious surgical
­consideration.

zz Suggested Reading
55 Di Rocco C, Battaglia D, Pietrini D, Piastra M,
Massimi L. Hemimegalencephaly: clinical
implications and surgical treatment. Childs
Nervous System. 2006;22(8):852–66.
55 Aicard J. Diseases of the Nervous System in
Childhood, 3rd ed. Mac Keith press; 2009.
p. 6.

?? 3. A 14-year-old girl in foster care with


no prior medical history came for an
evaluation. The foster parents believe
the child is autistic and perhaps men-
tally retarded. She drools excessively
and does not swallow well. She does
not socialize or smile. She has limited
speech. She has significant behavior
..      Fig. 3.2 With permission from ILAE.org.
Epilepsy­Diagnosis.org, Diagnostic Manual problem at home. On examination
she does not blink to light. She can-
not abduct either eye. She does not
What treatment option would you move her face.
use to achieve optimal seizure What is your most likely diagnosis?
­control? A. Poorly treated myasthenia gravis
A. Corpus Callosotomy B. Congenital myasthenic syndrome
B. Vagal nerve stimulator C. Athapaskan brain stem ­dysgenesis
C. Corticectomy D. Moebius syndrome
D. Hemispherectomy E. Fazio-Londe Disease
E. Ketogenic diet
vv Correct Answer is: D
vv Correct Answer is: D The non-smiling ophthalmolegic patient
Patients with uncontrolled seizures sec- with no history can present an interest-
ondary to hemimegencephaly should ing set of problems. Moebius classical
always be considered for surgery. The features are that of ophthamoparesis
problem sometimes is recognizing the (especially abducens paresis) and facial
disorder. Many patients are obvious diplegia. Lingual weakness and atrophy
but more subtle or partial hemimegen- may also be a significant cranial nerve
cephaly may be overlooked and the feature. Deafness is not a feature as it is
patient deprived of beneficial surgery. in Athapaskan brain stem dysgenesis.
Any patient with uncontrolled seizures Patients with Moebius syndrome may
should be looked at critically in this also have additional features which are
regard. The ideal age for surgery has important to know. They include scolio-
32 Chapter 3 · Genetic Disorders

sis, muscle hypoplasia, mental retarda- These radiological findings indicate


tion, hand and foot malformations. The Dandy-Walker malformation which is
psychological and social impact on the characterized by a dilated 4th ventricle
child who never smiles should not be with partial to complete agenesis of
overlooked. the cerebellar vermis. Hydrocephalus
almost always develops in the first
3 zz Suggested Reading year but on rare occasions does not
55 Verzijl HTFM, van der Zwaag B, Cruysberg develop until adulthood. Other fea-
JRM, Padberg GW. Moebius syndrome tures include elevation of the tento-
redefined: a syndrome of rhombencephalic rium and enlargement of the posterior
maldevelopment Neurology. 2003;61: fossa. Approximately 70% of cases are
327–33. ­associated with other CNS malforma-
tions including agenesis of the corpus
?? 4. You are seeing a full-­term baby girl callosum. The etiology is thought to
who was born after uneventful labor be due to improper closure of the
and delivery. Her prenatal ultrasound ventral neural tube with failure of the
performed at 34 weeks gestation foramina of Magendi to open. Despite
showed posterior fossa cyst. Her the ­presence of normal ­foramina
examination is normal and she is feed- of Luschka CSF flow is disrupted.
ing well. A brain MRI scan was ordered. Treatment requires shunting of both
What is the most accurate diagnosis? the lateral and 4th ventricles. There is
some suggestion that prognosis can be
predicted by the degree of agenesis of
the cerebellar vermis. The other causes
for cysts in the posterior fossa, listed
above, are not associated with dilation
of the 4th ventricle.

zz Suggested Reading
55 Klein O, Pierre-Kahn A, Boddaert N,
Parisot D, Brunelle F. Dandy-­Walker
malformation: prenatal diagnosis and
prognosis. Childs Nerv Syst. 2003;19:
484–89.

..      Fig. 3.3 Variant DWS with dysplasia of the pons ?? 5. You are seeing a full-­term infant
and cerebellum in an 8-year old. T2 weighted sagittal was with a normal vaginal delivery.
MRI. Hellerhoff. Wikipedia
The mother had trauma at 34 weeks
gestation, a prenatal ultrasound
A. Cerebellar hypoplasia and arach- performed at that time and showed
noid cyst enlarged ventricles and a question
B. Enlarged cisterna magna of holoprosencephaly. The infant
C. Dandy-Walker malformation has a normal general and neuro-
D. Type 1 Chairi ­malformation logical exam, without dysmorphic
E. Congenital cystic features. A T2-weighted MRI scan
­medulloblastoma is shown.
What is the most accurate
vv Correct Answer is: C ­diagnosis?
Genetic Disorders
33 3
malformations are linked to a poorer
prognosis. The etiology of this condition
continues to be debated and is most
often sporadic and linked to maternal
trauma, intrauterine vascular accidents
(such as the demise of a monozygotic
twin), and intrauterine viral infection
or is idiopathic. The incidence is esti-
mated at 1 in 100,000. Since rare familial
cases occur a genetic cause has been
sought. Initial reports of mutations in
the EMX2 homeodomain gene crucial
to CNS development have not been
found in further studies of groups of
children with schizencephaly. The major
manifestation is focal epilepsy and
hemiparesis in unilateral cases. Bilateral
cases can have a severe ­phenotype
that includes mixed forms of epilepsy
..      Fig. 3.4 With permission from ILAE.org. Epilepsy­ and spastic quadriparesis with severe
Diagnosis.org, Diagnostic Manual ­developmental retardation.

A. Lobar holoprosencephaly zz Suggested Reading


B. Schizencephaly 55 Denis D, Chateil JF, Brun M Brissaud O
C. Congenital hydrocephalus with Lacombe D Fontan D Flurin V Pedespan J.
arachnoid cyst Schizencephaly: clinical and imaging
D. Lissencephaly features in 30 infantile cases. Brain Dev
E. Agenesis of the corpus callosum 2000;22:475–83.
with colpocephaly 55 Tietjen I, Bodell A, Apse K, Mendoza AM,
Chang BS, Shaw GM, et al. Comprehensive
vv Correct Answer is: B EMX2 Genotyping of a Large
The MRI shows right side schizenceph- Schizencephaly Case Series. Am J Med
aly. The condition is defined as cleft(s) Genet 2007;143A:1313–16.
involving the cortex and white mater
from the ventricular ependyma to the ?? 6. You are evaluating a 6-month-old
pia. “Closed lip” schizencephaly refers to boy for his seizures which started at
a cleft with the edges fused and “open his second day of life. He was born
lip” refers to clefts as in this case with prematurely at 34 weeks gestation
CSF filling the space between the corti- by normal vaginal delivery. Baby
cal surfaces. The condition is thought to required oxygen after birth, but no
arise in the second month and is charac- artificial ventilation. His seizure was
terized by migration defects in the area consisted of lip-smacking move-
of the cortical ribbon lining the cleft. ments and arrest of behavior. Later
Other associated malformations includ- the seizures were on the left side of
ing absence of the septum ­pellucidum, his body. The mother has history of
agenesis of the corpus callosum and epilepsy treated with carbamazepine.
septo-optic dysplasia. These other He has developmental delay; he
34 Chapter 3 · Genetic Disorders

cannot reach for objects nor roll over. crogyric cortex. The most common loca-
On examination, he has inconsistent tion involves the insula and adjoining
visual tracking, generalized hypoto- pre/post central gyri. The clefts can be
nia, and fewer spontaneous move- unilateral or bilateral, “closed-lip” (small
ments of his left limbs compared to defect) or “open-lip” (large defect). The
the right. Brain MRI is shown. etiology is heterogeneous and can be
3 What is the most likely cause of his due to an early prenatal insult affect-
seizures? ing the germinal zone prior to neuronal
migration or can be genetic (mutation
in the homeobox gene Emx2 has been
identified in some cases). The presence
of gyri and sulci (gray matter) radiating
into the cleft aids in differentiating this
condition from a late intrauterine or
early postnatal MCA infarction. Patients
with unilateral closed-lip type can be
minimally affected or have hemipare-
sis and epilepsy. Those with open-lip
type, especially if bilateral, have mental
retardation, spasticity, epilepsy, and are
severely compromised.

zz Suggested Reading
55 Osborn AG, et al. Diagnostic imaging:
brain. Salt Lake City, Utah: Amerisys Inc.;
2004. P I1 70–3.

?? 7. You are seeing a 9-month-old baby


boy for his developmental delay. He
can roll over but cannot sit or scoot.
His past medical history is remarkable
..      Fig. 3.5 With permission from ILAE.org. Epilepsy­ for poor visual function and an oph-
Diagnosis.org, Diagnostic Manual thalmological assessment revealed
hypoplastic optic nerves. On exami-
A. Hemimegalencephaly nation, he has severed generalized
B. Maternal carbamazepine use hypotonia, limited visual fixation and
C. Perinatal hypoxic ischemic brain following, and nystagmus.
injury What other ­complications are you
D. Prenatal stroke expecting in this boy in the future?
E. Schizencephaly A. Deafness
B. Profound mental retardation
vv Correct Answer is: E C. Growth impairment
MRI demonstrate Schizencephaly, which D. Infantile spasms
is a congenital brain malformation char- E. Spastic quadriplegic cerebral
acterized by a cleft extending from the palsy
pial surface of the cerebral mantle to the
ventricle. The cleft is lined by polymi- vv Correct Answer is: C
Genetic Disorders
35 3
This is a case of Septo optic dysplasia but he does not track. Upon crying he
(Morsier syndrome). Optic nerve hypo- has symmetrical facial movements.
plasia is part of the variable phenotype Neck and shoulder girdle muscle tone
of Septo-optic dysplasia. This condition is decreased, but increased in the
is defined loosely by any combination of hands and ankles. DTRs are brisk, with
the triad of optic nerve hypoplasia, mid- bilateral ankle clonus. Axial T1 MRI is
line neuro-radiological abnormalities shown here.
(such as agenesis of the corpus callosum What is the most likely mechanism
and absence of the septum pellucidum)
and pituitary hypoplasia with conse-
quent hypopituitarism. The reported
incidence is 1/10,000 live births and
affects males and females equally.
Neurological deficit is common rang-
ing from global developmental delay
to focal deficits such as epilepsy or
hemiparesis. Approximately 75–80% of
patients exhibit optic nerve hypoplasia
(bilateral in 88%) and is often the first
presenting feature, with later onset of
endocrine dysfunction. Hypothalamic-­
pituitary dysfunction may manifest as
variable endocrine deficits ranging from
isolated growth hormone deficiency
(most common) to pan-hypopituitarism.
Although neurological manifestations
often occur, those listed above are not
typically seen, while pituitary dysfunc-
tion is found in 62% of these patients. ..      Fig. 3.6 Image of a baby with hydrancephaly, light
passes through the skull indicating lack of forebrain.
(Courtesy of D. P. Agamanolis 7 http://neuropathology-
zz Suggested Reading
web.org“. Wikipedia)
55 Kelberman D, Dattani MT. Septo-Optic
Dysplasia- Novel Insights into the Aetiol-
ogy. In: Horm Res 2008;69:257–65. to account for her condition?
A. Congenital infection
?? 8. You are evaluating a 7-month-old B. Inborn error of metabolism
boy for developmental delay and C. Migration brain anomaly
seizure-like spells. He was a product D. Perinatal hemorrhage
of uncomplicated full-term gestation. E. Prenatal ischemia
He is unable to roll over or sit, does
not consistently visually regard or vv Correct Answer is: E
track objects, and does not reach for This is a case of hydranencephaly, which
objects with his hands, but does put is characterized by severe dysgenesis
his hands in the mouth. On examina- of the cerebral hemispheres, with
tion, head circumference is at 5th relative preservation of the thalamus,
percentile. There are no dysmorphic cerebellum and brainstem. Advanced
features. Pupils are reactive to light neurological imaging techniques have
36 Chapter 3 · Genetic Disorders

showed that the affected regions are


usually localized within the territories
of the middle cerebral arteries. It has
been suggested that hydranencephaly
results from vasculopathy with carotid
artery occlusion. This hypothesis
3 was supported by the distribution of
the marked hemispheric destruction
observed in patients with hydranen-
cephaly. Experiments entailing ligation
of both the common carotid arteries
and the jugular veins in monkeys during
early gestation resulted in subsequent
cerebral dysgenesis and focal destruc-
tion, with the regions supplied by the
anterior and middle cerebral arteries
being reduced to thin transparent mem-
branes. The tissues supplied by the pos-
terior cerebral arteries, which receive
..      Fig. 3.7 With permission from ILAE.org. Epilepsy­
collateral blood flow from the vertebro-
Diagnosis.org, Diagnostic Manual
basilar system through the posterior
communicating arteries, were usually
mainly preserved. These findings sup- What is the most accurate descrip-
port the proposed pathologic process in tion of the condition causing her
hydranencephaly. seizures?
A. Band heterotopias
zz Suggested Reading B. Idiopathic
55 Tsai JD, Kuo HT, Chou IC. Hydranen- C. Pachygyria
cephaly in Neonates. In: Pediatric Neonatal D. Polymicrogyria
2008;49(4):154–7. E. Schizencephaly

?? 9. You are seeing a 9-year-old girl with a vv Correct Answer is: D


2 years history of seizure. Her seizures MRI shows poly-microgyria, which is
are described as unresponsiveness one of the most common malforma-
and lip smacking, sometimes also tions of cortical development. In this
with jerking movements of the right condition, the brain surface is irregular
arm. Her seizures have been partially and the normal gyral pattern is replaced
controlled with carbamazepine, by an excessive number of small and
lamotrigine, and levetiracetam. She partly fused gyri separated by shallow
has a remarkable family history of sulci. It is causally and histologically
generalized tonic-­clonic seizure in her heterogeneous, resulting from a devel-
maternal uncle, started at her teens. opmental disorder or injury that occurs
She has unremarkable past medical toward the end of the period of neuro-
history and she has normal develop- nal migration and in the early phase of
ment. She has normal examination. cortical organization. Causes include
Her MRI is shown here. congenital infection (particularly CMV),
localized or diffuse in utero ischemia,
Genetic Disorders
37 3
or mutations. It can be focal or diffuse, independently. He does not reach
unilateral (40%) or bilateral (60%). It for objects and does not consistently
can occur as an isolated lesion, in asso- visually track objects. On examina-
ciation with other brain malformations tion, he has poor head control,
such as heterotopias or white matter increased tone and reflexes in all
lesions, or as part of several multiple limbs, and lack of visual regard.
congenital anomaly/mental retarda-
tion syndromes. The severity of clinical
presentation and age at presentation
depend mostly upon the extent of
cortical involvement; bilateral involve-
ment and involvement of more than
half of a single hemisphere are poor
prognostic indicators, portending mod-
erate to severe developmental delay
and significant motor dysfunction. The
most common location is around the
Sylvain fissure. Several distinct patterns
of bilateral polymicrogyria have been ..      Fig. 3.8 With permission from ILAE.org. Epilepsy­
described, including frontal, front pari- Diagnosis.org, Diagnostic Manual
etal, perisylvian, lateral parietal, para-
sagittal parietooccipital (as in this case), What is most likely genetic mutation
and generalized Genetic mutation responsible for her condition?
involving GPR56 has been identified A. ARX
in patients with bilateral front parietal B. DCX
form of polymicrogyria and is inherited C. LIS1
in an autosomal recessive manner. D. MECP2
E. RELN
zz Suggested Reading
55 Barkovich AJ. Current concepts of polymi- vv Correct Answer is: C
crogyria. Neuroradiology. 2010;52:479–87. MRI shows characteristic of type 1
55 Jansen A, Andermann E. Genetics of the lissencephaly. Lissencephaly is a
polymicrogyria syndromes. J Med Genet. group of disorders characterized by
2005;42:369–78. an abnormally smooth surface of the
55 Chang B, Walsh CA, Apse K, Bodell cerebral cortex. Type 1 is also called
A. Polymicrogyria overview. GeneReviews “classic lissencephaly” while type 2 is
[Internet], 7 www.ncbi.nlm.nih.gov/books/ known as “cobblestone complex”. This
NBK1329/. severe brain malformation is associ-
ated with mutations in the LIS1 gene
?? 10. You are evaluating a 5-month-old boy on chromosome 17p13.3. Mutations in
who developed of infantile spasms this gene are associated with 2 clinical
for the last 2 months. The spasm has entities: Miller-Dieker syndrome and
been persistent despite treatment isolated lissencephaly sequence. The
using Phenobarbital, levetiracetam, former is characterized by dysmorphic
topiramate, and clonazepam. He facial features including a prominent
has global developmental delay and forehead, bi-temporal hollowing,
has been unable to roll over or sit and short nose with upturned nares,
38 Chapter 3 · Genetic Disorders

protuberant upper lip, thin vermillion


border, and small jaw. DCX, also known
as doublecortin, is a gene located on
the X chromosome and its mutation
in male’s causes’ lissencephaly, but in
female’s leads to a much milder pheno-
3 type, including subcortical band het-
erotopias (double cortex syndrome).
Both ARX and RELN are associated with
“variant lissencephaly”. ARX muta-
tions are associated with agyria or
pachygyria with posterior to anterior
gradient of severity, thick cortex (not
as thick as LIS1), and agenesis of the
corpus callosum, abnormal white mat-
ter signals, and poorly defined basal
ganglia. RELN is associated with gen-
..      Fig. 3.9 MRI of the head. Hyperintense basal
eralized pacchygyria, severe cerebellar ganglia lesions on T2-weighted images. (de Gurrola GC,
hypoplasia, and hippocampal abnor- Araúz JJ, Durán E, Aguilar-Medina M, Ramos-­Payán R,
malities. MECP2 is associated with Rett García-Magallanes N, Pacheco GV, Meraz EA. Kernic-
syndrome. terus by glucose-6-phosphate dehydrogenase
deficiency: a case report and review of the literature.
Journal of Medical Case Reports 2008, 2:146 7 https://
zz Suggested Reading doi.org/10.1186/1752-1947-2-146. Gladys Cossio de
55 Mochida GH. Genetics and Biology of Gurrola et al.)
Microcephaly and Lissencephaly. Semin
Pediatr Neurol. 2009;16:129–26. What is the most likely explanation
for her neurological condition?
?? 11. You are seeing a 35-month-old A. Hypoxic-ischemic encephalopathy
South American girl for her delayed B. Vitamin and nutritional deficiency
­development. She had no prenatal C. Leigh syndrome
care. Her birth weight was 3 kg. D. Kernicterus
Delivery was complicated but the E. Methylmalonic aciduria
child had respiratory distress and
jaundice in the neonatal period. vv Correct Answer is: D
Subsequently she had trouble swal- Nowadays, this condition has become
lowing and was followed carefully relatively uncommon. Few cases still
for nutritional deprivation. She is not have been seen. This patient’s neonatal
regressing. She can cruise, but does experience was unique but her clinical
not talk. Her examination shows a presentation of kernicterus was not. The
well-nourished child who is alert hypotonia, choreoathetosis, and lack of
and sociable. She is hypotonic and is speech suggesting hearing impairment
able to stand with support but has were all very characteristic and the diag-
poor balance. She has prominent nosis was confirmed by the MRI, which
choreic and dystonic movements of showed increased T2 signal in the glo-
the extremities. DTRs are intact and bus pallidus. You have to be aware that
plantar responses are down going. a general radiologist reading the MRI
Her MRI is shown. may easily miss the diagnosis.
Genetic Disorders
39 3
zz Suggested Reading synaptic plasticity. The frequency of
55 Shapiro SM. Definition of the clinical PTEN mutations as a cause of autism
spectrum of kernicterus and bilirubin- and macrocephaly range from 1% to
induced neurologic dysfunction (BIND). 17%. Both de novo and familial PTEN
J Perinatol. 2005;25(1):54–9. mutations have been identified in this
population. Children with ASD found
?? 12. You are evaluating a 4-year-old to have a PTEN mutation generally
because his mother and his Head have extreme macrocephaly ranging
Start teacher are concerned about from + 3.7 SD to + 9.6 SD (average:
his difficulty relating with others. He +5.4 SD). In addition, mutation of PTEN
wanders around the room and hums is not specific for autism: Varga et al.
during story time. He will flap his arms [2009] found children with macro-
and spin when he is excited. He has cephaly and intellectual disability but
a moderate speech delay and will be not autism who had a similar chance of
evaluated by speech therapy. He is a having a PTEN mutation.
picky eater; he only eats chicken nug-
gets and French fries. On examination zz Suggested Reading
he has macrocephaly with an OFC of 55 Miles JH. Autism Spectrum Disorders.
57 cm. GeneReviews. 1993–2014.
Mutations in which of the follow- 55 Macrocephaly/Autism Syndrome. OMIM #
ing gene or chromosome regions 605309.
are most likely responsible for the
child’s symptoms? ?? 13. Ocular manifestations are common in
A. FMR1 Lysosomal storage diseases.
B. PTEN The Lysosomal storage disease that
C. 22q11.2 deletion is least likely to have a macular
D. ARX cherry red spot is:
E. 17p11.2 deletion A. Silicosis
B. GM 1 Gangliosidoses
vv Correct Answer is: B C. Krabbe disease
The boy has symptoms consistent D. Farber disease
with Macrocephaly/Autism Syndrome E. Niemann-Pick disease
caused by mutations in the PTEN Gene.
The PTEN (phosphatase and tensin vv Correct Answer is: C
homolog) gene was initially described Krabbe disease is a lysosomal storage
as a tumor suppressor gene associ- disease characterized by infantile-
ated with a broad group of disorders onset progressive neurologic deterio-
referred to as PTEN hamartoma tumor ration and death before age 2 years
syndrome which includes Cowden (85–90% of individuals) or by onset
syndrome, Bannayan-Riley-Ruvalcaba between age 1 year and the fifth
syndrome, Proteus syndrome, and decade with slower disease progres-
Lhermitte-Duclos disease. More sion (10–15%). Children with the
recently, PTEN gene mutations have infantile form appear to be normal for
been associated with autism and mac- the first few months of life but show
rocephaly. PTEN is recognized to play extreme irritability, spasticity, and
an important role in brain develop- developmental delay before age 6
ment, including neuronal survival and months; psychomotor regression pro-
40 Chapter 3 · Genetic Disorders

gresses to a decerebrate state with no


voluntary movement. The onset and
progression in the late-onset forms
can be quite variable. Individuals can
be clinically normal until weakness,
vision loss, and intellectual regression
3 become evident. The onset of symp-
toms and clinical course can be vari-
able even among siblings.
In almost all individuals with Krabbe
disease, galactocerebrosidase (GALC)
enzyme activity is deficient (0–5% of nor-
mal activity) in leukocytes isolated from
whole heparinized blood or in cultured
skin fibroblasts.
Individuals with the infantile form of
Krabbe disease can present with any or
..      Fig. 3.10 With permission from ILAE.org.
all of the following features. However, it is EpilepsyDiagnosis.org, Diagnostic Manual
the least likely of all of the LSDs to have a
cherry red spot:
55 Irritability What is the most likely diagnosis?
55 Muscle hypertonicity A. Right hemispheric infarction
55 Progressive neurologic deterioration B. Left chronic subdural hematoma
55 Peripheral neuropathy C. Hemimegaloencephaly
55 Evidence of white matter disease on D. Lobar Holoprosencephaly
neuroimaging E. Neonatal adrenoleukodystrophy
55 Elevation of cerebrospinal fluid (CSF)
protein concentration vv Correct Answer is: C
There is an enlargement of the left
zz Suggested Reading hemisphere on the MRI consistent with
55 Wenger DA. Krabbe Disease. Gene hemimegaloencephaly. The left occipi-
Reviews. 2000. tal lobe is so large that it crosses mid-
55 Hofman KJ. et al. Cherry red spot in line into the territory where the right
association with galactosylceramide-β- occipital lobe occupies. This is called
galactosidase deficiency. J Inherit Metab the “occipital pole sign” and is a seen
Dis. 1987;10(3):273–4. in cases of hemimegaloencephaly. The
cerebral cortex on the left is thickened
?? 14. You are evaluating 3 days in the NICU and dysplastic.
for desaturation spells and neonatal Hemimegaloencephaly (HMC) is
seizures. His seizures are multifocal a congenital brain malformation that
clonic spells. He was born full term, includes hamaromatous overgrowth
after uncomplicated pregnancy, labor of one hemisphere of the brain. Occa-
or delivery. His brain MRI is shown sionally it can involve the cerebellum
below: and brainstem. It presents as part of a
Genetic Disorders
41 3
syndrome or in isolation. Usually it is What is the most common genetic
associated with a neuro-­cutaneous syn- abnormality in this syndrome?
drome. These include Neurofibromatosis, A. Succinic semi aldehyde dehydro-
Proteus syndrome, Klippel-­Trénaunay- genase deficiency
Weber syndrome and others. Other B. Respiratory chain complex IV defi-
abnormalities of cortical development ciency (COX)
can be seen in associated with HMC C. Coenzyme Q deficiency
including lissencephaly, pachygyria and D. Carnitine deficiency
polymicrogyria. Affected individuals typi- E. POLG deficiency
cally present in infancy with intractable
seizures, epileptic encephalopathy, and vv Correct Answer is: B
severe neurodevelopmental delay. Early This is a case of Leigh syndrome, which
hemispherectomy has been shown to is a severe neurological disorder that
an effective treatment for those with typically arises in the first year of life. It
severe intractable epilepsy. The right is the most common clinical phenotype
hemisphere in this MRI is actually the of mitochondrial disorders in childhood.
normal hemisphere. There is no evi- The diagnostic criteria are (1) progres-
dence of a chronic subdural hematoma. sive neurological disease with motor
There is no evidence of abnormal fusion and intellectual developmental delay;
of the brain as seen with holoprosen- (2) signs and symptoms of brainstem
cephaly. ­Neonatal adrenoleukodys- and/or basal ganglia disease; (3) raised
trophy is not felt to be associated with lactate levels in blood and/or cerebro-
­hemimegaloencephaly. spinal fluid (CSF); and (4) characteristic
symmetric necrotic lesions in the basal
zz Suggested Reading ganglia and/or brainstem.
55 Flores-Sarnat L. Hemimegaloencephaly: The most characteristic neuro-­
part 1. Genetic, clinical, and imaging radiological findings in Leigh syndrome
aspects. J Child Neurol. 2002;17(5):373–84; are bilateral, symmetric focal hyper-
discussion 384. intensities in the basal ganglia, thalamus,
substantia nigra, and brainstem nuclei at
?? 15. You are seeing a 2-year-old boy for various levels on T2-weighted MRI.
hypotonia and abnormal eye move- There are at least 3 major causes of
ment. He has a history of having Leigh syndrome, each transmitted by a
vomiting and diarrhea, and he has different mode of inheritance: X-linked
been choking on food and running recessive, mitochondrial, and autosomal
into things. He has failure to thrive. recessive.
On examination, he has hypotonic, Disruption of complex IV, also called
abnormal eye movements, optic atro- cytochrome c oxidase or COX, is the most
phy and difficulty with tracking, dys- common cause of Leigh syndrome. The
tonia and an ataxic gait. His brain MRI most frequently mutated gene in COX-
shows symmetrical necrotic lesions in deficient Leigh syndrome is called SURF1.
the basal ganglia, thalami and brain- This gene, which is found in nuclear DNA,
stem which appear hyper intense on provides instructions for making a pro-
the T2 weighted images. tein that helps assemble the COX protein
42 Chapter 3 · Genetic Disorders

complex (complex IV). Mutations in the from paternal uniparental disomy of


SURF1 gene typically lead to an abnor- 15q11.2-q13; and 2–3% result from
mally short SURF1 protein, resulting in imprinting defects. A subset of the
the absence of functional SURF1 protein. remaining 25% is caused by mutations
The loss of this protein reduces the for- in the gene encoding the ubiquitin-
mation of normal COX complexes, which protein ligase E3A gene (UBE3A).
3 impairs mitochondrial energy produc-
tion. The second most common cause is zz Suggested Reading
disruption of complex. 55 7 www.omim.org/entry105830

zz Suggested Reading This question is part of a series of two


55 Leigh Syndrome. Omim # 256000 ­questions
Kartikasalwah et al. Leigh Syndrome: MRI
findings in two children. Biomed Imaging ?? 17. You were asked to see a 2-month-old
Intervention J. 2010 Jan-Mar 2010. girl after the infant was noted to be
55 Morris et al. Deficiency of respiratory chain floppy and failure to thrive. The infant
complex I is a common cause of Leigh is brought to the ED for decreased
Disease. Ann Neurol. 1996;40(1):25–30. oral intake over the last week. On
examination, head is normo-cephalic
?? 16. Angelman syndrome (AS) is a neuro- but she has severe head lag and is
developmental disorder characterized floppy with muscle weakness. The
by severe mental retardation, absent infant has tachypnea. Her echo-
speech, ataxia, sociable affect, and cardiogram showed cardiomegaly.
dysmorphic facial features. If genetic Creatinine kinase and Aspartate Ami-
testing for the maternal deletion in notransferase were elevated.
the 15q11.2-q13 critical region was What is the next step in the
negative. ­evaluation?
What another genetic test should be A. Lysosomal enzyme screen
considered? B. Serum lactate
A. FMR C. Serum Ammonia
B. UBE3A D. Muscle biopsy
C. ARX E. MRI of the brain
D. LIS
E. TSC vv Correct Answer is: A
This is a case of Pompeo disease,
vv Correct Answer is: B Glycogen-­storage disease type II (GSDII),
Angelman syndrome is a neurodevel- an autosomal recessive disorder that
opmental disorder characterized by results from the deficiency of acid
mental retardation, movement or bal- alpha-­glucosidase, (GAA). It is a rare
ance disorder, typical abnormal behav- (estimated at 1 in every 40,000 births),
iors, and severe limitations in speech inherited and often fatal disorder.
and language. Most cases are caused Acid alpha-glucosidase is a lyso-
by absence of a maternal contribution somal hydrolase that is required for the
to the imprinted region on chromo- degradation of a 1–3% of cellular gly-
some 15q11-q13. Approximately 70% cogen. Because the main pathway for
of AS cases result from de novo mater- glycogen degradation is not deficient in
nal deletions involving chromosome glycogen-storage disease type II disease,
15q11.2-q13; approximately 2% result energy production is not impaired, and
Genetic Disorders
43 3
hypoglycemia does not occur. However, (LSDs) including Gaucher Types 1
the deficiency of this enzymatic activity and 3, Mucopolysaccharidosis Type 1
results in the accumulation of structurally (Hurler, Hurler Schie Syndrome),
normal glycogen in lysosomes and cyto- Mucopolysaccharidosis Type 2 (Hunter
plasm in affected individuals. Syndrome), Mucopolysaccharidosis
In the infantile form, clinically sig- Type 6 (Maroteaux-Lamy Syndrome),
nificant storage occurs in the heart, Fabry Disease and Pompe Disease. It
resulting in progressive cardiomegaly involves replacement of the deficient
with left ventricular (LV) thickening that enzyme by IV infusion of exogenous
eventually leads to outflow tract obstruc- enzyme. It is recommended that it be
tion. Storage in skeletal muscle leads to used in LSDs without mental retardation
hypotonia and weakness. The respiratory because the exogenous enzyme does
muscles are also affected, resulting in not cross the blood-brain barrier. ERT
hypoventilation and progressive respira- has proven to be effective in reducing
tory compromise. CNS involvement is many symptoms of LSDs.
primarily limited to the anterior horn
cells of the spinal cord and brain stem zz Suggested Reading
nuclei, although intellectual performance 55 Bruni S., et al. Update on Treatment of
remains normal. Lysosomal Storage Diseases. Acta
Symptoms begin in the first months of Myologica. 2007;XXVI: 87–92.
life, with feeding problems, poor weight
gain, muscle weakness, floppiness, and ?? 19. You were asked to see an infant who
head lag. is having difficulty with sucking and
has not passed meconium. On exami-
zz Suggested Reading nation the infant is dysmorphic with
55 NINDS Pompe information page. Ibrahim J. a large fontanel, high forehead, high
et al. Genetics of Glycogen-Storage Disease arched palate, right corneal opacity,
Type II (Pompe Disease). Medscape. palpable liver, hypotonic, floppy and
August 2014. does not react to visual or auditory
stimuli.
?? 18. The parents of the patient above have The signs that should raise suspicion
questions regarding management of a peroxisomal disease include all
and prognosis. of the following except:
Enzyme replacement therapy has A. Faciocranial dysmorphic features
been FDA approved in the all of the B. Patellar calcifications
following Lysosomal Storage Disor- C. Hirschsprung disease
ders except: D. Renal cysts
A. Gaucher types 1 and 3 E. Neuronal Migration Defects
B. Mucopolysaccharidosis Types 1, 2
and 6 vv Correct Answer is: C
C. Pompe Disease This is a clinical presentation of
D. Metachromatic Leukodystrophy Zellweger syndrome, which is the most
E. Fabry Disease frequent and severe of the peroxisomal
disorders in early infancy. It is a multi-
vv Correct Answer is: D system autosomal recessive disorder
Enzyme replacement therapy (ERT) associated with a complete absence of
has been FDA approved for use in peroxisomes in all tissues of the body
several Lysosomal Storage Disorders and a generalized loss of peroxisomal
44 Chapter 3 · Genetic Disorders

development. Signs of major diagnostic C. Lactic acid


importance are the typical craniofacial D. Serum amino acids
dysmorphology, ocular abnormalities, E. Urine organic acids
renal cysts and patellar calcifications.
In the typical patient, soon after birth it vv Correct Answer is: B
is noted that the child is hypotonic and This is a case of an X-linked
3 has impaired hearing and diminished Adrenoleukodystrophy. X-ALD is a
visual acuity. The craniofacial abnor- metabolic disorder characterized by
malities include a high forehead, large impaired peroxisomal beta-­oxidation
fontanels and metopic suture, and high of very long-chain fatty acids.
arched palate. Ocular abnormalities Consequently, there is an accumula-
include cataracts and corneal opacities tion of VLCFA in plasma and all tissues,
and retinal degeneration. Renal cysts including the white matter of the brain,
occur in all patients. Patellar calcifica- the spinal cord and adrenal cortex. It
tions are present in half the patients. is caused by mutations in the ABCD1
MRI may reveal abnormal neuronal gene located on the X-­chromosome.
migration. Mutations in this gene cause the
absence or dysfunction of ALDP, a per-
zz Suggested Reading oxisomal transmembrane protein that
55 Lyon et al. Neurology of hereditary transports VLCFacyl-CoA esters from
metabolic diseases of children. 2nd ed. the cytosol into the peroxisome. The
1996. onset of X-ALD is insidious, with defi-
cits in cognitive abilities that involve
?? 20. You are seeing a 10-year-old boy visuospatial and visual-motor func-
for Attention Deficit Hyperactivity tions or attention and reasoning. In
Disorder evaluation. He has always boys and adolescents, it initially results
been an excellent student but over in a decline of school performance.
the last year he has had difficulty These early clinical symptoms are
paying attention in the classroom often misdiagnosed as attention deficit
and has become hyperactive. He has hyperactivity disorder and can delay
a drop in his grades from A/B’s to the diagnosis of X-ALD. As the disease
C’s/D’s. On examination, he is a thin progresses, clinical manifestations
adolescent without acute distress, can include withdrawn or hyperac-
but he has melanoderma on his skin tive behavior, apraxia, astereognosia,
examination. He has delayed pro- auditory i­ mpairment, decreased visual
cessing, dysarthria, left hemiparesis acuity, hemiparesis or spastic tetrapa-
and hemianopia on his neurologi- resis, cerebellar ataxia and seizures.
cal examination. Your admitted him There can also be adrenal insufficiency
urgently to the hospital and ordered in which the patient presents with
a brain MRI, which shows increased Addison’s disease. The adrenal insuf-
signal intensity on T2-weighed and ficiency can be subtle in which the
FLAIR sequences in the parieto-­ only manifestation is bronzing of the
occipital region and the splenium of skin. Usually death occurs 2–4 years
the ­corpus callosum. after onset of symptoms. If X-ALD
The most appropriate laboratory is suspected in a male with neuro-
test would be: logical symptoms or Addison’s disease,
A. Creatinine Kinase elevated plasma VLCFAs confirms the
B. Very long chain fatty acids diagnosis.
Genetic Disorders
45 3
zz Suggested Reading retinal hypoplasia, m ­ icrophthalmia,
55 Lyon et al. Neurology of Hereditary cataracts, and iris ­malformations.
Metabolic diseases in children. 1996.
55 Engelen et al. X-linked adrenoleukodystro- zz Suggested Reading
phy (X-ALD): clinical presentation and 55 Iannaccone ST, et al. Congenital muscular
guidelines for diagnosis, follow-up and dystrophies and congenital myopathies.
management. Orphanet J. Rare Dis. 2012;7:51 Continuum 2013;19(6):1509–34.

?? 21. You are seeing a newborn neonate in ?? 22. You are evaluating a 6 who was a
the nursery who has dysmorphic fea- floppy infant. Family states that he is
tures, weak cry and suck. He has micro- not social, has repetitive behaviors,
cephaly, and he has a cataract and and is hyposensitive to pain. He has
iris malformation. He has decreased normal hearing and vision. On exami-
movement and severe hypotonia on nation he has dysmorphic features;
motor examination. His brain MRI you note that he has a long narrow
shows cobblestone ­lissencephaly. head, ptosis and prominent ears. He
The most common defect in this also has Autistic behavior, absent
­congenital muscular dystrophy is: speech, and hypotonia. He has a nor-
A. Merosin deficiency mal head circumference.
B. Collagen deficiency Mutations in which of the following
C. Abnormal dystrophin genes are most likely responsible for
D. Abnormal glycosylation of alpha- his condition?
Dystroglycan A. RAI1
E. Defect in glycogen metabolism B. FOXG1
C. PTEN
vv Correct Answer is: D D. SHANK3
This is a clinical presentation of Walker- E. ARX
Warburg Syndrome. It is the most severe
of the dystroglycanopathies which are a vv Correct Answer is: D
group of disorders caused by abnormal This patient has Phelan-Mc Dermid
glycosylation of alpha-dystroglycan. Syndrome- 22q13 deletion syndrome.
Alpha-­Dystroglycanopathies include Children with this disorder can have
Fukuyama congenital muscular dystro- mild dysmorphic features.
phy, muscle-eye-­brain disease, Walker- Durand et al. (2007) reported evi-
Warburg syndrome, and congenital dence showing that abnormal gene dos-
muscular dystrophy type 1C. These dis- age of SHANK3 is associated with severe
orders are associated with CNS pathol- cognitive deficits, including language and
ogy and structural eye abnormalities. speech disorder and autism spectrum
Patients present with paralysis at birth, disorder. They reported 3 families with
severe hypotonia, weak suck and cry and autism spectrum disorder and unambigu-
profound developmental delay. Brain ous alteration of 22q13 or SHANK3.
abnormalities include microcephaly, Mutations of RAI1 gene on chromo-
hydrocephalus, cobblestone lissenceph- some 17 causes Smith-Magenis Syn-
aly, severe hypoplastic cerebellum and drome (intellectual disability, behavioral
brainstem, with a Dandy-­Walker malfor- problems and sleep disturbance).
mation in 15–20%. Ocular a ­ bnormalities Mutations of FOXG1 cause symptoms
include severe congenital myopia, of an Autism Spectrum Disorder, develop-
glaucoma, and pallor of the optic discs, mental delay, and microcephaly.
46 Chapter 3 · Genetic Disorders

Mutations of the PTEN cause symp- impairment. A distinctive behavioral


toms of an Autism Spectrum Disorder, phenotype (with temper tantrums, stub-
macrocephaly and hamartomas. bornness, manipulative behavior, and
Mutations of the ARX gene cause obsessive-compulsive characteristics)
X-linked mental retardation. is common. Short stature is common;
characteristic facial features, strabis-
3 zz Suggested Reading mus, and scoliosis are often present,
55 Phelan-MC Dermid Syndrome. OMIM and non-insulin-dependent diabetes
Entry # 606232. Genetics Home Reference. mellitus often occurs in obese indi-
viduals. Hypopigmentation can also be
?? 23. You are evaluating a 7-year-old boy seen. The mainstay of diagnosis is DNA
who has always been delayed but he methylation testing to detect abnormal
is struggling in first grade although he parent-specific imprinting within the
is in special education. His past medi- Prader-Willi critical region (PWCR) on
cal history is significant for failure to chromosome 15; this testing determines
thrive, was “floppy” and had delays in whether the region is maternally inher-
motor development. Now he is over- ited only (i.e., the paternally contributed
weight because he eats all the time region is absent) and detects more than
and is always thirsty. He has been eval- 99% of affected individuals.
uated type 2 diabetes. On examination,
he is obese, pleasant boy who is eager zz Suggested Reading
to interact with you. He has almond 55 Driscoll DJ. Prader-Willi Syndrome.
shaped eyes and small hands and feet. GeneReviews Internet. October 1998.
He has a large area of hypopigmenta-
tion on his face, and hypotonic. He ?? 24. You are evaluating an 8-year-old
cannot write his name but he is able to boy for his developmental delay. His
draw a picture of himself; the drawing past medical history is remarkable of
is at a 3-year-old level. frequent ear infections and a heart
What laboratory test is most likely murmur due to valvular disease. On
would yield the diagnosis? examination, he has short stature,
A. Fragile X lordotic posture, slightly coarse facial
B. MRI of the brain features, and corneal clouding.
C. DNA methylation of chromosome What laboratory test would most
15 q likely define this condition?
D. Neuropsychological testing A. Amino acid analysis
E. FISH for chromosome 22q11 B. 7-dehydrocholesterol level
­deletion C. Lysosomal enzyme screen
D. CSF neurotransmitter levels
vv Correct Answer is: C E. Uric acid level
He has Prader Willi Syndrome (PWS).
Prader-Willi (PWS) syndrome is char- vv Correct Answer is: C
acterized by severe hypotonia and The mucopolysaccharidoses (MPSs) are
feeding difficulties in early infancy, a group of rare inherited lysosomal stor-
followed in later infancy or early age disorders caused by the ­deficiency
­childhood by excessive eating and or absence of specific lysosomal
gradual d­ evelopment of morbid obe- enzymes. The absence of these enzymes
sity. Motor milestones and language leads to accumulation of complex car-
development are delayed. All indi- bohydrates in cells and tissues and in
viduals have some degree of cognitive the cellular organelles, the lysosomes.
Genetic Disorders
47 3
These complex carbohydrates are also C. Sinus infection
known as mucopolysaccharides or gly- D. Cancer
cosaminoglycans (GAGs) serve as the E. All of the above
building blocks for connective tissues.
MPS III (Sanfilippo syndrome), is the vv Correct Answer is: E
most common of MPS disorders and results This is a case of Ataxia-telangiectasia,
from the deficiency or absence of 4 differ- which is a rare inherited disorder that
ent enzymes that are necessary to degrade affects the nervous system, immune
the GAG heparan sulfate. Each enzyme system, and other body systems. This
deficiency defines a different form of San- disorder is characterized by progressive
filippo syndrome. Individuals with type A ataxia beginning in early childhood.
lack the enzyme heparan sulfate sulfatase. Telangiectasia, which occur in the eyes
Individuals with type B lack the enzyme and on the surface of the skin, are also
N -acetyl-­alpha-­D-glucosaminidase. characteristic of this condition.
Patients with type C lack acetyl-CoA alpha- Mutations in the ATM gene, a gene
glucosaminide acetyltransferase. Patients the product of which is involved in DNA
with type D lack the enzyme N- acetylglu- repair, cause ataxia-telangiectasia.
cosamine-6-sulfatase. The particular form Patients with ataxia-telangiectasia have
of Sanfilippo syndrome cannot be deter- immunoglobulin deficiencies and many
mined based on clinical features, but rather develop chronic lung and sinus infec-
must rely on enzymatic assays. tions, including tuberculosis. They also
The neurological manifestations have an increased risk of developing
include mental regression beginning in cancer, particularly leukemia and lym-
the toddler years, coarse facial appear- phoma. Affected individuals are very
ance, but may be mild, sleep disturbance sensitive to the effects of radiation expo-
and progressive myelopathy due to sure. The life expectancy of people with
storage material in the spine. Seizures ataxia-­telangiectasia varies greatly, but
may occur. Initial clinical signs present affected individuals typically live into
as a change in behavior. Patients are early adulthood.
hyperactive and display aggressive and
­destructive behaviors. zz Suggested Reading
55 Perlman S, Becker-­Catania S, Gatti RA.
zz Suggested Reading Ataxia-telangiectasia: diagnosis and
55 Valstar MJ, Ruijter GJ, van Diggelen OP, treatment. Semin Pediatr Neurol.
Poorthuis BJ, Wijburg FA. Sanfilippo 2003;10(3):173–82.
syndrome: a mini-review. J Inherit Metab
Dis. 2008. ?? 26. You are evaluating a 16-year-old
female for her progressive gait
?? 25. You are seeing a 7-year-old adopted deterioration. She has a history of
boy because he has telangiectasias on psychiatric symptoms including
his face and sclera and walks with a inattention, impulsivity, and anger
walker because he is ataxic. The adop- outbursts for 1 year. Her past medical
tive family asking about the long-term history is otherwise unremarkable
implications of his genetic disease, you and family history is noncontribu-
should tell them he is at higher risk tory. On examination, she has a pale
than unaffected children of developing: optic nerves, low amplitude deep
A. Tuberculosis tendon reflexes, increased tone and
B. Inadequate responses to extensor planter responses. Her MRI
­vaccinations is shown.
48 Chapter 3 · Genetic Disorders

a b c

d e f

g h i

j k l

m n o

..      Fig. 3.11 Liaw, Hsiang-Ru, et al. “Late infantile 1. HR Liaw, HF Lee, CS Chi. This article is distributed
metachromatic leukodystrophy: Clinical manifesta- under the terms of the Creative Commons Attribution
tions of five Taiwanese patients and Genetic features 4.0 International License. a–c Patient #1; d–f Patient #2;
in Asia.” Orphanet journal of rare diseases 10.1 (2015): g–i Patient #3; j–l Patient # 4; m–o Patient # 5
Genetic Disorders
49 3
What is the most likely diagnosis? inability to stand, difficulty with
A. Adrenoleukodystrophy speech, deterioration of mental func-
B. Krabbe’s disease tion, increased muscle tone, pain in the
C. Metachromatic leukodystrophy arms and legs, generalized or partial
D. Alexander’s disease seizures, compromised vision and hear-
E. Canavan’s disease ing, and peripheral neuropathy. In the
final stages children have tonic spasms,
vv Correct Answer is: C decerebrate posturing, and general
This is a case of metachromatic leu- unawareness of their surroundings.
kodystrophy (MLD). MLD is due to Unexplained fevers and elevated CSF
Arylsulfatase A deficiency. There are protein may be seen.
three clinical subtypes: a late-infantile
MLD, a juvenile MLD, and an adult zz Suggested Reading
onset form of MLD. The disease course 55 Kaye EM. Update on genetic disorders
may be from three to ten or more years affecting white matter. Pediatr Neurol.
in the late-infantile form and up to 2001;24(1):11–24.
20 years or more in the juvenile and
adult forms. In the late infantile type, ?? 27. The child shown in the picture below
onset is between ages 1 and 2 years. has history of global developmen-
Typical presenting manifestations tal delay and seizures. A similarly
include weakness, hypotonia, clumsi- affected older sibling died at age 2
ness, frequent falls, toe walking, and due to complications of sepsis. His CT
slurred speech. Later signs include scan is shown.

a b

..      Fig. 3.12 a CT scan showing radiological findings of Holoprosencephaly. b A child is showing the facial
anomalies associated with Holoprosencephaly. Courtesy of Max Muenke, M.D., NIH
50 Chapter 3 · Genetic Disorders

What is the most likely diagnosis? decided to see her for follow up at
A. Aicardi syndrome 6 months of age, when she is noted to
B. Fetal alcohol syndrome have “sun setting” eyes. A ventriculo-
C. Velocardiofacial syndrome peritoneal shunt is placed. Her head
D. Lissencephaly MRI after shunt placement is shown
E. Holoprosencephaly below:
3
vv Correct Answer is: E
Holoprosencephaly (HPE) is the
most common structural malforma-
tion of the developing forebrain in
humans. It is characterized by failure
of the forebrain to divide into the two
separate hemispheres and ventricles.
Holoprosencephaly encompasses a
spectrum of brain malformations along
a continuum based on the degree of
non-­separation of the hemispheres. This
continuum includes alobar holoprosen-
cephaly (a single ventricle and no sepa-
ration of the cerebral hemispheres);
semilobar holoprosencephaly (the left
and right frontal and parietal lobes are
fused and the interhemispheric fissure ..      Fig. 3.13 Variant DWS with dysplasia of the pons
and cerebellum in an 8-year old. T2 weighted sagittal
is only present posteriorly); lobar holo-
MRI. Hellerhoff. Wikipedia
prosencephaly (most of the right and
left cerebral hemispheres and lateral
ventricles are separated but the most
rostral aspect of the telencephalon, the This child has a form of:
frontal lobes, are not separated, espe- A. Arnold Chiari malformation
cially ventrally), and syntelencephaly B. De Morsier malformation
or middle interhemispheric variant C. Dandy-Walker malformation
(failure of separation of the posterior D. Walker-Warburg malformation
frontal and parietal lobes, with varying
lack of cleavage of the basal ganglia vv Correct Answer is: C
and thalami, and absence of the body This is a case of Dandy-Walker
of the corpus callosum but presence of Syndrome, which is a congenital brain
the genu and splenium of the corpus malformation involving the cerebellum
callosum). and the fluid-filled spaces around it. The
key features of this syndrome are an
zz Suggested Reading enlargement of the fourth ventricle, a
55 Muenke M, Gropman A. partial or complete absence of cerebel-
Holoprosencephaly, In: Genereviews. lar vermis, and cyst formation near the
7 http://www.genereviews.org. lowest part of the skull. An increase in
the size of the fluid spaces surround-
?? 28. You saw a 4 weeks old girl for a big ing the brain as well as an increase
head. Her head CT is shown below. in pressure may also be present. The
She is doing well clinically and so you syndrome can appear dramatically or
Genetic Disorders
51 3
develop unnoticed. Symptoms, which obtunded. EEG shows slowing and
often occur in early infancy, include triphasic waves.
slow motor development and pro- What is the most appropriate next
gressive enlargement of the skull. In step in evaluation?
older children, symptoms of increased A. CSF exam
intracranial pressure such as irritability B. Ammonia
and vomiting, and signs of cerebellar C. Plasma lactate
dysfunction such as unsteadiness, lack D. MRI of the brain with MRS
of muscle coordination, or jerky move- E. CT brain scan
ments of the eyes may occur. Other
symptoms include increased head cir- vv Correct Answer is: B
cumference, bulging at the back of the Urea cycle disorders (UCDs) typically
skull, problems with the nerves that present in infancy, but the majority of
control the eyes, face and neck, and individuals UCD present outside the
abnormal breathing patterns. Dandy- neonatal period, frequently in child-
Walker Syndrome is frequently associ- hood. Signs and symptoms are often
ated with disorders of other CNS areas, vague, but recurrent; fulminant presen-
including absence of the corpus cal- tations associated with acute illness are
losum, and malformations of the heart, also common. A disorder of urea cycle
face, limbs, fingers and toes. metabolism should be considered in
children who have recurrent symptoms,
zz Suggested Reading especially neurologic abnormalities
55 NINDS Dandy-Walker Syndrome associated with periods of decompensa-
Information Page. (Accessed 10–10-2014) tion. Routine laboratory tests, includ-
7 http://www.ninds.nih.gov/disorders/ ing measurement of plasma ammonia
dandywalker/dandywalker.htm. concentrations, can indicate a poten-
55 Correa GG, Amaral LF, Vedolin LM. tial UCD; however, specific metabolic
Neuroimaging of Dandy-Walker malfor- testing and ultimately enzymatic or
mation: new concepts. Top Magn Reson molecular confirmation are necessary
Imaging. 2011;22(6):303–12. to establish a diagnosis. Treatment with
55 Spennato P, Mirone G, Nastro A, dietary protein restriction and medica-
Buonocore MC, Ruggiero C, Trischitta V, tions may be challenging in children.
Aliberti F, Cinalli G. Hydrocephalus in
Dandy-Walker malformation. Childs Nerv zz Suggested Reading
Syst. 2011;27(10):1665–81. 55 Smith W, Kishnani PS, Lee B, Singh RH,
Rhead WJ, Sniderman King L, Smith M,
Summar M. Urea cycle disorders: clinical
?? 29. You are evaluating a 9-year-old presentation outside the newborn period.
healthy female in the ED with a his- Crit Care Clin. 2005;21(4 Suppl):S9–17.
tory of vomiting and lethargy after a
holiday party. She has unremarkable ?? 30. You were asked to evaluate an
past medical history. Her develop- 8-month-old boy for a history of peri-
mental milestones were on target and ods of irritability, stiffness and regres-
she is an average student in the 3rd sion of skills for 3 months. He has a
grade. Family history is remarkable for normal birth and early development
a maternal cousin who died in early history. He also has periods of ele-
infancy from an unspecified condi- vated temperature without evidence
tion. When you examined her, she was of infection. He has no dysmorphic
52 Chapter 3 · Genetic Disorders

features on examination. There are enzyme activity is deficient in leukocytes


episodes of opisthotonus in response isolated from whole heparinized blood or
to tactile stimulus. Muscle tone is in cultured skin fibroblasts.
increased and reflexes are absent. A
spinal tap showed elevated protein zz Suggested Reading
but no other abnormalities. 55 Wenger DA, Suzuki K, Suzuki Y, Suzuki K.
3 What laboratory study would most Galactosylceramide lipidosis. Globoid cell
likely be abnormal in this patient? leukodystrophy (Krabbe disease). In:
A. Phenylalanine hydroxylase Scriver CR, Beaudet AL, Sly WS, Valle D,
B. 7-dehydrocholesterol Childs B, Vogelstein B, editors The meta-
C. Acid maltase bolic and molecular bases of inherited
D. Galactocerebrosidase disease. 8th ed. New York: McGraw-Hill;
E. Beta hexosamindase A 2001.

vv Correct Answer is: D ?? 31. You are evaluating a 9-month-old


This is a clinical presentation of Krabbe girl for her seizure which is described
disease. Krabbe disease is characterized as staring followed by a generalized
by an infantile-onset progressive neu- convulsion lasting few minutes. Her
rologic regressive disorder with death past medical history is remarkable for
before age 2 years of age in the majority hypotonia and developmental delay,
of individuals. Alternatively, onset may is unable to roll over and does not
be between age 6 months and the fifth reach for objects with her hands. On
decade with slower disease progression. examination, her head circumference
Children with the infantile form appear is above 95th percentile, while her
to be normal for the first few months of weight and length are at 25th percen-
life but show extreme irritability, spas- tile. She has optic atrophy and nystag-
ticity, and developmental delay before mus, central hypotonia and increased
age 6 months; psychomotor regression tone in the ankles, with clonus. Brain
progresses to a decerebrate state with MRI shows increased signal in the
no voluntary movement. In the late- white matter diffusely on T2-weighted
onset forms, individuals can be clinically images. Her MRS shows a strikingly
normal until weakness, vision loss, and elevated NAA peak.
intellectual regression become evident. What is your most likely diagnosis?
The clinical course is variable even A. Krabbe leukodystrophy
within the same family. B. Adrenoleukodystrophy
Stage I is characterized by irritabil- C. Mitochondrial encephalomyopathy
ity, stiffness, arrest of motor and mental D. Canavan disease
development, and episodes of tem- E. Metachromatic leukodystrophy
perature elevation without infection,
possibly caused by involvement of the vv Correct Answer is: D
hypothalamus. The child, apparently The triad of hypotonia, macrocephaly,
normal for the first few months after and head lag in an infant after the age
birth, becomes hypersensitive to audi- of 3–5 months should raise the sus-
tory, tactile, or visual stimuli and begins picion of Canavan disease. Diagnosis
to cry frequently without apparent of Canavan disease in symptomatic
cause. individuals relies upon demonstration
In almost all individuals with Krabbe of very high concentration of N-­acetyl
disease, galactocerebrosidase (GALC) aspartic acid (NAA) in the urine. NAA
Genetic Disorders
53 3
concentration is also elevated in the accumulation of harmful metabolites,
blood and cerebrospinal fluid (CSF) of including phenylketones. If untreated,
children with Canavan disease. In the PKU leads to mental retardation, sei-
brain, the elevated NAA can be visual- zures, psychoses, eczema and a distinc-
ized by 1H MRS as a single peak with a tive “mousy” odor. Pregnant women
resonance of 2.01 ppm on the spectrum. who have PKU and are not maintaining
dietary restrictions are at risk of deliver-
zz Suggested Reading ing infants with the effects of untreated
55 Janson CG, McPhee SW, Francis J, Shera D, maternal PKU and may also be at risk
Assadi M, Freese A, Hurh P, Haselgrove J, of developing neurologic symptoms
Wang DJ, Bilaniuk L, Leone P. Natural themselves if they don’t maintain life-
history of Canavan disease revealed by long dietary restrictions. The charac-
proton magnetic resonance spectroscopy teristic manifestations of maternal PKU
(1H-MRS) and diffusion-weighted MRI. syndrome include mental retardation,
Neuropediatrics. 2006;37(4):209–21. microcephaly, intrauterine growth retar-
dation, and congenital heart defects.
?? 32. You are seeing a 15-­old boy for micro- The risk for these defects is increased
cephaly and developmental delay. He with maternal plasma phenylalanine
and his family migrated to the United ­levels > 20 mg/dL).
States recently from Eastern Europe.
His mother had 2 other children (from zz Suggested Reading
different fathers) who both have mild 55 Bouchlariotou S, Tsikouras P, Maroulis G.
intellectual disability, microcephaly, Undiagnosed maternal phenylketonuria:
and delay in linguistic development, own clinical experience and literature
congenital heart defects and severe review. J Matern Fetal Neonatal Med.
growth retardation. The mother is 2009;22(10):943–8.
a vegetarian since her childhood
but currently does not practice any ?? 33. You are evaluating a 7-month-old
dietary restrictions. She barely gradu- infant for his developmental regres-
ated high school and has difficulty sion with progressive opisthotonus,
remembering appointments and stridor, and swallowing difficulties. He
­paying bills. was normal at birth. On examination,
What is the most likely explanation he has enlarged spleen, squint, head
for this child’s condition? retro flexion, spasticity, and trismus.
A. Chromosome disorder Analysis of which of the following
B. Infantile autism tissues would most likely define his
C. Leigh’s disease condition?
D. Maternal phenylketonuria A. Bone marrow
E. Urea cycle disorder B. Blood leukocytes
C. Muscle
vv Correct Answer is: D D. Skin
This is a case of phenylketonuria (PKU). E. Liver
PKU is an autosomal recessive disorder
of amino acid metabolism affecting vv Correct Answer is: B
approximately 1/5000–10,000 infants This is a clinical presentation of Gaucher
in North America. It is most often due disease. The diagnosis of Gaucher
to deficiency of the enzyme phenyl- disease relies on demonstration of
alanine hydroxylase which causes the deficient glucosylceramidase enzyme
54 Chapter 3 · Genetic Disorders

activity in peripheral blood leukocytes. differentiated from the elevated ALT


Carrier testing by assay of enzyme and AST of liver disease by the concomi-
activity is unreliable because of overlap tant and disproportionate elevation of
in enzyme activity between carriers “muscle function tests”, such as levels of
and non-carriers. Identification of two CPK and/or aldolase.
disease-­causing alleles in GBA, the
3 only gene known to be associated with zz Suggested Reading
Gaucher disease, ­provides additional 55 Korones DN, Brown MR, Palis J. “Liver
confirmation of the diagnosis. function tests” are not always tests of liver
function. Am J Hematol. 2001;66(1):46–8.
zz Suggested Reading 55 Stein MT, Tipnis NA, Schultz P. Fatigue,
55 Goker-Alpan O, Schiffmann R, Park JK, decreased interest in play, motor delay,
Stubblefield BK, Tayebi N, Sidransky E. and elevated liver function tests in a
Phenotypic continuum in neuronopathic 4-year-old boy. J Dev Behav Pediatr.
Gaucher disease: an intermediate pheno- 2002;23(1):37–41.
type between type 2 and type 3. J Pediatr.
2003;143(2):273–6.
?? 35. You are evaluating a 6-year-old male
?? 34. A pediatrician referred a 4-year-old because his family complaining that
boy because he felt the boy’s liver 1 he was completely normal until age 4,
finger breadth below the right costal when he began bumping into things
margin during a well-child check-up. and not responding to visual cues.
Liver function studies were done and He was seen by a pediatric ophthal-
showed abnormal values: ALT, 70 U/L; mologist who found that the child has
AST, 100 IU/L. The mother reported retinitis pigmentosa. He is now legally
that the child having trouble standing blind. He started to have a change
up when he has been sitting on the in his behavior 1 month ago. He will
floor playing with toys. repeat asking the same questions
If the lab has additional blood left over, several times and seemed not to be
you should ask the lab to test it for: satisfied with the answer. The family
A. Glucose-6-phosphate reports that he is anxious. Last week,
B. Bilirubin his teacher has questioned whether or
C. Alkaline phosphatase not he understands what she is saying
D. Creatine phosphokinase to him. He has official hearing testing
E. Prothrombin time which is normal. On examination, he
perseverates on verbal and manual
vv Correct Answer is: D tasks, to be anxious, and to have occa-
“Liver function tests” are tests for sional, single myoclonic jerks of one
enzymes that are expressed at high or the other of his arms or legs. His
levels in the liver, but are expressed in retinas have a tigroid a­ ppearance.
many other cell types, including muscle. What gene testing would you like to
As a consequence, elevated ALT and order?
AST blood levels occur in neuromuscu- A. Metachromatic Leukodystrophy
lar disorders that involve lysis or high B. Rett Syndrome
turnover rates of myocytes. Destruction C. Neuronal ceroid lipofuscinosis
of muscle in Duchenne muscular dys- D. Morquio syndrome
trophy is one such situation. It can be E. Zellweger syndrome
Genetic Disorders
55 3
vv Correct Answer is: C has multiple spells per day. His past
This is a presentation of Juvenile Batten medical history is remarkable for
disease, which is a neurodegenerative infantile spasms between the ages
disorder that often has its onset in late of 2 and 6 months. His MRI scan is
toddlerhood or early childhood. It typi- below.
cally starts with pigmentary changes in
the retina and progressive visual loss.
This is followed by progressive demen-
tia, myoclonus, and seizures. More than
60 different mutations in the CLN3
gene have been found associated with
juvenile Batten disease. But the func-
tion of the CLN3 gene product is not
completely understood. CLN3 protein
is found in the membranes of many cell
types; it appears to have some function
related to lysosomal trafficking, pH, and
size.

zz Suggested Reading
55 Bennett MJ, Rakheja D. The neuronal
ceroid-lipofuscinoses. Dev Disabil Res Rev. ..      Fig. 3.14 MRI from Wikipedia, 7 http://en.wikipedia.
2013;17(3):254–9. org/wiki/Lissencephaly, author: Ralpheig, accessed
10/5/2014
55 Cotman SL, Staropoli JF. The juvenile
Batten disease protein, CLN3, and its role
in regulating anterograde and retrograde The child most likely has mutations
post-Golgi trafficking. Clin Lipidol. in the gene that causes:
2012;7(1):79–91. A. Fragile X
55 Tuxworth RI, Chen H, Vivancos V, B. Miller-Dieker syndrome
Carvajal N, Huang X, Tear G. The Batten C. Tuberous Sclerosis
disease gene CLN3 is required for the D. Aicardi Syndrome
response to oxidative stress. Hum Mol E. Joubert Syndrome
Genet. 2011;20(10):2037–47.
vv Correct Answer is: B
?? 36. You are seeing a 2-year-old devel- Mutations in the PAFAH1B1 (Miller-­
opmentally delayed boy who has Dieker Syndrome), DCX (isolated lis-
3 different episodes type stared sencephaly), or TUBA1A gene can cause
at 10 months of age. While he is lissencephaly. PAFAH1B1 gene muta-
propped up in a sitting position, his tions are responsible for over half of
head just drops forward for a couple isolated lissencephaly sequence cases;
of seconds. Sometimes, without DCX gene mutations cause about 10%
warning, he has a single, vigorous of cases; and TUBA1A gene mutations
jerk of his whole body, extending cause a small percentage of isolated lis-
his arms and legs, after which he is sencephaly sequence. These genes code
sleepy for up to an hour. Also, he has for proteins that, through their interac-
a generalized tonic-clonic seizure tion with microtubules, are involved in
followed by sleepiness for hours. He neuronal migration.
56 Chapter 3 · Genetic Disorders

zz Suggested Reading diagnosis is suspected by the dysmor-


55 Fry AE, Cushion TD, Pilz DT. The genetics phic features and confirmed by analysis
of lissencephaly. Am J Med Genet C Semin of very long chain fatty acids (elevated
Med Genet. 2014;166C(2):198–210. VLCFA C26:0, C26:1; abnormal ratios of
55 Reiner O, Sapir T. LIS1 functions in normal C24/C22; C26/C22). Peroxisomes (which
development and disease. Curr Opin are missing in Zellweger’s) process
3 Neurobiol. VLCFA which, if they accumulate abnor-
mally, affect CNS and other cell mem-
?? 37. You are seeing a baby boy in the NICU branes. CNS dysmyelination occurs and
for episodes of clonic seizure activity in the severe form leads to problems
involving left and right side inde- with radial glial guides and migration
pendently lasting 30–60 s on several disorders (pachygyria, polymicrogyria,
occasions. He is a product of full- neuronal heterotopias, colopocephaly)
term pregnancy, and born after an resulting in severe encephalopathy
uneventful labor. He was depressed and seizures. Mutations in twelve dif-
at birth with generalized hypotonia ferent PEX genes, those that encode
and hypoventilation. On exam, he has peroxins, the proteins required for nor-
dysmorphic features consisting of mal peroxisome assembly, have been
micrognathia, broad nasal bridge, and identified in Zellweger’s syndrome.
high forehead, dolichocephalic skull Treatment is symptomatic and children
with large anterior and posterior fon- fail to develop, with death usually in
tanelles. Liver edge was firm and 1 cm the first year. Other PBD are neona-
below the costal margin. He is hypo- tal adrenoleukodystrophy, infantile
tonic with poor gag and suck/swal- Refsum disease (both considered part
low, absent reflexes and difficult to of the Zellweger’s Syndrome Spectrum)
elicit Moro response. The EEG showed and rhizomelic chondrodysplasia punc-
multifocal spikes on a mildly low volt- tata. These disorders can be separated
age background without state change clinically and 80% of PBD are among
but no electrographic seizures. CT the Zellweger’s Syndrome Spectrum
scan showed dilated posterior horns with an incidence of 1 in 50,000 births
(colopocephaly). in the US.
What is the most likely diagnosis?
A. Prader-Willi syndrome zz Suggested Reading
B. Zellweger’s syndrome 55 Steinberg SJ, Raymond GV, Braverman NE,
C. Myotonic dystrophy Moser AM, and Moser HW. Peroxisome
D. Spinal muscular atrophy Biogenesis Disorders, Zellweger Syndrome
E. Congenital CMV infection Spectrum. Gene Reviews: 7 http://www.
ncbi.nlm.nih.gov/bookshelf/br.fcgi?book=ge
vv Correct Answer is: B ne&part=pbd.
This is a clinical presentation of
Zellweger’s syndrome. It is an autoso- ?? 38. You are examining a 5-month-old boy
mal recessive disorder, is the severest who was normal at birth has been
form of the peroxisomal biogenesis startling easily for the past 2 weeks.
disorders (PBD) and presents at birth Head circumference had been in the
with dysmorphic features, liver cir- 50th percentile at birth, but is now in
rhosis, severe hypotonia and neonatal the 90th percentile. His retina photo-
encephalopathy with seizures. The graph is shown.
Genetic Disorders
57 3
and deaf with seizures, encephalopa-
thy and an increased startle to noise. A
rarer form can be seen in young adults
who present with a neuromuscular
­phenotype.

zz Suggested Reading
55 Shapiro BE, Logigian EL, Kolodny EH,
Pastores GM. Late-onset Tay-Sachs disease:
the spectrum of peripheral neuropathy in
30 affected patients. Muscle Nerve.
2008;38(2):1012–5.

?? 39. Children with Crouzon syndrome can


develop cognitive impairment.
This is because of:
..      Fig. 3.15 Cherry-red spot as seen in the retina in
A. Cerebral dysgenesis
Tay–Sachs disease. The fovea‘s center appears bright
red because it is surrounded by a whiter than usual B. Hypoxia from airway dysgenesis
area. 7 http://www.kellogg.umich.edu/theeyeshaveit/ C. Abnormal calcium metabolism
congenital/tay-sachs.html. Jonathan Trobe, M.D. D. Mitochondrial dysfunction
E. Multiple craniosynostoses

What would most likely be abnormal vv Correct Answer is: E


on laboratory testing? Management of the multiple simultane-
A. Aryl sulfatase A activity ous synostoses of the skull in Crouzon
B. Beta galactosidase A activity syndrome is an urgent matter, as they
C. Beta-hexosaminidase A activity can restrict outward brain growth
D. Glucocerebrosidase A activity in all directions and limit CSF egress
E. Very long chain fatty acids through occlusion of the foramina by
impingement from inward-­expanding
vv Correct Answer is: C ­parenchyma.
This photograph and the clinical pre-
sentation indicate Tay-Sachs disease, zz Suggested Reading
which is a fatal autosomal recessive lipid 55 Renier D, Lajeunie E, Arnaud E, Marchac D.
storage disorder in which the ganglio- Management of craniosynostoses. Childs
side GM2 build up in tissues and brain Nerv Syst. 2000;16(10–11):645–58.
neurons. The condition is caused by 55 Noetzel MJ, Marsh JL, Palkes H, Gado
insufficient activity of the enzyme beta- M. Hydrocephalus and mental retardation
hexosaminidase A that catalyzes the in craniosynostosis. J Pediatr.
biodegradation of acidic fatty materials 1985;107(6):885–92.
known as gangliosides. Infants with Tay
Sachs disease appear to develop nor- ?? 40. You are evaluating a 4-year-old boy
mally in the first few months of life. As for motor delay. He was a product
neurons become distended with storage of 38 weeks gestation with uncom-
material, the clinical course becomes plicated pregnancy. He was normal
one of relentless deterioration in which for the first few months of life, but
the child ultimately becomes blind, thereafter developed macrocephaly
58 Chapter 3 · Genetic Disorders

and was delayed in his milestones. zz Suggested Reading


He has unremarkable family history. 55 Janson CG, McPhee SW, Francis J, Shera D,
On examination, he is hypotonic and Assadi M, Freese A, Hurh P, Haselgrove J,
his head circumference is above the Wang DJ, Bilaniuk L, Leone P. Natural
95th percentile. His brain MRI scan is history of Canavan disease revealed by
shown. proton magnetic resonance spectroscopy
3 (1H-MRS) and diffusion-weighted
MRI. Neuropediatrics. 2006;37(4):209–21.

?? 41. You are evaluating a 9-year-old autis-


tic spectrum disorder (ASD) female
with history of a recent seizure which
began with staring and unresponsive-
ness and evolved into a generalized
tonic-clonic convulsion. A picture of
her face is shown.

..      Fig. 3.16 Radiology picture of the day.


(Dr. ­Laughlin Dawes. Canavan disease. Wikipedia)

What is the most likely diagnosis?


A. Adrenoleukodystrophy
B. Canavan disease ..      Fig. 3.17 A case of tuberous sclerosis showing
C. Krabbe disease facial angiofibromas in characteristic butterfly pattern.
D. Metachromatic leukodystrophy 7 http://cnx.org/content/m14895/latest/. Herbert
E. Vanishing white matter disease L. Fred, MD and Hendrik A. van Dijk

vv Correct Answer is: B Which of the following is most likely


The triad of hypotonia, macrocephaly, to be present on further evaluation?
and head lag in an infant after the age A. Conjunctival telangiectasias
of 3–5 months should raise the sus- B. Ash leaf spot
picion of Canavan disease. Diagnosis C. Family history consistent with
of Canavan disease in symptomatic recessive inheritance
individuals relies upon demonstration D. Multiple café au lait spots
of very high concentration of N-­acetyl E. Pseudo arthrosis
aspartic acid (NAA) in the urine. NAA
concentration is also elevated in the vv Correct Answer is: B
blood and cerebrospinal fluid (CSF) of This is a classic tuberous sclerosis (TSC)
children with Canavan disease. In the case. TS is a genetic disease with an
brain, the elevated NAA can be visual- incidence of about 1:40,000 that causes
ized by 1H MRS as a single peak with a benign tumors to grow in the brain and
resonance of 2.01 ppm on the ­spectrum. other organs such as the kidneys, heart,
Genetic Disorders
59 3
eyes, lungs, and skin. It commonly for MA of all causes in North America, and
affects the central nervous system. In 1/26,000 in China, but only a subset of
addition to the benign tumors that this is vitamin B12-responsive MA.
frequently occur in TSC, other common Usually it starts in infancy or early
symptoms include seizures, mental childhood with manifestations includ-
retardation, behavior problems, and ing lethargy, failure to thrive, recurrent
skin abnormalities. TSC may be present vomiting, dehydration, respiratory dis-
at birth, however, signs of the disorder tress, muscle hypotonia, hepatomegaly
can be subtle and full symptoms may and coma. They may also show signs
take some time to develop. The most of anemia (not megaloblastic), have
common skin lesion is the ash leaf spot. potentially life-threatening ketoacidosis
and/or hyperammonemia, and devel-
zz Suggested Reading opmental delay and intellectual deficit,
55 Gomez MR. Phenotypes of the tuberous with metabolic stroke affecting the brain
sclerosis complex with a revision of stem. MA frequently leads to end-stage
diagnostic criteria. Ann N Y Acad Sci. renal failure by adolescence or adult-
1991;615:1–7. hood. Patients with cblB are usually more
severely affected than patients with cblA.
?? 42. You electively admitted a 5-month-
old boy to the hospital for evaluation zz Suggested Reading
of failure to thrive, developmental 55 Orphanet. 7 http://www.orpha.net/consor/
delay, and generalized seizures. His cgi-bin/Disease_Search.php?lng=EN&data_
lab showed leukopenia. His evaluation id=3260&MISSING%20
includes urine organic acids and he CONTENT=Methylmalonic-acidemia--
is also found to have methylmalonic vitamin-B12-responsive&title=Methylmalonic
aciduria. -acidemia--vitamin-B12-responsive&search
He might have an abnormality =Disease_Search_Simple. Accessed
involving utilization of: 10–10-2014.
A. Vitamin D 55 Manoli I, Venditti CP. Methylmalonic
B. Vitamin E Acidemia. In: Pagon RA, Adam MP,
C. Vitamin C Ardinger HH, Bird TD, Dolan CR, Fong
D. Vitamin B12 CT, Smith RJH, Stephens K, editors.
E. Vitamin A GeneReviews® [Internet]. Seattle (WA):
University of Washington, Seattle; 1993–
vv Correct Answer is: D 2014. 2005 Aug 16 [updated 2010 Sep 28].
Vitamin B12-responsive methylmalonic 55 Parini R, Furlan F, Brambilla A, Codazzi D,
academia (MA) is an inborn error of Vedovati S, Corbetta C, Fedeli T, Merinero B,
vitamin B12 (cobalamin) metabolism Pérez B, Ugarte M. Severe Neonatal
characterized by recurrent ketoacidotic Metabolic Decompensation in
comas or transient vomiting, dehydra- Methylmalonic Acidemia Caused by CblD
tion, hypotonia and intellectual deficit, Defect. J Inher Metab Dis Rep.
which responds to vitamin B12. There 2013;11:133–7.
are three types: cblA, cblB and cblD-
variant 2 (cblDv2). ?? 43. You are evaluating a 14-year-old
To date, over 120 patients with cblA, male for severe generalized myoclo-
66 patients with cblB and 6 patients with nus, ataxia, and loss of color vision.
cblDv2 have been reported. Prevalence Funds exam showed cherry red spots,
of 1/48,000–1/61,000 have been reported and EEG demonstrated background
60 Chapter 3 · Genetic Disorders

slowing, and photosensitive bilateral features of this mitochondrial disorder


fast spike and wave activity. include myoclonus, epilepsy, myopathy,
What finding would establish the intellectual disability, and cerebellar
diagnosis? dysfunction. Patients may also have
A. An alpha-N-­acetylgalactosami short stature, deafness, optic atrophy,
nidase deficiency cutaneous lipomas, and neuropathy. The
3 B. decrease in cultured skin fibro- onset is typically in childhood or early
blast alpha-neuraminidase adulthood.
C. A response to oral biotin
D. A hexoaminidase A deficiency zz Suggested Reading
E. A muscle biopsy with ragged red 55 Verma A, Moraes CT. Mitochondrial
fibers disorders. In: Bradley WG, Daroff RB,
Fenichel GM, Jankovic J, editors.
vv Correct Answer is: B Neurology in Clinical Practice. 4th ed.
An alpha-N-acetylgalactosaminidase Elsevier, Inc.; 2004. p. 781–808.
deficiency is evident in Schindler dis- 55 Pastores GM, Kolodny EH. Inborn errors
ease, which is a rare autosomal recessive of metabolism of the nervous system. In:
disorder with acute infant and chronic Bradley WG, Daroff RB, Fenichel GM,
adult-onset forms. Infants demonstrate Jankovic J, editors. Neurology in Clinical
severe psychomotor deterioration with Practice. 4th ed. Elsevier, Inc.; 2004.
spaticity, cortical blindness, and myo- p. 1811–32.
clonic epilepsy with an exaggerated
startle response. The adult form is char- ?? 44. You are seeing an18-­year-­old male
acterized by mild cognitive deficits. with left-sided weakness and aphasia.
A presentation including severe gen- CT head showed an acute ischemic
eralized myoclonus, night blindness, and infarct in the territory of the right
loss of color vision, ataxia, cherry-red middle cerebral artery. On exam,
spots, and generalized convulsions in he is tall, thin, has long fingers, and
an adolescent is characteristic of sialido- high arched feet. His medical history
sis type 1. As described above, the EEG is remarkable for lens dislocation,
shows progressive slowing and bilateral, pigeon chest and osteoporosis.
photosensitive fast spike-wave dis- What single blood test is likely to be
charges. A decrease in cultured skin fibro- of diagnostic use in this patient?
blast alpha-neuraminidase is diagnostic. A. Peripheral blood smear
A response to oral biotin would be B. Thyroid-stimulating hormone
consistent with biotinidase deficiency. C. Homocysteine
This disorder is characterized by hypo- D. Branched chain amino acids
tonia, generalized tonic-clonic and myo- E. Lactic acid
clonic seizures, rash, and alopecia, with
onset in infancy. The EEG demonstrates vv Correct Answer is: C
multifocal spikes and slow waves. Genetics: AR; Cystathionine-13-synthase
A hexosaminidase A deficiency is evi- deficiency -- > elevation of blood and
dent in Tay-Sachs disease, with the onset urine levels of homocysteine and
of development delay, cherry-red spots, ­methionine.
and “startle seizures” in infancy. Symptoms and findings: Normal at
A muscle biopsy with ragged red birth; later developmental delay and
fibers is indicative of myoclonic epilepsy intellectual disability. Seizures and psy-
with ragged red fibers (MERRF). The core chiatric manifestations.
Genetic Disorders
61 3
Collagen metabolism is affected What is the most likely diagnosis in
-- > eye, bones, and the vascular system this patient?
-- > Ectopia lentis and lens dislocation. Mar- A. Essential tremor
fanoid habitus with tall and thin stature, B. Tardive dystonia and tremor
pectus carinatum, pes cavus, genu valgum, C. Wilson’s disease
and osteoporosis. Intimal thickening of the D. Early-onset Parkinson’s disease
blood vessel walls and high incidence of E. A psychogenic disorder
thromboembolism, including strokes.
Diagnosis – elevation of plasma and vv Correct Answer is: C
urine homocysteine, methionine in the This patient has Wilson’s disease based
plasma and CSF. on the history and MRI findings. It is an
Treatment – Majority of patient respond autosomal recessive disorder of copper
to pyridoxine, so give this first. Betaine, metabolism resulting from mutations
folate and vitamin 812 supplements. Low-­ of the gene encoding the copper-
protein diet, specifically low in methionine transporting P-type ATPase (ATP7B) on
with cysteine supplementation. chromosome 13. This enzyme normally
binds to and transports copper across
?? 45. You are seeing an 18-year-old female membranes. A defect in this enzyme
with a history of multiple abnormal leads to inability to excrete copper from
movements, including a moderate the liver into bile, leading to copper
frequency rest and postural tremor accumulation. The presenting symp-
and a twisting and in-turning of her toms of the disorder can be neurologic,
left foot. Her past medical history is hepatic, or psychiatric, or a combina-
remarkable for depression with sui- tion of these. Abnormal movements
cide attempt and significant anxiety. predominate the neurologic presenta-
A picture of her eye is shown below. tion, including Parkinsonism, dystonia,
She has elevated liver enzymes. tremor, ataxia, as well as dysarthria.
The tremor may have a variety of fea-
tures, but classically, it is proximal and
of high amplitude, giving the appear-
ance of “wing beating” when the arms
are abducted and the elbows flexed. A
characteristic grin with drooling also
occurs. Psychiatric symptoms include
depression, anxiety, and less commonly
psychosis. The liver disease may range
from mild to fulminant hepatic failure.
Laboratory findings in Wilson’s disease
include reduced serum levels of the
copper-binding protein.

?? 46. You are evaluating an 11-year old boy


for his headaches. His headaches are
..      Fig. 3.18 A brown ring on the edge of the cornea exacerbated by lying down and asso-
(Kayser–Fleischer ring) is common in Wilson’s disease, ciated with nausea and vomiting. On
especially when neurological symptoms are present.
his MRI scan, a 1 centimeter enhanc-
7 https://commons.wikimedia.org/wiki/File:Kayser-
Fleischer_ring.jpg. (Herbert L. Fred, MD, Hendrik A. ing mass is noted near the foramen
van Dijk) of Monro. He has a His past medical
62 Chapter 3 · Genetic Disorders

history is significant for cardiac rhab- The patient’s enhancing brain mass,
domyoma, discovered at birth. located near the foramen of Monro, is
Which of the following is most likely consistent with sub-ependymal giant
true? cell astrocytoma (SEGA). This, and the
A. The patient has a mutation in the patient’s cardiac rhabdomyoma, allow for
NF1 gene a diagnosis of definite tuberous sclerosis
3 B. The patient has a mutation in the complex.
TSC1 gene Mutations in the NF1 gene, encoding
C. The patient has a mutation in the for the protein neurofibromin, cause neu-
PTCH1 gene rofibromatosis type I. This is diagnosed in
D. The patient has a mutation in the the presence of 2 of the following:(1) the
PTEN gene presence of 6 cafe au lait macules (5 mm
E. The patient has a mutation in the in diameter pre-pubertal, 15 mm in diam-
NF2 gene eter post pubertal), (2) 2 neurofibroma or
1 plexiform neurofibroma, (3) freckling
vv Correct answer is: B of the inguinal or axillary region, (4) an
This is a case of tuberous sclerosis com- optic glioma, (5) 2:2 iris hamartomas, (6)
plex. This disorder is caused by muta- osseous lesion (thinning of long bone
tions in the TSC1 or TSC2 genes, coding cortex, sphenoid wing dysplasia) or (7)
for hamartin and tuberin, respectively. first degree relative with NF1.
Major features of this condition Mutations in the PTCH1 gene encod-
include facial angiofibromas, forehead ing for Patched 1 are associated with
plaques, ungula or periungual fibroma, nevoid basal cell carcinoma, or Gorlin
3 hypomelanotic macules, shagreen syndrome. Major criteria for this condi-
patch, retinal nodular hamartomas, cor- tion include; 2 basal cell carcinomas (12:
tical tubers, sub-ependymal nodules, is sufficient for people under 20 years of
sub-ependymal giant cell astrocytoma, age), odontogenic keratosis of the jaw, 3;
cardiac rhabdomyoma and lymphangio- palmar or plantar pits, bilamellar calcifi-
myomatosis/renal angiomyolipoma. cation of falx cerebri, rib abnormalities
Minor features include hamartoma- (bifid, fused, or markedly splayed), and a
tous rectal polyps, multiple pits in dental first degree relative with the condition.
enamel that are randomly distributed, Minor criteria include (1) frontal boss-
bone cysts, cerebral white matter radial ing/hypertelorism/cleft lip or palate (2)
migration lines, gingival fibromas, non- springe deformity/pectus/syndactyly of
renal hamartomas, retinal achromatic digits, (3) bridging of sella turcica/hemi-
patch, confetti skin lesions, and renal vertebrae flame shaped radiolucensies
cysts. (4) ovarian fibroma and (5) medulloblas-
The diagnosis of definite tuberous toma. A diagnosis of Nevoid basal cell
sclerosis complex can be made in the carcinoma can be made in the presence
presence of either 2 major features or 1 of either 2 major criteria or 1 major and 2
major feature and 2 minor features. The minor criteria.
presence of 1 major feature and 1 minor
feature is sufficient for a diagnosis of zz Suggested Reading
probable tuberous sclerosis complex. The 55 Jozwiak S, Migone N, Ruggieri M.The
presence of 1 major feature in isolation, tuberous sclerosis complex. In: Ruggieri M,
or the presence of at least 2 minor fea- Pascual­Castroviejo I, Di Rocco C, eds.
tures is suggestive of possible tuberous Neurocutaneous Disorders Phakomatoses
sclerosis complex. and Hamartoneoplastic Syndromes.
Genetic Disorders
63 3
Morlenbach: Springer-Verlag/Wien; 55 Geyer JR. Sposto R, Jennings M, Boyett JM,
2008:181–227. Axtell RA, Brieger D, Broxso E, Donahue B,
55 Tabori U, Laberge A-M, Ellezam B, Finlay JL, Goldwein JW, Heier LA,
Carret A-S. Cancer predisposition in Johnson DM, Mazewski C, Miller DC,
children with brain tumors. In: Scheinemann Packer R, Puccetti D, Radcliffe J, Tao ML,
K, Bouffet E, editors. Pediatric Neuro-­ Shiminski-Maher T. Multiagent chemo-
Oncology. New York: Springer; 2015:127–38 therapy and deferred radiotherapy in
.5. Lee J, Johnston DL. Chemotherapy. In: infants with malignant braintumors: a
Scheinemann K, Bouffet E, editors. Pediatric report from the Children’s Cancer. Group.J
Neuro-Oncology. New York: Springer; Clin Oncol. 2005;23(30):7621.
2015:69–896. 55 Wells EM, Packer RJ. Pediatric Brain Tumors.
55 Glass TD, Cochrane D, Rassekh SR, Continuum: Lifelong Learning in Neurology
Goddard K, Hukin J. Growingteratoma 2015; 21(2):373–96. 14. Fleming AJ. Rare
syndrome in intracranial non germinoma- tumors of the central nervous system in
tous germ cell tumors (iNGGCTs): a risk children. In: Scheinemann K, Bouffet E,
for secondary malignant transformation- a editors. Pediatric Neuro-­Oncology.
report of two cases. Child Nerv Syst. New York: Springer; 2015. p. 185–94.
2014;30(5):953–7.
55 Bartels U, Singhal A. Central nervous system ?? 47. A pediatrician refereed a 2-week-old
germ cell tumors. In: Scheinemann K, child to you for evaluation. He has a
Bouffet E, eds. Pediatric Neuro-­Oncology. distended abdomen and has had a
New York: Springer; 2015: p. 149–62. few bowel movements in comparison
55 Mach SC, Ramaswamy V, Wang X, Remke M, to other children at his age. On exami-
Sin-Chan P, Chan WSY, Bertrand KC, nation, he has a distended abdo-
Merino D, Zayne K, Huang A, Taylor MD. men with hypoactive bowel sounds.
Basic science of pediatric braintumors. In: X-ray of the abdomen shows dilated
Scheinemann K, Bouffet E, eds. Pediatric loops of bowel with excessive fecal
Neuro-Oncology. New York: Springer; material. You suspect a diagnosis of
2015. p. 59–67. Hirschsprung’s disease.
55 Jones DTW, Korshunov A, Pfister SM, Which of the following is not true
Tayllor MD, Northcott PA. regarding the enteric nervous
Medulloblastoma and CNS primitive ­system?
neuroectodermal tumors. In: Cagle PT, A. The sub mucous plexus is pre-
Matthias KA, Zagzag D, editors. Molecular dominantly involved in secretory
pathology of nervous system Tumors. functions of the gastrointestinal
New York: Springer; 2015. p. 121–42. tract.
55 Vanan Ml, Mehta V, Eisenstat, DD. B. Hirschsprung’s disease is due to
Diffuse intrinsic pontine glioma. In: congenital absence of the sub
Scheinemann K, Bouffet E, editors. mucous plexus.
Pediatric ­Neuro-­Oncology. New York: C. Hirschsprung’s disease is often
Springer; 2015. p. 117–26. focal, though rare cases involve
55 Rutkowski S, Bode U, Deinlein F, the entire colon.
Ottensmeier H, Warmuth-Metz M, D. Hirschsprung’s disease is due to
Soeroenson N, Grade N, Emser A, Pietsch T, congenital absence of the sub
Wolff JE, Kortmann RD, Kuehl J. Treatment mucous plexus.
of early childhood medulloblastoma by E. Hirschsprung’s disease is due to
post-­operative chemotherapy alone. N Eng J congenital absence of the myen-
Med. 2005;352(10):978. teric plexus.
64 Chapter 3 · Genetic Disorders

vv Correct answer is: E


The enteric nervous system consists of
the myenteric (or Auerbach’s) plexus
that is located between the outer
and inner smooth muscle layers of
the gastrointestinal tract and the sub
3 mucous (or Meissner’s) plexus, which
is located between the circular muscle
layer and the mucosa. The myenteric
plexus is predominantly involved in
gut motility, whereas the sub mucous
plexus is involved in secretory func-
tions. Together, the myenteric and
sub mucous plexus control the func-
tion of the gastrointestinal tract.
Hirschsprung’s disease is due to mal
development of the myenteric plexus.
Most commonly, focal congenital
absence of the myenteric plexus in the
internal anal sphincter or recto sig-
moid junction occurs, though in rare ..      Fig. 3.19 MRI image shows iron deposits in the
basal ganglia, the so-called eye-of-the-tiger sign
cases, the myenteric plexus may be
(T2w GRASE sequence). (Enro2002. Wikipedia)
absent throughout the gastrointestinal
system. Segments of the colon that
lack myenteric plexus cannot relax A. ATP1A3
leading to fecal retention and disten- B. GBA
tion of proximal colonic segments. C. MECP2
Some cases of Hirschsprung’s disease D. PANK2
are due to mutations in the RET proto- E. PMP22
oncogene.
vv Correct Answer is: D
zz Suggested Reading The clinical presentation and MRI
55 Comprehensive Review in Clinical showing the eye-of-the-tiger sign are
Neurology: A Multiple-Choice Question diagnostic of pantothenate kinase-
Book for the Wards and Boards © 2011 associated neurodegeneration (PKAN),
Wolters Kluwer Health Lippincott a type of neurodegeneration with brain
Williams & Wilkins. All rights iron accumulation. The disorder results
reserved. from mutations in the PANK2 gene.
Deletions in PMP22 are associated
?? 48. You are seeing a 10-year-old girl with with hereditary neuropathy, with liability
a history of dystonia, dysarthria, and to pressure palsies, while duplications in
rigidity. Her MRI is shown. Genetic this gene occur in Charcot-Marie-Tooth
testing will most likely demonstrate neuropathy type 1A.
a mutation in which of the following MECP2 mutations occur in Rett
genes? syndrome.
Genetic Disorders
65 3
Mutations in GBA are associated with What is the most likely diagnosis?
Gaucher disease. A. Double cortex syndrome
ATP1A3 mutations cause rapid-onset B. Hemimegalencephaly
dystonia-parkinsonism. C. Lissencephaly
D. Perisylvian syndrome
zz Suggested Reading E. Polymicrogyria
55 Gregory A, Hayflick SJ. Pantothenate
kinase-associated neurodegeneration. vv Correct Answer is: C
Seattle (WA): University of Washington, The MRI shows lissencephaly with com-
Seattle; 1993–2013. plete absence of cortical sulcation. The
combination of lissencephaly and the
?? 49. You are seeing a 2-day-­old neonate above malformations and dysmorphic
with dysmorphic features including features is characteristic of Miller-Dieker
widely spaced eyes, low set ears, a Syndrome. Typical manifestations include
short-upturned nose, and polydac- a prominent forehead with bi-temporal
tyly. Echocardiogram shows a small hollowing, short nose with upturned
ventricular septal defect and renal nares, and thickened upper lip with thin
ultrasound shows a small left kidney vermilion upper border, widely spaced
and absent right kidney. MRI of the eyes, low set ears, and a small jaw. Other
brain is shown. abnormalities include central hypotonia,
heart malformations, omphalocele, ingui-
nal hernia, duodenal atresia, pelvic and
cystic kidneys, sacral tails, cryptorchidism,
polydactyly, clinodactyly, ­camptodactyly,
transverse palmar creases, and polyhy-
dramnios during pregnancy. The underly-
ing genetic abnormality is a deletion of
the 17p13.3 locus, which involves LIS1
and YWHAE genes.

?? 50. You are seeing an 18-year-old male a


history of autism spectrum disorder and
moderate mental retardation. He has
epilepsy and takes topiramate and leve-
tiracetam. His last seizure was 3 years
ago. His brain T2 MRI scan is shown.
What skin findings would be
expected in this man?
A. adenoma sebaceum
B. café-au-lait macules
C. linear nevus sebaceous in the
scalp
D. lipoma over the lumbar spine
..      Fig. 3.20 With permission from ILAE.org. E. trigeminal distribution port-wine
EpilepsyDiagnosis.org, Diagnostic Manual stain
66 Chapter 3 · Genetic Disorders

to the number and size of the corti-


cal tubers seen on MRI. These hyper-
intense regions represent regions of
disorganized histogenesis of the brain.
Subependymal nodules can enlarge rap-
idly and may evolve into an obstructive
3 lesion near the foramen of Monro, called
a subependymal giant cell astrocytoma
(SEGA.) It is for this reason that serial
MRI imaging is recommended for this
patient population. Multiple organ sys-
tems are affected in tuberous sclerosis.
Cardiac involvement is most common
in early infancy, often with prenatal
atrial myxomas identified on prenatal
ultrasound. Though these lesions rarely
cause obstructed atrial outflow the nat-
ural history for most of these lesions is
to regress by the age of 12–24 months.
Cutaneous findings are prevalent
..      Fig. 3.21 With permission from ILAE.org.
in TS. Early in life, hypo-pigmented
EpilepsyDiagnosis.org, Diagnostic Manual
macules (ash-leaf spots) are present,
though may require use of a Wood lamp
vv Correct Answer is: A to identify in fair skinned individuals.
A must part of the evaluation in patients With time, the patient may also develop
such as this is detailed skin examina- some or all of the following: shagreen
tion for evidence of a neruo-cutaneous patch, adenoma sebaceum and periun-
syndrome. The MRI pictured shows evi- gual or gingival fibromas. Kidney, lung
dence of hyper-­intensities at the grey- and ophthalmologic involvement is also
white junction in multiple regions of common and should be looked for every
the brain (tubers) and sub-ependymal few years to prevent complications.
nodules, two classic imaging features Diagnosis in the past had been made
of tuberous sclerosis (TS). This disorder with clinical criteria; however, genetic
is inherited in an autosomal dominant testing is now available and especially
fashion, though over 50% of cases used when the patient does not meet
represent spontaneous mutations and diagnostic criteria.
will not have affected family members. Six or more cafe-au-lait macules
Two different genetic loci are respon- greater than 1.5 cm in size are one of the
sible (with varying clinical phenotype diagnostic criteria for neurofibromatosis.
depending on the gene involved) for Linear nevus sebaceous syndrome pres-
encoding the proteins tuberin and ents with varying degrees of epilepsy and
hamartin. Seizures, in particular infan- developmental delay and is associated
tile spasms, are often the presenting with the nevus lesion, usually in the scalp
feature of this disorder. Later in life or over the forehead. Often the seizure
multiple different seizure types pres- focus is ipsilateral to the skin lesion. Mul-
ent, often necessitating lifelong anti- tiple different midline cutaneous lesions
convulsant therapy. Varying degrees of can be seen with underlying tethered
intellectual disability exist, often related cord syndrome to include tufts of hair,
Genetic Disorders
67 3
port-­wine stain or lipoma. Finally, a port- neuronal migration characterized by a
wine stain in the V1 or V2 distribution of cleft that connects the cortical surface
the trigeminal nerve can be associated with the ventricular lumen. The cortical
with Sturge-­Weber syndrome. Realize tissue is usually abnormal in its edges
that only 10% of the time is the port-wine (either polymicrogyria, pachygyria or
stain associated with the full syndrome. both); closed-lip schizencephaly occurs
when the edges are juxtaposed, and
?? 51. You are evaluating an 11-year-old open-lip schizencephaly when the
boy for increasing frequent episodes edges are separated. Perinatal infarction
of staring and unresponsiveness. On of a branch of the middle cerebral artery
examination, he has difficulty with could lead to the same clinical findings,
rapid movements of the right hand, though the imaging study would show
brisk reflexes in the right arm and leg encephalomalacia with an intact ven-
and a tendency to toe walk on the tricular wall.
right. Examination of the skin and eyes Patients with tuberous sclerosis can
is normal. MRI of the brain is shown. have various features of disordered
neuronal migration, though not typi-
cally schizencephalic clefts, and notably
absent in this case are the classic skin
lesions (ash leaf macules) and hyper-
intensities on MRI at the cortical/subcor-
tical junction (cortical tubers) that would
suggest this as the correct diagnosis.
Imaging studies in patients with calcium
or sodium channel disorders would be
expected to be normal.

zz Suggested Reading
55 CONTINUUM: Lifelong Learning in
Neurology: June 2013 - Volume 19 - Issue
3, Epilepsy – p 623–42 doi: 10.1212/01.
CON.0000431379.29065.d3 Review
Articles Neuroimaging in Investigation of
..      Fig. 3.22 American Academy of Neurology
Institute, produced by permission
Patients with Epilepsy Cendes, Fernando
MD, PhD.

What is the most likely etiology of ?? 52. You are seeing a 15-year-old who
his epilepsy? complains of numbness, tingling, and
A. abnormal neuronal migration pain in his arms and legs, problems
B. calcium channel abnormality with balance and coordination, and
C. perinatal stroke vision problems for the last year. He
D. sodium channel abnormality was also diagnosed by the school
E. tuberous sclerosis psychologist as learning disabled
several years ago and placed in a
vv Correct Answer is: A special education class. He was previ-
The MRI pictured is consistent ously healthy, takes no medications,
with an open lip schizencephaly. and no previous hospitalizations. His
Schizencephaly is a type of abnormal vital signs are within normal limits.
68 Chapter 3 · Genetic Disorders

On examination, he has proximal zz Suggested Reading


muscle weakness throughout the 55 Normal Lab Values (7 https://
arms and legs, decreased peripheral s3.amazonaws.com/s3boardvitalsv3/exam_
vision, poor visual acuity, ataxia as resources/normal_lab_values.pdf).
well as sensory neuropathy in the
extremities, and extraocular move- ?? 53. You ordered an autopsy on a
3 ments and eyelid movements are 19-month-old boy who had a progres-
within normal limits. sive neurologic disorder. He had mac-
What is your diagnosis? rocephaly, psychomotor retardation,
A. Usher syndrome spasticity and seizures. Figure shows a
B. NARP syndrome histopathologic specimen obtained at
C. Kearns-Sayre syndrome autopsy.
D. Multiple sclerosis
E. Pompe’s disease

vv Correct Answer is: B


Neuropathy, ataxia, and retinitis pig-
mentosa (NARP) is characterized by a
constellation of symptoms including:
proximal muscle weakness (neuro-
genic), sensory-­motor neuropathy,
ataxia, and retinitis pigmentosa (or
other retinitis). It is an inherited disor-
der of mitochondrial DNA due to muta-
tions in the MT-ATP6 gene. Prevalence
is estimated to be around 1/100,000.
In later stages, cerebellar atrophy can
sometimes be seen on MRI.
Usher syndrome is characterized
by hearing loss (secondary to inner ear ..      Fig. 3.23 Brain of a 4-year-old boy with Alexander
abnormalities) and a gradual visual disease showing macroencephaly and periventricular
impairment (retinitis pigmentosa). leukomalacia (note brownish discoloration around the
cerebral ventricles). (Marvin 101)
Kearns-Sayre syndrome (KSS) also
known as oculocraniosomatic disease
is a rare mitochondrial myopathy that What is the most likely diagnosis?
presents usually before the age of 20. It is A. Canavan disease
characterized by isolated involvement of B. Adrenoleukodystrophy
the muscles controlling the eyelid: leva- C. Alexander disease
tor palpebrae and/or orbicularis oculi, D. Zellweger syndrome
as well as the extra-­ocular muscles. This E. Fabry disease
results in ptosis and ophthalmoplegia
respectively. KSS also involves bilateral vv Correct Answer is: C
pigmentary retinopathy and cardiac con- This is a case of Alexander disease,
duction abnormalities. Other symptoms which is a progressive disorder of astro-
can include cerebellar ataxia, proximal cytes caused by mutations in the gene
muscle weakness, or deafness. for glial fibrillary acidic protein. It has
Genetic Disorders
69 3
an autosomal dominant mode of inheri- ?? 54. Which of the following genetic
tance, although this is controversial. changes is implicated in Williams’s
There are infantile, juvenile, and adult syndrome?
forms. A. Chromosome 22q11.21 deletion
Patients with the infantile form B. Maternal chromosome 1Sq11-q13
have megalencephaly, developmental deletion
delay, seizures, psychomotor retarda- C. Paternal chromosome 1Sq11–13
tion, spasticity, and quadriparesis. The deletion
juvenile form has onset in childhood, D. Chromosome 7q11.23 deletion
and these patients have more significant
bulbar symptoms. The adult form mani- vv Correct Answer is: C
fests with bulbar signs, hyperreflexia, The characteristic features of Williams’s
dysautonomia, ataxia, and sleep apnea. syndrome include, aortic stenosis,
The brain MRI demonstrates diffuse hypercalcemia, elfin facies, and mental
white matter signal hyper-intensity, retardation. It is caused by a chromo-
predominantly in the frontal lobes and some 7q11.23 deletion. Individuals with
anterior cerebral regions, with involve- this condition tend to be very social and
ment of the U fibers. In the adult onset musically talented. Cognitive dysfunc-
form, the “tadpole sign” on sagittal MRI tion is not typically severe.
results from dramatic thinning of the Velocardiofacial syndrome is caused
upper cervical spinal cord. The brains by deletion of chromosome 22q11.21.
of these patients are large, and histo- It is characterized by hypocalcemia,
pathologically, there are Rosenthal conotruncal cardiac malformations, nasal
fibers. The specimen shown in the figure speech, hypertelorism, and cognitive dys-
shows multiple Rosenthal fibers, which function. About a quarter of individuals
are elongated eosinophilic fibers seen with this condition develop psychiatric
on hematoxylin eosin, and they are dif- disease.
fusely distributed throughout the brain Maternal chromosome 1Sq-q13 dele-
with clusters in the subpial, sub-ependy- tion may lead to Angelman syndrome;
mal, and perivascular areas. Rosenthal known as “happy puppet” syndrome.
fiber deposition is associated with severe This is characterized by microcephaly,
myelin loss and cavitation of the white mental retardation, speech difficulties,
matter. and seizures. Movements are often jerky
They are not pathognomonic for in nature and patients may laugh inap-
Alexander disease and are seen in other propriately, hence the nickname “happy
conditions associated with gliosis. There puppet” syndrome. Cerebral and cerebel-
is no specific treatment. lar atrophy are preset; cerebellar atrophy
is typically more severe.1.
zz Suggested Reading Paternal chromosome 1Sq11–13 dele-
55 Comprehensive Review in Clinical tion leads to Prader Willi syndrome which
Neurology: A Multiple-Choice Question is characterized by short stature, obesity,
Book for the Wards and Boards © 2011 mental retardation, and the tendency to
Wolters Kluwer Health Lippincott overeat. During infancy, patients may be
Williams & Wilkins. All rights hypotonic although this improves. Men-
reserved. tal retardation may be severe.
70 Chapter 3 · Genetic Disorders

zz Suggested Reading albumin and increased cholesterol levels


55 Menkes JH, Wilcox WR. Inherited metabolic are characteristic. Mutations in the SETX
diseases of the nervous system. In: Menkes JH, gene lead to ataxia with oculomotor
Sarnat HB, Maria BL. Child neurology. 7th ed. apraxia type 2. This is ­characterized by
Philadelphia: Lippincott Williams and Wilkins. ataxia, in this case starting by age 15,
2006 L. p. 29–142. oculomotor apraxia, myoclonus, chorea,
3 and neuropathy. Elevated alpha feto-
?? 55. Which of the following genes is impli- protein and creatine phosphokinase are
cated in Friedreich ataxia? often present.
A. Ataxia telangiectasia muted ATM
serine/ threonine kinase (ATM) zz Suggested Reading
B. Frataxin (FXN) 55 National Library of Medicine (US).
C. Tocopherol (alpha) transfer pro- Genetics Home Reference [Internet].
tein (TTPA) Bethesda (MD): The Library; 2015 Dec 7.
D. Aprataxin (APTX) FXN; [reviewed 201O May; cited 2015 Dec
13]; Available from: 7 http://ghr.nlm.nih.
vv Correct Answer is: B Gov/gene/FXN (7 http://ghr.nlm.nih.gov/
Friedreich ataxia (FA) is caused by gene/FXN).
increased GAA repeats in the gene 55 National Library of Medicine (US).
frataxin (FXN). FA is caused by repeats of Genetics Home Reference [Internet].
over 66 although those with fewer than Bethesda (MD): The Library; 2015 Dec 7.
300 repeats may not have symptoms TIPA; [reviewed 2008 Apr; cited 2015 Dec
until after age 25. It is characterized by 13]; Available from: 7 http://ghr.nlm.nih.
ataxia, decreased deep tendon reflexes, gov/geneITTPA (7 http://ghr.nlm.nih.gov/
positive Babinski, posterior column dys- geneITTPA).
function, dysarthria, and nystagmus. 55 National Library of Medicine (US).
Mutations in ATM lead to ataxia Genetics Home Reference [Internet].
telangiectasia. Patients present with Bethesda (MD): The Library; 2015 Dec 7.
ataxia, telangiectasias, immune defi- Ataxia with vitamin E deficiency;
ciency, and increased risk of malignancy. [reviewed 2008 Apr; cited 2015 Dec 13];
Those patients are sensitive to ionizing Available from: 7 http://ghr.nlm.nih.gov/
radiation. Mutations in TIPA lead to condition/ataxia-with-vitamin-edeficiency
ataxia with vitamin E deficiency. This (7 http://ghr.nlm.nih.Gov/condition/
is characterized by movement abnor- ataxia-with-vitamin-e-deficiency).
malities, dysarthria, lower extremity 55 National Library of Medicine (US).
areflexia, and peripheral neuropathy. Genetics Home Reference [Internet].
Patients may also have retinitis pig- Bethesda (MD): The Library; 2015 Dec 7.
mentosa. The alpha tocopherol transfer Ataxia with oculomotor apraxia; [reviewed
protein is necessary for processing of 2015 Apr; cited 2015 Dec 13]; Available
vitamin E in the diet. Mutations in the from: 7 http://ghr.nlm.nih.gov/condition/
gene APTX are implicated in ataxia with ataxia-with-oculomotor-apraxia (7 http://
oculomotor apraxia type 1. This gene is ghr.nlm.nih.gov/condition/ataxia-with-
implicated in DNA repair and leads to oculomotor-apraxia).
increased cerebellar cell death. This is 55 Bird TD, Jayadev. Genetic disease of the
characterized by ataxia starting by age nervous system, In: Rosenberg RN editor.
5 with myoclonus, oculomotor apraxia, Atlas of clinical neurology. Philadelphia:
chorea, and neuropathy. Decreased Springer; 2009.
Genetic Disorders
71 3
?? 56. You are seeing a 15-year-old male Inheritance of a phenotype (or dis-
with a progressive cerebellar ataxia, ease) requires the only one (dominant)
pigmentary retinopathy and a recent copy from either parent on an autosomal
diagnosis of progressive external oph- chromosome.
thalmoplegia.
What is the mode of inheritance in zz Suggested Reading
this case? 55 Angelini, C. Kearns-­Sayre syndrome. In:
A. X-linked Genetic ­Neuromuscular Disorders.
B. Autosomal recessive Springer International Publishing. 2014.
C. Autosomal dominate p. 225–8.
D. Maternal
?? 57. You are evaluating a 2-year-old girl with
vv Correct Answer is: D progressive visual loss since infancy,
This is a case of Kearns-Sayre syndrome associated with midface hypoplasia,
(KSS), which is a mitochondrial disorder. enophthalmos and hypermetropia?
Inherited mitochondrial pattern follows What is the likely diagnosis?
maternal inheritance. Egg cells, not A. Leber congenital amaurosis
sperm cells, contribute mitochondria B. Bardet-Biedle syndrome
DNA to the developing embryo, only C. Wolfram syndrome
females pass mitochondrial conditions D. Pituitary adenoma
to their children. Mitochondrial dis-
orders can appear in each generation vv Correct Answer is: A
of a family and affects both males and Leber congenital amaurosis (LCA) is a
females, Fathers do not pass mitochon- group of disorders of recessive retinal
drial traits to their children. dystrophies and is the most common
1. KSS is characterized by: Progressive cause of congenital visual impairment.
external ophthalmoplegia. Characteristics of LCA are moderate-
2. Disease onset before age 20 and severe visual impairment at or near
3. At least one of the following; short birth, along with nystagmus, and
stature, pigmentary retinopathy, sluggish pupillary responses. Physical
cerebellar ataxia, heart block, and stigmata include mid-facial hypoplasia,
elevated cerebrospinal fluid protein enophthalmos, hypermetric and refrac-
(>100 mg/dl). tive errors. Retinal examination shows
Many cases of KSS are believed to be no abnormality in infancy and early
sporadic mitochondrial DNA deletions or childhood, but progressive retinal stip-
duplications. pling and pallor of the disk gradually
Inheritance by a gene on the X chro- develop. Mental retardation may be
mosome that causes the phenotype to be associated.
expressed in males who are hemizygous Bardet-Biedle syndrome is character-
for the gene mutation and in females ized by pigmentary retinopathy, mental
who are homozygous for the gene muta- retardation, polydactyly, obesity and
tion (received copy of the gene on each hypogenitalism.
of their two X chromosomes. Inheritance Wolfram syndrome is characterized by
of a phenotype (or disease) requires the optic atrophy, bilateral sensory hearing
same copy of a gene in both copies a loss, diabetes insipidus and mellitus.
child receives each parent on an autoso- Pituitary adenomas account for 2%
mal chromosome. of childhood of all childhood intracranial
72 Chapter 3 · Genetic Disorders

tumors. Optic atrophy occurs in up to shoulder girdle and humeral muscle


20% OF patients. Visual field deficits weakness in more prominent.
include unilateral temporal, bi-temporal Juvenile progressive bulbar palsy
hemianopia, or even homonymous. includes weakness of tongue, facial, pha-
ryngeal and ocular muscles with a fluctu-
zz Suggested Reading ant course.
3 55 Weleber RG, Francis PJ, Trzupek KM, The most frequent clinical presenta-
Beattie C. (2013). Leber congenital tion of juvenile le myasthenia gravis is
amaurosis. QID: 55258. ptosis, which is often associated with
other ocular symptoms namely unilat-
?? 58. You are evaluating a 15- year-old eral or asymmetric ophthalmoplegia,
male for a progressive sensorineural strabismus, and lid twitch. Children with
hearing loss over the last 2 years. Few myasthenia gravis have lower frequency
months ago, he developed bilateral, of acetylcholine receptor antibodies
asymmetrical moderate facial weak- and a greater probability of achieving
ness and complained of trouble swal- ­remission.
lowing. He has tongue atrophy on
neurological exam otherwise the rest zz Suggested Reading
of the cranial nerves are intact. He 55 Foley AR, et al. Treatable childhood
had a mild increase in muscle tone in neuronopathy caused by mutations in
his extremities. riboflavin transporter RFVT2. Brain.
What is the most likely diagnosis? 2014;137(Pt 1):44–56.
A. Facioscapulohumeral dystrophy
B. Juvenile progressive bulbar palsy ?? 59. What is the most likely diagnosis in a
C. Pontobulbar Palsy with Deafness 4-year-old boy with congenital deaf-
D. Juvenile myasthenia gravis ness, who now presents with a goiter?
A. Pendred syndrome
vv Correct Answer is: C B. Infantile Refsum disease
Ponto-bulbar Palsy with Deafness C. Hypomelanosis of Ito
(Brown-Vialetto-Van Laere syndrome) is D. Vitiligo
an autosomal recessive neuronopathy
characterized by ponto-bulbar palsy vv Correct Answer is: A
affecting lower cranial nerves (primarily Sensorineural hearing impairment is
VII-XII). present at birth and is severe in about
Sensorineural deafness is often the half of the cases of Pendred syndrome.
leading sign, followed by facial weakness A diffuse, non-nodular goiter most often
and dysphagia. Tongue atrophy occurs develops during the first decade, often
in most cases, but masseter and ocular in infancy. No clinical signs of hypothy-
motor palsies are uncommon. About 50% roidism are present. Growth and intel-
will have pyramidal tract dysfunction. ligence are usually normal.
Riboflavin supplementation has been Infantile Refsum disease is a disorder
reported benefit some patients. Patients of peroxisome biogenesis and is charac-
mostly common die from respiratory terized by early onset cognitive impair-
insufficiency. ment, retinitis pigmentosa, sensorineural
Facioscapulohumeral dystrophy hearing deficit hepatomegaly, osteopo-
begins in the second decade of life rosis, failure to thrive and hypocholester-
and includes facial weakness, but the olemia.
Genetic Disorders
73 3
Hypomelanosis of Ito (Ito syndrome) 2. Infantile spasms.
produces symmetric depigmented streaks 3. Choreoretinal lacunae.
or swirls on the skin. Some individuals can Additional brain abnormalities in
have wide­set eyes or auricle anomalies, Aicardi syndrome include – asymmetry
but hearing loss is not associated. Vitiligo between hemispheres, polymicrogy-
is an autoimmune disorder characterized ria, cysts, and microcephaly. Other
by depigmented patches of skin. Hearing ophthalmic abnormalities include –
loss is not associated with this condition. microphthalmia or coloboma, which
can cause blindness. Facial features may
zz Suggested Reading include – small philtrum, a flat nose with
55 Smith RJ, Shearer AE, Hildebrand MS, Van an upturned tip, large ears, and sparse
Camp G. Deafness and hereditary hearing eyebrows. Other features include – small
loss overview. 2014. hands, scoliosis. They often have gastro-
intestinal problems such as constipation
?? 60. Polymicrogyria is associated or diarrhea, gastroesophageal reflux, and
with which one of the following difficulty feeding.
­conditions?
A. Chiari 1 malformation zz Suggested Reading
B. Aicardi syndrome 55 Mowzoon, Nima, Kelly D. Flemming.
C. Dandy walker malformation Neurology board review: an illustrated
D. Syringomyelia study guide. Rochester, MN: Mayo Clinic
E. Chiari II malformation Scientific, 2007. Print.

vv Correct Answer is: B ?? 61. You are evaluating a 16-year-old with


Polymicrogyria is some excessive, severe generalized myoclonus, ataxia,
numerous small gyri. It may occur in and loss of color vision. He has a
combination with pachygyria. It is cherry red spot on fundus exam. EEG
usually due to ischemic insult to the demonstrated background slowing,
underlying parenchyma but can be due and photosensitive bilateral fast spike
to defective migration. Other acquired and wave activity.
intrauterine insults include infections What choice is correct to establish
such as toxoplasmosis and cytomega- the diagnosis?
lovirus (and others, including rubella A. An alpha-N-­acetylgalactosami
and herpes simplex). Associated with nidase deficiency
X-linked dominant Aicardi’s syndrome B. A decrease in cultured skin fibro-
and may occur in the context of specific blast alpha-neuraminidase
inherited metabolic disorders, such as C. A response to oral biotin
peroxisomal disorders (e.g., Zellweger’s D. A hexosaminidase A deficiency
syndrome). Clinical presentation varies, E. A muscle biopsy with ragged red
depending on the associated condition. fibers
If involvement is focal, patients may be
asymptomatic or present with seizures. vv Correct Answer is: B
Aicardi syndrome is a disorder that Schindler disease is caused by a defi-
occurs almost exclusively in females. It ciency of alpha-N-acetylgalactosamin-
has three main features: idase. It is a rare autosomal recessive
1. Agenesis or dysgenesis of the corpus disorder with acute infant and chronic
callosum. adult-onset forms. Infants demonstrate
74 Chapter 3 · Genetic Disorders

severe psychomotor deterioration with ?? 62. Which of the following conditions is


spasticity, cortical blindness, and myo- associated with retinitis pigmentosa?
clonic epilepsy with an exaggerated A. HARP syndrome
startle response. The adult form is char- B. Glaucoma
acterized by mild cognitive deficits. C. Diabetic retinopathy
Sialidosis type 1 A manifestations D. Multiple Sclerosis
3 include severe generalized myoclonus, E. Mc leod Syndrome
night blindness, loss of color vision,
ataxia, cherry-red spots, and generalized vv Correct Answer is: A
convulsions. In this case, the EEG shows HARP stands for Hypopre­
progressive slowing and bilateral, pho- betalipoproteinemia, Acanthocytosis,
tosensitive fast spike-wave discharges. Retinitis pigmentosa, and Pallidal
A decrease in cultured skin fibroblast degeneration. It is a rare disease of
alpha-neuraminidase is diagnostic. childhood similar to Hallervorden-­Spatz
A response to oral biotin would be disease in that there is a defect in the
consistent with biotinidase deficiency. gene for pantothenate kinase (the first
This disorder is characterized by hypo- enzyme in the Coenzyme A synthe-
tonia, generalized tonic-clonic and myo- sis pathway). Glaucoma and diabetic
clonic seizures, rash, and alopecia, with retinopathy are important causes of
onset in infancy. The EEG demonstrates ­blindness but not retinitis
multifocal spikes and slow waves. pigmentosa.
A hexosaminidase A deficiency is Mcleod syndrome causes acanthosis,
present in Tay-Sachs disease, with the but not retinitis pigmentosa. The neuro-
onset of development delay, cherry-red acanthocytoses are a group of genetically
spots, and “startle seizures” in infancy. diverse conditions that cause movement
A muscle biopsy with ragged red disorders, neurological problems and
fibers is indicative of myoclonic epilepsy acanthosis (speculated red blood cells)
with ragged red fibers (MERRF). The core including chorea acanthocytosis, Mc
features of this mitochondrial disorder leod syndrome, Huntington’s disease-like
include myoclonus, epilepsy, myopathy, 2 (HDl2) and Hallervorden-­Spatz (also
intellectual disability, and cerebellar dys- called pantothenate kinase-associated
function. Patients may also have short neurodegeneration (PKAN)).
stature, deafness, optic atrophy, cutane-
ous lipomas, and neuropathy. The onset is ?? 63. Which one of the following condi-
typically in childhood or early adulthood. tions is associated with Chiari type I
­malformation?
zz Suggested Reading A. Myelomeningocele
55 Verma A, Moraes CT. Mitochondrial B. Holoprosencephaly
disorders. In: Bradley WG, Daroff RB, C. Syringomyelia
Fenichel GM, Jankovic J, editors. D. Cerebellar atrophy
Neurology in clinical practice. 4th ed. E. Corpus callosum agenesis
Elsevier, Inc.; 2004. p. 781–808.
55 Pastores GM, Kolodny EH. Inborn errors vv Correct Answer is: C
of metabolism of the nervous system. In: Chiari type I malformation: caudal dis-
Bradley WG, Daroff RB, Fenichel GM, placement of cerebellar tonsils through
Jankovic J, editors. Neurology in clinical the foramen magnum. Mean age at
practice. 4th ed. Elsevier, Inc.; 2004. presentation: 41 years. Chiari type I
p. 1811–32. malformation may be associated with:
Genetic Disorders
75 3
Hydrocephalus, Intermittent increase D. Gaucher disease
in intracranial pressure, Syringomyelia, E. GM1 gangliosidosis
Klippel-Feil syndrome, basilar impres-
sion, and occipitalization of the atlas, vv Correct Answer is: B
Compression of brainstem structures Fabry disease is an X-linked disorder
and some rostral extension of the caused by deficiency of the enzyme
medulla. Patient may be asymptom- A-­galactosidase, resulting in accumula-
atic. Presenting symptoms: headaches, tion of ceramide trihexoside in epithe-
especially brought on by neck extension lial, mesenchymal, and neural cells. The
or the Valsalva maneuver (most com- presenting manifestations start in child-
mon presenting symptom), sometimes hood or adolescence, presenting with
associated with tinnitus, nausea, and dysesthesias, lancinating pain, and epi-
vomiting. sodes of burning sensation from small
Various cerebellar symptoms, lower fiber neuropathy, which also may be
cranial nerve dysfunction, diplopia and associated with autonomic dysfunction.
downbeat nystagmus, dissociated sen- Dermatologic manifestations include
sory loss (loss of pinprick and tempera- the characteristic angiokeratomas,
ture sensation and relative preservation which are more prominent in the lower
of large fiber modalities), and possibly abdomen and legs, especially in the
pyramidal tract involvement. groins, hips, and periumbilical regions,
Surgical treatment: early surgery for and consist of cutaneous telangiecta-
symptomatic patients and observation sias. Cardiac involvement manifests
for asymptomatic ones. with valvular disease, arrhythmias,
cardiomyopathy, and ischemic heart
zz Suggested Reading disease. There is also renal involvement
55 Mowzoon N. Flemming KD. Neurology from endothelial and glomerular dam-
board review: an illustrated study guide. age, causing acute renal failure and
Rochester, MN: Mayo Clinic Scientific, eventually chronic renal disease leading
2007. Print. to hypertension and uremia. Vascular
compromise arises from endothelial and
?? 64. You are seeing an 18-year-old male vascular smooth muscle involvement
who has an acute ischemic stroke. and is associated with ischemic stroke.
His medical history is remarkable of Another frequent clinical finding is cor-
cardiomyopathy of uncertain cause, neal opacity.
severe hypertension, and end-stage Pathologically, there is lysosomal
renal disease. His first symptoms con- storage of birefringent lipids, with
sisted of a burning sensation in his membrane-bound lamellar deposits on
feet. His family history is remarkable electron microscopy.
of three brothers who died of end Treatment includes enzyme replace-
stage renal disease. His renal biopsy ment therapy.
was obtained and showed birefrin- Metachromatic leukodystrophy is
gent lipid deposits in the glomeruli, caused by deficiency of arylsulfatase A.
and electron microscopy showed Niemann-Pick Type A is caused by
membrane bound lamellar deposits. acid sphingomyelinase deficiency.
What is the most likely diagnosis? Gaucher disease is caused by defi-
A. Metachromatic leukodystrophy ciency of the enzyme glucocerebrosi
B. Fabry disease dase. GM1 gangliosidosis is caused by8-
C. Niemann-Pick Type A galactosidase deficiency.
76 Chapter 3 · Genetic Disorders

?? 65. You ordered a brain specimen accumulation of galactocerebroside in


obtained from an autopsy of a macrophages of the white matter in the
10-month-old baby. The patient had CNS, leading to the formation of glo-
irritability and hypersensitivity to boid cells and to progressive demyelin-
stimuli at 3 months of age. He was ation, but with sparing of the U fibers.
also blind and had prominent regres- The cause is a deficiency of the enzyme
3 sion by age 6 months. He also had galactosylceramidase (also known as
developed stiffness of all four limbs galactocerebroside galactosidase).
and later of the trunk to the point that It can involve the peripheral nervous
he was in an opisthotonic posture. system, leading to a demyelinating
neuropathy, but affects predominantly
the CNS.
There are three forms: –
1. Infantile form: the most common
variant and presents between 4 and
6 months of age with irritability,
hypersensitivity to stimuli, increasing
hypertonicity with eventual opisthoto-
nos, unexplained low-grade fevers,
and optic atrophy with blindness,
psychomotor developmental arrest,
and subsequent regression with loss of
previously achieved milestones. These
children also have a demyelinating
..      Fig. 3.24 Globoid cell leukodystrophy PAS –
­ ulinucleated macrophages (“globoid cells”) and loss
M polyneuropathy with arreflexia. They
of myelinated fibers in a case of Krabbe’s leukodystro- usually die by the age of 1 year.
phy. (Jensflorian. Wikipedia) 2. Juvenile form: the onset is between
3 and 10 years of age, with vision loss,
Which of the following is correct spasticity, ataxia, gait disturbance, and
regarding this condition? cognitive impairment.
A. It is autosomal dominant 3. Adult form: usually starts between the
B. The deficient enzyme is 8-gluco- third and the fifth decade of life, with
sylceramidase spastic paraparesis, weakness, vision
C. There is demyelination with rela- loss, and evidence of neuropathy, but
tive sparing of the U fibers intellectual function tends to be
D. It is only seen in newborns, and all normal.
patients die by 1 year of age Pathologically, there is symmetric
E. The cause is deficiency of the demyelination of the cerebral white mat-
enzyme A-galactosidase ter, with relative sparing of the subcorti-
cal arcuate or U fibers. The pathologic
vv Correct Answer is: C specimen shown in the above figure is
This patient has Krabbe disease on the typical of Krabbe disease, demonstrating
basis of the history and histopathologic clusters of globoid cells, which are multi-
findings. Krabbe disease or globoid cell nucleated macrophages with cytoplasmic
leukodystrophy is a disorder with auto- accumulation of galactocerebroside.
somal recessive inheritance, with the Radiologically, there is symmetric
affected gene mapped to chromosome periventricular white matter signal
14. This disorder is characterized by the abnormality and cerebral atrophy.
Genetic Disorders
77 3
NCS demonstrate slow conduction ?? 67. You are seeing a 10-month-old girl
velocities and prolonged distal with hypotonia. Her medical history
latencies. is remarkable to have low muscle
CSF examination shows elevated tone at 6 months. She has attained
­protein levels. few developmental milestones since
that time and recently was noted to
?? 66. You are evaluating a 15-year-old boy have an exaggerated started response
with growth retardation, generalized to loud noises. On examination, she
weakness, and ataxia. On examina- has an enlarged liver. She has been
tion, he has bilateral ptosis and referred to an ophthalmologist for
restricted gaze in all directions. An mild cornea clouding revealed a
electrocardiogram is obtained and cherry-red spot.
shows complete heart block. What is the hereditary genetic
What is the most likely diagnosis? ­pattern of this condition?
A. Kearns-Sayre syndrome A. Autosomal dominant
B. Mitochondrial encephalopathy, B. Autosomal recessive
lactic acidosis, and strokes C. X-linked
C. Myoclonic epilepsy with ragged D. Sporadic
red fibers
D. Leigh disease vv Correct Answer is: B
E. Myasthenia gravis Infantile GM1 gangliosidosis is an auto-
somal inherited disorder caused by a
vv Correct Answer is: A mutation in the GLB1 gene that pro-
This is a presentation of Kearns-Sayre duces beta-galactosidase.
syndrome, which is a disorder caused This results in an accumulation of
by multiple mt DNA deletions. The mucopolysaccharides in cells causing dys-
diagnosis is made with the triad of function and cell death. Onset is between
progressive external ophthalmoplegia, 6 and 18 months. Hypotonia progresses
onset before the age of 20 years, and to spasticity are accompanied with sei-
at least one of the following: short zures, cognitive and visual impairment.
stature, retinitis pigmentosa, cerebel- Death usually occurs by 7 years of age.
lar ataxia, heart block, and increased The absence of mucopolysaccharidiura
CSF protein (>100 mg/dl). Chronic and the presence of a cherry-­red spot
­progressive externalophthalmoplegia differentiate GM1 gangliosidosis from
may be an isolated finding seen in some Hurler syndrome.
patients. Patients with Kearns-Sayre Tay-sachs disease (infantile GM2
syndrome have a gradual progres- gangliosidosis) may present with similar
sion of s­ ymptoms and most will have symptoms, but occurs almost exclusively
cognitive regression by third or fourth in Ashkenazi Jews. Type II GM1 gangliosi-
decade of life. Most cases are sporadic. dosi s consists of intermediate forms of the
An e­ lectrocardiogram is required to condition. The third type of GM1 ganglio-
diagnose heart block, in which case, sidosis is an adult form and is the mildest
a pacemaker needed. Pathologically, manifestation of the disease spectrum.
patients may have muscles with
ragged red fibers and white matter zz Suggested Reading
showing spongy myelinopathy 55 Kannebley JS. Moryama LS, Bstos LOD,
without gliosis or ­macrophage Guerreiro MM, Amato A, Min LL, Steiner CE.
­reactions. (2014, April). Clinical and radiologic
78 Chapter 3 · Genetic Disorders

features in 12 patients’ with juvenile and zz Suggested Reading


adult GM1 Gangliosidosis. In: 13th 55 Mowzoon N, Flemming, KD. Neurology
International Child Neurology Congress board review: an illustrated study guide.
(ICNC2014). Rochester, MN: Mayo Clinic Scientific,
2007. Print.
?? 68. Which one of the following is
3 ­associated with Chiari type II ?? 69. Which of the following is not true
­malformation? regarding periventricular nodular het-
A. Anencephaly erotopia?
B. Encephalocele A. Heterotopias are clusters of defec-
C. Cerebellar atrophy with enlarged tive neurons in an area of other-
posterior fossa wise normal cortex
D. Lissencephaly B. It is a disorder of neuronal
E. Myelomeningocele ­migration
C. It is more common in females
vv Correct Answer is: E D. It is most commonly X-linked
Chiari type II malformation: caudal E. Seizures are a common clinical
displacement of cerebellar tonsils, manifestation
cervico-­medullary junction, pons,
fourth ventricle, and medulla. Chiari vv Correct Answer is: A
type II malformation is associated with: Heterotopia is a cluster of abnormally
Myelomeningocele; Hydrocephalus; located neurons that are otherwise nor-
Colpocephaly of the lateral ventricles; mal in morphology. Heterotopias result
Elongated fourth ventricle; interdigitat- from abnormal neuronal migration.
ing gyri (associated with hypoplastic The most common form of heteropia
falx cerebri); Skull abnormalities: lacu- is periventricular nodular heterotopia
nar skull, hypoplastic; tentorium cer- (also known as subependymal nodu-
ebelli, gaping foramen magnum. Other lar heterotopia), located, as the name
major findings: Medullary “spur” or implies, near the lateral ventricles, but
“kink” as a result of caudally; displaced, heterotopias can be found in various
elongated medulla. Tectal “beaking” regions (such as subcortical nodular
of the midbrain due to pressure from heterotopias, or transmantle heteroto-
herniated cerebellum. Hydrocephalus in pias, which extend from the ventricles
90% of patients. to the cortex).
Presentation: Variable age at onset In periventricular nodular heteroto-
usually neonatal to early childhood. pia, neurons are thought to have never
Neonatal presentation with rapid neu- begun migration but rather remained
rologic deterioration, including weak in the sub-­ventricular area (where corti-
or absent cry, respiratory failure, and cal neurons originate). This disorder is
strider, apneic spells, swallowing difficul- most often bilateral, but it can also be
ties from involvement of the medulla. unilateral. In more than half of cases,
Hydrocephalus, weakness including this disorder results from mutations in
facial weakness, lower cranial nerve the FLNa gene on chromosome X. This
abnormalities. FLNa gene encodes a protein involved
Treatment: surgical decompression in actin cytoskeleton reorganization,
and cerebrospinal fluid (CSF) shunt for and abnormalities in this protein lead to
hydrocephalus. disruption in cell migration by impeding
Genetic Disorders
79 3
the generation of forces needed for Trisomy 18, or Edwards’s syndrome,
cell movement. Periventricular nodular is characterized by intellectual disability,
heterotopias may be seen in isolation or pointed ears, micrognathia, microceph-
as part of a syndrome associated with aly, an occipital protuberance, a narrow
multiple other anomalies. Epilepsy is a pelvis, heart defects, and rocker bottom
common clinical manifestation, with par- feet. Intrauterine growth retardation
tial seizures being most common. Intel- and low birth weights are common.
ligence may be entirely normal; males are Many infants die before or shortly after
more likely to have developmental delay. birth, with only 5–10% living past the
first year.
?? 70. You were asked to see an infant who Trisomy 21, or Down syndrome, has
has moon-like facies, hypertelorism, multiple systemic manifestations and a
microcephaly, low set ears, hypotonia, variable phenotype. Common findings
and a high-pitched cry. include intellectual disability, hypotonia,
What chromosomal analysis are you characteristic facial features (e.g. upward
going to order? slanted eyes, protruding tongue, flat-
A. 5p monosomy tened face, small nose with a depressed
B. Trisomy 13 nasal bridge), and Brush field spots. Early
C. Trisomy 18 onset Alzheimer’s disease, heart defects,
D. Trisomy 21 and an elevated risk for leukemia are also
E. Partial monosomy 13q associated features.
Partial monosomy 13q occurs when
vv Correct Answer is: A a portion of the long arm (q) of chromo-
The clinical presentation indicates a some 13 is deleted or monosomic. The
5p monosomy. The syndrome is known phenotype is variable, depending on the
as “cri du chat,” meaning “cry of the size and location of the deletion. The syn-
cat,” reflecting the characteristic high- drome may be characterized by low birth
pitched cry. Associated manifestations weight, craniofacial malformations, ocu-
include moon-like facies, low set ears, lar abnormalities, defects of the hands
a small jaw, orbital hypertelorism, and/or feet, urogenital malformations,
microcephaly, MR, hypotonia, low birth and intellectual disability.
weight, and heart defects. Loss of the
CTNND2 gene on chromosome 5 may be zz Suggested Reading
associated with particularly severe intel- 55 Published 4/21/14. Accessed 4/28/14.
lectual disability. 7 http://ghr.nlm.nih.gov/condition/
Trisomy 13, or Patau syndrome, con- trisomy-13 (7 http://ghr.nlm.nih.gov/
sists of intellectual disability, microceph- condition/trisomy-13). Reviewed 11/13.
aly, holoprosencephaly, heart and spinal Published 4/21/14. Accessed 4/28/14.
cord defects, seizures, and hypotonia. 7 http://ghr.nlm.nih.gov/condition/
Other malformations include polydac- trisomy-18 (7 http://ghr.nlm.nih.gov/
tyly, a single palmar crease, cleft lip and condition/trisomy-18). Reviewed 3/ 12.
palate, low set ears, hernias, and crypt- Published 4/21/14. Accessed 4/28/14. Chen
orchidism. Patients may have coloboma, H. Down Syndrome. 7 http://emedicine.
microphthalmia, close-set eyes, or fusion medscape.com/article/943216-overview
into one eye. Many infants die shortly (7 http://emedicine.medscape.com/
after birth, with only 5–10% living past article/943216-overview). Updated 4/10/14.
1 year. Accessed 4/27/14.
80 Chapter 3 · Genetic Disorders

?? 71. A 5-month-old baby is referred to you development is normal, it will typically


because he has a small, midline tuft continue to be so. However, associated
of hair over the lower lumbar region. neurologic dysfunction may present
His examination is otherwise entirely future neurologic impairment. When
normal. You decided to see him for there is associated neurologic dysfunc-
follow-up at 3 years of age, the child tion in a child with a tuft of hair over
3 continues to have normal devel- the lumbar region but with no other
opment and a normal neurologic evidence of neural tube defect, the
­examination. disorder is named occult spinal dysra-
Which of the following statements phism. In occult spinal dysraphism, a
regarding this child is true? variety of developmental abnormalities
A. On the basis of the finding of this may be seen involving the spinal cord
tuft of hair, it can be concluded or roots and posterior fossa, and asso-
that he will eventually have sig- ciated findings may include dermoid
nificant cognitive delay, even if he or epidermoid cysts, intra-spinal or
has developed normally up until cutaneous lipomas, and tethered cord.
the age of 3 years Diastematomyelia, or splitting of the
B. On the basis of the finding of this spinal cord, may also be seen. Rarely,
tuft of hair, it can be concluded a sinus tract connects the dura with
that he will eventually have the surface of the skin. In occult spinal
significant motor and cognitive dysraphism, neurologic manifesta-
delay even if he has developed tions vary widely and may range from
normally up until the age of minimal motor deficits and ankle hypo-
3 years reflexia to bowel and bladder dysfunc-
C. This tuft of hair signifies a possible tion, sensory loss, and paraparesis or
underlying defect in the posterior paraplegia. Although patients may be
bony component of the vertebral initially asymptomatic, these neurologi-
column cal deficits can develop suddenly and
D. This tuft of hair signifies the pres- be irreversible.
ence of a myelomeningocele
E. On the basis of the finding of this ?? 72. You are evaluating a 4-year-old girl.
tuft of hair, it can be concluded Her family reports that, she seemed
that abnormalities in the spinal to be a very happy baby early on life,
cord will definitely be seen on she made eye contact and smiled
imaging and cooed all the time. She had met
motor milestones similar to her sib-
vv Correct Answer is: C lings: holding her head up, crawling,
This is a clinical presentation of spina and sitting unsupported. However,
bifida occulta. This is a defect in the around the age of 18 months, she
bony components along the poste- started to have difficulty sitting up
rior aspect of the vertebral column. and never developed any words. She
It can often be asymptomatic, but an later stopped smiling and laughing,
abnormal conus medullaris and filum and progressively lost use of her
terminale are possible. The presence hands, constantly rubbing her hands
of a tuft of hair, implying underlying against each other.
spina bifida occulta, does not necessar- Which of the following statements is
ily imply impending cognitive or motor true regarding the most likely diag-
delay. In fact, when early neurologic nosis in this patient?
Genetic Disorders
81 3
A. It is more common in males but of g
­ alactosylceramide beta-galactosi-
can also be seen in females dase activity in his leukocytes?
B. Macrocephaly is a common What is your diagnosis?
­finding A. GM1 gangliosidosis
C. It results from a mutation in the B. Tay-Sacks disease
gene encoding the MeCP2 protein C. Globoid cell leukodystrophy
D. It is autosomal dominant in D. Sphingomyelin lipidosis
­inheritance
E. It results from mutations in mito- vv Correct Answer is: C
chondrial DNA Globoid cell leukodystrophy, also
referred to as Krabbe disease, is a rap-
vv Correct Answer is: E idly progressive white matter demyelin-
This is a case of Rett syndrome, a syn- ating disorder that is transmitted by
drome of motor and cognitive regres- autosomal recessive inheritance of an
sion with eventual severe disability. enzyme defect in galactosylceramide
The presentation is one of early normal beta-galactosidase. After 2–4 months
development with subsequent regres- after onset, the children typically
sion at approximately 6–18 months of regress and develop the position of
age. Hand wringing and other motor opisthotonos.
stereotypies are a classic feature; GM1 gangliosidosis is caused by a
patients with Rett syndrome often place defect in beta-glactosidase and presents
their hands in their mouth or may hold with weakness and incoordination.
their hands fisted, with their fingers Tay-Sacks is caused by a deficiency in
flexed over their thumb. Arrest of head hexosaminase A and is associated with
growth with eventual microcephaly, progressive cognitive function decline
seizures, scoliosis, dysautonomia includ- and severe spasticity by 1 year.
ing respiratory dysfunction with apneas, Sphingomyelin liposis is caused by
and spasticity emerge as the disease a deficiency of sphingomyelinase that
progresses. It results from various types most commonly presents with feeding
of mutations in the gene that encodes difficulty, failure to thrive and hepato-
methyl CpG binding protein 2 (MeCP2), splenomegaly.
which is involved in binding to methyl-
ated DNA, modulating gene expression. zz Suggested Reading
It is most often seen in females, and it is 55 Gieselmann V, Wenger DA, Krageloh-
thought to be most often fatal in boys, Mann I. Metachromatic Leukodystrophy
although cases of MeCP2 mutation in and Globoid Cell Leukodystrophy.
male infants and children with intellec- Lysosomal Storage Disorders: A Practical
tual disability and other features have Guide, 2012. p. 70–9.
been identified. Another disorder of
severe intellectual disability in females ?? 74. You are seeing a 5-month-old baby
results from a mutation in the gene boy for psychomotor retardation.
CDKLS. After birth, he was noticed to have
prominent nystagmus, abnormal
?? 73. You are seeing a 6-month-old boy eye movements, and prominent
with a 2-month history of irritabil- ­incoordination. MRI demonstrates
ity and hyper-startle responses that diffuse white matter changes spar-
recently developed hypertonicity and ing the U fibers, with a “tigroid”
was found to have a severe deficiency ­appearance.
82 Chapter 3 · Genetic Disorders

Which statement is correct regard- Alexander disease, which is caused by


ing this disorder? a defect in the gene for glial fibrillary
A. It is inherited in an autosomal acidic protein (GFAP).
recessive fashion
B. The gene involved is PLP1 zz Suggested Reading
C. Central and peripheral myelin are 55 Comprehensive Review in Clinical
3 affected Neurology: A Multiple-Choice Question
D. This patient has Alexander disease Book for the Wards and Boards ©2011
E. The mutation is in the gene for Wolters Kluwer Health Lippincott
glial fibrillary acidic protein (GFAP) Williams & Wilkins. All rights reserved.

vv Correct Answer is: B ?? 75. Which of the following statements


The presentation indicative of is not true regarding Dandy-Walker
Pelizaeus-­M erzbacher disease, which ­malformation?
is a demyelinating disorder inherited A. It includes cerebellar vermis
in an X-linked recessive fashion. The ­hypoplasia
gene involved is PLP1 on chromo- B. It includes fourth ventricle cystic
some Xq22, and the mutation leads dilatation
to abnormal synthesis of proteolipid C. Lateral and third ventricular
protein 1. Mutations in this same hydrocephalus is common
gene also account for one form of D. It may present in infancy with
hereditary spastic paraplegia. The macrocephaly or may be asymp-
onset of clinical manifestations is in tomatic into adulthood
the first few months of life, with inter- E. It is associated with the “molar
mittent nodding movements of the tooth sign” on imaging
head, pendular nystagmus, and other
abnormal eye movements. Ataxia, vv Correct Answer is: E
chorea, athetosis, dystonia, spasticity, The molar tooth sign is seen in various
and laryngeal strider also occur, and disorders associated with cerebellar
psychomotor development arrests hypoplasia, including Joubert syndrome,
with subsequent regression. Late COACH syndrome (cerebellar vermis
manifestations include seizures and hypoplasia, oligophrenia, congenital
optic atrophy. Patients with later onset ataxia, coloboma, and hepatic fibrocir-
may have slower progression, and rhosis), and Leber congenital amaurosis.
some patients survive into adulthood. However, this radiologic sign is not seen
The MRI demonstrates diffuse demy- in Dandy-Walker malformation. Dandy-
elination. Pathologically, there is a Walker malformation is characterized
non-inflammatory demyelination spar- by cerebellar vermis hypoplasia, fourth
ing the U fibers and islands of white ventricular cystic dilatation, and eleva-
matter, giving it a “tigroid” appear- tion of the torcula and the tentorium
ance. Peripheral myelin is spared; and cerebelli. Posterior fossa enlargement
therefore, peripheral nerves are not and hydrocephalus are common. It is
involved. Genetic testing is available associated with various chromosomal
for diagnosis, and there is no specific anomalies. Neural tube defects, including
treatment. This patient does not have encephalocele, and anomalies in other
Genetic Disorders
83 3
organ systems, ­including the heart, may zz Suggested Reading
occur. Another association is with facial 55 Fenichel GM. Clinical pediatric neurology.
hemangiomas. Clinical presentation is 5th ed. Philadelphia: Elsevier Saunders;
variable and depends on the presence of 2005. p. 129, 140.
hydrocephalus and associated anomalies.
In severe forms, there is neonatal macro- ?? 77. The triad of Kearns-­Sayre
cephaly from hydrocephalus, brain stem Syndrome includes which visual
dysfunction, and feeding and respiratory abnormality?
problems. Severe developmental delay A. Progressive external
and ataxia may be present. However, in ­ophthalmoplegia
other cases, no symptoms may be present B. Bilateral optic neuropathy
and the malformation may be detected C. Cortical blindness
only incidentally on imaging in adulthood. D. Opsoclonus
E. Jerk nystagmus
?? 76. Globoid cells are characteristic of:
A. Metachromatic leukodystrophy vv Correct Answer is: A
B. Krabbe disease Kearns-Sayre Syndrome (KSS) consists
C. Zellweger syndrome of the triad of: 1) Progressive external
D. Adrenoleukodystrophy ophthalmoplegia (PEO), causing ­ptosis
E. Refsum disease and impaired eye movements. 2) Onset
prior to 20 years of age, and, 3) At
vv Correct Answer is: B least one of the following symptoms:
Krabbe disease, also known as globoid short stature, pigmentary retinopathy,
cell leukodystrophy, is a deficiency of ­cerebellar ataxia, heart block,
galactosylceramide galactosidase. and/or elevated cerebrospinal
This enzyme normally converts galacto- protein levels. Patients may also have
sylceramide to ceramide. Globoid cells are intellectual disability, muscle weak-
multinucleated macrophages that phago- ness, deafness, renal disease, and
cytize the accumulated galactosylceramide ­diabetes. Muscle biopsy reveals ragged
in the CNS white matter. The disorder is red fibers. The disorder is caused by
inherited in an autosomal recessive man- sporadic ­mutations of mitochondrial
ner, with mutations of chromosome 14. It DNA.
results in fibrillary astrocytosis in regions of
demyelination. In the infantile form, there zz Suggested Reading
is an exaggerated startle, low-grade fever, 55 Verma A, Moraes CT. Mitochondrial
irritability, developmental regression, disorders. In: Bradley WG, Daroff RB,
hyporeflexia, seizures, peripheral neuropa- Fenichel GM, Jankovic J, editors.
thy, and spastic weakness resulting in opis- Neurology in clinical practice. 4th ed.
thotonos. The juvenile form presents with Elsevier, Inc.; 2004. p. 1833–46.
cortical blindness, ataxia, developmental 7 http://ghr.nlm.nih.gov/condition/
regression, and spasticity, in the absence of kearns-sayre-syndrome (7 http://ghr.nlm.
peripheral neuropathy. MRI is notable for nih.Gov/condition/kearnssayre-syndrome).
diffuse demyelination. Reviewed 12/11. Published 5/26/14.
Treatment is with bone marrow trans- Accessed 5/29/14.
plant prior to symptom onset.
84 Chapter 3 · Genetic Disorders

?? 78. Which of the following statements is dard deviations below the mean. A
not true regarding the abnormality variety of causes of microcephaly exist.
depicted in the figure? It can be a normal variant that is often
hereditary, without clinical implications.
When pathologic, causes include in
utero infections, toxin exposure (such
3 as alcohol, tobacco, and prescription
drugs such as chemotherapeutic agents
and anti-­epileptics), hypoxic-ischemic
injury, birth trauma, and metabolic dis-
orders such as prolonged hypoglycemia.
A wide variety of hereditary disorders
including enzyme deficiencies can lead
to microcephaly, as can chromosomal
abnormalities. Clinical manifestations
depend on the underlying cause and
can range from none (asymptomatic
microcephaly) to severe developmental
delay and seizures. Macrocephaly is
defined as a head circumference greater
than 2 standard deviations above the
mean. Macrocephaly may be a normal
variant that is often hereditary, may
result from increased CSF, as in hydro-
cephalus due to a variety of causes,
..      Fig. 3.25 With permission from ILAE.org.
EpilepsyDiagnosis.org, Diagnostic Manual from mass lesions such as tumors or
subdural hematomas, or may result
from megalencephaly, an oversized
A. Hemimegalencephaly frequently brain (brain weight greater than 2
presents with seizures standard deviations above the mean).
B. Macrocephaly may be due to Causes of megalencephaly include stor-
hydrocephalus or megalencephaly age diseases such as mucopolysacchari-
C. Hemimegalencephaly rarely leads doses or Tay-Sachs disease, Canavan
to motor manifestations disease, Alexander disease, genetic
D. Megalencephaly is an oversized disorders including Sotos syndrome,
brain, with brain weight greater and others. In the latter disorders, even-
than 2 standard deviations above tual cell loss with subsequent atrophy
the mean typically occurs. Megalencephaly may
E. Megalencephaly may be a benign also be a benign familial finding. Clinical
finding in some cases manifestations depend on the under-
lying cause. Neurologic deficits may
vv Correct Answer is: C be absent in the benign familial form.
This is the radiological manifestation of Hemimegalencephaly, or enlargement
left hemispheric hemimegalencephaly, of only one brain hemisphere, as in this
which frequently leads to contralateral case, invariably presents with seizures
hemiparesis. Microcephaly is defined and hemiparesis. Hemimegalencephaly
as head circumference less than 2 stan- is associated with various genetic dis-
Genetic Disorders
85 3
orders, including Beckwith-­Wiedemann motor weakness, scoliosis/kyphoscolio-
syndrome, which is characterized by sis, dysarthria, high arches or pes cavus,
gigantism, macroglossia, and midline diabetes in 20%, dysphagia, absent
abdominal wall defects. DTRs, muscle wasting, an extensor
plantar response, nystagmus, cardiac
?? 79. You are evaluating a previously involvement (cardiomegaly, cardio-
healthy 8-year-old boy for his abnor- myopathy). Treatment is symptomatic,
mal gait, in the last few months he and aims to support and maintain
has been “walking funny”. His mother ambulation as long as possible. Most
reports that he has been clumsier, patients with FA are wheelchair-bound
tripping frequently as well. The child by age 15.
started to have slurred speech a
week ago. On exam, he has a wide- ?? 80. You are seeing a 9-month-old baby
based slow gait and steppage gait. for his significant developmental
His motor strength in the distal lower delay with psychomotor retardation
extremities is 2/5 and proximal is 3/5. and failure to thrive. On examination,
Lower extremity DTRs are absent. he also has a cherry-­red spot on reti-
There is an extensor plantar response, nal examination and hepatospleno-
and his feet are significant for high megaly. Bone marrow specimen
plantar arches. He has scoliosis on demonstrates foam cells. He carries a
spine exam. diagnosis is of Niemann-Pick disease.
In what way this disorder is What is correct regarding this
acquired? ­diagnosis?
A. X-linked A. This patient has Niemann-­Pick
B. Autosomal dominant Type A
C. Autosomal recessive B. This patient has Niemann-­Pick
D. Post-infectious Type C
E. Random (non-­infectious and C. The activity of the enzyme acid
­non-genetic) sphingomyelinase is increased
D. The cause is a defect in intracel-
vv Correct Answer is: C lular cholesterol trafficking
This is a Friedreich’s Atax ia (FA) case. E. Interstitial lung disease does
It is an autosomal recessive disease not occur in any of the types of
caused by a mutation on chromosome Niemann­Pick disease
9 in the gene encoding the protein
frataxin. Therefore, it occurs in males vv Correct Answer is: A
and females equally. An important thing This patient has Niemann-Pick type
to know is that it is the only disease A. Niemann-Pick types A and B are
caused by a GAA trinucleotide repeat. caused by acid sphingomyelinase
The more GAA repeats, the more pro- deficiency, leading to accumulation of
found the deficit in frataxin protein. sphingomyelin. This disorder is auto-
Patients with Friedreich’s Ataxia usu- somal recessive. Type A involves the
ally present between the ages of 5–15, CNS and other viscera and manifests in
though cases do occur even younger infancy with feeding difficulty, failure
than this. Late-onset FA usually presents to thrive, psychomotor retardation
in the 2nd or 3rd decades. Presenting with regression, hypotonia, and fail-
symptoms and signs include ataxia, ure to thrive. Cherry-red spot is com-
86 Chapter 3 · Genetic Disorders

monly seen, and these patients have of age, a juvenile form with onset in
massive hepatosplenomegaly. Most late childhood and early teens, and the
children die by age 3. Type B is purely adult form with onset in the 20s or 30s.
visceral and does not affect the CNS, The infantile form manifests with clum-
­presenting with ­hepatosplenomegaly siness, frequent falls, slurred speech,
and interstitial lung disease. In both and is associated with weakness and
3 types, bone marrow biopsy will demon- hypotonia. With progression of the
strate vacuolated histiocytes with lipid disease, these children are unable
accumulation and foam cells, in which to stand, and their tone increases.
the sphingomyelin adopts the form of They have loss of vision and hearing,
concentric lamellar bodies. The foamy peripheral neuropathy, and progressive
histiocytes can be seen also in the deterioration of mental function to a
spleen, lymph nodes, hepatic sinusoids, vegetative state and death. The juve-
and pulmonary alveoli. The diagnosis is nile form has a slower progression. The
based on detecting deficient activity of adult form presents with behavioral
acid sphingomyelinase. changes, psychosis, and dementia. MRI
demonstrates T2 hyper-intense signal
?? 81. A 5-year-old boy with a history of changes in periventricular and subcor-
progressive neurologic deterioration. tical white matter, sparing the U fibers.
Symptoms started around 1 year of Cerebellar white matter is also involved.
age with hypotonia and inability to Pathologically, there are confluent sym-
walk, with subsequent visual and metric lesions in the white matter and
hearing loss. He is now unable to walk later atrophy. Central and peripheral
and has generalized spasticity and demyelination is seen, with accumu-
cognitive regression. Brain MRI shows lation of metachromatic material in
T2 hyper intense signal changes in macrophages. Nerve conduction veloci-
the periventricular and subcortical ties are initially normal, but slowing is
white matter sparing the U fibers. seen later in the course. The diagnosis
Arylsulfatase A deficiency is detected is suspected on the basis of typical MRI
on leukocyte analysis. findings and confirmed by demonstrat-
What is the most likely diagnosis? ing deficiency of arylsulfatase A in leu-
A. Metachromatic leukodystrophy kocytes and/or fibroblasts. Treatment
B. Niemann-Pick Type C is supportive. Niemann-Pick Type C is a
C. Niemann-Pick Type A disorder of intracellular cholesterol traf-
D. Krabbe disease ficking. Niemann-Pick Type A is caused
E. Tay-Sachs disease by acid sphingomyelinase deficiency
disease is caused by deficiency of galac-
vv Correct Answer is: A tosylceramidase. Tay-Sachs disease is
Metachromatic leukodystrophy is an caused by hexosaminidase A deficiency.
autosomal recessive disorder caused
by deficiency of the lysosomal enzyme zz Suggested Reading
arylsulfatase A with accumulation of 55 Comprehensive Review in Clinical
sulfatide, resulting in demyelination of Neurology: A Multiple-Choice Question
the central and peripheral nervous sys- Book for the Wards and Boards ©2011
tem. There are three forms: an infantile Wolters Kluwer Health Lippincott
form with onset between 1 and 3 years Williams & Wilkins. All rights reserved.
Genetic Disorders
87 3
?? 82. Which of the following is not true ?? 83. Which of the following gene is impli-
regarding abetalipoproteinemia or cated in congenital neuronal ceroid
Bassen­Kornzweig syndrome? lipofuscinosis?
A. It is autosomal dominant A. Ceroid lipofuscinosis neuronal 2
B. There is demyelination of the pos- (CLN2)
terior columns of the spinal cord B. Ceroid lipofuscinosis, neuronal 3
C. There is demyelination of periph- (CLN3)
eral nerves C. Ceroid lipofuscinosis, neuronal 6
D. There is vitamin E deficiency (CLN6)
E. It is associated with fat D. Ceroid lipofuscinosis, neuronal 8
­malabsorption (CLN8)
E. Ceroid lipofuscinosis, neuronal 10
vv Correct Answer is: A (CLN10)
Abetalipoproteinemia, or Bassen-­
Kornzweig syndrome, is an autosomal vv Correct Answer is: E
recessive disorder caused by a molecular Ceroid lipofuscinosis, neuronal 10
defect in the gene for the microsomal (CLN10) codes for the lysosomal aspar-
triglyceride transfer protein, which is tic protease cathepsin D; CLN10 is also
localized to chromosome 4q22.24. This referred to as Cathepsin D (CTSD). This
protein normally catalyzes the transport is diagnosed in the neonatal period and
of triglyceride, cholesteryl ester, and patients often pass away within the first
phospholipid from phospholipid surfaces. week. Seizures, microcephaly, and a lack
The defect of this protein results in fat of reflexes may be present.
malabsorption and liposoluble vitamin The infantile form of neuronal ceroid
deficiency, especially of vitamin E, which lipofuscinosis is caused by mutations
is the culprit of most of the clinical mani- in CN1; onset is usually within the first
festations. This condition manifests since year of life. This is characterized by visual
birth, with failure to thrive, vomiting, and loss, seizures, cognitive decline, motor
loose stool. During infancy, there is pro- decline, and spasticity. Alterations in the
gressive psychomotor retardation with same gene are implicated in a form of late
cerebellar ataxia and gait disturbance. infantile onset disease; additionally, alter-
Proprioceptive sensation is lost in the ations in this gene may lead to a form of
hands and feet, with less compromise of adult onset disease.
pinprick and temperature sensation. Deep Mutations affecting CLN6 (also
tendon reflexes are depressed. This is referred to as CLN4) are implicated in
likely from demyelination of posterior col- both a late infantile or adult variation
umns and peripheral nerves. Visual distur- of this disease (also called Kufs disease).
bance is the result of retinitis pigmentosa, In addition to CLN6/CLN4, mutations in
and nystagmus is common. Laboratory DnaJ heat shock protein family (Hsp40)
studies demonstrate acanthocytosis, member CS (DANJCS) lead to an adult
absence of very low-density lipoproteins, onset form of the disease. The latter is
absence of apolipoprotein B, low levels of inherited in an autosomal dominant
vitamin E, and severe anemia. Treatment ­manner.
involves the restriction of triglycerides in Mutations in CLN8 lead to late infan-
the diet, and large doses of vitamin E with tile onset disease. This often starts with
supplementation of vitamins A, D, and K. seizures, occurring prior to the age of 10,
88 Chapter 3 · Genetic Disorders

with decline of motor and cognitive func- D. Lactate levels are reduced during
tion and visual loss. Defects in this gene episodic exacerbations
can also cause prominent myoclonic sei- E. Clinical manifestations most
zures with cognitive and motor decline; ­commonly begin during late child-
in the latter phenotype, visual loss may hood
not be present.
3 vv Correct Answer is: B
zz Suggested Reading Leigh disease or acute necrotizing
55 National Library of Medicine (US). encephalomyelopathy is a manifesta-
Genetics Home Reference [Internet]. tion of mitochondrial disorders that can
Bethesda (MD): The Library; 2015 Dec 7. be sporadic or familial, with only some
CTSD; [reviewed 2013 Jul; cited 2015 Dec cases with the typical maternal inheri-
13); Available from: 7 http://ghr.nlm.nih. tance pattern. This condition affects
gov/condition/congenital-neuronal-ceroid- neurons of the brain stem, thalamus,
lipofuscinosis (7 http://ghr.nlm.nih.gov/ basal ganglia, and cerebellum. Most of
condition/congenital-neuronal-ceroid- the affected patients have onset of neu-
lipofuscinosis). rologic manifestations in the first year of
55 National Library of Medicine (US). life, but there are forms with late onset.
Genetics Home Reference [Internet]. Clinical features manifest with decom-
Bethesda (MD): The Library; 2015 Dec 7. pensation associated with intercurrent
PPT1; [reviewed 2013 Aug; cited 2015 Dec illnesses. In infancy, patients present
13]; 7 http://ghr.nlm.nih.gov/gene/PPT1 with hypotonia, loss of head control,
(7 http://ghr.nlm.nih.gov/gene/PPT1). poor sucking, vomiting, irritability, sei-
55 National Library of Medicine (US). zures, and myoclonic jerks. If the onset
Genetics Home Reference [Internet]. is beyond the first year, patients present
Bethesda (MD): The Library; 2015 Dec 7. with gait disturbance, cerebellar ataxia,
TPP1; [reviewed 2013 Aug; cited 2015 Dec dysarthria, psychomotor retardation,
13]; 7 http://ghr.nlm.nih.gov/gene/TPP1 spasticity, external ophthalmoplegia,
(7 http://ghr.nlm.nih.gov/gene/TPP1). and nystagmus, abnormal movements
55 National Library of Medicine (US). with chorea or dystonias, and periph-
Genetics Home Reference [Internet]. eral neuropathy in some cases with
Bethesda (MD): The Library; 2015 Dec 7. autonomic failure. The disorder is pro-
CLN3; [reviewed 2013 Aug; cited 2015 Dec gressive with episodic deterioration.
13]; 7 http://ghr.nlm.nih.gov/gene/CLN3 Lactate level is increased in blood and
(7 http://ghr.nlm.nih.gov/gene/CLN3). CSF. Lactate and pyruvate levels in blood
55 National Library of Medicine (US). are elevated during exacerbations.
Genetics Home Reference [Internet]. MRI of the brain demonstrates bilat-
Bethesda (MD): The Library; 2015 Dec 7. eral symmetric hyperintense T2 signal
CLN6; [reviewed 2013 Sept; cited 2015 Dec abnormalities in the brain stem and/or
13); 7 http://ghr.nlm.nih.gov/gene/CLN6 basal ganglia, and in some cases in the
(7 http://ghr.nlm.nih.Gov/gene/CLN6). spinal cord. Treatment is supportive and
­symptomatic.
?? 84. Which of the following options is true
regarding Leigh disease? zz Suggested Reading
A. It is a static encephalopathy 55 Comprehensive Review in Clinical
B. It is due to mitochondrial abnor- Neurology: A Multiple-Choice Question
malities Book for the Wards and Boards © 2011
C. It is inherited in an autosomal Wolters Kluwer Health Lippincott
recessive fashion Williams & Wilkins. All rights reserved.
Genetic Disorders
89 3
?? 85. You are seeing a 17-month-old boy Mutations in this gene do not typically
with a history of progressive motor lead to the earlier onset, infantile form of
dysfunction with hypotonia, started the disease.
at 5 months. He lost his vision at Mutations in CLN10 lead to dys-
12 months of age and he now has function of Cathepsin D resulting in a
refractory seizures. Neuronal ceroid congenital form of the disease. Patients
lipofuscinosis is suspected. usually die in the neonatal period and
Which protein is most likely present with microcephaly and respira-
­abnormal in this case? tory troubles. Mutations in CLN 10 do not
A. Palmityl protein thioesterase 1 typically causes the infantile form of the
(PPT) disease. Mutations in DNAJCS, coding for
B. Tripeptidyl peptidase 1 (TPP1) cysteine string protein alpha (CSPa) lead
C. Cathepsin D (CTSD) to an autosomal dominant adult onset
D. Cysteine string protein alpha disease. This starts after age 30 and visual
loss is usually not present.
vv Correct Answer is: A Mutations in ceroid lipofuscinosis
The patient most likely has the infantile neuronal 4 (CLN4) were initially thought
form of neuronal ceroid lipofuscinosis to underlie most adult onset disease.
based on the age of onset and manifes- It was then discovered that the gene
tations described above. This is caused affected is the same as that causing a
by a mutation in ceroid lipofuscinosis 1 late infantile onset variant of ceroid lipo-
(CLN1) which codes for the protein pal- fuscinosis, (CLN6). Mutations in DNAJCS
mitoyl-protein thioesterase. The gene do not typically cause infantile onset
is also referred to as palmitoyl-protein disease.
thioesterase 1 (PPT1). Clinical features
are usually present by 6 months of age ?? 86. Mutations of the ALD (ABCD1) gene
and patients typically die within the are most likely to cause:
first decade of life. Patients usually have A. Frontal white matter abnormali-
seizures, microcephaly, hypotonia, and ties
visual loss. Defects in the same gene can B. CNS and renal neoplasms
lead to later onset disease, with symp- C. A deficiency of very long chain
toms appearing between ages 2 and fatty acids
4; seizures, developmental arrest, and D. Cerebral inflammatory
visual loss may be present. CLN1 muta- ­demyelination
tions may lead to another, late onset E. Infantile hypotonia
form of the disease starting during
adulthood (usually after age 19) with vv Correct Answer is: D
decline in cognitive function, visual loss, Mutations of the ALD (ABCD1) gene on
ataxia, and Parkinsonism. the X chromosome causes adrenoleu-
Defects in Tripeptidyl peptidase 1 kodystrophy. The gene encodes a per-
(TTP1) caused by mutations in CLN2 (also oxisomal protein (ALDP) involved very
referred to as Tripeptidyl peptidase I long chain fatty acid (VLCFA) oxidative
(TPP1)) lead to the late infantile neuronal degradation. Mutations lead to an accu-
ceroid lipofuscinosis. Patients between mulation of VLCFAs, and adrenal failure
the ages of 2 and 4 present with general- with a deficiency of androgens and glu-
ized tonic clonic, absence, or myoclonic cocorticoids. This results in perivascular
seizures. Deterioration of cognitive lymphocytes, and demyelination, begin-
function, visual loss, and ataxia are also ning in the parietal and occipital lobes
present and death may occur by age 15. and sparing arcuate fibers. MRI demon-
90 Chapter 3 · Genetic Disorders

strates T2-weighted hyper-intensities Treatment for adrenoleukodystrophy


of the posterior white matter. Several involves glucocorticoids and bone mar-
clinical phenotypes may result, includ- row transplant. Dietary fatty acid supple-
ing childhood onset, adolescent onset, mentation (“Lorenzo’s oil”), has been
adult onset, adrenomyeloneuropathy, used, but with limited data to suggest
Addison’s disease only, and women with efficacy.
3 symptomatic X-ALD.
1. Childhood onset adrenoleukodystro- zz Suggested Reading
phy: The age of onset ranges from 55 Engelen M, et al. X-linked adrenoleukodys-
2.5–10 years. Symptoms include trophy (X-ALD): clinical presentation and
extensive CNS demyelination, hyper- guidelines for diagnosis, follow-up and
activity, dementia, ataxia, vision loss, management. Orphanet J Rare Dis.
hearing deficits, and seizures, with 2012;7:51. Rai PVS, et al. Childhood
endocrine dysfunction. The course is adrenoleukodystrophy – Classic and
rapidly progressive. variant – Review of clinical manifestations
2. Adolescent onset adrenoleukodystro- and magnetic resonance imaging. J Pediatr
phy: Symptoms become evident Neurosci. 2013;8(3):192–7.
between ages 10 and 21 years, and are
rapidly progressive, with CNS demy- ?? 87. You are evaluating an 8-year-old
elination, behavioral and cognitive female with a history of multiple
changes, and endocrine dysfunction. complaints gradually worsening over
3. Adult onset adrenoleukodystrophy: the last few months. She has chronic
This phenotype consists of a rapidly pain and numbness in her hands and
progressive dementia, psychiatric feet that feels like “burning achy pins
changes (e.g. psychosis, affective and needles”. Also, she has chronic
disorders), CNS demyelination, abdominal pains and diarrhea and
endocrine dysfunction, and possible feels her vision “isn’t so good some-
myelopathy or neuropathy. times”. She has kidney dysfunction by
4. Adrenomyeloneuropathy: The age of laboratory investigation.
onset is typically in early adulthood What is your diagnosis?
with a progressive spastic paraparesis, A. Raynaud phenomenon
bowel and bladder incontinence, B. Multiple sclerosis
sexual dysfunction, mild dementia, C. Fabry disease
and psychiatric changes (i.e. affective D. Rheumatic fever
disorders),as well as sensorimotor, E. Lupus
primarily axonal neuropathy. This
phenotype may be either slowly or vv Correct Answer is: C
rapidly progressive. Fabry disease in children can present
5. An Addison’s only phenotype consists with acroparesthesias, GI symptoms/
of isolated adrenal dysfunction. pain, corneal opacity, hypohydrosis,
6. Heterozygous females typically angiokeratomas (which are often mis-
develop symptoms by age 60. The taken for petechiae), and tinnitus or
disorder is slowly progressive, and hearing loss. It is an X-linked recessive
may present with peripheral neuropa- disorder which often manifests variably
thy, fecal incontinence, and pain in the severe symptoms even in heterozy-
legs. White matter lesions and gous females (X-linked recessive with
endocrine dysfunction are rare. incomplete penetrance in heterozygous
Genetic Disorders
91 3
females), and is caused by accumulation Miller-Dieker syndrome (MOS), one form
of a glycoprotein called globotriao- of lissencephaly type I, is character-
sylceramide caused by a deficiency of ized by lissencephaly associated with
alpha galactosidase A (AGA). It occurs microcephaly, typical facies including
in 1/40,000–120,000 live births. Renal micrognathia, low-set ears, thin upper
and cardiac involvement is fairly com- lip, and other features. Clinical manifes-
mon and are common causes of death tations include global developmental
in these patients, and many end up on delay, hypotonia and later spasticity,
dialysis or with renal transplants. and intractable seizures. Life expec-
tancy is often not beyond 1 year. MDS
?? 88. You are seeing a 6-month-old boy for has been associated with microdele-
his intractable seizures. On examina- tions on chromosome 17 in the LIS1
tion, he has a short small chin, thin gene. LIS1 gene encodes a protein
upper lip, and low-set ears. He has involved in regulation of microtubules
spastic quadriparesis and requires a and dynein function, and mutations
feeding tube because of recurrent interfere with microtubule-­directed
aspirations. His brain MRI shows migration of neurons from the ventricu-
essentially smooth frontal, parietal, lar zone. Mutations in LIS1 gene can
and occipital lobes, with a thick cortex also lead to isolated lissencephaly (so-
and without sulci or gyri. called isolated lissencephaly sequence,
What is your most likely diagnosis? without other features of MDS). X-linked
A. Miller-Dieker syndrome lissencephaly is another form that
B. Lissencephaly type II results from mutations in the DCX gene
C. Subcortical band heterotopias on chromosome X when occurring in
D. Cobblestone lissencephaly males. This gene encodes for the pro-
E. Polymicrogyria tein doublecortin, which is involved in
microtubule organization and stabiliza-
vv Correct Answer is: A tion. In another form of Lissencephaly
The clinical and radiological presen- that results from mutations in the gene
tation consistent with Miller-Dieker ARX, which encodes for a transcription
syndrome, a form of lissencephaly type factor involved in non-radial migration
I. In lissencephaly type I (classic lissen- of cortical interneurons, the basal gan-
cephaly), the cortex is thick but consists glia are abnormal in addition to the cor-
of only four layers or less (often two or tex and corpus callosum. In some forms
three layers, as opposed to the normal of lissencephaly, there may be an ante-
six neocortical layers). Lissencephaly rior to posterior (or vice versa) gradient
is a malformation of cortical develop- of gyral formation (e.g. with some gyral
ment resulting from abnormal neuronal formation in the anterior or posterior
migration resulting in impaired forma- aspects of the brain). Cobblestone lis-
tion of gyri. It is characterized by the sencephaly, also known as lissencephaly
presence of reduced cortical gyration type II, is seen in several disorders
and, in the most severe form, no gyri, including Walker-Warburg syndrome,
or agyria, resulting in a smooth brain. Fukuyama muscular dystrophy, and
Associated malformations may include muscle-eye-brain disease of Santavuori.
agenesis of the corpus callosum or In polymicrogyria, there are excess,
hypoplasia of the cerebellum, with spar- abnormal gyri. Subcortical band het-
ing of the thalamus and basal ganglia. erotopia, also known as “double ­cortex”
92 Chapter 3 · Genetic Disorders

syndrome, refers to a band of subcorti- paresthesia in the extremities, acropar-


cal heterotopia neurons, located mid- esthesias) and the CNS.CNS manifesta-
way between the ventricles and the tions include stroke and other vascular
cerebral cortex. The disorder is seen events related to vascular ectasia, with
primarily in females and typically causes often marked dolichoectatic enlarge-
varying degrees of intellectual disability ment of the basilar artery.
3 and almost all of them have epilepsy. Hereditary hemorrhagic telangiec-
tasia, or Osler-Weber-Rendu syndrome,
?? 89. Which of the following statements is an autosomal dominant disorder in
is not true regarding disorders that which telangiectasia occurs in the skin,
involve the skin and CNS? mucous membranes, and several organs
A. Multiple intracranial arterio- including the retina and gastrointestinal
venous malformations are a tract. Recurrent epistaxis is a common
manifestation of hereditary manifestation. CNS involvement results
hemorrhagic telangiectasia (Osler- from single or multiple arteriovenous
Weber-­Rendu syndrome) malformations (AVMs) or cerebral embo-
B. Patients with pseudoxanthoma lization from pulmonary AVMs. It results
elasticum are at increased risk of from a mutation in the HHT1 gene on
cerebral arterial occlusive disease chromosome 9 that encodes for endo-
C. Ehlers-Danlos syndrome is associ- glin, a ­protein that binds transforming
ated with increased risk of intra- growth factor-B (TGF-B},or from a muta-
cranial aneurysms and carotid tion in the HHT2 gene on chromosome
dissection 12. Wyburn-Mason syndrome is another
D. Xeroderma pigmentosa is associ- neurocutaneous disorder in which
ated with peripheral neuropathy, multiple AVMs occur on the face, in the
cognitive decline, ataxia, and retina, and intracranially. In pseudoxan-
hyperkinetic involuntary move- thoma elasticum (Gronblad­Strandberg
ments syndrome), a connective tissue disorder
E. Fabry disease is a multiorgan that may be autosomal dominant or
disorder associated with neuropa- recessive, yellowish xanthomas occur in
thy and cerebral artery ectasia various skin regions, on mucous mem-
and results from a defect in the branes, and in the retina. Neurological
enzyme sphingomyelinase manifestations relate to vascular occlu-
sions and intracranial carotid artery
vv Correct Answer is: E aneurysms.
Fabry disease is an X-linked disorder Ehlers-Danlos syndrome exists in 10
that results from a mutation in the subtypes, with types I, II, and Ill being the
gene encoding the lysosomal enzyme most common. These subtypes share the
A-galactosidase. Multiple organs are occurrence of hyperelastic skin, hyper
involved; renal involvement leads to extensible joints, and vascular lesions.
renal failure and cardiac involvement to This syndrome results from mutations
cardiomyopathy. Cutaneous manifesta- in various genes encoding for differ-
tions in Fabry disease include purplish ent types of collagen. The main neuro-
angiokeratomas often occurring in logic significance of this disorder is the
the groin. Neurologic manifestations increased risk of intracranial aneurysms,
involve both the peripheral nervous carotid-cavernous fistulas (that may be
system (including small fiber neuropa- spontaneous or due to mild trauma), and
thy with attacks of cold-­induced painful arterial dissection.
Genetic Disorders
93 3
Xeroderma pigmentosum (XP) is a zz Suggested Reading
neurocutaneous disorder marked by 55 Comprehensive Review in Clinical
sensitivity to ultraviolet light that pre- Neurology: A Multiple-Choice Question
disposes affected individuals to skin Book for the Wards and Boards © 2011
freckling and multiple cutaneous malig- Wolters Kluwer Health Lippincott
nancies including melanoma, basal cell Williams & Wilkins. All rights reserved.
carcinoma, and squamous cell carcinoma
as well as other cutaneous and systemic ?? 90. You are evaluating an 11-month-
tumors. Neurologic abnormalities include old baby for regression started at
progressive cognitive dysfunction, hear- 6 months of age. Initially, she had
ing loss, tremor, chorea, and ataxia as gained some developmental mile-
well as peripheral neuropathy. stones. On examination, she has
XP is autosomal recessive and results coarse facial features, generalized
from mutations on chromosome 9 that spasticity, and motor impairment.
result in abnormal DNA repair. Cockayne A retinal picture is shown in the
syndrome is a related disorder. Another Figure. Enzymatic analysis showed
disorder of DNA repair associated with a B-­galactosidase deficiency in
neurologic features is ataxia telangiectasia. ­leukocytes.

a b

..      Fig. 3.26 a and b showing the accumulation of the gangliosides in the retina. JAMA Ophthalmology.
Wikipedia

Which statement is correct? ?? 91. You are evaluating an 18-year-old


A. This patient has GM2 male in hospital with a 12-h history
­gangliosidosis of right-sided headache, nausea and
B. Gangliosides accumulate in the light sensitivity, and had a general-
brain and visceral organs ized tonic-clonic seizure shortly
C. Sandhoff disease is caused by this before admission. His past medical
enzymatic deficiency history is remarkable for migraine.
D. Tay-Sachs disease is caused by this His family history is remarkable of a
enzymatic deficiency sister with history of diabetes mel-
E. Mucopolysacchariduria is common litus, his mother and a maternal
aunt had had strokes in their 40s.
vv Correct Answer is: B On examination, he was confused
94 Chapter 3 · Genetic Disorders

with a GCS of 11. He had a left-sided involved than distal muscles. Additional
homonymous hemianopia. Blood features on neurologic examination
pressure was 175/95 mmHg but there may include ataxia, tremor, myoclonus,
were no other abnormalities. Work-up and dystonia, visual disturbances, and
revealed: CBC/Electrolytes/ESR – Nor- cortical blindness. Some patients may
mal, Lactic acid – 14.2 mmol/L, Blood present with ophthalmoplegia and pto-
3 Glucose – Normal, MRI brain w/o con- sis. On ophthalmologic examination,
trast – Diffusion restriction on DWI in patients have presented with pigmentary
the right occipital lobe which appears retinopathy. Sensorineural deafness has
hypo-intense on ADC. been reported as part of the disorder
What is your most likely diagnosis? in approximately 25% of patients with
A. Familial hemiplegic migraine MELAS syndrome.
B. Venous sinus thrombosis Cardiomyopathy with signs of con-
C. cerebral autosomal dominant gestive heart failure (CHF) may also be
arteriopathy with subcortical observed upon physical examination.
infarcts and leukoencephalopathy Short stature may be the first mani-
D. Mitochondrial encephalomyopa- festation of MELAS syndrome in many
thy with lactic acidosis and stroke- patients. Familial hemiplegic migraine
like episodes may present in a similar manner but is
E. Cerebral vasculitis not associated with seizures. Although
subtle changes on MR imaging have
vv Correct Answer is: D been reported those shown here are far
This is a clinical presentation of mito- too extensive. Cerebral venous throm-
chondrial encephalomyopathy with bosis may be clinically indistinguishable
lactic acidosis and stroke like episodes. from this case, though the family history
Stroke like episodes are the hallmark is not explained. MR findings are usually
feature of this disorder. Stroke like that of a hemorrhagic infarct not pertain-
episodes may present with convul- ing to a single arterial territory and CSF
sions, visual abnormalities, numbness, opening pressures were not elevated in
hemiplegia, and aphasia. Episodes may this case. A diagnosis of CADASIL would
be followed by transient hemiplegia or be consistent with recurrent migraine
hemianopia, which lasts a few hours to and a dominant family history. Imaging
several weeks. Patients may also experi- in CADASIL shows deep white matter
ence episodes of seizures and visual abnormalities, in contrast to the findings
abnormalities followed by hemiplegia. in this patient.
Migraine or migraine-like headaches
observed in these patients may also zz Suggested Reading
reflect the stroke like episodes. Pedigrees 55 Testai FD, Gorelick PB. Inherited metabolic
of patients with classic MELAS syndrome disorders and stroke part 1: Fabry disease
identify many members whose only and mitochondrial myopathy, encepha-
manifestations are migraine headaches. lopathy, lactic acidosis, and stroke like
Patients may have visual complaints episodes. Arch Neurol. 2010;67(1):19–24.
due to ophthalmoplegia, and they may 55 Scarpelli M, Zappini F, Filosto M,
experience blindness because of optic Russignan A, Tonin P, Tomelleri
atrophy and difficulties with night vision G. Mitochondrial sensorineural hearing
due to pigmentary retinopathy. Myopa- loss: a retrospective study and a description
thy presents with hypotonia and weak- of cochlear implantation in a MELAS
ness. Proximal muscles tend to be more patient. Genet Res Int. 2012;2012:287432.
Genetic Disorders
95 3
55 Singmaneesakulchai S, Limotai N, Jagota P, vv Correct Answer is: B
Bhidayasiri R. Expanding spectrum of This is a clinical and radiological mani-
abnormal movements in MELAS syndrome festations of septo-optic dysplasia.
(mitochondrial encephalomyopathy, lactic The malformations include hypoplasia
acidosis, and stroke-like episodes). Mov or absence of the septum pellucidum,
Disord. Oct 2012;27(12):1495–7. optic nerve and optic chiasm hypo-
55 Fayssoil A. Heart diseases in mitochondrial plasia, dysgenesis of the corpus cal-
encephalomyopathy, lactic acidosis, and losum and anterior commissure, and
stroke syndrome. Congest Heart Fail. fornix detachment from the corpus
2009;15(6):284–7. callosum. Arrhinencephaly (agenesis
of only the olfactory bulb and tract)
and/or ­hypothalamic hamartomas
?? 92. Physical complications of spina bifida may be associated features. Other less
include which of the following? commonly associated abnormalities
A. Allergy to latex include cerebellar vermis defects and
B. Leg weakness and paralysis hydrocephalus. Septo-optic dysplasia
C. Club foot can also be associated with lobar holo-
D. (B) and (C) prosencephaly and other malforma-
E. (A) and (C) tions of cortical development. Clinical
manifestations include vision loss,
vv Correct Answer is: E ataxia when the cerebellum is involved,
68% of children with spina bifida have and symptoms of hydrocephalus when
an allergy to latex, ranging from mild it is present, and endocrinology distur-
to life-­threatening. The common use bances. Endocrinology disturbances
of latex in medical facilities makes this can range from pan-hypopituitarism
a particularly serious concern. Leg to isolated hormone deficiencies.
paralysis or weakness and orthopedic Mutations in the transcription factors
abnormalities such as club foot, hip HESX1, a homeobox gene, and SOX
dislocation or scoliosis, and urinary and may be implicated in this disorder.
bowel control problems are frequently Cavum septum pellucidum, in which
present. the septum pellucidum is not fused but
rather exists in two separate pieces of
?? 93. You are seeing an 8-year-old boy tissue, is considered non-pathologic,
with a short stature due to growth with little clinical implications. The
hormone deficiency and is also being presence of normal cortex excludes
treated for hypothyroidism. He has a holoprosencephaly, and lissencephaly.
normal motor and cognitive develop- This patient has several features
ment. He has reduced visual acuity. ­making septo-­optic dysplasia the
He has an absence of the septum pel- likely diagnosis rather than
lucidum and hypoplasia of the optic ­arrhinencephaly.
nerves and chiasm on his MRI. The
cortex and other areas appear normal. zz Suggested Reading
What is your most likely diagnosis? 55 Comprehensive Review in Clinical
A. Holoprosencephaly Neurology: A Multiple-Choice Question
B. Septo-optic dysplasia Book for the Wards and Boards © 2011
C. Lissencephaly Wolters Kluwer Health Lippincott
D. Arrhinencephaly Williams & Wilkins. All rights
E. Cavum septum pellucidum reserved.
96 Chapter 3 · Genetic Disorders

?? 94. You are evaluating a 5-year-old boy initially the posterior regions and spar-
with a history of behavioral problems ing the U fibers. There is involvement
with aggressiveness for 1 year. He has of other tissues, including the adrenal
also had progressive cognitive dete- cortex and Leydig cells of the testis.
rioration and spasticity and visual Histopathological, cerebral lesions are
loss. Plasma levels of very long-chain characterized by the presence of peri-
3 fatty acids are elevated, and MRI vascular cuffing, with predominance of
shows white matter T2 hyper-intensi- T cells.
ties, which are symmetric, involving Adrenomyeloneuropathy is the
posterior regions, and sparing the U most common phenotype and mani-
fibers. fests with paraparesis that begins after
What is the most likely diagnosis? age 20, and is slowly progressive into
A. Canavan disease adulthood. There is also a sensory neu-
B. Adrenoleukodystrophy ropathy in these patients. Some degree
C. Alexander disease of cognitive impairment and adrenal
D. Zellweger syndrome insufficiency may be seen. Pathologi-
E. Fabry disease cally, there is long tract degeneration
with axon and myelin loss. Peripheral
vv Correct Answer is: B nerve demyelination without inflamma-
Adrenoleukodystrophy is an X-linked tion also occurs. Patients with adreno-
disorder caused by a deficiency of the leukodystrophy have increased plasma
peroxisomal enzyme acyl coenzyme levels of very long-­chain fatty acids, and
A synthetase, leading to the impaired ACTH is increased secondary to adrenal
ability to oxidize very long-chain fatty ­insufficiency.
acids, with subsequent accumulation Treatment involves supportive care
in tissues and plasma. There are four with steroid replacement therapy for
phenotypes: childhood cerebral type, adrenal insufficiency. “Lorenzo’s oil”,
adrenomyeloneuropathy, pure adrenal which consists of 4:1 glyceryl trioleate­
insufficiency, or asymptomatic. glyceryl trierucate, has been shown to
Female carriers may have mild symp- reduce levels of very long-­chain fatty
toms of adrenomyeloneuropathy. acids in plasma. Dietary use of Lorenzo’s
The cerebral type has an onset oil may be beneficial in young asymp-
between 4 and 8 years of age, with tomatic patients but not in patients
behavioral changes progressing to with neurologic deficits. Bone marrow
cognitive impairment, spasticity, dis- transplantation may have a role in early
turbances of gait and coordination, and stages of the disease.
vision and hearing loss. The prognosis is
poor, and these patients die early. MRI ?? 95. You are evaluating a 3-year-old boy
demonstrates confluent T2 hyper-inten- with a history of developmental delay
sity in the white matter affecting more and ataxia. On examination, he has
predominantly the parieto-occipital extraocular muscle abnormalities,
regions and the posterior corpus callo- but visual acuity is unaffected and his
sum. Pathologically, there is symmetric irises appear normal. His brain MRI is
and confluent demyelination affecting shown in the figure.
Genetic Disorders
97 3
ataxia, oculomotor abnormalities, and
respiratory difficulties. The molar tooth
sign is also seen in COACH syndrome
(cerebellar vermis hypoplasia, oligo-
phrenia, congenital ataxia, coloboma,
and hepatic fibrosis), features of which
are not present in this case. It is also
seen in Leber congenital amaurosis, in
which vision loss occurs due to rod and
cone dystrophy. The molar tooth sign
has not been associated with ataxia
with oculomotor apraxia types I and II.

?? 96. Which of the following statements


is not true regarding polymicrogyria
and schizencephaly?
A. Polymicrogyria is characterized by
excessive abnormal gyri that are
small and separated by shallow
sulci
..      Fig. 3.27 Axial T2-weighted MRI. (Courtesy of B. Schizencephaly is a deep cleft
Dr. Manikum Moodley and Dr. Gary Hsieh) that extends from the pial surface
to the ventricle and is lined with
What is the most likely diagnosis? ­cortex
A. Ataxia with oculomotor apraxia C. Polymicrogyria most often occurs
type I in the perisylvian region
B. Ataxia with oculomotor apraxia D. Porencephaly describes a type of
type II schizencephaly
C. Joubert syndrome E. They are disorders of cortical
D. Leber congenital amaurosis organization
E. COACH syndrome
vv Correct Answer is: D
vv Correct Answer is: C Porencephaly is distinguished from
There is the molar tooth sign shown on schizencephaly in that the cleft of
the MRI, which results from cerebellar schizencephaly is lined uniformly with
vermis hypoplasia with fourth ven- gray matter. Porencephalic cysts are
tricular enlargement, a large interpe- not due to malformations of cortical
duncular fossa, and abnormal superior development but are rather CSF-filled
cerebellar peduncles. It is seen in a cysts that most often result from in
variety of conditions associated with utero infarction or other insult. In mal-
cerebellar hypoplasia, including Joubert formations of cortical organization,
syndrome. Joubert syndrome is an neurons migrate relatively normally
autosomal recessive disorder character- but formation of the cortical layers
ized clinically by developmental delay, or cortical-cortical connections are
98 Chapter 3 · Genetic Disorders

abnormal. These thus occur later in loss. He is now unable to walk and has
gestation. Malformations of cortical generalized spasticity and cognitive
organization include polymicrogyria, regression. There are T2 hyper-intense
in which there are excessive abnormal signal changes in the periventricular
gyri that are small and separated by and subcortical white matter sparing
shallow sulci, and schizencephaly, a the U fibers on brain MRI. Arylsulfa-
3 deep cleft that extends from the pial tase A deficiency is detected on leu-
surface to the ventricle and is lined kocyte analysis.
with cortex. These often co-occur in What is the most likely diagnosis?
the same patient. Polymicrogyria can A. Metachromatic leukodystrophy
be unilateral or bilateral, generalized, B. Niemann-Pick Type C
perisylvian, predominantly frontal, C. Niemann-Pick Type A
or in a variety of other patterns. The D. Krabbe disease
perisylvian form is the most common. E. Tay-Sachs disease
Polymicrogyria often occurs as part of
various syndromes, either sporadically vv Correct Answer is: A
or in familial forms. Perisylvian poly- Metachromatic leukodystrophy is an
microgyria occurs particularly in the autosomal recessive disorder caused
setting of peroxisomal disorders such by deficiency of the lysosomal enzyme
as Zellweger syndrome. Several genetic arylsulfatase A with accumulation of
mutations have been identified in asso- sulfatide, resulting in demyelination
ciation with polymicrogyria. Clinical of the central and peripheral nervous
manifestations depend on the location ­system. There are three forms: an
and the extent of the abnormality; infantile form with onset between 1
epilepsy is common. Schizencephaly, and 3 years of age, a juvenile form with
or cleft brain, most often occurs in the onset in late childhood and early teens,
perisylvian region but can occur any- and the adult form with onset in the
where. In closed-lip schizencephaly, 20s or 30s.
the cerebral cortical walls on either The infantile form manifests with
side of the cleft are in c­ ontact. In open- clumsiness, frequent falls, slurred speech,
lipped schizencephaly, the two walls and is associated with weakness and
are separated by CSF. Schizencephaly hypotonia. With progression of the dis-
is most often an isolated finding or is ease, these children are unable to stand,
associated with polymicrogyria but can and their tone increases. They have loss
rarely be seen in patients with septo­ of vision and hearing, peripheral neu-
optic dysplasia. Mutations in homeobox ropathy, and progressive deterioration of
genes, which encode for transcrip- mental function to a vegetative state and
tion factors expressed during different death.
times of embryologic development The juvenile form has a slower progression.
and modulate neuronal proliferation The adult form presents with behav-
and migration, may be implicated in ioral changes, psychosis, and dementia.
schizencephaly. MRI demonstrates T2 hyper-intense
signal changes in periventricular and sub-
?? 97. You are evaluating a 5-year-old boy cortical white matter, sparing the U fibers.
for his progressive neurologic dete- Cerebellar white matter is also involved.
rioration. It stared at 1 year of age Pathologically, there are confluent
with hypotonia and inability to walk, symmetric lesions in the white matter
with subsequent visual and hearing and later atrophy. Central and peripheral
Genetic Disorders
99 3
demyelination is seen, with accumu- What is your ­diagnosis?
lation of metachromatic material in A. Miller-Dieker syndrome
­macrophages. B. Lissencephaly type 11
Nerve conduction velocities are ini- C. Subcortical band heterotopia
tially normal, but slowing is seen later in D. Cobblestone lissencephaly
the course. The diagnosis is suspected E. Polymicrogyria
on the basis of typical MRI findings and
confirmed by demonstrating deficiency vv Correct Answer is: C
of arylsulfatase A in leukocytes and/or The MRI shows subcortical band hetero-
fibroblasts. Treatment is supportive. topia. The neuronal migration disorder
Niemann-Pick Type C is a disorder associated with muscular dystrophy
of intracellular cholesterol trafficking. is cobblestone lissencephaly. The lis-
Niemann-­Pick Type A is caused by acid sencephaly syndromes result from
sphingomyelinase deficiency disease is neuronal migration abnormalities and
caused by deficiency of galactosylcerami- include subcortical band heterotopia,
dase. Tay-Sachs disease is caused by hex- or double cortex, in which there is rela-
osaminidase A deficiency. tively normal cortex with an underlying
band of white matter, underneath which
?? 98. You are seeing a 5-year-old girl with is a band of gray matter. This disorder
a history of intractable seizures. On results from a mutation in the DCX
examination, she has spastic quadri- gene on chromosome X, which encodes
paresis and requires a feeding tube for the protein doublecortin which is
because of recurrent aspirations. Her involved in microtubule organization
brain MRI is shown. and stabilization. The same mutation
can lead to classic lissencephaly when
occurring in males. This difference in
manifestations in females as compared
with males is thought to result from
lyonization (random X inactivation) in
females, such that in neurons in which
the mutated gene is inactivated, nor-
mal migration occurs. Clinical features
include intractable seizures, microceph-
aly, hypotonia, spastic quadriparesis,
recurrent aspirations necessitating feed-
ing tube, and shortened life expectance.
Lissencephaly type I characterized by
agyria or pachygyria rather than the
presence of two bands of gray matter
separated by a band of white matter.
Cobblestone lissencephaly (rather than
subcortical band heterotopia), or lis-
sencephaly type II, is seen in several
disorders, including Walker-Warburg
syndrome, Fukuyama muscular dystro-
phy, and muscle-eye-brain disease of
..      Fig. 3.28 With permission from ILAE.org. Santavuori. In polymicrogyria, there are
EpilepsyDiagnosis.org, Diagnostic Manual excess, abnormal gyri.
100 Chapter 3 · Genetic Disorders

?? 99. You are evaluating a 15-year-old boy Which of the following statements is
for growth retardation, generalized correct regarding microcephaly?
weakness, and ataxia. On examina- A. It always implies an underlying
tion, he has bilateral ptosis and neurologic disorder and develop-
restricted gaze in all directions. His mental delay is invariably present
EKG shows complete heart block. B. Causes include infection, trauma,
3 What is the most likely diagnosis? and hypoxic-ischemic insult
A. Kearns-Sayre syndrome C. Microcephaly is defined as a head
B. Mitochondrial encephalopathy, circumference less than 1 stan-
lactic acidosis, and strokes dard deviation below the mean
C. Myoclonic epilepsy with ragged D. Seizures are infrequently seen in
red fibers children with microcephaly
D. Leigh disease E. Maternal exposure to AEDs has
E. Myasthenia gravis not been associated with micro-
cephaly
vv Correct Answer is: A
This is a clinical presentation of Kearns-­ vv Correct Answer is: B
Sayre syndrome, which is a disorder Microcephaly is defined as head circum-
caused by multiple mtDNA deletions. ference less than 2 standard deviations
The diagnosis is made with the triad below the mean. A variety of causes of
of progressive external ophthalmople- microcephaly exist. It can be a normal
gia, onset before the age of 20 years, variant that is often hereditary, without
and at least one of the following: short clinical implications.
stature, retinitis pigmentosa, cerebellar When pathologic, causes include in
ataxia, heart block, and increased CSF utero infections, toxin exposure (such as
protein (>100 mg/dl). Chronic progres- alcohol, tobacco, and prescription drugs
sive external ophthalmoplegia may such as chemotherapeutic agents and
be an isolated finding seen in some anti-epileptics), hypoxic-ischemic injury,
patients. Patients with Kearns-Sayre birth trauma, and metabolic disorders
syndrome have a gradual progression of such as prolonged hypoglycemia. A wide
symptoms and most will have cognitive variety of hereditary disorders including
regression by third or fourth decade of enzyme deficiencies can lead to micro-
life. Most cases are sporadic. An elec- cephaly, as can chromosomal abnormali-
trocardiogram is required to diagnose ties. Clinical manifestations depend on
heart block, in which case, a pacemaker the underlying cause and can range from
is needed. Pathologically, patients may none (asymptomatic microcephaly) to
have muscles with ragged red fibers and severe developmental delay and sei-
white matter showing spongy myeli- zures. Macrocephaly is defined as a head
nopathy without gliosis or macrophage circumference greater than 2 standard
reactions. deviations above the mean.
Macrocephaly may be a normal vari-
?? 100. You are seeing a 2-year-old child ant that is often hereditary, may result
for his small head size noted by the from increased CSF, as in hydrocephalus
child’s pediatrician. The child’s head due to a variety of causes, from mass
size had been small at birth and the lesions such as tumors or subdural
rate of growth had been below aver- hematomas, or may result from megalen-
age over time. cephaly, an oversized brain (brain weight
Genetic Disorders
101 3
greater than 2 standard deviations above vv Correct Answer is: B
the mean). Causes of megalencephaly Lissencephaly is a malformation of
include storage diseases such as muco- cortical development resulting from
polysaccharidoses or Tay-Sachs disease, abnormal neuronal migration resulting
Canavan disease, Alexander disease, in impaired formation of gyri. It is char-
genetic disorders including Sotos syn- acterized by the presence of reduced
drome, and others. In the latter disorders, cortical gyration and, in the most severe
eventual cell loss with subsequent atro- form, no gyri, or agyria, resulting in a
phy typically occurs. Megalocephaly may smooth brain. In classic lissencephaly,
also be a benign familial finding. or lissencephaly type I, the cortex is
Clinical manifestations depend on thick but consists of only four layers
the underlying cause. Neurologic deficits (or less often two or three layers, as
may be absent in the benign familial opposed to the normal six neocorti-
form. Hemimegalencephaly, or enlarge- cal layers). Associated malformations
ment of only one brain hemisphere, may include agenesis of the corpus
invariably presents with seizures and callosum or hypoplasia of the cerebel-
hemiparesis. Hemimegalencephaly is lum, with sparing of the thalamus and
associated with various genetic disor- basal ganglia. Miller-Dieker syndrome
ders, including Beckwith-Wiedemann (MDS), one form of lissencephaly type I,
syndrome, which is characterized by is characterized by lissencephaly associ-
gigantism, macrogloss ia, and midline ated with microcephaly, typical facies
abdominal wall defects. including micrognathia (small jaw),
low-set ears, thin upper lip, and other
?? 101. You were asked to see a 6-month- features. Clinical manifestations include
old boy for intractable seizures. On global developmental delay, hypotonia
examination, he has a short small and later spasticity, and intractable
chin, thin upper lip, and low-set ears. seizures. Life expectancy is often not
He has spastic quadriparesis and beyond 1 year. MDS has been associated
requires a feeding tube because of with microdeletions on chromosome
recurrent aspirations. His brain MRI 17 in the LIS1 gene. LIS 1 gene encodes
shows essentially smooth frontal, a protein involved in regulation of
­parietal, and occipital lobes, with a microtubules and dynein function, and
thick cortex and without sulci or gyri. mutations interfere with microtubule-
Which of the following statements is directed migration of neurons from the
not true regarding this disorder? ventricular zone. Mutations in LIS1 gene
A. It results from abnormal neuro- can also lead to isolated lissencephaly
nal migration (so-called isolated lissencephaly
B. The cortex in this disorder con- sequence, without other features of
sists of six layers MDS).X-linked lissencephaly is another
C. Associated malformations form that results from mutations in
including corpus callosum agen- the DCX gene on chromosome X when
esis may occur occurring in males. This gene encodes
D. LIS1 mutation disrupts micro- for the protein doublecortin, which is
tubule-directed neuronal involved in microtubule organization
­migration and stabilization. In another form of
E. Microcephaly and intractable lissencephaly that results from muta-
seizures occur tions in the gene ARX, which encodes
102 Chapter 3 · Genetic Disorders

for a transcription factor involved in optic chiasm) and pituitary infundibu-


non-radial migration of cortical inter- lum. It can be associated with either
neurons, the basal ganglia are abnormal 1) Schizencephaly or 2) Constellation
in addition to the cortex and corpus cal- of diffuse white matter hypoplasia,
losum. In some forms of lissencephaly, ventriculomegaly, and abnormal
there may be an anterior to posterior hypothalamic-­pituitary function. MRI
3 (or vice versa) gradient of gyral forma- of the brain will show - absent septum
tion (e.g. with some gyral formation in pellucidum; hypoplastic pituitary stalk;
the anterior or posterior aspects of the hypoplastic optic chiasm/optic nerves
brain). Cobblestone lissencephaly, also and globes. 60–90% of these patients
known as lissencephaly type II, is seen present with blindness or symptoms
in several disorders including Walker- of visual impairment. Signs related
Warburg syndrome, Fukuyama muscular to visual loss include absent fixation,
dystrophy,and muscle-eye-brain disease searching nystagmus, visual inattentive-
of Santavuori. In polymicrogyria, there ness and strabismus. 75% of patients
are excess, abnormal gyri. Subcortical may have intellectual disability, epi-
band heterotopia. lepsy or cerebral palsy or a
­ ttention
deficit disorder or autism. Other
?? 102. You are seeing a 6-year-old boy developmental abnormalities include
for vision impairment because he underdeveloped mid-facial structures,
bumps into things at home. His hypoglycemia, and hyperbilirubinemia,
past medical history is remarkable disturbance in temperature regulation,
of horizontal nystagmus at around muscular hypotonia, microgenitalism,
6 months of age and seizure disor- low Apgar scores and failure to thrive.
der. He has not met developmental
milestones with below average zz Suggested Reading
height. He has a low glucose levels. 55 Parker K, et al. J Pediatr Endocrin Metab
Ophthalmologic exam revealed 2002; 15:697–700. Lehnstein Campbell,
small optic nerves and visual acuity C. Optometry 2003; 74:417–26.
of 20/200 both eyes. 55 Phillips PH, Brodsky MC. Congenital optic
What do you ­except to see on his ne abnormalities, In: Pediatric ophthalmol-
brain MRI? ogy and strabismus, 2nd ed. Wright KW,
A. Occipital lobe stroke Spiegel, PH, editors. New York: Springer,
B. Optic nerve tumor bilaterally 2003. p. 918–22.
C. Absent septum pellucidum 55 Egan RA, Kerrison JB. Ophthalmol Clin N
D. Frontal lobe meningioma com- Am 2003;16:595–605.
pressing optic nerves
E. Lissencephaly ?? 103. You are evaluating a 6-month-old
baby for seizures. He has multiple
vv Correct Answer is: C dysmorphic features and severe
This is a case of septo-optic dysplasia. hypotonia, and brain MRI shows
Septo-optic dysplasia is described as pachygyria. There is calcific stippling
absence or dysgenesis of the septum of the patella on skeletal radio-
pellucidum with hypoplasia of the graphs. Very long-chain fatty acid
optic nerve (sometimes involving the levels are elevated in plasma.
Genetic Disorders
103 3
Which of the following is not true kidney disease. A very typical feature is
regarding this condition? chondrodysplasia punctata with bony
A. This is a peroxisomal disorder stippling of the patella. Other congenital
B. Liver cirrhosis occurs in these abnormalities have been reported, such
patients as ventricular septal defects and other
C. Kidney cysts can be seen in these cardiac abnormalities.
patients Diagnostic workup demonstrates
D. White matter is not involved increased plasma very long-chain fatty
E. Polymicrogyria may be seen in acids, decreased red blood cell plasmalo-
these patients gens, and decreased or absent hepatic
peroxisomes. Pathologically, the brain
vv Correct Answer is: D may be enlarged, and there is evidence
This is a clinical presentation of of neuronal migration abnormality, white
Zellweger syndrome, which is a per- matter changes with lipid accumulation,
oxisomal disorder in which the white and sometimes pachygyria and/or poly-
matter is involved. Peroxisomes are microgyria. The treatment of Zellweger
organelles involved in the biosynthesis syndrome is symptomatic. Most of these
of ether phospholipids and bile acids, patients die early. The other peroxisomal
oxidation of very long­chain fatty acid, abnormalities are much milder. Neonatal
prostaglandins, and unsaturated long- adrenoleukodystrophy patients have liver
chain fatty acids, and the catabolism of cirrhosis, adrenocortical atrophy, and
phytanate, pipecolate, and glycolate. brain abnormalities such as polymicrogy-
The infantile syndromes of peroxisomal ria, subcortical heterotopia, and cerebel-
dysfunction include Zellweger syn- lar dysplasia but no pachygyria. Infantile
drome, neonatal adrenoleukodystrophy, Refsum disease manifests with psycho-
and infantile Refsum disease. Zellweger motor retardation, sensorineural hear-
is the most severe form and is caused by ing loss, retinal degeneration, anosmia,
mutations in the PEX genes, the majority and mild dysmorphic features and there
with PEX1 resulting in abnormal peroxi- may be cirrhosis and adrenal atrophy. In
somal biogenesis. Zellweger syndrome, Refsum disease, phytanic acid levels are
or “cerebrohepatorenal syndrome”, is elevated and peroxisomes are reduced or
characterized by dysmorphic features absent. These patients may survive into
such as a high forehead, large fonta- adulthood.
nelles, flat supraorbital ridges, hyper-
telorism, epicanthal folds, broad nasal ?? 104. You are seeing a 14-year-ofd boy
bridge, micrognathia, and flat occiput. with a history of migraines has short
These children have cataracts, retinal stature, ataxia, proximal weakness,
dystrophy, sensorineural hearing loss, mild cognitive impairment, deaf-
severe hypotonia, decreased sucking ness, and various types of seizures.
and crying, hyporeflexia, deformities in He is admitted after a viral illness,
flexion of the lower limbs with arthro- becomes dehydrated, and is found
gryposis, profound intellectual disabil- to have lactic acidosis. He has been
ity, and seizures. The brain demonstrates diagnosed with a progressive myo-
white matter changes and abnormalities clonic epilepsy (PME); a muscle
in neuronal migration. There is liver biopsy has been obtained and
dysfunction with cirrhosis and polycystic shown in the figure.
104 Chapter 3 · Genetic Disorders

Point mutations in mitochondrial DNA


have been detected in this disorder.
Mutation affecting cystatin B is seen
with EPM1 mutations in Unverricht Lun-
dborg syndrome. Cherry red spot is seen
in various conditions, including sialidosis,
3 which is another progressive myoclonic
epilepsy. EPM2A mutation is seen in
Lafora body disease, in which Lafora bod-
ies are also detected on skin biopsy.

zz Suggested Reading
55 Delgado-Escueta AV, Ganesh S, Yamakawa K.
..      Fig. 3.29 Modified_Gomori_trichrome_stain_ Advances in the genetics of progressive
showing_several_ragged_red_fibers_.jpg. Abu-­Amero myoclonus epilepsy. Am J Med Genet.
KK, Al-Dhalaan H, Bohlega S, Hellani A, Taylor RW. 2001;106:129–38.
derivative work: CopperKettle (talk) 55 Schmiedel J, Jackson S, Schafer J, et al.
Mitochondrial cytopathies. J Neurol.
Which of the following is a feature of 2003;250:267–77.
this patient’s condition?
A. Mutation affecting cystatin B ?? 105. You were asked to see an 8-month-
B. Cherry red spot on the fundo- old boy who recently stopped
scopic examination visual tracking and nystagmus was
C. This is a mitochondrial disorder noted. On examination, the pupil-
D. EPM2 A mutation lary response is sluggish and the
E. Lafora bodies on skin biopsy reminder of his neurological exam is
within normal limits.
vv Correct Answer is: C What is your diagnosis?
This is a case of myoclonic epilepsy A. Retinoblastoma
with ragged red fibers (MERFF), which B. Craniopharyngioma
is a mitochondrial disorder. The patient C. Kearns-Sayre syndrome
has various characteristics suggestive D. Leber Congenital amaurosis
of a mitochondrial disorder, including
migraines, short stature, ataxia, cogni- vv Correct Answer is: D
tive impairment, deafness, epilepsy, This is a case of Leber congenital amau-
and elevated lactate. He has general- rosis, it is a retinal dystrophy and is the
ized proximal weakness suggesting a most common genetic cause of congeni-
myopathy. Mitochondrial disorders are tal visual impairment. Severe impair-
a heterogeneous group of disorders ment begins in the first 3–4 months of
that can affect both the peripheral life followed by nystagmus and sluggish
and central nervous system. MERRF is pupillary responses. Visual acuity will
a mitochondrial disorder that usually rarely be better than 20/400 when they
starts in the second or third decade of get older. Some children display mid-
life, and is maternally inherited (like facial hypoplasia and may have associ-
other disorders of mitochondrial DNA). ated mental retardation.
CSF studies will show elevation of pyru- Retinoblastoma features include uni-
vate and lactate, and the serum creatine lateral progressive visual loss in a young
kinase may be elevated. MRI of the brain child and strabismus. The most common
usually demonstrates cerebral atrophy. initial sign is leucocoria, where the white
Genetic Disorders
105 3
tumor reflects light and blocks view of Which of the following is true regard-
the red colored retina. ing this condition?
Craniopharygioma is the most com- A. It is autosomal d ­ ominant
mon non-glial tumor in children and most B. The deficient enzyme is B-gluco-
commonly occurs between the ages of sylceramidas
6–14. There is a 1–2-year history of progres- C. There is demyelination with rela-
sive visual loss, delayed sexual maturation tive sparing of the U fibers
growth failure and bi-temporal hemianopia. D. It is only seen in newborns, and
Kearns-Sayre syndrome is a mitochon- all patients die by 1 year of age
drial disorder and consists of a triad of E. The cause is deficiency of the
symptoms including: progressive exter- enzyme A-galactosidase
nal ophthalmoplegia, onset before age
20 and one of the following; short stat- vv Correct Answer is: C
ure, pigmentary retinopathy, cerebellar This is a case of Krabbe disease on the
ataxia, heart block, cerebellar ataxia, and basis of the history and histopathologic
elevated cerebrospinal protein. findings. Krabbe disease (globoid cell
leukodystrophy) is a disorder with auto-
zz Suggested Reading somal recessive inheritance, with the
55 Weleber RG, Francis PJ, Trzupek KM, affected gene mapped to chromosome
Beattie C. Leber congenital amaurosis. 2013. 14. This disorder is characterized by the
accumulation of galactocerebroside in
?? 106. You saw a baby who presented with macrophages of the white matter in the
irritability and hypersensitivity to CNS, leading to the formation of glo-
stimuli at 4 months of age, but he boid cells and to progressive demyelin-
died at 11-month of age and a brain ation, but with sparing of the U fibers.
specimen obtained from an autopsy The cause is a deficiency of the enzyme
is shown below. The baby was also galactosylceramidase (also known as
blind and had prominent regression galactocerebrosidegalactosidase). It can
by age 6 months. He also had devel- involve the peripheral nervous system,
oped stiffness of all four limbs and leading to a demyelinating neuropathy,
later of the trunk to the point that he but affects predominantly the CNS.
was in an opisthotonic posture. There are three forms: - Infantile form:
the most common variant and presents
between 4 and 6 months of age with
irritability, hypersensitivity to stimuli,
increasing hypertonicity with eventual
opisthotonos, unexplained low-grade
fevers, optic atrophy with blindness,
psychomotor developmental arrest, and
subsequent regression with loss of previ-
ously achieved milestones. These children
also have a demyelinating polyneuropa-
thy with arreflexia. They usually die by
the age of 1 year. -Juvenile form: the onset
is between 3 and 10 years of age, with
vision loss, spasticity, ataxia, gait distur-
..      Fig. 3.30 Globoid cell leukodystrophy PAS – Muli-
nucleated macrophages (“globoid cells”) and loss of bance, and cognitive impairment. - Adult
myelinated fibers in a case of Krabbe’s leukodystrophy. form: usually starts between the third
(Jensflorian. Wikipedia) and the fifth decade of life, with spastic
106 Chapter 3 · Genetic Disorders

paraparesis, weakness, vision loss, and Which of the following statements


evidence of neuropathy, but intellectual regarding this patient’s disorder is
function tends to be normal. false?
Pathologically, there is symmetric A. Chiari 1 malformations may
demyelination of the cerebral white mat- be asymptomatic and do not
ter, with relative sparing of the subcorti- ­necessarily require surgical
3 cal arcuate or U fibers. The pathologic ­intervention
specimen shown in the Figure is typical of B. Myelomeningocele occurs
Krabbe disease, demonstrating clusters in patients with Chiari II
of globoid cells, which are multinucleated ­malformation
macrophages with cytoplasmic accumu- C. Hydrocephalus is a complication
lation of galactocerebroside. of Chiari malformation
Radiologically, there is symmetric D. Management of Chiari malforma-
periventricular white matter signal tions may include sub-occipital
abnormality and cerebral atrophy. NCS decompression
demonstrate slow conduction velocities E. Syringomyelia, which may occur
and prolonged distal latencies. CSF exam- in association with a Chiari I
ination shows elevated protein levels. malformation, is an enlargement
of the central canal of the spinal
?? 107. You are evaluating an 8-year-old girl cord
with gait abnormalities and head-
ache. On examination, she has nys- vv Correct Answer is: E
tagmus, and truncal and limb ataxia. While syringomyelia and hydromyelia
are erroneously used interchangeably
by some, the two are distinct by defini-
tion. Syringomyelia is a fluid-filled cavity
within the spinal cord that is separate
from the central canal; hydromyelia is
the termed used to describe an enlarge-
ment in the central canal itself. Chiari 1
malformation is defined as displace-
ment of the cerebellum and cerebellar
tonsils downward through the fora-
men magnum. The MRI shows a Chiari I
malformation with 1.3 cm downward
displacement of the cerebellar tonsils.
In minor downward displacement of
less than 1 cm, the patient may be
asymptomatic and care should be taken
in attributing nonspecific neurologic
symptoms to the Chiari malformation.
In more severe downward displace-
ment, as in this case, headache, cranial
nerve abnormalities, and other brain
stem symptoms, nystagmus, and ataxia
may occur. Associated findings include
..      Fig. 3.31 Transferred from 7 en.wikipedia to syringomyelia. The pathophysiology
Commons. Cyborg Ninja at English Wikipedia of Chiari I malformation may relate to
Genetic Disorders
107 3
­ osterior fossa overcrowding due to pos-
p vv Correct Answer is: D
terior fossa hypoplasia. In symptomatic This is a presentation of Tangier dis-
cases, treatment includes posterior fossa ease, which is an autosomal reces-
decompression and duraplasty. Chiari sive familial neuropathy, caused by
II malformation, also known as Arnold- mutation affecting the adenosine
Chiari malformation, includes displace- triphosphate cassette transporter pro-
ment of the cerebellar vermis and tonsils tein, resulting in a deficiency of high
in association with a myelomeningocele. density lipoprotein (HDL) and very low
Brain stem dysfunction is often promi- serum cholesterol and high triglycer-
nent, including prominent cranial nerve ide concentrations. Given the severely
abnormalities, stridor, apnea, and feed- reduced HDL level, cholesteryl esters
ing difficulties. Fourth ventricle com- accumulate in various tissues, includ-
pression leads to hydrocephalus. ing tonsils, peripheral nerves, cornea,
Pathophysiological, Chiari II malfor- bone marrow, and other organs of the
mation is thought to be secondary to the reticuloendothelial system. A typical
presence of the caudal myelomeningocele, clinical finding is the enlarged orange
producing downward traction and hence tonsils. Peripheral neuropathy is com-
herniation of the brain stem and cerebel- mon and manifests with sensory loss
lum through the foramen magnum. Man- to pain and temperature that may
agement of Chiari II malformation includes have a pattern in the upper extremities
shunting for hydrocephalus and surgical similar to that seen in syringomyelia
intervention for myelomeningocele and and may affect the entire body. Motor
may include posterior fossa decompres- involvement may manifest with weak-
sion through suboccipital craniotomy. ness that affects the upper and lower
Management of complications includ- extremities and particularly the hand
ing seizures, feeding difficulties, and muscles. A symmetric polyneuropathy
bowel and bladder dysfunction is also is common; however, a mono-neurop-
necessary. Chiari Ill malformation is cere- athy ­presentation can also be seen.
bellar herniation into a cervical encepha- Deep tendon reflexes are depressed.
locele. Chiari IV malformation is the term Cranial nerves may also be involved.
previously used to describe ­cerebellar Premature atherosclerosis also occurs.
hypoplasia but is no longer used. The diagnosis is suspected on the basis
of clinical features, and a lipid profile
?? 108. A 12-year-old boy is found to have showing HDL deficiency, with low total
large orange-colored tonsils, hepa- cholesterol and high triglyceride
tosplenomegaly, and sensory defi- levels. Foamy macrophages are
cits in the upper limbs. present in the bone marrow and
Which of the following is true regard- other tissues. There is no specific
ing this condition? ­treatment.
A. It is caused by deficiency of low-
density lipoproteins ?? 109. You are evaluating a 16-year-old
B. It is autosomal dominant in male with growth and psycho-
inheritance motor retardation, sensorineural
C. Peripheral nerves are not involved hearing loss, frequent headaches,
D. It is caused by a mutation in the seizures, and multiple stroke-like
adenosine triphosphate cassette episodes. CT scan and MRI from dif-
transporter protein ferent time points are shown in the
E. Total cholesterol level in the figure. ­Lactate level is found to be
serum is very elevated elevated.
108 Chapter 3 · Genetic Disorders

a b

c d

..      Fig. 3.32 a A computed tomography brain scan spectroscopy showing inversion of J-coupling
showing bilateral basal ganglia calcification; the phenomenon at 1.3 ppm, indicating lactate peak.
cerebellum shows prominent folia indicating mild (Abu-Amero et al. Journal of Medical Case Reports 2009
cerebellar atrophy. b Axial T2 brain magnetic 3:77 7 https://doi.org/10.1186/1752-1947-3-77). A
resonance image scan showing left temporo-parieto patient with typical clinical features of mitochondrial
occipital ischemic lesion. c Axial T2 brain magnetic encephalopathy, lactic acidosis and stroke-like
resonance image scan showing the extension of the episodes (MELAS) but without an obvious genetic
parietal temporal region to the occipital lobe, and also cause: a case report. Abu-Amero KK, Al-Dhalaan H,
showing a right occipital lesion. d Magnetic resonance Bohlega S, Hellani A, Taylor RW. Wikipedia

What is the most likely diagnosis? ­ itochondrial disorder with typical


m
A. Kearns-Sayre syndrome maternal inheritance; however, sporadic
B. Mitochondrial encephalopathy, cases may occur. The most common
lactic acidosis, and strokes mutation is in the mtDNA MTTL1 gene
C. Myoclonic epilepsy with ragged encoding for an mtDNA tRNA. The onset
red fibers is often between 2 and 10 years of age,
D. Leigh disease although symptoms may begin at any
E. Neuronal ceroid lipofuscinosis age, including adulthood. Patients are
normal at birth, later manifesting with
vv Correct Answer is: B seizures, migraine headaches, vomit-
Mitochondrial encephalopathy, lac- ing with anorexia, exercise intolerance,
tic acidosis and strokes (MELAS) is a and weakness. Failure to thrive, growth
Genetic Disorders
109 3
retardation, and progressive deafness basal ganglia, and cerebellum. Most of
are common in these children. Stroke- the affected patients have onset of neu-
like episodes occur, presenting with rologic manifestations in the first year of
transient hemiparesis, cortical blind- life, but there are forms with late onset.
ness, and altered consciousness, with Clinical features manifest with decom-
cumulative residual effects leading pensation associated with inter-current
to gradually progressive neurologic illnesses. In infancy, patients present
impairment, leading to mental dete- with hypotonia, loss of head control,
rioration and encephalopathy. The MRI poor sucking, vomiting, irritability, sei-
shown in the figure demonstrates multi- zures, and myoclonic jerks. If the onset
focal infarcts that do not correlate with is beyond the first year, patients present
definite vascular territories. Initially, the with gait disturbance, cerebellar ataxia,
infarcts occur in the posterior cerebral dysarthria, psychomotor retardation,
regions, with eventual involvement of and spasticity, external ophthalmople-
other cerebral and cerebellar cortices, gia, nystagmus, abnormal movements
basal ganglia, and the thalamus. The with chorea or dystonia, and periph-
infarcts are extremely epileptogenic. eral neuropathy in some cases with
Lactate level is elevated in the blood autonomic failure. The disorder is
and CSF. Muscle biopsy may demon- progressive with episodic deteriora-
strate ragged red fibers. There is no spe- tion. Lactate level is increased in blood
cific treatment. These patients may have and CSF. Lactate and pyruvate levels
some benefit from coenzyme Q1O and in blood are elevated during exacerba-
L-carnitine. tions. MRI of the brain demonstrates
L-arginine may attenuate the severity bilateral symmetric hyper- intense T2
of strokes and reduce the frequency of signal abnormalities in the brain stem
these events. and/or basal ganglia, and in some cases
in the spinal cord. Treatment is supportive
?? 110. Which of the following options is and symptomatic.
true regarding Leigh disease?
A. It is a static encephalopathy ?? 111. You are seeing a 5-month-old
B. It is due to mitochondrial abnor- baby for evaluation of seizures. He
malities has a prominent startle response
C. It is inherited in an autosomal and developmental delay. He had
recessive fashion reached some developmental
D. Lactate levels are reduced during milestones but has started regress-
episodic exacerbations ing now. It is also noticed that he
E. Clinical manifestations most is macrocephalic and spastic. A
commonly begin during late cherry-red spot is seen on oph-
childhood thalmologic examination, and he
also has prominent hepatospleno-
vv Correct Answer is: B megaly. There is suspicion for a GM2
Leigh disease or acute necrotizing gangliosidosis.
encephalomyelopathy is a manifesta- Which of the following is true regard-
tion of mitochondrial disorders that can ing this patient’s condition?
be sporadic or familial, with only some A. Hexosaminidase A is the only
cases with the typical maternal inheri- enzyme affected
tance pattern. This condition affects B. Both hexosaminidases A and B
neurons of the brain stem, thalamus, are affected
110 Chapter 3 · Genetic Disorders

C. T-Galactosidase deficiency is the involved, whereas in Sandhoff disease,


most common cause the GM2 gangliosides accumulate in
D. Sphingomyelinase deficiency is the brain and viscera, causing hepato-
the cause splenomegaly that is not seen in Tay-
E. This patient has Tay-­Sachs Sachs disease. The diagnosis is based
­disease on these clinical features and con-
3 firmed with analysis of the enzymatic
vv Correct Answer is: B activity of the enzymes involved.
This patient has GM2 gangliosidosis,
more specifically Sandhoff disease ?? 112. You are seeing an 18-year-old female
caused by deficiency of hexosamini- in the ED for severe abdominal pain,
dase A and B. Galactosidase deficiency along with nausea, vomiting, fever,
causes GM1 but not GM2 gangliosi- and tachycardia. She has been evalu-
dosis. Sphingomyelinase deficiency ated multiple times over the past
causes Niemann-Pick types A and year for similar symptoms and has
B. GM2 gangliosidosis is caused by undergone evaluation for appendi-
deficiency of ­hexosaminidase A in citis and other gynecologic causes,
Tay-Sachs disease or hexosaminidase including exploratory laparoscopy
A and B in Sandhoff disease. Both are in the past. There are no cutaneous
autosomal recessive. Tay-Sachs disease manifestations. She has increased
or infantile GM2 gangliosidosis is more aminolevulinic acid and porphobi-
common in Ashkenazi Jews but may linogen (PBG) concentrations in the
also occur in other populations with urine, with no increase in urinary or
less frequency, and the CNS is the only fecal coproporphyri n Ill.
affected organ. Onset is between 3 Which of the following is true regard-
and 6 months of age, with increased ing this disorder?
startle response and subsequent motor A. The enzyme affected is PBG
regression, spasticity, blindness with deaminase
optic atrophy, and seizures. There is B. The enzyme affected is copro-
a delay in reaching developmental porphyrinogen oxidase
milestones, with subsequent regres- C. The enzyme affected is protopor-
sion. A cherry-­red spot in the macula phyrinogen oxidase
is commonly seen, and these patients D. Seizures can occur and should be
have macrocephaly. Progression to treated with phenobarbital
severe intellectual disability occurs, E. A symmetric distal demyelin-
and most children die by the age of ating polyneuropathy occurs
5 years. The diagnosis is suspected in ­predominantly in the lower
patient with psychomotor retardation limbs
and a cherry-­red spot and is confirmed
with the detection of hexosaminidase A vv Correct Answer is: A
deficiency with normal activity of hex- This is a case of porphyria, likely acute
osaminidase B. Treatment is supportive. intermittent porphyria (AIP) given
Sandhoff disease occurs from com- the lack of cutaneous manifestations
bined deficiency of hexosaminidase A and the absence of increased copro-
and B. The clinical features are similar porphyrin Ill in the urine and stool,
to those seen in Tay-Sachs disease; therefore making variegate porphyria
however, as mentioned, in Tay­Sachs (VP) and hepatic coproporphyria (HCP)
disease, the brain is the only organ less likely. The enzyme involved in
Genetic Disorders
111 3
AIP is porphobilinogen deaminase. Seizures can occur from neurologic
Porphyrias are a group of metabolic dis- involvement or may result from the
orders caused by deficiency of a specific hyponatremia that is seen in these
enzyme involved in the heme biosyn- patients.
thetic pathway. Most of the porphyrias Neuropsychiatric symptoms such as
are inherited in an autosomal domi- anxiety, insomnia, depression, disorienta-
nant fashion, except 5-­aminolevulinic tion, hallucinations, and paranoia may
acid dehydratase deficient porphyria, occur. The diagnosis is based on clinical
which is autosomal recessive. AIP has suspicion and elevated urinary excre-
primarily neurologic manifestations, tion of ALA and PBG during the attacks.
HCP and VP have a combination of neu- Genetic testing and enzyme analysis are
rologic and cutaneous manifestations also helpful.
with photosensitivity, and porphyria Management is focused on prevent-
cutanea tarda has cutaneous, with no ing attacks by avoiding precipitating fac-
neurologic, manifestations. The acute tors. During attacks, these patients may
porphyrias manifest with attacks of need hospitalization for hydration and
neurovisceral symptoms, with markedly pain control. Carbohydrates decrease the
elevated levels of plasma and urinary synthesis of porphyrins, and an infusion
concentrations of the porphyrin pre- may be required. Hematin infusions may
cursors aminolevulinic acid (ALA) and be helpful. For seizures, clonazepam and
porphobilinogen (PBG). Levels are usu- gabapentin may be helpful.
ally elevated during attacks and may be Unfortunately, many AEDs precipitate
normal in between. The attacks may be attacks, especially barbiturates, which
triggered by drugs such as barbiturates, should be avoided. HCP is caused by
sulfonamides, AEDs, and hormones coproporphyrinogen oxidase deficiency,
among other medications. Attacks may and VP is caused by protoporphyrinogen
also be triggered by a low-­carbohydrate oxidase deficiency. In both, the neurovis-
diet, infections, or other intercurrent ceral manifestations are similar to AIP, but
illnesses. AIP is the most common of in HCP and VP, there are cutaneous mani-
the acute porphyrias with neurovisceral festations with photosensitivity, abnor-
symptoms and is caused by deficiency mal skin fragility, and bullous skin lesions
of PBG deaminase, with the gene local- on sun-exposed areas. HCP and VP cause
ized on chromosome 11. Onset occurs increased urinary and fecal levels of cop-
after puberty, with acute attacks of roporphyrin Ill.
abdominal pain, sometimes associ-
ated with nausea, vomiting, diarrhea, ?? 113. An Ashkenazi Jewish family of
fever, tachycardia, and leukocytosis. 6-month-old boy brought the
Commonly these patients experience baby for evaluation of seizures. He
limb pain and muscle weakness result- has prominent startle response
ing from a peripheral neuropathy that and developmental delay. He had
is predominantly motor and axonal, reached some early developmen-
affecting more proximal than distal tal milestones but then began to
segments and more the upper than regress. It is also noticed that he
the lower extremities. Deep tendon is macrocephalic and spastic, but
reflexes are depressed. The radial nerve there is no visceromegaly. A cherry-
has been described as being classically red spot is seen on ophthalmologic
involved, and in severe cases, there may examination. There is suspicion for a
be bulbar, and respiratory ­involvement. GM2 gangliosidosis.
112 Chapter 3 · Genetic Disorders

What is correct regarding this disease; however, as mentioned, in


patient’s condition? Tay-Sachs disease, the brain is the only
A. This patient has Sandhoff disease organ involved, whereas in Sandhoff
B. Hexosaminidase A is the only disease, the GM2 gangliosides accumu-
cause late in the brain and viscera, causing
C. It is autosomal dominant hepatosplenomegaly that is not seen
3 D. It occurs only in Ashkenazi Jews in Tay-Sachs disease. The diagnosis is
E. B-Galactosidase deficiency is the based on these clinical features and
most common cause confirmed with analysis of the enzy-
matic activity of the enzymes involved.
vv Correct Answer is: B
The baby has a GM2 gangliosidosis, ?? 114. Which of the following statements
more specifically Tay-Sachs disease is correct regarding disorders of cor-
caused by hexosaminidase A defi- pus callosum development?
ciency. Galactosidase deficiency causes A. They result from abnormalities in
GM1 but not GM2 gangliosidosis. the third trimester of pregnancy
Sphingomyelinase deficiency causes B. Most commonly, complete
Niemann-Pick types A and B. GM2 agenesis of the corpus callosum
­gangliosidosis is caused by deficiency occurs
of hexosaminidase A in Tay-Sachs C. They result from abnormalities in
disease or hexosaminidase A and B in the commissural plate
Sandhoff disease. Both are autosomal D. Corpus callosum agenesis is
recessive. Tay-Sachs disease or infantile most often seen in isolation
GM2 gangliosidosis is more common in E. Severe developmental delay is
Ashkenazi Jews but may also occur in invariably present
other populations with less frequency,
and the CNS is the only affected organ. vv Correct Answer is: A
Onset is between 3 and 6 months of During the fifth week of gestation,
age, with increased startle response within the lamina terminalis, the com-
and subsequent motor regression, missural plate develops and serves as
spasticity, blindness with optic atrophy, a bridge over which axonal processes
and seizures. There is a delay in reach- decussate. The corpus callosum is fully
ing developmental milestones, with developed by week 17th of gestation.
subsequent regression. A cherry-­red Abnormalities in the commissural plate
spot in the macula is commonly seen, lead to agenesis or dysgenesis of the
and these patients have macrocephaly. corpus callosum. Agenesis of the cor-
Progression to severe intellectual dis- pus callosum may be complete or more
ability occurs, and most children die commonly partial and may occur in
by the age of 5 years. The diagnosis is isolation or more commonly is associ-
suspected in patients with psychomotor ated with dysplasias of other prosen-
retardation and a cherry-­red spot and cephalon derivatives and aplasia of the
is confirmed with the detection of hex- cerebellar vermis. It may also be seen
osaminidase A deficiency with normal in a variety of syndromes, including
activity of hexosaminidase Treatment Aicardi syndrome, or may be associ-
is supportive. Sandhoff disease occurs ated with metabolic disorders such as
from combined deficiency of hexosa- nonketotic hyperglycinemia. Clinical
minidase A and B. The clinical features manifestations may be absent, or there
are similar to those seen in Tay-Sachs may be subtle cognitive or perceptual
Genetic Disorders
113 3
abnormalities; developmental delay and ?? 116. You are evaluating a 6-year-old boy
seizures may also occur. Hypertelorism for seizures. He has had gradual
is seen in many. Coronal MRI shows ver- vision loss and progressive psycho-
tically oriented lateral ventricles (“steer motor retardation. Neuronal ceroid
horn sign,” or “racing car sign” on axial lipofuscinosis (NCL) is within the dif-
images). ferential diagnosis.
Which of the following is true
?? 115. Which of the following malforma- ­regarding NCL?
tions is not paired with the appropri- A. NCL occurs only in childhood
ate underlying mechanism? B. It is autosomal dominant
A. Focal cortical dysplasia-­disorder C. Myoclonic seizure is the least
of cell proliferation common type of seizure seen in
B. Lissencephaly-­disorders of neu- these patients
ronal migration D. The infantile form is more com-
C. Polymicrogyria-­disorder of corti- mon in Finnish people
cal organization E. Electron microscopy is useless as
D. Periventricular nodular heteroto- there are no characteristic findings
pias-disorder of cell p
­ roliferation
E. Schizencephaly-­disorder of corti- vv Correct Answer is: D
cal organization Neuronal ceroid lipofuscinosis is a
group of autosomal recessive disorders
vv Correct Answer is: D characterized by progressive psycho-
Periventricular nodular heterotopia motor retardation, seizures, and blind-
is a disorder of neuronal migration. ness, which can present in infantile,
Malformations of cortical development late infantile, juvenile, and adult forms.
are divided into three categories based Mutations in eight genes causing the
on the underlying cause: disorders of disorder (CLN1 through CLN8) have
cell proliferation, migration, or cortical been described. The most commonly
organization. involved genes are CLN1 and CLN2.
Disorders of neuronal proliferation CLN1 encodes for palmitoyl protein
include some forms of megalencephaly thioesterase 1 (PPT1), and CLN2 encodes
and focal cortical dysplasia. for tripeptidyl peptidase 1 (TPP1). The
Disorders of neuronal migration storage products are saposins A and
include lessencephaly, periventricular D in the infantile forms and subunit
nodular heterotopias, and others. C of ATP synthase in the other forms.
Disorders of cortical organization The infantile form is most common in
include polymicrogyria and schizen- Finland and is associated with CLN1
cephaly. gene mutations. These patients are nor-
Focal cortical dysplasia may be seen mal at birth, with onset of symptoms
in isolation or in the setting of tuber- between 6 and 24 months, including
ous sclerosis. One type of focal cortical microcephaly, hypotonia, myoclonus,
dysplasia is characterized pathologically seizures, ataxia, progressive psycho-
by the presence of balloon cells, which motor retardation, and blindness.
result from proliferation of abnormal cells Neurons accumulate membrane-bound
within the germinal matrix. This is a com- osmophilic deposits (seen on electron
mon pathology in focal epilepsy, with the microscopy) and neuronal loss with
seizures often being intractable to medi- cortical atrophy will ensue. The late-
cal therapy. infantile form presents between 2 and
114 Chapter 3 · Genetic Disorders

4 years of age and is associated with vv Correct Answer is: B


CLN2 mutations, and these patients This patient’s history and imaging find-
develop seizures, myoclonus, ataxia, ings are consistent with septo-optic
involuntary movements, blindness, dysplasia, a group of malformations
and psychomotor retardation. Neurons that include hypoplasia or absence of
accumulate curvilinear bodies, which the septum pellucidum, optic nerve and
3 are seen on electron microscopy. The optic chiasm hypoplasia, dysgenesis of
juvenile form begins between 4 and 10 the corpus callosum and anterior com-
years of age and is associated with CLN3 missure, and fornix detachment from
mutation, and these patients develop the corpus callosum. Arrhinencephaly
visual loss, seizures, myoclonus, psycho- (agenesis of only the olfactory bulb and
motor retardation, and focal neurologic tract) and/or hypothalamic hamartomas
deficits. The storage product consists may be associated features. Other less
of fingerprint bodies, which are seen commonly associated abnormalities
on electron microscopy. The adult form include cerebellar vermis defects and
is evident at approximately 30 years of hydrocephalus. Septo-­optic dysplasia
age and is ­characterized by myoclonus, can also be associated with lobar holo-
ataxia, behavioral abnormalities, and prosencephaly and other malformations
dementia. The storage material consists of cortical development. Clinical mani-
of a combination of fingerprint bodies, festations include vision loss, ataxia
granular osmiophilic deposits, and recti- when the cerebellum is involved, and
linear profiles. The diagnosis is based on symptoms of hydrocephalus when it
the clinical presentation and supported is present, and endocrinologic distur-
with electron microscopic examination bances. Endocrinologic disturbances
of lymphocytes or cells from other tis- can range from panhypopituitarism
sues. Enzyme activity studies for PPT1 (with deficiencies in both anterior and
and TPP1 are also available, as is genetic posterior pituitary hormones) to iso-
testing for all of the identified CLN lated hormone deficiencies. Mutations
genes. The treatment is symptomatic. in the transcription factors HESX1, a
homeobox gene, and SOX may be impli-
?? 117. You are seeing a 6-year-old boy cated in this disorder. Cavum septum
with short stature secondary to pellucidum, in which the septum pel-
growth hormone deficiency and lucidum is not fused but rather exists in
is also being treated for hypothy- two separate pieces of tissue, is consid-
roidism. His motor and cognitive ered non-pathologic, with little clinical
development has been normal. He implications. The presence of normal
has reduced visual acuity. He has cortex excludes holoprosencephaly, and
absence of the septum pellucidum lissencephaly. This patient has several
and hypoplasia of the optic nerves features making septa-optic dysplasia
and chiasm on MRI. The cortex and the likely diagnosis rather than arrhin-
other areas appear normal. encephaly.
What is your likely diagnosis in this
patient? ?? 118. You are evaluating a 5-year-old girl
A. Holoprosencephaly for her involuntary writhing move-
B. Septo-optic dysplasia ments in her arms with small, ran-
C. Lissencephal dom, superimposed twitches. Her
D. Arrhinencephaly past medical history is remarkable
E. Cavum septum pellucidum for asthma and frequent pneumonia
Genetic Disorders
115 3
and sinusitis. On examination, she chest radiography and sunbathing. Het-
has hypometric saccades, forced erozygotes are also at risk for malignancy,
blinking to initiate saccades, truncal especially breast cancer.
ataxia, and choreoathetosis. Skin
examination shows telangiectasias zz Suggested Reading
on the bulbar conjunctivae, on the 55 Perlman S, Becker-­Catania S, Gatti RA.
tops of the ears, along the malar Ataxia-telangiectasia: diagnosis and
prominences of the face, and within treatment. Semin Pediatr Neurol
the flexor creases of the elbows and 2003;10:173–82.
knees.
What test is most likely to be ?? 119. You are evaluating an infant with
­diagnostic? epileptic encephalopathy. He has
A. Alpha-fetoprotein (AFP) and car- microcephaly and prominent devel-
cinoembryonal antigen (CEA) opmental delay. On CSF analysis, the
B. Chest radiograph glucose level is 30 mg/dl, whereas
C. EMG/NCS the serum glucose level is 112 mg/dl.
D. MRI of the brain and lumbar Other CSF tests are normal.
puncture What is correct regarding this
E. Urine for catecholamines ­condition?
A. Brain MRI usually suggests the
vv Correct Answer is: A diagnosis
This patient has Ataxia telangiectasia B. It is caused by deficiency in the
(AT). This child has the classic triad of glucose transporter type 1
ataxia, telangiectasia, and frequent sin- C. Phenobarbital is the treatment of
opulmonary infections, which suggests choice
a diagnosis of ataxia­telangiectasia (AT). D. Ketogenic diet has been shown
AT is an autosomal recessive disorder to be ineffective for seizure
affecting the ATM gene on chromo- ­control
some 11, which is a gene involved in E. It is inherited in an X-­linked
cell-cycle progression and DNA repair. ­fashion
Ataxia begins proximally and spreads
distally, and choreoathetosis is fre- vv Correct Answer is: B
quent. Telangiectasias appear between This patient has a glucose transporter
age 2 and 10 years, and mild intellectual type 1 (GLUT-1) deficiency. The brain
disability occurs. There is an increased utilizes glucose as its primary source of
tendency to develop malignancies, energy. In fasting conditions, glycogen
especially lymphomas and leukemias. is exhausted within minutes, and since
The immune system is also impaired, amino acids and fat cannot be used for
accounting for recurrent infections. AFP the production of energy in the brain,
is often elevated, CEA may be elevated, ketones become the alternative fuel.
and lgG (especially subclasses 2 and 4), Glucose crosses the blood brain bar-
lgA, and lgE are depressed. Together rier facilitated by GLUT-1, which is a
these tests have up to 90% sensitivity. membrane-bound protein encoded
Specific testing for the ATM gene by the SLC2A 1 gene on chromosome
is also available. Brain MRI may show 1p35–31.3. GLUT-1 deficiency syndrome
cerebellar atrophy. Management of AT is inherited in an autosomal dominant
includes aggressive treatment of infec- fashion and causes a defect in glucose
tions and avoiding radiation, including transport across the blood-­brain barrier
116 Chapter 3 · Genetic Disorders

and into brain cells, manifesting as an does not show specific abnormalities.
epileptic encephalopathy with infantile- Later-onset forms with episodic move-
onset seizures, developmental delay, ment disorders and ataxia, paroxysmal
microcephaly, and complex movement exertional dyskinesia, or early-onset
disorders. CSF glucose level is low with atypical absence epilepsy have been
a normal serum glucose level, and other described. A ketogenic diet should
3 CSF studies are normal, excluding other be started as soon as the diagnosis is
causes of hypoglycorrhachia (such as CNS suspected, since this treatment option
infection). EEG may show 2.5–4 Hz spikes improves seizure control and the abnor-
and waves and the inter-ictal EEG findings mal movements; however, it is less effec-
may improve with glucose. Neuroimaging tive for the psychomotor impairment.
117 4

Congenital

© Springer International Publishing AG, part of Springer Nature 2018


Y. M. Awaad, Absolute Pediatric Neurology, https://doi.org/10.1007/978-3-319-78801-2_4
118 Chapter 4 · Congenital

?? 1. You are evaluating a one-month-old zz Suggested Reading


baby for severe hypotonia since birth. 55 Steinberg S, Raymond GV. Peroxisome
His pregnancy and delivery were biogenesis disorders, Zellweger syndrome
normal. On examination, he has a flat spectrum. Gene Revi. At 7 http://www.
occiput, frontal bossing, large fonta- genereviews.org.
nelles, epicanthal folds, shallow supra-
orbital ridges and hepatomegaly. ?? 2. You are seeing a 2-year-old girl for her
What test is most likely to be diag- progressive neurological disorder. She
4 nostic? was a full-term baby with normal and
A. Chromosome analysis an uncomplicated pregnancy. She
B. CT scan of the brain was reportedly normal at birth. She
C. Plasma carnitine levels achieved her normal milestones in the
D. Plasma very long chain fatty acid first year of life, with walking and bab-
levels bling by 14 months. But at 16 months
E. Skeletal X-rays of age, she began falling and became
clumsy with her hands. At 18 months
vv Correct Answer is: D she stopped babbling and walking;
Peroxisomes are cytoplasmic organelles subsequently she developed loss of
in all mammalian cells except RBC; single visual function, myoclonic seizures,
membrane. They contain no nucleic acid; and progressive microcephaly.
the proteins are encoded by nuclear genes What is the most likely diagnosis?
and translated on free cytoplasmic ribo- A. Methylmalonic academia
somes. They have many cellular anabolic B. Mitochondrial disorder
and catabolic functions including degrada- C. Neuronal ceroid lipofuscinosis
tion of very long chain fatty acids (C22, D. Ornithine transcarbamylase defi-
C24, and C26), early steps of plasmalogen ciency
biosynthesis, degradation of phytanic acid, E. SCN1A gene mutation
selected steps in cholesterol biosynthesis,
degradation of pipecolic acid, synthesis vv Correct Answer is: C
of bile acid intermediates, and glyoxylate The neuronal ceroid lipofuscinoses (NCLs)
metabolism. Peroxisomal disorders result are the most common group of genetic
from failure to form or maintain the per- storage disorders that lead to progressive
oxisomes or a defect in one of the enzymes diseases of children and young adults.
normally found in the peroxisome. They are characterized by neurological
Zellweger syndrome is due to total regression, epilepsy, and visual loss due
absence of liver peroxisomes. Patients to retinal degeneration. Pathologically,
present with dysmorphic facies, hypotonia, they are storage disorders with accumu-
seizures, mental retardation, neuronal lation of an auto-fluorescent material,
heterotopias, cataracts and/or glaucoma, ceroid lipofuscin, in combination with
and renal cysts; about 50% have epiphyseal the degeneration of neuronal cells. There
calcifications. Death usually occurs by 6–12 are at least 10 genetically distinct NCLs,
months. The diagnosis depends on designated CLN1 to CLN10 and genetic
showing lack of all peroxisomal function. testing is available for many of them.
No peroxisomes would be seen on EM. Several NCLs exhibit a widely variable
Elevated VLCFA or low plasmalogens are clinical picture, depending on the sever-
good screening tests. ity of the individual mutation. NCL may
Congenital
119 4
be suspected by the clinical history and vv Correct Answer is: B
supplemented by the finding of leuko- This clinical presentation should raise the
cyte inclusions on skin biopsy or buffy suspicion of Canavan disease because
coat analysis. of the triad of hypotonia, macrocephaly,
and head lag in an infant after the age of
zz Suggested Reading three to 5 months. Very high concentra-
55 Kohlschütter A, Schulz A. Towards tion of N-acetyl aspartic acid (NAA) in
understanding the neuronal ceroid the urine is diagnostic of symptomatic
lipofuscinoses. Brain Dev. 2009;31(7): Canavan disease patients. NAA concen-
499–502. tration is also elevated in the blood and
cerebrospinal fluid (CSF) of children with
?? 3. You are evaluating a 4-year-old boy Canavan disease. The elevated NAA can
for his motor delay. He was born at full be seen by 1H MRS as a single peak with
term after normal and an uneventful a resonance of 2.01 ppm on the spec-
pregnancy. He was normal at birth trum.
and in the first few months of life, but
thereafter developed macrocephaly zz Suggested Reading
and was delayed in his milestones. He 55 Janson CG, McPhee SW, Francis J, Shera
has no family history of such case. On D, Assadi M, Freese A, Hurh P, Haselgrove
your examination, he is hypotonic and J, Wang DJ, Bilaniuk L, Leone P. Natural
his head circumference is above the history of Canavan disease revealed by
95th percentile. A brain MRI scan is proton magnetic resonance spectroscopy
obtained and is shown.    (1H-MRS) and diffusion-weighted
MRI. Neuropediatrics. 2006;37:209–21.

?? 4. You are seeing a 15-yr. old boy with


history of intermittent falling for
2 years. His examination at that time
revealed ataxia, dysarthric speech and
multifocal myoclonus, and he was
diagnosed with myoclonic seizures.
One year ago, he developed intrac-
table seizures including myoclonic
and generalized tonic clonic seizures.
He also complained of transient epi-
sodes of visual loss. He also developed
progressive multifocal myoclonus at
times brought on by auditory stimula-
..      Fig. 4.1 Radiology picture of the day. Dr. Laughlin
tion. Now, he lost his ability to walk
Dawes. Canavan disease. Wikipedia
and became non-ambulatory, has a
tracheostomy to help with handling
What is the most likely diagnosis? of secretions, is nonverbal and does
A. Adrenoleukodystrophy not follow commands. Brain neuro-
B. Canavan disease imaging showed severe diffuse brain
C. Krabbe disease atrophy. His optical fundus examina-
D. Metachromatic leukodystrophy tion was normal. A skin biopsy slid is
E. Vanishing white matter disease shown.   
120 Chapter 4 · Congenital

a b

..      Fig. 4.2 Lafora bodies on skin biopsy. a Histo- sweat glands, corresponding to Lafora bodies (arrows).
pathologic examination of the skin biopsy taken from Periodic acid–Schiff stain, 200×. b Lafora bodies in a
the axillary region reveals round-to-oval intracytoplas- magnified view (arrows). Periodic acid–Schiff stain,
mic, periodic acid–Schiff-positive, diastase-resistant 400×. Print ISSN: 0028–3878 Online ISSN: 1526-632X
inclusions within the myoepithelial and acinar cells of

What is the most likely diagnosis? shows global atrophy in later stage which
A. Neuronal ceroid lipofuscinosis is normal initially. Electroencephalograms
B. Lafora body disease show paroxysmal spike wave bursts and
C. Dravet syndrome poly spike and wave with an occipital
D. Tay Sach’s disease predominance. Diagnosis can be made by
E. Juvenile Myoclonic Epilepsy skin biopsy with characteristic PAS posi-
tive eccrine duct deposits as shown in the
vv Correct Answer is: B picture below. The putative genes have been
At age 13 he developed progressive myo- named EPM2A and EPM2B (epilepsy with
clonic seizures, progressive stimulus induced progressive myoclonus).
myoclonus and loss of milestones eventually Juvenile neuronal ceroid lipofuscinosis
leading to a vegetative state. This is con- could present in a similar fashion, but
sistent with a neurodegenerative disorder, typically presents at a younger age, around
specifically one associated with progressive 5–8 yrs. of age; the retinal exam would show
myoclonic epilepsy. There are a group of a pigmentary retinopathy, with progressive
disorders that fall into this description. These loss of vision occurring early on.
include neuronal ceroid lipofuscinosis, Unver- Dravet syndrome is a disorder, which
richt Lundborg, Disease (ULD), Lafora Body typically presents in infants and children with
Disease, myoclonic epilepsy with ragged red a history of multiple febrile seizures, febrile
fibers (MERRF), and sialidosis. Lafora body hemiclonic status epilepticus and myoclonic
disease patients have normal development seizures. There is slowing of development
until 13 to 17 years of age when they develop starting around the age of 2 years, and
ataxia, dysarthria and eventually progressive typically has a slower progression.
myoclonus. They develop myoclonic epilepsy, Juvenile myoclonic epilepsy is associ-
occipital seizures with episodic loss of vision, ated with early morning myoclonic jerks,
and progressive myoclonus. They experience typically nocturnal generalized tonic clonic
eventual loss of all acquired milestones, with seizures and poly-spike and wave on EEG. It
progressive dementia and typically leading is not typically associated with neurodegen-
to a vegetative state. Initial Neuroimaging eration.
Congenital
121 4
Juvenile Tay-Sach’s disease usually Wegrzyn G. Drug-resistant epilepsia and
presents around 2–10 years of age. fulminant valproate liver toxicity. Alpers-
Symptoms include loss of cognitive Huttenlocher syndrome in two children
milestones, difficulty with gait and at times confirmed post mortem by identification
seizures. It is related to a deficiency of of p. W748S mutation in POLG gene. Med
hexosaminidase A. Progression is slower Sci Monit. 2011;17:CR203–209.
than Lafora Body disease and the skin
biopsy would not show the results seen for ?? 6. You are seeing a 5-month-old
this patient. adopted Chinese boy because the
family noticed episodes of incon-
zz Suggested Reading solable screaming, arching of his
55 Zhao X, Scherer SW, Ackerley CA, Berge back and neck, and apparent lack
A. Minassian Early-onset Lafora body of awareness of his surroundings
disease. Brain. 135(9):2684–98. right after they bring him home.
55 Anna C Jansen, MD, PhD and Eva On your exam, he has no deep ten-
Andermann, MD, PhD, FCCMG. Progressive don reflexes but has paradoxically
Myoclonus Epilepsy, Lafora Type. Gene increased tone both axially and
Reviews. Initial Posting: December 28, 2007; appendicularly.  
Last Update: November 3, 2011. What abnormalities would you
55 Zupanc ML, Legros B. Progressive expect to see on his brain MRI?
myoclonic epilepsy. Cerebellum. A. Skull
2004;3(3):156–71. B. Grey matter
C. White matter
?? 5. What anticonvulsant is likely to cause D. Venous sinuses
life-threatening side effects in the E. Basal ganglia
patient with mitochondrial disorders?
A. Oxcarbazepine vv Correct Answer is: C
B. Levitiracetam This a classic description of Krabbe disease
C. Clonazepam (leukodystrophy). Because he was not born
D. Valproic acid in the U.S., it explains the absence of the
E. Ethosuccimide newborn testing that might have made this
family aware of the diagnosis before the
vv Correct Answer is: D adoption.
Valproic acid that, in their metabolism,
generate reactive oxygen species can zz Suggested Reading
cause irreversible, and life-threatening 55 Kohlschütter A. Lysosomal leukodystro-
liver disease in patients with mitochon- phies: Krabbe disease and metachromatic
drial disorders and other disorders of leukodystrophy. Handb Clin Neurol.
redox regulation. 2013;113:1611–18.

zz Suggested Reading ?? 7. You are seeing a 4-year-old girl South


55 Nanau RM, Neuman MG. Adverse drug American for her progressive refusal
reactions induced by valproic acid. Clin to walk and absent deep tendon
Biochem. 2013;46:1323–38. reflexes. When you examined her,
55 Pronicka E, Weglewska-Jurkiewicz A, you noticed that she keeps her hips
Pronicki M, Sykut-Cegielska J, Kowalski P, and knees flexed, even when you
Pajdowska M, Jankowska I, Kotulska K, support her from both axillae to
Kalicinski P, Jakobkiewicz-Banecka J, hold her up in vertical suspension.
122 Chapter 4 · Congenital

When you try to put the soles of her Activity of which of the following
feet onto the examining table, she enzymes is most likely to be abnor-
screams as if she is in pain. Rubbing mal?
the bottoms of her feet with your A. Arylsulfatase
hands elicits the same response. B. Glucocerebrosidase
When you lay her flat on the table C. Hexosaminidase
and passively extend her legs, you D. Sialidase
notice they are bowed. Her family
4 states that her diet consists of juice vv Correct Answer is: B
and chicken nuggets.    Gaucher disease is the most common
You think you know what is wrong and lysosomal storage disease. It is inherited
test your hypothesis by ordering: as an autosomal recessive disorder due
A. An MRI of the brain to deficiency of lysosomal glucocerebro-
B. A serum lactates sidase (gene locus 1q21). Type I is a non-
C. Serum calcium, phosphate, alka- neuronopathic (visceral) Gaucher disease
line phosphatase, and vitamin D and neuronopathic glucocerebrosidase
D. A somatosensory evoked poten- deficiency was referred to as type II (acute
tial study. neuronopathic) or type III (subacute
neuronopathic). Meanwhile, it has been
vv Correct Answer is: C recognized that all forms represent a
Longstanding vitamin D deficiency in broad spectrum of manifestations rather
toddlers causes a painful neuropathy, than clearly distinguishable disorders.
bowing of the legs, a low serum calcium, Neuronopathic type is found in less than
a low serum phosphate, and a high 1:100,000 live births. So far, more than
serum alkaline phosphatase. The neu- 200 mutations have been described, but
ropathy may be evident on an EMG/NCV lack a genotype-phenotype correlation.
study, but this study will not allow deter- Patients with acute neuronopathic Gau-
mination of the etiology of the neuropa- cher disease exhibit severe neurological
thy. The other studies would be expected symptoms during the first year of life
to be unhelpful in this child. including stridor, strabismus due to palsy
of the ocular muscles, and dysphagia.
zz Suggested Reading Some patients show affection of the pyra-
55 Elder CJ, Bishop NJ. Rickets. Lancet. midal tract with spasm, retroflexion of the
2014;383:1665–76. head or opisthotonus. Acute neurono-
pathic Gaucher disease is usually fatal
?? 8. You are evaluating a 4-month-old within the first 2 years of life.
baby for muscle stiffness with hyper-
extension of his neck and back. He has zz Suggested Reading
had failure to thrive with swallowing 55 Staretz-Chacham O, Lang TC, LaMarca
difficulties. He might have a possible ME, Krasnewich D, Sidransky E. Lysosomal
seizure. He is extremely irritable. storage disorders in the newborn.
Examination is notable for an irritable, Pediatrics. 2009;123(4):1191–207.
cachexic infant with strabismus, bulbar 55 Hoffmann B, Mayatepek E. Neurological
dysfunction and hypertonia. There is manifestations in lysosomal storage
hepatosplenomegaly. A previous child disorders – from pathology to first
in the family had a similar presentation therapeutic possibilities. Neuropediatrics.
and died at age 3 years.   2005;36(5):285–9.
Congenital
123 4
?? 9. The girl shown in the picture has history C. Very long chain fatty acids
of hypotonia and global non-regressive D. 7 dehydrocholesterol
developmental delay with autistic fea- E. Buffy coat
tures. She has poor school performance.
She has cataracts on examination.    vv Correct Answer is: D
This child has Smith-Lemli-Opitz syn-
drome (SLOS). It is a complex malfor-
mation syndrome with prenatal and
postnatal growth retardation, micro-
cephaly, moderate to severe mental
retardation, and multiple major and minor
malformations. The facial features can
be subtle or severe. The malformations
include distinctive facial features, cleft
palate, cardiac defects, underdeveloped
external genitalia in males, and 2–3 syn-
dactyly of the toes. Many children present
due to developmental delay and autism.
The diagnosis is often missed. The defect
lies in the final step of cholesterol metabo-
..      Fig. 4.3 Smith-Lemli-Opitz_syndrome. 7 http:// lism. Testing for 7 dehydrocholesterol is
learn.genetics.utah.edu/content/disorders/ diagnostic as many milder patients have
whataregd/slos/index.html 7 http://emedicine.
normal plasma cholesterol levels. Testing
medscape.com/article/949125-overview 7 http://
en.wikipedia.org/wiki/Smith-Lemli-Opitz_syndrome of plasma cholesterol is not diagnostic.

zz Suggested Reading
55 Pickler L, Elias E. Genetic evaluation of
the child with an autism spectrum
disorder. Pediatr Ann. 2009;38(1):26–9.

?? 10. A pediatrician referred a 6-month-old


male because he has periods of irrita-
bility, stiffness and regression of skills
since he was 4 months of age. He has
normal birth and early development
history. He also has periods of elevated
temperature without evidence of
infection. Examination shows no dys-
morphic features. There are episodes
..      Fig. 4.4 My feet with limited partial syndactyly
of opisthotonus in response to tactile
(and some other ugly defects). (AzaToth. Own work.
Wikipedia) stimulus. Muscle tone is increased and
reflexes are absent. His CSF testing
showed elevated protein but no other
What is the best diagnostic test to
abnormalities.   
consider in this patient?
Which of the following laboratory
A. Mitochondrial point mutation and
studies would most likely be abnor-
deletion panel
mal in this patient?
B. Microarray analysis
124 Chapter 4 · Congenital

A. Phenylalanine hydroxylase Vogelstein B, editors. The metabolic and


B. 7-dehydrocholesterol molecular bases of inherited disease, 8th ed.
C. Acid maltase New York: McGraw-Hill; 2001.
D. Galactocerebrosidase
E. Beta hexosamindase A ?? 11. You were asked to see a full-term baby
boy in the NICU who was born after
vv Correct Answer is: D non-complicated labor but appeared
this is a presentation of a Krabbe disease, depressed at birth with generalized
4 which is characterized by an infantile- hypotonia and hypoventilation. He
onset progressive neurologic regressive started to have spells of clonic seizure
disorder with death before 2 years of age activity involving left and right side
in the majority of individuals. Alternatively, independently lasting 30–60 seconds
onset may be between age 6 months on several occasions. On your exami-
and the fifth decade with slower disease nation, he has micrognathia, broad
progression. Children with the infantile nasal bridge, and high forehead, doli-
form appear to be normal for the first few chocephalic skull with large anterior
months of life but show extreme irritabil- and posterior fontanelles. A firm liver
ity, spasticity, and developmental delay edge was noted 1 cm below the costal
before age 6 months; psychomotor regres- margin. On neurological exam, he has
sion progresses to a decerebrate state sever generalized hypotonia with poor
with no voluntary movement. In the late- gag and suck/swallow, absent reflexes
onset forms, individuals can be clinically and difficult to elicit Moro response.
normal until weakness, vision loss, and His EEG showed multifocal spikes on a
intellectual regression become evident. mildly low voltage background with-
The clinical course is variable even within out state change but no electrographic
the same family.Stage I is characterized seizures. CT scan showed dilated pos-
by irritability, stiffness, arrest of motor terior horns (colopocephaly).   
and mental development, and episodes What is the most likely diagnosis?
of temperature elevation without infec- A. Zellweger syndrome
tion, possibly caused by involvement of B. Prader-Willi syndrome
the hypothalamus. The child, apparently C. Myotonic dystrophy
normal for the first few months after birth, D. Spinal muscular atrophy
becomes hypersensitive to auditory, tac- E. Congenital CMV infection
tile, or visual stimuli and begins to cry fre-
quently without apparent cause.In almost vv Correct Answer is: A
all individuals with Krabbe disease, galac- the child has Zellweger’s syndrome. It is an
tocerebrosidase (GALC) enzyme activity is autosomal recessive disorder, is the sever-
deficient in leukocytes isolated from whole est form of the peroxisomal biogenesis
heparinized blood or in cultured skin fibro- disorders (PBD) and presents at birth with
blasts. dysmorphic features, liver cirrhosis, severe
hypotonia and neonatal encephalopathy
zz Suggested Reading with seizures. The diagnosis is based on
55 Wenger DA, Suzuki K, Suzuki Y, Suzuki the dysmorphic features and confirmed by
K. Galactosylceramide lipidosis. Globoid cell analysis of very long chain fatty acids (ele-
leukodystrophy (Krabbe disease). In: Scriver vated VLCFA C26:0, C26:1; abnormal ratios
CR, Beaudet AL, Sly WS, Valle D, Childs B, of C24/C22; C26/C22). Peroxisomes (which
Congenital
125 4
are missing in Zellweger’s) process VLCFA vv Correct Answer is: B
which, if they accumulate abnormally, Gaucher disease diagnosis is based on
affect CNS and other cell membranes. CNS the demonstration of deficient glucosyl-
dysmyelination occurs and in the severe ceramidase enzyme activity in peripheral
form leads to problems with radial glial blood leukocytes. Carrier testing by assay
guides and migration disorders (pachygy- of enzyme activity is unreliable because
ria, polymicrogyria, neuronal heterotopias, of overlap in enzyme activity between
colopocephaly) resulting in severe enceph- carriers and non-carriers. Identification
alopathy and seizures. Mutations in twelve of two disease-causing alleles in GBA, the
different PEX genes, those that encode only gene known to be associated with
peroxins, the proteins required for normal Gaucher disease, provides additional con-
peroxisome assembly, have been identi- firmation of the diagnosis.
fied in Zellweger’s syndrome. Treatment is
symptomatic and children fail to develop, zz Suggested Reading
with death usually in the first year. Other 55 Goker-Alpan O, Schiffmann R, Park JK,
PBD are neonatal adrenoleukodystrophy, Stubblefield BK, Tayebi N, Sidransky
infantile Refsum disease (both considered E. Phenotypic continuum in neurono-
part of the Zellweger’s Syndrome Spec- pathic Gaucher disease: an intermedi-
trum) and rhizomelic chondrodysplasia ate phenotype between type 2 and
punctata. These disorders can be separated type 3. J Pediatr. 2003;143(2):
clinically and 80% of PBD are among the 273–6.
Zellweger’s Syndrome Spectrum with an
incidence of 1 in 50,000 births in the US. ?? 13. You are evaluating an 8-year-old boy
for his developmental delay. His medi-
zz Suggested Reading cal history is remarkable for frequent
55 Steinberg SJ, Raymond GV, Braverman ear infections and a heart murmur due
NE, Moser AM, and Moser to valvular disease. On examination,
HW. Peroxisome biogenesis disorders, he has short stature, lordotic posture,
Zellweger syndrome spectrum. Gene Rev. slightly coarse facial features, and cor-
7 http://www.ncbi.nlm.nih.gov/bookshelf/ neal clouding.   
br.fcgi?book=gene&part=pbd. What laboratory test would most
likely define his condition?
?? 12. You are evaluating a 5-month-old A. Amino acid analysis
baby who was normal at birth, but has B. 7-dehydrocholesterol level
devolved regression with progressive C. Lysosomal enzyme screen
opisthotonus, stridor, and swallowing D. CSF neurotransmitter levels
difficulties. On examination, he has E. Uric acid level
enlarged spleen, squint, head retro-
flexion, spasticity, and trismus.    vv Correct Answer is: C
What tissue analysis would most the child has Sanfilippo syndrome. It is one of
likely define his condition? the mucopolysaccharidoses (MPSs) disorders.
A. Bone marrow It is inherited lysosomal storage disorders
B. Blood leukocytes caused by the deficiency or absence of
C. Muscle specific lysosomal enzymes. The absence of
D. Skin these enzymes leads to accumulation of com-
E. Liver plex carbohydrates in cells and tissues and in
126 Chapter 4 · Congenital

the cellular organelles, the lysosomes. These On examination she is obtunded. Her
complex carbohydrates are also known as EEG shows slowing and tri-phasic
mucopolysaccharides or glycosaminoglycans waves.   
(GAGs) serve as the building blocks for con- What is your next step in evaluation?
nective tissues. A. Ammonia
MPS III, or Sanfilippo syndrome, is the B. CSF exam
most common of MPS disorders and results C. Plasma lactate
from the deficiency or absence of 4 different D. MRI of the brain with MRS
4 enzymes that are necessary to degrade the E. CT brain scan
GAG heparan sulfate. Each enzyme
deficiency defines a different form of vv Correct Answer is: A
Sanfilippo syndrome. Individuals with type this is a clinical presentation of ornithine
A lack the enzyme heparan sulfate sulfatase. transcarbamylase deficiency. Although classi-
Individuals with type B lack the enzyme N cally associated with presentation in infancy,
-acetyl-alpha-D-glucosaminidase. Patients the majority of individuals with urea cycle
with type C lack acetyl-CoA:alpha-glucosa- disorders (UCDs) present outside the neonatal
minide acetyltransferase. Patients with type period, frequently in childhood. Signs and
D lack the enzyme N- acetylglucosamine- symptoms are unclear but recurrent; fulmi-
6-sulfatase. The particular form of Sanfilippo nant presentations associated with acute
syndrome cannot be determined based on illness are also common. A disorder of urea
clinical features, but rather must rely on cycle metabolism should be considered in
enzymatic assays. The neurological children who have recurrent symptoms, espe-
manifestations include mental regression cially neurologic abnormalities associated
beginning in the toddler years, coarse facial with periods of decompensation. Routine
appearance, but may be mild, sleep laboratory tests, including measurement of
disturbance and progressive myelopathy plasma ammonia concentrations, can indicate
due to storage material in the spine. a potential UCD; however, specific metabolic
Seizures may occur. Initial clinical signs pres- testing and ultimately enzymatic or molecular
ent as a change in behavior. Patients are confirmation are necessary to establish a
hyperactive and display aggressive and diagnosis. Treatment with dietary protein
destructive behaviors. restriction and medications may be challeng-
ing in children.
zz Suggested Reading
55 Valstar MJ, Ruijter GJ, van Diggelen OP, zz Suggested Reading
Poorthuis BJ, Wijburg FA. Sanfilippo 55 Smith W, Kishnani PS, Lee B, Singh RH,
syndrome: a mini-review. J Inherit Metab Rhead WJ, Sniderman King L, Smith M,
Dis. 20. Summar M. Urea cycle disorders: clinical
presentation outside the newborn period.
?? 14. A normal healthy 9-year-old female Crit Care Clin. 2005;21(4 Suppl):S9–17.
is brought to the ED with vomiting
and lethargy after a birthday party. ?? 15. You are evaluating an 8-year-old boy with
She has unremarkable past medi- a history of a progressive gait disturbance
cal history. Her early developmental and abnormal involuntary movements
milestones were on target and she is for 1 year. He has unremarkable past
an average student in the 3nd grade. medical history. On examination, he has
Family history is remarkable for a remarkable dystonia and equinovarous
maternal cousin who died in early deformity in his feet, rigidity in his legs,
infancy from an unspecified condition. and chorea. His brain MRI is shown.   
Congenital
127 4
Approximately 50% of the patients have
identifiable mutations in the PANK2 gene.
All individuals with NBIA who are found
to have the ‘eye-of-the-tiger’ sign on
T2-weighted magnetic resonance imaging
have been found to have at least one muta-
tion in PANK2.

zz Suggested Reading
55 Gregory A, Polster BJ, Hayflick SJ. Clinical
and genetic delineation of neurodegenera-
tion with brain iron accumulation. J Med
Genet. 2009;46(2):73.

?? 16. You are seeing a 7-month-old girl with


a history of a seizure described as star-
ing followed by a generalized convul-
sion lasting 5 min. Her past medical
history is remarkable of hypotonia and
developmental delay, is not able to roll
..      Fig. 4.5 MRI image shows iron deposits in the over and does not reach for objects
basal ganglia, the so-called eye-of-the-tiger sign with her hands. On examination, her
(T2w GRASE sequence). (Enro2002. Wikipedia) head circumference is above 95th per-
centile, while her weight and length are
Which of the following metals can at 25th percentile. She has optic atro-
case this metabolic disorder? phy and nystagmus, central hypotonia
A. Copper and increased tone in the ankles, with
B. Iron clonus. Her brain MRI shows increased
C. Mercury signal in the white matter diffusely on
D. Lead T2-weighted images. Her MRS shows a
E. Zinc strikingly elevated NAA peak.
What is the most likely diagnosis?
vv Correct Answer is: B A. Krabbe leukodystrophy
This is a presentation of Pantothenate B. Adrenoleukodystrophy
kinase-associated neurodegeneration C. Mitochondrial encephalomyopathy
(PKAN). It is a form of neurodegeneration D. Canavan disease
with brain iron accumulation (NBIA for- E. Metachromatic leukodystrophy
merly called Hallervorden-Spatz syndrome)
characterized by progressive dystonia and vv Correct Answer is: D
basal ganglia iron deposition with onset Canavan disease diagnosis can be made
that usually occurs before age 10 years. based on the triad of hypotonia, macro-
The most clinical manifestations include cephaly, and head lag in an infant after
dysarthria, rigidity, and pigmentary reti- the age of 3–5 months. Diagnosis of
nopathy. About 25% of affected individuals Canavan disease in symptomatic individu-
have an ‘atypical’ presentation with later als relies upon demonstration of very
onset (after age 10 years), prominent high concentration of N-acetyl aspartic
speech defects, psychiatric disturbances, acid (NAA) in the urine. NAA concentra-
and more gradual progression of disease. tion is also elevated in the blood and
128 Chapter 4 · Congenital

cerebrospinal fluid (CSF) of children clumsiness, frequent falls, toe walk-


with Canavan disease. In the brain, the ing, and slurred speech. Later signs
elevated NAA can be visualized by 1H include inability to stand, difficulty with
MRS as a single peak with a resonance of speech, deterioration of mental function,
2.01 ppm on the spectrum. increased muscle tone, pain in the arms
and legs, generalized or partial seizures,
zz Suggested Reading compromised vision and hearing, and
55 Janson CG, McPhee SW, Francis J, Shera peripheral neuropathy. In the final stages
4 D, Assadi M, Freese A, Hurh P, Haselgrove children have tonic spasms, decerebrate
J, Wang DJ, Bilaniuk L, Leone P. Natural posturing, and general unawareness of
history of Canavan disease revealed by their surroundings. Unexplained fevers
proton magnetic resonance spectroscopy and elevated CSF protein may be seen.
(1H-MRS) and diffusion-weighted
MRI. Neuropediatrics. 2006;37(4): zz Suggested Reading
209–21. 55 Kaye EM. Update on genetic disorders
affecting white matter. Reference: Pediatr
?? 17. A 14-year-old female with 1-year his- Neurol. 2001;24(1):11–24.
tory of psychiatric symptoms including
inattention, impulsivity, and anger out- ?? 18. Which of the following is correct
bursts came to your office for evalua- regarding sialidosis?
tion of progressive deterioration in her A. Sialidosis type I has infantile onset
gait. She has unremarkable past medi- B. Sialidosis type II has adult onset
cal history and noncontributory family C. A cherry-red spot is rarely seen in
history. Her examination demonstrates this condition
pale optic nerves, low amplitude deep D. Coarse facial features are almost
tendon reflexes, increased tone, and never seen in sialidosis type II
extensor plantar responses. Her brain E. These patients have myoclonic
MRI is shown. epilepsy
What is your likely diagnosis?
A. Adrenoleukodystrophy vv Correct Answer is: E
B. Krabbe’s disease Sialidosis is one of glycoproteinoses
C. Metachromatic leukodystrophy group, and myoclonic epilepsy is com-
D. Alexander’s disease mon in these patients. Glycoproteinoses
E. Canavan’s disease are a group of lysosomal storage disor-
ders of autosomal recessive inheritance,
vv Correct Answer is: C in which the enzymatic defect leads to
This is a case of Metachromatic leukodys- accumulation of oligosaccharides, glyco-
trophy (MLD). It is due to Arylsulfatase peptides, and glycolipids. Accumulation
A deficiency. There are three clinical in the brain and viscera leads to vacu-
subtypes: a late-infantile MLD, a juve- olization of multiple cell types. There
nile MLD, and an adult onset form of are multiple phenotypes depending on
MLD. The disease course may be from the enzyme affected. In general, these
three to ten or more years in the late- patients have coarse facial features,
infantile form and up to 20 years or skeletal abnormalities, and psychomo-
more in the juvenile and adult forms. In tor retardation. Sialidosis is caused by
the late infantile type, onset is between deficiency of lysosomal A-neuraminidase
1–2 years of age. Typical presenting (sialidase), leading to increased urinary
findings include weakness, hypotonia,
Congenital
129 4

a b c

d e f

g h i

j k l

m n o

..      Fig. 4.6 Liaw, Hsiang-Ru, et al. “Late infantile meta- Liaw, HF Lee, CS Chi. This article is distributed under
chromatic leukodystrophy: Clinical manifestations of the terms of the Creative Commons Attribution 4.0
five Taiwanese patients and Genetic features in Asia.” International License. (a–c) Patient #1, (d–f) Patient #2,
Orphanet journal of rare diseases 10.1 (2015): 1. HR (g–i) Patient #3, (j–l) Patient # 4, (m–o) Patient # 5
130 Chapter 4 · Congenital

excretion of sialic acid-containing oligo-


saccharides.
Type I sialidosis (cherry red spot
myoclonus syndrome) has its onset in
adolescence to adulthood and manifests
with myoclonic epilepsy, visual deteriora-
tion, and cherry-red spots without
dysmorphism.
4 Type II is the childhood form, and these
patients have not only myoclonic epilepsy
and cherry-red spots in the retina but also
severe neurologic abnormalities, coarse
facial features, severe dysostosis, and
psychomotor retardation.
The neonatal form is characterized by
hydrops fetalis, nephrotic syndrome, and
early death.
Other glycoproteinoses include
A-­mannosidosis caused by A-mannosidosi-
dase deficiency, B-mannosidosis caused by
B-mannosidosidase deficiency, fucosidosis
caused by fucosidase deficiency, aspartyl- ..      Fig. 4.7 Hemimegalencephaly From 7 ILAE.org,
glucosaminuri a caused by aspartylglucos- 7 EpilepsyDiagnosis.org, Diagnostic Manual
aminidase deficiency, and Schindler
disease caused by A-N acetylgalactosamin- B. Macrocephaly may be due to hydro-
idase deficiency. cephalus or megalencephalyc.
Abnormal urinary excretion of C. Hemimegalencephaly rarely leads
oligosaccharides and glycopeptides, as to motor manifestations
well as vacuolated lymphocytes with D. Megalencephaly is an oversized
membrane-bound vacuoles, can be seen. brain, with brain weight greater
Definitive diagnosis is made with analysis than 2 standard deviations above
of enzyme activity. the mean
E. Megalencephaly may be a benign
zz Suggested Reading finding in some cases
55 Comprehensive review in clinical neurol-
ogy: a multiple-choice question book for vv Correct Answer is: C
the wards and boards © 2011 This is a radiological presentation of
Wolters Kluwer Health left hemispheric hemimegalencephaly,
Lippincott Williams & Wilkins. All rights which frequently leads to contralateral
reserved. hemiparesis. Microcephaly is defined as
head circumference less than 2 standard
?? 19. Which of the following statements deviations below the mean. A variety of
is not true regarding the abnormal- causes of microcephaly exist. The heredi-
ity shown in the Figure? Coronal tary form could be a normal variant, and
T2-weighted MRI has no clinical implications. In utero infec-
A. Hemimegalencephaly frequently tions, toxin exposure (such as alcohol,
presents with seizures tobacco, and prescription drugs such as
Congenital
131 4
chemotherapeutic agents and antiepilep- Kluwer Health Lippincott Williams &
tic), hypoxic-ischemic injury, birth trauma, Wilkins. All rights reserved.
and metabolic disorders such as prolonged
hypoglycemia are the pathological causes ?? 20. You are seeing a 6-month-old baby
for the microcephaly. A wide variety of boy for his seizures. The family recently
hereditary disorders including enzyme emigrated from Colombia, and he
deficiencies can lead to microcephaly, as did not undergo newborn screening
can chromosomal abnormalities. Clinical at birth. He has significant develop-
manifestations depend on the underlying mental delay, hypotonia, has failure to
cause and can range from none (asymp- thrive, and is microcephalic. His urine
tomatic microcephaly) to severe develop- has a musty odor.
mental delay and seizures. Which of the following is not true
Macrocephaly is defined as a head regarding this condition?
circumference greater than 2 standard A. Phenylalanine transaminase defi-
deviations above the mean. Macrocephaly ciency is the cause of this condition
may be a normal variant that is often B. Tetrahydrobi opterin is a cofactor
hereditary, may result from increased CSF, in the conversion of phenylala-
as in hydrocephalus due to a variety of nine to tyrosine
causes, from mass lesions such as tumors C. Dietary restriction of phenylalanine
or subdural hematomas, or may result from is the treatment for this condition
megalencephaly, an oversized brain (brain D. Cognitive impairment and sei-
weight greater than 2 standard deviations zures are common
above the mean). E. The musty odor in the urine and
Causes of megalencephaly include stor- sweat is caused by phenylacetic acid
age diseases such as mucopolysaccharidoses
or Tay-Sachs disease, Canavan disease, Alex- vv Correct Answer is: A
ander disease, genetic disorders including This is a clinical presentation of phenylke-
Sotos syndrome, and others. In the latter dis- tonuria (PKU). It is a disorder of phenylala-
orders, eventual cell loss with subsequent nine metabolism, caused by a deficiency of
atrophy typically occurs. Megalencephaly phenylalanine hydroxylase. This enzyme
may also be a benign familial finding. Clinical converts phenylalanine to tyrosine, and
manifestations depend on the underlying its deficiency leads to accumulation of
cause. Neurologic deficits may be absent in phenylalanine, which is then metabolized
the benign familial form. by phenylalanine transaminase to phenyl-
Hemimegalencephaly, or enlargement pyruvic acid, which is subsequently oxidized
of only one brain hemisphere, invariably to phenylacetic acid, responsible for the
presents with seizures and hemiparesis. musty odor of the sweat and urine of these
Hemimegalencephaly is associated with patients. It is autosomal recessive disorder.
various genetic disorders, including Children with PKU are normal at birth, with
Beckwith-Wiedemann syndrome, which is a rise in the phenylalanine levels after initia-
characterized by gigantism, macroglossia, tion of feeding. These children will have
and midline abdominal wall defects. developmental delay, cognitive impairment,
microcephaly, seizures, hypotonia, and
zz Suggested Reading severe behavioral disturbances. A musty
55 Comprehensive review in clinical neurol- odor, as described, is characteristic. These
ogy: a multiple-choice question book for children are fair, with blond hair, blue eyes,
the wards and boards ©2011 Wolters and pale skin given the lack of tyrosine and
132 Chapter 4 · Congenital

melanin pigment production. Newborn demyelination occurs in peripheral nerves


screening detects hyperphenylalaninemia, and cell death in central oligodendroglia
and the diagnosis is made on the basis of cells. Sparing of the subcortical U-fibers
elevation of phenylalanine levels in blood. on MRI is found in other leukodystrophies
Tetrahydrobiopterin is a cofactor for phenyl- (x-linked adrenoleukodystrophy andgloboid
alanine hydroxylase, and its deficiency may cell leukodystrophy) as well in the inborn
also produce hyperphenylalaninemia (and errors of metabolism disorders, such as
PKU). A reduction of phenylalanine levels in phenylketonuria, maple syrup urine disease,
4 blood and urine after a trial of tetrahydrobi- Lowe’s disease and hypohomocysteinemia.
opterin is used to make the diagnosis of this Krabbe disease, or globoid cell leuko-
condition. The treatment of PKU is dietary dystrophy, is caused by a deficiency of
restriction of phenylalanine, and these galactosylceramide beta-galactosidase and
patients should be placed on a low-protein symptoms begin between 2–6 years with
diet and phenylalanine-free feeding formula mental regression, cortical blindness and
as soon as possible after birth, which will spasticity.
prevent neurologic complication. Untreated Adrenomyeloneuropathy is the most
patients have severe intellectual disability. common phenotype of adrenoleukodys-
Despite treatment, some patients still have trophy and is caused by an inability to
varying degrees of mild to moderate cogni- oxidize very-long chain fatty acids.
tive delays. Tetrahydrobiopterin is used as a Paraplegia begins after the age of 20 and
treatment adjunct in select patients. slowly progresses throughout life.
Limb-Girdle myasthenia is an immune-
?? 21. You are evaluating a 6-year-old boy mediated myasthenia gravis, with an age
who reached his normal motor and of onset after 10 years, and presents with
cognitive development by 1 year of progressive proximal weakness of the
age, but at 16 months the child began limbs.
to have frequent falls and looked
“unsteady” when walking. He was seen zz Suggested Reading
by another neurologist at 2 years of age 55 Von Figura K., Gieselmann V, Jaecken
and he documented, ataxia, distal leg J. Metachromatic leukodystrophy:
weakness and absent tendon reflexes. Lysosomal disorders. In: the metabolic and
His brain MRI showed subcortical molecular bases of inherited disease 2011.
demyelination with a posterior pre- p. 3695–724.
dominance and sparing of the subcorti-
cal U-fibers. Since then he experienced ?? 22. You are seeing a 5- years old with
significant cognitive decline, optic atro- severe peripheral motor and sensory
phy and progressive spasticity.   neuropathy, has tightly curled hair
What is the most likely diagnosis? (unlike either parent) and cognitive
A. Krabbe disease impairment with a sural nerve biopsy
B. Metachromatic leukodystrophy remarkable for enlarged axons filled
C. Adrenomyeloneuropathy with disrupted neurofilaments.   
D. Limb-Girdle myasthenia What is the most likely diagnosis?
vv Correct Answer is: B A. Infantile neuroaxonal dystrophy
This is a presentation of the late infantile B. Late infantile neuronal ceroid lipo-
form of metachromatic leukodystrophy, fuscinosis
which is an inherited disorder of myelin C. Giant axonal neuropathy
metabolism and is caused by a deficiency D. Sub-acute necrotizing encephalo-
of the enzyme arylsulfatase A. Segmental myelopathy
Congenital
133 4
vv Correct Answer is: C motor units. On your examination she
In giant cell neuropathy, the underlying had severe weakness, muscle atrophy
pathology is disruption of intermediate and exaggerated tendon reflexes. Cranial
filament organization that predominantly nerves were intact, except for atrophy
affects the neurofilaments. Usually the and fasciculation of the tongue.   
first symptoms to be seen before 5 years of Which of the following tests could
age. Both central and peripheral axons are confirm the diagnosis?
disrupted and manifest impaired function. A. Deficiency in hexosaminidase-A
Boys and girls are equally affected. Sym- activity in cultured leukocytes
metrical distal atrophy is universal with B. Reduced erythrocyte galactose-
impairment of vibratory sensation and 1-phosphate uridyltransferase
proprioception with diminished tendon (GALT)
reflexes. C. Elevated dermatan sulfate and
The neuropathology of infantile heparin sulfate
neuroaxonal dystrophy is characterized by D. Muscle biopsy evidence of fiber
the presence of large axonal eosinophilic necrosis and regeneration
spheroids, forming as a result of axonal
swelling, in the cerebral and anterior horn vv Correct Answer is: A
cell grey matter. This is a presentation of GM2 ganglio-
Symptoms of late infantile neuronal sidosis (Tay-Sachs disease), which may
ceroid lipofuscinosis begin between 2 and present with a variety of phenotypes. The
4 years of age and include progressive initial features can be similar to spinal
cognitive impairment and abnormal muscular atrophy. Wasting, weakness and
movements. Visual impairment begins by cramps of proximal leg muscles are initial
4 years of age and progresses to blindness. symptoms followed by distal leg weak-
Subacute necrotizing encephalomy- ness along with proximal and distal arm
elopathy (Leigh disease), present between weakness. As the disease progresses there
3 and12 months of age with neurological are both upper and lower motor neuron
decompensation associated with lactic symptoms. Some children develop cere-
acidosis. bral symptoms.
Galactosemia is a transference defi-
zz Suggested Reading ciency that presents with early signs of
55 Kuhlenbaumer G, et al. Giant axonal cataracts. Physical symptoms are provoked
neuropathy. Gene Rev. October 9, 2014. by the first feeding after birth including
7 http://www.ncbi.nlm.nih.gov/books/ vomiting, diarrhea and jaundice. Even with
NBK1136/ (7 http://www.ncbi.nlm.nih.gov/ diet modifications, developmental delays
books/NBK11361). and truncal ataxia develops.
Elevated dermatan sulfate and heparin
?? 23. You were asked to see a 2-year-old girl for sulfate are present in mucopolysacchari-
developmental delay evaluation. Her par- doses, including Hunter Syndrome.
ents reported that at 12 month of age the Cognitive regression occurs as a result of
child was no longer able to stand with progressive neuronal storage of gangliosi-
assistance. She has muscle wasting and dosis after 2 years of age.
weakness at her 18 months regular exam. In this age group, fiber necrosis with
Serum CK levels and an MRI of the brain signs of regeneration is most consistent
were normal. She has a normal motor with children born with congenital muscular
and sensory nerve conduction velocities dystrophy. They display hypotonia, poor
1 week ago. EMG showed neuropathic suck and respiratory distress at birth.
134 Chapter 4 · Congenital

zz Suggested Reading 55 Adult onset adrenoleukodystrophy: This


55 Scriver CR. Physician’s guide to the phenotype consists of a rapidly progres-
laboratory diagnosis of metabolic diseases. sive dementia, psychiatric changes (e.g.
In: Blau N, Duran M, Blaskovics ME, psychosis, affective disorders), CNS
Gibson KM, editors. Springer Science & demyelination, endocrine dysfunction, and
Business Media; 2012. possible myelopathy or neuropathy.
55 Adrenomyeloneuropathy: The age of onset
?? 24. Mutations of the ALD (ABCD1) gene is typically in early adulthood with a
4 are most likely to cause: progressive spastic paraparesis, bowel and
A. Frontal white matter abnormali- bladder incontinence, sexual dysfunction,
ties mild dementia, and psychiatric changes (i.e.
B. CNS and renal neoplasms affective disorders), as well as sensorimotor,
C. A deficiency of very long chain primarily axonal neuropathy. This pheno-
fatty acids type may be either slowly or rapidly
D. Cerebral inflammatory demyelin- progressive.
ation 55 An Addison’s only phenotype consists of
E. Infantile hypotonia isolated adrenal dysfunction.
55 Heterozygous females typically develop
vv Correct Answer is: D symptoms by age 60. The disorder is slowly
Mutations of the ALO (ABCD1) gene on the progressive, and may present with periph-
X chromosome causes adrenoleukodystro- eral neuropathy, fecal incontinence, and pain
phy. The gene encodes a peroxisomal pro- in the legs. White matter lesions and
tein (ALDP) involved very long chain fatty endocrine dysfunction are rare.
acid (VLCFA) oxidative degradation. Muta-
tions lead to an accumulation of VLCFAs, Treatment for adrenoleukodystrophy
and adrenal failure with a deficiency of involves glucocorticoids and bone marrow
androgens and glucocorticoids. This results transplant. Dietary fatty acid supplemen-
in perivascular lymphocytes, and demyelin- tation (“Lorenzo’s oil”), has been used, but
ation, beginning in the parietal and occipital with limited data to suggest efficacy.
lobes and sparing arcuate fibers. MRI dem-
onstrates T2-weighted hyper-intensities of zz Suggested Reading
the posterior white matter. Several clinical 55 Engelen M, et al. X-linked adrenoleuko-
phenotypes may result, including child- dystrophy (X-ALD): clinical presentation
hood onset, adolescent onset, adult onset, and guidelines for diagnosis, follow-up
adrenomyeloneuropathy, Addison’s disease and management. Orphanet j Rare Dis.
only, and women with symptomatic X-ALD. 2012;7:51.
55 Childhood onset adrenoleukodystrophy: 55 Rai PVS, et al. Childhood adrenoleukodys-
The age of onset ranges from 2.5–10 years. trophy – classic and variant – review of
Symptoms include extensive CNS demy- clinical manifestations and magnetic
elination, hyperactivity, dementia, ataxia, resonance imaging. J Pediatr Neurosci.
vision loss, hearing deficits, and seizures, 2013;8(3):192–7.
with endocrine dysfunction. The course is
rapidly progressive. ?? 25. On which chromosome is the respon-
55 Adolescent onset adrenoleukodystrophy: sible gene located in a 7-year-old boy
Manifestations start between ages with a previous diagnosis of develop-
10–21 years, and are rapidly progressive, with mental delay and cerebral palsy that
CNS demyelination, behavioral and cognitive now presents with serious biting of his
changes, and endocrine dysfunction. fingers, lips and cheeks?
Congenital
135 4
A. 3 Which of the following is not correct
B. 21 regarding this condition?
C. X A. The genetic defect is in the gene
D. Unknown etiology HPRT1
B. It is autosomal recessive
vv Correct Answer is: C C. Hypoxanthine guanine phospho-
The hallmark sign of Lesch-Nyhan disease ribosyltransferase is the deficient
is self-mutilation. This is a disorder of enzyme
hypoxanthine guanine phosphoribsyl- D. It is caused by an enzymatic
transferase deficiency mapped to the defect in the purine salvage path-
gene locus Xq26. Uric acid in the blood way
and urine is elevated. E. Patients may have choreoath-
Patients usually appear normal at birth, etotic movements
then develop hypertonia and delayed
motor milestone attainment. Progressive vv Correct Answer is: B
spasticity frequently leads to an initial This is a case of Lesch-Nyhan syndrome
diagnosis of cerebral palsy. Occasionally (LNS), which is inherited in an X-linked
aggressive behavior towards caretakers is fashion and is caused by deficiency of the
also present. enzyme hypoxanthine guanine phospho-
Syndromes associated with chromo- ribosyltransferase (HGPRT), which partici-
some 3 include van Hippel-Landau, pates in the salvage pathway of purines.
3-methylcrotonyl-CoA carboxylase HGPRT is encoded by the gene HPRT1
deficiency and Charcot-Marie-Tooth on chromosome Xq26, and deficiency of
disease. this enzyme leads to the accumulation of
The most common childhood disorder purines with their subsequent conversion
associated with chromosome 21 is Down’s to uric acid.
syndrome. There is a wide presentation of the
Cerebral palsy can be a manifestation disease, ranging from the classic LNS form
of many disorders and often a specific to milder forms without neurologic
etiology is not known. manifestations. The classic LNS form may
manifest in the newborn period with
zz Suggested Reading severe hypotonia. These children will have
55 Torres RJ, Puig JG, Jinnah HA. Update delayed motor development, progressive
on the phenotypic spectrum of Lesch- limb and neck rigidity with dystonia,
Nyhan disease and its attenuated vari- choreoathetotic movements, facial
ants. Curr Rheumatol Rep. grimacing, seizures, spasticity, and
2012;14(2):189–94. intellectual disability. These patients have
aggressive and severe self- mutilation
?? 26. You are seeing a 3-year-old boy for his behavior.
follow-up visit. He has a history of devel- Milder forms include those with less
opmental delay and intellectual disabil- severe neurologic disease, and those in
ity since early in life and has developed which there are no neurologic manifesta-
torticollis and spasticity in his limbs. He tions, presenting with hyperuricemia,
also has a history of seizures and aggres- gout, and nephrolithiasis. The clini-
sive behavior. He began biting himself cal ­picture, including neurologic
to the point of bleeding and constantly ­manifestations, self-mutilation behavior,
self-inflicts injuries. He has had kidney and hyperuricemia suggests the
stones and hyperuricemia.    ­diagnosis.
136 Chapter 4 · Congenital

HGPRT activity can be assessed in recessive except for Hunter disease,


fibroblasts, and genetic testing can also which is X-linked. In general, MPS are
be used. a group of progressive multi-systemic
Treatment includes purine restricted disorders that affect the cornea, carti-
diet, hydration to prevent kidney stones, lage, bone, connective tissue, reticulo-
allopurinol to decrease the production of endothelial system, and nervous system.
uric acid, and supportive care to prevent Hurler syndrome is MPS type I and is
self-inflicted injuries and control abnor- caused by a-L-iduronidase deficiency.
4 mal movements. Levodopa and tetrabena- The gene is localized to chromosome
zine have been tried for the 4, and there is accumulation of both
neuropsychiatric manifestations. dermatan and heparan sulfate. Children
are normal at birth, but within the first
?? 27. You are evaluating a 6 -year-old boy 2 years of life, they will develop coarsen-
in your office. On examination, he ing of facial features, with progressive
has coarse facial features, short stat- skeletal dysplasia with dysostosis multi-
ure with dysmorphic appearance, plex and growth impairment (dwarfism).
prominent psychomotor retardation, Those children have restricted range
hepatosplenomegaly, and several of motion of the joints, hearing loss,
nodular lesions that are ivory colored corneal clouding, macroglossia, hernias,
on his upper back. The activity of idu- visceromegaly, valvular heart disease,
ronate sulfatase in leukocytes is almost and prominent intellectual disability.
absent.    Mild MPS type I is also known as Scheie
Which of the following is correct syndrome, and the intermediate form is
regarding this condition? known as Hurler-Scheie syndrome. Most
A. It is inherited in an X-linked fashion recently, MPS type I has been classified
B. Corneal clouding is a prominent as severe MPS type I and attenuated
feature in this syndrome MPS type I. The diagnosis of MPS type I
C. He has Hurler syndrome is based on elevated urinary excretion
D. Heparan sulfate, but not derma- of dermatan and heparan sulfate and
tan sulfate, will be elevated in the confirmed with enzyme analysis in leu-
urine kocytes and fibroblasts. Pathologically,
E. This patient has mucopolysac- there are cells with vacuolated appear-
charidoses type ance, expansion of perivascular spaces in
the CNS, and neuronal lipidosis. Electron
vv Correct Answer is: A microscopy demonstrates reticulogranu-
These are the manifestations of Hunter lar material in epithelial and mesen-
syndrome. It is inherited as an X-linked chymal cells, and lamellar material in
disorder. Patients with Morquio syn- neurons, some of which adopt a layered
drome have no intellectual disability. appearance and are called zebra bodies.
Mucopolysaccharidoses (MPS) are due Enzyme replacement therapy can be
to impaired lysosomal degradation of used to treat non-central nervous sys-
glucosaminoglycans, which are long tem manifestations of the disease. Stem
unbranched molecules of repeating cell transplantation can be potentially
disaccharides. Many enzymatic defects helpful.
lead to the accumulation of glucosami- Hunter syndrome or MPS type II is
noglycans in lysosomes and the extra- due to a defect in iduronate sulfatase,
cellular matrix. All MPS are autosomal with accumulation of dermatan sulfate
Congenital
137 4
and heparan sulfate. These patients have of these glycans. Clinical features are
the Hurler phenotype but lack the widely variable, with multiple subtypes,
corneal clouding and have characteristic including combinations of failure to thrive,
nodular ivory-colored lesions on the developmental delay, dysmorphic features,
back, shoulders, and upper arms. These hypotonia, ataxia, and weakness, and
children have short stature, macroceph- retinitis pigmentosa, short stature with
aly, macroglossia, hoarse voice, hearing skeletal abnormalities, polyneuropathy,
loss, visceromegaly, dysostosis multi- and stroke- like episodes, liver dysfunction,
plex, joint contractures, entrapment hypogonadism, and multisystem involve-
neuropathies such as carpal tunnel ment.
syndrome, and intellectual disability. Lipodystrophy with prominent fat
The diagnosis is based on mucopolysac- pads in the buttocks and suprapubic area
chariduria and confirmed with analysis and inverted nipples are distinctive
of the enzyme activity. Treatment is features. Type II CDGs are caused by
symptomatic. abnormal processing and modification of
glycans and lead to profound intellectual
?? 28. You are seeing a 4 years old boy for his disability but no cerebellar ataxia or
developmental delay, hypotonia, and peripheral neuropathy.
dysmorphic features. On examination, CDGs are characterized by the
he has inverted nipples and prominent presence of a carbohydrate-deficient
fat pads in the buttocks area. A diag- transferrin in the serum and CSF, and the
nosis of a congenital disorder of glyco- analysis of the glycoforms of this protein
sylation is suspected.    is a diagnostic test. Genetic confirmation
Which of the following is not true is available. Treatment is supportive.
regarding this condition?
A. It affects multiple organ systems zz Suggested Reading
B. Analysis of transferrin glycoforms 55 Comprehensive review in clinical neurol-
can help make the diagnosis ogy: a multiple-choice question book for
C. Patients can present with stroke- the wards and boards © 2011 Wolters
like episodes Kluwer Health Lippincott Williams &
D. Some patients may have hypogo- Wilkins. All rights reserved.
nadism
E. It is inherited in an X-linked fashion ?? 29. You are seeing a 5-year-old trisomy 21
girl for her annual checkup.   
vv Correct Answer is: E Which of the following statements is
Congenital disorders of glycosylation not true regarding this disorder?
(CDG), previously known as carbohydrate- A. Intellectual disability of varying
deficient glycoprotein syndrome, are a degrees of severity may be present
group of genetic disorders inherited as B. Patients with trisomy 21 are at
an autosomal recessive disorder. It is a increased risk of leukemia
multiple- systems disorder, especially the C. MRI of the brain would show
CNS. The defect is abnormal synthesis, hypertrophy of the frontal lobes,
transport, modification, and/or processing with macrocephaly
of the carbohydrate moieties or glycans D. There is increased risk of atlanto-
of glycoproteins, therefore affecting pro- axial dislocation
tein components in many tissues. Type I E. Early dementia with Alzheimer
CDGs are caused by abnormal synthesis type pathology is seen
138 Chapter 4 · Congenital

vv Correct Answer is: C Trisomy 13 or Patau syndrome is


MRI findings in Down syndrome, or tri- characterized by microcephaly, microph-
somy 21 are, the frontal lobes are small thalmia, iris coloboma, low-set ears, cleft
and under developed, and the superior lip and palate, polydactyly (excess number
temporal gyri are small and thin. of fingers), prominence of the heels, and
Developmental delay is defined by cardiac abnormalities. Life expectancy is
performance on standardized tests of typically not beyond early childhood.
function, that is, 2 standard deviations Trisomy 18 is characterized by
4 below the mean. Intellectual disability is a microcephaly, ptosis, overlapping of the
diagnosis made on the basis of testing third finger over the second finger,
intelligence quotient (IQ) and is character- rocker-bottom feet, umbilical hernia,
ized as mild, moderate, or severe on the congenital heart disease, and other
basis of the IQ, the degree of impairment, findings. Life expectancy is typically not
and the level of assistance in daily beyond early infancy.
activities and other activities that are In Klinefelter syndrome, two X
required. Mild intellectual disability is chromosomes are present in a male:
defined by an IQ of 55 to 70, whereas XXY. Clinical features include intellectual
severe intellectual disability is defined by disability, a wide arm span, high-pitched
an IQ of 25 to myriad of causes for voice, gynecomastia, and small testes.
developmental delay exist, including but
not limited to genetic disorders, toxin ?? 30. You are evaluating a 6-year-old boy
exposure, and metabolic disorders. Down who has coarse facial features, short
syndrome, or trisomy 21, results from stature with dysmorphic appearance,
trisomy of the 21st chromosome or, less prominent psychomotor retardation,
commonly, chromosomal translocations. hepatosplenomegaly, and several
Diagnosis is made in the majority through nodular lesions that are ivory colored
karyotyping. Increased maternal age is a on his upper back. The activity of idu-
risk factor. The frontal lobes are small and ronate sulfatase in leukocytes is almost
under-developed, and the superior absent.   
temporal l gyri are small and thin. Clinical Which of the following is correct
manifestations include presence of medial regarding this condition?
epicanthal folds, slanting palpebral A. It is inherited in an X-linked fash-
fissures, micrognathia (small mouth) ion
leading to an apparently large tongue, the B. Corneal clouding is a prominent
so-called simian crease (in which there is a feature in this syndrome
single palmar crease), Brush field spots C. He has Hurler syndrome
(white spots of depigmentation in the D. Heparan sulfate, but not derma-
iris), clinodactyly (incurving of the tan sulfate, will be elevated in the
fingers), short stature, and other features. urine
In addition to varying degrees of intellec- E. This patient has mucopolysac-
tual disability, seizures, hematologic charidoses type I
malignancies, and congenital heart
defects occur. Early dementia with vv Correct Answer is: A
Alzheimer type pathology is seen, as the This is a case of Hunter syndrome (MPS
B-amyloid gene is on chromosome 21. type II) is caused by a defect in iduronate
These patients are at risk of cervical spinal sulfatase, with accumulation of dermatan
cord compression due to atlantoaxial sulfate and heparan sulfate. These patients
instability. have the Hurler phenotype but lack the
Congenital
139 4
corneal clouding and have characteristic on year of age, she started to have dif-
nodular ivory-colored lesions on the ficulty sitting up and never developed
back, shoulders, and upper arms. These any words. She later stopped smiling
children have short stature, macrocephaly, and laughing, and progressively lost
macroglossia, hoarse voice, hearing loss, use of her hands, constantly rubbing
visceromegaly, dysostosis multiplex, joint her hands against each other.   
contractures, entrapment neuropathies Which of the following statements is
such as carpal tunnel syndrome, and intel- correct?
lectual disability. The diagnosis is based A. It is more common in males but
on mucopolysacchariduria and confirmed can also be seen in females
with analysis of the enzyme activity. Treat- B. Macrocephaly is a common find-
ment is symptomatic. ing
Other MPS include types III, IV, VI, VII, C. It results from a mutation in the
and IX as follows: - MPS type Ill or gene encoding the MeCP2 protein
Sanfilippo syndrome has several subtypes D. It is autosomal dominant in inheri-
(A, B, C, and D) and is associated with tance
accumulation of heparan sulfate only. The E. It results from mutations in mito-
main manifestation is intellectual disabil- chondrial DNA
ity. - MPS type IV or Morquio syndrome
has two subtypes (A and B), and they vv Correct Answer is: C
manifest with corneal clouding, dysostosis This is a presentation of Rett syndrome, a
multiplex, and heart disease, with normal syndrome of motor and cognitive regres-
intelligence. - MPS type VI is also known as sion with eventual severe disability. The
Maroteaux-Lamy syndrome and manifests presentation is one of initially normal
with normal intelligence, dysostosis development with subsequent regression
multiplex, corneal clouding, heart disease, at approximately 6 to 18 months of age.
and other features similar to Hurler Hand wringing and other motor stereo-
syndrome. - MPS type VII is also known as typies are a classic feature; patients with
Sly syndrome, and these patients have Rett syndrome often place their hands in
hydrops fetalis, intellectual disability, their mouth or may hold their hands fisted,
dysostosis multiplex, corneal clouding, with their fingers flexed over their thumb.
and other features of Hurler syndrome. Arrest of head growth with eventual
microcephaly, seizures, scoliosis, dysauto-
zz Suggested Reading nomia including respiratory dysfunction
55 Comprehensive review in clinical neurol- with apneas, and spasticity emerge as the
ogy: a multiple-choice question book for disease progresses. It results from vari-
the wards and boards ©2011 Wolters ous types of mutations in the gene that
Kluwer Health Lippincott Williams & encodes methyl CpG binding protein 2
Wilkins. All rights reserved. (MeCP2), which is involved in binding to
methylated DNA, modulating gene expres-
?? 31. You are evaluating a 4-year-old female sion. It is most often seen in females, and
for her developmental delay. By family it is thought to be most often fatal in boys,
report, she seemed to be a very happy although cases of MeCP2 mutation in
baby early on, she made eye contact male infants and children with intellectual
and smiled and cooed all the time. disability and other features have been
She had met motor milestones similar identified. Another disorder of severe
to her siblings: holding her head up, intellectual disability in females results
crawling, and sitting unsupported. By from a mutation in the gene CDKLS.
140 Chapter 4 · Congenital

?? 32. Which of the following is not true ?? 33. You are evaluating a 9-month-old
regarding abetalipoproteinemia or baby for his hypotonia, growth
Bassen Kornzweig syndrome? retardation and poor psychomotor
A. It is autosomal dominant development. He also has a history of
B. There is demyelination of the pos- episodes of incoordination, especially
terior columns of the spinal cord with high-carbohydrate meals and
C. There is demyelination of periph- during systemic infections. On labora-
eral nerves tory evaluation, he is found to have
4 D. There is vitamin E deficiency elevated levels of lactate and pyruvate,
E. It is associated with fat malab- with a low lactate: pyruvate ratio. Defi-
sorption ciency of the enzyme pyruvate dehy-
drogenase (PDH) is detected.   
vv Correct Answer is: A Which of the following is not true
Abetalipoproteinemi a, or Bassen-Kornz- regarding this condition?
weig syndrome, is an autosomal recessive A. PDH is responsible for oxidative
disorder caused by a molecular defect in decarboxylation of pyruvate to
the gene for the microsomal triglyceride carbon dioxide and acetyl coen-
transfer protein, which is localized to chro- zyme A
mosome 4q22 0.24. This protein normally B. Only inherited in an autosomal
catalyzes the transport of triglyceride, recessive fashion
cholesteryl ester, and phospholipid from C. Can present with severe neonatal
phospholipid surfaces. The defect of this acidosis
protein results in fat malabsorption and D. High-carbohydrate diet can pre-
liposoluble vitamin deficiency, especially cipitate episodes of ataxia
of vitamin E, which is the culprit of most of E. Ketogenic diet has been used to
the clinical manifestations. This condition treat patients with this condition
manifests since birth, with failure to thrive,
vomiting, and loose stool. During infancy, vv Correct Answer is: B
there is progressive psychomotor retarda- Pyruvate dehydrogenase (PDH) deficiency
tion with cerebellar ataxia and gait distur- is caused by defects of the PDH complex,
bance. Proprioceptive sensation is lost in which is responsible for the oxidative
the hands and feet, with less compromise decarboxylation of pyruvate to carbon
of pinprick and temperature sensation. dioxide and acetyl coenzyme A. This enzy-
Deep tendon reflexes are depressed. This matic complex has three main components
is likely from demyelination of posterior called E1 (A and B subunits), E2, and E3.
columns and peripheral nerves. Visual E1 deficiency is the most common and is
disturbance is the result of retinitis pig- inherited in an X-linked fashion, whereas
mentosa, and nystagmus is common. the other defects are autosomal recessive.
Laboratory studies demonstrate acan- Since the brain derives energy primar-
thocytosis, absence of very low-density ily from glucose oxidation, neurologic
lipoproteins, absence of apolipoprotein dysfunction is one of the main clinical
B, low levels of vitamin E, and severe ane- features. The clinical presentation is vari-
mia. Treatment involves the restriction of able, ranging from severe neonatal lactic
triglycerides in the diet, and large doses acidosis with death in the neonatal period,
of vitamin E with supplementation of vita- to less severe forms that are manifested in
mins A, D, and K. infancy, in which patients have lactic and
Congenital
141 4
pyruvic acidosis, and episodic or progres- A. Autosomal dominant
sive ataxia, nystagmus, dysarthria, leth- B. Autosomal recessive
argy, weakness with arreflexia, hypotonia, C. Post-infectious
and psychomotor retardation, which can D. Random (non-infectious and non-
be profound. These patients have periodic genetic)
exacerbations, which can be spontaneous
or triggered by infections, stress, or high- vv Correct Answer is: B
carbohydrate meals. Some patients may This a case of Friedreich’s Ataxia (FA). It
have a presentation of Leigh disease. The is an autosomal recessive disease caused
diagnosis is suspected in children with by a mutation on chromosome 9 in the
clinical manifestations as described and gene encoding the protein frataxin.
with elevations of lactate and pyruvate Therefore, it occurs in males and females
levels, with a low lactate: pyruvate ratio. equally. It is the only disease caused by a
Enzyme analysis can be performed in GAA trinucleotide repeat. The more GAA
leukocytes, cultured fibroblasts, muscle, repeats, the more profound the deficit
or liver biopsy specimens. Pathologically, in frataxin protein. Those patients usu-
there may be cystic lesions in the white ally present between the ages of 5 to
matter and basal ganglia, and certain cases 15, though cases do occur even younger
of the neonatal form may have agenesis of than this. Late-onset FA usually presents
the corpus callosum. Management of PDH in the 2nd or 3rd decades. Presenting
deficiency includes ketogenic diet (high symptoms and signs include ataxia,
fat with low carbohydrates) and thiamine motor weakness, scoliosis/kyphoscolio-
supplementation. Carnitine, coenzyme sis, dysarthria, high arches or pes cavus,
Q10, and biotin supplementation may be diabetes in20%, dysphagia, absent DTRs,
given, but their efficacy is not well estab- muscle wasting, an extensor plantar
lished. Acetazolamide may be used for the response, nystagmus, cardiac involve-
treatment of episodes of ataxia. ment (cardiomegaly, cardiomyopathy).
Most patients with FA are wheelchair-
?? 34. You are seeing an 8-year-old boy who bound by age 15.
is referred to you because his funny Treatment is symptomatic, and aims to
walk for the last few months. His support and maintain ambulation as long
mother reports that he has been clum- as possible.
sier, tripping frequently as well. While
you are examining him, you notice ?? 35. You are seeing a 7-month-old boy with
that his speech seems slightly slurred, history of seizures and developmen-
and his mother states this just started tal delay for follow-up visit. He has
a week ago. On examination, he has had increased head circumference,
a wide-based slow gait and steppage progressive psychomotor retardation,
gait. Motor strength in the distal lower spasticity and qudriparesis for the
extremities is 2/5 and proximal is 3/5. last 2 months. His brain MRI showed
Lower extremity DTRs are absent. There white matter hyperintesity mainly in
is an extensor plantar response, and the frontal lobes and anterior cere-
his feet are significant for high plantar bral regions, involving the cortical U
arches. Also, he has scoliosis.    fibers.   
What is the mode of inheritance in What is the most likely etiology for
this disease? his presentation?
142 Chapter 4 · Congenital

A. Hexosamindiase A deficiency On examination, he has enlarged calf


B. Galactocerbrocidase beta-galac- muscles.   
tosidase deficiency What the inheritance of this condi-
C. Gene mutation for glial fibrillary tion?
acidic protein (GFAP) A. Sporatic/unknown
D. Aspartocylase deficiency B. Most common in Ashkenazi Jews
E. Mitochondrial DNA deletion C. Autosomal dominant
D. Autosomal recessive
4 vv Correct Answer is: C E. X-linked
This is a patient of Alexander disease. Eti-
ology is gene mutation for glial fibrillary vv Correct Answer is: E
acidic protein (GFAP). It is a progressive Duchenne Muscular Dystrophy (DMD)
disorder of astrocytes. There are three and Becker Muscular Dystrophy have vari-
forms: Infantile form – features include able phenotypic expression of the gene
megalencephaly, developmental delay, defect on Xp21. In DMD the gene product
seizures, psychomotor retardation, spas- dystrophin is present at 0–5% of the
ticity, and quadriparesis. normal level and in BMD between 5–20%.
Juvenile form - in childhood, more The incidence of DMD is 1 per 3500 male
significant bulbar symptoms. births and typically presents earlier than
Adult form – features include bulbar BMD. Mutation analysis is now routine for
signs, hyperreflexia, dysautonomia, ataxia, diagnosis, carrier detection and fetal diag-
sleep apnea. nosis. At the time of diagnosis, the serum
MRI shows diffuse white matter signal concentration is usually 10 times normal
hyperintensity, mainly in the frontal lobes in boys with DMD.
and anterior cerebral regions, involve- Dystrophin analysis on muscle biopsy
ment of U fibers. In the adult-onset form, can aid in differentiating the two forms of
the “tadpole sign” on sagittal MRI results the condition. Several of the inherited
from dramatic thinning of the upper metabolic disorders are more common in
cervical spinal cord. the gene pool of Ashkenazi Jews.
Histopathology, there is Rosenthal
fibers - elongated eosinophilic fibers seen zz Suggested Reading
on H&E, and they are diffusely distributed 55 Yiu EM, Kornberg AJ. Duchenne muscular
throughout the brain with clusters in the dystrophy. J Pediatr Child Health. 2015.
sub-pial, sub-ependymal, and perivascular
areas. Severe myelin loss and cavitation of ?? 37. Which of the following statements is
the white matter is seen. not true regarding the syndromes of
cobblestone lissencephaly?
zz Suggested Reading A. Muscle-eye-brain disease of San-
55 Mowzoon N, Flemming KD. Neurology tavuori is most often seen in Fin-
board review: an illustrated study guide. land
Rochester: Mayo Clinic Scientific; 2007. B. Fukuyama muscular dystrophy is
Print. most often seen in Japan
C. Muscular dystrophy is seen in all
?? 36. You are seeing a 6 years old boy for his these syndromes
abnormal gait which started at 4 years D. Eye abnormalities seen include
of age, and followed by toe walking. retinal hypoplasia and glaucoma
He has a history of frequent falls with E. They are autosomal dominant in
a subtle loss of hand grasp strength. inheritance
Congenital
143 4
vv Correct Answer is: E Her head circumference is 45 cm, her
The cobblestone lissencephalies are anterior fontanelle is soft and small, and
mainly autosomal recessive in inheritance no other dysmorphic or cutaneous fea-
although X-linked forms have also been tures are noted. You ordered a MRI, which
identified. Cobblestone lissencephaly, or shows a left posterior leptomeningeal
lissencephaly type II, is a neuronal migra- angioma with prominent, tortuous veins
tion disorder in which the cortical gray coursing toward the vein of Galen.   
matter has reduced number of gyri and What is the inheritance pattern of
sulci that appear like cobblestones. There this disorder?
is reduced and abnormal white matter, A. Autosomal dominant
and the cerebellum and brain stem are B. Autosomal recessive
hypoplastic. Microscopically, the cortex C. Maternal mitochondrial
has no recognizable layers. Hydrocephalus D. Sporadic
frequently occurs because of the presence E. X-linked
of fibroglial bands and abnormal vascular
channels disrupting the subarachnoid vv Correct Answer is: D
space. It has three autosomal recessive This is a case of Sturge Weber syndrome
syndromes: Walker-Warburg syndrome, (SWS). It is a “congenital, non-familial
Fukuyama muscular dystrophy, and mus- disorder of unknown incidence and
cle-eye-brain disease of Santavuori. These cause.” In Takeoka’s article the incidence is
three syndromes share similar clinical estimated at 1 per 50,000. The neuroimag-
features, including microcephaly, global ing findings can be unilateral or bilateral,
developmental delay, epilepsy, hypotonia, even with unilateral skin findings, though
and evidence of muscular dystrophy. if unilateral, it is generally on the same
Walker-Warburg is the most severe form. side as the port wine stain. Glaucoma can
Several gene mutations have been identi- be present at birth or develop later.
fied in patients with Walker-Walburg and
muscle-eye-brain disease of Santavuori. zz Suggested Reading
Eye abnormalities in the latter two condi- 55 7 http://www.sturge-weber.org/about-sws/
tions include retinal hypoplasia, optic conditions/12-sturge-weber-syndrome.html.
nerve atrophy, glaucoma, and cataracts. 55 Sturge-Weber Syndrome. On 7 http://
Muscle eye-brain disease of Santavuori is emedicine.medscape.com/article/1177523-
seen most commonly in Finland but also overview, author M Takeoka. Updated Jan
occurs in other European populations. 5, 2010.
Fukuyama muscular dystrophy, which
results from a mutation in the gene fuku- ?? 39. You are seeing a 6-year-old boy for his
tin, is most common in Japan and is rare in motor development delay. His history
other populations. It has the least amount is remarkable of unsteady gait ever
of cortical abnormalities, but the muscular since he started to walk independently
dystrophy is severe with progressive weak- at 3 years of age. Also, he has frequent
ness and joint contractures. An elevated involuntary movements in his arms
level of serum creatine kinase is seen. and hands. On examination he has
choreoathetoid movements in the
?? 38. You are seeing a 9-month-old baby in the upper extremities. He prefers to turn
ED for a new-onset seizure. She has been his head rather than his eyes during
healthy, with no fever or other signs of horizontal tracking. He has decrease
illness. On examination, she has a reddish sensation to all modalities and absent
patch over her right eyelid and forehead. deep tendon reflexes. Gait and
144 Chapter 4 · Congenital

coordination examination revealed in the ATM gene on chromosome 11


both limb and truncal ataxia. Eye exam is the cause of the disorder, and in
showed that he has prominent blood addition to the neurologic phenotype,
vessels in the conjunctiva.   results in impaired DNA repair leading
What is correct regarding this to an increased risk of hematologic and
patient’s most likely diagnosis? other malignancies. Hence exposure to
A. The clinical picture is consistent radiation even for routine procedures
with a pure cerebellar ataxia. is strongly discouraged. MRI may show
4 B. It is associated with the Fragile X signs of progressive cerebellar atrophy.
chromosome. Patients usually have hypogammaglo-
C. This condition is secondary to a binemia and are prone to infections. In
GAA repeat expansion. fact, malignancies and recurrent infec-
D. Patients with this condition tions are the most common cause of
typically have hypogammaglobu- death in these individuals eventually.
linemia. Freidreich’s ataxia is another autosomal
E. Serial CT scans of the head should recessive ataxia syndrome in children,
be performed to monitor the pro- results from a GAA expansion in the
gression of cerebellar atrophy. frataxin protein coding region. Patients
with this genetic condition typically
vv Correct Answer is: D have a later onset of ataxia (between
This a presentation of ataxia–telangiec- ages 10–15) and do not have the above
tasia, an autosomal recessive disorder mentioned cutaneous changes or the
that presents in early childhood with extrapyramidal or ocular features
ataxia. The telangiectasias are venous (though signs of dorsal column and
plexuses that are commonly present in peripheral nerve dysfunction are also
the bulbar conjunctiva, ear lobes and present.) Fragile X typically leads to
other areas. It is important to know that severe intellectual disability in children.
these hallmark cutaneous signs may not Adults with milder phenotypes may
be present until several years after the present with ataxia as part of the Fragile
neurologic features of the disorder. In X- Tremor- Ataxia syndrome.
addition to the truncal and limb ataxia,
affected patients may have frequent zz Suggested Reading
choreoathetoid movements, oculomo- 55 Ropper A, Samuels M, Klien J. Adams and
tor apraxia (evidenced by a defect of Victor’s Principles of Neurology. 10th ed.
initiation of spontaneous saccades forc- New York: Mcgraw Hill; 2014. p. 946.
ing a patient to compensate by head- 55 Pandolfo M, Manto M. Cerebellar and
thrusting in the desired direction) and afferent ataxias. Continuum Lifelong
signs of a diffuse neuropathy. Mutation Learning Neurol. 2013;19(5):1312.
145 5

Demyelinating Disorders

© Springer International Publishing AG, part of Springer Nature 2018


Y. M. Awaad, Absolute Pediatric Neurology, https://doi.org/10.1007/978-3-319-78801-2_5
146 Chapter 5 · Demyelinating Disorders

?? 1. Which of the following is inconsistent symptoms, in the absence of pleocytosis


with a diagnosis of Guillain-Barre syn- (“albuminocytological dissociation”)
drome 55 Nerve conduction slowing or block on
A. Sensory involvement nerve conduction studies
B. Persistent asymmetry of weakness 55 Recovery over week
C. Cranial nerve palsies Findings that are inconsistent with a
D. Dysautonomia diagnosis of Guillain-Barre syndrome
E. Increased cerebrospinal fluid pro- include:
tein levels for >1 week 55 Persistently asymmetric weakness
55 A clear sensory level
55
5 vv Correct Answer is: B
Acute inflammatory demyelinating
Severe bowel or bladder dysfunction at the
onset of symptoms
polyradiculoneuropathy (AIDP), or 55 A marked cerebrospinal fluid pleocytosis
“Guillain-­Barre syndrome,” is an autoim- should also raise suspicion for an alterna-
mune, areflexic, ascending weakness tive diagnosis.
with mild sensory changes that pro-
gresses over a few weeks’ time. It is clas- ?? 2. A 15-year-old female came to the
sically a demyelinating process, ED with a 2 days history of right eye
although axonal damage may occur in blurry vision that has rapidly pro-
cases of severe demyelination. gressed to an inability to read print
Predominantly axonal variants also with that eye. This morning, she had
exist (e.g. Miller-Fisher syndrome, acute complained of a frontal headache,
motor axonal neuropathy). It is often pain with eye movement, and flash-
associated with a preceding respiratory ing lights in her central visual field,
infection, gastrointestinal infection, sur- which she attributed to migraine.
gery, or vaccination. She had an upper respiratory tract
Characteristic manifestations include: infection 2 weeks ago with full
55 Progressive upper and lower extremity recovery. On examination, her visual
weakness, which may also involve facial, acuity is 20/20 OS and finger count-
oropharyngeal, and respiratory muscles ing at 3 feet OD. She has a right
55 Areflexia or hyporeflexia afferent pupillary defect and eleva-
55 Progression over days to weeks. By tion of the right optic nerve head.
definition, progression ends 4 weeks after Head MRI is shown. CSF analysis is
onset unremarkable.
55 Symmetry of weakness Admission to the hospital with high-
55 Mild sensory changes dose methylprednisolone treatment is
55 lnter-scapular or low back pain recommended for which of the follow-
55 Cranial nerve palsies (45–75% of cases), ing reasons?
most often bilateral and involving the A. Preventing blindness
facial nerves B. Preventing involvement of the
55 Dysautonomia (e.g. orthostasis, anhidrosis, unaffected eye
urinary retention, arrhythmias), occurring C. Speeding recovery of the affected
in approximately two-thirds of patients eye
55 Lack of fever D. Preventing the subsequent devel-
55 Elevated cerebrospinal fluid protein levels opment of multiple sclerosis
persisting beyond the first week of E. Treating status migrainosus
Demyelinating Disorders
147 5
up this morning. He vomited several
times and has complained of head-
ache and neck pain. He has unsteady
gait and needed help to walk. Past
medical history is unremarkable, but
he had an upper respiratory illness
with sore throat and cough 1 week
ago. He has no known allergies and
taken no medications. On examina-
tion he is afebrile with normal vital
signs. He is lethargic and does not
follow commands. He has mild neck
stiffness and complains of headache.
Cranial nerves are intact. Tendon
..      Fig. 5.1 Optic Neuritis (Transferred from en. reflexes are brisk in the right limbs
wikipedia to Commons. Topperdoc at English Wikipedia)
and the right plantar response is
extensor. He is unable to walk. EEG
vv Correct Answer is: C shows diffuse slowing and MRI is
Optic neuritis is an acute inflammatory shown. CSF exam shows protein
condition of the optic nerve. It occurs as 62 mg/dL, glucose 60 mg/dL, 35 lym-
a separate entity or as a manifestation phocytes/hpf, and 0 RBC/hpf.  
of many other demyelinating diseases What is the most likely diagnosis?
(multiple sclerosis, neuromyelitis optica, A. Acute disseminated encephalo-
and acute disseminated encephalomy- myelitis
elitis) and collagen vascular disorders B. Herpes simplex virus encephalitis
(lupus). There are no controlled trials C. Measles encephalitis
regarding the treatment of optic neuri- D. Multiple sclerosis
tis in children; however, the Optic E. Mitochondrial encephalomyopathy
Neuritis Treatment Trial (ONTT) demon-
strated that treatment with high-dose vv Correct Answer is: A
intravenous methylprednisolone short- Acute disseminated encephalomyelitis
ened the time to initial recovery but did (ADEM) is an immune-mediated demye-
not change the long-­term visual course linating disease which most commonly
in adults. Most physicians treat pediatric affects children (mean age 7 years). It
optic neuritis with intravenous methyl- most commonly presents in winter and
prednisolone at a dose that is compara- spring. Majority of patients have history
ble to that used in the ONTT. of a recent infection with a mean
latency of 2 weeks. Encephalopathy is a
zz Suggested Reading must symptom; typical symptoms
55 Brady KM, Amarpreet SB, Lee AG, Coats include headache, vomiting, confusion,
DK, Paysse EA, Steinkuller PG. Optic drowsiness, and meningismus, symp-
neuritis in children: clinical features and toms which are uncommon in multiple
visual outcome. J AAPOS. 1999. sclerosis. Other neurologic signs relate
to the specific areas of white matter
?? 3. An 8-year-old boy is brought by his involvement and can include pyramidal,
family to the ED because he has been cerebellar, and brain stem signs.
sleepy and confused since waking Distinguishing ADEM from multiple
148 Chapter 5 · Demyelinating Disorders

..      Figs. 5.2 and 5.3 MRI showing white matter changes in ADEM disease. American Academy of Neurology
Institute, produced by permission

sclerosis (MS) can be difficult. In MS, CSF can the initial presentation helps in
usually shows intrathecal s­ ynthesis of establishing a correct diagnosis? Arch Dis
oligo clonal bands. The MRI shows mul- Child. 2005;90:636–9.
tifocal white matter lesions in both 55 Menge T, Kieseier BC, et al. Acute dissemi-
ADEM and MS, but there are differences. nated encephalomyelitis: an acute hit
Lesions in ADEM tend to have less dis- against the brain. Curr Opin Neurol.
tinct margins, do not involve the peri- 2007;20(3):247–54.
ventricular white matter, but do involve
deep gray matter. Lesions in MS typi- ?? 4. A 4-year-old female started having trou-
cally have distinct margins (plaques) ble walking for 2 days. Today, she is not
and are most commonly seen in the able to bear weight, has trouble raising
periaqueductal, corpus callosum, and her arms above her head, and cannot
periventricular white matter. Repeat crawl. She is not complaining of any
MRI scans 6 months after presentation pain at rest but when you performed
most often show complete or partial the Kernig and Brudzinski maneuvers
resolution of the lesions and no new on her, there is discomfort in the thighs.
lesions in ADEM. Treatment of ADEM She is areflexic but her cranial nerves
usually involves 10–30 mg/kg/day intra- are intact. She has a normal MRI.
venous methylprednisolone for 3 days What are you going to do to confirm
followed by a 2–6 week tapering course the diagnosis?
of oral prednisolone. A. CSF protein
B. Repetitive nerve stimulation study
zz Suggested Reading C. MRI of the spine with contrast
55 Dole RC, Branson JA. Acute disseminated D. Nerve conduction velocity study
encephalomyelitis or multiple sclerosis: E. Response to IVIG infusion
Demyelinating Disorders
149 5
vv Correct Answer is: D
The nerve conduction studies are the
best studies to confirm acute demyelin-
ating polyneuropathy. Typically, conduc-
tion velocity is slowed but there may be
prominent delays in F waves and con-
duction blocks as well. CSF protein may
not be elevated early in the clinical
course and thus is not reliable for diag-
nostic confirmation. On occasion MRI
may identify inflamed nerve roots but
this is not a reliable way to make the
diagnosis. Repetitive stimulation would
be normal in these patients as there is no
defect in neuromuscular transmission.
Response to treatment is not a reliable
approach to diagnosis, particularly a
nonspecific therapy such as immuno-
globulin.

zz Suggested Reading
55 Soliven B. Acute demyelinating polyra- ..      Fig. 5.4 MRI showing white matter changes in
Multiple sclerosis. (James Heilman, MD. Wikipedia)
diculoneuropathy In: Gilman S, Goldstein
GW, Waxman SG, editors. Med link. 12th
ed. San Diego: Arbor Publishing; 2007 vv Correct Answer is: D
55 Bromberg MB. Comparison of electrodi- Typically, the diagnosis of MS requires
agnostic criteria for primary demyelin- the identification of more than one epi-
ation in chronic polyneuropathy. Muscle sode of neurological dysfunction with
Nerve. 1991;14(10):968–76. more than one locale within the central
nervous system. The increasing use of
?? 5. A 16-year-old female complains of MRI and the variability of the clinical
acute onset of numbness in her right presentation in the pediatric age group
arm, which disappeared few days have led to a change in the diagnostic
later. Her medical history is unremark- criteria for MS in this population.
able and she takes no medications. The criteria for MS on MRI in pedi-
A brain MRI is obtained. A FLAIR atrics requires 2 of the following three
Sequence sample is shown. features;
Which of the following test results 1. Five or more lesions on T-2 weighted
listed below is needed to diagnose MRI scans.
MS? 2. Two or more periventricular lesions.
A. Demonstration of abnormal 3. At least one lesion in the brainstem.
SSEPs
B. Demonstration of abnormal VEPs The importance of making a firm diag-
C. Monoclonal antibodies in the CSF nosis of MS, even in the pediatric age
D. Five or more lesions on T-2 group is enhanced by the fact that there
weighted MRI are now therapies available which have
E. T-2 weighted MRI showing lesions the potential to modify the long-term
in the spine outcome of these patients.
150 Chapter 5 · Demyelinating Disorders

zz Suggested Reading treated with Lamotrigine. 1 week ago,


55 Callen DJ, et al. Role of MRI in the she had cough and congestion and
differentiation of ADEM from MS in she was treated with amoxicillin, oth-
children. Neurology. 2009;72:968–73. erwise she is a health girl. She has nor-
mal vital signs. She is drowsy, but you
?? 6. You are seeing a 9-year-old female in can awaken her with vigorous stimula-
the ED for her gait disturbance and tion. She is not speaking or following
drowsiness. 2 days ago, she woke up commands. Her pupils are 6 mm and
with walking difficulty, veering to the sluggish. She has normal fundus exam.
right side. By the end of the day, she Muscle tone is normal and tendon
5 became drowsier and has to stay in
bed. She has an absence seizure disor-
reflexes are brisk, with bilateral exten-
sor plantar responses. A brain MRI is
der which began last year and is being obtained and is shown here.

..      Figs. 5.5 and 5.6 MRI showing white matter changes in ADEM. American Academy of Neurology Institute,
produced by permission

What is your explanation for her inflammatory demyelination with mul-


condition? tiple CNS sites neurological deficits most
A. Acute demyelinating encephalo- often accompanied by encephalopathy
myelopathy (defined as altered behavior or conscious-
B. Herpes simplex encephalitis ness). MRI features of diffuse, bilateral
C. Clinically isolated syndrome lesions are supportive, but not diagnostic,
D. Multiple sclerosis of ADEM. ADEM occurs more commonly in
E. Tuberculous meningitis children less than 10 years of age. ADEM
may be preceded by infectious symptoms,
vv Correct Answer is: A although a specific pathogen is rarely
Acute demyelinating encephalomyelopa- found. Meningism, fever, and seizures are
thy (ADEM) is defined as a first episode of more commonly seen in ADEM than in
Demyelinating Disorders
151 5
“clinically isolated syndrome”. Complete
recovery is expected in approximately
70%. This patient’s drowsiness and lack
of prior neurological events distinguish
ADEM from either clinically isolated
syndrome or multiple sclerosis. Herpes
simplex encephalitis typically affects the
temporal lobe rather than the brainstem.
The MRI findings and her clinical course
are much more characteristic of ADEM
than tuberculous meningitis.

zz Suggested Reading
55 Dale RC, Brilot F, Banwell B. Pediatric
central nervous system inflammatory
demyelination: acute disseminated
encephalomyelitis, clinically isolated
syndromes, neuromyelitis optica, and ..      Fig. 5.7 Herpes encephalitis. Coronal T2-weighted
multiple sclerosis. Curr Opin Neurol. MR image shows high signal in the temporal lobes
including hippocampal formations and parahippogam-
2009;22:233–40.
pal gyrae, insulae, and right inferior frontal gyrus. A brain
biopsy was performed and the histology was consistent
?? 7. A 4-year-old boy has been evaluated with encephalitis. PCR was repeated on the biopsy
for his first left onset partial seizure specimen and was positive for HSV. (7 http://www.
with secondarily generalization. EEG radpod.org/2007/03/24/herpes-simplex-encephalitis/.
dr Laughlin Dawes)
showed right sided spike and sharp
wave and also slowing on the same
side. Because his high peripheral seizure, particularly if it is secondary to
white blood cell count at 18.5, you a known process. One might also con-
ordered a lumbar puncture which sider an MS panel on the CSF, because in
showed 18 WBC/uL and a protein of children under 10 years of age multiple
45 mg/dl. His MRI is shown here.   sclerosis more often has large white
What is your next step in his man- matter lesions and more often has sei-
agement? zures associated with the onset.
A. Administer Fosphenytoin 20 mg/kg
B. Administer high-dose intravenous zz Suggested Reading
corticosteroid medication. 55 Aicardi J. Diseases of the nervous system
C. Administer intravenous acyclovir. in childhood. 2nd ed. Cambridge
D. Begin plasmapheresis. University Press; 1998. p. 444.
E. Send CSF for HSV PCR.
?? 8. You are evaluating a 5-year-old boy
vv Correct Answer is: E for his irritability and unsteadiness in
The clinical presentation is most likely walking and using his arms for the last
ADEM but it is very important to rule 2 days. He was crying from pain in his
out herpes encephalitis. One should legs and back and now he is unable
also consider visual fields and visual to walk without assistance. He had
acuity testing and inquire about the sta- gastroenteritis 1 week which has been
tus of the spinal cord function. Anti- resolved with no treatment. He takes
epileptics are not indicated after his first no medications. His family history is
152 Chapter 5 · Demyelinating Disorders

unremarkable. On your examination, ?? 9. A 13-year-old female has been evalu-


he has normal ocular movements and ated because of her vertigo, nausea,
pupillary reactions. He has mild facial vomiting, and visual disturbances
weakness, and mild weakness in both which started 2 weeks ago after a bas-
shoulders, but a good handgrip. He ketball game. She reported that she
has action coarse tremor when reach- felt unsteady just before starting the
ing for objects. He cannot support his game, but she did not pay too much
own weight and is unable to sit unsup- attention to it. She was struck by the
ported. He is areflexic and plantar ball in the face and felt stunned. Two
responses are flexor. Sensation is intact. hours later, she noticed a spinning
5 What is your diagnosis?
A. Acute cerebellar ataxia
sensation associated with nausea and
vomited several times. The following
B. Acute transverse myelitis day, her vertigo continued and she
C. Botulism was unable to walk. Two days later
D. Guillain Barre Syndrome she began to experience double
E. Polymyositis vision. An MRI was performed and
results are not available. Her symp-
vv Correct Answer is: D toms started to resolve gradually over
The association of facial and limb weak- the following 2 weeks. You examined
ness with areflexia of acute onset and her today and she has normal exam.
rapid progression are most compatible What is the most likely diagnosis?
with acute inflammatory demyelinating A. Acute demyelinating encephalo-
polyradicuopathy, although a significant myelopathy
subset of these patients have an axonal B. Multiple sclerosis
form rather than demyelinating. Guillain C. Clinically isolated syndrome
Barre Syndrome is an eponym for acute D. Neuromyelitis optica
or chronic, axonal or demyelinating E. Traumatic brain injury
inflammatory neuropathy. Presentation
can be different in young children than vv Correct Answer is: C
in adults. 1/3 of pediatric cases occur in The girl has a first episode of neurologi-
less than 3 years of age children. In the cal dysfunction, the onset of which pre-
pediatric group, pain is a common pre- dated the basketball injury. Multiple
senting complaint, involving the back or sclerosis diagnosis would require a sec-
limbs. Ataxia and disturbances of equi- ond demyelinating event. ADEM diag-
librium are more common in children nosis requires ­encephalopathy (altered
than adults. Electrophysiological studies behavior or consciousness) as a clinical
performed in the first week of the illness feature. Neuromyelitis optica is charac-
most often demonstrate prolonged or terized by sequential or concomitant
absent F-responses (88%), prolonged optic neuritis and transverse myelitis,
distal latency (75%), conduction block patient does not have either. Clinically
(58%), and reduction in conduction isolated syndrome (CIS) is the most
velocity (50%). accurate diagnosis for this patient. CISs
are more common than ADEM in chil-
zz Suggested Reading dren, particularly in those > 10 years of
55 Delanoe C, Sebire G, et al. Acute inflam- age. CIS is a clinically heterogeneous
matory demyelinating polyradiculopathy entity and has a higher incidence of
in children: clinical and electrodiagnostic subsequent MS diagnosis than
studies. Ann Neurol. 1998;44:350–6. ADEM. CIS can be mono-focal affecting
Demyelinating Disorders
153 5
a localized part of the CNS (optic neuri- cases of inflammatory optic neuritis, it is
tis, transverse myelitis, brainstem syn- not the most likely finding. A macular
drome, hemispheric syndrome) or star may be seen in some cases of severe
multifocal but must not include enceph- optic nerve edema, but this finding indi-
alopathy. cates neuro-retinitis. It may be seen in
association with infectious etiologies as
zz Suggested Reading well as non-infectious etiologies, includ-
55 Dale RC, Brilot F, Banwell B. Pediatric ing multiple sclerosis, sarcoidosis and
central nervous system inflammatory other inflammatory disorders.
demyelination: acute disseminated Inflammatory cells in the vitreous may
encephalomyelitis, clinically isolated suggest uveitis: this finding is not sug-
syndromes, neuromyelitis optica, and gestive of optic neuritis. An increased
multiple sclerosis. Curr Opin Neurol. cup-to-disc ratio is seen in glaucoma,
2009;22:233–40. and is indicative of increased intraocular
pressure.

?? 10. T
 he following scenario is part of ?? 11. T
 he following scenario is question 2
a 7-question series, Discussion & of the 7-question series, Discussion
References included at the end of & References included at the end of
series. this series.
The following 7 questions pertain Imaging is performed:
to the patient history below: Image 1: axial T2-FLAIR image of
A 10-year-old boy is complaining the brain.
of 5 days history of frontal headache Image 2: T2-weighted image of
which affected him playing his last orbits.
volleyball game. He also complains of Image 3: T1-weighted image of
blurred vision. His visual acuity is the orbits with contrast.
count fingers on the left and 20/60 on Image 4: T2-weighted sagittal
the right, with color desaturation on image of the spine.  
the left. What findings are you seeing on the
Given this patient’s history, which of MRI?
the features below is most likely to A. T2 hyper intensities of the ON
be seen? with gadolinium enhancement;
A. Optic nerve (ON) pallor T2 lesion in the spinal cord, brain
B. Macular Star normal
C. Bilateral papillitis B. T2 hyper intensities of the ON
D. Cells in the vitreous bilaterally, Brain, and Spinal Cord,
E. Increased cup to disc ratio with leptomeningeal enhance-
ment
vv Correct Answer is: C C. Brain, spine and ON within normal
This is a clinical presentation of acute limits
optic neuritis. Bilateral papillitis will be D. Bilateral ON hyper intensities with
the most expected findings on his exam. gadolinium enhancement, brain
Optic nerve pallor is indicative of a and spine within normal limits.
chronic process, and therefore is less E. Longitudinally extensive spinal
likely to be seen in acute ON. While a cord lesion, ON within normal lim-
macular star might be seen in extreme its, brain within normal limits
154 Chapter 5 · Demyelinating Disorders

..      Figs. 5.8, 5.9, 5.10 and 5.11 Bilateral ON hyper intensities with gadolinium enhancement in 7 Fig. 5.9, all
other figures are normal. By permission from Radiopaedia

vv Correct Answer is: D ?? 12. T


 he following scenario is question 3
Imaging shows bilateral optic nerve T2 of the 7-question series, Discussion
weighted hyper intensities with gado- & References included at the end of
linium enhancement on T1 weighted this series.
imaging bilaterally. The other imaging is What might be seen on visual field
normal. testing?
Demyelinating Disorders
155 5
A. Normal visual fields A. 1/3
B. Bilaterally increased blind spot B. ½
C. Bilaterally decrease blind spot C. ¾
D. Bilateral quadrantanopia D. 1/8
E. Homonymous homonopsia
vv Correct Answer is: C
vv Correct Answer is: B In a small series of children with clini-
The most common visual field abnor- cally established recurrent NMO, about
mality seen in bilateral optic neuritis is ¾ (78%) are positive for the antibody.
an enlarged central scotoma, or on field
testing, bilaterally increased blind spot. zz Suggested Reading
55 Banwell et al. Neurology. 2008 Jan 29;
?? 13. T
 he following scenario is question 4 70(5):344–52. Epub 2007 Dec 19.
of the 7-question series, Discussion
& References included at the end of ?? 15. The following scenario is question 6
this series. of the 7-question series, Discussion
Testing is performed. & References included at the end of
Which key serological test may be this series.
important in this case? What would you expect the visual
A. ANA evoked potential to show in this
B. CRP child?
C. ESR A. Bilaterally decreased amplitude
D. NMO-IgG B. p100, no response on the left,
E. NMDAR ab markedly delayed on the right
C. W-wave bilaterally
vv Correct Answer is: D D. Delay of n75 bilaterally
NMO-IgG. Neuromyelitis optica is char-
acterized by severe optic neuropathy vv Correct Answer is: B
and longitudinally extensive transverse The child has bilateral optic neuritis
myelitis. It is positive in approximately with severe involvement of the left and
3/4 of children with a clinical diagnosis moderate involvement on the right.
of relapsing neuromyelitis optica. Prolongation of the p100 is the classical
finding on PRVEP in optic neuritis.
zz Suggested Reading
55 Banwell et al. Neurology. 2008 Jan ?? 16. T
 he following scenario is question 7
29;70(5):344–52. Epub 2007 Dec 19. of the 7-question series, Discussion
& References included at the end of
?? 14. T
 he following scenario is question 5 this series.
of the 7-question series, Discussion What would your first line treatment
& References included at the end of of choice for this acute presentation
this series. be?
The parents ask what percentage A. IVIG 1 g/kg per day for 2 days
of children who have the disease B. Solumedrol 30 mg/kg/day for
being evaluated for in the question 3 days
above have a positive serum test. C. Cyclophosphamide 750 mg/m2
What would your answer to this D. Plasmapheresis, 5 cycles
family be?
156 Chapter 5 · Demyelinating Disorders

vv Correct Answer is: B vv Correct Answer is: B


The first line therapy of choice is pulse Small percentage of children have
IV steroids, other treatments might be underlying neurologic disease for their
used in severe optic neuritis. The optic acute psychosis. Several series of
neuritis treatment trial suggests that patients with first onset schizophrenia,
the use of pulse IV Solumedrol for depression, and borderline personality
3 days speeds recovery of ON in com- have not revealed an increased likeli-
parison to placebo and oral prednisone hood of pathologic antibodies to the
at a dose of 1 mg/kg/d. Beck et al. N NMDA-receptor (NR1 IgG) in these
Engl J Med. 1992 Feb 27;326(9):581–8. patients.
5 Pulse steroid dosing in the pediatric
population is 20–40 mg/kg/d for 3 days. zz Suggested Reading
While recovery may be hastened, there 55 Kayser et al. JAMA Neurol. 2013
is no evidence for long term benefit September 1;70(9): Of the answers pro-
of steroids over placebo for return to vided, an acute or sub-acute onset might
normal VA (Gal RL, Vedula SS, Beck R, point to an autoimmune etiology, but is not
Corticosteroids for treating optic neu- adequate in itself to suggest this (Suleiman
ritis, Cochrane Database Syst Rev. 2012 et al. Epilepsia 54(6)1036–45, 201).
Apr 18;4:CD001430. doi: 7 https://doi.
org/10.1002/14651858.CD001430.pub3.). ?? 18. T
 he following scenario is part 2 of
a 7-question series, Discussion &
zz Suggested Reading References included at the end of
55 Beck et al. N Engl J Med. 1992 Feb series.
27;326(9):581–8. The family reports occasional
55 Gal RL, Vedula SS, Beck R, Corticosteroids transient facial twitching not
for treating optic neuritis. Cochrane accompanied by loss of
Database Syst Rev. 2012;(4):CD001430. consciousness. You order an EEG.
7 https://doi.org/10.1002/14651858. Which EEG feature may point spe-
CD001430.pub3. cifically to an autoimmune encepha-
litis?
?? 17. T
 he following scenario is part of a A. Frontal predominance of sharp
7-question series. waves
The following 7 questions B. Focal slowing
pertain to the patient history below: C. Extreme delta brush
You are seeing a 14-year-old boy D. Intermittent rhythmic delta
with an 8-week history of behavioral E. Triphasic waves
changes and hallucinations. He was
­previously healthy and a good vv Correct Answer is: C
student at school. He has no history of Extreme delta brush has been reported
psychiatric problems, and no family specifically in NMDAR patients and is
history of psychiatric problems. His usually not present in other encepha-
parents got separated recently. lopathies.
What is in the history might point to
an autoimmune etiology? zz Suggested Reading
A. Presence of mood disturbance 55 Schmitt et al. Neurology. 2012 Sep
B. Acute or sub-acute onset 11;79(11):1094–100. 7 https://doi.
C. Diagnosis of borderline personality org/10.1212/WNL.0b013e3182698cd8.
D. Insight into symptoms Epub 2012 Aug 29.
Demyelinating Disorders
157 5
?? 19. The following scenario is part 3 of a zz Suggested Reading
7-question series, Discussion & Ref- 55 Dalmau et al. Lancet Neurol. 2011
erences included at the end of series. Jan;10(1):63–74. 7 https://doi.
You see a suggestive pattern on org/10.1016/S1474-4422(10)70253-2;
EEG. Titulaer MJ et al. Lancet Neurol. 2013 Feb;
What test might be helpful? 12(2):157–65. 7 https://doi.org/10.1016/
A. Rheumatoid factor S1474-4422(12)70310-1. Epub 2013 Jan 3.
B. Anti-TTG
C. NMO IgG ?? 21. T
 he following scenario is part 5 of
D. NMDAR antibodies a 7-question series, Discussion &
E. Anti-gliadin antibodies References included at the end of
F. Oligo clonal bands series.  
Which ancillary testing should you
vv Correct Answer is: D ordered?
NMDAR antibodies may be helpful in A. Pulmonary function tests
this case, although the testing should B. Abdominal ultrasound
not be limited to this. The story suggests C. Ultrasound of the pelvis
the presence of an autoimmune enceph- D. Chest CT
alitis, and the behavioral changes sug- E. Thyroid scan
gest the possibility of a limbic
encephalitis. This story, combined with vv Correct Answer is: C
extreme delta brush on EEG, suggests NMDAR encephalitis was first described
the need for testing for NMDAR antibod- in relation to ovarian teratoma (Dalmau
ies. The other antibody/rheumatologic 2004); while the likelihood of teratoma
tests listed (RF, anti-TTG, Anti gliadin ab, is lower in the pediatric population (6%
NMO-­IgG) have not been associated of one large series), resection is curative.
with autoimmune encephalitis).
zz Suggested Reading
?? 20. T
 he following scenario is part 4 of 55 Tuzun et al. Acta Neuropathol.
a 7-question series, Discussion & 2009;118(6):737–43. 7 https://doi.
References included at the end of org/10.1007/s00401-009-0582-4; Dalmau
series. et al. Lancet Neurol. 2011;10(1):63–74.
The child’s testing is positive. 7 https://doi.org/10.1016/S1474-
What would you use as a first line 4422(10)70253-2.
therapy?
A. Cyclophosphamide ?? 22. T
 he following scenario is part 6 of a
B. IVIG 1 g/kg x 2 days 7-question series, Discussion & Ref-
C. Interferon beta 1a erences included at the end of series.
D. Solumedrol 30 mg/kg/d x 3 days Based on presently available
E. Glatiramer Acetate knowledge.
What will you tell the family about
vv Correct Answer is: D risk of recurrence?
While no clear prospective evidence for A. 20%
therapy for NMDAR encephalitis is avail- B. 10%
able, retrospective analyses suggest C. 30%
that first line therapy with pulse steroids D. 40%
is effective in half of patients. E. 50%
158 Chapter 5 · Demyelinating Disorders

vv Correct Answer is: B B. 50% chance of seizures long term


Risk of recurrence is 10% in large series. C. Ongoing behavioral problems in
greater than half of children
zz Suggested Reading D. Full recovery seen in 75% of
55 Dalmau et al. Lancet Neurol. patients
2011;10(1):63–74. 7 https://doi. E. Recurrent disease is seen in 50%
org/10.1016/S1474-4422(10)70253-2; of children
Titulaer MJ et al. Lancet Neurol.
2013;12(2):157–65. 7 https://doi. vv Correct Answer is: D
org/10.1016/S1474-4422(12)70310-1. Full recovery is seen in 75% of children.
Epub 2013 Jan 3.
5 zz Suggested Reading
?? 23. T
 he following scenario is part 7 of 55 Dalmau et al. Lancet Neurol.
a 7-question series, Discussion & 2011;10(1):63–74. 7 https://doi.
References included at the end of org/10.1016/S1474-4422(10)70253-2.
series.
The family asks you whether ?? 24. A 12-year-old boy is complaining of
recovery is expected. double vision for 1 day. When you
What would you tell the family? examine him you find internuclear
A. 70% of children suffer from irre- opthalmolplegia. An MRI is per-
versible cognitive impairment formed.

FLAIR (3T) Phase (7T)

..      Figs. 5.12 and 5.13 MS. FLAIR (3 T) and GRE phase Li S, Swamy E, et al. (2013) Iron Is a Sensitive Biomarker for
(7 T) images of a patient with active relapsing-remitting Inflammation in Multiple Sclerosis Lesions. PLoS ONE 8(3):
MS.FLAIR images show numerous white matter MS lesions e57573. 7 https://doi.org/10.1371/journal.pone.0057573.
of which 2 are magnified (inset, red arrows). Phase imag- 7 http://www.plosone.org/article/info:doi/10.1371/jour-
ing at 7 T phase/GRE reveals a hypointense ring corre- nal.pone.0057573#pone-0057573-g005. Veela Mehta, Wei
sponding with one lesion on FLAIR. The other lesion is not Pei, Grant Yang, Suyang Li, Eashwar Swamy, Aaron Boster,
visible on 7 T GRE (inset, arrows). (Mehta V, Pei W, Yang G, Petra Schmalbrock, David Pitt)
Demyelinating Disorders
159 5
Given the T2 FLAIR Axial and presentation of acute disseminated
Sagittal MRI brain images below, what encephalomyelitis.
will allow you to make the diagnosis 1. MS Diagnosis, from Krupp et al. MS
of MS? Journal 2013. Two or more nonen-
A. 3 more lesions in the brain cephalopathic (e.g. not ADEM-like),
B. A lesion in the spinal cord and a clinical CNS events with presumed
gadolinium enhancing lesion inflammatory cause, separated by
C. One gadolinium enhancing lesion more than 30 days and involving
and one hypo intense lesion on T1 more than one area of the CNS.
weighted imaging 2. One nonencephalopathic episode
D. White blood cells in the CSF and typical of MS which is associated with
black hole MRI findings consistent with 2010
E. Increased opening pressure on LP Revised McDonald criteria for DIS
and black hole and in which a follow up MRI shows
at least one new enhancing or non-
vv Correct Answer is: C enhancing lesion ­consistent with dis-
Current diagnostic criteria for MS, the semination in time (DIT) MS criteria
revised McDonald Criteria (reference 3. One ADEM attack followed by a non-
Polman et al. Ann Neurol. 2011 Feb; encephalopathic clinical event, three
69(2):292–302. 7 https://doi. or more months after symptom onset
org/10.1002/ana.22366), are acceptable that is associated with new MRI
for use in pediatric cases of multiple lesions that fulfill 2010 Revised
sclerosis over 12 years of age (Krupp McDonald DIS criteria
et al. Multiple Sclerosis Journal 2013 4. A first, single, acute event that does
7 https://doi. not meet ADEM criteria and whose
org/10.1177/1352458513484547). MRI findings are consistent with the
These criteria allow for the MRI diag- 2010 Revised McDonald criteria for
nosis of multiple sclerosis at first clini- DIS and DIT (applies only to children
cal event, as long as there is evidence 12 years or older)
for dissemination in space and time
on the MRI scan in children older than zz Suggested Reading
12 years of age. The 2010 revision of the 55 MS Diagnosis, from Krupp et al. MS J. 2013.
McDonald Criteria specifies that one
new enhancing or non-enhancing lesion ?? 25. A 12-year-old boy woke up this morn-
be seen in order to make a diagnosis of ing noticing that he could not see out
MS. of his left eye. He complains of blurry
In this case, the boy has had one vision in the left eye and rubbing
clinical attack and would need MRI evi- the eye or squinting did not make it
dence for dissemination in time. This better. He had no pain in the eye. He
would be fulfilled with another lesion has no medical history of previous
which was enhancing on T1 weighted trauma, fever, respiratory symptoms,
imaging or evidence for an old lesion, as or headache. The mother made an
represented by a hypo intense lesion on appointment with an optometrist
T1 weighted imaging (“black hole”). for him to be seen today. While he is
The following are specific pediatric waiting to be seen, the mother had to
diagnostic criteria for MS. Note the take him to the ED because the boy
importance of having more clinical complained of mid-back pain and
episodes in the young child with a first rapidly developed difficulty urinating,
160 Chapter 5 · Demyelinating Disorders

stiffness of his legs, and tingly sensa- In the past, NMO was considered to
tion when he ran his hand over his be a severe variant of multiple sclerosis
abdomen below his belly button. ED (MS) because both can cause attacks
attending admitted him for evalua- of optic neuritis and myelitis. Recent
tion and treatment.   discoveries, however, suggest it is a
The attending most likely checked separate disease. NMO is different from
him for: MS in the severity of its attacks and
A. Anti-NMDA receptor antibodies its tendency to solely strike the optic
B. Anti-microsomal antibodies nerves and spinal cord at the beginning
C. Anti-thyroid antibodies of the disease. Symptoms outside of the
5 D. Anti-aquaporin 4 antibodies
E. Anti-neuronal antibodies
optic nerves and spinal cord are rare,
although certain symptoms, including
uncontrollable vomiting and hiccups,
vv Correct Answer is: D are now recognized as relatively specific
Neuromyelitis optica (NMO) is an symptoms of NMO that are due to brain-
uncommon disease syndrome of the stem involvement.
central nervous system (CNS) that The recent discovery of the anti-­
affects the optic nerves and spinal cord. aquaporin 4 antibody in many individu-
Individuals with NMO develop optic als with NMO gives doctors a reliable
neuritis and transverse myelitis. NMO biomarker to distinguish NMO from
leads to loss of myelin. MS. The antibody, known as NMO-IgG,
Historically, NMO was diagnosed seems to be present in about 70 percent
in patients who experienced a rapid of those with NMO and is not found in
onset of blindness in one or both eyes, people with MS or other similar condi-
followed within days or weeks by vary- tions.
ing degrees of paralysis in the arms
and legs. In most cases, however, the zz Suggested Reading
interval between optic neuritis and 55 Wingerchuk DM, Weinshenker
transverse myelitis is significantly lon- BG. Neuromyelitis optica (Devic’s syn-
ger, sometimes as long as several years. drome). Handb Clin Neurol. 2014;122:581
After the initial attack, NMO follows an
unpredictable course. Most individuals ?? 26. A family of previously healthy 6-year-
with the ­syndrome experience clus- old boy brought him to the ED
ters of attacks months or years apart, because he cannot walk. For the last
followed by partial recovery during few days he has developed progres-
periods of remission. This relapsing sive weakness of both legs, ataxia,
form of NMO primarily affects women. left facial weakness, blurred vision,
The female to male ratio is greater than lethargy, and an epileptic seizure. He
4:1. Another form of NMO, in which has no medical history of any sick-
an individual only has a single, severe ness. He was seen by his pediatrician
attack extending over a month or two, 10 days ago and had his routine vac-
is most likely a distinct disease that cinations. On examination, he is som-
affects men and women with equal nolent, and cannot sit up in bed. He
frequency. The onset of NMO varies has brisk reflexes in all 4 extremities
from childhood to adulthood, with two with an up-going Babinski response
peaks, one in childhood and the other bilaterally. MRI scan of the brain is
in adults in their 40s. shown.  
Demyelinating Disorders
161 5

..      Figs. 5.14 and 5.15 MRI showing white matter changes indicating ADEM disease. American Academy of
Neurology Institute, produced by permission

What would be the most reasonable the responsible antibody (not typically
treatment for him at that time? known) via plasmapheresis. Most cases
A. Albendazole are monophasic, although 11% can be
B. Beta-interferon multiphasic over time. Beta-interferon
C. Methylprednisolone has been used anecdotally for recurrent
D. Pyridostigmine ADEM but not for acute monophasic
E. Temozolomide ADEM. Pyridostigmine, which acts at the
neuromuscular junction, is used in the
vv Correct Answer is: C treatment of myasthenia gravis, and
The presentation is consistent with have no use in ADEM. Albendazole is a
acute disseminated encephalomyelitis benzimidazole used to treat cysticerco-
(ADEM), supported by the clinical and sis and other parasitic infections, not
radiographic findings. ADEM is an ADEM. Temozolomide is an alkylating
immune-mediated, demyelinating dis- chemotherapy drug used to treat glio-
ease of the CNS that presents typically blastomas and other tumors, not ADEM.
in children, often following a defined
viral illness or vaccination. Although zz Suggested Reading
there are no randomized, controlled 55 Tenembaum S, Chitnis T, Ness J, Hahn
treatment trials for ADEM, standard clin- JS. Acute disseminated encephalomyelitis.
ical practice involves some type of Neurology. 2007;68(16 suppl 2):S23–36.
immunosuppresant therapy, such as 55 Chitnis T. Pediatric demyelinating
high dose steroids or intravenous immu- diseases. Continuum (Minneap Minn).
noglobulin, or attempts at removal of 2013;19(4):1023–45.
162 Chapter 5 · Demyelinating Disorders

?? 27. You are seeing a 9-year-old girl in maternal aunt has multiple sclerosis.
the PICU for mental status changes On examination, she is febrile with
and worsening headache for the temperature of 38.5 °C; pulse 90/min;
last 2 days. Her mother reports that respirations 20/min; blood pressure
she has been increasingly lethargic 110/65. She is asleep but when you
and confused with a diffuse continu- stimulate her vigorously she will open
ous headache which improves only her eyes. She will not speak nor follow
slightly with acetaminophen. Her commands. Cranial nerves are intact
mother also report that she is weak, with no papilledema. Movements are
unable to sit or stand on her own. symmetric but diffusely weak. DTR are
5 She has a flu-like illness 2 weeks ago.
Her immunization is not up-to-date.
3+ and symmetric with flexor plantar
responses. Brain MRI is shown. CSF
She came to the US with her family shows 8 lymphocytes/mm 3, protein
from South America 2 years ago. Her 65 mg/dL, glucose 70 mg/dL.  

..      Figs. 5.16 and 5.17 MRI showing white matter changes indicating ADEM disease. American Academy of
Neurology Institute, produced by permission

What is the most likely diagnosis? vv Correct Answer is: A


A. acute disseminated encephalomy- This girl has an acute encephalopathy
elitis following a predated illness. The clinical
B. herpes encephalitis presentation in association of CSF and
C. multiple sclerosis MRI findings is most indicative of acute
D. neurocysticercosis disseminated encephalomyelitis
E. subacute sclerosing panencepha- (ADEM). Herpes encephalitis has a pre-
litis dilection for the temporal lobes, which
Demyelinating Disorders
163 5
does not show up on MRI. Because this 2 days to 4 weeks after a predated ill-
is her first episode of neurologic symp- ness. Approximately 70% to 77% of
tomatology makes multiple sclerosis patients report a clinically evident pre-
(MS) an inappropriate diagnosis. dated infection or vaccination during
Distinguishing between patients with the prior few weeks. The typical symp-
ADEM and those with a first demyelinat- toms and signs of ADEM include a rapid
ing episode which eventually becomes onset encephalopathy associated with
MS can be very difficult. Studies of chil- a combination of multifocal neurologic
dren after a first episode of demyelin- deficits. A prodromal phase with fever,
ation have identified disease onset in malaise, headache, nausea and vomit-
adolescence, intrathecal oligoclonal ing may be observed shortly before the
bands and optic neuritis as associated development of meningeal signs and
with a higher MS risk; whereas prepu- drowsiness. The clinical course is rap-
bertal onset, presence of multifocal fea- idly progressive and usually develops
tures with encephalopathy and over hours to maximum deficits within
transverse myelitis have been associ- days (mean: 4.5 days). There is a wide
ated with a lower risk of subsequent variation in the severity of the illness.
MS. Although she is from South America Occasionally ADEM can present as a
where parasitic diseases like neurocysti- subtle disease with nonspecific irritabil-
cercosis are common, most such ity, headache and somnolence, or may
patients present with seizures and the show a rapid progression of symptoms
MRI finding is not supportive. Despite and signs to coma and decerebrate
not being immunized against measles rigidity. Respiratory failure secondary
the likelihood of subacute sclerosing to brainstem involvement or severely
panencephalitis is much lower than impaired consciousness occurs in 11%
ADEM. to 16% of cases.
Acute disseminated encephalomy- Neuroimaging is extremely impor-
elitis (ADEM) is an immune-mediated tant in establishing the diagnosis of
inflammatory disorder of the CNS, com- ADEM. MRI abnormalities are most
monly preceded by an infection and frequently identified on T2-weighted
mainly affecting the brain and spinal and fluid-attenuated inversion recovery
cord white matter. In the absence of (FLAIR) sequences as patchy, poorly
specific biologic markers the ­diagnosis marginated areas of increased signal
of ADEM is based on the clinical and intensity. Lesions in ADEM are typi-
radiologic features. Although ADEM cally large, multiple and asymmetric.
usually has a monophasic course They typically involve the subcortical
recurrent or multiphasic forms have and central white matter and cortical
been reported, raising diagnostic dif- gray-white junction of both cerebral
ficulties in distinguishing these cases hemispheres, cerebellum, brainstem
from multiple sclerosis (MS). ADEM can and spinal cord. The gray matter of the
occur at any age but it is more common thalami and basal ganglia are frequently
in pediatric patients than in adults. involved, typically in a symmetric pat-
A seasonal distribution in the winter tern. The periventricular white mat-
and spring months has been found in ter is also frequently involved, being
studies conducted in the US. ADEM is described in 30% to 60% of cases.
classically described as a monophasic Lesions confined to the corpus callosum
disorder which typically begins within are less common.
164 Chapter 5 · Demyelinating Disorders

There is no standard therapy for ?? 28. You are seeing a 16-year-old boy in
ADEM. Most treatment approaches have the ED with a history of headache,
employed some form of nonspecific difficulty walking and numbness and
immunosuppressant therapy similar to paresthesia’s in both arms for the
that used for MS and other autoimmune last 3 days. His family states he has
diseases, including steroids, IV immuno- also been increasingly confused and
globulin (IVIG) or plasmapheresis. Most lethargic over the past 24 h. He has
of the data describing treatment for some mild upper respiratory viral
ADEM are derived from case reports and infection 2 weeks ago, otherwise his
small series. To date there have been past medical history is unremarkable.
5 no randomized, controlled trials for the
treatment of ADEM in either children or
He is difficult to arouse and unable to
follow commands when you examined
adults. him. He has spastic paraparesis of
both legs, brisk deep tendon reflexes
zz Suggested Reading and up going toes bilaterally. He has a
55 Tenembaum S, Chitnis T, Ness J, et al. generalized tonic-clonic seizure lasting
Acute disseminated encephalomyelitis. 5 min immediately after you finished
Neurology. 2007;68(16 Suppl 2): S23–36. his exam. Head CT is normal. CSF anal-
55 Dale RC, Brilot F, Banwell B. Pediatric ysis shows 50 white blood cells with a
central nervous system inflammatory protein level of 95 ug/dL. Brain MRI is
demyelination: acute disseminated shown. You started him on 5 days of
encephalomyelitis, clinically isolated high dose intravenous solumedrol. A
syndromes, neuromyelitis optica, and follow up MRI is performed 4 weeks
multiple sclerosis. Curr Opin Neurol. later, at which point he has fully recov-
2009;22(3):233–40. ered back to normal.

..      Figs. 5.18 and 5.19 ADEM, white matter changes on MRI. American Academy of Neurology Institute,
produced by permission
Demyelinating Disorders
165 5
What is the most likely diagnosis for (though this has rarely been reported),
this patient? nor would near complete resolution of
A. Acute Disseminated Encephalo- the radiographic features be expected
myelitis with these progressive and degerative
B. Adrenoleukodystrophy disorders of cerebral white matter.
C. Metachromatic leukodystrophy PML (due to proposed reactivation
D. Multiple sclerosis of the JC virus in the central nervous
E. Progressive multifocal leukoen- system) would not be expected to occur
cephalopathy (PML) in a young, healthy, immune-compe-
tent patient, nor would the lesions be
vv Correct Answer is: A expected to resolve so quickly.
The patient manifests clinical features of
upper motor type weakness and enceph- zz Suggested Reading
alopathy. Additionally, noted are non- 55 Wingerchuk D, Weinshenker B. Acute
contrast enhancing diffuse confluent disseminated encephalomyelitis, trans-
periventricual white matter lesions on verse myelitis, and neuromyelitis optica.
MRI. Given the acute/sub-acute onset, Continuum Lifelong Learning Neurol.
acute disseminated encephalomyelitis 2013;19(4):944.
(ADEM) should be strongly considered. 55 Rezai MS, et al. Acute disseminated
There is no one diagnostic test to sub- encephalomyelitis: a case series and review
stantiate the diagnosis, however, and the of literatures. J Pediatr Rev. 2013;1(2):88.
patients clinical and radiographic exami-
nation must be followed closely to ensure ?? 29. You are re-evaluating an 18-year-old
marked improvement if not resolution female for peripheral demyelinat-
has occurred. This condition is often “pro- ing neuropathy, who you saw her
voked” by a preceding illness or immuni- 6 weeks in the hospital. 2 weeks
zation, and is often treatable with ago, she started to have progres-
steroids or intravenous immune globulin sive weakness and paresthesias in
(IVIG). It is typically a monophasic illness, the distal portions of her both lower
although the consensus criteria by the extremities. CSF had been done and
Pediatric Multiple Sclerosis Study Group revealed no cells with an elevated
allows for recurrence of symptoms within protein level. You diagnosed her
3 months of the initial illness. The lesions with an acquired demyelinating
on the MRI typically resolve on follow up polyradiculoneuropathy, treated and
imaging after recovery. responded to intravenous immuno-
The patient does not present with globulin and a diagnosis of Guillain-
the necessary dissemination in time Barre syndrome on discharge. She
criteria (though dissemination in improved clinically after discharge
space may be present if enough of the 4 weeks ago, but not fully. In the last
neuroaxis is imaged) for a diagnosis 2 weeks she has an increase in her
of multiple sclerosis. Additionally, the paresthesias and weakness in her
presence of such degree of encepha- lower legs. When you examined her
lopathy, including a seizure, would be today, she has a new mild weakness
extremely unusual for a presentation of in her upper extremities and are-
multiple sclerosis. flexia throughout all limbs. Her EMG
A leukodystrophy would not be showed a multifocal disturbance of
expected to have such an acute onset conduction velocity.
166 Chapter 5 · Demyelinating Disorders

What is her diagnosis? zz Suggested Reading


A. Recurrent Guillain-­Barre syndrome 55 Sung JY, Tani J, Park SB, Kiernan MC, Lin
B. Chronic demyelinating polyra- CSY. Early identification of ‘acute­onset
diculoneuropathy chronic inflammatory demyelinating
C. Charcot-Marie-Tooth Disease 1 polyneuropathy. Brain. 2014;137(8):2155–63.
D. Adult form of metachromatic leu-
kodystrophy ?? 30. You are evaluating a 10-year-old girl
in the hospital because of progressive
vv Correct Answer is: B weakness. 1 week ago, she started
Guillain-Barre is the acute form of to complain of tingling and muscle
5 acquired demyelinating polyradiculo-
neuropathy and the chronic inflamma-
cramping in her right foot and right
lower leg. She became “clumsy”
tory demyelinating 2 days later and last night she was
polyradiculoneuropathy is the chronic unable to walk up the stairs and she
form. In both forms, EMG and NCV find- has to be carried. Today, she woke
ings show evidence of “patchy” or multi- up and was unable to stand. On your
focal abnormalities in conduction examination, she had flaccid paralysis,
velocity and elevated spinal fluid protein. impaired sensation and areflexia in
Mandatory diagnostic criteria are; 1) her lower legs, and a left arm drift
progressive or relapsing motor and sen- on arm extension and left-hand
sory dysfunction of more than one limb, weakness. She has a history of “cold”
developing over a minimum of 8 weeks 2 weeks ago.  
and 2) areflexia or hyporeflexia usually What is the most likely diagnosis?
affecting all limbs. A. Myasthenia gravis
Recurrent Guillain-Barre syndrome B. Acute demyelinating polyradicu-
is rare and is estimated to occur in only loneuropathy
1–3% of the cases. C. Polio secondary to immunization
Charcot-Marie-Tooth 1 (CMT1), also D. Heavy metal neuropathy
referred to as hereditary motor and
sensory neuropathy type I (HMSN1), is a vv Correct Answer is: B
demyelinating neuropathy that typically Guillain-Barre syndrome (acute demye-
begins in the second decade of life and linating polyradiculoneuropathy), is an
progresses gradually throughout the acute monophasic demyelinating neu-
patient’s life. The distal nerves in legs ropathy. Majority of patients have an
are affected first causing weakness and antecedent viral infection. An abnormal
foot drop. A uniform slowing of conduc- immune response generates a response
tion velocity characterizes CMT1. targeting peripheral nerve antigens.
Metachromatic leukodystrophy is Insidious sensory symptoms predate
an inherited disorder of myelin metabo- the onset of motor weakness.
lism and is caused by a deficiency of the Myasthenia gravis does not have
enzyme arylsulfatase A. In the childhood sensory symptoms and the presentation
form segmental demyelination occurs is of generalized weakness.
in peripheral nerves and cell death in Polio symptoms following immuni-
central oligodendroglia cells. The adult zation is rare but can occur. The polio-
form accounts for about 20% of affected virus attacks the anterior horn cells
individuals and typically begins in the exclusively and there are not sensory
teenage years with psychiatric symp- disturbances. In addition, the acellular
toms and eventually cognitive decline. polio vaccine is typically administered
Demyelinating Disorders
167 5
as part of the DPT immunizations in zz Suggested Reading
younger childhood. 55 Bosch EP, Smith BE. Disorders of periph-
Heavy metal neuropathy is typically eral nerves. In: Bradley WG, Daroff RB,
more gradual and acute presentations Fenichel GM, jankovic J, editors.
are associated with GI and systemic Neurology in clinical practice. 4th ed.
symptoms such as nausea, vomiting, Elsevier, Inc.; 2004. p. 2299–401.
renal failure or pneumonitis.
?? 32. You are seeing a 14-year-old girl in
zz Suggested Reading the hospital, who developed head-
55 Yuki N, Hartung HP. GuiIlain-Barre ache, fever, visual blurring, trouble
syndrome. N Engl J Med. walking, and paresthesias of her limbs
2012;366(24):2294–304. 2 days ago after she had a mild upper
respiratory tract infection 2 weeks
?? 31. Which bacterial infection is most ago. Her MRI is shown below. On
commonly associated with Guillain- examination, she has bilateral papil-
Barre syndrome? litis, brisk reflexes with some limb
A. Staphylococcus aureus ataxia, and up-going toes. She is con-
B. Streptococcus pneumoniae fused and febrile. CSF shows a mild
C. Enterococci lymphocytic pleocytosis and negative
D. Campylobacter jejuni oligoclonal banding.  
E. Listeria monocytogenes How is this related to multiple
­sclerosis?
vv Correct Answer is: D A. It appears to be a separate neuro-
Majority of Guillain-Barre syndrome immunologic disorder
(GBS) patients have a predated infec- B. It usually transitions into multiple
tion, surgery, or vaccination within the sclerosis
month prior to the onset of symptoms. C. It never transitions into multiple
Campylobacter jejuni is the most sclerosis
common bacterial infection associated D. It is the pediatric equivalent of
with GBS. Estimates vary, with C. jejuni multiple sclerosis
found in 17–42% of patients with GBS. It E. It is the same as multiple sclerosis
is specifically associated with the axonal
variant, acute motor axonal neuropathy vv Correct Answer is: A
(AMAN), with C. jejuni infections evident The clinical and radiological presenta-
in up to 76% of such patients. tion is most consistent with acute dis-
Mycoplasma pneumonia is often seminated encephalomyelitis (ADEM).
seen in children with GBS. GBS has also ADEM is usually a monophasic syndrome
been linked to Hemophilus influenzae. usually predated by an infection or vacci-
Viral infections associated with GBS nation. It is a multifocal demyelinating
include cytomegalovirus, Epstein-Barr syndrome with large lesions that can be
virus, varicella zoster, hepatitis A, hepa- peripheral and may involve the basal
titis B, and HIV. ganglia. CSF may show a pleocytosis, but
Isolated cases have been associated usually does not show oligoclonal bands.
with vaccinations (e.g. influenza, teta- Patients are often febrile and encephalo-
nus, diphtheria, rabies, and oral polio) pathic. It may leave residual deficits.
and drugs (e.g. heroin, streptokinase). First-line treatment for ADEM is intra-
The risk of GBS with influenza vaccina- venous methylprednisolone. All treat-
tion is approximately 1:1,000,000. ment recommendations are anecdotal.
168 Chapter 5 · Demyelinating Disorders

..      Figs. 5.20 and 5.21 ADEM on MRI. MRI showing white matter changes in (ADEM) disease. American Academy
of Neurology Institute, produced by permission

Treatment consists of initial intra- with lymphocytic predominance, a


venous methylprednisolone and if this protein level of 70 mg/dl (normal up
fails, trial of plasmapheresis. There is no to 45 mg/dl) with normal glucose, and
indication for antibacterial or antiviral high immunoglobulin G index. MRI of
agents. the brain shows symmetric subcorti-
cal white matter lesions, and MRI of
zz Suggested Reading the spine shows an expanding intra-
55 Tenembaum 5, Chitnis T, Nessj, et al. medullary lesion between T5 and T10
ADEM. Neurology. 2007; 68:523–536. with gadolinium enhancement.  
Comprehensive review in clinical neurol- What is the most likely diagnosis?
ogy: a multiple-choice question book for A. Multiple sclerosis
the wards and boards ©2011 Wolters B. Acute disseminated encephalo-
Kluwer Health Lippincott Williams & myelitis
Wilkins. All rights reserved. C. Neuromyelitis optica
D. Idiopathic transverse myelitis
?? 33. You are seeing a 14-year-old girl with E. Bacterial meningitis
a history of headache, altered mental
status, inability to move her legs, and vv Correct Answer is: B
urinary incontinence. She had fever The clinical presentation of an acute
and cough 1 week ago. When you neurologic disorder followed a recent
examined her, she was confused, had febrile illness is indicative of an inflam-
no meningeal signs, and had a flaccid matory myelopathy and encephalopa-
paraplegia with areflexia and sensory thy. At ­presentation, the illness was
loss below her waist. You ordered CSF monophasic, with symmetric involve-
analysis which shows 40 WBCs/mm3 ment of cerebral white matter and
(normal up to 5 lymphocytes/mm3) simultaneous spinal cord involvement.
Demyelinating Disorders
169 5
CSF findings support an inflammatory
disorder. The most likely explanation for
her clinical picture is acute dissemi-
nated encephalomyelitis (ADEM).
ADEM is an inflammatory demyelin-
ating disorder of childhood in which
there is a monophasic immunologic reac-
tion to a viral illness. Patients have an
encephalopathy, with clear white matter
changes on MRI and inflammatory mark-
ers in the CSF. Occasionally, the spinal
cord is also affected with features of a
transverse myelitis, in which the lesion
affects more than three segments of the
spinal cord. The treatment is intravenous
steroids in high doses.
Encephalopathy is not a feature of
neuromyelitis optica; the patient may have
some white matter lesions, but in contrast to
ADEM, they are not symmetric or confluent.
Multiple sclerosis is an inflammatory
demyelinating disorder that presents
with a relapsing and remitting or a
progressive course. When it affects the
spinal cord, the lesions are usually in
short segments, not like in this case. ..      Fig. 5.22 EMG showing findings of ( CIDP). MRI
Acute encephalopathy is not common in showing white matter changes in ADEM disorder. By
permission from J Phys Ther Sci. 2013 Oct; 25(10)
multiple sclerosis.
The diagnosis of idiopathic trans-
verse myelitis is made when the patient potential, and slowed conduction have
has a transverse myelitis with no other shown on the EMG. This is consistent
explanation, and usually without other with Chronic Inflammatory
features, which is not the case here. Demyelinating Polyneuropathy (CIDP).
The clinical, imaging, and CSF findings Chronic inflammatory demyelinat-
do not correlate with bacterial meningitis. ing polyradiculoneuropathy usually
starts insidiously and evolves in either
?? 34. Which of the following would be a in a slowly progressive (<60%) or relaps-
suspected diagnosis in the study ing process, with some level of recovery
shown below? between recurrences. Presenting symp-
A. Amyotrophic lateral sclerosis (ALS) toms include motor weakness (most com-
B. Chronic Inflammatory Demyelin- mon), and some have sensory and other
ating Polyneuropathy (CIDP) symptoms such as autonomic complaints.
C. Brachial plexus injury Most literature agrees on a mini-
mum duration of 8 weeks to diag-
vv Correct Answer is: B nose CIDP.80% of CIDP patients have
The EMG shows Conduction block, pro- elevated protein in the CSF, and EMG
longed distal latencies, temporal disper- is used to determine the presence of
sion of compound muscle action demyelinating disease.
170 Chapter 5 · Demyelinating Disorders

?? 35. Acute disseminated encephalomyeli- 55 ADEM may be treated with steroids,


tis (ADEM) differs from multiple scle- immunomodulatory medications, and
rosis (MS) in that: plasmapheresis.
A. There is a lack of oligoclonal
bands in the cerebrospinal fluid of zz Suggested Reading
patients with ADEM Cerebrospinal 55 Olek MJ, Dawson MJ. Multiple sclerosis
fluid immunoglobulin levels are and other inflammatory demyelinating
typically normal in ADEM diseases of the central nervous system. In:
B. Episodes of ADEM do not usually Bradley WG, Daroff RB, Fenichel GM,
recur Jankovic J, editors. Neurology in clinical
practice. 4th ed. Elsevier, Inc.; 2004.
5 C. The development of ADEM has
been linked to recent vaccination p. 1631–64.
D. All of the above
?? 36. You are seeing a 6-year-old boy
vv Correct Answer is: D presents with confusion and left
ADEM is an acute, monophasic, demye- sided weakness. On exam he has
linating process, often predated by hyperreflexia on the left side and
recent viral infections or vaccinations. It the patient being difficult to arouse.
manifests as perivascular inflammation, Brain MRI showed diffuse white mat-
edema, and demyelination of the CNS, ter lesions.  
with the rapid onset of focal or multifo- What is the next step in evaluation
cal neurologic deficits. and management?
The abrupt onset of simultaneous A. Lumbar puncture
bilateral optic neuritis, impaired conscious- B. Methylprednisolone 20 mg/kg/d
ness, meningismus, loss of deep tendon for 5 days
reflexes with intact abdominal reflexes and C. IVIG 2 g/kg spread out between
a positive Babinski sign, elevated tempera- 2–5 days
ture, seizures, and limb pain are the char- D. MRI of the cervical and thoracic spine
acteristic symptoms. Symptoms may be E. Brain Biopsy
preceded by a prodromal of fever, malaise,
and myalgias for several days. vv Correct Answer is: A
ADEM differs from MS in that: The clinical and radiological picture is
55 Patients recover more rapidly from ADEM, most consistent with a diagnosis of
in some cases within days. acute disseminated encephalomyelitis.
55 Recovery is more complete from ADEM, An infectious cause should be ruled out
with total recovery in approximately first so a lumbar puncture should be
50–70% of patients. done to exclude this etiology. Cervical
55 In contrast to MS, follow-up MRI in several and thoracic spine MRI can also be
weeks demonstrates at least partial done, but lumbar puncture should be
resolution of ADEM lesions, without the performed first. If no infectious etiology
formation of new lesions. is noted, treatment with IV methylpred-
55 Episodes of ADEM do not usually recur. nisolone should be started. IVIG is con-
55 There is a lack of oligoclonal bands in the sidered a second line therapy.
cerebrospinal fluid of patients with ADEM.
55 Cerebrospinal fluid immunoglobulin levels zz Suggested Reading
are typically normal in ADEM. 55 Chitnis T. Pediatric demyelinating disease.
55 The development of ADEM has been linked to Continuum (Minneap Minn). 2013;19(4
recent vaccination and a variety of infections. Multiple Sclerosis):1023–45.
171 6

Developmental Disorders

© Springer International Publishing AG, part of Springer Nature 2018


Y. M. Awaad, Absolute Pediatric Neurology, https://doi.org/10.1007/978-3-319-78801-2_6
172 Chapter 6 · Developmental Disorders

?? 1. An 8 year old Duchenne Muscular psychological testing performed


Dystrophy boy in the office for a 3 years ago, his full scale IQ was
follow up visit with his family. The 62. A Vineland Adaptive Behavioral
family showed up their concern that Scales-II (VABS-II) found “Moderately
the child is having difficulty with aca- Low” scores for most domains. On
demics in the 3rd grade. His difficulty examination, head circumference at
is in all subjects. The family would the 10th percentile, and mild speech
like to know whether there can be and motor dyspraxia. Deep tendon
cognitive involvement in a child with reflexes are 3+ and symmetric, and he
DMD. is mildly hypotonic throughout. The
What will be your response? patient is conversant, and describes
A. Intellectual disability is common to you his favorite sport show.
in all children with DMD There are no abnormal movements
6 B. Cognitive involvement is common noted.
in DMD In addition to the underlying
C. DMD children do not present with neurological diagnoses, what is the
Attention Deficit Hyperactivity most appropriate DSM-­5 diagnosis?
Disorder A. Autism Spectrum Disorder
D. The mean IQ in children with DMD B. Mental Retardation-­Mild
is 60 C. Intellectual Disability-­Mild
E. Delayed language development is D. Childhood-onset fluency disorder
uncommon E. Specific Learning Disorder

vv Correct Answer is: B vv Correct Answer is: C


Cognitive involvement is common in As a result of federal statute 111–256
DMD. 30% DMD patients have intellec- (“Rosa’s Law”), “intellectual disability”
tual disability. Eighty five is the average has replaced “mental retardation” in
patient’s IQ, one standard deviation the DSM-5. In addition, the DSM-5 has
below the mean. Performance IQ is less emphasized the need to demonstrate
affected than Verbal IQ, and delayed deficient adaptive skills rather than only
language development is common. IQ when diagnosing intellectual dis-
The prevalence of comorbid neurode- ability.
velopmental /psychiatric disorders is
more prevalent in DMD: 10–15% having zz Suggested Reading
attention deficit hyperactivity disorder, 55 Greenspan S, Woods GW. Intellectual
3–6% having Autism spectrum Disorder, disability as a disorder of reasoning and
and 5% having obsessive compulsive judgement: the gradual move away from
disorder. intelligence quotient-­ceilings. Curr Opin
Psychiatry. 2014;27(2):110–6. 7 https://
zz Suggested Reading doi.org/10.1097/YCO.0000000000000037.
55 Muscle disease. Continuum. 2013;19(6). 55 Harris JC: New terminology for mental
retardation in DSM-5 and ICD-11. Curr
?? 2. A 16-year-old male has been evalu- Opin Psychiatry. 2013;26:260–62.
ated by you. He has been receiving 7 https://doi.org/10.1097/
special education services throughout YCO.0b013e32835fd6fb.
his school years, currently main- 55 Papazoglou A, Jacobson LA, McCabe M,
tained in a self-­contained classroom. Kaufmann W, Zabel TA. To ID or not to
You reviewed his school records, ID? Changes in classification rates of
Developmental Disorders
173 6
intellectual disability Using DSM-5. Developmental Disorder-Autistic
Intellect Dev Disabil. 2014;52(3):165–74. Disorder) associated with no expres-
7 https://doi.org/10.1352/1934-9556- sive language and impaired receptive
52.3.165. language abilities, intellectual dis-
55 Public Law. 111–256, §2(b) (2), 5 Oct ability, and epilepsy, with focal onset
2010, 124 Stat. 2643 (Rosa’s Law). seizures with evolution to bilateral
convulsive seizures. Her mother men-
?? 3. Which of the following syndromes are tioned that the child has occasional
typically associated with an intellec- episodes of sudden and explosive
tual disability? aggression associated with an identi-
A. Fragile X syndrome fiable antecedent, such as transition-
B. Rett Syndrome ing away from a preferred activity.
C. de Morsier’s syndrome Overall, the patient has displayed
D. Treacher Collins Syndrome neurodevelopmental regression. In
spite of receiving lamotrigine, her
vv Correct Answer is: A, B, and C seizures occurring more than once
Fragile X is the leading inherited cause monthly. She was on Valproic acid
of intellectual disability in boys. Rett 5 years ago but she had elevations in
syndrome is a common X-linked cause liver enzymes and she had to stop of
of syndromic intellectual disability. the medication. On examination, she
Intellectual disability is a common find- has ataxia and occasional myoclonic
ing in de Morsier’s syndrome (septo- movements.
optic dysplasia) and can occur in 50% of Which of the following tests is most
the patients. likely to provide a definitive
Treacher Collins Syndrome is a biological diagnosis?
syndrome involving the development of A. Neuronal Ceroid Lipofuscinosis
the facial bones. Those affected have (NCL) testing
normal intelligence. B. Random lactate level
C. Electromyography and nerve con-
zz Suggested Reading duction velocities
55 Mefford, H. et al. Genomics, intellectual D. Mitochondrial genome sequenc-
disability, and autism. N Engl J Med. ing, deletion detection, and het-
2012;366:733–43. eroplasmy analysis, and exome
55 Signorini et al. Septo-optic dysplasia in DNA sequencing of nuclear mito-
childhood: the neurological, cognitive and chondrial genes
neuro-ophthalmological perspective. Dev E. Comparative Genomic Hybridiza-
Med Child Neurol. 2012;54(11):1018–24. tion Microarray analysis
7 https://doi. F. Karyotype with molecular Fragile
org/10.1111/j.1469-8749.2012.04404.x. X analysis
Epub 2012 Aug 27.
55 Treacher Collins syndrome. 7 http://ghr. vv Correct Answer is: C
nlm.nih.gov/condition/treacher-collins- The clinical presentation indicates a
syndrome. mitochondrial disorder. Mitochondrial
disorders can presented with non-­
?? 4. A 10-year-old female was evaluated in mitochondrial disorders symptoms and
your office with a previous diagnosis signs. The history of autism, develop-
of Autism Spectrum Disorder-Moder- mental regression, intellectual disability,
ate/Severe (under DSM-IV, Pervasive intractable epilepsy, valproate liver toxic-
174 Chapter 6 · Developmental Disorders

ity associated with myoclonus and ataxia 55 Parikh S, Goldstein A, Koenig MK, et al.
is highly suggestive of a mitochondrial Practice patterns of mitochondrial disease
disorder involving pathological gene physicians in North America. Part 1:
variations of POLG. Diagnostic and clinical challenges.
Classically, mitochondrial pheno- Mitochondriaon. 2014;14:26–33. 7 https://
types can include various combina- doi.org/10.1016/j.mito.2013.07.116. Epub
tions of neurodevelopmental delay or 2013 Jul 26.
regression, seizures/epilepsy, ptosis/ 55 Srivastava S, Cohen JS, Vernon H,
external ophthalmoplegia, hearing Barañano K, McClellan R, Jamal L, Naidu
loss, cardiomyopathy or conduction S, Fatemi A. Clinical Whole Exome
defects, hepatopathy, myopathy, Sequencing In Child Neurology Practice.
neuropathy, movement disorder/ Ann Neurol. 2014;76:473–83. 7 https://
ataxia, and/or progressive optic doi.org/10.1002/ana.24251. [Epub 2014
6 atrophy or retinitis pigmentosa. It is Aug 30].
difficult to diagnosis a mitochondrial
disorder, also neuroimaging, labora- ?? 5. You were called to evaluate a
tory and tissue biochemical studies 6-year-old boy with Autism who
often are non-diagnostic. However, the was admitted to the hospital for gait
identification of mitochondrial abnormality. His family reported that
disorders has advanced along with the he has refused to walk for several
growth in detection of mitochondrial days. He also is more irritable. He
regulatory genes, and the growing was eating his typical diet of French
availability of comparative genomic fries, hamburger patty and artificially
hybridization microarray analysis, flavored water. On examination, he
targeted DNA exome sequencing and is very thin. His gums start bleeding
enzymology. after you look into his mouth. Also
he has bruising on lower legs and
zz Suggested Reading his buttocks. His left leg is tender to
55 Cohen BH, Chinnery PF, Copeland ­palpation. CBC was ordered which
WC. POLG-related disorders. In: Pagon showed anemia. An XRAY of his lower
RA, Adam MP, Ardinger HH, et al. editors. legs reveal ground glass osteopenia.
GeneReviews [Internet]; Seattle: What is the most likely etiology for
University of Washington, Seattle; 1993– the above symptoms?
2014. Initial Posting: March 16, 2010; Last A. Lyme disease
Update: Oct 11, 2012. B. Vitamin C deficiency
55 Cohen BH, Naviaux RK. The clinical C. Celiac disease
diagnosis of POLG disease and other D. Lead poisoning
mitochondrial DNA depletion disorders. E. Non accidental trauma
Methods 2010;51:364–73.
55 Legido A, Jethva R, Goldenthal vv Correct Answer is: B
MJ. Mitochondrial dysfunction in This a clinical presentation of scurvy. It
autism. Semin Pediatr Neurol. is an ancient disease that has become
2013;20:163–75. more common in developmentally
55 Lu JT, Campeau PM, Lee BH. Genotype-­ delayed children with restricted
phenotype correlation-Promiscuity in the diets. Nowadays Scurvy is usually
era of next-generation sequencing. N Engl found among the elderly, the poor,
M Med. 2014;371:593–96. alcoholics, and those restricted to
Developmental Disorders
175 6
macrobiotic diets or other diets that poorer responses in the management of
contain < 10 mg/day of vitamin C. Clini- aggression, hyperactivity, impulsivity,
cal presentations of scurvy are small and self-injurious behaviors. Risperi-
hemorrhages, abnormal mitochondrial done is the only other neuroleptic spe-
metabolism, and abnormal collagen cifically FDA approved for use in autism
synthesis and may manifest as bleeding spectrum disorders.
gums and hemarthroses, arthralgias, The other medications can be useful
malaise, and weakness. in the treatment of ASD, but none have
In reviewing the literatures, 23 case been FDA approved.
reports of scurvy in children with It is important to determine if there
behaviorally restricted diets, including is a biological cause or contributor to
2 autistic children and another with an the adverse behavior before using
unspecified behavioral problem, 1 with medications for maladaptive behaviors
trisomy 21, 1 with static encephalopa- in ASD children. Functional behavior
thy after pertussis-­associated encepha- analysis (FBA) has to be done to
litis, 2 with cerebral palsy, 4 with determine if there is an identifiable
mental retardation, and 5 with global functional cause maintaining maladap-
developmental delay; 7 otherwise tive behaviors (e.g. attention, escape
reportedly normal children, including 1 from demands) or if the behavior is
morbidly obese teenager and another automatically maintained (i.e. biologi-
with a pathologic grief reaction, had cally driven). The assessment and
scurvy in association with unusual treatment team for those patients with
diets and isolated food-avoidant maladaptive behavior patterns should
behaviors. include behavioral specialists who can
perform FBAs. If a biological cause or
zz Suggested Reading contributor to maladaptive behavior is
55 Noble, JM, et al. Scurvy and rickets identified (for example, a tooth
masked by chronic neurologic illness: abscess), treatment should be directed
revisiting psychologic malnutrition. to that source. If the behavior is
Pediatrics. 2007;119(3):e783–90. determined to be mainly functional,
behavioral management (i.e. applied
?? 6. Which medication from the list behavior analysis and related methods)
below has been approved by the should be instituted and the response
FDA to treat an autistic child having monitored. If behavioral interventions
“­irritability”? are not enough to control the behavior,
A. Aripriprazole medications can be added. Alpha-2
B. Clonidine adrenergic agonists, psychostimulants,
C. Quetiapine antiepileptic drugs, anxiolytics/
D. Methylphenidate antidepressants, and beta blockers can
E. Oxcarbazepine be useful in select patients with
maladaptive behavior patterns,
vv Correct Answer is: A although not approved for ADS. Other
In two double blind, placebo controlled medication classes or nutritional
studies, Aripiprazole was found to be supplements may be indicated if
efficacious, safe and well tolerated in specific ion channelopathy or mitochon-
long-term extension studies. However, drial disorder/dysfunction, have been
there are many reports indicating ­identified as the cause.
176 Chapter 6 · Developmental Disorders

zz Suggested Reading least two domains (speech/language,


55 Marcus RN, Owen R, Manos G, et al. social, fine motor or gross motor)
Safety and tolerability of aripiprazole for which are 2 SD below the mean in com-
irritability in pediatric patients with parison to their age related peers, he/
autistic disorder: a 52-week, open-label, she will be considered having global
multicenter study. J Clin Psychiatry. developmental delays. It that Genetic
2011;72:1270–76. testing is recommended to be done in
55 Minshawi NF, Hurwitz S, Morriss D, this case. Microarray is the genetic test
McDougle CJ. Multidisciplinary assess- with the highest diagnostic yield in the
ment and treatment of self-injurious child with unexplained GDD/ID. Karyo-
behavior in autism spectrum disorder and typing and FISH testing have a lower
intellectual disability: Integration of sensitivity. Fragile X may also be con-
psychological and biological theory and sidered in a male with GDD/ID. MECP2
6 approach. J Autism Dev Disord. 2014. may also be considered in a female
[Epub ahead of print]. with GDD.
55 Mintz M, Allesandri M, Curatolo
P. Treatment approaches for autistic zz Suggested Reading
spectrum disorders. In: Tuchman R, 55 Michelson DJ et al. Evidence report:
Rapin I, editors. Autism: A neurological genetic and metabolic testing on children
disorder of early brain development, with global developmental delays: Report
International Child Neurology of the Quality Standards Subcommittee of
Association. London: Mac Keith Press; the American Academy of Neurology and
2006. p. 281–307. the Practice Committee of the Child
Neurology Society. Neurology.
?? 7. A 22 months child came with his 2011;77:1629–35.
mother because he is not walking yet.
On your examination, he is babbling ?? 8. Angelman syndrome (AS) is a neuro-
only vowel sounds, has a raking grasp, developmental disorder characterized
does not have a pincer grasp, and by severe mental retardation, absent
sits up but does not crawl. He cannot speech, ataxia, sociable affect, and
build a tower of two blocks. His brain dysmorphic facial features.
MRI is normal. If genetic testing for the maternal
Which genetic test has the highest deletion in the 15q11.2-q13 critical
diagnostic yield in a child with region was negative, what genetic
unexplained global developmental test should be considered next?
delay/intellectual disability? A. FMR
A. MECP2 B. UBE3A
B. Karyotype C. ARX
C. Current generation oligonu- D. LIS
cleotide-based chromosome E. TSC
­microarray
D. Fragile X vv Correct Answer is: B
E. FISH Angelman syndrome is a neurodevel-
opmental disorder characterized by
vv Correct Answer is: C mental retardation, movement or bal-
The clinical presentation is for a global ance disorder, typical abnormal behav-
developmental delays. If the child is iors, and severe limitations in speech
below the age of 6 has delays in at and language. Most cases are caused
Developmental Disorders
177 6
by absence of a maternal contribution vv Correct Answer is: C
to the imprinted region on chromo- This a clinical presentation of scurvy. It
some 15q11-q13. Approximately 70% is an ancient disease that has become
of AS cases result from de novo mater- more common in developmentally
nal deletions involving chromosome delayed children with restricted
15q11.2-q13; approximately 2% result diets. Nowadays Scurvy is usually
from paternal uniparental disomy of found among the elderly, the poor,
15q11.2-q13; and 2 to 3% result from alcoholics, and those restricted to
imprinting defects. A subset of the macrobiotic diets or other diets that
remaining 25% is caused by mutations contain < 10 mg/day of vitamin C. Clini-
in the gene encoding the ubiquitin- cal presentations of scurvy are small
protein ligase E3A gene (UBE3A). Sev- hemorrhages, abnormal mitochondrial
eral patients with a clinical diagnosis metabolism, and abnormal collagen
of Angelman-like syndrome have been synthesis and may manifest as bleeding
reported with mutations in the MECP2 gums and hemarthroses, arthralgias,
gene. malaise, and weakness.
In reviewing the literatures, 23 case
zz Suggested Reading reports of scurvy in children with
55 7 www.omim.org/entry105830. behaviorally restricted diets, including
2 autistic children and another with an
?? 9. You have admitted your 6-year-old unspecified behavioral problem, 1 with
patient with Autism to the hospital trisomy 21, 1 with static encephalopa-
due to gait abnormality. His parents thy after pertussis-­associated encepha-
state that he has refused to walk litis, 2 with cerebral palsy, 4 with mental
for several days. He also is more irri- retardation, and 5 with global develop-
table. His parents state that he has mental delay; 7 otherwise reportedly
not been eating very much. In fact, normal children, including 1 morbidly
his parents state for the past several obese teenager and another with a
months his food aversion has wors- pathologic grief reaction, had scurvy in
ened; he only eats chicken nuggets association with unusual diets and
and drinks only water. On examina- isolated food-avoidant behaviors.
tion you note he is very thin. His
gums start bleeding after you look zz Suggested Reading
into his mouth. You also note bruis- 55 Noble, JM, et al. Scurvy and rickets
ing on lower legs and his buttocks. masked by chronic neurologic illness:
His right leg is tender to palpation. revisiting psychologic malnutrition.
You order a CBC which revealed ane- Pediatrics. 2007;119(3):e783–90.
mia. A BMP was negative. An XRAY
of his lower legs reveal ground glass ?? 10. Parents of a 7-year-­old boy with
osteopenia. Duchenne Muscular Dystrophy bring
A deficiency in which Vitamin their child in for a follow up visit. They
should be considered in this child are concerned that their child is hav-
with restricted intake of foods? ing difficulty with academics in the
A. Vitamin A 2nd grade. He is having difficulty with
B. Vitamin B all subjects. The teacher is refusing to
C. Vitamin C recommend psychoeducational test-
D. Vitamin D ing at this time because many chil-
E. Vitamin E dren in her class are having difficulty
178 Chapter 6 · Developmental Disorders

and he is not failing any subjects. Which medication does not increase
The parents would like your opinion the risk of intellectual disability in
on whether there can be cognitive utero exposure?
involvement in a child with DMD. A. Valproic acid
What would be the proper response? B. Phenytoin
A. Cognitive involvement is uncom- C. Isotretinoin
mon in DMD D. Lithium
B. All children with DMD have intel- E. All of the medications increase the
lectual disability risk of intellectual disability in the
C. Attention Deficit Hyperactivity fetus
Disorder is prevalent in DMD
D. The mean IQ in children with DMD vv Correct Answer is: D
is 100 Fetus exposure to Phenytoin can
6 E. Delayed language development is lead to the fetal hydantoin syn-
uncommon drome, which is consisting of facial
dysmorphism, cleft lip and palate,
vv Correct Answer is: C cardiac defects, digital hypoplasia,
Cognitive involvement is common in and nail dysplasia. IQ points could
DMD. Of patients with DMD, 30% have be decreased by Phenytoin expo-
intellectual disability. The average sure. Neural tube defects, Autism
patient’s IQ is 85, one standard devia- and intellectual disability have been
tion below the mean. Verbal IQ is more reported with Valproic acid exposure.
affected than performance IQ, and Isotretinoin has been shown to cause
delayed language development is com- microcephaly, hydrocephalus and
mon. There is a prevalence of comor- intellectual disability. Lithium has
bid neurodevelopmental /psychiatric been shown to cause Ebstein anomaly
disorders is more prevalent in DMD: but not an intellectual disability.
Attention deficit hyperactivity disorder
is 10–15%, Autism spectrum Disorder zz Suggested Reading
3–6%, and obsessive compulsive disor- 55 Boyle RJ. Effects of certain prenatal drugs
der is 5%. on the fetus and newborn. Pediatr Rev.
2002;23:17–24.
zz Suggested Reading 55 American Academy of Pediatrics
55 Muscle disease. Continuum. 2013;19(6). Committee on Drugs. Use of Psychoactive
Medication during Pregnancy and Possible
?? 11. An 18-year-old female came for her Effects on the Fetus and Newborn.
appointment, she mentioned that she Pediatrics 2000;105(4):880–7.
is not pregnant, but she has a concern 55 Meador KJ. Cognitive/behavioral
about her prescribed medications ­teratogenetic effects of antiepileptic
effect on the fetus if she got preg- drugs. Epilepsy Behav. 2007;11(3):
nant. She takes phenytoin for epi- 292–302.
lepsy. Her psychiatrist has prescribed
Valproic acid but has added lithium ?? 12. Epilepsy syndromes can present in
for severe bipolar disorder. She also infancy and childhood with co morbid
states that her dermatologist is con- intellectual disability. Genetic defects
sidering treating her severe acne with have been identified in several syn-
isotretinoin. (Accutane). dromes.
Developmental Disorders
179 6
Which epilepsy syndrome-gene X-linked West syndrome cases (ISSX1
defect pair is incorrect? and ISSX2). ARX (Aristaless-related
A. Severe myoclonic epilepsy of homeobox) is a transcription factor
childhood (SMEI) - SCN1A involved in brain development. It is
B. X-linked West Syndrome – ARX frequently mutated in a spectrum of X
C. Rett syndrome variant with intrac- chromosome-­linked phenotypes
table epilepsy – CDKL5 characterized by intellectual disability
D. Glut1 deficiency – SLC2A1 as their cardinal feature.
E. Early Infantile Epileptic Encepha- CDKL5 mutations are associated
lopathy With Suppression-­burst - with West syndrome, intellectual
MECP2 disability, and ASD, and also with a Rett
syndrome variant that is characterized
vv Correct Answer is: E by infantile spasms and intractable
Many of distinct clinical syndromes of seizures occurring before the appear-
pediatric genetic epilepsy have been ance of the typical features of Rett
described and linked to specific gene syndrome (Hanefeld variant).
defects. Phenotypes may include, in Altered glucose transport to the
addition to epilepsy, variable degrees brain due to mutations in the GLUT1
of intellectual disability, elements of transporter gene SLC2A1 causes
autism spectrum disorders, other psy- epilepsy of variable severity and
chiatric disorders, and motor impair- comorbidities. Classic GLUT1 deficiency
ment. is associated with very low cerebrospi-
Severe myoclonic epilepsy of nal fluid (CSF) glucose concentrations
infancy (SMEI) is characterized by and causes a severe early-onset
early-onset generalized febrile seizures, (<1 year of age) metabolic encepha-
followed by very frequent afebrile lopathy characterized by movement
generalized tonic–clonic seizures disorders, epilepsy, and mental
(GTCS), myoclonic seizures, absences retardation, typically associated with
and focal seizures, with arrest of de novoSLC2A1 mutations.
psychomotor development, intellectual
disability and motor impairment. zz Suggested Reading
Encode Sodium channel subunits, in 55 Pandolfo, M. Pediatric epilepsy genetics.
particular SCN1A, is the suspected gene Curr Opin Neurol. 2013;26(2):137–45.
in this case, which is mutated in 61–87%
of SMEI. ?? 13. A 15-month-old boy migrated with
West syndrome is a common severe his family from Eastern Europe is
infantile epileptic syndrome character- evaluated for microcephaly and
ized by typical brief tonic spasms, a developmental delay. He has 2 other
profoundly abnormal electroencepha- maternal siblings (with different
lography pattern called hypsarrhyth- fathers) who both have mild mental
mia, and intellectual disability. Early retardation, microcephaly, and delay
infantile epileptic encephalopathy in linguistic development, congeni-
with suppression-burst (EIEE) is the tal heart defects and severe growth
earliest infantile epileptic syndrome, retardation. The mother was “vegetar-
often evolving to West syndrome. ian” as a child but currently does not
Mutations in the X chromosome genes practice any dietary restrictions. She
ARX and CDKL5 are found in some barely graduated high school and has
180 Chapter 6 · Developmental Disorders

difficulty remembering appointments slower compared to the rest of his


and paying bills. classmates and that he may need an
What is the likely explanation for extra year of preschool. The family is
this child’s condition? wondering if he is ready for Kinder-
A. Maternal phenylketonuria garten this coming school year. His
B. Infantile autism physical and neurological exam are
C. Leigh’s disease normal.
D. Chromosome disorder What is your next step in
E. Urea cycle disorder management?
A. EEG
vv Correct Answer is: A B. Standardized neurocognitive test-
Phenylketonuria (PKU) is an autoso- ing
mal recessive disorder of amino acid C. Head MRI
6 metabolism affecting approximately D. Observe
1/5000–10,000 infants in North America. E. Atypical antipsychotic medication
It is most often due to deficiency of
the enzyme phenylalanine hydroxy- vv Correct Answer is: B
lase which causes the accumulation of A child with a history of developmen-
harmful metabolites, including phenylk- tal delay is at risk for learning dis-
etones. If untreated, PKU leads to mental abilities or mental retardation. Mental
retardation, seizures, psychoses, eczema retardation is defined as “significantly
and a distinctive “mousy” odor. Preg- sub-average general intellectual func-
nant PKU women who are not following tioning with an intelligence quotient
dietary restrictions are at risk of deliver- score (IQ) of < 70, existing concurrently
ing infants with the effects of untreated with deficits in adaptive behavior and
maternal PKU and may also be at risk of manifested during the developmental
developing neurologic symptoms them- period, which adversely affects a child’s
selves if they don’t maintain lifelong educational performance”. Stanford
dietary restrictions. Mental retardation, Binet Intelligence Scales-Fifth Edition
microcephaly, intrauterine growth retar- (SB5), Wechsler Intelligence Scales for
dation, and congenital heart defects are children, Wechsler Individual Achieve-
the characteristic features of maternal ment tests and, Mullen scales of early
PKU syndrome. The risk for these defects learning and Wide Range Test for
is increased with maternal plasma phe- Achievement are the cognitive testing
nylalanine levels > 20 mg/dL). can be used for evaluation. Because
the concurrent difficulties in adaptive
zz Suggested Reading behavior. The common questionnaires
55 Bouchlariotou S, Tsikouras P, Maroulis for an adaptive behavior include Vine-
G. Undiagnosed maternal phenylketon- land scales can identify these children
uria: own clinical experience and literature at risk for these disorders and to guide
review. J Matern Fetal Neonatal Med. the school, clinicians and families with
2009;22(10):943–8. regards to his educational needs in
school. AAN guidelines for the assess-
?? 14. A 5-year-old-boy has been in pre- ment of the child with developmental
school for 2 years. The teachers delay might also include MRI and
mentioned to the mother that he is genetic testing.
Developmental Disorders
181 6
zz Suggested Reading is not a universal finding. Neurologic
55 American Psychiatric Association. hallmarks include hypotonia and
Diagnostic and statistical manual of mental global developmental delay, with most
disorders, 4th ed. Washington DC; 1994. affected adults exhibiting mild intellec-
tual disability. Sotos syndrome can be
?? 15. A 5-year-old Caucasian boy in your caused by either intragenic truncating
office for evaluation of developmen- mutations or microdeletions that result
tal delay. He was born at full-term in haplo insufficiency of the nuclear
after an unremarkable pregnancy receptor binding SET domain protein 1
but stayed in the regular nursery for (NSD1), located on chromosome 5q35.3.
2 days secondary to problems with Intragenic mutations account for more
feeding and jaundice. He was 7 lbs. than 80% of cases in European-origin
8 oz. (25–50%ile) and his length was white patients, where’s microdeletions
22 in. (> 95%ile). On examination, he occur with a higher incidence in the Jap-
is tall and macrocephalic (both > 97th anese population. The majority of cases
percentile) with mildly dysmorphic are sporadic, though autosomal domi-
facial features, including a high, broad nant pedigrees have been reported.
forehead, down-slanting palpebral
fissures, and a pointed chin. His has zz Suggested Reading
an enlarged hands and feet. His 55 Leventopoulos G, et al. A clinical study of
neurologic examination is non-­focal. Sotos syndrome patients with review of the
There are no family history of similar literature. Pediatr Neurol. 2009;40:357–64.
affected cases.
Which genetic test is most likely to ?? 16. A 5-year-old boy evaluated by you
yield a positive result? for language delay. The mother
A. Chromosome microarray reported that he said his first words
B. FISH for chromosome 22q11 dele- at 23 months, but his motor develop-
tion ment was on target. He has trouble
C. Karyotype understanding instructions in his pre-
D. Methylation analysis of chromo- school class, but his mother reports
some 15q that he is able to hear loud speech
E. Nuclear receptor SET domain con- even when there is significant back-
taining protein-1 (NSD1 testing) ground noise, such as a birthday
party.
vv Correct Answer is: E What is the reason for his speech
Sotos syndrome (cerebral gigantism), delay?
is a genetic condition that affects 1: A. Mild language disorder
10,000 to 1: 50,000 individuals. The syn- B. Sensorineural hearing loss
drome is characterized by prenatal and C. Conductive hearing loss
postnatal overgrowth, macrocephaly, D. No delays, speech and hearing is
and characteristic facial features—spe- within normal variations.
cifically a high anterior hair line; fronto-
temporal hair sparsity, a long, thin face; vv Correct answer is: C
frontal bossing; down slanting palpe- Speech delay is the most common
bral fissures; and a prominent mandible. symptom of hearing impairment in
Advanced bone age is ­common but young children. Normally majority
182 Chapter 6 · Developmental Disorders

of children use their first words by compared to children exposed to carba-


16 months and after 21 months, this is mazepine, lamotrigine, and phenytoin,
considered a delay. External or middle by a mean of 8–11 points. The FDA
ear problems can cause conductive stated that:
hearing impairment. The deficient Valproate is contraindicated for the
in conductive hearing impairment is prevention of migraine headaches in
Loss of amplification is the deficient in the setting of pregnancy, labeled
conductive hearing impairment, and category X (“the risk of use in pregnant
therefore these children are able to women clearly outweighs any possible
hear loud speech in a noisy background benefit of the drug”).
better than soft speech in a quiet back- Valproate should only be prescribed
ground. if other medications are not effective or
The history suggests a hearing otherwise unacceptable when treating
6 disorder. If the hearing was tested and epilepsy or bipolar disorder. In these
normal, formal testing would be settings, the drug is considered
required to diagnose a language category D (“the potential benefit of the
disorder. drug in pregnant women may be
Children with sensorineural hearing acceptable despite its potential risks,”
impairment have difficulty discriminat- with human data suggesting risk to the
ing speech in the presence of back- fetus). In women who are of childbear-
ground noise. ing age but not pregnant, valproate
should not be taken unless essential,
zz Suggested Reading and if prescribed, should be used with
55 Smith RJ, Shearer AE, Hildebrand MS, Van birth control.
Camp G. 2014. Deafness and hereditary On the other hand, women having
hearing loss overview. epilepsy who become pregnant while
on valproate should not abruptly
?? 17. Which of the following drugs was discontinue the medication, and they
shown to decrease the IQ of children have to discuss the treatment plan with
born to mothers taking anticonvul- their physicians.
sant medication?
A. Levetiracetam zz Suggested Reading
B. Phenobarbital 55 Meador KJ, et al. Fetal antiepileptic drug
C. Valproic acid exposure and cognitive outcomes at age
D. Carbamazepine 6 years (NEAD study): a prospective
E. Primidone observational study. Lancet Neurol.
2013;12(3):244–52.
vv Correct Answer is: C
In 2013 the FDA issued a warning ?? 18. A 15-year-old boy has a known history
regarding valproate sodium (valproic of intellectual disability with a family
acid, and divalproex sodium), if was history (brother and two maternal
taken during pregnancy. The Neuro- cousins) with intellectual disability as
developmental Effects of Antiepileptic well. On examination, he has an elon-
Drugs (NEAD) study, showed that chil- gated face, with a high forehead and
dren exposed to valproate products in elongated jaw, and p ­ rotuberant ears.
utero had decreased IQs at age 6 years His testes are enlarged.
Developmental Disorders
183 6
Which statement is incorrect ?? 19. Which of the following statements
regarding the most likely diagnosis regarding acquired intellectual dis-
in this boy? ability are incorrect?
A. It is the most common inherited A. Congenital infections such as cyto-
form of intellectual disability megalovirus or rubella are associ-
B. Females may be affected, ated with intellectual disability
although to a lesser degree B. Intellectual disability has been
C. It results from expansion of the associated with exposure to radia-
CGG repeat in the familial intellec- tion during the first trimester
tual disability 1 gene on chromo- C. In utero exposure to AEDs can
some X lead to varying degrees of intel-
D. A family history of intellectual lectual disability and other CNS
disability may be present, but not abnormalities
necessarily D. Maternal alcohol intake is a com-
E. Severe intellectual disability is mon cause of acquired intellectual
invariably present disability
E. In utero exposure to alcohol does
vv Correct Answer is: E not lead to abnormalities outside
Intellectual disability has too many of the CNS
causes. Fragile X syndrome is the most
common inherited form of intellectual vv Correct Answer is: E
disability. There is an expansion of the Fetus exposed alcohol in utero can have
CGG repeat in the familial intellectual a congenital heart disease. There are
disability 1 gene on chromosome X. The a myriad of established or suspected
function of this gene is yet to be fully exposures associated with intellectual dis-
understood. Because this gene is on the ability. Congenital cytomegalovirus and
X chromosome, and because random rubella lead to intellectual disability with
X inactivation (lyonization) occurs in a variety of other manifestations such as
females, females are less often and less congenital blindness and periventricular
severely affected. Clinical manifesta- calcifications. Exposure to radiation dur-
tions in males include an elongated ing the first trimester of pregnancy have
face, with a high forehead and elon- also been associated with intellectual dis-
gated jaw, and protuberant ears and ability. Malnutrition during the first few
enlarged testes. Intellectual disability months of life can lead to reversible cog-
will range from mild and subtle to nitive delay. Maternal intake of alcohol,
severe. A family history of intellectual antiepileptics, vitamin A, and thalidomide
disability in males may be present. are also associated with various nervous
Other neurologic manifestations may system and systemic malformations as
occur in adults with a permutation. well as intellectual disability. Maternal
alcohol intake is a common cause of
zz Suggested Reading acquired (non-­genetic) intellectual disabil-
55 Comprehensive review in clinical neurol- ity. Other manifestations of fetal alcohol
ogy: a multiple-choice question book for syndrome include behavioral p ­ roblems
the Wards and Boards ©2011 Wolters such as hyperactivity, microcephaly,
Kluwer Health Lippincott Williams & short palpebral fissure (a short distance
Wilkins. All rights reserved. between the inner and outer canthi of the
184 Chapter 6 · Developmental Disorders

eyes), presence of epicanthal folds, and of reading disorder, mathematics dis-


hypoplasia of the maxilla, micrognathia, order, disorder of written expression,
and thin upper lip with flattened philtrum, and learning disorder not otherwise
and congenital heart disease. specified.
Specific learning disorder in the
?? 20. A 15-year-old boy has significant DSM-5 is defined as “A neurodevelop-
schoolwork problems mainly in read- mental disorder of biological origin
ing. Evaluation shows that he reads manifested in learning difficulty and
below the 10th percentile for his age, problems in acquiring academic skills
however his IQ is measured to be 100. markedly below age level and mani-
What is your most likely diagnosis? fested in the early school years, lasting
A. Specific learning disorder for at least 6 months; not attributed to
B. Mild intellectual disability intellectual disabilities, developmental
6 C. ADHD disorders, or neurological or motor
D. Vision problems disorders”.
E. Social communication disorder You can then specify if it is either:
With impairment in reading, with
vv Correct Answer is: A impairment in written expression, or
DSM-5 diagnosis of specific learning dis- with impairment in mathematics. Also,
order combines the DSM-IV ­diagnoses Mild, Moderate, or Severe.
185 7

Eating Disorders

© Springer International Publishing AG, part of Springer Nature 2018


Y. M. Awaad, Absolute Pediatric Neurology, https://doi.org/10.1007/978-3-319-78801-2_7
186 Chapter 7 · Eating Disorders

?? 1. An 18-year-old woman is referred to with SSRI is the treatment of choice in


her college’s health counselor after a this case. Bulimia nervosa is commonly
pharmacist detects several unusual associated with borderline personality
prescriptions for laxatives purchased disorder as comorbidity.
by the patient. Her body mass index In patients with eating disorder not
had been in the range of 23–24 since otherwise specified, patients may avoid
adolescence (normal 18.5–24.9). The food, binge with or without purging or
patient admits to an intense fear of participating excessively in exercise,
being fat for years, wishing to be thin- and partake in other methods of mini-
ner. She reported trying not to eat, mizing weight loss, but the patient's
but then gets so hungry that several weight is maintained within the normal
times a week she would binge for range, the episodes of binging occur
hours, would subsequently feel guilty, less than twice per week for 3 months,
and would take 30–40 pills of laxa- and menstruation remains regular. The
tives at once. binge eating form of eating disorder not
7 What is the most likely diagnosis in otherwise specified is the most common
this patient? eating disorder, followed by bulimia and
A. Bulimia nervosa anorexia nervosa. Teenage years is peak
B. Anorexia nervosa, restricting type onset of eating disorders, though they
C. Anorexia nervosa, binge eating/ may begin at any age. Eating disorders
purging type are more common in females than in
D. Eating disorder not otherwise males, with a 3:1 ratio, however males
specified are likely underdiagnosed. The patho-
E. Impulse control disorder physiology relates to be both envi-
ronmental and genetic factors; there
vv Correct Answer is: A is a 50–80% concordance rate among
Patients suffering from bulimia nervosa monozygotic twins. Comorbid mood,
binge eat over a discrete period of time anxiety, and personality disorders are
at least twice a week for 3 months, common among patients with eating
they lose over the extent of food intake disorders.
during that time period, followed by
compensatory behaviors to prevent zz Suggested Reading
weight gain such as excessive exercise, 55 Comprehensive review in clinical neurol-
induction of emesis or misuse of laxa- ogy: a multiple choice question book for
tives, diuretics, or other medications. the wards and boards ©2011 Wolters
Treatment of bulimia nervosa involves Kluwer Health Lippincott Williams &
Psychotherapy combined in some cases Wilkins. All rights reserved.
187 8

Epilepsy 1

© Springer International Publishing AG, part of Springer Nature 2018


Y. M. Awaad, Absolute Pediatric Neurology, https://doi.org/10.1007/978-3-319-78801-2_8
188 Chapter 8 · Epilepsy 1

?? 1. A 10-year-old boy is brought by his drooping side of my mouth.” He had


family because of frequent episodes one nocturnal seizure 6 months ago,
of drooping of the right or left side of and no work up has been done. He
his mouth that last 1–3 min each. He has unremarkable history and physical
is completely awake and alert during exam. He is otherwise a healthy boy
the episodes, but has difficulty talking. who does quite well in school.
He said to you that I am embarrassed You decide to get an EEG. A trac-
by them because “Spit drips out of the ing from it is below:

..      Fig. 8.1 EEG shows Centro-temporal spike. With a permission from 7 ILAE.org, Diagnostic Manual, and
7 EpilepsyDiagnosis.org

What are you going to tell the family? in dysarthria or drooling. There is no loss
A. He has a facial tic. of consciousness (simple partial seizure).
B. The seizure and the mouth droop- EEG shows Centro-temporal spike, often
ing are unrelated. with a surface dipole in which the spike is
C. He needs to have a video EEG to surface-negative in the Centro-temporal
capture an episode. region, and surface-positive over the
D. He has a form of epilepsy that he frontal region. It has a favorable prog-
is likely to outgrow. nosis, and seizures usually remit around
E. He needs to take ethosuximide. puberty. However, atypical forms of BECTS
are also recognized.
vv Correct Answer is: D
Benign Epilepsy with Centro-Temporal zz Suggested Reading
Spikes (BECTS- Benign Rolandic Epilepsy), 55 Guerrini R, Pellacani S. Benign childhood
it is very common in clinical practice, focal epilepsies. Epilepsia. 2012;53 Suppl
accounting for 13–23% of all cases of 4:9–18.
epilepsy during childhood. Patients are 55 Hughes JR. Benign epilepsy of childhood
usually otherwise normal, and the most with centro-temporal spikes (BECTS): to
common seizure occurs shortly after treat or not to treat, that is the question.
going to sleep, with focal motor activity Epilepsy Behav. 2010;19(3):197–203.
affecting the face and tongue, resulting
Epilepsy 1
189 8
?? 2. A 7-year-old female recently stared to seizures or spell have been reported.
have spells for 5–10 s multiple times She is an excellent student, at or
per day, both mother and teachers above grade level academically, and is
noted them, with behavioral inat- otherwise medically well. A 10 s page
tention and activity arrest. No other from her outpatient EEG is shown.

..      Fig. 8.2 With a permission from 7 ILAE.org, Diagnostic Manual, and 7 EpilepsyDiagnosis.org

What is your diagnosis? generalized spike-wave discharges may


A. Juvenile myoclonic epilepsy be seen, particularly during sleep.
B. Benign epilepsy with centrotem-
poral spikes zz Suggested Reading
C. Localization-related epilepsy with 55 Sadleir LG, Farrell K, Smith S, et al.
complex partial seizures Electroclinical features of absence seizures
D. Panayiotopoulos Syndrome in childhood absence epilepsy. Neurology.
E. Childhood Absence Epilepsy 2006; 67:413–8.

vv Correct Answer is: E ?? 3. A 6-month-old girl is brought to the


Childhood Absence Epilepsy (CAE) is very ED for excessive fussiness and colicky
common in clinical practice. The peak age behavior. On exam, she has repetitive
for presentation is 6–7 years. Seizures clusters of stereotyped flexion move-
are usually frequent, have short duration ments of the trunk, associated with a
with abrupt onset and resolution, and cry. Her EEG shows hypsarrhythmia. A
lack post-­ictal confusion. The seizures are few flexion movements are captured
commonly provoked by hyperventilation, and are associated with a generalized
which is a useful maneuver for revealing slow wave followed by an electro-
the correct diagnosis during both the bed- decrement. She has several scattered
side exam and the EEG. Abundant focal hypo-pigmented spots on the trunk
inter-ictal EEG spikes are not expected, and extremities. A brain MRI scan is
although fragmentary forms of the shown.
190 Chapter 8 · Epilepsy 1

zz Suggested Reading
55 Hancock EC, Osborne JP, Edwards
SW. Treatment of infantile spasms.
Cochrane Database Syst Rev.
2008;8(4):CD001770.
55 Willmore LJ, Abelson MB, Ben-Menachem
E, Pellock JM, Shields WD. Vigabatrin:
2008 update. Epilepsia. 2009;50(2):163–73.

?? 4. A 15-year-old female with absence


epilepsy for 3 years, has been well
controlled with ethosuximide.
Recently, she fells quick, lightning-­
like jerks of her shoulders and upper
extremities, in early morning hours
and when fatigued. They come after a
period of clumsiness with her hands
and involuntary flinging of writing
8 implements or kitchen utensils. Her
..      Fig. 8.3 With a permission from 7 ILAE.org, Diag- exam is remarkable for BMI of 28.
nostic Manual, and 7 EpilepsyDiagnosis.org
The interictal EEG is shown. The ictal
EEG shows 10–16-­Hz polyspike dis-
What is the best treatment option? charges.
A. Valproic acid Which of the following anticonvul-
B. Felbamate sant medications is the best treat-
C. Ralfinamide ment option?
D. Pyridoxine A. Carbamazepine
E. Vigabatrin B. Levetiracetam
C. Phenytoin
vv Correct Answer is: E D. Pregabalin
Vigabatrin (VGB) is a structural analogue E. Valproic acid
of the inhibitory neurotransmitter gamma-
aminobutyric acid (GABA) and irreversibly vv Correct Answer is: B
inhibits the GABA degradative enzyme Many anticonvulsants used in juvenile
GABA-transaminase (GABA-T), thus myoclonic epilepsy (JME). The primary
increasing brain levels of GABA. VGB is very agents are valproic acid, lamotrigine and
effective against infantile spasms associ- levetiracetam. Carbamazepine, phenytoin,
ated with tuberous sclerosis complex (TSC), pregabalin, and oxcarbazepine are limited
and it is the first-choice drug of in this case spectrum agents useful only in the adjunc-
based on many reports. One of VGB side tive treatment of partial-onset seizures.
effects is peripheral visual field defects Due to concerns regarding weight gain,
(VFDs), which varies depending on age and hyperinsulinemia and an increased risk of
degree of exposure to the medication. The hyper-­androgenism, valproic acid is now
prevalence in children is estimated at 15% commonly avoided in female patients. In
and retinal defects in infants range from males, all 3 medications are safe and used
15% to 31%. Cognitive, age-appropriate successfully. Levetiracetam is the only
visual field testing is required at baseline drug has FDA-­approved indication for the
and should be repeated at regular intervals adjunctive treatment of JME in patients
in patients who are on long-term therapy. 12 years and older.
Epilepsy 1
191 8
..      Fig. 8.4 With
a permission from
7 ILAE.org, Diagnostic
Manual, and 7 Epi-
lepsyDiagnosis.org

zz Suggested Reading to the ground. He has broken a few


55 Welty TE. Juvenile myoclonic epilepsy: teeth and sustained scalp contusions
epidemiology, pathophysiology, and and lacerations which have required
management. Paediatr Drugs. 2006; sutures. His EEG shows diffuse back-
8(5):303–10. ground slowing, as well as multifocal
55 Verrotti A, Manco R, di Marco G, Chiarelli and generalized slow spike-wave dis-
F, Franzoni E. The treatment of juvenile charges occurring at a frequency of
myoclonic epilepsy. Expert Rev. 1.5–2.5 Hz.
Neurother. 2006;6(6):847–54. What is the best treatment option at
this time?
?? 5. A 3-year-old developmentally delayed A. Phenobarbital
boy has multiple paroxysmal events B. Carbamazepine
for few months. He was diagnosed C. Oxcarbazepine
with infantile spasms at 5 months of D. Topiramate
age, and had a complete responder E. Felbamate
to ACTH. He did not have any further
seizures after he was weaned off vv Correct Answer is: D
until 6 months ago. Family reports Lennox-Gastaut syndrome is refractory
nocturnal events involving whole- to medical treatment, but better seizure
body stiffening, and while awake he control has been achieved by the newer
has multiple brief periods of altered generation broad-spectrum medications.
awareness with eyelid fluttering, eye Topiramate and felbamate are good
rolling and head nodding. He also options, but felbamate is not a first-line
has quick jerking movements of his medication because its side effects.
extremities. The scariest events are Rufinamide and clobazam were recently
“falling seizures” during which he approved for this condition. There are a
suddenly loses muscle tone and falls number of effective non-pharmacological
192 Chapter 8 · Epilepsy 1

..      Fig. 8.5 Slow spike-wave IED typical of the Lennox- reserved. (Figure courtesy of Jeffrey W. Britton, MD
Gastaut syndrome. Also note the associated underlying Electroencephalography (EEG): An Introductory Text and
background slowing, indicative of generalized cerebral Atlas of Normal and Abnormal Findings in Adults, Chil-
dysfunction and an epileptic encephalopathy. Ipsilateral dren, and Infants ISBN: 978-0-9979756-0-4, Copyright
ear referential montage. Copyright 2013. Mayo Foun- c2016 by American Epilepsy Society, 135 S. LaSalle St.,
dation for Medical Education and Research. All rights Suite 2850, Chicago, IL 60603)

options, including the ketogenic diet, VNS ?? 6. The family of an 8-year-old boy
and corpus callosotomy. Finally, immu- reports that their son is having a
nomodulation with IVIG or steroids can night events. They describe the
be helpful in some patients. In summary, event as a rhythmic jerking of the
most successful treatment strategies left side of the face, with drooling
appear to involve combination pharma- and dysarthria. The boy is fully alert
cological and non-pharmacological thera- and understand and following their
pies which may provide complementary instructions. He has a normal exam
benefits. and doing well in school. His routine
EEG is shown
zz Suggested Reading What is the child’s prognosis?
55 Arzimanoglou A, French J, Blume WT, A. Controlled, but lifelong, epilepsy
Cross JH, Ernst JP, Feucht M, Genton P, B. Worsening myoclonus, ataxia, and
Guerrini R, Kluger G, Pellock JM, Perucca dementia
E, Wheless JW. Lennox-Gastaut syndrome: C. Worsening myoclonus and ataxia,
a consensus approach on diagnosis, but without dementia
assessment, management, and trial D. Disappearance of seizures and
methodology. Lancet Neurol. 2009; EEG abnormalities by mid-teens
8(1):82–93. E. Progressive aphasia
Epilepsy 1
193 8

..      Fig. 8.6 With a permission from 7 ILAE.org, Diagnostic Manual, and 7 EpilepsyDiagnosis.org

vv Correct Answer is: D ?? 7. A 5-year-old boy, who had a hypoxic


Benign Rolandic Epilepsy (Benign Epilepsy ischemic encephalopathy complica-
with Centro-Temporal Spikes (BECTS) is a tion at birth. He starts having general-
very common childhood seizure (13–23%). ized tonic-clonic seizures for 1 year.
Children are normal, and the seizure Recently, he starts having generalized
occurs shortly after going to sleep. There tonic seizures at night, drops, and
is a focal motor activity affecting the face staring episodes during daytime. EEG
and tongue, resulting in dysarthria or is shown.
drooling. There is no loss of conscious- What is the most likely diagnosis?
ness. EEG shows Centro-temporal spike, A. Benign familial infantile seizures
often with a surface dipole in which the B. Childhood absence epilepsy
spike is surface-negative in the Centro- C. Dravet syndrome
temporal region, and surface-­positive D. Lennox-Gastaut syndrome
over the frontal region. Prognosis is favor- E. West syndrome
able, and seizures usually remit around
puberty. Atypical forms of BECTS have vv Correct Answer is: D
been reported. EEG is characteristic of Lennox-Gastaut
syndrome with slow sharp and slow wave
zz Suggested Reading complexes of 1.5–2 Hz. This is different
55 Bouma PAD, Bovenkerk AC, Westendorp than EEG with faster frequency seen in pri-
RGJ, Brouwer OF. The course of benign mary generalized epilepsy, like childhood
partial epilepsy of childhood with centro- or juvenile absence epilepsy. Also, the
temporal spikes: a meta-analysis. clinical scenario is consistent with Lennox-
Neurology 1997. Gastaut syndrome. There are different
194 Chapter 8 · Epilepsy 1

..      Fig. 8.7 Slow spike-wave IED typical of the Lennox- rights reserved. (Figure courtesy of Jeffrey W. Britton,
Gastaut syndrome. Also note the associated underlying MD Electroencephalography (EEG): An Introductory
background slowing, indicative of generalized cerebral Text and Atlas of Normal and Abnormal Findings in
dysfunction and an epileptic encephalopathy. Ipsilat- Adults, Children, and Infants ISBN: 978-0-9979756-
eral ear referential montage. Copyright 2013. Mayo 0-4, Copyright c2016 by American Epilepsy Society,
Foundation for Medical Education and Research. All 135 S. LaSalle St., Suite 2850, Chicago, IL 60603)

generalized seizure types (generalized went to sleep. She witnesses him


tonic-clonic, generalized tonic and drop having twitching of his left side of
attacks, and probably atypical absence), his face and he was drooling and
and the EEG shows the characteristic dif- his speech was not clear. Also, she
fuse slow spike-and-wave complexes (less noticed some twitching of his left
than 2.5 Hz). hand. He is a healthy and excellent
student. His EEG is shown.
zz Suggested Reading What is the most likely diagnosis?
55 VanStraten AF, Ng YT. Update on the A. benign epilepsy with centrotem-
management of Lennox-­Gastaut syn- poral spikes (BECTS)
drome. Pediatr Neurol 2012;47(3):153–61. B. early onset benign childhood
55 Trevathan E. Infantile spasms in Lennox-­ occipital epilepsy (Panayiotopou-
Gastaut syndrome. J Child Neurol los type)
2012;17(Suppl 2):2S9–22. C. epilepsy with myoclonic
absences
?? 8. The mother of a 6-year-old boy D. late onset childhood occipital epi-
rushed to her son room after she lepsy (Gastaut type)
heard her son calling her after he E. Lennox-Gastaut syndrome
Epilepsy 1
195 8

..      Fig. 8.8 With a permission from 7 ILAE.org, Diagnostic Manual, and 7 EpilepsyDiagnosis.org

vv Correct Answer is: A ?? 9. A 6-year-old female wakes up after


The history and EEG are characteristic of she went into her sleep. She vomited
benign childhood epilepsy with Centro and was able to answer her family. Few
temporal spikes (BECTS). The EEG shows minutes later she has forced eye devia-
central and temporal epileptiform dis- tion to the left but she was responsive.
charges with a normal background (these Then she closed her eyes, became
may be bilateral, synchronous or asyn- unresponsive, and had a generalized
chronous; often with horizontal dipole). convulsion. She then was postictal for
This seizure is the most common partial few minutes. Her EEG is shown.
epilepsy of childhood (2/3 of all idiopathic What is her most likely diagnosis?
focal epilepsies in childhood). It occurs A. benign familial infantile seizures
between 5 and 8 years of age. Usually, it is B. early-onset benign childhood
focal motor seizures, unilateral involving occipital epilepsy (Panayiotopou-
the face, the oropharyngeal muscles, and los type)
upper extremities. It occurs often during C. Landau-Kleffner syndrome
sleep; and less than 25% occurs during D. mesial temporal lobe epilepsy
wakefulness. Treatment is not necessary with hippocampal sclerosis
because patients usually will have 1–2 E. migrating partial seizures of infancy
seizures, and prognosis is excellent.
vv Correct Answer is: B
zz Suggested Reading EEG is classic for Panayiotopoulos type
55 Guerrini R, Pellacani S. Benign childhood (early onset benign childhood occipital
focal epilepsies. Epilepsia 2012;53(Suppl epilepsy). The EEG has occipital spikes,
4):9–18. which can be either bilateral or unilateral.
196 Chapter 8 · Epilepsy 1

..      Fig. 8.9 With a permission from 7 ILAE.org, Diagnostic Manual, and 7 EpilepsyDiagnosis.org

The occipital spikes can be seen with eye ?? 10. A 5-year-old male with long events
closure, or with fixation and central vision of vomiting, unresponsiveness, loss
are eliminated. The spikes are often reac- of tone, eye deviation, and face flush-
tive and decrease in frequency when the ing. He has a normal development
eyes are open. The clinical manifestations history and a normal. His interictal
are autonomic and behavioral distur- EEG shows a normal background and
bances, vomiting, deviation of the eyes, focal occipital sharp wave discharges
and often impairment of consciousness which happened more during his
that progresses to convulsions. Duration is sleep.
usually long. It is 28% of all benign child- What is the most likely diagnosis?
hood partial seizures. Prognosis is excel- A. Angelman syndrome
lent, it occurs as some single or infrequent B. Doose syndrome
seizures that subside within 1–2 years. C. Landau-Kleffner syndrome
Onset is between 2 and 12 years, with a D. Lennox-Gastaut syndrome
median of 5 years. E. Panayiotopoulos syndrome

zz Suggested Reading vv Correct Answer is: E


55 Panayiotopoulos CP. Benign childhood This is a clinical presentation of
epileptic syndrome with occipital spikes: Panayiotopoulos syndrome, a benign child-
new classification proposed by the Internal hood seizure disorder with a good prog-
League Against Epilepsy. J Child Neurol nosis. Angelman syndrome is a genetic
2000;15(8):648–52. epilepsy with onset in the infantile period.
Epilepsy 1
197 8
Doose syndrome, also known as epilepsy ?? 11. A 7-month-old boy was diagnosed by
with myoclonic atonic seizures, with onset his PCP with colic at age 5 months. He
in the infantile period, is characterized by has spells of waist flexion and arms
myoclonic, atonic, atypical absence, and extension. They are brief, 1–2 s, and
bilateral convulsions. Landau-Kleffner occur in clusters; mainly upon waken-
syndrome is a childhood onset epileptic ing from a nap, or during transitions
encephalopathy characterized by aphasia from sleep to wake, or wake to sleep.
with continuous epileptiform discharges The mother had uncomplicated preg-
during sleep. Lennox-­Gastaut syndrome nancy and he is a healthy child. His
is a severe epileptic encephalopathy with EEG is shown.
multiple seizure types, intellectual disabil- What is the most likely diagnosis?
ity, and slow spike-­and-­wave on EEG. A. benign familial infantile seizures
B. benign familial neonatal seizures
zz Suggested Reading C. early myoclonic encephalopathy
55 Ferrie C, Caraballo R, Covanis A, et al. (EME)
Panayiotopoulos syndrome: a consensus D. migrating partial seizures of
view. Dev Med Child Neurol. infancy
2006;48(3):236240. E. West syndrome

..      Fig. 8.10 With a permission from 7 ILAE.org, Diagnostic Manual, and 7 EpilepsyDiagnosis.org

vv Correct Answer is: E zz Suggested Reading


This inter-ictal EEG shows chaotic non-­ 55 Pellock JM, Hrachovy R, Shinnar S, et al.
rhythmic disorganized high-voltage spike Infantile spasms: a US consensus report.
and slow wave activity which is consistent Epilepsia 2010;51(10):2175–89.
with hypsarrhythmia. The spike and slow 55 Kossoff EH. Infantile spasms. Neurologist
waves are varied from moment to moment 2010;15(2):69–75.
in duration and location. The background
is often greater than 300 microvolts. The ?? 12. A 2-week old baby boy is brought
hypsarrhythmic usually occur during non-­ to the ED by his mother because his
REM sleep. seizures. The seizures are lasting for
198 Chapter 8 · Epilepsy 1

5–10 s, many time a day, come in What is the most likely diagnosis?
clusters of stiffening of all extremi- A. benign familial neonatal seizures
ties. His labs are normal. Head CT is B. Dravet syndrome
also normal. He has a normal birth C. infantile spasms
history with Apgar scores of 9 and 9, D. migrating partial seizures of
at 1 and 5 min respectively. His EEG is infancy
shown. E. Ohtahara syndrome

..      Fig. 8.11 American Academy of Neurology Institute. (Reproduced with permission)

vv Correct Answer is: E zz Suggested Reading


The history is indicative of Ohtahara 55 Beal JC, Cherian K, Moshe SO. Early-
syndrome. The EEG shows a symmetric onset epileptic encephalopathies: Ohtahara
burst suppression pattern. The bursts are syndrome and early myoclonic encephalo-
of high amplitude spikes and poly-spikes thopy. Pediatr Neurol. 2012;47(5):
(symmetric or asymmetric). The pattern 317–23.
is unchanged during wakefulness and 55 Pavone P, Spalice A, Polizzi A, et al.
sleep. Onset is in early infancy, within Ohtahara syndrome with emphasis on
the first 3 months, and often within the recent genetic discovery. Brain Dev.
first 2 weeks. Manifestations are tonic 2012;34(6):459–68.
spasms (generalized or lateralized), can
occur both singly or in clusters, and are ?? 13. An 8-year-old boy with multiple sei-
independent of sleep cycle. Tonic spasms zure types came to your office for an
typically last up to 10 s and can occur evaluation. His seizures started as a
hundreds of times per day. Structural neonate; he would grimace his face
brain malformations or metabolic or with altered consciousness. Then
genetic conditions could be the under- he starts to have complex partial
ling etiology, and in some no etiology is ­seizures, drop attacks, and general-
found. Genetic conditions which should ized tonic-clonic seizures. He has
be looked for include Abnormalities of behavioral difficulties and devel-
the ARX and STXBP1, among others. opmental delay. MRI of the brain is
Treatment is often not successful. shown.
Epilepsy 1
199 8
What is his most likely diagnosis? C. Lennox-Gastaut syndrome
A. epilepsy with myoclonic absences D. mesial temporal lobe epilepsy
B. seizures with hypothalamic ham- with hippocampal sclerosis
artoma E. Rasmussen syndrome

..      Fig. 8.12 With


a permission from
7 ILAE.org, Diagnostic
Manual, and 7 Epi-
lepsyDiagnosis.org

vv Correct Answer is: B Surgical considerations and outcomes.


MRI shows left hypothalamic hamar- Neurosurg Focus. 2013;34(6):E-7.
toma. Hypothalamic hamartoma is
hyper-­intense relative to the cortical and ?? 14. You are evaluating a 7-year-old boy for
deep grey matter on the T2-weighted staring episodes. His teacher told his
image. The clinical history confirms parents that she has witnessed many
the diagnosis, gelastic episodes (facial spells of staring off space for 5–10 s
grimace) beginning in infancy, later and the child cannot remember what
development of other seizure types as happened. He has no change in his
children get older, typically associated tone and no associated motor activity.
with behavior difficulties and an epilep- On one occasion, the class was leaving
tic encephalopathy. the classroom; the child stood up to
leave, but did not. He then asked the
zz Suggested Reading teacher what they were supposed to
55 Mittal S, Mittal M, Montes JL, et al. be doing. His EEG is shown.
Hypothalamic hamartomas Part 1. What is the best treatment?
Clinical, neuroimaging, and neurophysi- A. oxcarbazepine
ological characteristics. Neurosurg Focus. B. methylphenidate
2013;34(6):E-6. C. lamotrigine
55 Mittal S, Mittal M, Montes JL, et al. D. ethosuximide
Hypothalamic hamartomas Part 2. E. valproic acid
200 Chapter 8 · Epilepsy 1

..      Fig. 8.13 With a permission from 7 ILAE.org, Diagnostic Manual, and 7 EpilepsyDiagnosis.org

vv Correct Answer is: D choice is ethosuximide. Lamotrigine is less


Absence seizures is a childhood disorder, effective, and valproic acid causes more
starts between 4 and 10 years and the cognitive side effects. Oxcarbamazepine is
peak onset is between 5 and 7 years, and not effective for absence and can make it
usually begin. Childhood absence epilepsy worse. His attention problems are related
is more common in girls than boys. It is to his seizures, so methylphenidate would
frequent during daytime. Manifestations not be appropriate.
are loss of awareness, unresponsiveness,
and behavioral arrest. Most children have zz Suggested Reading
a complete cessation of activity, but a 55 Matricardi S, Verrotti A, Chiarelli F, et al.
few can continue activities in an altered Current advances in childhood absence
fashion. The classic l EEG shows bilateral epilepsy. Pediatr Neurol. 2014;50(30):205–12.
symmetric rhythm discharges 3 Hz (spike 55 Glauser TA, Cnaan A, Shinnar S, et al.
and slow wave complexes) with sudden Ethosuximide valproic acid and lamotrig-
onset and offset. Juvenile absence epi- ine in childhood absence epilepsy. N Engl
lepsy typically occurs in older children and J Med. 2010;362(9):790–9.
is associated with generalized tonic-­clonic
seizures. Patients with epilepsy, with ?? 15. A 14-year-old girl has her first
generalized tonic-clonic seizures alone, do generalized tonic-clonic seizure
not have absence seizures. Treatment of while she was taking a shower.
Epilepsy 1
201 8
She has bilateral arm jerks before What is the most appropriate for
her seizure, she has similar jerks first-line therapy?
after awakening when she is sleep A. carbamazepine
deprived. She has no episodes of B. levetiracetam
staring or unresponsiveness. She has C. patient does not require AED
a normal neurologic examination therapy
and head CT. EEG before treatment D. phenytoin
is shown. E. valproate

..      Fig. 8.14 Atypical generalized spike-wave IED. This Copyright 2013. Mayo Foundation for Medical Educa-
IED is most commonly seen in children with juvenile tion and Research. All rights reserved. (Figure courtesy
absence or myoclonic epilepsy syndromes. The dis- of Jeffrey W. Britton, MDElectroencephalography (EEG):
charges are induced by the activating procedure of An Introductory Text and Atlas of Normal and Abnormal
photic stimulation, representative of a so-called photo Findings in Adults, Children, and Infants ISBN: 978–0–
paroxysmal response, which often occurs in those with 9,979,756-0-4, Copyright c2016 by American Epilepsy
primary idiopathic generalized epilepsy syndromes. Society, 135 S. LaSalle St., Suite 2850, Chicago, IL 60603)

vv Correct Answer is: B be quite appropriate for a young man).


The clinical history and EEG are typical Carbamazepine and phenytoin are not
of juvenile myoclonic epilepsy (JME). broad-spectrum agents and would not be
Valproate and levetiracetam both are a first choice for treatment of this primary
effective medications for JME, but because generalized (or genetic in the new clas-
the risk of birth defects and impact on sification) epilepsy. While sleep deprivation
fetal cognitive development valpro- and alcohol can be provocative for the sei-
ate should not be first- (or second-) line zures of JME, this teenager is undoubtedly
therapy for a young woman (it would going to be sleep deprived in the future, so
202 Chapter 8 · Epilepsy 1

therapy at this time is indicated. In addition, intervals for 5–6 months. He was tried
patients with JME have a high likelihood of on arbamazepine, levetiracetam, topi-
recurrent seizures without treatment. ramate, pregabalin, and zonisamide.
His seizures are staring off space and
zz Suggested Reading left-hand automatisms, sometimes
55 Noachtar S, Andermann E, Mewisch P, lip smacking. They are 40 s in dura-
et al. Levetiracetam for the treatment of tion; then, 3–4 min later his language
idiopathic generalized epilepsy with function returns to baseline. Some
myoclonic seizures. Neurology. typical events are captured on video
2008;70:607–16. EEG monitoring, showed left temporal
lobe ictal activity. Neuropsychologi-
?? 16. A 16-year-old boy with some medically cal evaluation was done and revealed
resistant complex partial seizures. He remarkable impairment in naming and
started to have seizures at 10 years verbal memory, with intact other cog-
of age, he had some seizure-free nitive abilities. His MRI is shown.

..      Fig. 8.15 American

8 Academy of Neurology
Institute. (Reproduced
with permission)

What is the best next step in his abnormal MRI and a confirmatory
management? EEG. Neuropsychological evaluation
A. obtain a magnetoencephalogram confirms the left temporal lobe dysfunc-
B. obtain PET of the brain tion. He is a perfect candidate for anterial
C. perform additional video EEG temporal resection and most probably he
monitoring with intracranial elec- will respond fully to antiepileptic medica-
trodes tions. Magnetoencephalogram can be
D. refer the patient for a clinical trial helpful, particularly when the MRI is non-
of a novel antiepileptic drug lesional; however, it is not necessary prior
E. resect the left anterial mesial tem- to surgery. As documented in the AAN
poral lobe 2003 Practice Parameter, the benefits of
anterial mesial temporal lobe resection
vv Correct Answer is: E are greater than continuing treatment
The patient has a drug resistant with antiepileptic medications, and the
left temporal lobe epilepsy, with an risks are comparable.
Epilepsy 1
203 8
zz Suggested Reading eye lead), shows a response to inter-
55 Engle J, Wiebe S, French J, et al. Practice mittent photic stimulation marked by
Parameter: temporal lobe and localized the tick marks at the bottom of the
neocortical resections for epilepsy. Report tracing.
of the Quality Standards Subcommittee of What is the best explanation to this
the American Academy of Neurology, in response?
Association with the American Epilepsy A. abnormal posterior spiking
Society and the American Association of B. asymmetrical driving
Neurological Surgeons. Neurology. C. driven eye blinks
2003;60(4):538–47. D. prominent, but normal, occipital
driving
?? 17. EEG of a 10-year-old boy with staring E. spike-waves
spells (IOS = left eye lead; IOD = right

..      Fig. 8.16 American Academy of Neurology Institute. (Reproduced with permission)

vv Correct Answer is: D methods. Neurophysiol Clin. 1999;29(4):


The tick marks at the bottom of the trac- 318–24.
ing mark the time of light flashes during
intermittent photic stimulation (IPS). ?? 18. An EEG was ordered on an 18-year-old
Photic responses are abnormal when they female to rule out encephalopathy.
produce spikes that outlast the stimuli or She was diagnosed with lupus, steroid
spread far anteriorly, seizures, slowing or psychosis, and bizarre behavior.
marked asymmetry. Posterior spikes not The frontal leads rhythmical activity
outlasting the flashes can be normal. indicates the following activities?
A. delta activity from the brain
zz Suggested Reading B. eye movements
55 Trenite DG, Binnie CD, Harding GF, et al. C. phantom spike waves
Medical technology assessment photic D. possible frontal seizure
stimulation–standardization of screening E. spike-wave activity
204 Chapter 8 · Epilepsy 1

..      Fig. 8.17 Eye movement artifact. Electroencepha- and Infants ISBN: 978-0-9979756-0-4, Copyright c2016
lography (EEG): An Introductory Text and Atlas of by American Epilepsy Society, 135 S. LaSalle St., Suite
Normal and Abnormal Findings in Adults, Children, 2850, Chicago, IL 60603

8
vv Correct Answer is: B 50 μV calibration. The EEG shows all the
The fronto-polar leads show rhythmical features of hypsarrhythmia: multifocal
slow waves, in this instance with little dis- spikes, diffuse slowing, periods of relative
tribution to other leads. Leads below the flattening of the background (“burst-
eye (IOS, IOD) are out of phase with Fp1 suppression-­like”). Hypsarrhythmia can
and Fp2, documenting that these are verti- be seen in infantile spasms and West syn-
cal eye movements, likely resulting from drome. Herpes simplex would likely pro-
partial blinking called eye flutter. Potentials duce focal temporal epileptiform patterns.
from the frontal lobe would be in phase
with electrodes above and below the eyes. zz Suggested Reading
55 Kramer U, Sue WC, Mikati
zz Suggested Reading MA. Hypsarrhythmia: frequency of
55 Croft RJ, Barry RJ. Removal of ocular variant patterns and correlation with
artifact from the EEG: a review. etiology and outcome. Neurology.
Neurophysiol Clin. 2000;30(1):5–19. 1997;48(1):197–203.
?? 19. A 16 month--old boy presents with a
history of infantile spasms. He is taken ?? 20. A 1-day-old full term infant has been
valproic acid and he was on ACTH evaluated to rule out seizure. The infant
before. His EEG (IOS = left eye lead; had distress at birth, a nuchal cord, and
IOD = right eye lead) indicate which of low Apgar scores; and now has occa-
the following diagnoses? sional respiratory pauses and right leg
A. herpes simplex encephalitis tremors. EEG tracing is shown (IOS = left
B. hypsarrhythmia eye lead; IOD = right eye lead).
C. low voltage What is the most likely diagnosis?
D. simple partial seizures A. encoche frontalis
E. status epilepticus B. interictal spikes
C. rhythmic artifact
vv Correct Answer is: B D. right temporal
The EEG has a very high voltage (200 μV ­electroencephalographic seizure
calibration), as opposed to the more typical E. tracé alternant
Epilepsy 1
205 8

..      Fig. 8.18 Hypsarrhythmic EEG pattern in an infant All rights reserved. (Figure courtesy of Jeffrey W. Brit-
with West syndrome. EEG of an 8-month-old infant ton, MD. Electroencephalography (EEG): An Introduc-
with infantile spasms and developmental regression. tory Text and Atlas of Normal and Abnormal Findings
An ictal electrodecremental response (black brackets) in Adults, Children, and Infants ISBN: 978-0-9979756-
is seen during an epileptic spasm. Copyright 2013. 0-4, Copyright c2016 by American Epilepsy Society,
Mayo Foundation for Medical Education and Research. 135 S. LaSalle St., Suite 2850, Chicago, IL 60603)

..      Fig. 8.19 American Academy of Neurology Institute, produced by permission

vv Correct Answer is: D not be expected to be so localized; but in


The EEG shows a localized rhythmical 4/ a neonatal brain, with wider spaced elec-
second to 5/second spikes occur at T4. EEG trodes, limited myelination and reduced
of an older child or an adult, activity would connectivity, ictal events can remain
206 Chapter 8 · Epilepsy 1

restricted to one or a few electrodes. Any episodes lasting up to 20 s, multiple


focal, rhythmic evolving event in a neona- times per day. No treatment was given.
tal EEG should be considered suspicious for In her EEG shown, (IOS = left eye lead;
ictal activity, unless it is an obvious artifact. IOD = right eye lead).
What do you see during hyperven-
zz Suggested Reading tilation?
55 Abend NS, Wusthoff CJ. Neonatal seizures A. asymmetrical slowing
and status epilepticus. J Clin Neurophysiol. B. eye flutter artifact
2012;29(5):441–8. C. generalized spike-­waves
D. left temporal and parietal sharp
?? 21. A 6-year-old boy has a history of waves
2-month of staring and unresponsive E. normal response

..      Fig. 8.20 Absence seizure. During an absence Copyright 2013. Mayo Foundation for Medical Education
seizure, the typical ictal EEG pattern is an extension of and Research. All rights reserved. (Figure courtesy of
the interictal pattern (i.e., 3-Hz generalized spike-wave Erik K. St. Louis, MD Electroencephalography (EEG): An
of a prolonged duration, with usual clinical accompani- Introductory Text and Atlas of Normal and Abnormal
ments of staring with behavioral arrest, or with variable Findings in Adults, Children, and Infants ISBN: 978-0-
accompaniment by oral and manual automatisms such 9979756-0-4, Copyright c2016 by American Epilepsy
as blinking, lip smacking, or hand-fumbling movements. Society, 135 S. LaSalle St., Suite 2850, Chicago, IL 60603)

vv Correct Answer is: C zz Suggested Reading


Hyperventilation produced early diffuse 55 Hughes JR. Absence seizures: a review of
slowing, followed by a sudden 3/second recent reports with new concepts. Epilepsy
to 4/second generalized spike-wave Behav. 2009;15(4):404–12.
discharge with bilateral frontocentral
maximum. This pattern is associated with ?? 22. A 2-month-old baby boy has a history of
primary generalized epilepsy, such as multiple clonic seizures in the past 24 h.
childhood absence epilepsy. He was delivered vaginally at full term
Epilepsy 1
207 8
pregnancy. The mother had gestational holoprosencephaly (HPE). There are both
diabetes, controlled with diet and insu- environmental and genetic factors, which
lin. On day one, he had feeding prob- result in the fetal cerebral hemispheres,
lems and required nasogastric feeding and often diencephalon, fail to separate to
for several days. Otherwise he is healthy. varying degrees. Facial malformations are
Family history is negative of seizures. common, and include mild facial defor-
On examination, his head size is below mities, such as hypotelorism and a flat
5rd percentile, mild hypotelorism, and a nasal bridge, to the more severe cyclopia
depressed nasal bridge. Neurologically, (a single midline eye) and a proboscis (a
he is alert and has brief visual regard rudimentary single nostril nose above the
but does not track objects. He has hypo- eye). The condition varies from the most
tonic, with brisk reflexes bilaterally. You severe alobar form (complete failure of
started him on phenobarbital and he hemisphere and diencephalic cleavage),
became seizure-free 24 h later. His brain to the mildest interhemispheric variant
coronal MRI is shown. (where most of the hemisphere is cleav-
age normal, and only the most inferior
portions of the diencephalon and/or cor-
pus callosum are malformed).
All types include microcephaly,
developmental delay, and seizures forms
as some common features (though most
prevalent in the alobar forms.) Endocrine
dysfunction secondary to subtle malfor-
mation of the hypothalamic/pituitary axis
is very common, with diabetes insipidus
(and associated hypernatremia) being the
most common, occurring in up to 68% of
cases. Growth hormone deficiency,
hypothyroidism, and hypocortisolism are
less common features. Alobar forms
commonly lead to progressive hydroceph-
..      Fig. 8.21 American Academy of Neurology Insti- alus, through an associated midline dorsal
tute. (Produced by permission) cyst, which develops due to impaired
egress of spinal fluid from the third
Which of the following laboratory ventricle related to blockage from a
findings is most likely associated malformed surrounding fused diencepha-
with this ­abnormality? lon. VP shunting is common, and serial
A. elevated ratio of very long-chain head circumferences should be followed
fatty acids closely.
B. elevated serum lactate The sonic hedgehog (SHH) protein plays
C. hypernatremia an integral role in this normal cleavage
D. hyperthyroidism process, and seems to be the pathway
E. hypocalcemia susceptible to environmental and genetic
influences leading to the malformation.
vv Correct Answer is: C Maternal diabetes appears to be the
The MRI shows incomplete telencephalic strongest environmental factor, with an
cleavage consistent with a diagnosis of incidence of HPE of up to 1% (100 times the
208 Chapter 8 · Epilepsy 1

risk in the general population). A distinct but cannot walk; she makes eye con-
chromosomal abnormality is identified in tact and babbles, but has no speech.
30–50% of cases, and HPE can be part of Her EEG is shown.
larger genetic syndromes, such as Smith- Which of the following epilepsy syn-
Lemli-Opitz. Other malformations of dromes is most likely?
cortical development can be present as A. absence epilepsy with eyelid myo-
well, most commonly grey matter heteroto- clonia
pias.Based on the severity of the malforma- B. benign centrotemporal epilepsy
tion, developmental and feeding ability C. Landau-Kleffner syndrome
vary, with alobar forms usually being D. Lennox-Gastaut syndrome
entirely dependent on others for care, E. Panayiotopoulos syndrome
including frequent need for gastrostomy
tube feeding and failure to sit or walk vv Correct Answer is: D
independently. Lobar forms may develop All children with epilepsy should be
independent ambulatory skills and categorized into epilepsy syndromes, if
functional use of hands and typically feed possible. These syndromes carry specific
exclusively by mouth. Life span is information about age of onset, seizure
decreased, inversely related to the degree types, associated neurocognitive issues,
8 of malformation, though even alobar forms and etiologic origin; they provide informa-
can survive (with appropriate attention to tion regarding long-term outcome from
feeding safety, seizure control, prevention a seizure control/resolution and eventual
of fatal hydrocephalus and management of cognitive/quality of life standpoint. There
endocrine abnormalities) for up to 15 years. are different epilepsy syndromes for dif-
ferent age children, with onset in the
zz Suggested Reading neonatal, infantile, childhood, and ado-
55 Swaiman K, Ashwal S, Ferriero D, Schor lescent age groups. These syndromes can
N, editors. Swaiman’s pediatric neurology: otherwise be subdivided into focal or gen-
principles and practice. 5th ed. eralized epilepsies and idiopathic (primary)
Philadelphia: Elsevier Saunders, 2012; versus symptomatic (secondary) epilepsies,
145–50. with symptomatic cases having a known
or presumed underlying associated brain
?? 23. A 6-year-old female came to the office dysfunction, such as developmental delay.
with a history of frequent seizures. Other references may refer to “benign” and
She has been having seizures since “catastrophic” epilepsies, usually referring
her first year of life, the mother called to both ease of seizure control and even-
them “spasms.” Her seizures stopped tual cognitive outcome.The case presented
with treatment; however, she started here entails intractable epilepsy and sig-
having different seizures over the past nificant developmental delay, indicating
several years. She has multiple seizures a symptomatic or catastrophic epilepsy.
per day, despite she is on topiramate, Of the options, only Landau-Kleffner syn-
levetiracetam, and clonazepam. Her drome or Lennox-Gastaut syndrome would
spells are, stares unresponsively for fit this description. Absence epilepsy with
10–20 s, head drop and brief tonic eyelid myoclonia (Jeavon’s syndrome),
stiffening of her extremities. She is a benign centrotemporal (Rolandic) epilepsy,
healthy girl, but she has severe devel- and Panayiotopoulos syndrome are well
opmental delay, and she is completely defined idiopathic epilepsies of childhood,
dependent on her mother for her daily none of which are associated with develop-
living activities. She can sit and crawls, mental delay.
Epilepsy 1
209 8

..      Fig. 8.22 Slow spike-wave IED typical of the Len- All rights reserved. (Figure courtesy of Jeffrey W. Britton,
nox-Gastaut syndrome. Also note the associated under- MDElectroencephalography (EEG): An Introductory Text
lying background slowing, indicative of generalized and Atlas of Normal and Abnormal Findings in Adults,
cerebral dysfunction and an epileptic encephalopathy. Children, and InfantsISBN: 978-0-9979756-0-4, Copy-
Ipsilateral ear referential montage. Copyright 2013. right c2016 by American Epilepsy Society, 135 S. LaSalle
Mayo Foundation for Medical Education and Research. St., Suite 2850, Chicago, IL 60603)

Lennox-Gastaut syndrome is classically require other nonpharmacologic therapies


defined by a triad consisting of: (1) multiple such at the ketogenic diet, vagal nerve
seizure types, to include tonic, atonic, stimulator, or even corpus callosotomy for
myoclonic and atypical absence, (2) an inter- refractory atonic seizures.
ictal EEG pattern, similar to the figure, Landau-Kleffner syndrome (LKS), (and
showing generalized 1.5–2.5 Hz slow the related electrical status epilepticus in
spike-and-wave complexes on an abnor- sleep or ESES), typically presents with
mally slow background, and (3) cognitive normal initial development, followed by a
dysfunction. A majority of cases have period of regression with language, or
pre-existing structural causes, either from more global regression. Associated seizures
perinatal distress, malformations of cortical typically predate the onset of regression,
development, or other neurocutaneous, though can occasionally start concurrently.
genetic, or metabolic disorders. In these The seizures themselves in LKS typically
cases, infantile spasms often predate have onset from the temporal region, and
evolution into this syndrome. The seizures usually manifest as nocturnal hemiclonic
themselves are often intractable and may seizures. Treatment of the seizures
210 Chapter 8 · Epilepsy 1

themselves is not very difficult, though have noted multiple events of staring
improvement of the diffuse electrical status off space and eye blinking lasting
pattern often requires adjunctive therapy 10–20 s. She has multiple events
with corticosteroids, high dose nocturnal per day. She does not respond for
benzodiazepines, and even calling her name during the events
IVIG. Occasionally, standard anticonvul- and seems “in her own world.” She
sants such as valproic acid or levetiracetam has trouble sleeping at night and
can also lead to improvement (and rarely she wakes up screaming. Her EEG is
resolution) of this pattern. Aggressive shown. Her PCP started her on meth-
treatment early in the course is required to ylphenidate for presumed ADHD, but
prevent permanent language dysfunction. continues to have events.
What is your next step in her man-
zz Suggested Reading agement?
55 Wirrell E, Nickels KC. Pediatric epilepsy A. change medication to ethosuximide
syndromes. Continuum Lifelong Learning B. change methylphenidate to
Neurol 2010;16(3):57–82. mixed amphetamine salts
C. perform brain MRI
?? 24. A 7-year-old female has some atten- D. perform polysomnogram
8 tion problems, and poor school per- E. refer for neuropsychiatric
formance. Her teacher and parents ­evaluation

..      Fig. 8.23 With a permission from 7 ILAE.org, Diagnostic Manual, and 7 EpilepsyDiagnosis.org
Epilepsy 1
211 8
vv Correct Answer is: A ?? 25. An 8-year-old healthy boy had a sei-
The history is indicative of absence epi- zure last night. His mother reports
lepsy. EEG shows a 3 Hz generalized spike-­ that he was asleep in the back seat
and-­slow wave rhythm, classic for absence of the car when she saw him having
epilepsy. Patients will have many daily stiffening and shaking of his limbs,
spells of brief staring and eye blinking, twitching of his face and upward
without postictal confusion. Because of her deviation of his eyes. The event lasted
age, she has childhood absence epilepsy, approximately 30 s, and he was sleepy
which has a peak age of onset of 5–8 years. afterwards. His examination is normal.
Juvenile absence epilepsy has onset at EEG is shown.
10–12 years of age. The first-choice medica- What is your diagnosis?
tion for childhood absence epilepsy is eth- A. benign epilepsy with centrotem-
osuximide. Alternative choices are valproic poral spikes
acid and lamotrigine. On the other hand, B. frontal lobe epilepsy
oxcarbazepine, carbamazepine, tiagabine, C. idiopathic generalized epilepsy
gabapentin, and vigabatrin worsen the D. Panayiotopoulos syndrome
condition. Usually, childhood absence E. temporal lobe epilepsy
epilepsy is a transient disorder, but some
patients will develop myoclonic and tonic- vv Correct Answer is: A
clonic seizures and are diagnosed then The EEG shows benign childhood epilepsy
with juvenile myoclonic epilepsy. with centrotemporal spikes (BECTS). The
The juvenile absence epilepsy is not as location of the epileptiform discharges
benign, with seizures typically persisting (central and temporal) and morphol-
into adulthood, and these patients are also ogy (polyphasic dipole tangential to the
at higher risk of having generalized Rolandic region) are characteristic. BECTS
tonic-­clonic seizures as well, although is the most common partial epilepsy of
usually few in number. Methylphenidate childhood, accounting for approximately
and mixed amphetamine salts (Adderall) 2/3 of all idiopathic focal epilepsies of
are useful for ADHD, but are ineffective for childhood. The onset is 5–8 years of age.
absence seizures. Brain MRI is not indicated Focal motor seizures are the most com-
for absence seizure evaluation. mon seizure type and are often unilateral,
Polysomnogram might be indicated for involving the face, oropharynx and
further evaluation of the sleep disturbance, upper extremity. The seizures most often
but is not the best next step in her manage- occur during sleep; fewer than 25% of
ment. Neuropsychiatric evaluation would the seizures occur during wakefulness.
be indicated if scholastic difficulties persist Treatment can be delayed because many
despite adequate treatment of the absence patients will not have more than 1–2, and
seizures. remission of the epilepsy is expected.

zz Suggested Reading zz Suggested Reading


55 Glauser TA, Loddenkemper 55 Bergqvist CA. Idiopathic pediatric epilepsy
T. Management of childhood epilepsy. syndromes. Continuum Lifelong Learning
Continuum Lifelong Learning Neurol. Neurol. 2007;13(4):106–20.
2013;19(3):656–81.
55 Glauser TA, Cnaan A, Shinnar S, et al. ?? 26. A 17-year-old male collapsed to the
Ethosuximide, valproic acid, and lamotrig- ground at school. He denies any loss
ine in childhood absence epilepsy. N Engl of consciousness. He reported hav-
J Med. 2010;362:790–9. ing some jerking movements of his
212 Chapter 8 · Epilepsy 1

8 ..      Fig. 8.24 With a permission from 7 ILAE.org, Diagnostic Manual, and 7 EpilepsyDiagnosis.org

arms that morning after staying up this case and is often the preferred AED
studding late the night before. He has for men with generalized epilepsy. Other
those jerky movements of his arms for AEDs that may be used for this syndrome
several years, usually in the morning. include levetiracetam, zonisamide, topira-
He has a normal examination. EEG is mate and lamotrigine.
shown. Carbamazepine may worsen general-
What is the best treatment you can ized epilepsies, particularly myoclonic and
provide? absence types. Propranolol is not an anti-
A. carbamazepine seizure medication. Although sleep
B. clonazepam as needed deprivation is a known seizure trigger for
C. counsel patient to always get a patients with myoclonic epilepsy and
full night’s sleep avoidance of sleep deprivation should be
D. divalproex sodium recommended, this would not be ade-
E. propranolol quate as a single best treatment. Likewise,
clonazepam as needed might be helpful
vv Correct Answer is: D for patients with a tendency toward
The clinical history is consistent with myo- sustained seizure clusters, but would not
clonic seizures and it is confirmed by the be the best single treatment.
generalized 4–5 Hz spike-wave complexes
and polyspikes on his EEG. He has no his- zz Suggested Reading
tory of generalized tonic-clonic seizure, 55 Rudzinski LA, Shih JJ. Classification of
but he is at high risk for one and should seizures and epilepsy syndromes.
be treated with an antiepileptic drug Continuum Lifelong Learning Neurol.
(AED). ­Divalproex sodium is very useful in 2010;16(3):15–35.
Epilepsy 1
213 8

..      Fig. 8.25 Atypical generalized spike-wave IED. This Copyright 2013. Mayo Foundation for Medical Educa-
IED is most commonly seen in children with juvenile tion and Research. All rights reserved. (Figure courtesy
absence or myoclonic epilepsy syndromes. The dis- of Jeffrey W. Britton, MDElectroencephalography (EEG):
charges are induced by the activating procedure of An Introductory Text and Atlas of Normal and Abnormal
photic stimulation, representative of a so-called photo Findings in Adults, Children, and InfantsISBN: 978-0-
paroxysmal response, which often occurs in those with 9979756-0-4, Copyright c2016 by American Epilepsy
primary idiopathic generalized epilepsy syndromes. Society, 135 S. LaSalle St., Suite 2850, Chicago, IL 60603)

?? 27. A 14-year-old male has episodes of What is your diagnosis?


staring off space and Lip-­smacking for A. behavioral manifestations of
3 months. Past medical history is remark- ADHD
able of attention deficit hyperactivity B. childhood absence epilepsy
disorder (ADHD) and a family history of C. frontal lobe epilepsy
epilepsy. He is on atomoxetine. He has a D. juvenile absence epilepsy
normal examination. EEG is shown. E. medication side effects
214 Chapter 8 · Epilepsy 1

8 ..      Fig. 8.26 With a permission from 7 ILAE.org, Diagnostic Manual, and 7 EpilepsyDiagnosis.org

vv Correct Answer is: D the past 3 weeks. The family brought


The EEG is typical of juvenile absence epi- a video EEG, showed a repetitive
lepsy (JAE) with the characteristic 3.5–4 Hz symmetric flexion of his both arms
generalized spike- and polyspike-wave and head, lasting up to 5 s each and
discharges. The age onset for JAE is later repeating every 5–10 s for 15 min.
than childhood absence epilepsy, with He will cry and be fussy in between
typical onset at 10–16 years of age. The the events. The events occur in clus-
absence seizure semiology is similar, but ter around sleep-wake transitions.
the seizures are less frequent and longer Recently he showed mild delays in
in duration, with a higher incidence of motor skills with worsening of head
absence status and occasional generalized control and less vocal output since
tonic-clonic seizures. Rarely, they have these movements started. He has a
remission. The best treatment options are normal general examination, but the
valproic acid and lamotrigine, which are neurologic examination shows only
effective for both absence and tonic-clonic mildly decreased truncal tone. EEG is
seizures. shown.
What is the most appropriate diag-
zz Suggested Reading nosis?
55 Wirrell E, Nickels KC. Pediatric epilepsy A. benign myoclonic epilepsy of
syndromes. Continuum Lifelong Learning infancy
Neurol 2010;16(3):57–85. B. benign sleep myoclonus of
infancy
?? 28. A previously healthy 5-month-old C. infantile spasms
boy started to have an abnormal D. Lennox Gastaut syndrome
twitching of his arms and legs for E. Sandifer syndrome
Epilepsy 1
215 8

..      Fig. 8.27 With a permission from 7 ILAE.org, Diagnostic Manual, and 7 EpilepsyDiagnosis.org

vv Correct Answer is: C Hypsarrhythmia. Benign sleep myoclonus


The EEG pictured shows a very chaotic and of infancy and Sandifer syndrome are
disorganized background, with a mixture nonepileptic paroxysmal events. They
of very high amplitudes, and multifocal are commonly noted in young infants
and generalized spike and sharp com- and sometimes mistaken for seizures or
plexes. Additionally, noted is a brief 1-s infantile spasms, though they would not
segment of suppression of background, be associated with any EEG abnormalities.
referred to as “electrodecrement.” This Lennox Gastaut syndrome (LGS) is another
pattern comprises hypsarrhythmia, the type of epileptic encephalopathy in which
EEG pattern typical for infantile spasms patients have several seizure types (includ-
(and one of the key features of the triad of ing tonic, myoclonic, tonic-clonic, atypical
the initial description of West syndrome). absence, and partial) along with cognitive
The disorganized EEG background belies impairment and typically occurs in children
the prominent encephalopathy seen in older than 2 years of age. Infantile spasms
these patients. Although the myoclonic typically present in the 4- to 9-month-­old
seizures of benign myoclonic epilepsy of child, often with preceding developmental
infancy sometimes resemble an epileptic delay, though occasionally in a previously
spasm, the interictal EEG background normal infant. They typically appear as
for this epilepsy syndrome is normal for clusters of flexor or extensor movements
age and would not be confused with of the limbs, often with a quick myoclonic
216 Chapter 8 · Epilepsy 1

phase (and sometimes a very brief tonic 55 Go CY, Mackay MT, Weiss SK, et al.
phase lasting no more than 1–2 s). If in a Evidence-based guideline update: Medical
seated position, the child also will usu- treatment of infantile spasms. Report of
ally have head and neck atonia and lose the Guideline Development Subcommittee
postural control. Although these are of the American Academy of Neurology
frequently misdiagnosed as startle spells and the Practice Committee of the Child
or colic (due to some of the associated Neurology Society. Neurology.
fussiness during clusters), the movements 2012;78(24):1974–80.
are classic enough that diagnosis is often
able to be made by a qualified neurologist ?? 29. A 6-month-old boy is brought by
simply from viewing the spells in person or his family because of clusters of
on a video recording. Whether full hypsar- unusual movements. In the office
rhythmia is present on EEG or not, current you witnessed an episode of sudden
best practice recommends attempts to head drop and extension of his arms
stop the clinical spasms (and improve in front of this body. These tend to
the EEG) as soon as possible. Although cluster around sleep-wake transi-
outcome is typically most highly depen- tions, though he has clearly had clus-
dent on the specific cause, there is some
8 thought that duration of spasms may have
ters while already awake. His EEG is
shown.
an independent inverse prognostic effect? What is the best treatment for this
Common causes include birth related young child?
hypoxic-ischemic injury, cortical malforma- A. Adrenocorticotropic hormone
tions, genetic and metabolic disorders, B. Diazepam
neurocutaneous syndromes (especially C. Phenobarbital
tuberous sclerosis) and congenital infec- D. Reassurance only, he has benign
tions. Treatment is typically with some type myoclonus of infancy
of steroid therapy (ACTH versus predni- E. Vigabatrin
sone), though agents such as vigabatrin
have been particularly effective for spasms vv Correct Answer is: A
secondary to tuberous sclerosis. Other This a typical clinical history and EEG find-
more traditional anticonvulsant therapies ings of infantile spasms. The EEG shows
or pyridoxine have been tried, though hypsarrhythmia, with a very high-ampli-
often with poor overall success (except in tude and disorganized background with
the setting of true pyridoxine dependent both generalized spike and slow bursts
epilepsy.) The spasms usually go into and multifocal discharges. The EEG of the
remission with, or without treatment, over benign myoclonus of infancy, EEG is nor-
time; though the risk for ongoing devel- mal, and the spells would be aborted by
opmental delay and other seizure types waking the child up. Adrenocorticotropic
remains high. hormone (ACTH) is the first line therapy
for infantile spasms not associated with
zz Suggested Reading tuberous sclerosis. Vigabatrin is used in
55 Wirrell E, Nickels KC. Pediatric epilepsy children with infantile spasms associated
syndromes. C­ ontinuum Lifelong Learning with tuberous sclerosis. Phenobarbital
Neurol. 2010;16(3):70–5. is not a first line treatment modality.
Epilepsy 1
217 8

..      Fig. 8.28 Hypsarrhythmic EEG pattern in an infant All rights reserved. (Figure courtesy of Jeffrey W. Brit-
with West syndrome. EEG of an 8-month-old infant ton, MD Electroencephalography (EEG): An Introduc-
with infantile spasms and developmental regression. tory Text and Atlas of Normal and Abnormal Findings
An ictal electrodecremental response (black brackets) in Adults, Children, and Infants ISBN: 978-0-9979756-
is seen during an epileptic spasm. Copyright 2013. 0-4, Copyright c2016 by American Epilepsy Society,
Mayo Foundation for Medical Education and Research. 135 S. LaSalle St., Suite 2850, Chicago, IL 60603)

Benzodiazepines such as diazepam can be What is the most likely diagnosis?


used in conjunction with other agents for A. Childhood absence epilepsy
refractory spasms, but would not be used B. Frontal lobe epilepsy
as a first-line agent. C. Juvenile myoclonic epilepsy
D. Panayiotopoulos syndrome
zz Suggested Reading E. Reflex epilepsy
55 Arya R, Shinnar S, Glauser
TA. Corticosteroids for the treatment of vv Correct Answer is: A
infantile spasms: a systematic review. J The EEG shows the classic 3-Hz gen-
Child Neurol. 2012;27(10):1284–8. eralized spike-wave discharges of the
childhood absence epilepsy (CAE). The
?? 30. A 10-year-old female in your office disorder starts between 4 and 10 years
for an evaluation because she has a of age. The seizures are brief, lasting less
school difficulty. Her teachers noted than 10 , and may occur up to hundreds
that she frequently stare off space of times a day. Automatisms and brief
with twitching movements in her clonic jerks can be seen during absence
face. These episodes happen every seizures. The seizures can be provoked by
time she blows balloons. Her EEG is hyperventilation in approximately 90%
shown. of children.
218 Chapter 8 · Epilepsy 1

..      Fig. 8.29 With a permission from 7 ILAE.org, Diagnostic Manual, and 7 EpilepsyDiagnosis.org

zz Suggested Reading vv Correct Answer is: D


55 7 http://journals.lww.com/continuum/ She is severing of photosensitive seizures
Fulltext/2010/06000/PEDIATRIC_EPILEPSY_ and episodes of myoclonus, which is typi-
SYNDROMES.6.aspx. cal of juvenile myoclonic epilepsy. The EEG
shows generalized fast poly-spike wave dis-
?? 31. An 18-year-old female repots epi- charges which is consistent with JME. JME
sodes of time lapses for few years. is a common epilepsy syndrome (4–10% of
The time lapses usually occur while all patients with epilepsy). It starts between
is driving her car or jogging in the 12 and 18 years of age. Children often have
park, especially if the sun is flickering months to years of episodes of myoclonus
through the trees. She also reports and absence seizures. Routine EEG is usually
that she is clumsy in the morning and abnormal in most children with JME and
will sometimes give up on braiding shows 4– 6-Hz generalized spike and poly-­
her hair since she keeps dropping her spike wave discharges. Focal discharges
brush. Her EEG is shown. may also be present. Photosensitivity
What is your diagnosis? is present in 50–75% of people with
A. Childhood absence epilepsy JME. Neither childhood or juvenile absence
B. Frontal lobe epilepsy epilepsy have prominent myoclonus, nor
C. Juvenile absence epilepsy do the bursts of generalized epileptiform
D. Juvenile myoclonic epilepsy activity occur classically at 3–4 Hz. There
E. Progressive myoclonic epilepsy are several forms of progressive myoclonus
Epilepsy 1
219 8

..      Fig. 8.30 Myoclonic seizure in a patient with reserved. (Figure courtesy of Erik K. St. Louis, MDElec-
JME. The patient had a generalized axial myoclonic troencephalography (EEG): An Introductory Text and
jerk during second 14, which coincided with the gen- Atlas of Normal and Abnormal Findings in Adults, Chil-
eralized spike-wave discharge and electrodecremental dren, and Infants ISBN: 978-0-9979756-0-4, Copyright
pattern seen on EEG. Copyright 2013. Mayo Founda- c2016 by American Epilepsy Society, 135 S. LaSalle St.,
tion for Medical Education and Research. All rights Suite 2850, Chicago, IL 60603)

epilepsy, typically with myoclonus and left or right side of the face 2 years
some prominent cognitive regression or ago. Initial EEG showed bilaterally
visual loss. Frontal lobe epilepsy would independent triphasic sharp and
be expected to have frontally-­based focal slow complexes. Her pediatrician
epileptiform discharges (though interictal started her on carbamazepine, with
EEG is often normal), without absence or good control of her seizures until
myoclonic seizure types. 5 months ago, in conjunction with her
regressed language skills. A repeat
zz Suggested Reading EEG is ordered, with a segment of her
55 7 http://journals.lww.com/continuum/ sleep recording attached here.
Fulltext/2010/06000/PEDIATRIC_EPILEPSY_ What is the most appropriate diag-
SYNDROMES.6.aspx. nosis for this patient?
A. Landau-Kleffner syndrome
?? 32. A 6-year-old female starts to have B. Lennox-Gastaut syndrome
a language regression in the last C. Pervasive developmental disorder
5 months. She stated to have few D. Rett syndrome
brief focal seizures, involving the E. Rolandic epilepsy
220 Chapter 8 · Epilepsy 1

..      Fig. 8.31 With a permission from 7 ILAE.org, Diagnostic Manual, and 7 EpilepsyDiagnosis.org

vv Correct Answer is: A anticonvulsants such as valproic acid,


The EEG shows continuous generalized levetiracetam or lamotrigine. Use of high
spike and slow wave discharges seen in dose nightly benzodiazepines or even oral
sleep. Language regression with severe steroid treatment can also be helpful in
sleep-­activated epileptiform EEG is con- many cases. A normal development during
sistent with Landau-Kleffner syndrome, the first 5 years of life is not compatible
which has also been termed acquired with pervasive developmental disorder or
epileptiform aphasia. Classically those Rett syndrome. Lennox-Gastaut syndrome
children have normal development until is characterized by multiple generalized
sometime between the ages of 2–5, when seizure types, cognitive dysfunction, and
expressive and even receptive language EEG showing diffuse slow spike-and-wave
skills regress. Clinical seizures typically complexes. Again, most patients with an
occur prior to, or early on in the course of eventual diagnosis of this epilepsy syn-
the regression, though in some cases the drome are never developmentally normal.
seizures may occur later. Often, the initial
EEG can have features of rolandic epilepsy, ?? 33. A 6-year-old female complains of
and worsen with treatment with tradi- occasional staring spells for the last
tional sodium channel blocking agents 3 months. She has multiple spells a day
such as carbamazepine or oxcarbazepine. and last about 10 s. Occasionally they
Although epileptiform discharges in are accompanied by eyelid fluttering.
Rolandic epilepsy increased in sleep, Also, she has trouble paying attention
they do not generalize as seen in Landau- in the classroom. Her medical history is
Kleffner syndrome. Treatment of Landau- unremarkable and her examination is
Kleffner involves use of broader spectrum normal. Her EEG is shown.
Epilepsy 1
221 8
What is best medications to be used C. lamotrigine
first? D. methylphenidate
A. carbamazepine E. valproic aid
B. ethosuximide

..      Fig. 8.32 Absence seizure. During an absence Education and Research. All rights reserved. (Figure
seizure, the typical ictal EEG pattern is an extension of courtesy of Erik K. St. Louis, MD Electroencephalogra-
the interictal pattern (i.e., 3-Hz generalized spike-wave phy (EEG): An Introductory Text and Atlas of Normal
of a prolonged duration, with usual clinical accompani- and Abnormal Findings in Adults, Children, and Infants
ments of staring with behavioral arrest, or with variable ISBN: 978-0-9979756-0-4, Copyright c2016 by American
accompaniment by oral and manual automatisms such Epilepsy Society, 135 S. LaSalle St., Suite 2850, Chicago,
as blinking, lip smacking, or hand-fumbling move- IL 60603)
ments. Copyright 2013. Mayo Foundation for Medical

vv Correct Answer is: B zz Suggested Reading


The clinical presentation is consistent with 55 Glauser, TA, Cnaan A, Shinnar S, et al.
a diagnosis of childhood absence epilepsy Ethosuximide, valproic acid and lamotrig-
and supported with the EEG showing the ine in childhood absence epilepsy. N Engl
3 Hz spike-wave discharges. Ethosuximide, J Med 2010;362(9):790–9.
lamotrigine and valproic acid can all be
used to treat this syndrome, but ethosuxi- ?? 34. An 18-year-old female with right
mide has been shown to be more effective temporal lobe epilepsy has her first
than lamotrigine and associated with less healthy baby. She is planning to
side effects compared to valproic acid. breastfeed her baby but she does
Carbamazepine may worsen absence sei- not know the risks of her anti- seizure
zures and is contraindicated in this situa- medications on her baby. She had no
tion. Her attentional problems may be due seizures during her pregnancy and is
to uncontrolled seizure activity and meth- currently on levetiracetam and carba-
ylphenidate is not indicated at this time. mazepine.
222 Chapter 8 · Epilepsy 1

Based on evidence-­based medicine for the past 2 months. He has a history


which of the following statements is of sleep walking but does not sleepwalk
most accurate regarding breastfeed- during these episodes. There is a family
ing in this situation? history of his father and paternal uncle
A. carbamazepine and levetiracetam had childhood epilepsy and his mater-
both cross the placenta so further nal uncle has Tourette syndrome. He
exposure via breastmilk would not has a normal examination and develop-
contraindicate breastfeeding mental milestones. EEG is shown.
B. risks are clearly defined and she Which of the following is the most
should not breastfeed accurate statement regarding this
C. risks are unknown and it is better patient?
not to breastfeed A. diagnosis is Panayiotopoulos syn-
D. she can breastfeed, but should drome
expect increased fussiness and B. diagnosis is benign Rolandic epi-
lethargy in the infant lepsy
E. transference of carbamazepine C. First-line treatment is low dose
into the breast milk is much clonazepam before bedtime
greater than levetiracetam D. genetic testing for autosomal
8 dominant frontal lobe epilepsy is
vv Correct Answer is: A indicated
There are no controlled studies comparing E. polysomnogram with full montage
symptomatic effects from AEDs in infant’s EEG during one of the episodes is
breast fed by women with epilepsy. At this necessary for accurate diagnosis
point the clinical consequences for the
newborn ingesting AEDs via breast milk are vv Correct Answer is: B
unknown. Phenobarbital, primidone, phe- The EEG shows benign childhood epilepsy
nytoin, carbamazepine, levetiracetam, gab- with centrotemporal spikes (BECTS). The
apentin, lamotrigine, oxcarbazepine and location of the epileptiform discharges
topiramate cross the placenta and may be (central and temporal) and morphol-
factored into the clinical decision making. ogy (polyphasic dipole tangential to the
Studies have shown significant penetration Rolandic region) are characteristic. BECTS is
of levetiracetam into breast milk (as well the most common partial epilepsy of child-
as gabapentin, lamotrigine, topiramate hood, accounting for approximately 2/3 of
and primidone). Valproate, phenobarbital, all idiopathic focal epilepsies of childhood.
carbamazepine and phenytoin do not sig- The onset is 5–8 years of age. Focal motor
nificantly penetrate into the breast milk. seizures are the most common seizure type
and are often unilateral, involving the face,
zz Suggested Reading oropharynx and upper extremity. The sei-
55 Harden CL, Pennell BP, Koppel BS, et al. zures most often occur during sleep; fewer
Practice Parameter update: Management than 25% of the seizures occur during wake-
issues for women with epilepsy—focus on fulness. Treatment can be delayed because
pregnancy (an evidence-based review): many patients will not have more than 1–2,
Vitamin K, folic acid, blood levels and and remission of the epilepsy is expected.
breastfeeding. Neurology 2009;73:142–149.
zz Suggested Reading
?? 35. A mother of 6-year-old boy said that her 55 Bergqvist CA. Idiopathic pediatric epilepsy
son has a weekly event of drooling and syndromes. Continuum Lifelong Learning
movements of his mouth while sleeping Neurol 2007;13(4):106–20.
Epilepsy 1
223 8

..      Fig. 8.33 With a permission from 7 ILAE.org, Diagnostic Manual, and 7 EpilepsyDiagnosis.org

?? 36. A family of 5-month-­old girl brought superior compared to lower dos-


her because of recurrent episodes ing regimens (20–30 IU qD)
of abnormal movements, occur at B. this condition typically resolves
sleep-wake transitions over the last without treatment by 2 years of
6 weeks. She has some mild gross age; therefore, patients should
motor milestones delay and she has be spared the high likelihood of
been less active and visually attentive corticosteroid induced side effects
over the past 4 weeks. You reviewed a and should only be treated if they
video recording brought by the fam- develop other types of seizures
ily, shows a brief cluster episode of C. oral prednisone is more effica-
3-second of head drop, arm and leg cious than injectable preparations
flexion associated with fussiness in of steroid treatment
between. The spells are not rhythmic, D. pyridoxine (vitamin B6) has been
but are recurrent and stereotyped. shown to be effective in the treat-
Her general examination is normal, ment of most patients with this
but her neurological exam showed condition
poor ocular tracking, hypotonia in E. vigabatrin may be as effective as
the neck and truncal muscles, with corticosteroids in the treatment
slightly brisk deep tendon reflexes. of children with this condition,
Brain MRI is normal. Her EEG is shown. especially in symptomatic cases
Which of the following statements associated with tuberous sclerosis
is true regarding the treatment of
this patient’s condition according to vv Correct Answer is: E
a recent updated evidenced-based Infantile spasms are a well-known age
guideline from the American Acad- dependent seizure manifestation most
emy of Neurology and the Child often occurring in infants 4–12 months of
Neurology Society? age. Considered one of the earliest epi-
A. high dose ACTH (150 IU/m2 qD) leptic encephalopathies, developmental
has clearly been shown to be regression commonly occurs due to the
224 Chapter 8 · Epilepsy 1

8
..      Fig. 8.34 Hypsarrhythmic EEG pattern in an infant All rights reserved. (Figure courtesy of Jeffrey W. Brit-
with West syndrome. EEG of an 8-month-old infant ton, MD Electroencephalography (EEG): An Introduc-
with infantile spasms and developmental regression. tory Text and Atlas of Normal and Abnormal Findings
An ictal electrodecremental response (black brackets) in Adults, Children, and Infants ISBN: 978-0-9979756-
is seen during an epileptic spasm. Copyright 2013. 0-4, Copyright c2016 by American Epilepsy Society,
Mayo Foundation for Medical Education and Research. 135 S. LaSalle St., Suite 2850, Chicago, IL 60603)

consistently abnormal EEG manifest- as well as normalization of the EEG offers


ing extremely high amplitude focal and the best potential for long-term cognitive
generalized bursts of sharply contoured outcomes. The outcome, however, remains
and spike activity, often intermixed with highly dependent on the etiology, with
an electro-decremental response and symptomatic causes (prior brain injury,
disorganized background. Although the CNS malformations, chromosomal and
causes of infantile spasms are diverse, the metabolic disorders) having a much higher
clinical and electrographic features remain risk of long-­term intellectual disability and
quite consistent. The events can be quite intractable epilepsy than the idiopathic
subtle at onset and often escape diagnosis cases. Even this last group, however,
for weeks to months. During this period of has nearly a 50–75% risk of long-term
time there can also be loss of previously neurologic disability. Although practices
acquired milestones such as loss of head or vary greatly worldwide, corticosteroid
trunk control, language abilities or visual treatment remains the mainstay for the
attention. Although limited natural history majority of patients treated in developed
studies show a spontaneous remission countries. The recent 2012 updated guide-
rate of spasms in up to 30% of cases, it has lines from the AAN mention that amongst
traditionally been accepted that prompt the steroid treatment options, injectable
recognition and treatment of the spasms, ACTH appears to have the best evidence,
Epilepsy 1
225 8
though lower end dosing appears to be a zz Suggested Reading
very reasonable option. There was insuf- 55 Go CY, MacKay MT, Weiss SK, et al.
ficient evidence to recommend using alter- Evidence-based guideline update: Medical
native oral or intravenous preparations as treatment of infantile spasms. Report of
being as effective as ACTH for short-term the Guideline Development Subcommittee
treatment of spasms, though clearly indi- of the American Academy of Neurology
vidual case series have shown these agents and the Practice Committee of the Child
to be potentially effective. Vigabatrin has Neuro.
also been recommended as a potential
treatment, especially in the setting of
symptomatic spasms associated with ?? 37. Spike and wave seizure-­detection
tuberous sclerosis. The evidence from the software has flagged this portion of
guideline suggests that ACTH may be more an EEG recording.
effective when considering all etiologic What is the most likely diagnosis?
conditions. Although used as either initial A. electrode pop artifact
therapy or as adjunctive treatment for B. filtering artifact
refractory infantile spasms there remains C. left parietal spike-and-­wave dis-
insufficient evidence to recommend other charge
therapies such as valproic acid, pyridoxine, D. right parietal spike-­and-­wave dis-
levetiracetam, zonisamide, topiramate and charge
the ketogenic diet. E. sneeze artifact

..      Fig. 8.35 American Academy of Neurology Institute, produced by permission


226 Chapter 8 · Epilepsy 1

vv Correct Answer is: A zz Suggested Reading


Any unusual waveform that is confined 55 Ebersole JS, Pedley TA. Current practice of
to a single or common electrode, as in clinical electroencephalography. 3rd ed.
this tracing at T5, must be considered Philadelphia: Wolters Kluwer; 2003. 180
artifactual. Note the sudden high ampli- p. 282–3.
tude deflection that involves only a single
electrode. Epileptiform spike-and-wave ?? 38. The EEG tracing shown is typical for
discharges would be expected to demon- which diagnoses?
strate a phase reversal with involvement A. benign myoclonus of childhood
of adjacent electrodes. A sneeze, or other B. childhood absence epilepsy
generalized bodily movement, would C. infantile spasms (epileptic
introduce generalized artifact. The deflec- spasms)
tion is not due to an incorrect filter setting D. juvenile myoclonic epilepsy
since it affects only one electrode. E. Landau-Kleffner syndrome

..      Fig. 8.36 With a permission from 7 ILAE.org, Diagnostic Manual, and 7 EpilepsyDiagnosis.org

vv Correct Answer is: C which typically have an age of onset of


The EEG shows hypsarrhythmia, which is 4–7 months old.
characterized by a chaotic, high amplitude,
asynchronous background. Generalized zz Suggested Reading
spike-wave activity, periods of focal or 55 Ebersole JS, Pedley TA, editors.
generalized voltage attenuation or focal Current practice of clinical electroen-
spike-­wave activity may be present in the cephalography. 3rd ed. Philadelphia:
recording. Hypsarrhythmia is associated Lippincott, Williams & Wilkins;
with infantile spasms (epileptic spasms), 2003.
Epilepsy 1
227 8
?? 39. A 6 -year-old female complains of D. juvenile myoclonic epilepsy
spells characterized by staring, eye E. Lennox-Gastaut syndrome
flutter, and unresponsiveness lasting
5–10 s. Her EEG is shown. vv Correct Answer is: B
What is the most likely diagnosis? The clinical scenario indicates absence
A. benign rolandic epilepsy of child- seizures. The EEG shows a 3 Hz general-
hood ized spike-wave pattern, which is classic
B. childhood absence epilepsy for absence seizures. Childhood absence
C. juvenile absence epilepsy epilepsy begins before age 8.

..      Fig. 8.37 With a permission from 7 ILAE.org, Diagnostic Manual, and 7 EpilepsyDiagnosis.org

zz Suggested Reading vv Correct Answer is: B


55 Rudzinski LA, Shih JJ. The classification of The electrocardiogram (EKG) shown was
seizures and epilepsy syndromes. recorded at the scalp and represents a far
Continuum Lifelong Learning Neurol. field potential, most often seen in referential
2010;16(3):15–35. montages. Note that the sharply contoured
waveforms are uniform across the cerebral
?? 40. The EEG tracing shown demon- hemisphere and are time locked with the R-R
strates which of the following intervals in the EKG tracing at the bottom of
­findings? the image. EKG artifact can mimic cerebral
A. cerebral sharp waves sharp waves or spikes and are distinguished
B. electrocardiogram artifact from them by the distribution and timing.
C. periodic lateralized epileptiform
discharges zz Suggested Reading
D. periodic spike-wave discharges 55 Ebersole JS, Pedley TA. Current practice of
E. pseudoperiodic lateralized epilep- clinical electroencephalography. 3rd ed.
tiform discharges Philadelphia: Wolters Kluwer; 2003. p. 275–6.
228 Chapter 8 · Epilepsy 1

..      Fig. 8.38 American Academy of Neurology Institute, produced by permission

?? 41. In a 2-week-old full term neonate, the C. tonic seizures


trace alternant pattern is seen during D. transition from active sleep to
8 which of the following states? wakefulness
A. active sleep E. wakefulness
B. quiet sleep

..      Fig. 8.39 Trace alternant. This is a neonatal Electroencephalography (EEG): An Introductory


EEG sample of a 25-day-old girl born at 39 weeks Text and Atlas of Normal and Abnormal Findings in
GA. The sample shows a segment of quiet sleep Adults, Children, and Infants ISBN: 978-0-9979756-
with trace alternant. (Figure courtesy of Elia 0-4, Copyright c2016 by American Epilepsy Society,
M. Pestana-Knight, MD, Cleveland Clinic Foundation 135 S. LaSalle St., Suite 2850, Chicago, IL 60603)
Epilepsy 1
229 8
vv Correct Answer is: B Lippincott Williams & Wilkins, 2003.
In the term or immediate post-term p. 183–5.
neonate, the trace alternant pattern is a
normal sleep pattern present only in quiet ?? 42. The tracing shown represents which
sleep. of the following?
A. 6 Hz spike-and-wave
B. sleep spindles
zz Suggested Reading C. small sharp spikes
55 Ebersole JS, Pedley TA, editors. D. temporal intermittent rhythmic
Current practice of clinical electroenceph- delta activity (TIRDA)
alography. 3rd ed. Philadelpia: E. wicket waves

..      Fig. 8.40 Typical wicket waves. Note the arciform Research. All rights reserved. (Figure courtesy of
appearance, lack of after-going slow wave, and Jeffrey W. Britton, MD.Electroencephalography
lack of background disruption or disturbance. The (EEG): An Introductory Text and Atlas of Normal and
wicket waves are seen in the left temporal region Abnormal Findings in Adults, Children, and Infants
with phase-reversal at T7 in seconds 3 and 4 of the ISBN: 978-0-9979756-0-4, Copyright c2016 by Ameri-
tracing, longitudinal bipolar montage. Copyright can Epilepsy Society, 135 S. LaSalle St., Suite 2850,
2013. Mayo Foundation for Medical Education and Chicago, IL 60603)

vv Correct Answer is: E most commonly mistaken for epileptiform


Wicket waves are a benign variant often discharges. They are typically mid-temporal,
seen during drowsiness and light sleep 5–11 Hz, and typically occur in bursts. There
in adults. It is one of the benign variants is no focal slowing or slow wave component.
230 Chapter 8 · Epilepsy 1

zz Suggested Reading gelastic seizures (associated with ictal


55 Tatum WO, Husain AM, Benbadis SR, laughter). Gelastic seizures are particularly
Kaplan PW. Normal adult EEG and associated with hypothalamic lesions,
patterns of uncertain significance. J Clin which may project seizure discharges to
Neurophysiol. 2006;23:194–207. various parts of the cortex, but it can be
seen in cortical epilepsy. In fact, it may
?? 43. A 7-year-old female came for an be very challenging to demonstrate clear
evaluation for her inappropriate and ictal evolution during a scalp EEG record-
uncontrollable laughing spells for ing (as noted in the case above, where
2 years. She has features of precocious only diffuse suppression of the waking
puberty. Video/EEG monitoring was background is noted), depth electrodes
ordered and captured multiple laugh- may be required to better characterize
ing fits. During laughing, the ictal EEG the ictal pattern. Surgery in the vicinity
showed a diffuse suppression of back- of the hypothalamus is very hazardous as
ground rhythm; MRI is shown. it can be very disruptive of many critical
What is the etiology of her events? neuroendocrine pathways, though more
A. cordoma selective radiosurgery offers promise of a
B. frontal lobe epilepsy slightly safer intervention strategy
8 C. hypothalamic hamartoma
D. psychogenic zz Suggested Reading
E. skull base meningioma 55 Berkovic SF, Azrimanoglou A, Kuzniecky
R, et al. Hypothalamic hamartoma and
seizures: a treatable epileptic encephalopa-
thy. Epilepsia. 2003;44(7):969–73.

?? 44. A 5-year-old female with a history of


occasional spells of lip and right hand
clonic movements for few seconds
and she cannot talk during those
spells and accompanied with drooling
as well. Her medical history is unre-
markable. She is a good student with
no learning difficulties. Her brain MRI
is normal. Her EEG is shown.
What is your diagnosis?
A. Benign Rolandic epilepsy
B. Landau-Kleffner syndrome
C. Panayiotopoulos syndrome
..      Fig. 8.41 With a permission from 7 ILAE.org,
D. Rett syndrome
Diagnostic Manual, and 7 EpilepsyDiagnosis.org
E. Right mesial temporal sclerosis

vv Correct Answer is: C vv Correct Answer is: A


The clinical and radiographic features The EEG shows benign childhood epilepsy
are of the syndrome of hypothalamic with centrotemporal spikes (BECTS). The
hamartoma with epilepsy. This diagnosis location of the epileptiform discharges
is often quite devastating, with varying (central and temporal) and morphol-
degrees of mental retardation and, often, ogy (polyphasic dipole tangential to the
multiple seizure types that are often rolandic region) are characteristic. BECTS
refractory, including the well described is the most common partial epilepsy of
Epilepsy 1
231 8

..      Fig. 8.42 With a permission from 7 ILAE.org, Diagnostic Manual, and 7 EpilepsyDiagnosis.org

childhood, accounting for approximately and a n


­ ormal MRI and bilaterally inde-
2/3 of all idiopathic focal epilepsies of child- pendent discharges would not suggest a
hood. The onset is 5–8 years of age. Focal diagnosis of right sided mesial temporal
motor seizures are the most common sei- sclerosis.
zure type and are often unilateral, involving
the face, oropharynx and upper extremity. zz Suggested Reading
The seizures most often occur during sleep; 55 Wirrell E, Nickels K. Pediatric epilepsy
fewer than 25% of the seizures occur dur- syndromes. Continuum: Lifelong Learning
ing wakefulness. Treatment can be delayed Neurol. 2010;16(3): 57–85.
because many patients will not have more
than 1 to 2, and remission of the epilepsy is
expected. ?? 45. A previously healthy 8-month old
Panayiotopoulos syndrome is a benign infant boy came with his mother to
occipital onset epilepsy in early child- see you for recurrent clusters of sud-
hood, with interictal discharges in the den flexion at the trunk and neck with
posterior brain regions and clinical arms extension spells. They last for
semiology of ictal vomiting, and eye 1–2 s and occur in multiple times a
version with complex partial or secondary day. They have increased in frequency
generalized seizures. Landau-­Kleffner in the last month and he starts to
syndrome is one of the pediatric epileptic struggle to sit up and does not babble
encephalopathies, where dramatic as often. EEG and MRI are shown.
language regression accompanies What is the most likely etiology?
seizures, and an EEG that shows nearly A. Tuberous sclerosis
continuous epileptiform activity in slow B. Down Syndrome
wave sleep. C. Trauma
Rett syndrome would not be sug- D. Mitochondrial disease
gested in a typically developing child E. Infection
232 Chapter 8 · Epilepsy 1

..      Figs. 8.43 and 8.44 With a permission from 7 ILAE.org, Diagnostic Manual, and 7 EpilepsyDiagnosis.org
Epilepsy 1
233 8
vv Correct Answer is: A
This is a case of infantile spasms, which
can have many different etiologies. The
EEG shows slow disorganized background
and continuous high amplitude polymor-
phic generalized slowing and multifocal
spikes which is consistent hypsarrhythmia.
West syndrome is the triad of infantile
spasms, hypsarrhythmia and develop-
mental delay. West syndrome has many
different etiologies as well. Approximately
70% of patients with West syndrome have
abnormal MRI findings. The most common
etiology is tuberous sclerosis as seen in
this MRI. The MRI shows multiple focal
areas of broadened gyri, blurring of the
gray-white junction and increased signal
in the subcortical white matter typical of
cortical tubers. There are also subependy-
mal nodules consistent with sub-ependy-
mal hamartomas.

zz Suggested Reading
55 7 http://journals.lww.com/continuum/
Fulltext/2015/04000/BenignBrainTumorsand
TumorsAssociatedWith.12.aspx.
..      Figs. 8.45 and 8.46 With a permission from 7 ILAE.
?? 46. A 7 y7-year-oldy just moved with org, Diagnostic Manual, and 7 EpilepsyDiagnosis.org
his family to your area. His family
brought a previous CT to the visit
(see figure). capillary-­venous malformation. The pres-
Which of the following features, if ence of facial capillary malformation (a
present, would be most specific in port wine stain) in the 1st or 2nd division of
making a definitive diagnosis? the trigeminal nerve in this setting would
A. Autosomal recessive inheritance be diagnostic of Sturge-Weber syndrome
pattern (SWS). This neuro-cutaneous finding is
B. Facial capillary malformation typically ipsilateral to the cerebral vascular
C. Failure to meet developmental malformation. An autosomal dominant
milestones in the first few months inheritance pattern would be atypical.
of life SWS, although congenital, is a sporadic
D. Retinal astrocytoma on dilated disease and so unlikely to occur in siblings
eye exam or offspring of affected individuals.
E. Seizures in the first year of life Glaucoma is a common but inconsis-
tent feature of SWS, occurring in 30–70%
vv Correct Answer is: B of patients with SWS. Glaucoma may be
The MRI shows right parieto-occipital pial congenital or develop later in life. In the
enhancement, gyral calcifications, and rare patient without facial capillary
atrophy consistent with leptomeningeal malformation, the presence of glaucoma
234 Chapter 8 · Epilepsy 1

and typical MRI pattern is highly sugges- zz Suggested Reading


tive of SWS. Retinal astrocytoma is seen in 55 Thomas-Sohl KA, Vaslow DF, Maria
patients with tuberous sclerosis, not SWS. BL. Sturge-Weber syndrome: a review.
Though seizures and developmental Pediatr Neurol. 2004;30(5):303–10.
delay occur commonly in patients with
SWS, (with 95% having developed ?? 47. The tracing shown is recorded initially
seizures by age five), neither are specific while the patient is reading. Beside
to SWS and may occur in a multitude of the eye closure. What another wave is
congenital and developmental diseases. presented here?

..      Fig. 8.47 Lambda waves. Lambda waves over of Jeffrey W. Britton, MDElectroencephalography (EEG):
posterior head regions, elicited by complex pattern An Introductory Text and Atlas of Normal and Abnor-
viewing. Note the surface positive waveforms over mal Findings in Adults, Children, and Infants ISBN:
both occipital regions. Longitudinal bipolar montage. 978–0–9,979,756-0-4, Copyright c2016 by American
Copyright 2013. Mayo Foundation for Medical Educa- Epilepsy Society, 135 S. LaSalle St., Suite 2850, Chicago,
tion and Research. All rights reserved. (Figure courtesy IL 60603)

A. benign lambda waves vv Correct Answer is: A


B. benign small sharp spikes Although sharply contoured, lambda
C. benign vertex waves (“V waves”) waves are a normal finding and occur in
D. epileptiform lambda waves occipital leads when the subject is scan-
E. epileptiform sharp waves ning with rapid saccadic eye movements
Epilepsy 1
235 8
such as during reading. They are surface zz Suggested Reading
positive and occur bilaterally. They can 55 Tatum WO, Husain AM, Benbadis SR,
be asymmetric and easily misinterpreted Kaplan PW. Normal adult EEG and
as epileptiform discharges. They are not patterns of uncertain significance. J Clin
associated with epilepsy, and they do not Neurophysiol. 2006;23:194–207.
occur over the vertex. They can be abol-
ished by closing the eyes or eliminating
the visual stimulus producing them.
237 9

Epilepsy 2

© Springer International Publishing AG, part of Springer Nature 2018


Y. M. Awaad, Absolute Pediatric Neurology, https://doi.org/10.1007/978-3-319-78801-2_9
238 Chapter 9 · Epilepsy 2

?? 1. You have been consulted on an with febrile status epilepticus compared


actively seizing 16-month-old boy to children with simple febrile seizures.
brought to the ED with a fever of Also, there is no increased risk for epi-
104 °f. He has been seizing for approx- lepsy or for recurrent febrile seizures or
imately 30 min despite his family gave febrile status epilepticus. On the other
him 0.5 mg/kg diazepam rectally prior hand, children with preexisting neuro-
to the arrival of EMS and he received logical deficits have a much higher risk
another 0.3 mg/kg diazepam intrave- of recurrent febrile status epilepticus
nously upon arrival to the ED. The sei- and rescue medicine should be consid-
zure ultimately stopped after he was ered. It has also been shown that intra-
given a 20 mg/kg load of intravenous venous phenytoin is much less effective
phenobarbital. Past medical history in aborting status epilepticus than intra-
is remarkable for a prior self-limited venous phenobarbital.
febrile seizure at age 12 months,
and a paternal cousin with a febrile zz Suggested Reading
seizures in. He has a normal develop- 55 Ismail S, Levy A, Tikkanen H, et al. Lack
mental milestone. of efficacy of phenytoin in children
Which statement below is correct? presenting with febrile status epilepticus.
A. At the time of discharge, this Am J Emerg Med. 2012;30(9):2000–4.
child should be sent home with 55 Maytal J, Shinnar S. Febrile status epilepti-
9 rectal diazepam due to a high risk cus. Pediatrics. 1990;86(4):611–6.
of developing recurrent febrile
­status epilepticus. ?? 2. A 4-year-old girl came with her
B. Due to the risk of respiratory sup- adoptive mother for evaluation of
pression with the combination frequent brief episodes of shaking-
of benzodiazepines and barbitu- type movements affecting her limbs
rates, intravenous fosphenytoin and trunk and often causing falls.
should have been used to break There is no loss of consciousness with
the seizure as opposed to intrave- these episodes and no apparent aura
nous phenobarbital. or post-ictal state following. They
C. An EEG should be performed due occur multiple times per day and
to this child’s increased risk of they are a couple of seconds in dura-
developing epilepsy. tion, though longer episodes have
D. The parents of this child should also been recorded. On examination,
be counseled about seizure man- she has normal tone, plantar flexor
agement as this patient has an responses, and easily elicitable DTR’s.
approximately 30% risk of febrile There was no tremor or ataxia. There
seizure recurrence. were no involuntary movements
E. This child should be sent home on seen.
maintenance phenobarbital due The next step in her diagnostic evalua-
to his increased risk of neurologi- tion should be:
cal morbidity due to his febrile A. Trial of low-dose carbamazepine
status epilepticus. B. Overnight video EEG to get an
ictal recording
vv Correct Answer is: D C. Asking the father to record the
There is no increased risk of death or events on home video
neurological morbidity in neurologically D. Answers B and C
and developmentally normal children E. Any of the above (A, B, or C)
Epilepsy 2
239 9
vv Correct Answer is: E receiving intravenous phenobarbital
The clinical presentation looks like a par- and levetiracetam with high thera-
oxysmal movement disorder that was not peutic levels and oral pyridoxine at
directly witnessed in the office, with an 30/mg/kg/day QID. To protect the
essentially normal neurological exami- child against central apnea, the child
nation. Paroxysmal events may include is also receiving oral caffeine. Yet
seizures, migraine, Channelopathies, despite these interventions, the infant
behavioral, or involuntary movements. is still having multiple daily seizures
Because the short duration of these often associated with oxygen desatu-
patients’ events, an ictal EEG recording rations. The neonatologists ordered a
may be helpful for eliminating seizures. brain MRI which is normal as well as a
Video clips brought by the family of their lumbar puncture with 12 WBC’s, protein
children when they are having an event of 60 mg/dl, glucose of 42 mg/dl with
can be extremely useful. The neurologist a serum glucose of 80 mg/dl. Gram
will be able to actually see the details of stain and cultures are both negative.
the events in “real time” instead of just The child is receiving ampicillin, cefo-
creating a mental image of the event from taxime and acyclovir as prophylaxis
the family’s history. Given the age of onset for a possible CNS infection and the
and details of her spells, it is highly likely child’s newborn screen results are
that she has paroxysmal kinesiogenic normal. You are considering glucose
choreoathetosis. So forth, a diagnostic transporter deficiency and are consid-
trial of low dose carbamazepine (other ering initiation of the ketogenic diet.
anticonvulsants have also been success- What other interventions should
fully used) is also a reasonable approach. be undertaken to help control this
Autosomal dominant familial paroxysmal patient’s epilepsy?
choreoathetosis has been linked in many A. Add intravenous Valproic acid.
families to mutations in the PRRT2 gene B. Discontinue the levetiracetam.
on chromosome 16q11.2-q12.1. The disor- C. Discontinue the phenobarbital
der is autosomal dominant with variable and caffeine.
penetrance. As this child was adopted, her D. Increase the serum glucose con-
family history, which may also be helpful centration by using 15% dextrose
with defining the cause of the movement intravenous fluids to overcome
disorder, was unknown. the relatively low CSF glucose
concentrations.
zz Suggested Reading E. Perform another lumbar puncture
55 Bhatia KP. Familial (idiopathic) paroxys- to measure CSF and concurrent
mal dyskinesias. Semm Neurol. glucose levels since the first
2001;21:69–74. results were borderline.
55 Erro R, Sheerin UM, Bhatia KP.
Paroxysmal dyskinesias revisited: a review vv Correct Answer is: C
of 500 genetically proven cases and a new The clinical presentation meets diag-
classification. Mov Disord. nostic criteria for glucose transporter
2014;29(9):1108–16. deficiency with a CSF glucose of 42 mg/
dl. In 90% of patients with Glucosec-1
?? 3. You have been consulted on a full transporter deficiency (GluT1DS) CSF
term 10 days old male infant in the glucose concentrations is below 40 mg/dl,
NICU, who has multiple refractory the other 10% have glucose concentra-
focal motor and tonic seizures. He is tions between 40 and 60 mg/dl. Also,
240 Chapter 9 · Epilepsy 2

while this patient’s CSF/serum ratio is was waiting for a neurology appoint-
0.51 and above the typical cut-­off value ment. One day before her scheduled
of less than 0.4, total CSF glucose con- office visit with you, she developed
centrations have been found to be more one of her usual headaches associ-
reliable in diagnosing GluT1DS than ated with blurry vision as well as
just the ratio. The ketogenic diet is the severe nausea. On examination, the
treatment of choice for patients with patient’s visual acuity is 20/200 OU at
GluT1DS as the result is to create ketone 20 feet and 20/40 OU at near. Fundo-
bodies that use a different transporter scopic examination and her extraocu-
system to cross the blood-­brain-­barrier lar movements are normal. The rest
and provide an alternative energy of her neurological examination is
source for neurons. Medications such normal.
as barbiturates, Valproic acid, benzodi- At this time, the best course of action
azepines and methylxanthines should should be to:
be avoided in patients with GluT1DS as A. Add acetazolamide to the topira-
they tend to competitively inhibit the mate to treat presumptive benign
brain glucose transporter. intracranial hypertension without
papilledema.
zz Suggested Reading B. Increase the topiramate to 100 mg
55 De Giorgis V, Veggiotti P. GLUT1 defi- twice a day to get her in the thera-
9 ciency syndrome 2013: current state of the peutic range for epilepsy.
art. Seizure. 2013;22(10):803–11. 7 https:// C. Schedule her for an outpatient
doi.org/10.1016/j.seizure.2013.07.003. Epub optometrist visit to evaluate her
2013 Jul 26. Review. for glasses.
55 Wang D, Pascual JM, DeVivo D. Glucose D. Immediately discontinue the topi-
Transporter Type 1 Deficiency Syndrome. ramate and arrange for an urgent
In: Pagon RA, Adam MP, Ardinger HH, ophthalmology consultation
Bird TD, Dolan CR, Fong CT, Smith RJH, through the emergency room.
Stephens K, editors. GeneReviews® E. Given her past psychiatric history,
[Internet]. Seattle: University of diagnose her with conversion
Washington, Seattle; 1993–2014. 2002 Jul disorder and schedule her for an
30 [updated 2012 Aug 09]. outpatient psychology evaluation.

?? 4. A 17-year-old female came to your vv Correct Answer is: D


office with a history of migraine The clinical scenario is of bilateral acute
headaches and epilepsy. She was angle closure glaucoma (AACG). This
diagnosed with generalized anxiety idiosyncratic side effect of topiramate
disorder and has been taking par- usually occurs around 2 weeks after
oxetine 40 mg per day for the past starting the medication. The mechanism
9 months which has helped with her of action is believed to be secondary to
symptoms. Currently she has daily ciliary body edema which then results
migraine headaches. She had her sec- in anterior displacement of the iris and
ond convulsive seizure 3 months ago. closure of the anterior chamber angle
Her PCP prescribed topiramate, 50 mg of the eye. Symptoms are blurred vision
once a day, 2 weeks ago in an attempt and myopia with or without ocular pain
to treat both her epilepsy and and/or conjunctival injection. SSRI’s
migraine headaches while the patient given concurrently with topiramate,
Epilepsy 2
241 9
may help precipitate (AACG) by adding C. Allow her to drive under the
an element of pupillary block. The first ­condition that she wears
step to treat AACG is the immediate ­sunglasses.
discontinuation of the topiramate and D. Order visual evoked potentials to
in some cases, the addition of cyclople- look for optic pathway electrical
gia (atropine drops) and intravenous conductance problems.
­methylprednisone. E. Start her on sodium valproate
with 4 mg of folic acid and coun-
zz Suggested Reading sel her about pregnancy risks.
55 Boentert M, Aretz H, Ludemann P. Acute
myopia and angle closure glaucoma vv Correct Answer is: E
induced by topiramate. Neurology Idiopathic photosensitive occipital lobe
2003;61(9):1306. epilepsies show hallmark EEG findings
55 Rhee DJ, Goldberg MJ, Parish RK. Bilateral of occipital epileptiform discharges
angle closure glaucoma and ciliary body and photosensitivity. They are clas-
swelling from topiramate. Arch sified as either focal or generalized
Opthalmol. 2001;119(11):1721–3. depending on the semiology of the
seizures. Seizure types may include
?? 5. You are evaluating a developmentally multicolored visual hallucinations with
normal 14-year-­old female in the associated blindness, autonomic symp-
ED who was found by her mother toms such as retching and ictal vomit-
earlier that morning to be convuls- ing, myoclonic jerks, absence seizures
ing for approximately 3 min. She and generalized convulsive seizures.
returned to her baseline after being Watching video games or flickering
sleep and ataxic for 1 h after the lights can trigger these seizures. EEG
seizure. There is no family history of may show occipital paroxysms of spikes
epilepsy. She reports that she has or polyspikes or more generalized
not been allowed to drive to school photoparoxysmal responses. Given
for the last 3 months due to frequent this patient’s generalized photopar-
whole-body jerks that seem to be oxysmal discharges and her history of
triggered by flickering morning sun- photic-induced myoclonic seizures, it
shine. Her EEG performed in the ED would be wise to start her on a broad-
shows a normal background with rare spectrum anticonvulsant and restrict
occipital sharp waves. During photic her driving privileges until she remains
stimulation, occipital paroxysms of seizure-free and myoclonic jerk-free for
high voltage spike and polyspike 6 months. Polarized sunglasses may be
complexes are seen as well as more useful for patients with photic-induced
generalized polyspike and wave seizures.
­discharges.
The best management strategy at this zz Suggested Reading
point is: 55 Guerrini R, Dravet C, Genton P, et al.
A. Load her with phenobarbital until Idiopathic photosensitive occipital lobe
the occipital epileptiform dis- epilepsy. Epilepsia. 1995;36:883–91.
charges abate. 55 Panayiotopoulos CP. Benign childhood
B. Do not start anticonvulsants since partial seizures and related epileptic
she has had only one seizure and syndromes. London:John Libby and
her risk for recurrence is low. Company Ltd.; 1999.
242 Chapter 9 · Epilepsy 2

?? 6. What is the correct statement about cyanosis), thermoregulatory and cardio-


Panayiotopoulos syndrome? respiratory (heart and respiratory rate
A. Seizures are frequent disturbances) changes also occur.
B. There is focal slowing consistently
over one area zz Suggested Reading
C. Head imaging is required to make 55 International League Against Epilepsy.
the diagnosis 55 7 https://www.epilepsydiagnosis.org/
D. Autonomic features are required syndrome/panayiotopoulos-overview.html.
for the diagnosis
?? 7. A 6-year-old boy with autistic behav-
vv Correct Answer is: D iors came for an evaluation. He has
The onset of Panayiotopoulos syn- a normal developmental history, his
drome is usually in the early childhood difficulties started around 4 years of
of autonomic seizures that are often age when he stopped responding to
prolonged. The EEG commonly shows his name being called. Eventually he
high amplitude focal spikes and may be developed severe global aphasia with
activated by sleep. Seizures are infre- mutism as well as hyperactive and
quent in most patients, with 25% only aggressive behavior. He has no his-
having a single seizure (which may be tory of seizure. He is on a combination
autonomic status epilepticus) and 50% of risperidone and clonidine which do
9 having six seizures or less. Seizures are not seem to be very helpful in con-
self-­limiting with remission typically trolling his behavior. His sedated EEG
within a few years from onset. shows multifocal and bisynchronous
10% of patients have a normal posterior temporal sharp and slow
single routine EEG. Multifocal high wave complexes that activate during
voltage repetitive spikes or sharp and sleep.
slow-waves are seen in 90% of patients, The next step in his management
these often are present in different focal should be to:
areas on sequential EEGs. All focal brain A. Obtain a high-­resolution MRI of
regions may be affected but posterior the brain.
regions is commonly affected, with B. Start oxcarbazepine to treat the
occipital spikes seen on EEG in 60% of interictal epileptiform discharges.
patients. Low voltage spikes and gen- C. Add a stimulant medication to
eralized discharges may be seen in a help control his hyperactivity.
minority of cases. D. Start the child on high-dose ACTH
Focal slowing consistently over one therapy.
area is not seen; therefore, if present con- E. Consider a trial of sodium valpro-
sider structural brain abnormality. ate and/or clobazam therapy.
Neuroimaging is usually normal. If
the diagnosis of Panayiotopoulos syn- vv Correct Answer is: E
drome has been made and there are no The clinical history of subacute regres-
atypical features, neuroimaging is not sion in speech and language, including
necessary. acquired auditory agnosia, after 3 years
The mandatory seizure type for this of age with previously normal social and
syndrome is the presence of seizures with language development is strongly sug-
prominent autonomic features mainly gestive of Landau-Kleffner syndrome.
emetic (nausea, retching, vomiting) but LKS is much less common than autistic
pupillary (mydriasis), circulatory (pallor, spectrum disorders which begin prior
Epilepsy 2
243 9
to 3 years of age with delayed develop- C. Rett syndrome variant with intrac-
ment of language and social skills. 75% table epilepsy – CDKL5
patients with LKS experiences seizures; D. Glut1 deficiency – SCN1A
generally frontal lobe complex partial E. A and D
seizures or secondarily generalized
convulsive, but 25% never have any sei- vv Correct Answer is: E
zures. Interictal EEG in the awake state A number of distinct clinical syn-
may be normal or show posterior tem- dromes of pediatric genetic epilepsy
poral epileptiform discharges that may have been described and linked to
be bisynchronous or multifocal. The specific gene defects. Phenotypes may
discharges increase in frequency during include, in addition to epilepsy, vari-
sleep. Slow wave sleep is not necessary able degrees of intellectual disability,
to obtain a diagnosis of LKS, although it elements of autism spectrum disorders,
is necessary to diagnose epilepsy with other p­ sychiatric disorders, and motor
continuous spike and waves during ­impairment.
slow wave sleep. First-line agents such Severe myoclonic epilepsy of infancy
as sodium valproate, benzodiazepines, (SMEI) is characterized by early-onset
or ethosuximide should be used first in generalized febrile seizures, followed
the treatment of LKS, but they are often by very frequent afebrile generalized
not effective in improving language tonic–clonic seizures (GTCS), myoclonic
and cognitive functioning. Second line seizures, absences and focal seizures,
treatment is high dose oral steroids for with stagnation of psychomotor develop-
at least 3 months or a trial of ACTH. The ment, intellectual disability and motor
monitoring of any treatment modality impairment. The main genes involved in
should include serial neurocognitive these conditions encode sodium channel
testing as well as awake and sleep EEG. subunits, in particular SCN1A, which is
mutated in 61–87% of SMEI.
zz Suggested Reading West syndrome is a common severe
55 Lerman P, Lerman-Sagie T, Kivity, S. Effect infantile epileptic syndrome character-
of early corticosteroid therapy for Landau- ized by typical brief tonic spasms, a
Kleffner Syndrome. Dev Med Child Neuro. profoundly abnormal electroencepha-
1991;33(3): 257–60. lography pattern called hypsarrhyth-
55 Smith M, Hoeppner TJ. Epileptic encepha- mia, and intellectual disability. Early
lopathy of late childhood: Landau-Kleffner infantile epileptic encephalopathy with
syndrome and the syndrome of continuous suppression-burst (EIEE) is the earliest
spikes and waves during slow-wave sleep. infantile epileptic syndrome, often evolv-
J Clin Neurophysiol. 2003;20(6):462–72. ing to West syndrome. Mutations in the
X chromosome genes ARX and CDKL5 are
?? 8. Epilepsy syndromes can present in found in some X-linked West syndrome
infancy and childhood with comorbid cases (ISSX1 and ISSX2). ARX (Aristaless-
intellectual disability. Genetic defects related homeobox) is a transcription
have been identified in several syn- factor involved in brain development. It
dromes. is frequently mutated in a spectrum of X
Which epilepsy syndrome-gene defect chromosome-­linked phenotypes charac-
pair is incorrect? terized by intellectual disability as their
A. Severe myoclonic epilepsy of cardinal feature.
childhood (SMEI) – SLC21A CDKL5 mutations are associated with
B. X-linked West ­Syndrome – ARX West syndrome, intellectual disability,
244 Chapter 9 · Epilepsy 2

and ASD, and also with a Rett syndrome ­ odulate the M-current, a low-thresh-
m
variant that is characterized by infantile old K+ current modified by muscarinic
spasms and intractable seizures occur- receptors.
ring before the appearance of the typi- Juvenile myoclonic epilepsy (JME) is
cal features of Rett syndrome (Hanefeld a myoclonic seizure and primarily gen-
­variant). eralized convulsive seizures which usu-
Altered glucose transport to the brain ally occur in the early morning hours. It
due to mutations in the GLUT1 trans- occurs in adolescence or early adulthood
porter gene SLC2A1 causes epilepsy of and most patients do not attain remis-
variable severity and comorbidities. Clas- sion. Usually JME is sporadic inheritance,
sic GLUT1 deficiency is associated with though many patients with an autosomal
very low cerebrospinal fluid (CSF) glu- dominant pattern of inheritance have
cose concentrations and causes a severe been found to have mutations of the
early-onset (<1 year of age) metabolic gene encoding the α-1 subunit of the
encephalopathy characterized by move- GABAA channel (GABRA1). CACNA1A is
ment disorders, epilepsy, and mental the alpha subunit of the neuronal calcium
retardation, typically associated with de channel. Mutations of the CACNA1A gene
novoSLC2A1 mutations. result in familial hemiplegic migraine,
spinocerebellar ataxia type 6 and epi-
zz Suggested Reading sodic ataxia.
9 55 Pandolfo M. Pediatric epilepsy genetics. Dravet syndrome (Severe myoclonic
Curr Opin Neurol. 2013;26(2):137–45. epilepsy of infancy “SMEI”) starts within
the first year of life with prolonged or
?? 9. What is the wrong match of the focal convulsive febrile seizures followed
following pathological membrane later by myoclonic, atypical absence, and
­Channelopathies? complex partial and generalized con-
A. Benign Familial Neonatal Seizures vulsive seizures. The seizures are refrac-
and KCNQ2α tory to medication and associated with
B. Autosomal Dominant Juvenile a severe epileptic encephalopathy and
Myoclonic Epilepsy and CACNA1A ataxia. SCN1A encodes for α subunit of
C. Severe Myoclonic Epilepsy and the voltage-gated sodium channel NAV1.1
SCN1A which allows for fast Na+ influx within the
D. Autosomal Dominant Nocturnal dendrites of neurons.
Frontal Lobe Epilepsy and CHRNA4 Autosomal dominant nocturnal frontal
E. Migrating Partial Seizures of lobe epilepsy (ADNFLE) usually occurs in
Infancy and KCNT1 late childhood with nocturnal frontal lobe
seizures. The seizures often have dystonic
vv Correct Answer is: D or hyperkinetic qualities, are of abrupt
Benign familial neonatal seizures (BNFS) onset, and are generally not associated with
is an autosomal dominant epileptic any post-ictal state. CHRNA4 and CHRNB2
seizure that presents on the second or are nicotinic, pre-synaptic, acetylcholine
third day of life in otherwise healthy receptors. As a non-selective cation chan-
term infants. The seizures are brief but nel, these receptors modulate the release of
may occur multiple times per day. They neurotransmitters due to an overall depo-
are tonic seizures with associated auto- larization of the neuronal membrane due to
nomic features. KCNQ2α and KCNQ3α an influx of Na+ ions which is greater than
are both potassium channels that the outflow of K+ ions.
Epilepsy 2
245 9
Migrating partial seizures of infancy C. Taper off his ethosuximide and
(MPSI) usually occurs within the first start him on lamotrigine.
10 months of life with refractory focal D. Taper off his ethosuximide and
migrating motor seizures, epileptic start sodium valproate.
encephalopathy and loss of previously E. Order an overnight video EEG
attained developmental milestones. The to capture epochs of slow wave
KCNT1 gene encodes the pore-forming sleep.
alpha subunit of a sodium-activated
potassium channel. Mutations in the vv Correct Answer is: D
gene result in a gain of function for the This child in the case starts to have
channel which may be at least partially astatic seizures in addition to his
blocked by quinidine. absence seizures. The term “astatic”
is not necessarily synonymous with
zz Suggested Reading “atonic”. Astatic seizures refer to
55 Alfred G. Inherited Channelopathies either atonic, myoclonic or brief tonic
associated with epilepsy. Epilepsy Curr. seizures that can cause loss of erect
2004;4(2):65–70. posture. The patient has both atonic
55 Kullman D. The neuronal channelopathies. (head drops) and myoclonic (shoul-
Brain. 2002;125(6):1177–95. der jerks) which noticed by you. As a
result, he now has absence seizures,
?? 10. A 6-year-old boy came for a second myoclonic seizures and atonic seizures.
opinion regarding his seizures man- He is also neurologically and develop-
agement. He was diagnosed with mentally normal with a normal MRI of
Childhood Absence Epilepsy. He is tak- the brain. As such he meets criteria for
ing ethosuximide 500 mg po qAM and Doose syndrome or myoclonic-astatic
750 mg po qHS (39 mg/kg/day). He epilepsy. A slow wave sleep seen on
has a fair control of his absence sei- an overnight EEG is not necessary to
zures. His mother reports that he has order because this diagnosis can be
been falling a lot lately. You witnessed made clinically or by performing a
a couple of brief head drops and bilat- routine awake EEG to show that astatic
eral upper extremity shoulder jerks. and myoclonic jerks have an electrical
His EEG 1 year ago showed a normal origin with polyspike and slow wave
awake background with periods of discharges correlating with myoclonic
3 per second, frontally dominant, jerks and atonic seizures. Valproate
generalized spike and slow wave dis- acid is a broader spectrum anti-­seizure
charges lasting 1–20 s in duration and medication which can cover all three
activated by hyperventilation. Also, seizure types better than ethosuxi-
his Brain MRI a year ago was normal. mide. Ethosuximide may be useful
His developmental milestones are adjunctively if he begins to experience
normal. His neurological examination an increase in his absence seizures.
is normal. Vigabatrin, phenytoin, carbamazepine,
The most appropriate medication and oxcarbazepine should be avoided
adjustment should be to: as these medications may increase
A. Add vigabatrin to his current dose absence seizures. Lamotrigine may
of ethosuximide. exacerbate myoclonic seizures. Topira-
B. Taper off his e­ thosuximide and mate may not be terribly effective for
replace it with topiramate. absence seizures.
246 Chapter 9 · Epilepsy 2

zz Suggested Reading C. Chromosomal microarray analysis


55 Doose H, Gerken H, Leonhardt T, et al. of her parents.
Centrencephalic myoclonic-astatic petit D. Genetic analysis for Channelopa-
mal. Clinical and genetic investigation. thies causing infantile epileptic
Neuropadiatrie. 1970;2:59–78. encephalopathies.
55 Kelley S, Kossoff E. Doose syndrome E. Lysosomal storage disorders
(myoclonic-astatic epilepsy): 40 years of ­battery.
progress. Dev Med Child Neuro.
2010;52(11):988–93. vv Correct Answer is: D
This is a case of Partial Migrating
?? 11. You are seeing a previously healthy Seizures of Infancy (MPSI) based on
8-month-old girl in the PICU who is the clinical course and EEG correlate.
having nearly continuous seizures This disorder usually occurs in the
despite being loaded with phenobar- first 10 months of life with refrac-
bital, levetiracetam and fosphenytoin tory focal migrating motor seizures,
as well as being placed on a continu- epileptic encephalopathy and loss of
ous midazolam drip. Her seizures are previously attained developmental
rapidly migrating focal clonic and milestones. The etiology of MPSI has
asymmetric tonic seizures with EEG been attributed to a variety of genetic
findings of focal onset rhythmic sharp channelopathies including (but not
9 and slow wave discharges that sud- limited to) mutations in the genes
denly start and evolve from different SCN1A, SCN1B, SLC25A22, and KCNT1.
areas of the cortex. She reached all The latter gene encodes for the pore-
of her developmental milestones on forming alpha subunit of a sodium
time and her seizures began 4 weeks activated potassium channel which is
before admission. Another neurologist highly expressed in neurons and car-
started her on levetiracetam which diomyocytes. Patients with MPSI due to
has not been effective in controlling heterozygous mutations in the KCNT1
her seizures. She has had an extensive gene have all shown gain of function
diagnostic work up including a brain mutations resulting in constitutive
MRI, analysis of serum amino acids, activation of the channel. Treatment
urine organic acids, serum acylcar- with quinidine has been successful in
nitine profile, serum very long chain significantly reducing seizure frequency
fatty acids, serum ammonia, serum in some patients with KCNT1 gain of
biotinidase activity, smf CSF analysis function mutations but not necessarily
of neurotransmitters, amino acids, and with improving their developmental
glucose levels. All tests were normal. ­outcomes.
She has been given trials of pyridoxal
phosphate and folinic acid and was zz Suggested Reading
recently initiated on the ketogenic 55 Bearden D, Strong A, Ehnot J, et al.
diet but no improvement. A genetic Targeted treatment of migrating partial
analysis including chromosomal seizures of infancy with quinidine. Ann
microarray analysis has been normal. Neurol. 2014;76:457–61.
What is your next step in evaluating her? 55 Coppola G, Plouin P, Chiron C, et al.
A. Diagnostic muscle biopsy. Migrating partial seizures in infancy: a
B. Serum and urine studies for disor- malignant disorder with developmental
ders of creatinine synthesis. arrest. Epilepsia. 1995;36:1017–24.
Epilepsy 2
247 9
?? 12. A 13-year-old boy with medically- seizures (ICEGTC) to Severe Myoclonic
refractory convulsive seizures came Epilepsy of Infancy or Childhood (Dravet
to your office for an evaluation. He syndrome). Many patients with GEFS+
has a history of febrile convulsions have a multigenerational family his-
beginning at age 9 months and tory of febrile seizures or other more
continuing through 10 years of age. malignant epilepsy syndromes. Patients
He has a family history of febrile sei- on the more severe end of the GEFS+
zures, his mother and maternal aunt, syndrome generally have more intrac-
as well as refractory epilepsy with table generalized seizure types, such
developmental delay in his maternal as myoclonic, absence, and/or atonic
aunt’s 7-year-old son. He has been seizures and their long-term neurologi-
having 3 to 5 generalized unprovoked cal prognosis for seizure control and
convulsive seizures weekly. He has cognitive outcome is poor. Patients with
recently started having myoclonic GEFS+, especially at the more severe
seizures over the past 4 months since end of the spectrum should avoid using
lamotrigine (100 mg/kg/day) was carbamazepine, lamotrigine, or vigaba-
introduced. He also on topiramate trin as these medications may worsen
(12 mg/kg/day) and phenobarbital seizures, especially myoclonic seizure.
(4 mg/kg/day). He was previously an Benzodiazepines, stiripentol, topira-
honor roll student in school but over mate, ethosuximide, and sodium valpro-
the past year he has shown declin- ate are preferred medications for use in
ing grades and difficulties with his patients with GEFS+.
­behavior at school.
Which of the following is unlikely to be zz Suggested Reading
true of this patient’s medical care? 55 Fujiwara T. Clinical spectrum of mutations
A. He likely has a mutation of the in SCN1A gene: severe myoclonic epilepsy
genes encoding for SCN1A, in infancy and related epilepsies. Epilepsy
SCN1B, SCN2A, or GABRG2. Res. 2006;70(Suppl 1):S223–30.
B. An increase in the dose of his 55 Scheffer JE, Berkovic SF. Generalized
lamotrigine is likely to improve his epilepsy with febrile seizures plus: a
epilepsy control. genetic disorder with heterogeneous
C. The use of sodium valproate is clinical phenotypes. Brain. 1997;120:
relatively contraindicated for use 479–90.
in this patient.
D. His prognosis for seizure control ?? 13. A full-term newborn was being dis-
and cognitive outcome is ques- charged on day 2 of life when his
tionable at this point. mother observed staring followed
by chewing and then bilateral limb
vv Correct Answer is: C jerking. His nurse witnessed the end
Febrile seizures beyond age 6 years of the event in which the baby had
indicates “Febrile Seizures +” categori- rhythmic clonic activity of both upper
zation. GEFS+ is a spectrum of epilep- extremities and face lasting approxi-
tic syndromes that include recurrent mately 20 s. The whole event lasted
febrile seizures at the mildest end of 2 min. He had been born unevent-
the spectrum. More severe manifesta- fully with Apgar scores of 9/10. His
tions include Intractable Childhood blood lab work and head ultrasound
Epilepsy with Generalized Tonic Clonic immediately following the episode
248 Chapter 9 · Epilepsy 2

were unremarkable. An interictal EEG days to 1 week of life, although there


shows normal active and quiet sleep is some variability in age at onset. Sei-
with a normal state change. He has zures are usually brief, but recurrence
family of a father who had neonatal may occur until age 2–3 months, when
seizures which resolved by 2 months spontaneous resolution typically occurs.
of age. The interictal EEG is usually normal. The
What is the underlying pathophysi- treatment for benign familial neonatal
ological mechanism for his condition? convulsions is similar to that for other
A. Unknown neonatal ­seizures.
B. Acetylcholine receptor
­abnormality zz Suggested Reading
C. Sodium channel abnormality 55 Mulley JC, Scheffer IE, Harkin LA,
D. Potassium channel abnormality Berkovic SF, Dibbens LM. Susceptibility
E. Glutamate metabolism genes for complex epilepsy. Hum Mol
­abnormality Genet. 2005;14:R243–49.

vv Correct Answer is: D ?? 14. A family of 9-year-old girl complain-


Benign familial neonatal convulsions ing that their daughter has multiple
(BFNC) is a well-known cause for neo- stereotyped episodes each week. She
natal seizures and the diagnosis is reports that, “I see a funny shape in
9 made by the family history of similar front of my eyes” with each of these
events in an infant with no risk factors episodes. Then the room feels like it
for neonatal seizures and a normal tilts to the side for a little while. Then I
interictal EEG. The syndrome occurs get sick to my stomach and feel dizzy.
in 14.4 per 100,000 live births. It is an Sometimes it only lasts a few minutes.
autosomal dominant inheritance with But if it lasts longer somewhere when
penetration of 85 percent. BFNC is one I can lie down and go to sleep, I can
of several epileptic disorders character- make it go away.” As a child she had
ized as a channelopathy. The two major benign infantile episodic head tilt
genes identified in this condition are from 2 to 6 months of age. She has a
both voltage-gated potassium chan- very strong family history of migraine,
nels (KCNQ2, KCNQ3) that are respon- her mother, sister, and maternal
sible for the M-current. This current grandmother. Her neurological exami-
is a slowly repolarizing current that is nation is normal, including response
responsible for modulating neuronal to Hallpike maneuver. During a partic-
bursting by hyperpolarizing the cell ularly long episode, she had a 30-min
membrane. In mice disruption of kcnq2 “awake only” EEG, and it was normal.
is lethal and only a 25% disruption of Due to the frequency of episodes:
the M-current in humans is sufficient to A. Give her a dose of sumatriptan
produce BFNC (making the mutation when her next episode starts.
of a single allele potentially pathologic B. Consideration of a preventative
and explaining the dominant mode of medication such as nortriptyline.
inheritance). Clinical features of BFNC C. Send her to an ENT surgeon to get
are characterized by focal or multifocal tubes put into her ears.
clonic or tonic seizures, a family his- D. Get an MRI scan with particular
tory of neonatal seizures, and no other attention to her posterior fossa.
neurologic abnormalities. The onset of E. Send her to a physical therapist
seizures typically occurs within a few for vestibular conditioning.
Epilepsy 2
249 9
vv Correct Answer is: B D. You would like the family to be
Those episodes have the time course seen by a psychiatrist.
and accompanying and attenuating E. The boy needs to be treated with
features of migraine equivalents. She zolpidem.
can make them go away with sleep. The
sensory distortions are accompanied by vv Correct Answer is: B
nausea. She has a past medical history This is the clinical presentation of a
of benign intermittent infantile torticol- night terrors which occurs in young pre-
lis. She has a positive family history for school and school age children. They are
migraine and a negative awake EEG. She characterized by appearing to be awake
is getting episodes frequently, so pro- but not being aware of surroundings or
phylactic treatment is advised. others in them. Screaming, fearfulness,
excited babbling, and crying are charac-
zz Suggested Reading teristic as well. They are very frightening
55 Dooley JM, Augustine HF, Gordon KE, for the family, but the child is not aware
Brna PM, Westby E. Alice in wonderland they occurred the following morning.
and other migraine associated phenom- They typically remit before age 10 years.
ena-evolution over 30 years after headache Reassurance of the family is the only
diagnosis. Pediatr Neurol. 2014;51(3): thing usually needed.
321–3.
zz Suggested Reading
?? 15. A 6-year-old boy came to the office 55 Carter KA, Hathaway NE, Lettieri CF.
with his parents because they have a Common sleep disorders in children. Am
concern about his recent nocturnal epi- Fam Physician. 2014;89(5):368–77.
sodes. He sleeps at his regular time, but 55 Haupt M, Sheldon SH, Loghmanee D. Just
at around midnight two or three nights a scary dream? A brief review of sleep
per week, he will scream and cry as if terrors, nightmares, and rapid eye move-
frightened. When they go to see him, ment sleep behavior disorder. Pediatr Ann.
they find him sitting upright with his 2013;42(10):211–6.
eyes wide open, but they cannot confer
him or get him to calm down. It is as if ?? 16. A mother of an 8-month-old child
he does not hear them at all. Eventu- reports a recurrent event of flaccid
ally, he goes back down and go back paralysis of one side of the body, or the
to sleep. He is not incontinent during other, with complete recovery of motor
these episodes and does not have function between individual events.
abnormal movements of his limbs or The child has no previous history of
eyes. He does not remember the event medical problems or developmental
the following morning. His past medi- concerns, and there is no family his-
cal history, family history, and physical tory of similar condition. Every event
examination are unremarkable. lasts for approximately 4 h, and do not
What can you tell his parents? seem to be distressing to the child. The
A. The boy should have a video EEG mother reports that some of the events
as soon as possible. seem to be precipitated by a bath or
B. The boy is likely to outgrow these sudden change in environmental tem-
episodes. perature. He does not lose conscious-
C. You are asking Child Protective ness during the spell. Brain MRI and MR
Services to take the boy out of angiography are normal. Video-EEG
their custody. was ordered and captured one event,
250 Chapter 9 · Epilepsy 2

showing EEG slowing over the cerebral confused and sleepy for almost an hour
hemisphere contralateral to the weak- afterwards. The boy does not remem-
ness, but no epileptiform abnormalities. ber the episodes, but remembers drop-
What is the correct diagnosis for this ping cups and feeling clumsier prior to
patient? the episodes. The feeling of clumsiness
A. Multiple sclerosis, relapsing and occurs more upon awakening. Family
remitting type history is unremarkable of epilepsy
B. Temporal lobe epilepsy or seizures. His past medical history is
C. Transient ischemic ­attack negative. He has normal general and
D. Alternating hemiplegia of childhood neurological ­examinations.
E. Segawa disease His EEG is most likely to show:
A. Generalized 6 Hz polyspike dis-
vv Correct Answer is: D charges on a slow background.
Alternating hemiplegia of childhood is B. No discharges on a normal back-
a rare disorder which occurs in perhaps ground.
1 in 1000,000 births. The majority of C. Generalized 4–5 Hz spike-and-
patients present with first symptoms wave discharges on a normal
during infancy or even in the newborn background.
period. The pathogenesis of unknown, D. Lateralized 6 Hz polyspike dis-
although some clinical features are charges on a normal background.
9 shared with conditions such as migraine E. Decremental beta discharges on a
and mitochondrial disorders (mitochon- slow background.
drial encephalopathy with lactic acido-
sis and stroke-like episodes [MELAS]). vv Correct Answer is: C
Large vessel occlusion or vasospasm This is a presentation of juvenile
appears to be an unlikely mechanism. myoclonic epilepsy, likely sometimes
triggered by bright light (e.g., being
zz Suggested Reading outside in the sun). He has normal cog-
55 Sweney MT, Silver K, Gerard-Blanluet M, nitive function and not likely to have
et al. Alternating hemiplegia of childhood: background slowing on his interictal
early characteristics and evolution of a EEG. But discharges are common inter-
neurodevelopmental syndrome. Pediatrics. ictally in patients with this syndrome.
2009;123:e534–41. It runs in families and at least one gene
responsible for the syndrome is inher-
?? 17. A 16-year-old boy came to your office ited as an autosomal dominant trait, it is
because of episodes of loss of con- not surprising that his family history is
sciousness. He had 4 of such events in negative, as the inheritance patterns of
the last 6 months. His mother witnessed other associated genes and the genetics
one of the event. She was in another of the disorder in many patients are not
room and when she heard a loud thud. understood.
She ran into the room into which her
son had just walked from outside and zz Suggested Reading
found him limp and unconscious on 55 Panayiotopoulos CP, Obeid T, Tahan AR.
the floor. He remained unconscious Juvenile myoclonic epilepsy: a 5-year
with rapid, non-­rhythmic eye flutter for prospective study. Epilepsia.
3–4 min, and then awakened but was 1994;35(2):285–96.
Epilepsy 2
251 9
?? 18. A mother of a 1­ 4-year-­old boy has zz Suggested Reading
witnessed recent paroxysmal events 55 Ryvlin P, Rheims S, Risse G. Nocturnal
of her son shortly after he is transi- frontal lobe epilepsy. Epilepsia.
tioning to sleep at night. He has no 2006;47(Suppl 2):83–6.
significant academic or behavioral
problems. Brain MRI was normal, but ?? 19. A 5-year-old female has a spell of
video-EEG seizure monitoring showed sudden confusion and became
a subtle but consistent ictal pattern pale. 10 min later she began to
over the frontal head regions at the vomit repeatedly, and she was
time of the spells. He has a family his- unresponsive when she arrived to
tory of nocturnal seizures in an older the ED, with tonic eye deviation to
brother and maternal uncle. her left side. An intravenous dose
Which of the following is the most of lorazepam was given and all her
accurate description of the seizures symptoms were resolved. She has a
expected in this patient? normal brain MRI, and EEG showed
A. Unresponsive staring, with chew- independent focal spikes over the
ing movements of the mouth, and occipital head regions bilaterally. The
non-purposeful fumbling move- family reported that she had similar
ments of the hands but slightly briefer spells 6 months
B. Sudden rage behaviors, with ago, which was attributed to food
directed violence, after minor ­poisoning.
frustration What is your diagnosis?
C. Sudden, shock-like contractions of A. Lennox-Gastaut syndrome
head, neck, and both arms B. Cyclic vomiting
D. Brief paroxysmal events during C. Panayiotopoulos syndrome
sleep with thrashing and yelling D. Benign Rolandic epilepsy
E. Throbbing headaches associated E. Landau-Kleffner syndrome
with vomiting
vv Correct Answer is: C
vv Correct Answer is: D Initially reported by Panayiotopoulos as
Autosomal Dominant Nocturnal Fron- a distinct clinical entity in 1989, this syn-
tal Lobe Epilepsy (ADNFLE) has been drome is now recognized by the Interna-
associated with gain-of-function muta- tional League Against Epilepsy (ILAE) as
tions in genes encoding subunits of the Early-Onset Benign Childhood Seizures
neuronal nicotinic acetylcholine recep- with Occipital Spikes. It is an increas-
tors (nAChRs) in some affected families. ingly recognized syndrome, notable for
The seizures are frequent, brief, usually mimicking other paroxysmal or acute
nocturnal, and may involve unusual disorders, such as syncope, gastroen-
motor features with posturing or thrash- teritis, or toxic ingestion. Autonomic
ing with intact awareness. Muscle symptoms such as pallor, mydriasis, and
and movement artifact make the ictal tachycardia are common. Individual sei-
changes on the EEG difficult to define, zures are usually at least 5 min in dura-
and onset of seizure activity from mesial tion, and status epilepticus is common.
frontal or orbitofrontal regions. Most However, seizures tend to be relatively
patients respond well to conventional infrequent, and remit for many affected
antiepilepsy drug (AED) therapy. patients.
252 Chapter 9 · Epilepsy 2

zz Suggested Reading retardation is usually observed dur-


55 Covanis A. Panayiotopoulos syndrome: a ing the second year after the onset of
benign childhood autonomic epilepsy seizures. Ataxia and corticospinal tract
frequently imitating encephalitis, syncope, signs as progressive neurologic deficits
migraine, sleep disorder, or gastroenteritis. develop subsequently. Diagnosis of Dra-
Pediatrics. 2006;118:e1237–43. vet syndrome is based on electroclinical
features, and the majority of affected
?? 20. A 2-year-old boy was admitted to patients have demonstrable muta-
a local ED with multiple seizures of tions in the SCN1A gene which encodes
either whole body jerks or rhythmic the alpha subunit of a voltage-gated
shaking of the extremities, often sodium channel.
involving only one side. He has nor-
mal perinatal history, but he had a zz Suggested Reading
history of recurrent febrile seizures 55 Dravet C, Bureau M, Oguni H, Fukuyama Y,
beginning at 6 months of age, 2 of Cokar O. Severe myoclonic epilepsy in
which were prolonged and required infancy: Dravet syndrome. Adv Neurol.
high-dose of pentobarbital. He has a 2005;95:71–102.
normal early developmental history
but recently his language skills have ?? 21. You are evaluating a 10-month-old
lagged behind compared to other girl for her extreme irritability, failure
9 children the same age. His EEG shows to thrive, vomiting, and possible sei-
abundant generalized spike-­waves zures. She was full term, birth weight
and polyspike waves with diffuse 3.8 kg, with fetal distress during labor
background slowing. Brain MRI scan is and associated with uterine rupture
normal. and placental abruption. She was
What is the most likely diagnosis? delivered by emergency caesarean
A. Benign myoclonic epilepsy of section. She had significant hypoxic-
infancy ischemic encephalopathy with severe
B. Dravet syndrome metabolic acidosis and neonatal
C. Lennox-Gastaut syndrome depression. There were no seizures.
D. Doose syndrome Her development was very slow, with
E. Generalized epilepsy with febrile signs of increased tone, and micro-
seizures plus cephaly. Over the past 3 months she
has been vomiting several times daily,
vv Correct Answer is: B with increased irritability, and poor
This is the clinical presentation of Dra- weight gain. In the last 2 weeks she
vet syndrome, also known as severe has also developed peculiar spells,
myoclonic epilepsy of infancy. Children with tonic extension of the neck and
have normal development before the back, while turning the head force-
onset of seizures. Usually, it occurs in fully to the right side and crying
the first year of life with febrile convul- as if in pain. These events may last
sions, often with unilateral features. for 20 min. Her EEG showed a mild
Later, they can have other seizure types degree of generalized slowing, but no
which include generalized clonic, tonic-­ epileptiform abnormality.
clonic, absence, and myoclonic seizures, Which of the following is the most
or unilateral hemiclonic and/or complex likely explanation for these spells?
partial seizures. The occurrence of sta- A. Frontal lobe seizures
tus epilepticus is frequent. Psychomotor B. Infantile spasms
Epilepsy 2
253 9
C. Pallid breath-holding spells Which of the following EEG patterns
D. Sandifer syndrome is most likely to be observed in this
E. Increased intracranial pressure patient?
A. Generalized 1.5–2.5 spike-wave
vv Correct Answer is: D B. Generalized 3 Hz spike-wave
Sandifer syndrome is secondary to C. Generalized 5–6 Hz spike-wave
gastro-­esophageal reflux disease D. Normal awake EEG background
(GERD) and it results from irritability E. Unilateral temporal lobe epilepti-
and discomfort associated with reflux form discharges
esophagitis. The symptoms are epi-
sodic dystonic posturing with torticol- vv Correct Answer is: A
lis, usually with crying or other signs Lennox-Gastaut Syndrome (LGS) diag-
of distress. It is more likely to occur in nosis is based upon three elements: (1)
children with pre-existing neurologi- refractory epilepsy with multiple seizure
cal abnormalities, who are at increased types (usually including tonic, atonic,
risk for GERD, but it can occur in normal and atypical absence, in addition to
children with severe untreated reflux. others), (2) mental retardation, and (3)
Many of these children have epilepsy, a characteristic EEG pattern, consisting
and the events of Sandifer syndrome of generalized “slow spike-wave”, typi-
may be mistaken for epileptic seizure cally at 1.5–2.5 Hz, superimposed upon
events. Identifying the correct diagnosis a slow and disorganized awake back-
is important, as anti-reflux therapy may ground. On the other hand, infantile
be dramatically effective. spasms is an epileptic syndrome with
multiple etiologies, including struc-
zz Suggested Reading tural, metabolic, and genetic causes. A
55 Kabakus N, Kurt A. Sandifer syndrome: a large subgroup is cryptogenic, without
continuing problem of misdiagnosis. In: a definite identified etiology. Seizures
Pediatrics International 2006;48:622–25. are usually treatment-­resistant. Those
patients can be treated with alterna-
?? 22. A family of a 10-year-­old boy is tive treatments such as ketogenic diet,
seeking a second opinion for their vagus nerve stimulation, or corpus cal-
son’s treatment-resistant epilepsy. losotomy (the latter may be particularly
He has the diagnosed of Lennox- beneficial for those with frequent drop
Gastaut syndrome in the last 7 years attacks leading to physical injury). Gen-
of age, although prior studies have eralized 3 Hz spike-wave is usually asso-
not ­identified a specific underlying ciated with Childhood Absence Epilepsy
etiology. He has multiple brief tonic (CAE), and “rapid” 5–6 Hz spike-wave is
­seizures per day. These occur sud- more likely to be associated with Juve-
denly and unexpectedly, and often nile Myoclonic Epilepsy (JME).
lead to falls, resulting in injuries to
the head and face. He wears a helmet zz Suggested Reading
when he is outside the home to pro- 55 Arzimanoglou A, French J, Blume WT,
tect him of physical injury associated Cross JH, Ernst JP, Feucht M, Genton P,
with these drop seizures. He also Guerrini R, Kluger G, Pellock JM, Perucca E,
has frequent absence seizures, and Wheless JW. Lennox-­Gastaut syndrome: a
sometimes generalized tonic-­clonic consensus approach on diagnosis, assess-
seizures. His developmental quotient ment, management, and trial methodology.
(DQ) is 45. Lancet Neurol. 2009;8:82–93.
254 Chapter 9 · Epilepsy 2

?? 23. A 1-week-old girl is seen for unusual the legs or the face, and are irregularly
jerking movements during sleep. She repetitive. They are less than a minute
was a full-term gestation, with no in most patients, but can be longer,
complication during pregnancy and lasting up to 30 min or even an hour if
delivery. She was discharged home the child is not awakened. The EEG is
after 1 day without recognized prob- normal between and during the spells.
lems. She seems otherwise healthy If the diagnosis is correct, a normal
and alert to her parents. However, neurological and developmental out-
for the last 3 days, shortly after she come is expected. Anti-epilepsy drugs
goes to sleep she develops repeti- (AEDs) are not advised, and have been
tive, arrhythmical jerks of both arms, reported to worsen some patients with
sometimes stronger on one side of this condition.
the body or the other. They are 2 min
in duration then they disappear. They zz Suggested Reading
do not awaken her. These can some- 55 Maurer VO, Rizzi M, Bianchetti MG,
times be provoked by touching her Ramelli GP. Benign neonatal sleep
or picking her up while asleep, but myoclonus: a review of the literature.
she does not have them while she is Pediatrics. 2010;125:e919–24.
awake. No family history of similar
condition. She has a normal awake ?? 24. A 9-month-old girl starts having
9 exam, with normal findings for her infantile spasms. She has a slower
age. Her routine EEG is normal, while development than her older siblings
awake and during sleep, although no at the same age, but she had not
clinical events occurred during the undergone evaluation. On exami-
EEG study. nation, she has multiple hypopig-
What is the single best treatment mented macules. Brain MR shows
choice for this patient? many small subependymal nodules,
A. Adrenocorticotropin hormone and several high signal abnormali-
(ACTH) injections daily ties in the subcortical white matter
B. Diazepam rectal gel given at the consistent with cortical tubers. An
times of the events echocardiogram shows a right intra-
C. Phenobarbital given orally ventricular rhabdomyoma. A muta-
D. Pyridoxine intravenous trial tion in the TSC2 gene supports the
E. Re-assurance and continued clinical diagnosis. Different treatment
observation options have been discussed with the
family. After informed consent, they
vv Correct Answer is: E are eager to begin therapy with oral
Benign Neonatal Sleep Myoclonus is vigabatrin.
probably under-diagnosed relative to For which of the following side effect
its true incidence in the population. It should this patient be regularly
occurs exclusively during sleep. Myoc- assessed?
lonus may occur in any sleep stage, but A. Gastrointestinal bleeding
they are reportedly less common during B. Glucose intolerance
active sleep. Usually, they occur during C. Hypertension
the first 2 weeks of life, and disappear D. Increased risk of serious infection
within 3–6 months. The jerks are often E. Permanent loss of peripheral
bilateral, affect the arms more than vision
Epilepsy 2
255 9
vv Correct Answer is: E return of his breathing and color to
Vigabatrin was approved for the treat- normal. He regained consciousness
ment of infantile spasms (IS) between quickly, but seemed tired and behav-
the ages of 1 month and 2 years by the iorally subdued. He was a full-term
United States Food and Drug Adminis- gestation, and had a negative medi-
tration (US FDA) in August 2009. Viga- cal history, and normal development
batrin works by binding irreversibly for age. There is no family history for
to and thereby inhibiting the function epilepsy or similar events. He has a
of GABA-transaminase, the enzyme normal examination.
responsible for breaking down GABA What is the most likely diagnosis?
at synaptic junctions. Vigabatrin results A. Cataplexy
in increased brain levels of GABA, the B. Complicated migraine
major inhibitory neurotransmitter in C. Pallid breath-holding spell
mammalian brain. However, vigabatrin D. Post-traumatic seizures
is also associated with an increased E. Anoxic seizures
risk for permanent changes in vision
in up to 30% of adult patients, usually vv Correct Answer is: C
with asymptomatic concentric visual Breath-holding spells (BHS) are involun-
field loss. Monitoring children for these tary, non-epileptic paroxysmal events
changes is difficult, although ocular that are provoked by an external cir-
coherence tomography (OCT) appears cumstance. Pallid BHS (associated by a
to be a promising technology. The pale appearance) are usually provoked
other adverse events listed above are by pain, often involving the head or
more likely to occur with intramuscular scalp, whereas cyanotic BHS are more
adrenocorticotropin hormone (ACTH) likely to be provoked by some emo-
therapy. tional upset (such as an older sibling
taking a toy). Most BHS tend to have
zz Suggested Reading one subtype or the other, although 20%
55 Lawthom C, Smith PEM, Wild JM. Nasal of patients can have both. The patho-
retinal nerve fiber layer attenuation: a physiology is not well-understood,
biomarker for vigabatrin toxicity. although there is circumstantial evi-
Ophthalmology. 2009;116:565–71. dence for autonomic nervous system
dysregulation in a significantly higher
?? 25. A 14-month-old boy is evaluated for percentage of BHS patients compared
the recent onset of spells which are to normal age-matched controls. The
triggered by minor head injury or events occur at the end of the expira-
impact. Recently, he tripped on the tory phase (rather than at the end of the
carpet and striking his head against inspiratory phase). Iron-deficiency ane-
living room furniture, leading to a mia is higher than otherwise expected
spell where he seemed to be upset in children with BHS. Anti-­epilepsy
and in pain, but unable to take his drugs (AEDs) are not effective for
next breath. He was very pale. As the ­treatment.
event progressed, he lost conscious-
ness, associated with rolling up of the zz Suggested Reading
eyes, and generalized opisthotonic 55 DiMario FJ. Prospective study of children
posturing. He remained rigid for sev- with cyanotic and pallid breath-holding
eral seconds, and then relaxed, with spells. Pediatrics 2001;107:265–69.
256 Chapter 9 · Epilepsy 2

?? 26. An 18-year-old female was admitted zz Suggested Reading


to the ED because she has a new- 55 Kochar et al. Acute intermittent porphyria
onset generalized tonic clonic seizure presenting with neurological emergency:
this morning, which lasted about Review of six cases. Neurology India. 2007.
4 min. She has been diagnosed with 55(4):413–5.
an acute intermittent porphyria at 55 Solinas et al. Neurological complications
13 years of age. Her classic attack has of porphyria. J Clin Neurosci.
previously included abdominal pain 2008;15(3):263–8.
with nausea and vomiting as well
as numbness in her legs. She has a ?? 27. A 5-month-old girl was born at
remarkable family for porphyria in her 38 weeks gestation with microcephaly,
mother and sister. On neurological petechiae, and hepatosplenomegaly.
examination, she has mild proximal Upon her pediatrician as a newborn,
weakness in all limbs and decreased she had intracranial calcifications
tendon reflexes. and sensorineural hearing loss, and
Which of the following antiepileptic he diagnosed her with congenital
medication would be safe to use if cytomegalovirus (CMV) infection.
­prophylactic medication is required? There weeks ago, she developed body
A. Carbamazepine jerks, with sudden, forceful flexion of
B. Gabapentin/Keppra the head and neck, torso, and arms,
9 C. Lamotrigine occurring in clusters, more likely upon
D. Phenobarbital awakening from sleep. She cries briefly
E. Valproic acid between each jerk, which are 4–6 s
apart. The clusters last up to 3 min in
vv Correct Answer is: B duration, although some are shorter.
Acute intermittent porphyria is an auto- Now those clusters are occurring daily.
somal dominant disorder, and results The EEG pattern you are going see with
from partial deficiency of porphobilino- this new symptom is:
gen deaminase enzyme. Classic presen- A. Burst-suppression background
tation is with an acute abdomen, but B. Hypsarrhythmia
the neurological symptoms can include C. Photo paroxysmal occipital dis-
peripheral neuropathy, respiratory charges
paralysis, seizures and loss of conscious- D. Spike transients over the right
ness. You have to be aware of that the temporal head region
use of enzyme-inducing antiepileptic E. 3-Hz generalized spike-wave dis-
drugs can cause worsening of seizures, charges
which can be fatal. Carbamazepine,
clonazepam and other benzodiaz- vv Correct Answer is: B
epines, lamotrigine, phenytoin, sodium The presentation of the infantile spasms
valproate, barbiturates including primi- (IS) includes clusters of myoclonic/
done, topiramate, and tiagabine can tonic spasms, each cluster can last up to
worsen and exacerbate porphyria or several minutes in duration, and more
affect metabolism of porphyrins. The likely to occur with sleep-wake transi-
incidence of the disease may be under- tions. They usually begin between 4
estimated, so high clinical suspicion is and 12 months of age, although older
warranted. ­children may also have clusters of epi-
Epilepsy 2
257 9
leptic spasms. Many children with IS D. Reassurance and anticipatory
have pre-existing neurological symp- guidance
toms. The list of etiologies for IS is very E. Admission to the hospital for fur-
long, including structural, metabolic, ther evaluation and observation
and genetic causes. 10% of IS patients
have a favorable prognosis, but the vv Correct Answer is: D
outcome for the majority includes at Benign febrile convulsions occur in
least a mild degree of developmental 2–5% of the normal population of
abnormality and the later appearance of infants, typically between the ages of
other seizure types. Hypsarrhythmia (or 6 months and 3 years. Maximal inci-
one of its several “modified” forms, or dence is at 18–20 months of age. Simple
variants) is the EEG pattern most likely febrile seizures last less than 15 min,
to be seen in IS patients, although the without focal or lateralizing features
absence of hypsarrhythmia does not with ictal semiology, and do not recur
exclude the clinical diagnosis. within the same 24-h time period.
For those children over 18 months of
zz Suggested Reading age without previously recognized
55 Zupanc ML. Clinical evaluation and neurological symptoms or signs, the
diagnosis of severe epilepsy syndromes of evaluation should be limited to a care-
early childhood. J Child Neurol. ful history and physical examination. If
2009;24(Suppl 8):6S–14S. this is reassuring (including the absence
of meningeal signs suggesting an acute
?? 28. A 17-month-old boy came to the ED neurological infection), then routine
after a 5-min generalized tonic-clonic laboratory tests, EEG, brain imaging, and
convulsion in association with high lumbar puncture are not recommended,
fever at home. His parents report that and hospital admission is not necessary.
he was not, but he was very hot to Families should be educated, however,
the touch when the seizure occurred. about the expected natural history of
In ED he has a temperature of 103.8° benign febrile seizures, and indications
rectally. On exam, he has otitis media, for medical intervention if febrile sei-
after he was given acetaminophen, zures should occur in the future.
he became happy and playful as his
baseline. His prior history is notable zz Suggested Reading
for birth at term gestation, and subse- 55 Capovilla G, Mastrangelo M, Romeo A,
quent normal development. His fam- Vigevano F. Recommendations for the
ily history is remarkable of his mother management of “febrile seizures”.
who she had three seizures with high Epilepsia. 2009;50(Suppl 1):2–6.
fever when she was an infant.
What is your next step in management? ?? 29. A 4-day-old baby was admitted to the
A. Intravenous loading dose of Emergency Department for suspected
phenobarbital followed by oral seizures. His mother has no problems
­treatment during her pregnancy, and she deliv-
B. Lumbar puncture for CSF ered him at term via uncomplicated
­examination vaginal delivery. He went home
C. EEG to be scheduled later as an with his mother after 1 day. He has
outpatient normal feeding pattern and seemed
258 Chapter 9 · Epilepsy 2

otherwise well, but had 3 episodes ?? 30. A 6-year-old female came to the ED
of unresponsiveness, stiffening, tonic because she has for two recent noc-
eye deviation, and cyanosis. Every turnal spells. The family found her in
episode was less than a minute, and the early morning in her room having
he seemed to recover quickly. He has retching behavior, followed by sev-
a family history of similar episodes in eral bouts of vomiting. She was pale,
his mother and other maternal rela- disoriented, and unable to coherently
tives. In the emergency department respond to her family. 10 min later, she
he is awake and has a normal vital started to have subtle clonic jerking to
signs and intact examination. her right side, including face, neck and
What is your most likely diagnosis? arm, lasting another 8 min. Her evalua-
A. Benign familial neonatal convulsions tion in the ED was unrevealing, includ-
B. Encephalitis with herpes simplex ing head CT imaging. A month later,
virus she had a second identical spell in her
C. Ischemic stroke in the middle sleep. She has a normal neurological
cerebral artery examination, and a normal develop-
D. Methylmalonic aciduria ment for her age. She has negative
E. Pyridoxine dependency family history of epilepsy. You ordered
an EEG, shows normal background,
vv Correct Answer is: A but also frequent spikes from the left
9 Newborns with Benign Familial Neona- occipital head region, which appeared
tal Convulsions (BFNC) typically begin to suppress with eye opening. During
to have seizures on the third day of life. sleep, spikes were more frequent, and
Those ­seizures are brief, and are associ- appeared independently from both
ated with limited post-ictal manifesta- the right and left occipital regions.
tions, but they may be repetitive. This What is your diagnosis?
condition is one explanation for “well- A. Benign Rolandic epilepsy of child-
baby seizures”. CNS infection, meta- hood
bolic perturbations or consequences B. Migraine with aura
of trauma must be considered and C. Obstructive sleep apnea
excluded. Seizures usually permanently D. Panayiotopoulos syndrome
remit in 1–3 months, without residual E. Rasmussen syndrome
neurological or developmental impair-
ment. However, 10–20% of patients may vv Correct Answer is: D.
have seizures later in life. BFNC have Beginning in 1988, and in subsequent
been associated with loss-of-­function reports, Panayiotopoulos has described
mutations in KCNQ2 and KCNQ3 genes, this benign epilepsy syndrome, charac-
which express neuronal voltage-­ terized by infrequent but nevertheless
dependent potassium channel subunits often prolonged seizure events, usually
Kv7.2 and Kv7.3, respectively. nocturnal, dominated by autonomic
symptoms, including pallor, nausea,
zz Suggested Reading retching and vomiting, sometimes
55 Rosen GM, Rosales TO, Connolly M, associated with confusion and occasion-
Anderson VE, Leppert M. Seizure charac- ally concluding in a focal clonic seizure
teristics in chromosome 20 benign familial event. EEG is characterized by interic-
neonatal convulsions. Neurology. tal spikes located most likely over the
1993;43:1355–60. occipital regions, but can be observed
Epilepsy 2
259 9
over other areas as well. Approximately What is your most likely diagnosis?
only 10% of patients report visual symp- A. DiGeorge syndrome (chromo-
toms. Because of these features, now some 22q11.2 deletion syndrome)
confirmed by multiple reports, it has B. Down syndrome (trisomy 21)
been proposed that Panayiotopoulos C. Group B streptococcal sepsis and
syndrome should be classified as an meningitis
autonomic epilepsy, rather than as an D. Propionic aciduria
occipital epilepsy. Most patients have E. Prenatal hypoxic-­ischemic brain
only a few seizures, and do not require injury
daily anti-epilepsy drug (AED) therapy.
The prognosis is always good. vv Correct Answer is: A
In the 1960’s and 1970’s, neonatal
zz Suggested Reading hypocalcemia (and seizures) were com-
55 Covanis A. Panayiotopoulos syndrome: a monly encountered as an unintended
benign childhood autonomic epilepsy result of high phosphate levels in cow’s
frequently imitating encephalitis, syncope, milk-derived infant formulas. In current
migraine, sleep disorder, or gastroenteritis. clinical practice, neonatal hypocalcemic
Pediatrics. 2006;118:e1237–43. seizures are uncommon, and usually
result from deficiency of parathyroid
?? 31. You are evaluating a 2-week-old boy hormone due to hypoplasia of the para-
with congenital heart disease for pos- thyroid glands. This is often associated
sible seizures. He is a full-term baby. with various forms of congenital heart
He became hypotensive shortly after disease including the cardiac outflow
birth. While he was at NICU, an evalu- tract malformations associated with
ation was done which determined DiGeorge syndrome (chromosome
that he has an interrupted aortic 22q11.2 syndrome). Treatment consists
arch, and has been dependent upon of restoring physiological levels of cal-
a prostaglandin infusion to maintain cium. It should be recognized that the
systemic blood flow through the seizures and encephalopathy associated
ductus arteriosus. On general exam, with hypocalcemia may take several
he has dysmorphic features with days to resolve after normal calcium lev-
micrognathia and low-set ears. He has els have been achieved.
been irritable since birth and he have
repeated events involving clonic jerk- zz Suggested Reading
ing of the extremities for 24 h, usually 55 Kossof EH, Silvia MT, Maret A,
on the left side, although the nurs- Carakushansky M, Vining EPG. Neonatal
ing staff also documents right-sided hypocalcemic seizures: case report and
clonic events as well. Serum calcium literature review. J Child Neurol.
is abnormally low (ionized calcium 2002;17:238–41.
2.8 mg/dl, normal range 4.1–5.2 mg/
dl). CT head is normal, but the EEG is ?? 32. A 25-month-old female is being
markedly abnormal, with low ampli- evaluated for unexplained spells. She
tude irregular slowing, and high developed normally until episodes of
amplitude multifocal independent unusual eye movements were noted
spikes. He has been on intravenous around 4 months of age, consisting of
administration of calcium gluconate nystagmus, sometimes affecting only
and phenobarbital. one eye. She never has an alteration
260 Chapter 9 · Epilepsy 2

of consciousness. At 8 months of ­ emiplegia), with episodes charac-


h
age she developed episodes of flac- terized by quadriplegia, dysphagia,
cid weakness, affecting one side of dysarthria, and possibly respiratory
the body or the other. These events compromise, (3) some episodes may
occurred only while awake, and were consist of unilateral dystonia, rather
sometimes provoked by bathing. than flaccid weakness, and (4) virtually
Weakness would last for several hours all patients develop persisting co-mor-
before recovery to baseline. If she bid symptoms in between hemiplegic
goes to sleep after the onset of the episodes, with developmental delay,
episode, she would always a­ waken ataxia, and/or epilepsy. Sleep predict-
without the unilateral weakness. ably relieves the hemiplegic episodes,
Therapeutic trials of medications for and induction of sleep is the most effec-
epilepsy and migraine have proven tive way of acutely treating an event.
ineffective. On physical examination, Flunarizine can be effective in some
she is poorly socialized and has only a patients to reduce the frequency and
few words. She appears to be clumsy severity of attacks.
and mildly ataxic, despite the fact that
she is on no medications at this time. zz Suggested Reading
Her eye movements are normal. She 55 Sweney MT, Silver K, Gerard-Blanluet M,
was mildly hypotonic, but showed Pedespan JM, Renault F, Arzimanoglou A,
9 no lateralizing features on her motor Schlesinger-Massart M, Lewelt AJ, Reyna SP,
exam. Swoboda KJ. Alternating hemiplagia of
What is your most likely diagnosis? childhood: early characteristics and
A. Alternating hemiplegia of child- evolution of a neurodevelopmental
hood syndrome. Pediatrics. 2009;123: e534–41.
B. Hypokalemic periodic paralysis
C. Mitochondrial encephalopathy ?? 33. A 5-year-old girl has been evalu-
with lactic acidosis and stroke-like ated for several acute episodes of
syndrome unsteadiness and difficulty walking
D. Narcolepsy with cataplexy lasting for few minutes. The family
E. Simple partial epilepsy describes the child as appearing pale
and “sickly”, and with unusually fussy
vv Correct Answer is: A behavior. The young girl was awake
Alternating hemiplegia of childhood and interactive with the family during
(AHC) is a rare disorder with very spe- these spells, and she wanted to be
cific symptomatology. The correct diag- held closely. When the family encour-
nosis is usually delayed for several years aged her to walk, she had a wide-
relative to the onset of first symptoms. based wobbly gait. Additionally, they
AHC is in some ways an inadequate noted unusual jerking movements of
descriptive name for the syndrome: (1) the eyes to the sides. Afterwards, she
the first symptoms are usually episodes would generally fall asleep and upon
of abnormal eye movements, occurring awakening was back to baseline.
in early infancy, (2) unilateral flaccid There were no previous illnesses or
hemiplegia occurring independently symptoms, and no family history of
on each side is the classic expectation, seizures or epilepsy.
but many patients have simultane- What is your most likely diagnosis?
ous involvement of both sides of the A. Drug intoxication
body for at least some events (double B. Benign paroxysmal vertigo
Epilepsy 2
261 9
C. Acute post-infectious cerebellitis no sign of acute infection. Head CT
D. Neuroblastoma, with opsoclonus- and lumbar puncture were normal.
myoclonus His EEG shows on-going generalized
E. Complex partial seizure epileptiform ictal activity despite ces-
sation of overt clinical seizures.
vv Correct Answer is: B What is your next step in manage-
Benign paroxysmal vertigo is thought ment?
to be a migraine variant and occurs A. MRI brain scan
usually in toddlers or young children B. High-dose pentobarbital
who have family history of migraine. C. Pyridoxine 100 mg IV
Nystagmus or vomiting can occur, and D. Ganciclovir 6 mg/Kg IV every 4 h
in older children, unilateral throbbing E. Ativan 1 mg/Kg IV
headache may be present. The attacks
are generally brief, last from a few min- vv Correct Answer is: C
utes to several hours, and they often Patient having status epilepticus with-
resolve with sleep. Between attacks, the out any obvious cause at 1 month with
child is perfectly normal. These episodes a history of irritability, suggestive of
are benign, have a favorable prognosis, an encephalopathy. This patient has
and no treatment is needed. Despite the normal general lab, CSF and CT scan,
prevailing view that BPV is a migraine he is in the category of neonatal epi-
variant, there is no strong evidence that leptic encephalopathy. A trial of IV
these events presage the development pyridoxine is warranted in all children
of typical migraine in adolescence or with status of unknown etiology as
early adulthood. pyridoxine dependency is a treatable
cause of epilepsy in children less than
zz Suggested Reading 3 years of age. Pyridoxine dependency
55 Drigo P, Carli G, Laverda AM. Benign (PD) is an autosomal recessive and
paroxysmal vertigo of childhood. Brain recent discovery of mutations in fami-
Dev. 2001;23(1):38–41. lies with the condition involving the
55 Al-Twaijri WA, Shevell MI. Pediatric gene ALDH7A1 (antiquitin) is making
migraine equivalents: occurrence and this a rapidly changing field. PD is cur-
clinical features in practice. Pediatr rently a clinical diagnosis that depends
Neurol. 2002;26(5):365–8. on the response to pharmacological
doses of pyridoxine (Vitamin B6) with
?? 34. A previously healthy 1-month-old cessation of clinical and electrographic
boy was admitted to the ED 2 h ago, seizures. 100 mg of IV pyridoxine with
he is in generalized status epilep- EEG monitoring is recommended by
ticus which began 30 min prior to many authors. The paroxysmal activity
arrival. ED physician intubated him (both clinical and/or electrographic)
and gave him phenobarbital 20 mg/ should cease. If there is no positive
kg and fosphenytoin 20 mg/kg.which result, a trial of p
­ yridoxal phosphate
stopped the convulsion after an hour. is warranted. If other AEDs have been
1 week ago, he started to vomit and used to treat the seizures (this is likely
to be irritable. He has also been noted as pyridoxine-dependent seizures are
to have intermittent twitching of usually intractable) then after a time
the limbs since birth. He is afebrile, the other AEDs need to be withdrawn. If
comatose, with a flat anterior fonta- the seizures remain controlled on daily
nelle. Lab work including CRP showed oral pyridoxine (at 15 mg/Kg) then a
262 Chapter 9 · Epilepsy 2

trial without pyridoxine, which results syndrome that begins in infancy with
in recurrence of the seizures which are onset of febrile hemi-clonic status
again quickly controlled by oral pyridox- epilepticus and evolves to a pattern of
ine, confirms the diagnosis. Pyridoxine multiple seizure types, including focal
should then be continued for life. A DNA motor, myoclonic, absence, and gener-
diagnosis should be pursued. Pyridox- alized tonic-clonic seizures associated
ine dependency is a different disorder with marked slowing or stagnation of
than the pyridoxine-deficiency which development often accompanied by
is an acquired condition that occurs in behavioral disturbances. Metabolic
neonates as a result of a deficient diet investigations are normal. EEG may
usually associated with powdered goat’s show generalized spike and slow wave
milk diet. complexes and a photoparoxysmal
response. Many patients have a charac-
zz Suggested Reading teristic defect in the SCN1A gene which
55 Basura GJ, Hagland SP, Wiltse AM and encodes the alpha one subunit of the
Gospe SM. Clinical features and the neuronal sodium channel.
management of pyridoxine-­dependent and
pyridoxine-responsive seizures: review of zz Suggested Reading
63 North American cases submitted to a 55 Petrelli C, Passamonti C, Cesaroni E, et al.
patient registry. Eur J Pediatr. 2008 [Epub Early clinical features in Dravet Syndrome
9 ahead of print]. PMID: 18762976. patients with and without SCN1A.
Epilepsy. 2012;99(1–2):21–7.
?? 35. An 11-month-old baby boy presents 55 Brunklaus A, Ellis R, Reavey E, et al.
with febrile hemi-clonic status epi- Prognostic, clinical and demographic
lepticus. He is treated acutely. Two features in SCN1A mutation – positive
months later, he has another episode Dravet syndrome. Brain. 2012;
of febrile hemi-­clonic status epilep- 135(Pt 8):2329–36.
ticus involving the other side of his
body. He then begins having apparent ?? 36. An 8-month-old boy has seizures,
absence seizures, focal motor seizures, characterized by stiffening of arms
and generalized tonic-clonic seizures and legs, lasting 15–20 s.
associated with illnesses. The parents What is the most likely seizure type?
note stagnation of his psychomo- A. absence seizure
tor development. MRI of the brain is B. focal seizure
normal. Genetic studies disclose an C. infantile spasms
abnormality in the SCN1A gene. D. myoclonic seizure
What is his most likely diagnosis? E. tonic seizure
A. Dravet syndrome
B. epilepsy with myoclonic absences vv Correct Answer is: E
C. Lennox-Gastaut syndrome The seizure description is most consis-
D. migrating partial seizures of tent with a tonic seizure. No focality
infancy with tonic seizure. Absence seizures
E. West syndrome would be atypical in a 10-month-old.
The duration of the stiffening is too
vv Correct Answer is: A prolonged for infantile spasms or myo-
The patient history is consistent with clonic seizures, and did not include flex-
Dravet syndrome, a distinct epilepsy ion component.
Epilepsy 2
263 9
zz Suggested Reading Doose and other epilepsy syndrome
55 Berg AT, Berkovic SF, Brodie MJ, et al. occur during infancy are characterized
Revised terminology and concepts for by myoclonic, atonic, atypical absence,
organization of seizures and epilepsies: and bilateral convulsions. Landau-­
report of the ILAE Commission on Kleffner syndrome is a childhood-onset
Classification and Terminology, 2005– epileptic encephalopathy characterized
2009. Epilepsia. 2010;51:676–85. by aphasia with continuous epilepti-
form discharges during sleep.
?? 37. A 7-year-old boy with a history of
treatment-resistant epilepsy with zz Suggested Reading
multiple seizure types, secondary to 55 Berg AT, Berkovic SF, Brodie MJ, et al.
perinatal hypoxic-­ischemic encepha- Revised terminology and concepts for
lopathy. This resulted in severe organization of seizures and epilepsies:
cognitive and motor impairment. report of the ILAE Commission on
He had multifocal electrographic Classification and Terminology, 2005–
seizures during the first week of life 2009. Epilepsia. 2010;51:676–85.
that resolved with phenobarbital.
At 6 months of age he developed ?? 38. A 4-year-old boy has been evaluated
infantile spasms that were initially for frequent head jerks, staring spells,
responsive to treatment with adre- and infrequent bilateral convulsions.
nocorticotrophin hormones. His He has a happy demeanor, but suf-
seizures recurred, characterized by fers global developmental delay
brief stiffening, drops, and episodes with ataxia. His examination shows
of unresponsiveness. He has been a nonverbal child with diffuse hypo-
having daily seizures despite multiple tonia, and jerking movements which
medications. include hand flapping and tremor.
What is the most likely diagnosis? His brain MRI is normal. Interictal
A. Angelman syndrome EEG shows intermittent bursts of dif-
B. Doose syndrome fuse polymorphic delta activity, with
C. Landau-Kleffner syndrome admixed sharp wave discharges, giv-
D. Lennox-Gastaut syndrome ing it a “notched” appearance.
E. West syndrome What is your most likely diagnosis?
A. Angelman syndrome
vv Correct Answer is: D B. Doose syndrome
The clinical presentation indicates C. Landau-Kleffner syndrome
Lennox-­Gastaut syndrome. Infantile D. Lennox-Gastaut syndrome
spasms may follow perinatal hypoxic- E. West syndrome
ischemic encephalopathy and subse-
quently evolve into Lennox-Gastaut vv Correct Answer is: A
syndrome. The onset and evolution The patient has Angelman syndrome, a
from West syndrome to Lennox-­Gastaut genetic epilepsy occurs in the infantile
syndrome are the key clinical features period. Typically, patients have devel-
to differentiate from the other choices. opmental delay, seizure onset usually
Angelman syndrome is a genetic epi- before age 3 years, happy demeanor,
lepsy which occurs during infancy. ataxia with hand flapping movements,
Doose syndrome, also known as epi- and EEG with characteristic pattern with
lepsy with myoclonic atonic seizures, large amplitude slow-­spike waves. West
264 Chapter 9 · Epilepsy 2

syndrome is a severe infantile-onset vv Correct Answer is: C


epileptic encephalopathy character- The clinical scenario is consistent with a
ized by a triad of infantile spasms, diagnosis of benign epilepsy with cen-
hypsarrhythmia on EEG, and develop- trotemporal spikes (BECTs, also known
ment regression. Doose syndrome, also as benign rolandic epilepsy.) Typically,
known as myoclonic astatic epilepsy seizures in this condition begin between
(myoclonic seizure followed immedi- the ages of 3–13 (peaking between ages
ately by an atonic seizure), which occurs 8 and 12) and are associated with uni-
in the infantile period and is charac- lateral facial sensorimotor symptoms,
terized by myoclonic, atonic, atypical hypersalivation, speech arrest, and/
absence, and bilateral convulsions; EEG or oropharyngeal symptom. 50% of
shows regular and irregular bilaterally patients can have secondarily general-
synchronous 2- to 3-Hz spike-waves ize seizures. Interictal EEG shows high
and polyspike patterns with a 4–7-Hz amplitude central spikes, often with
background. Landau-Kleffner syndrome shifting laterality. Seizures typically
is a childhood-onset epileptic encepha- resolve by age 18.
lopathy characterized by aphasia with
continuous epileptiform discharges dur- zz Suggested Reading
ing sleep. Lennox-­Gastaut syndrome is 55 Guerrini R, Pellacani S. Benign childhood
a severe epileptic encephalopathy with focal epilepsies. Epilepsia. 2012;
9 multiple seizure types, intellectual dis- 53(Suppl 4):9–18.
ability, and slow spike-­and-­wave on EEG.
?? 40. A 7-month-old girl develops spells of
zz Suggested Reading sudden bending forwards, and then
55 Valente KD, Koiffmann CP, Fridman C, her arms and legs stiffen and extend.
et al. Epilepsy in patients with Angelman She can lift her head without sup-
syndrome caused by deletion of the port, and she has a social smile; how-
chromosome 15q11–13. Arch Neurol. ever, she is not able to sit without
2006; 63(1):122–8. support or turn from her back onto
her ­stomach.
?? 39. A 10-year-old boy has a history of What is the most likely interictal EEG
three nocturnal seizures associated features?
with left-sided facial, mouth and A. 3 Hz generalized spike-and-wave
tongue paresthesias and twitching of discharges
the left side of his face with associated B. asymmetric beta activity
hypersalivation. He had one second- C. hypsarrhythmia
arily generalized tonic-clonic seizure D. suppression-burst pattern
followed one of his regular seizure. E. temporal sharp waves
Brain MRI scan is unremarkable.
Which of the following interictal EEG vv Correct Answer is: C.
features is most likely? The clinical description is most con-
A. 1 Hz to 2 Hz generalized spike- sistent with a diagnosis of West
and-wave discharges syndrome. The spells the infant is
B. 4 Hz to 5 Hz generalized poly- experiencing are most consistent with
spike-and-wave discharges infantile spasms. West syndrome is
C. bilateral centrotemporal spikes characterized by developmental delay,
D. bilateral occipital sharp waves hypsarrhythmia on EEG, and infantile
E. generalized paroxysmal fast activity spasms.
Epilepsy 2
265 9
zz Suggested Reading brief stiffening episodes in sleep and
55 Hrachovy RA, Frost JD Jr. Infantile frequently exhibited inappropriate
epileptic encephalopathy with hypsar- laughter.
rhythmia (infantile spasms/West syn- What can you find on his MRI?
drome). J Clin Neurophysiol. A. focal cortical dysplasia in the right
2003;20(6):408–25 frontal lobe
B. hypothalamic hamartoma
?? 41. A 12-year-old boy with normal devel- C. mesial temporal sclerosis
opment presents with new-onset D. normal MRI of the brain
epilepsy with focal seizures. His E. perinatal right MCA infarct
seizures are characterized by a rising
sensation, followed by unresponsive- vv Correct Answer is: B
ness and lip smacking for several This patient’s presentation is most con-
minutes. He has 2–3 seizures per sistent with gelastic, tonic, and focal
month despite appropriate anticon- seizures secondary to hypothalamic
vulsant. hamartoma. Although other structural
Which of the following conditions is causes of epilepsy may manifest with
most likely to be present in his medical similar focal seizures, the inappropriate
history? laughter is most commonly associated
A. brain tumor with this localization.
B. family history of epilepsy
C. febrile status epilepticus zz Suggested Reading
D. hypoxic-ischemic encephalopathy 55 Berkovic SF, Arzimanoglou A, Kuzniecky R,
E. traumatic brain injury et al. Hypothalamic hamartoma and
seizures: a treatable epileptic encephalopa-
vv Correct Answer is: C thy. Epilepsia. 2003;44:969–73.
The clinical history indicates temporal
lobe epilepsy which may be secondary ?? 43. A 10-month-old boy with normal
to mesial temporal sclerosis following development has a history of pro-
an episode of febrile status epilepticus longed alternating hemiconvulsive
during childhood. Although the other seizures, usually associated with fever
options are risk factors for epilepsy, the and/or illness. Seizure began 1 day
age of onset would be most consistent after his 4-month vaccinations. He has
with this specific clinical scenario. 2–3 seizures per month despite two
anticonvulsants, and seem to worsen
zz Suggested Reading with carbamazepine.
55 Hesdorffer DC, Shinnar S, Lewis DV, et al. What is your the most likely diagnosis?
FEBSTAT study team. Design and phe- A. atypical febrile seizures
nomenology of the FEBSTAT study. B. complex febrile seizures
Epilepsia. 2012; 53:1471–80. C. Dravet syndrome
D. infantile spasms
?? 42. A 7-year-old boy has intractable epi- E. simple febrile seizures
lepsy with multiple seizure types, and
a progressive encephalopathy. He vv Correct Answer is: C
started to have seizures at 2 years old This patient is most likely has Dravet
and initially characterized by a behav- syndrome. Development is typically
ioral arrest, eye deviation, tachycar- normal very early in the course with
dia, and facial flushing. He also had later deterioration. Febrile seizures,
266 Chapter 9 · Epilepsy 2

either simple or atypical, are usually not elimination for lamotrigine. If the oral
so frequent, are usually do not require contraceptive pill is withdrawn, then
prophylactic antiepileptic medications, deinduction of the lamotrigine metabo-
and are not considered a diagnosis of lism occurs quickly, typically within days
epilepsy. The clinical presentation is not and maximally by 1 week. This results in
consistent with infantile spasms, which increased serum levels of lamotrigine,
are clusters of quick jerks that are typi- with typical symptoms of medication
cally associated with developmental toxicity such as nausea, vomiting, and
regression and an EEG background of ataxia. This is best managed by lowering
hypsarrhythmia. the dose of lamotrigine; the symptoms
should abate within a couple of days.
zz Suggested Reading
55 Berg AT, Berkovic SF, Brodie MJ, et al. zz Suggested Reading
Revised terminology and concepts for 55 Christiansen J, Petrenaite V, Atterman J,
organization of seizures and epilepsies: et al. Oral contraceptives induce lamotrig-
report of the ILAE Commission on ine metabolism: Evidence from a double-
Classification and Terminology, 2005– blind, placebo-­controlled trial. Epilepsia.
2009. Epilepsia. 2010; 51:676–85. 2007;48(3):484–9.

?? 44. An 18-year-old girl with complex ?? 45. A 6-year-old boy has been having
9 partial seizures that have been well myoclonic, myoclonic-­atonic, and
controlled for the last 3 years. In the atonic seizures. He has early normal
past 2 years, she was taken lamotrig- development. His seizures have not
ine and a combination type oral responded adequately to treatment
contraceptive. She takes lamotrigine with levetiracetam, valproate, clo-
300 mg tablets twice daily. She has bazam, lamotrigine, or topiramate.
not had any seizures for 3 years and He is diagnosed with epilepsy with
has regular menses. She came to you myoclonic-atonic seizures.
because she recently stopped taking What is the best next step in his
her oral contraceptive medication, ­management?
and approximately 1 week later had A. ketogenic diet
intermittent nausea with vomiting, B. obtain a magnetoencephalogram
and she became ataxic. C. refer the patient for a clinical trial
What is your next step in her of a novel antiepileptic drug
­management? D. responsive neurostimulation
A. lower her lamotrigine dose to E. video EEG monitoring with inter-
200 mg twice daily cranial electrodes
B. obtain MRI of the brain to exclude
space occupying lesion vv Correct Answer is: A
C. obtain urine toxicology The patient has myoclonic-atonic sei-
D. perform a lumbar puncture zures. This is an epilepsy syndrome
E. perform a serum pregnancy test occurs in childhood between the ages
of 7 months and 6 years. Normal devel-
vv Correct Answer is: A opment is expected. The ketogenic
Oral contraceptives induce the metabo- diet has been shown to be effective
lism of lamotrigine by inducing uridine in several series of patients with myo-
diphosphate glucuronosyltransfer- clonic-atonic epilepsy. Ketogenic diet
ase (UGT), which is a major root of in the treatment of myoclonic-atonic
Epilepsy 2
267 9
epilepsy is very effective if used early ?? 47. The family of a 6-month-old boy
in the course of treatment. Given the reports clusters of brief neck flexion
failure of multiple medications in this and arm extension and developmental
child, this would be the next best step in regression. His EEG shows a chaotic
­management. high-voltage background, polymor-
phic delta activity with admixed
zz Suggested Reading multifocal spike-wave discharges
55 Nangia S, Carbaloo RH, Kang HC, et al. Is and electrodecrements. His birth,
the Ketogenic diet effective in specific past medical history, and family his-
epilepsy syndromes?. Epilepsy Res. tory were unremarkable. He has no
2012;100(3):252–7. hypopigmented macules on examina-
tion. MRI brain is normal. Standard
?? 46. The implementation of the ketogenic metabolic evaluation of CSF, serum,
diet can prove harmful in which of the and urine is unrevealing. He was diag-
following conditions associated with nosed with West syndrome and treated
epilepsy? with adrenocorticotropic hormone.
A. glucose transporter type 1 (Glut1) The parents reported clinical cessation
deficiency of infantile spasms within 1 week.
B. myoclonic astatic epilepsy What is the recommended EEG type
C. pyruvate carboxylase deficiency and time of testing from the time
D. pyruvate ­dehydrogenase (E1) treatment was initiated?
­deficiency A. 1-h routine EEG after 2 months
E. tuberous sclerosis related Lennox- B. 1-h routine EEG after 2 weeks
Gastaut syndrome C. 1-h routine EEG after 4 weeks
D. 24-h inpatient video EEG after
vv Correct Answer is: C 2 months
The ketogenic diet is safe and effective E. 24-h inpatient video EEG after
in management of epilepsy associated 2 weeks
with glucose transporter type 1 (Glut1)
deficiency, pyruvate dehydrogenase vv Correct Answer is: E
(E1) deficiency, infantile spasms, myo- The recommendation form the 2010 US
clonic astatic epilepsy, and severe consensus report is short duration ther-
myoclonic epilepsy of infancy, tuber- apy with a first-line treatment (ACTH or
ous sclerosis complex, Lennox-Gastaut vigabatrin) and timely EEG evaluation
syndrome and Rett syndrome. On the of treatment effectiveness. The patient
other hand, the ketogenic diet is con- has been improved clinically on ACTH,
traindicated in pyruvate carboxylase so a 24-h inpatient video EEG is recom-
deficiency, primary carnitine deficiency, mended after 2 weeks of treatment to
carnitine palmitoyl transferase I and evaluate for subtle persistent infantile
II deficiencies, carnitine translocase spasms. If difficult infantile spasms
deficiency, beta-oxidation defects and or hypsarrhythmia are detected, then
porphyria. prompt treatment modification may still
be needed.
zz Suggested Reading
55 Lee RR, Kossoff EH. Dietary treatments zz Suggested Reading
for epilepsy: management guidelines for 55 Pellock JM, Hrachovy R, Shinnar S, et al.
the general practitioner. Epilepsy Behav. Infantile spasms: a U.S. consensus report.
2011;21(2):115–21. Epilepsia. 2010;51:2175–89.
268 Chapter 9 · Epilepsy 2

?? 48. What is the mechanism of action of vv Correct Answer is: E


vigabatrin? Treatment of status epilepticus is initi-
A. activation of GABA-B receptors ated with benzodiazepines. Because of
B. allosteric binding to GABA-A greater lipid solubility, diazepam has a
receptors more rapid onset of action. On the other
C. increasing GABA synthesis hand, lorazepam has a longer lasting
D. inhibiting neuronal and glial reup- anticonvulsant activity with less respira-
take of GABA tory side effects due to smaller volume
E. inhibition of GABA transaminase distribution. As shown in the Rapid Anti-
convulsant Medication Prior to Arrival
vv Correct Answer is: E Trial (RAMPART) study, intramuscular
Vigabatrin is an irreversible inhibitor midazolam was noninferior by a margin
of GABA transaminase, and its antiepi- of 10% to that of intravenous lorazepam
leptic effects are based on inhibition of in patients treated by paramedics for
metabolic breakdown of GABA. Pheno- status epilepticus and had comparable
barbital and benzodiazepines bind to safety. All these first-line agents act on
GABA-A receptors; tiagabine inhibits a subset of benzodiazepine-sensitive
neuronal and glial reuptake of GABA; GABAA receptors, whose reduction in
baclofen (not an AED) activates GABA- surface expression during status epi-
B receptors; valproic acid is thought lepticus parallels their time-dependent
9 to possibly increase GABA synthesis, efficacy. In systematic review of prior
among other actions. studies for treatment of refractory
status epilepticus, pentobarbital was
zz Suggested Reading associated with the lowest rates of acute
55 Browne TR, Holmes GL. Handbook of failure and withdrawal seizures, but the
epilepsy. 4th ed. Philadelphia: Lippincott: highest rates of systemic hypotension
Williams & Wilkins; 2008. requiring pressors.

?? 49. Which of the following statements is zz Suggested Reading


most true about the management of 55 Treiman DM. The role of benzodiazepines
status epilepticus? in the management of status epilepticus.
A. among the anesthetic agents, Neurology. 1990;40(5 Suppl 2):32–42.
intravenous midazolam has the Review.
lowest rates of breakthrough 55 Claassen J, Jirsch LJ, Emerson RG.
­seizures Treatment of refractory status epilepticus
B. intramuscular midazolam has infe- with pentobarbital, propofol, or mid-
rior efficacy compared to intrave- azolam: a systematic review. Epilepsia.
nous lorazepam 2002;43(2):146–53. Review.
C. intravenous diazepam has longer 55 Goodkin HP, Joshi S, Mtchedlishvili Z,
lasting anticonvulsant activity et al. Subunit-specific trafficking of
than lorazepam GABA(A) receptors during status
D. intravenous lorazepam has more ­epilepticus. J Neurosci. 2008;28(10):
rapid onset than diazepam 2527–38.
E. there is a time-­dependent efficacy 55 Silbergleit R, Lowenstein D, Durkalski V,
of first-line agents Conwit R; NETT Investigators. Lessons
Epilepsy 2
269 9
from the RAMPART study--and which is 55 Smith MC, Byrne R, Kanner AM, et al.
the best route of administration of Corpus callosotomy and multiple subpial
benzodiazepines in status epilepticus. transection. In: Wyllie E, Cascino G, Gidal B,
Epilepsia. 2013;54(Suppl 6):74–7. editors. Wyllie’s treatment of epilepsy:
55 Browne TR. The pharmacokinetics of principles and practice. Philadelphia:
agents used to treat status epilepticus. Lippincott Williams & Wilkins; 2011.
Neurology. 1990;40(5 Suppl 2):28–32.
Review. ?? 51. Which cardiac conduction abnormal-
ity has been reported with rufin-
?? 50. A focal disconnection surgical proce- amide?
dure would be favored over a resec- A. prolonged PR interval
tive procedure with which of the B. prolonged QT interval
following conditions? C. right bundle branch block
A. Hemimegalencephaly D. shortened PR interval
B. Cortical dysplasia E. shortened QT interval
C. Rasmussen encephalitis
D. Sturge-Weber syndrome vv Correct Answer is: E
E. Landau-Kleffner syndrome Rufinamide has been reported to
shorten the QT interval, and is con-
vv Correct Answer is: E traindicated in patients with short QT
Rasmussen encephalitis, hemimega- interval. It has been advised to obtain a
lencephaly, Sturge-Weber syndrome baseline EKG prior to initiating therapy
and cortical dysplasias are commonly with rufinamide.
associated with refractory epilepsy
that can be surgically curable with a zz Suggested Reading
radical resection such as a modified 55 Spanaki MV, Barkley GL. An overview of
hemispherectomy. Landau-­Kleffner on third-generation antiseizure drugs:
the other hand is a form of acquired Clobazam, lacosamide, rufinamide and
epileptic aphasia arising from the pos- vigabatrin. Neurol Clin Pract. 2012;2:
terior temporal region of the dominant 236–41.
hemisphere and is either treated with
medications or with disconnection ?? 52. Which of the following sporting
surgical procedures, such as multiple activities is classified as low risk for
subpial transections, aiming to preserve persons with epilepsy?
the eloquent cortex in that region. A. baseball
B. scuba diving
zz Suggested Reading C. gymnastics
55 Gonzalez-Martinez JA, Gupta A, Gidal B, D. hockey
et al. Hemispherectomies, hemispheroto- E. soccer
mies and other hemispheric disconnec-
tions. In: Wyllie E, Cascino G, Gidal B, vv Correct Answer is: A.
editors. Wyllie’s treatment of epilepsy: Sports activities for an epilepsy patient
principles and practice. Philadelphia: should be decided on an individual
Lippincott Williams & Wilkins; 2011. basis. Scuba diving, hang gliding,
boxing, and hockey are classified as a
270 Chapter 9 · Epilepsy 2

­ igh-risk activity. On the other hand,


h C. gelastic seizures
football, soccer, basketball, gymnastics, D. juvenile absence epilepsy
horseback riding are moderate risk E. mesial temporal lobe epilepsy
activities, but golf, bowling, ping-pong with hippocampal sclerosis
and baseball are low risk activities.
vv Correct Answer is: A
zz Suggested Reading The clinical presentation of this patient
55 ILAE Commission report. Restrictions for would be consistent with frontal lobe
children with epilepsy. Commission of epilepsy. Given the almost exclusive
Pediatrics of the ILAE. International nocturnal events, family history, and
League Against Epilepsy. Epilepsia. description, this would make autoso-
1997;38(9):1054–6. mal dominant nocturnal frontal lobe
55 Drazkowski JF, Sirven JI. Driving and epilepsy likely. The genetic studies per-
social issues in epilepsy. In: Wyllie E, formed confirmed the classic chromo-
Cascino G, Gidal B, editors. Wyllie’s somal abnormality associated with this
treatment of epilepsy: principles and condition.
practice. Philadelphia: Lippincott
Williams & Wilkins; 2011. zz Suggested Reading
55 Mesad SM. Epilepsy and the athlete. In: 55 Combi R, Dalpra L, Tenchini ML,
Jordan BD, Tsairis P, Warren R, editors. Ferini-Strambi L. Autosomal dominant
9 Sport neurology. 2nd ed. Philadelphia: nocturnal frontal lobe epilepsy – A
Raven Press; 1998. critical overview. J Neurol.
2004;251(8):923–34.
?? 53. A 14-year-old male has been evalu- 55 Oldani A, Zucconi M, Asselta R, et al.
ated for nocturnal seizures. His Autosomal dominant nocturnal frontal
birth and past medical history are lobe epilepsy--A video-polysomno-
unremarkable. There is a family his- graphic and genetic appraisal of forty
tory nocturnal seizure in his father patients and delineation of the epileptic
that began in adolescence, are well syndrome. Brain. 1998;121(Pt 2):
controlled, and persisted into adult- 205–23.
hood. The boy’s seizures happened
exclusively in sleep, are brief (lasting ?? 54. Blue discoloration of the skin and
less than 30 s), and characterized by retinas has been associated with use
sudden eye opening, head raising or of which of the following antiepileptic
sitting up in bed, often with a fright- medications?
ened expression, and sometimes A. eslicarbazepine
vocalization. His interictal awake and B. ezogabine
drowsy EEG is normal. Genetic studies C. lacosamide
are performed, showing a mutation D. perampanel
in the CHRNA4 gene coding the alpha E. rufinamide
subunit of the neuronal nicotinic ace-
tylcholine receptor (nAChR). vv Correct Answer is: B.
What is your most likely diagnosis? Prolonged use of ezogabine (also
A. autosomal dominant nocturnal known as retigabine) has been associ-
frontal lobe epilepsy (ADNFLE) ated with a bluish discoloration of lips,
B. familial focal epilepsy with vari- fingernails and skin, as well as bluish
able foci discoloration of the retina.
Epilepsy 2
271 9
zz Suggested Reading treatment of the obstructive sleep apnea
55 Faulkner MA, Burke RA. Safety profile of with continuous positive airway pres-
two novel antiepileptic agents approved for sure (CPAP). In patients who are over-
the treatment of refractory partial seizures: weight and have a history of snoring and
ezogabine (retigabine) and perampanel. excessive daytime sleepiness, a sleep
Expert Opin Drug Saf. 2013;12:847–55. study would be the next best strategy in
55 FDA safety warning. their management. If the study shows
evidence of obstructive sleep apnea,
?? 55. An 18-year-old male came for evalua- treatment with CPAP could improve the
tion of medically resistant, idiopathic patient’s seizure control, excessive day-
complex partial seizures. He started to time sleepiness, and quality of life before
have his seizures in his adolescence; further interventions are performed. All
brain MRI is normal. His physical and the antiepileptic medications he used
neurologic exams are normal. He can increase weight. If the patient is
failed pregabalin, carbamazepine, overweight, changing to an antiepileptic
gabapentin, and valproate. His current medication not associated with weight
medication is valproate. The seizures gain should be considered, as obesity
occur in the day time and in sleep, can contribute to sleep apnea.
but more often in sleep and are char-
acterized by awakening, staring off, zz Suggested Reading
automatisms and nonresponsiveness 55 Derry CP, Duncan S. Sleep and epilepsy.
for up to 90 s. Approximately 25% go Epilepsy Behav. 2013;26(3):394–404.
on to secondary generalized tonic- 55 Mallow BA, Fromes GA, Aldrich MS.
clonic activity. He has a significant his- Usefulness of polysomnography in
tory of snoring at night and excessive epilepsy patients. Neurology.
daytime sleepiness for the last 2 years. 1997;48(5):1389–94.
What is the best next step in his
­management? ?? 56. Medical management would be
A. addition of perampanel to the favored over surgical evaluation in
patient’s drug regimen which of the following cases of refrac-
B. counsel the patient on weight loss tory epilepsy?
C. obtain a sleep study A. a 9-month-old boy presents with
D. refer the patient for a clinical trial refractory infantile spasms in
of a novel antiepileptic drug the context of multifocal tubers
E. vagus nerve stimulation therapy on MRI and TSC1 mutation on
genetic testing
vv Correct Answer is: C B. a 15-year-old boy presents with
People with epilepsy frequently report refractory nocturnal frontal lobe
day time sleepiness and poor quality seizures in the context of normal
of sleep. Patients with excessive day MRI and presence of CHRNA4
time sleepiness have significantly lower mutation on genetic testing
quality of life scores. The association C. a 20-year-old woman presents
between obstructive sleep apnea and with refractory generalized con-
epilepsy has been known for years. vulsions in sleep, in the context of
Obstructive sleep apnea is common in right frontal lobe dysplasia on MRI
men who are overweight. Seizure con- and 15q14 chromosome mutation
trol will improve significantly after the on genetic testing
272 Chapter 9 · Epilepsy 2

D. a 25-year-old man presents with current practice. Curr Neurol Neurosci


refractory complex partial seizures Rep. 2012;12(4):445–55.
in the context of cavernous mal- 55 Dashti SR1, Hoffer A, Hu YC, Selman WR.
formations in the left temporal Molecular genetics of familial cerebral
pole and the right frontal lobes cavernous malformations. Neurosurg
and CCM2 mutation on genetic Focus. 21(1):e2.
testing
E. a 30-year-old woman presents ?? 57. An 18-year-old female with idiopathic
with refractory complex partial complex partial seizures. She has
seizures in the context of right been seizure-free for years on 400 mg
mesial temporal sclerosis on MRI carbamazepine twice daily. One
and GABRA1 mutation on genetic month ago, her primary care physi-
testing cian added fluoxetine because of her
mood disorder. She is complaining
vv Correct Answer is: B of diplopia, nausea, and gait and
Autosomal dominant nocturnal frontal ­balance difficulties.
lobe epilepsy is an inherited genetic What is the best next step in her
condition due to widespread dysfunc- ­management?
tion in the nicotinic acetylcholine A. discontinue fluoxetine and
receptors and would therefore not be replace it with fluvoxamine
9 surgically remediable. The other con- B. discontinue fluoxetine and
ditions listed could still demonstrate replace it with citalopram
potentially focal epileptogenicity, and C. obtain a urine toxicology study
thus merit presurgical evaluation. D. obtain an MRI of the brain
E. send her to laboratory for a serum
zz Suggested Reading sodium level
55 Steinlein OK, Kaneko S, Hirose S.
Nicotinic acetylcholine receptor muta- vv Correct Answer is: B
tions. In: Noebels JL, Avoli M, Rogawski Carbamazepine is metabolized exten-
MA, et al. editors. Jasper’s basic mecha- sively through the P450 CYP3A4
nisms of the epilepsies. 4th ed. Bethesda: enzyme system. Medications used to
National Center for Biotechnology influence mood may affect this metabo-
Information (US); 2012. lism. Both fluoxetine and fluvoxamine
55 Loddenkemper T, Cosmo G, Kotagal P, inhibit metabolism of the CYP3A4
et al. Epilepsy surgery in children with isoenzyme, resulting in increasing car-
electrical status epilepticus in sleep. bamazepine levels significantly. This
Neurosurgery. 2009;64(2):328–37. medication interaction can produce the
55 Jeha LE, Morris HH, Burgess RC. symptoms of carbamazepine toxicity,
Coexistence of focal and idiopathic namely diplopia, ataxia, and gastroin-
generalized epilepsy in the same patient testinal upset. If a medication is needed
population. Seizure. 2006;15(1):28–34. for mood disorder, both citalopram
55 Bernasconi A, Bernasconi N, Bernhardt BC, and escitalopram have no effect on the
Schrader D. Advances in MRI for ‘crypto- CYP450 system and would not have
genic’ epilepsies. Nat Rev Neurol. an interaction. Either of these medica-
2011;7(2):99–108. tions would be a good alternative for
55 Michelucci R, Pasini E, Riquzzi P, et al. this patient. The urine toxicology test is
Genetics of epilepsy and relevance to likely to be n
­ egative given the history.
Epilepsy 2
273 9
Carbamazepine can produce hyponatre- absence epilepsy, seizures may persist
mia, but the sudden onset of her symp- for many years, but the prognosis is
tom after years of treatment would be usually good and response to treat-
unlikely due to hyponatremia. ment is excellent. Similarly, in juvenile
myoclonic epilepsy, response to treat-
zz Suggested Reading ment is good in 85% of patients, but
55 Mula M. Anticonvulsant--antidepressant it is a lifelong treatment. On the other
pharmacokinetic drug interactions: the hand, patients with hippocampal
role of the CYP450 system in psychophar- sclerosis are likely to be intractable in
macology. Curr Drug Metab. about 90% of cases.
2008;9(8):730–7.
55 Spina E, Santoro V, D’Arrigio C. Clinically zz Suggested Reading
relevant pharmacokinetic drug interac- 55 Bergqvist C. Idiopathic pediatric epilepsy
tions with second-generation antidepres- syndromes. Continuum Lifelong Learn
sants: an update. Clin Ther. Neurol. 2007;13(4):106–20.
2008;30(7):1206–27. 55 D’Argenzio L, Cross JH. Hippocampal
sclerosis and dual pathology. In: Wyllie E,
?? 58. Which of the following patients car- Cascino G, Gidal B, editors. Wyllie’s
ries the highest chances of becoming treatment of epilepsy: principles and
seizure free? practice. Philadelphia: Lippincott
A. an 8-year-old boy with benign Williams & Wilkins; 2011.
childhood epilepsy with centro-
temporal spikes ?? 59. The predominant site of action of the
B. an 11-year old boy with benign new antiepileptic (antiseizure) medi-
childhood epilepsy with occipital cation perampanel is on:
spikes A. AMPA receptors
C. a 12-year-old girl with juvenile B. NMDA receptors
absence epilepsy C. postsynaptic GABA receptors
D. a 20-year-old woman with juve- D. presynaptic SV2A receptors
nile myoclonic epilepsy E. voltage-dependent sodium
E. a 35-year-old man with lesional ­channels
mesial temporal lobe epilepsy
vv Correct Answer is: A
vv Correct Answer is: A Perampanel is a selective noncompeti-
In benign childhood epilepsy with cen- tive antagonist of AMPA receptors, a
trotemporal spikes (Rolandic epilepsy), major subtype of excitatory amino
remission occurs almost universally acid receptors activated by glutamate.
by adolescence and treatment during It has no other known major mecha-
symptomatic period is controversial; nism of action. Benzodiazepines and
1% of patients develop refractory barbiturates act on GABA receptors,
seizures into adulthood. In benign while phenytoin, carbamazepine, and
childhood occipital epilepsy of Gastaut lamotrigine are voltage-dependent
type, the remission rates are much sodium channels. Levetiracetam acts
than Rolandic epilepsy. In childhood on SV2A receptors to modulate neu-
absence epilepsy, spontaneous remis- ronal activity. Ketamine blocks NMDA
sion occurs in 65% to 70% of patients receptors, but there are no chronic oral
during adolescence. In juvenile seizure medications that act on this
274 Chapter 9 · Epilepsy 2

excitatory amino acid receptor; actions benign familial neonatal convulsions,


at this site cause a variety of major side and mutations in the KCNQ2 gene can
effects, predominantly psychological also result in a more severe disorder
and sedative. with early-onset epileptic encephalopa-
thy. KCNT1 associated with migrating
zz Suggested Reading partial seizures of infancy. ALDH7A1 is
55 Rogawski MA. AMPA Receptors as a associated with pyridoxine-dependent
molecular target in epilepsy therapy. Acta epilepsy.
Neurol Scand Suppl. 2013;(197):9–18.
zz Suggested Reading
?? 60. A 5-year-old boy has known epilepsy 55 De Vivo DC, Trifiletti RR, Jacobson RI,
with multiple seizure types and mild et al. Defective glucose transport across
developmental delay. Seizure started the blood-brain barrier as a cause of
at 5 months of age, with focal sei- persistent hypoglycorrhachia, seizures, and
zures characterized by staring, and developmental delay. N Engl J Med.
cyanosis. His EEG showed multifocal 1991;325:703–9.
spike-wave discharges. Seizures were
refractory to initial anticonvulsant ?? 61. An 8-month-old male infant devel-
trials. He developed new seizure ops seizures 2 days after receiving a
types including atypical absence, vaccine for Haemophilus influenza.
9 myoclonic, and tonic-clonic, with Initially, his seizures are relatively
generalized epileptiform discharges infrequent clonic seizures. At the
on EEG. Seizures were completely age of 10 months, he started to have
controlled with the introduction of frequent, prolonged myoclonic sei-
the ketogenic diet. zures. He also has infrequent absence
Mutation of which of the following seizures. He subsequently develops
genes would confirm the diagnosis? two episodes of refractory status
A. SLC2A1 epilepticus. His interictal EEG shows
B. SCN1A generalized polyspike-and-­wave
C. KCNQ2 and spike-and-wave discharges of
D. KCNT1 1.5–2 Hz.
E. ALDH7A1 Screening for mutations in which of
the following genes is most likely
vv Correct Answer is: A. to reveal the etiology of this child’s
The clinical presentation indicates glu- ­epilepsy?
cose transporter type I deficiency syn- A. CHRNA4
drome (GLUT1-DS) that is characterized B. EPM1
by infantile onset of seizures that are C. EPM2A
refractory to anticonvulsants, and the D. SCN1A
development of generalized seizures E. TSC1
in early childhood. GLUT1-DS is known
as “De Vivo disease” and diagnosed vv Correct Answer is: D
with careful CSF glucose concentration This child has severe myoclonic epi-
measurements. SLC2A1 gene muta- lepsy of infancy, also known as Dravet
tion is known to cause GLUT1-DS; and syndrome. 75% of Dravet syndrome
testing can confirm the clinical suspi- cases are secondary of mutations to the
cion. SCN1A is associated with Dravet SCN1A gene. Fever or vaccination can
syndrome. KCNQ2 is associated with trigger the seizures in 30% to 40% of
Epilepsy 2
275 9
infants with severe myoclonic epilepsy. patients with new-onset epilepsy and
Initial seizures are relatively infrequent in patients in remission. Incidence is
clonic or tonic-clonic seizures. How- about 0.35 cases/1000 person-years
ever, affected individuals subsequently in population-­based incidence cohort
developed abundant severe, prolonged of epilepsy. Incidence is consider-
myoclonic seizures, as well as absence ably higher in patients with chronic
seizures and focal motor seizures. Fre- epilepsy, 1–2/1000 person-years, and
quent episodes of status epilepticus are highest with severe, refractory seizures,
not uncommon in this condition. Muta- 3–9/1000. The highest rates occur from
tions in CHRNA4 are typically seen in 20 to 40 years. Refractory epilepsy with
individuals with nocturnal frontal lobe convulsive seizures and onset at an
epilepsy. Mutations in EPM1 and EPM2A early age, as well as long duration and
lead to Unverricht-Lundborg and Lafora polytherapy are other reported risk fac-
body disease, respectively, which are tors. Some studies suggest that postic-
two forms of progressive myoclonic epi- tal generalized electroencephalography
lepsy that affect children and teenag- suppression greater than 20 s following
ers. Mutations in TSC1 lead to tuberous generalized convulsions is a risk factor
sclerosis. for SUDEP, but this contention as an
independent risk factor, as opposed
zz Suggested Reading to an epiphenomenon, is still debated.
55 Marini C, Scheffer IE, Nabbout R, et al. Similarly, some animal and human data
The genetics of Dravet syndrome. have suggested that SSRIs are associ-
Epilepsia. 2011;52. ated with reduced likelihood of ictal
oxygen desaturation in patients with
?? 62. Sudden unexplained death in epi- partial seizures.
lepsy (SUDEP) is best characterized by
which of the following factors? zz Suggested Reading
A. may be associated with profound 55 Tomson T, Walczak T, Sillanpaa M,
and prolonged postictal EEG sup- Sander JW. Sudden unexpected death in
pressions epilepsy: a review of incidence and risk
B. more common in patients with factors. Epilepsia. 2005;46(Suppl 11):54–61.
refractory atonic seizures 55 Bateman LM et al. Serotonin reuptake
C. more frequent in children inhibitors are associated with reduced
D. occurs from status epilepticus severity of ictal hypoxemia in medically
E. risk may be increased in patients refractory partial epilepsy. Epilepsia.
on selective serotonin reuptake 2010;51(10):2211–4.
inhibitors (SSRIs) 55 Lhatoo SD, Faulkner HJ, Dembny K, et al.
An electroclinical case-control study of
vv Correct Answer is: A sudden unexpected death in epilepsy. Ann
Sudden unexplained death in epilepsy Neurol. 2010;68(6):787–96.
(SUDEP) is defined as unexpected, wit-
nessed or unwitnessed, nontraumatic ?? 63. You are requested to evaluate a
and nondrowning deaths in patients 3-day-old boy, who is irritable with
with epilepsy, with or without evidence poor feeding. He is having multiple
of a seizure and excluding documented focal seizures characterized by apnea
status epilepticus. Postmortem exami- and clonic jerking. His mother had
nation does not reveal a toxicologic or no complication during pregnancy
anatomic cause of death and is rare in and delivery. He has no family history
276 Chapter 9 · Epilepsy 2

of epilepsy. MRI brain is normal. partial seizures, possibly from the fron-
The interictal EEG background is tal lobe. His seizures have been totally
excessively discontinuous for stated controlled with carbamazepine.
postconceptional age. He was refrac- This is associated with a genetic muta-
tory to phenobarbital, but seizures tion that encodes for which of the
were subsided with a trial of oral ­following areas?
­pyridoxine. A. NMDA receptor
Which gene is the most likely respon- B. one of the voltage-­gated potas-
sible for his seizures? sium channels
A. ALDH7A1 C. SCN1A sodium channel
B. KCNQ2 D. subunit of the GABA-A receptor
C. KCNT1 E. subunit of the nicotinic acetylcho-
D. SCN1A line receptor
E. SLC2A1
vv Correct Answer is: E
vv Correct Answer is: A The clinical scenario is consistent with
The clinical history is most consistent autosomal dominant nocturnal frontal
with pyridoxine-dependent epilepsy, lobe epilepsy (ADNFLE)--a disorder that
caused by mutations in ALDH7A1. Dra- produces brief, often multiple nocturnal
vet syndrome is usually secondary to frontal lobe seizures during slow wave
9 mutations in the SCN1A gene. In the sleep. This is an autosomal dominant dis-
past, KCNQ2 was associated only with order with70% to 80% penetrance, and
benign familial neonatal convulsions, it is associated with a mutation in the
but recently different mutations are gene CHRNA4, encoding the alpha 4 sub-
responsible for previously undiagnosed unit of a neuronal nicotinic acetylcholine
cases of Ohtahara syndrome. SLC2A1 is receptor (nAchR). This was the first ion
the only gene in which mutations are channel mutation found in an ­inherited
known to cause GLUT1-DS and testing form of epilepsy. How this mutation pro-
can confirm the clinical suspicion. KCNT1 duces epilepsy is still unknown.
is the causative gene for the migrating
partial seizures of infancy. zz Suggested Reading
55 Steinlein OK, Mulley JC, Propping P et al.
zz Suggested Reading A missense mutation in the neuronal
55 Gospe SM. Pyridoxine-­dependent nicotinic acetylcholine receptor apha 4
epilepsy. In: Pagon RA, Adam MP, subunit is associated with autosomal
Ardinger HH, et al. editors. GeneReviews® dominant nocturnal frontal lobe epilepsy.
[Internet]. Seattle: University of Nat Genet 1995;11:201–3.
Washington, Seattle; 1993–2014. 55 Guerrini R, Marini M, Mantegazza
M. Genetic epilepsy syndromes without
?? 64. A 12-year-old boy is evaluated for structural brain abnormalities: clinical
his history of hyperkinetic nocturnal features and experimental models.
events, sometimes with tonic or dys- Neurotherapeutics. 2014;11:269–85.
tonic features. They are stereotyped,
often cluster and may have relatively ?? 65. A 4-day-old boy with normal birth his-
preserved consciousness. His family tory has tonic seizures, characterized
history showed a similar disorder in his by stiffening and apnea. His inter-
father and older sister. He had a video- ictal EEG shows burst suppression.
EEG monitoring that showed nocturnal Pyridoxine trial was ineffective. MRI
Epilepsy 2
277 9
brain was normal. CSF, serum, and shows predominantly asynchronous
urine tests results were negative. His bursts of medium to high voltage
seizures remain refractory to multiple delta activity of approximately 3–5 s
anticonvulsant trials, but gradually duration alternating with 1–2 s of
decrease in frequency at 2 years old. interburst periods of 25 μV to 50 μV,
He has severe cognitive and motor mixed theta and delta activity.
impairments without progression. What is the EEG pattern consistent
Which gene mutation will most likely with?
confirm the diagnosis? A. activité moyenne
A. ALDH7A1 B. burst suppression
B. KCNQ2 C. theta pointu alternant
C. KCNT1 D. tracé alternant
D. SCN1A E. tracé discontinu
E. SLC2A1
vv Correct Answer is: D
vv Correct Answer is: B. This is the normal trace alternant EEG
KCNQ2 was previously associated only pattern, which is the normal quiet sleep
with benign familial neonatal convul- background for a term neonate. Trace
sions. Different mutations were recently discontinu is another normal sleep back-
found with previously undiagnosed ground seen in preterm neonates which
cases of Ohtahara syndrome. SLC2A1 is is more discontinuous, as the name
the only gene in which mutations are implies. Activité moyenne is the normal
known to cause GLUT1-DS and has been wake background in a term neonate
associated with early onset seizures and characterized by continuous mixed
(e.g., absence). SCN1A is associated frequency activity. Burst suppression is
with Dravet syndrome. KCNT1 is associ- a severely abnormal background char-
ated with migrating partial seizures of acterized by invariant bursts and pro-
infancy. ALDH7A1 is associated with longed low-voltage interburst intervals.
pyridoxine dependent epilepsy. Theta pointu alternant is characterized
by runs of unilateral alternating theta
zz Suggested Reading activity in admixed sharp waves; this
55 Weckhuysen S, Ivanovic V, Hendrickx R, pattern is nonspecific and is an interictal
et al. on behalf of the KCNQ2 Study finding that may be seen associated with
Group. Extending the <em>KCNQ2</em> multiple causes of neonatal seizures.
encephalopathy spectrum.
zz Suggested Reading
?? 66. A 2-day-old baby has a routine EEG 55 Tsuchida TN, Wusthoff CJ, Shellhaas RA,
in the nursery because he has inter- et al. American Clinical Neurophysiology
mittent episodes of apnea. He was Society Critical Care Monitoring
born by vaginal delivery after uncom- Committee.
plicated pregnancy at 39 weeks 55 American clinical neurophysiology society
gestation. Family history is negative standardized EEG terminology and
for epilepsy. The infant is otherwise categorization for the description of
healthy, vigorous, and feeding well. continuous EEG monitoring in neonates:
He has normal exam. The EEG showed report of the American Clinical
quiet sleep that was clinically charac- Neurophysiology Society critical care
terized by closed eyes and little move- monitoring committee. J Clin
ment. The EEG background activity Neurophysiol. 2013; 30:161–73.
278 Chapter 9 · Epilepsy 2

?? 67. To apply for the social security admin- ?? 68. 3-year-old girl came to the ED by EMS
istration disability benefits in relation after she abruptly became limp and
to epilepsy, the presence of which exhibited shaking of her arms and
of the following factors would be legs in her daycare. While she was
­sufficient? playing with other children, she fell
A. Three daytime nonconvulsive sei- down, cried briefly, and then she lost
zures per month, which interfere her conciseness. In the ambulance,
with awareness she woke up, started crying for her
B. history of prior employment mother. She has normal general and
C. nocturnal convulsions without neurologic examinations. Her mother
prolonged postictal confusion reports that she had previous 3 such
D. One convulsive seizure every episode and all of them have been
2 months provoked by minor injuries. Her EKG
E. significant comorbid conditions was normal after the first episode.
What is your next step in her
vv Correct Answer is: E ­management?
Regardless of prior employment, con- A. perform a 1-week cardiac event
sideration of social security adminis- monitor
tration disability benefits can be given B. perform MRI of the brain
in patients with epilepsy if they have C. perform routine electroencepha-
9 one of the following: (1) at least four logram (EEG)
partial seizures per month that are of D. prescribe levetiracetam
sufficient severity to cause clouding E. reassurance
of consciousness or significant impair-
ment of daytime activities (2) at least vv Correct Answer is: E
one daytime convulsive seizure per Pediatric neurologists always have a lot
month (3) predominantly nocturnal of paroxysmal spells referrals. Usually,
convulsive seizures with aftereffects the concern is about possible seizure.
that persist during the day, or (4) Typically, the pediatric neurologist does
significant neuropsychiatric or medi- not witness the event and children are
cal comorbidities exist. The treating normal between events, so the neuro-
physician usually is asked to submit a logic examination is of limited benefit.
medical documentation and opinion A full detailed history is the way to
on the disability claim and can there- figure out the nature of the spell and
fore function as an advocate for the determining whether further evalu-
patient by helping to ensure that they ation or treatment is needed. Video
are fairly considered for employment recording by caregivers is off utmost
or benefits. importance. The patient spells are trig-
gered by minor injuries and probably
zz Suggested Reading represent a vasovagal attack (pallid
55 Social Security website [accessed May breath-holding spells.) Other children
2014]. cry vigorously and get “caught” in the
55 Drazkowski JF, Cascino G, Gidal BE, et al. exhalation phase of breathing. In this
Driving and social issues in epilepsy. In: case, their face becoming deep red,
Wyllie E, Cascino G, Gidal BF, editors. their bodies often becoming rigid, they
Wyllie’s treatment of epilepsy: principles lose consciousness; and then they begin
and practice. Philadelphia: Lippincott breathing again (cyanotic breath-hold-
Williams & Wilkins; 2011. ing spells). Children with these spells
Epilepsy 2
279 9
require no further testing or treatment, ing, and complex migraine headaches.
only parental reassurance.Different Children with developmental delay
types of paroxysmal non-epileptic spells and autism of all ages commonly have
come different ages. In infancy, it is periods of disengagement, or other
quite common for the immature cortex stereotyped movements. For all of these
to fail to inhibit unwanted, or undesired, conditions, a detailed history is the
movements. These can occur in wakeful- most important aspect of evaluation.
ness, in the case of clonus, which is not
always a sign of pathology in a young ?? 69. A 6-month-old girl is admitted to
infant; or during sleep, as in the setting the hospital for a cluster of sei-
of benign sleep myoclonus of infancy. zures. Her seizures are typically last
On the other hand, gastroesophageal 1–2 min and associated with sud-
reflux can cause an unwanted move- den behavioral arrest, followed by
ments, sometimes associated with color clonic activity of her extremities.
changes, which can trigger laryngo- She has been having those seizures
spasm and affect breathing--leading to for the last month; however, they
dystonic posturing. During the toddler were shorter and only one every few
years, breath-­holding spells and infan- days, until the day of admission. She
tile gratification disorders are common; became less interactive, especially
and they often mark the onset of the with visual attention over the past
sleep parasomnias, such as night ter- 2 weeks. She is not sick, has no fever
rors. Breath-holding spells in particular or upper respiratory symptoms.
can often mimic seizures; breathing Her head circumference at the 10th
is affected, often followed by loss of percentile, with height and weight
postural tone and then abnormal move- at the 25th percentile. She does not
ments (similar to those associated with have any dysmorphic features or
syncope.) Color change, either pallor or neurocutaneous lesions are seen.
cyanosis, occurs. A history of consistent She is alert, has normal visual regard,
provocation of the spells, by anger or conjugate tracking, and normal suck-
pain, should raise suspicion.Older chil- ing abilities. She has mild decrease
dren can have continued sleep related in the truncal and extremity tone,
events (parasomnias, periodic limb but tendon reflexes are normal. She
movements in sleep, confusion arous- does not make any attempts to reach
als) which can be mistaken as seizures. or bear weight in sitting or stand-
Tic disorders typically begin in child- ing. Her brain MRI is normal. EEG
hood and the history of willful control, shows interictal sharp waves from
which can help distinguish tics from both sides of the brain, and during
simple partial seizures, is often lacking a seizure shows ictal epileptiform
in young children. Syncope causes col- patterns arising from the left central
lapse, often with a series of associated region. Her serum sodium is142;
myoclonic limb movements, which lead glucose 75; calcium of 9.2; and mag-
to a wrong seizure diagnosis. Other nesium of 2.2. Seizures are resistant
causes of transiently altered awareness to treatment with phenobarbital,
that can mimic absence or complex par- levetiracetam, and carbamazepine.
tial seizures include hypersomnia from Further evaluation includes a lumbar
multiple causes, including narcolepsy puncture which reveals 1 WBC; 0
and obstructive sleep apnea, attention RBC; glucose 30 mg/dL; and protein
deficit disorders, simple daydream- 20 mg/dL.
280 Chapter 9 · Epilepsy 2

What is the most likely diagnosis? a mutation in the SLC2A1 gene region
A. cerebral folate deficiency on chromosome 1, leads to inability to
B. glucose-1-transporter deficiency efficiently transport glucose across the
C. pyridoxine dependent epilepsy blood-­brain barrier. The hallmark labo-
D. severe myoclonic epilepsy of ratory finding in such patients is a low
infancy ­cerebrospinal fluid glucose level (with
E. tuberous sclerosis complex over 90% of patients having values less
than 40 mg/dL). When collected fast-
vv Correct Answer is: B ing, with a paired serum glucose level,
The child has a history of intractable there is also a low CSF/blood glucose
infantile-onset epilepsy, of multifo- ratio of less than 0.4 (with normal being
cal origin, and delayed development. over 0.5). Although typically presenting
Differential diagnosis is extensive, with seizures prior to the age of 2, there
including pre- and perinatal insult, are several other variants, including
malformations of cortical development, intermittent dystonia, choreoathetosis,
neurocutaneous disorders (including and ataxia. The disease occurs as a de
tuberous sclerosis), and inborn errors novo mutation in 90% of cases, with the
of metabolism, chromosomal dele- remaining 10% inherited in an autoso-
tion/duplication syndromes, vitamin/ mal dominant fashion.Treatment is cen-
cofactor responsive seizures, and the tered on utilization of ketones bodies as
9 symptomatic generalized epilepsy syn- an alternative source of energy for the
dromes of childhood.Our efforts have brain, and when instituted promptly,
to be focused on treatable causes of the ketogenic diet can lead to full con-
refractory epilepsy in infants and young trol of seizures, and even prevention
children. Until recently, empiric trials of the intellectual disability and micro-
of vitamins/cofactors such as pyridox- cephaly which result from chronically
ine, pyridoxal phosphate (the active low brain glucose levels. Although all
metabolite of pyridoxine) and folinic of the other options can lead to refrac-
acid were given for vitamin B6 depen- tory epilepsies in children, only GLUT-1
dency conditions, but now these disor- has the finding of a low cerebrospinal
ders are diagnosed by urine/blood/CSF ­glucose level.
metabolite studies or genetic testing.
Each one of these disorders can be par- zz Suggested Reading
tially responsive to conventional anti- 55 Pagon RA, Adams MP, Bird TD, et al.
epileptic therapies, though complete editors. GeneReviews. Seattle: University
seizure control is not achieved until the of Washington, Seattle; 1993–2013.
appropriate cofactor is supplemented in
the diet.Both cerebral folate deficiency ?? 70. A 17-year-old male hit his head after
and glucose-1-transporter deficiency accidental fall followed by a 2 min
(GLUT-1) are diagnosed chiefly by generalized tonic-clonic seizure. In
examination of the spinal fluid (though the ambulance, he came back to his
confirmatory genetic sequencing tests baseline. He was evaluated at the ED
are available for both.) Cerebral folate and found no injuries. He has a nor-
deficiency has a wide spectrum of mal brain CT. Serum toxicology was
neurologic presentations, including positive for cocaine. They kept him
seizures, and tends to respond at least overnight for observation and the fol-
partially to use of folinic acid.Glucose-­1-­ lowing morning had a normal neuro-
transporter deficiency (GLUT-1), due to logic examination and EEG.
Epilepsy 2
281 9
Which of the following is the best An abnormality affecting which of the
recommendation regarding duration following cellular structures is most
of antiepileptic drug therapy for this likely?
patient? A. AMPA receptors
A. 0 days B. nicotinic acetylcholine receptors
B. 14 days C. potassium channels
C. 6 months D. sodium channels
D. 1 year E. T-type calcium channels
E. 2 years
vv Correct Answer is: B
vv Correct Answer is: A This family history and seizure semiol-
7 days of antiepileptic drug (AED) pro- ogy is consistent with autosomal domi-
phylaxis is recommended after severe nant nocturnal frontal lobe epilepsy
traumatic brain injury. However, in a (ADNFLE). Those seizures are typically
mild injury, there is no definite con- brief, tonic and hyperkinetic motor
sensus. This patient had an immediate seizures occurring during the Non-REM
provoked seizure and had no sequelae, sleep. Vocalizations are common. Usual
therefore an AED is often not recom- onset is in childhood around 10 years of
mended. He also had two reasons for age. Bilateral frontal lobe epileptiform
a provoked seizure, the head trauma discharges are common during ictal
with the fall and possible acute use of EEG, but EEG is normal interictally. ADN-
cocaine. In cases of a provoked seizure, FLE is secondary to mutations in the
AED prophylaxis is not generally recom- CHRNA4, CHRNB2, and CHRNA2 encod-
mended. In addition, even after a single ing subunits of the neuronal nicotinic
nonprovoked seizure, AED prophylaxis is acetylcholine receptor. Nicotine treat-
not generally used. ment seems to reduce seizure frequency
in these patients.
zz Suggested Reading
55 Practice parameter: Antiepileptic drug zz Suggested Reading
prophylaxis in severe traumatic brain 55 Chyung A, Ptacek LJ. Genetics of epilepsy.
injury Report of the Quality Standards Continuum Lifelong Learn Neurol.
Subcommittee of the American Academy 2005;11(2):79–94.
of Neurology 55 Steinlein O, Mulley J, Propping P, et al.
A missense mutation in the neuronal
?? 71. A 16-year-old male came for his sei- nicotinic acetylcholine receptor alpha 4
zure evaluation. His seizures started subunit is associated with autosomal
at 10 years of age. They only happen dominant nocturnal frontal lobe epilepsy.
at night and involve hand clench- Nat Genet.
ing, rapid arm movements, and knee
bending. He shouts and screams ?? 72. A 5-year-old boy has been evaluated
during the seizures. He was misdi- for episode of unresponsiveness.
agnosed of having nightmares, but He awoke from sleep with vomit-
seizures have stopped with the use ing, appeared pale, and had dilated
of carbamazepine. He has a normal pupils. He vomited for 15 min, then
brain MRI and EEGs. His family history he suddenly became limp and unre-
is remarkable of his father, paternal sponsive with eyes open and deviated
uncle, and paternal grandmother all up and to the left. 15 min later, he
have similar seizures. began to respond and complained
282 Chapter 9 · Epilepsy 2

of a headache. His family reported 3 When she woke up. She was disori-
other episodes in the past 6 months ented, but then able to provide a his-
of awakening from sleep and vomit- tory of depression and one other GTC
ing repeatedly, but he never became seizure in the past, as well as morning
unresponsive with those spells. He episodes of arm jerking and dropping
has a normal neurologic examination objects. She takes citalopram and her
in the ED. PCP increased her dose recently to
Which of the following test results is 80 mg daily. Her EEG in the ED shows
most likely? bilateral theta slowing and occasional
A. EEG showing occipital spikes frontal intermittent rhythmic delta
B. extreme somatosensory evoked activity.
potentials What is her seizure etiology?
C. genetic testing showing a point A. cerebral vasospasm
mutation in the CACNA1A gene B. increased intracranial pressure
D. MRI showing pontine mass C. juvenile myoclonic epilepsy
E. serum test positive for antigliadin D. serotonin syndrome
antibodies E. sleep deprivation

vv Correct Answer is: A vv Correct Answer is: C


Panayiotopoulos syndrome (early onset, The clinical scenario of morning myoc-
9 benign focal epilepsy with occipital lonus and a generalized tonic-clonic
paroxysms) is a common focal epilepsy seizure is indicative of juvenile myo-
syndrome with a peak age of onset at clonic epilepsy. A postictal EEG may
5 years. The typical seizures consist pre- show interictal 4 Hz to 6 Hz generalized
dominantly of autonomic symptoms, spike-wave and polyspikes, but could be
more often arise from sleep, and tend normal or show postictal slowing. Post-
to be prolonged. A typical seizure is ictal slowing may consist of generalized
characterized by the child maintaining theta and delta slowing, as well as fron-
consciousness, complaining of feeling tal intermittent rhythmic delta activity
sick, and then turns pale and vomits. (FIRDA). Morning myoclonus is not
Other autonomic features may also be typical of increased intracranial pres-
present such as cyanosis, mydriasis, sure. Serotonin syndrome is not typi-
incontinence, or hypersalivation. 20% of cally associated with convulsions. Sleep
the seizures progress into unresponsive- deprivation can exacerbate seizures in
ness and loss of tone (ictal syncope). The individuals with epilepsy, but is not part
seizures may last from 30 min to several of this history.
hours. The interictal EEG shows multifo-
cal, occipitally predominant, spike- and zz Suggested Reading
slow waves accentuated by sleep. 55 Bergqvist C. Idiopathic pediatric epilepsy
syndromes. Continuum Lifelong Learn
zz Suggested Reading Neurol. 2007;13(4):106–20.
55 Bergqvist C. Idiopathic pediatric epilepsy
syndromes. Continuum Lifelong Learn
Neurol 2007;13(4):106. ?? 74. An 18-year-old male, with Lennox-
Gastaut syndrome, starts to have an
?? 73. An 18-year-old ­female was admitted increase in his seizures. Usually, he
to the ED after a witnessed general- has 1 to 5 seizures a day consisting of
ized tonic-clonic seizure at home. body stiffening and head drop. Four
Epilepsy 2
283 9
weeks ago, his seizures increased to received vaccines. He has a family his-
up to 20 seizures a day, and he has tory of a brother had febrile seizures
fallen twice in the past week. It is and now has 10–15 seizures a day,
not safe for him to walk unassisted despite taking multiple antiepileptic
because of the seizures. He is on rufin- drugs. His mother and maternal aunt
amide, levetiracetam, and zonisamide. had febrile seizures. His family would
He has tried multiple antiepileptic like to know more about the cause of
drugs in the past. his seizures.
What do you recommend as a next This patient most likely has a mutation
step in his management? in the gene associated with which of
A. add on perampanel the following cellular structures?
B. prescribe a wheelchair A. Calcium channel
C. prescribe gait belt B. GABA receptor
D. refer for a vagal nerve stimulator C. nicotinic acetylcholine receptor
E. refer for transcranial magnetic D. potassium channel
stimulation E. sodium channel

vv Correct Answer is: D vv Correct Answer is: E


Vagal nerve stimulation (VNS) is effica- Generalized epilepsy with febrile sei-
cious as well as the new antiepileptic zures plus (GEFS+) is an autosomal
drugs. A 2009 AAN Guideline concludes dominant epilepsy syndrome, with
that “sufficient evidence exists to rank approximately 60% to 70% penetrance.
VNS for epilepsy as effective and safe, Individuals can have multiple differ-
based on a preponderance of Class I evi- ent phenotypes. Some patients have
dence”. A study by Lundgren et al. found isolated simple febrile seizures and oth-
among 6 children with Lennox-Gastaut ers may have refractory epilepsy with
syndrome, 5 sustained at least a 90% multiple seizure types and associated
reduction of seizures. Transcranial mag- developmental compromise. The far end
netic stimulation and perampanel could of the spectrum is Dravet syndrome (aka
be effective, but have less evidence severe myoclonic epilepsy of infancy).
for use in this situation. A w
­ heelchair The most common genetic mutations
or gait belt would improve safety, but associated with this syndrome is in the
would not address the seizures increase. sodium channel SCN1A (also impli-
cated in severe myoclonic epilepsy of
zz Suggested Reading infancy).The patient in this vignette,
55 Fisher RS, Handforth A. Reassessment: and his brother, probably have Dravet
Vagus nerve stimulation for epilepsy. syndrome, a severe infantile-onset epi-
Neurology. 1999;53:666. lepsy syndrome with a distinctive but
55 Lundgren J, Amark P, Blennow G, et al. complex electroclinical presentation. A
Vagus nerve stimulation in 16 children healthy, developmentally normal infant
with refractory epilepsy. Epilepsia. present at around 6 months of age with
1998;39:809–13. convulsive status epilepticus, which
may be hemiclonic or generalized; sei-
?? 75. A 15-month-old boy is brought by zures may be triggered by fever, illness
his parents because he had 3 febrile or vaccination. The infant typically has
generalized tonic clonic seizures, further episodes of status epilepticus
each seizure lasted 30–45 min, one of every month or two, often triggered
which occurred the same day as he by fever. Other seizure types including
284 Chapter 9 · Epilepsy 2

focal dyscognitive seizures, absence and topiramate, valproic acid, levetirace-


myoclonic seizures develop between tam, lamotrigine, and zonisamide. The
1 and 4 years. Atonic drop attacks and most recent side effect is weight gain
episodes of nonconvulsive status may of 13 kg on valproic acid.
occur. Early development is normal, but She tried diet and exercising, but
slows in the second year. Developmen- kept gaining weight, so she stopped
tal regression may occur, particularly taking her medication 1 week ago
with status epilepticus. EEG studies and had a tonic-­clonic seizure 2 days
are initially normal, but after 2 years ago. She is not interested in more
they show generalized spike-wave and treatment for her epilepsy as “the side
polyspike-wave activity with multifo- effects of medication are worse than
cal discharges. Photosensitivity may be the seizures.”
seen. Imaging is normal or shows non- Which of the following approaches is
specific findings, such as atrophy.Dravet most likely to lead to the best possible
syndrome is associated with mutations outcome and compliance with further
of the gene encoding the alpha-1 sub- treatment attempts?
unit of the sodium channel, SCN1A, A. agree to observe without any fur-
in more than 70% of patients. These ther medications, but notify the
include sequencing mutations and state motor vehicle department of
copy number variant anomalies; 90% her treatment noncompliance
9 of mutations arise de novo. PCDH19 B. continue valproic acid and tell her
mutational analysis is a second-tier to go on a stricter diet and get
test for girls with a Dravet-like picture more exercise
who do not have SCN1A mutations. C. refer for psychiatric evaluation to
Outcome is poor, with intellectual dis- determine competency
ability in most patients and ongoing D. refer for surgical evaluation
seizures. Intellectual impairment varies E. review prior medication doses
from severe in 50% patients, to moder- and side effects in detail, with a
ate and mild intellectual disability each discussion of other non-­medical
accounting for 25% cases. Rare patients treatment options, including
have normal intellect. The long-term vagal nerve stimulator or diet
course involves ongoing, brief nocturnal modifications
­convulsions and a characteristic deterio-
ration in gait. vv Correct Answer is: E
It is not easy to deal with teenage hav-
zz Suggested Reading ing chronic medical conditions, it is
55 Wirrell E, Nickels KC. Pediatric epilepsy very important to respect their need for
syndromes. Continuum Lifelong Learn autonomy and for them to have a say in
Neurol. 2010;16(3):57–85. their own medical decision making. To
55 Scheffer IE. Diagnosis and long-term determine if an adolescent has the ability
course of Dravet syndrome. Eur J Paediatr to make medical decisions, it is crucial
Neurol. 2012;16(Suppl 1):S5–S8. to understand their mental abilities and
insight into their own medical illness.
?? 76. An 18-year-old female who was diag- Including parents in the decision-­making
nosed with juvenile myoclonic epilepsy process is also crucial, though their opin-
at 13 years of age, came for her routine ions should not necessarily carry more
follow-up visit. She has unwanted side weight than the teenager’s.This girl has
effects from the following medications, a lot of frustration regarding side effects;
Epilepsy 2
285 9
therefore, specifically detailing the side vv Correct Answer is: B
effects of previous medications would Children have 2 types of breath holding
be worthwhile. Highlighting the positive spells. Cyanotic breath-holding is usu-
and negative effects for each specific ally provoked by anger or frustration.
dose, of each medication, would help Pallid breath-holding spells are typically
designing a treatment plan using ratio- induced by pain, which results in exces-
nal polytherapy. It would be important sive vagal tone leading to bradycardia
to discuss other adjunctive treatments or asystole. Although cataplexy and
(surgical and dietary) to achieve the psychogenic nonepileptic events often
goal of minimizing the effect of seizures have an emotional trigger, presentation
on her quality of life; including safety, at age 3 would be unusual. Absence and
employability, and the ability to drive.If a temporal lobe seizures are the 2 types
teenager has an emotional, or cognitive of nonconvulsive seizures, presenting
disability, or not able to process informa- with episodes of behavioral arrest, not
tion for medical decision making, con- typically provoked by anger.
sultation from mental health providers,
and/or legal counsel, would be necessary ?? 78. A 4-year-old boy came to your office
to determine whether the patient is because of he has episodes of recur-
competent to assist in decision making. rent nausea and vomiting and also,
It seems clear that the patient presented he appears pale. An empiric trial
here has reasonable concerns, so these of ranitidine was not helpful, and
steps are likely to be unnecessary. upper endoscopy was performed and
normal. Cyproheptadine and later
zz Suggested Reading amitryptiline were used to treat a pre-
55 Rodgers-Neame N, Orlowski JP. Ethical sumed cyclic vomiting, but they were
perspectives in neurology. Continuum not either. You ordered an EEG after
Lifelong Learn Neurol. 2009;15(6):203–6. the family notices gaze deviation and
speech arrest around the most recent
?? 77. A family of a 3-year-­old girl brought a event and shows multifocal spikes,
video of one of their daughter recur- with greatest prevalence in the right
rent episodes of loss of consciousness occipital region.
which occurred in the last 2 months. What is your most likely diagnosis?
She was playing with a doll and her A. Panayiotopoulos syndrome
sister took the doll from her. She B. Reflex epilepsy
became angry and her face turns red C. Rolandic epilepsy
and then cyanotic. She fell to her side D. Subacute sclerosing
and has several jerking movements. ­panencephalitis
One minute later, she is alert and E. Temporal lobe epilepsy
walking normally. She is otherwise
well and has normal developmental vv Correct Answer is: A
milestones and a normal examination. It is very common to see occipital
Which of the following is the most spikes in Panayiotopoulos Syndrome
likely diagnosis? (early onset childhood benign occipital
A. Absence seizure epilepsy). 80% of cases are associated
B. Breath holding spell with ictal vomiting. Nausea or retching
C. Cataplexy may occur during or before the vomit-
D. Psychogenic nonepileptic event ing. Pallor is the most frequent auto-
E. Temporal lobe seizure nomic manifestation. The autonomic
286 Chapter 9 · Epilepsy 2

manifestations can be quite prolonged vv Correct Answer is: B


at times, and it might get confused with Parents are able and have the right to
the cyclic vomiting. This autonomic make most treatment decisions for their
status epilepticus can require use of children. But in this situation, the par-
rescue benzodiazepines. Although gen- ents are not enforcing a medically nec-
eralized convulsions are rare with this essary and clearly beneficial treatment
syndrome, often the initial autonomic with a favorable benefit-to-risk ratio
phase can be followed by gaze devia- and are acting counter to the child’s
tion, speech arrest and confusion. As best interests. An ethics consultation
with many of the benign epilepsies of is helpful in situations that cannot be
childhood, the spontaneous remission resolved with more communication and
rate is quite high with this disorder. education. Parental non-compliance is
often due to ignorance, anxiety or other
zz Suggested Reading stressors. An initial approach of deter-
55 Panayiotopoulos CP. Benign childhood mining the parent’s knowledge and per-
epileptic syndromes with occipital spikes: spective will increase the possibility of a
new classification proposed by the resolution.
International League Against Epilepsy.
J Child Neurol. 2000;15:548–52. zz Suggested Reading
55 Wirrell E, Nickels K. “Pediatric Epilepsy 55 Menahem S, Halasz G. Parental non-
9 Syndromes,” Continuum Lifelong Learn compliance–a paediatric dilemma. A
Neurol. 2010;16(3):57–85. medical and psychodynamic perspective.
Child Care Health Dev. 2000;26(1):61–72.
?? 79. A 12-year-old boy with epilepsy is
having monthly seizures at school. ?? 80. A 13-month-old female started to
On several occasions, his anti-seizure have her febrile seizures at 8 months
medication level has been very low or of age. They are generalized convul-
undetectable. While in the hospital, sions lasting 2 min and occasion-
on the same dose, his level is within ally precede her fever by several
a therapeutic range. His parents have hours. She has now had five, the last
taken him to a faith healer. of which was 2 days ago. She has
Which of the following is the best ini- normal development and physical
tial approach in communicating with examination. You are preparing to
the parents? print out her discharge paperwork,
A. Ask them to stay outside the room when she states she is not leaving
while you speak to the patient in without some testing to find out
private why her daughter is having seizures.
B. Determine what the parents know You are already 1 h behind schedule
about epilepsy and what their with three patient’s waiting to be
concerns are seen.
C. Inform them that child protective What is the best way to address the
services will be notified mother’s concern?
D. Set up a conference call and A. Begin treatment with levetirace-
include the faith healer, school tam to minimize any future sei-
nurse and counselor zures
E. Tell them you will no longer be B. Demand that she leaves the office
able to care for the patient as he is immediately, other testing in this
at risk for SUDEP case is a waste of resources
Epilepsy 2
287 9
C. Perform an EEG to prove she arms and legs for a few seconds. After
does not have a tendency toward about a minute she started to wake
unprovoked seizures up and was able to stand and walk
D. Refer mom for counseling to within 5 min. She does not have a sig-
address her obvious anxiety disor- nificant past medical history except
der she feels lightheaded upon standing
E. Suggest that a follow up appoint- several times every day. She takes no
ment can be scheduled to talk medications. She has a family history
some more about mom’s concerns of first degree relatives with both
migraine and seizures. Her neurologic
vv Correct Answer is: E examination is normal.
A good prognosis is the norm for simple Which of the following is the best next
febrile seizures and they have less than step in her management?
5% incidence of epilepsy even in chil- A. bedside orthostatic heart rate and
dren with multiple episodes. Febrile blood pressure measurement
seizures are the most common seizure B. perform electroencephalogram
disorder in childhood and on average C. prolonged cardiac event
affect 2% to 5% of children between the ­monitoring
ages of 6 and 60 months. Simple febrile D. serum glucose
seizures are defined as brief (<15-­min) E. urine drug testing
generalized seizures that occur once
during a 24-h period in a febrile child vv Correct Answer is: A
and absence of preexisting neurologic This is a presyncope/syncope case.
abnormality. Despite they are very The myoclonic jerks which often occur
benign, seizures of any type or cause during these episodes can mistakenly
can cause a lot of fear and anxiousness interpreted as seizures. A careful history
in any parent. It is important to find out is important to order further testing and
how to try and gently alleviate these treatment. The most important clues
concerns without ordering unnecessary are the sequence of symptoms and pre-
tests or placing a child on medication syncopal symptoms provide by stand-
that they do not need. If family reas- ing up. The patient describes several
sureness is not successful, a polite refer- core features consistent with neurally-­
ral to a colleague for another opinion, or mediated syncope including an aura of
offering assistance from a social worker visual dimming and hearing loss with
or counselor could be considered, but systemic ­pallor. The patient’s frequent
again, typically with better communica- complaints of near syncope on standing
tion strategies, this challenge can be should prompt careful assessment of
resolved in the office. orthostatic tolerance (or intolerance) and
bedside measurement of blood pressure
?? 81. A 14-year-old female has been evalu- and heart rate should be checked in the
ated of possible seizures. She has had supine, seated and standing positions.
two episodes in the last month. She There are many reasons for syncope,
will feel dizzy, then her vision will but the most common is the vasovagal
fade in both eyes, she cannot hear, (also referred to as neurally mediated)
then she passes out and falls to the syncope. The precise pathophysiology
ground. One event was witness by the remains incompletely understood, but
mother, who said that her daughter it is the most frequent cause of unex-
was pale and there was jerking of her plained transient loss of consciousness.
288 Chapter 9 · Epilepsy 2

This response can be triggered centrally ?? 82. A 3-year-old boy is admitted to the
(fear or anger), by other peripheral ED when he became lethargic and
stimuli affecting mechanical and/or che- fever of 39.4 °C. In the ED he has a
moreceptors (wearing of a stiff collared generalized convulsive activity which
shirt, micturition, defecation and cough- does not stop spontaneously. He is a
ing), or by some “hemodynamic stressor” healthy child with no history of sei-
(abrupt standing, prolonged standing or zures. He met all of his developmental
relative dehydration). Abnormal motor milestones and he is not taking any
movements with sudden and transient medications. There is a history of sei-
cerebral anoxia are commonly seen in zures in a distant cousin. He has been
syncope. Most are myoclonic in nature, sick for the last 72 h with vomiting,
but they can be tonic posturing or clonic cough, rhinorrhea and intermittent
activity. This so-called “convulsive syn- low-grade fevers. He received 2 doses
cope”, seen in 40–90% of all syncopal of lorazepam (0.1 mg/kg each) and
events, contributing to the confusion of fosphenytoin (20 mg/kg), but the con-
whether a true epileptic event occurred. vulsive activity have not stopped and
When the history is consistent with has been continued for 40 min. Beside
syncope and despite these movements, all anticonvulsant medications to stop
EEG is not necessary. Syncope due to the seizure which of the following
cardiac disease can arise either from an tests should be considered immedi-
9 arrhythmia or some type of obstructive ately in the care of this patient?
heart lesion (aortic stenosis). Red flags A. electroencephalogram
suggesting a potential cardiac etiol- B. lumbar puncture
ogy include lack of any aura, exertional C. MRI of the brain
syncope or a family history of sudden D. serum glucose
cardiac death or arrhythmia. 12-lead EKG E. urine organic acid measurement
is recommended, though other more
detailed cardiac monitoring should be vv Correct Answer is: D
saved for atypical cases or patients with Status epilepticus (established) defined
the above red flags. Other causes for as seizure of 30 min or more, or two or
syncope are variations in blood glucose more sequential seizures without full
regulation, toxic ingestion, iatrogenic recovery of consciousness inbetween.
(medication), primary autonomic fail- The incidence in children has ranged
ure (neurodegenerative disorders) and from 10–58 / 100,000 per year ages
brainstem/ventricular masses (colloid 1–19 years, with the highest in under
cysts). It would be unlikely that a spot 1 year of age. Status epilepticus is a com-
serum glucose would be diagnostic in mon occurrence in children with epilepsy
this patient during the office visit when and can also be the presenting manifes-
not experiencing symptoms, though tation of epilepsy; although increased
a sample taken immediately during or recognition of nonconvulsive status epi-
after an event would not be unreason- lepticus, especially in the intensive care
able. A urine drug screen is not indicated setting, has developed in both pediatric
as a first step in straightforward syncope. and adult age patients. Status epilep-
ticus can be divided into acute symp-
zz Suggested Reading tomatic cases (CNS infection, electrolyte
55 Drazkowski JF, Chung SS. Differential disturbance, sepsis, hypoxia, trauma or
diagnosis of epilepsy. Continuum Lifelong intoxication), remote symptomatic cases
Learn Neurol. 2010;16(3):36–56. (cortical malformation, prior traumatic
Epilepsy 2
289 9
brain injury or insult, chromosomal zz Suggested Reading
disorders, inborn errors of metabolism) 55 Riviello JJ, Ashwal S, Hirtz D, et al.
and idiopathic febrile status epilepticus. Practice parameter: Diagnostic assessment
Diagnose of idiopathic febrile status of the child with status epilepticus (an
epilepticus is challenging because no evidence-based review): Report of the
tests to confirm. Further testing must Quality Standards Subcommittee of the
be done to exclude a direct infection American Academy of Neurology and the
of the CNS. Although a LP should be Practice Committee of the Child
deferred until an imaging study has Neurology Society. Neurology.
excluded a space occupying lesion with 2006;67:1542–50.
local mass effect. Antibiotics must be
started empirically. Performing a lum- ?? 83. An 18-year-old man with a past medi-
bar puncture in children with status cal history of depression and post-
epilepticus without fever varies based traumatic stress disorder came to the
on the clinical scenario and may be ED with his roommate. He witnesses
performed at the discretion of the treat- multiple episodes of screaming and
ing physician. Identification of readily thrashing movements during the
reversible acute symptomatic causes of night. While in the emergency room
status epilepticus such as hypoglycemia he appears to be asleep and is wit-
or electrolyte imbalance should always nessed to have an event consisting of
be a priority as there is a prompt treat- yelling and moving all extremities in
ment that may abort the event. Serum a desynchronized fashion. The move-
glucose in particular should be checked ments last 40 s; then he stops moving
early in the course of a patient’s seizure. suddenly and is fully awake and ori-
Brain MRI and screening tests for inborn ented. His examination, EEG and MRI
errors of metabolism are indicated when are normal. Prolactin level measured
etiology cannot be established with 10 min after the event is normal.
initial testing. Serum anticonvulsant What do you recommend as a next
levels are strongly suggested in patients step?
who are taking anticonvulsant medica- A. diagnose him with psychogenic
tion. Serum and urine toxicology panels nonepileptic seizures
should be considered when no clear B. prescribe clonazepam
cause is identified by the history as up to C. recommend a­ dmission to an inpa-
3.6% of cases of pediatric status epilep- tient psychiatric unit
ticus are associated with a toxic inges- D. refer for inpatient video-EEG
tion. EEG has multiple dications in the monitoring
pediatric patient with status epilepticus E. refer for polysomnogram
including exclusion of nonconvulsive
status in the recovery period from the vv Correct Answer is: D
convulsive seizure, appropriateness of This patient may have frontal lobe
response to more aggressive IV infusions seizures, which occur in approximately
for refractory status epilepticus, exclu- 30% of patients with focal epilepsy.
sion of pseudo status epilepticus and They are often mistaken as psycho-
interictally to determine if there are focal genic nonepileptic seizures or para-
or generalized abnormalities that may somnias due to the bizarre clinical
influence other diagnostic or treatment semiology. They are often hypermotor,
decisions. Assessment of glucose level is brief and without postictal confusion.
more important than EEG in this patient. A normal prolactin level does not
290 Chapter 9 · Epilepsy 2

rule out an epileptic seizure as it has Which of the following treatment


a sensitivity of approximately 46% in approaches may foster the best long-
complex partial seizures and 60% in term compliance in this situation?
generalized tonic-clonic seizures and A. refer him to a psychiatrist for a
is possibly less sensitive in frontal lobe possible mood disorder
seizures. Inpatient video-EEG monitor- B. speak with him alone, make sure
ing is helpful in diagnosis. The EEG may he is not having any side effects
be obscured by movement artifact but of the medication and discuss the
the clinical semiology can be recorded long-term risks and benefits of
to allow for assessment of stereotyped staying on medication for seizures,
events, which can be helpful in diag- including safety with driving
nosis. C. tell him he will never outgrow his
tendency for seizures and he bet-
zz Suggested Reading ter get used to taking medication
55 Rudzinski LA, Shih JJ. The classification of D. tell him it’s OK to stop his medica-
seizures and epilepsy syndromes. tion but if he has a seizure he won’t
Continuum Lifelong Learn Neurol. be able to drive for 6 months
2010;16(3):15–35. E. tell his mother he is not yet of
55 Chen DK, So YT, Fisher, RS. Use of serum adult age and she needs to make
prolactin in diagnosing epileptic seizures. him take his medication
9 Report of the Therapeutics and
Technology Assessment Subcommittee of vv Correct Answer is: B
the American Academy of Neurology. Management of epilepsy can be compli-
Neurology. 2005;65:668–75 cated enough without adding medica-
tion noncompliance into play. Teenage
?? 84. A 16-year-old boy in the office with patients in particular are often unhappy
his mother for follow-­up visit for that they require medication on a daily
his juvenile myoclonic epilepsy. He basis to “be normal.” They feel impen-
is carrying this diagnosis since he etrable and vulnerable at the same time
was 14 after his first generalized and despite physical maturity sometimes
convulsive seizure. He still has a few do not have the cognitive or emotional
clusters of myoclonic seizures in the maturity to take care of their own medi-
early morning hours but no seizure cal needs. The treating physician’s duty
of any type for the past 8 months. He is to ensure the best interests of the
is otherwise healthy without medical patient are being met regardless of
complications. He is taken only leve- emotional, cognitive or age-dependent
tiracetam, 750 mg twice daily. His limitations that may arise. Although a
prior EEG’s have shown generalized teenage patient may not be completely
4–6 Hz spike and polyspike and slow independent from a medical decision-
waves with some photosensitivity. making standpoint, it is essential that
He often forgets to take his medica- they understand the implications of their
tion, but he has not had any seizures medical condition on their life and that
and told his mother in the morning they feel like they have an important
this must mean he does not need voice in the ongoing management of
medication any more. In the office their condition. In the above scenario a
he and his mother begin a loud ver- careful assessment of the patient’s men-
bal disagreement and he leaves the tal state, including how much insight
room. the patient has of the possible risks of
Epilepsy 2
291 9
medication noncompliance, is essential. C. frequency of 6–7 Hz
There may be a very obvious reason the D. maximal over the occipital region
patient is not taking the medication as E. most commonly observed in
directed, such as untoward side effects. stage II sleep
Having the patient “buy in” to the impor-
tance of medication compliance is always vv Correct Answer is: A
ideal, rather than forcing them to take The mu waveform occurs in the central
the medication. If it becomes apparent, regions in an awake patient and not
despite a constructive conversation, that in drowsiness, which can be markedly
compliance will continue to be an issue, asymmetrical. It is usually in the alpha
safety measures especially regarding frequency range, with rounded posi-
driving must be put into place, and the tive aspects on one side and sharpened
patient will have to understand these negative aspects on the other. It is not
potential consequences of failing to take attenuated with eye opening, but will
the medication as needed. Referral to an resolve with movement of the contra-
appropriate counselor may be of some lateral limb or even with the thought of
benefit especially if there is still an issue such movement.
adjusting to the diagnosis of a chronic
disease. zz Suggested Reading
55 Fisch B. Fisch and Spehlmann’s EEG
zz Suggested Reading Primer. 3rd ed. Amsterdam: Elsevier; 1999.
55 Rodgers-Neame NT, Orlowski JP. Ethical 55 Niedermeyer E, Lopes da Silva
perspectives in neurology. Continuum. F. Electroencephalography: basic prin-
2009;15(6):203–6. ciples, clinical applications, and related
fields. 5th ed. Baltimore: Willliams &
?? 85. A posterior dominant alpha rhythm of Wilkins; 2005.
9 Hz is usually reached by what age?
A. 10 ?? 87. After gently scrubbing EEG scalp
B. 12 electrodes, used on an HIV-positive
C. 14 patient in which there is no contact
D. 16 with blood, which of the following
E. 18 is the most appropriate cleaning
method?
vv Correct Answer is: A A. air dry
By the age of 9, the mean alpha fre- B. soak in distilled water
quency is 9 Hz in 65% of controls. C. soak in hydrogen peroxide
D. soak in tap water
zz Suggested Reading E. soak in tenofovir
55 Ebersole JS, Pedley TA, editors. Current
practice of clinical electroencephalogra- vv Correct Answer is: C
phy. 3rd ed. Philadelphia: Lippincott, There are no reported cases of HIV
Williams, & Wilkins; 2003. p. 102–3. transmission via scalp EEG electrodes.
Universal precautions are the standard
?? 86. Which of the following characteristics of care for sterilization of EEG elec-
of the mu rhythm is most accurate? trodes after use in patients who are
A. attenuated with contralateral HIV-positive, HIV-negative, or in whom
hand movement HIV status is unknown. Tap or distilled
B. attenuated with eye opening water is inadequate for disinfection
292 Chapter 9 · Epilepsy 2

purposes and antiretroviral medica- vv Correct Answer is: E


tion is unnecessary, according to the Hyperventilation is most often used to
Committee on Infectious Diseases. activate the 3-per-second, spike-and-
Electrodes should be soaked in a non- wave discharge of primary generalized
corrosive disinfectant solution such as epilepsy. In some patients, discharges
hydrogen peroxide, glutaraldehyde, are only seen during hyperventilation.
sodium hypochlorite, or a combination The patient is asked to mouth-breathe
of isopropanol, 2-butoxyethanol, and deeply for 3 min; however, if there
benzethonium chloride. is suspicion of absence seizures, the
patient should hyperventilate for 5 min.
zz Suggested Reading Generalized slowing of the background
55 Report of the Committee on Infectious activity in the theta range is the normal
Diseases. J Clin Neurophysiol. response to hyperventilation. Absence
1994;11(1):128–32. of slowing is not abnormal and depends
55 Beltrami EM, Williams IT, Shapiro CN, on effort, age, and time of hyperven-
Chamberland ME. Risk and management tilation. Hypoglycemia may increase
of blood-borne infections in health care slowing. Because hyperventilation may
workers. Clin Microbiol Rev. 2000;13: cause hypocapnia and alkalosis, vaso-
385–407. spasm and impaired cerebral perfusion
55 Kelen GD. Human immunodeficiency may occur; therefore, hyperventilation
9 virus and the emergency department: risks should not be performed in patients
and risk protection for health care provid- with cerebrovascular disease or intracra-
ers. Ann Emerg Med. 1990;19:242–8. nial hemorrhage.
55 Center for Disease Control
Recommendations for Prevention of HIV zz Suggested Reading
Transmission in Healthcare Settings. 55 Misulis KE, Head TC. Essentials of clinical
Published at: 7 www.cdc.gov/mmwr/pre- neurophysiology. 3rd ed. Philadelphia:
view/mmwrhtml/00023587 [accessed Butterworth-­Heinemann; 2003.
4-10-13].
?? 89. An 18-year-old male is unconscious
?? 88. A 4-year-old girl might have absence and unresponsive to painful stimuli
seizures because of multiple daily epi- upon admission to the hospital after
sodes of staring and decreased level he had sustained head trauma while
of attention. 1-min hyperventilation downhill skiing. On examination,
was done during her EEG which was there is no pupillary responses, cor-
normal. neal reflexes, oculocephalic reflexes,
Which of the following facts about gag reflex, and spontaneous breath-
hyperventilation is most accurate? ing. He has an EEG 4 h after admission
A. considered a safe activating pro- as part of the clinical assessment for
cedure for individuals of all age brain death, shows no detectable
groups electrical activity.
B. 1 min is an adequate amount of What additional information is
time and should capture abnor- required before concluding that the
mal activity EEG is consistent with brain death?
C. requires rapid shallow breathing A. core temperature greater than 32 °C
D. should be performed for 15 min B. electrocardiogram evidence of
E. should be performed for 5 min asystole
Epilepsy 2
293 9
C. electrocardiogram evidence of zz Suggested Reading
bradycardia 55 Torres F, Anderson C. The normal EEG in
D. systolic blood pressure less than the human newborn. J Clin Neurophysiol.
80 mm Hg 1985;2(2):89–103.
E. the patient has an advance
­directive ?? 91. An 18-year-old female with a history
of sexual abuse is suspected of hav-
vv Correct Answer is: A ing psychogenic seizures. Long-term
Exclusion of hypothermia (a core tem- video EEG monitoring is performed,
perature greater than 32 °C) is a condi- during which she exhibits hip thrust-
tion of the diagnosis of brain death ing and side-to-­side flailing move-
by neurologic exam, and diagnosis ments, which are not followed by a
of electrocerebral silence by EEG. The postictal period. She has a normal
clinician should consider hypother- neurologic exam.
mia in this case, especially since the Which of the following findings on
patient was in a cold outdoor environ- EEG is considered a normal variant
ment hour before the EEG. Whether an on EEG?
advance directive exists is irrelevant A. lambda waves
to the clinical diagnosis, although it B. parietal sharp waves
would direct clinical decision making C. periodic lateralized epileptiform
following the diagnosis. Cardiac arrest discharges
(asystole), bradycardia, or hypoten- D. rolandic spikes
sion is not necessary for a diagnosis of E. temporal sharp waves
brain death.
vv Correct Answer is: A
zz Suggested Reading Lambda waves are a normal find-
55 Practice Parameters for determining brain ing on EEG and are seen on a waking
death: (Summary Statement). Report of record, when the subject is viewing a
the Quality Standards Subcommittee of scene. These positive occipital waves
the American Academy of Neurology. are blocked by eye closure. Parietal
Neurology. 1995;45:1012–4. sharp waves are often associated with
versive head and eye movements
?? 90. Spindle delta brushes are most com- or sensory seizures. Temporal sharp
monly seen in infants of what age? waves are often associated with partial
A. premature complex and generalized tonic clonic
B. full-term infants seizures. Psychological complaints
C. 2-month-old infants may be seen in individuals with tem-
D. 4-month-old infants poral sharp waves. Although rolandic
E. between full-term birth and spikes may be seen in subjects without
1 month old seizures, they are also seen in benign
rolandic epilepsy. Periodic lateralized
vv Correct Answer is: A epileptiform discharges may be either
Delta brushes are present from unilateral or bilateral and are due to
29 weeks of conceptional age to any destructive process. Common
1-month post-term. Their peak expres- causes of PLEDs include anoxia, her-
sion is at 32–33 weeks of conceptional pes simplex encephalitis, stroke, and
age. tumor.
294 Chapter 9 · Epilepsy 2

zz Suggested Reading C. anterior


55 Misulis KE, Head CH. Essentials of clinical D. central
neurophysiology. 3rd ed. Philadelphia: E. posterior
Butterworth-­Heinemann; 2003.
vv Correct Answer is: E
?? 92. A 16-year-old female came for an evalua- Compared to older children and adoles-
tion of spells characterized by thrashing cents, hyperventilation-induced slow
movements of the arms, vocalizations, activity in young children is usually
and rocking movements of the trunk maximal over the posterior head region.
and body. Each spell lasts about 30 s. She
does not respond during the spell, but zz Suggested Reading
appropriately answers questions within 55 Daube J. Clinical neurophysiology. 2nd ed.
10 s after movement cessation. Over Oxford: Oxford University Press; 2002.
50% of the spells occur in sleep.
What is your diagnosis? ?? 94. A 17-year-old boy complaining
A. frontal lobe seizures of spells characterized by a warm
B. night terrors ascending sensation in the epigas-
C. parietal lobe seizures trium, associated with a sensation of
D. psychogenic nonepileptic events fear, followed by loss of contact with
(psychogenic seizures) the environment and oromandibular
9 E. temporal lobe seizures automatisms. During a video EEG, his
left hand continually rubs his leg and
vv Correct Answer is: A his right arm remains immobile and
Frontal lobe onset seizures are charac- stiff. He has difficulty following com-
terized by an abrupt onset of stereo- mands during the seizure.
typed hypermotor behavior and may Where is the location of the ictal
include vocalizations, gestural or sexual ­discharge?
automatisms, and bilateral leg automa- A. left lateral temporal lobe
tisms consisting of pedaling or bicycling B. left mesial frontal lobe
movements. Frontal lobe seizures often C. left mesial temporal lobe
arise in sleep. In contrast, temporal lobe D. left perirolandic region
seizures are characterized by behavioral E. right lateral temporal lobe
arrest and hypomotoric movements.
vv Correct Answer is: C
zz Suggested Reading Mesial temporal lobe epilepsy is one
55 Jobst BC, Siegel AM, Thadani VM, et al. of the most common types of epilepsy
Intractable seizures of frontal lobe origin: referred for epilepsy surgery and is
clinical characteristics, localizing signs, often refractory to antiepileptic drugs.
and results of surgery. Epilepsia. The age of onset is between late child-
2000;41(9):1139–52. hoods to mid adolescence. Patients
often had febrile convulsions in infancy
?? 93. Compared to older children and ado- or early childhood. Most patients report
lescents, hyperventilation in young chil- an aura. Common auras include an epi-
dren tends to show maximal slowing in gastric sensation (a rising sensation,
which of the following head regions? butterflies, and nausea), fear, olfactory
A. all head regions hallucinations, lightheadedness, and
B. alternates between the anterior déjà vu. Complex partial seizure semiol-
and central head regions ogy may also consist of ipsilateral upper
Epilepsy 2
295 9
extremity automatisms and ipsilateral seizures can also manifest after awaken-
early non-forced head turn. Contralat- ing and are triggered by sleep depriva-
eral dystonic posturing, Todd paralysis, tion, alcohol, stress, and fatigue. The EEG
and late forced head turn prior to sec- recording is consistent with generalized
ondary generalization can also be seen. 10–16 Hz polyspike discharges, fol-
lowed by slow waves.In Lennox-Gastaut
?? 95. A 16-year-old male has a witnessed syndrome, EEG shows slow 1.5 to 2.5
generalized tonic seizure early in generalized spike-­and-­wave discharges,
the morning by his mother. His past predominantly in the frontal regions on
medical history is unremarkable. He an abnormal slow background. In benign
just came back from a trip to Europe childhood epilepsy with centrotemporal
and reports still feeling jet lagged, spikes, EEG shows biphasic high-voltage
but otherwise well. He has a normal central temporal spikes, followed by
neurologic exam. Vital signs, labs, and slow waves and a normal background
CT of the head are normal. There is no activity. Are patterns seen in childhood
history of head injury, drug, or alcohol absence and juvenile absence epilepsy,
abuse. He noticed that he is occasion- EEG shows generalized 3- and 4.5–5 Hz
ally clumsy, especially in the morning. spike-and-wave discharges with normal
He spills his drink or drops items from background activity respectively.
his hands. Jerking movements of the
head, neck, and upper extremities zz Suggested Reading
have also been noticed. 55 Dodson WE, Pellock JM. Pediatric
What are you going to see on his EEG? epilepsy: diagnosis and therapy. New York:
A. 1.5–2.5 generalized spike-and-wave Demos; 1993. p.171–81.
discharges predominantly in the 55 Fenichel GM. Clinical pediatric neurology.
frontal regions, superimposed on an 3rd ed. Philadelphia: WB Saunders; 1997.
abnormal slow background activity p. 28–31.
B. frequent centrotemporal spikes 55 Schmitz B, Saunder T, editors. Juvenile
C. generalized 10–16 Hz polyspike myoclonic epilepsy: the Janz syndrome.
discharges followed by slow Wrightson Biomedical; 2000.
waves 55 Wyllie, E. The Treatment of epilepsy:
D. generalized 4–5 Hz spike-and- principles and practice. Philadelphia: Lea
wave discharges and Febiger; 1993. p. 552–83.
E. regular symmetrical 3 Hz spike- 55 De Vivo DC. Early onset epilepsy.
and-wave discharges with a nor- Continuum Lifelong Learn Neurol.
mal background 2000;6:94–113.
55 Menkes JH, Sarnat HB. Paroxysmal
vv Correct Answer is: C disorders. In: Child neurology. 6th ed.
The clinical scenario is suggestive of Philadelphia: Lippincott Williams &
juvenile myoclonic epilepsy (JME), which Wilkins; 2000. p. 919–1026.
accounts for 10% of all epilepsies. The
normal neurologic exam and myoclonic ?? 96. A 14-year-old boy presents with a his-
movements in the morning are important tory of perinatal asphyxia. His seizures
findings. JME is an autosomal dominant are drop attacks, staring spells, myo-
trait, characterized by myoclonic tonic- clonic jerking and generalized tonic-
clonic seizures that tend to occur in the clonic seizures. His IQ is 62. His EEG
morning in individuals between 12 and shows 1.5–2.0 Hz generalized spike-
18 years old. Generalized tonic-clonic wave discharges as a baseline pattern.
296 Chapter 9 · Epilepsy 2

What is the most likely diagnosis? antiepileptic medications are the treat-
A. Baltic myoclonus ment of choice. Narrow spectrum antiepi-
B. Lennox-Gastaut syndrome leptic medications such as carbamazepine
C. myoclonic astatic epilepsy and gabapentin can exacerbate myo-
D. Ohtahara syndrome clonic seizures, as can phenytoin. Valproic
E. West syndrome acid should be avoided in women of child-
bearing age because of its potential for
vv Correct Answer is: B teratogenicity. Zonisamide is considered a
Lennox-Gastaut syndrome (LGS) is second-line agent for treating myoclonic
one of the epileptic encephalopathies. and generalized tonic-clonic seizures.
Usually, it starts before 8 years of age,
with a peak age of onset between 3 zz Suggested Reading
and 5 years of age. Rarely, the disorder 55 Auvin S. Treatment of juvenile myoclonic
can present in early adulthood. The epilepsy. CNS Neurosci Ther.
syndrome is characterized by a triad of 2008;14(3):227–33.
multiple seizure types (tonic and atypi-
cal absence are the most common), slow ?? 98. Photic stimulation (PS) is one of the
spike-­and-­wave on EEG (1–2.5 Hz), and activation methods used to enhance
some degree of mental retardation. EEG changes, both normal and abnor-
mal. PS is essentially done to evaluate
9 zz Suggested Reading responses that can occur in photosen-
55 Rudzinski LA, Shih JJ. The classification of sitive seizures.
seizures and epilepsy syndromes. Which of the following factors is most
Continuum Lifelong Learn Neurol. accurate regarding PS?
2010;16(3):15–35. A. normal results of PS include signif-
icantly asymmetric photic driving
?? 97. A 16-year-old female has a 3-year responses
history of seizures, occur in the early B. occurrence of photoparoxysmal
morning and characterized by rapid response in a seizure-free indi-
jerks of her limbs and trunk. She has vidual is common
occasional generalized tonic-clonic C. photomyogenic response can be
seizures, which usually occur in seen in hypercalcemia
the setting of sleep deprivation, or D. photomyogenic response is an
increased stress. She was initially on epileptic response to PS
lamotrigine and then levetiracetam, E. photoparoxysmal response con-
but could not tolerate either drug. sists of generalized polyspike-
What is the most appropriate next wave alterations
treatment option?
A. carbamazepine vv Correct Answer is: E
B. gabapentin Normal results of PS include symmet-
C. phenytoin ric photic driving. The photomyogenic
D. valproic acid response is an exaggerated nonepileptic
E. zonisamide motor response to PS and can be enhanced
in hypocalcemia and withdrawal from
vv Correct Answer is: E sedatives. A photoparoxysmal response is
The clinical presentation of juvenile frequently associated with juvenile myo-
myoclonic epilepsy, which is a primary clonic epilepsy, and it is a rare finding in a
generalized epilepsy. Broad spectrum seizure-free individual (less than 2%).
Epilepsy 2
297 9
zz Suggested Reading ?? 100. A 6-month-old baby girl is seen in
55 Schomer D, Lopes da Silva, editors. the emergency room due to a sei-
Niedermeyer’s electroencephalography: zure. The seizure involved left arm
basic principles, clinical applications, and shaking and then generalized. It
related fields. Philadelphia: lasted 15 min. Her birth and delivery
Lippincott Williams & Wilkins; 2011. history are unremarkable. Her devel-
p. 221–2 opmental milestones were on time.
Her father had febrile seizures as a
?? 99. Activation methods are used to child. On examination, she smiles
enhance EEG changes, both normal and coos, but does not reach with
and abnormal. Which of the following her left hand. She has signs of a left
statements is most accurate regard- otitis media and is febrile.
ing hyperventilation (HV) activation Which of the following is consistent
procedure? with a simple febrile seizure in this
A. buildup of slow waves after the patient?
end of HV (rebuildup) may be a A. Age of the patient
diagnostic finding in moyamoya B. Family history
disease C. Length of seizure
B. HV is rarely achieved in young D. Neurologic examination
children E. Seizure semiology
C. HV procedure is indicated in
patients with acute stroke vv Correct Answer is: A
D. HV-induced slowing is always Febrile seizures are extremely common
abnormal in adults in the young child, with an age range
E. HV-induced slowing is the result of onset typically listed as 3 months to
of increased cerebral blood flow 6 years of age. Simple febrile seizures
are characterized by seizures accom-
vv Correct Answer is: A panied by fever that are generalized in
Buildup of slow waves after the end onset, last less than 15 min, and do not
of HV (rebuildup) may be a diagnostic occur more than once in 24 h. Febrile sei-
finding in moyamoya disease. Sickle cell zures are complex due to either longer
disease and acute stroke are contrain- duration, focal signs at onset, or recur-
dications to HV. HV-induced slowing is rence within 24 h. Normal achievement
the result of hypocapnic cerebral vaso- of neurodevelopmental milestones
constriction with reduction of CBF. HV- and a normal neurologic exam are also
induced slowing is normal unless it is typically required to classify the event
significantly asymmetric or contains as a febrile seizure. Prominent encepha-
definite spikes. In young children, HV can lopathy prior to or after the seizure
be achieved by asking the child to blow should raise suspicion of a secondary
on a pinwheel. cause such as a central nervous system
infection and focal onset seizures with
zz Suggested Reading post-ictal hemiparesis may suggest a
55 Schomer D, Lopes da Silva FH, editors. structural cause such as tumor, stroke or
Niedermeyer’s electroencephalography: abscess. Although the genetics of febrile
basic principles, clinical applications, and seizures is still not well known (other
related fields. Philadelphia: than for syndromes such as generalized
Lippincott Williams & Wilkins; 2011. epilepsy febrile seizures plus- GEFS+),
p. 216–217. a family history of such events is quite
298 Chapter 9 · Epilepsy 2

commonly obtained, though not effec- essential for screening. If normal, the
tive in differentiating between simple patient should have exercise testing
and complex febrile seizures. or be referred to pediatric cardiology,
as occasionally the initial resting EKG
zz Suggested Reading can be normal. Most patients with
55 Waruiru C, Appleton R. Febrile seizures: a congenital long QT syndrome have a
consensus of their significance, evaluation, normal cardiac exam on echocardiog-
and treatment. Consensus development raphy so this is not a high priority test
conference of febrile seizures. National in this population. A routine EEG may
Institute of Health. be warranted if the ECG is normal or the
Pediatrics.1980;66:1009–12. patient has recurrent episodes outside
55 Shinnar S, Glauser TA. Febrile Seizures. the setting of exercise. Typically, how-
J Child Neuro. 2002:17(1):s44–52. ever, convulsions associated with long
QT syndrome are due to anoxic seizures,
?? 101. A 10-year-old female has been eval- and would not be likely to reveal any
uated for loss of consciousness. She interictal abnormalities on EEG. Brain
was playing soccer for the first time MRI is not indicated for congenital long
this season, she has a sudden loss of QT syndrome but may be indicated
consciousness with witnessed con- if the patient is felt to have epilepsy
vulsions and urinary incontinence. rather than anoxic seizures. A tilt table
9 After a few minutes she returns to test may be helpful is cases neurocar-
her baseline of consciousness and diogenic syncope due to orthostatic
reports no recall of any warning hypotension. Those patients typically do
symptoms. She has a normal devel- not have syncope during active phases
opmental history. Her father had a of exercise.
seizure in his 20 s after diving into
cold water. A paternal uncle had zz Suggested Reading
childhood seizures and died while 55 McLeod KA. Syncope in childhood. Arch
swimming as a teenager. She has a Dis Child. 2003;88(4):350–3.
normal general medical and neuro-
logical examinations. ?? 102. A 16-year-old female with juvenile
Which of the following is the most myoclonic epilepsy is here for her
urgent diagnostic test? annual check up. She has been
A. 12-lead electrocardiogram seizure free for 1 year. She is on
B. Transthoracic echocardiogram lamotrigine.
C. Routine EEG What are the current guidelines
D. MRI brain in her case?
E. Tilt table test A. There is no need to discuss con-
traception and pregnancy until
vv Correct Answer is: A she is older.
This is a clinical presentation of B. Annual counseling of how epi-
exercise-­induced syncope with convul- lepsy and its treatment may
sions. Her family history is indicative of affect contraception or preg-
autosomal dominant pattern of convul- nancy is recommended.
sions possibly induced by cold water C. Before discussing contraception
immersion. This is most concerning for or pregnancy, you must obtain
a congenital long QT syndrome and the permission of a parent or
a 12-lead electrocardiogram (ECG) is legal guardian.
Epilepsy 2
299 9
D. Prophylactic use of an intra- exaggerated startle response with
uterine device is recommended muscle spasms involving the whole
given the interaction of oral con- body. They also reported that symp-
traceptives and lamotrigine. toms will continue to recur each time
E. Pregnancy test should be the stimulation is repeated. It became
done annually and prior to any very difficult for them to take care of
changes in antiepileptic drug him as he would become very stiff
therapy. every time they will pick him up for
change of diapers or to feed him. His
vv Correct Answer is: B muscle tone appears normal when
One of the most important issues in lying in bed by himself.
epilepsy is “Counseling for Women of What is the most likely diagnosis of
Childbearing Potential with Epilepsy”, this patient?
and it is one of the core quality mea- A. Serum anti-glutamic acid decar-
surables for caregivers of patients with boxylase (GAD) antibodies will
epilepsy. All female epilepsy patients of be positive.
childbearing potential (12 to 44 years B. EEG will show generalized 3 Hz
old) should be counseled or referred spike and wave activity.
for counseling for how epilepsy and its C. CSF will be positive for 14-3-3
treatment may affect contraception OR protein.
pregnancy at least once yearly. Post- D. Autosomal recessive inheritance
menopausal females, or those who are is most common.
surgically sterile are excluded. E. It involves mutation in the gene
Parental presence or permission for for alpha-1 subunit of the glycine
this discussion is not mandated, nor receptor.
pregnancy testing unless clinical concern
is present. Lamotrigine does not lessen vv Correct Answer is: E
the effectiveness of oral contraceptive This is a typical presentation of exagger-
agents (unlike topiramate), though ated startle disease or hyperekplexia.
concomitant use of oral contraceptives The event in hyperekplexia is markedly
can induce the metabolism of lamotrigine, exaggerated and does not habituate
resulting in lower serum levels. Typically, with repetition of the stimulus which
adjustments can be made in the dose of differentiates it from normal infantile
lamotrigine if this occurs, and would not startle response. The child will massive
necessitate another form of birth control, flexion of the neck (severe enough to
unless desired by the patient. at times compromise the airway of the
infant) and trunk with abduction and
zz Suggested Reading flexion of the arms. It can also result in
55 Fountain NB, Van Ness PC, Bennett A, tonic muscle spasms. Although these
Absher J, et al. Quality improvement in may resemble tonic seizures, the consis-
neurology: Epilepsy Update Quality tent ability to be provoked by a stimulus
Measurement Set. Neurology. 2015; is atypical for seizures. An EEG will show
84(14):1483–7. only movement artifact during the spells.
The condition most commonly is due to
?? 103. A 5-month-old boy has been evalu- mutations in the glycine receptor alpha1-
ated for a history of spasms. His subunit (GLRA1) gene and inherited as
family reports that he is very sensi- an autosomal dominant trait (though
tive to any noise and reacts with an autosomal recessive and x-linked inheri-
300 Chapter 9 · Epilepsy 2

tance has been reported.) Mutations E. Ask the patient to further discuss
typically result in deficient production his thoughts on antiepileptic
and activity of glycine (a major inhibi- drug treatment and the risks of
tory neurotransmitter) in the central not taking the medication
nervous system. Anti-GAD antibodies are
negative in this case, it is positive in “stiff- vv Correct Answer is: E
person syndrome.” CSF exam is typically Noncompliance is very common for dif-
normal in these patients (not consis- ferent reasons and the patient’s expla-
tently showing abnormal glycine levels). nation and understanding of the risks
Though the condition can improve over should be thoroughly explored by the
time, benzodiazepines, such as clonaz- physician. The risk of having a seizure is
epam, provide effective symptomatic very high, including while driving, and
treatment for this condition. should be advised not to drive until he
is compliant and the level is back to his
zz Suggested Reading therapeutic range. Honesty between the
55 Espay A, Chen R. Myoclonus. Continuum patient and the physician in a key prin-
Lifelong Learn Neurol. 2013;19(5):1264. ciple in the management of patient’s
55 Bode A, Lynch J. The impact of human with epilepsy and is the reason that
hyperekplexia mutations on glycine most states do not have a mandatory
receptor structure and function. Molecular reporting law to the DMV after a seizure.
9 Brain. 2014:7:2.
zz Suggested Reading
?? 104. An 18-year-old male has focal epi- 55 Bacon D, Fisher RS, Morris JC, Rizzo M,
lepsy due to an intrauterine stroke. Spanaki MV. American Academy of
He denies any seizures since his Neurology position statement on physi-
last visit 6 months ago. He takes cian reporting of medical conditions that
carbamazepine 400 mg bid, but may affect driving competence. Neurology.
read recently that it can lower tes- 2007;68(15): 1174–7.
tosterone levels and decided to take
it only once a day. His serum level is ?? 105. A 15-year-old male came to the
undetectable. He is currently driving. office for a second opinion. He was
What is the most appropriate action diagnosed with epilepsy 8 months
by you? ago and is taking carbamazepine.
A. Bring his parents into the exami- He has had 6 seizures. Seizures
nation room to discuss his risky occurred 1–2 h after waking from
behavior sleep, some early in the morning
B. Report the patient to the Depart- and some after awakening from a
ment of Motor Vehicles (DMV), midday nap. No loss of conscious-
so that his license can be tempo- ness and he did not have twitches
rarily suspended of the hands have been reported
C. Request the patient sign a form by the patient or family or friends.
that notifies him that he is at risk He has no family history of epilepsy.
of death and driving is not rec- He has a normal examination and
ommended MRI of the brain. EEG shows 3–4 Hz
D. Prescribe testosterone in addi- generalized spike wave discharges
tion to carbamazepine to and photosensitivity. There are rare
increase adherence to the treat- discharges with a left sided predom-
ment regimen inance.
Epilepsy 2
301 9
What is your most accurate diagnosis? zz Suggested Reading
A. Autosomal dominant nocturnal 55 Janz D. Epilepsy with grand mal on
frontal lobe epilepsy (ADNFLE) awakening and sleep-waking cycle. Clin
B. Epilepsy with grand mal seizures Neurophysiol. 2000;111(S2):S103–10.
on awakening 55 Rudzinski LA, Shih JJ. The Classification of
C. Left frontal lobe epilepsy Seizures and Epilepsy Syndromes.
D. Juvenile myoclonic epilepsy Continuum 2010;16(3): 15–35.
E. Progressive myoclonic epilepsy
?? 106. A 16-month-old boy has an unpro-
vv Correct Answer is: B voked generalized tonic clonic seizure
This is a clinical presentation of epilepsy (GTC). He has a history of 2 complex
with grand mal seizures upon awaken- febrile seizures, they were prolonged
ing which is an idiopathic generalized unilateral convulsions (one right and
epilepsy (IGE) syndrome characterized one left-sided). Recently his mother
by generalized tonic clonic seizures has noted some jerking movements
occurring predominantly on awakening of his arms. This jerking occurred
(independent of the time of day.) The EEG as increased clusters prior to onset
shows predominantly generalized spike of the GTC. Mom has also reported
and slow waves (though focal abnormali- increased falls. You admitted the
ties can also be noted) and photosensitiv- patient for evaluation and an EEG
ity. This syndrome shares many features and brain MRI were ordered, both of
with the more common juvenile myo- which are normal. Forty-eight hours
clonic epilepsy, including similar provok- later, he is awake, but has some mild
ing features (sleep deprivation, alcohol truncal ataxia on exam.
use and bright flickering lights), though What gene abnormalities would best
a formal diagnosis of JME would typically explain his condition?
require either clinical absence or myo- A. CACNA1A
clonic events in addition to generalized B. DMPK gene
tonic-clonic seizures. Treatment for both C. KCNQ2
disorders is a broad spectrum agents such D. SCN1A
as lamotrigine, levetiracetam, sodium E. TSC1
valproate, topiramate or zonisamide. Nar-
row spectrum sodium channel blocking vv Correct Answer is: D
agents such as carbamazepine would be Dravet syndrome is one of a genetic epilep-
relatively contraindicated. tic encephalopathy. In typical cases, unpro-
Though frontal lobe seizures can occur voked seizures begin in the first or second
during sleep-wake transitions, they most year of life in an otherwise healthy infant.
commonly occur out of sleep, and not 1–2 h However, recurrent spells of hemiconvul-
after awakening. Additionally, although sive febrile status epilepticus in the first
very rapidly generalizing discharges can be year of life may also herald the diagnosis.
present on EEG, photosensitivity is rare. Also known as severe myoclonic epilepsy
Without a suggestive family history, a of infancy (SMEI), this naming is not in
diagnosis of ADNFLE would be hard to favor because a lot of cases are being iden-
confidently make. The patient has not been tified without obvious myoclonic events.
identified to have myoclonic seizures, and Development may be normal at the onset
has not have the typical progressive of seizures, though most patients will have
cognitive decline seen with the disorders very obvious delays in motor, language
leading to a progressive myoclonic epilepsy. and cognitive development by 3–4 years
302 Chapter 9 · Epilepsy 2

of age. Progressive ataxia has seen in this vv Correct Answer is: E


syndrome. Up to 80–90% of cases meet- The patient may be better joined a struc-
ing clinical criteria of Dravet syndrome will tured clinical trial if he or she is interest
have a mutation identified in the SCN1A in unproven medications or alternative
gene region. treatments especially if a clinical trial is
Mutations in the CACNA1A region lead ongoing. There are many patients and
to multiple phenotypes including sporadic parents who use alternative and unproven
and familial hemiplegic migraine, episodic treatments. An open discussion of poten-
ataxia and spinocerebellar ataxia, type 6. tial risks and benefits is important in these
KCNQ2 mutations typically lead to benign circumstances. Use of cannabinoid oil
familial neonatal convulsions, though a as a treatment for medically intractable
more aggressive phenotype with a severe epilepsy is not “unethical”, though the
epileptic encephalopathy has been efficacy and side effect profile has as of yet
described (though without myoclonic not been well studies through randomized
seizures or prolonged hemiconvlusive clinical trials. Each year, additional states
febrile status). TSC1 mutations lead to are passing legislature making the prod-
tuberous sclerosis, unlikely given the uct legally obtainable if used for intrac-
normal MRI scan in this patient and DMPK table patients and grown within state
mutations are seen in myotonic dystrophy. boundaries. Even when obtained through
other methods, there is no requirement
9 zz Suggested Reading for the physician to report the patient or
55 Dravet C. The core Dravet syndrome family to law enforcement officials.
phenotype. Epilepsia. 2011;52(s2):3–9.
zz Suggested Reading
?? 107. A 9-year-old girl with Lennox- 55 Gloss DS, Maa EH. Medical marijuana:
Gastaut syndrome came to your between a plant and a hard place.
office for follow-up. She has been Neurology. 2015; 5(4):281–4.
having daily seizures despite treat-
ment with sodium valproate, cloba- ?? 108. A 6-year-old boy has seizures that are
zam and rufinamide. She would like associated with stroke-like episodes
to try cannabinoid oil but they don’t of cortical blindness, most likely has
live in a state where there is a legal which type of metabolic disorder?
prescription for the cannabinoid, it is A. Glycogen storage
available on the internet. B. Lysosomal
Which of the following statements is C. Mitochondrial
most ethically appropriate? D. Copper metaboli
A. Cannabinoid oil use is unethical
B. The physician is obligated to vv Correct Answer is: C
report the parents to the Drug Mitochondrial encephalopathy, lac-
Enforcement Agency tic acidosis, and stroke-like episodes
C. In order to legally use cannabi- (MELAS) is a multisystem disorder which
noid oil, the patient must move is associated with generalized tonic-
to Colorado clonic seizures, recurrent headaches,
D. The child should be referred to anorexia and recurrent vomiting. Occipi-
a social worker for an interdisci- tal lobe seizures that often manifest
plinary approach as transient periods of blindness is a
E. The mother should be encouraged hallmark feature of this condition. The
to participate in a clinical trial cumulative effects of the stroke­like
Epilepsy 2
303 9
­ pisodes progressively impair motor
e What is the likely etiology of this
abilities, vision and cognition. child’s seizures?
Glycogen storage diseases are A. Meningococcemia
characterized by cellular build up of B. Group B streptococci
glycogen causing organ dysfunction. C. Whole cell pertussis vaccine
Affected newborns my developed D. Acellular pertussis vaccine
generalized seizures as a result of hypo-
glycemia. vv Correct Answer is: C
Lysosomal storage diseases are Presentation of post-immunization
characterized by the cellular accumulation encephalopathy due to whole cell per-
of intracellular products that cannot be tussis vaccine includes seizures with or
broken down by lysosomal enzymes. Severe without fever occurring within 3 days
motor symptoms and cognitive deficits of immunization and persistent, severe,
typically are present in infancy. Seizures are inconsolable crying for 3 or more hours
associated with some conditions. Wilson’s within 48 h of immunization. Dissemina-
disease is a disorder of copper metabolism tion of Neisseria meningitides presents
resulting in copper accumulation and with fever, headache, fatigue, muscle
dysfunction in the liver and specific brain aches, nuchal rigidity and with subse-
regions. Kayser-Fleisher rings are copper quent signs of sepsis. The child in the
colored rings in the cornea; Neurological vignette has normal vital signs and he
symptoms are not common in children. has no associated characteristic rash.
Seizures are not a common manifesta-
zz Suggested Reading tion of meningitis. Group B streptococci
55 Munnich A, Rotig A, Chretien D, Cormier is a common cause of sepsis in new-
V, Bourgeron T, Bonnefont JP, Rustin borns and may progress to meningitis.
P. Clinical presentation of mitochondrial Acellular pertussis vaccination is not
disorders in childhood. J Inherit Metab associated with post­immunization
Dis. 1996;19(4):521–7. encephalopathy and seizures.

?? 109. An 18-month-old healthy boy was zz Suggested Reading


admitted to the PICU after having a 55 Patel MK, Patel TK, Tripathi
2-min generalized tonic-clonic sei- CB. Diphtheria, pertussis (whooping
zure in your office. He had a second cough), and tetanus vaccine induced
prolonged seizure, he was loaded recurrent seizures and acute encephalopa-
with phenobarbital. He is obtunded, thy in a pediatric patient: Possibly due to
but has non-focal examination. pertussis fraction. J Pharmacol
He has a normal general physical Pharmacother. 2012;3(1):71.
examination with stable vital signs 55 Barlow WE, Davis RL, Glasser JW, Rhodes
with the exception of a temperature PH, Thompson RS, Mullooly JP, Destefano
of 100.5 F. Two days ago, his PCP F. The risk of seizures after receipt of
diagnosed him with strep throat whole-cell pertussis or measles, mumps,
and started him on amoxicillin. His and rubella vaccine. N Engl J Med.
mother reported the child was irrita- 2001;345(9):656–61.
ble and crying for many hours 1 day
before your visit. His past medical his- ?? 110. An 18-year-old male is referred
tory is unremarkable, he has a nega- to your practice because he has a
tive family history of seizures and his medically refractory seizure. He has
immunizations are up-­to­date. a complex partial seizure in which
304 Chapter 9 · Epilepsy 2

he initially has an “out of body” C. Mitochondrial encephalomyopa-


type feeling, then he feels fearful, thy, lactic acidosis, and stroke-
because he knows a seizure is com- like episodes
ing on. He has been told that he D. Lafora disease
will then start to stare, and at times E. Dentatorubral-­pallidoluysian
makes a sound like he has been atrophy
holding his breath, then lets out a
cry/scream. vv Correct Answer is: D
Occasionally, he had some Unverricht-Lundborg disease, myoclonic
generalized tonic clonic seizures epilepsy with ragged red fibers (MERRF),
after this. His MRI scan showing mitochondrial encephalomyopathy,
hippocampal atrophy, and was lactic acidosis, and stroke-like episodes
referred to you for confirmatory PET (MELAS), Lafora disease, and dentatoru-
scan and further management of bra l-­pallidoluysian atrophy (DRPLA) are
his seizures. progressive myoclonus epilepsies. They
What is your diagnosis? started in childhood, consisting of myoc-
A. absence seizures lonus, ­generalized tonic-clonic seizures,
B. Auditory seizures and neurological deterioration. Lafora
C. temporal lobe epilepsy disease is distinct because it has partial
D. Kojenikow’s syndrome visual seizures of occipital onset. Presen-
9 E. none of the above tation includes visual hallucinations, and
transient blindness or scotomas. Patients
vv Correct Answer is: C may also have generalized tonic-clonic
Temporal lobe epilepsy is the most seizures. In Lafora disease, patients
common form of focal epilepsy in develop resting and action myoclonus,
adults. Absence seizures are general- with cognitive decline. Death typically
ized seizures. Auditory seizures, are occurs within several years of onset.
not the most common form of focal It is an autosomal recessive disorder,
epilepsy, and can form a subset of the with mutations of the EPM2A or NHLRC1
temporal lobe epilepsies. Kojenikow’s genes. The EPM2A gene encodes laforin, a
syndrome, is chronic progressive epi- tyrosine kinase inhibitor. The NHLRC1
lepsia partialis continua of childhood, gene encodes the protein malin.
and is fairly rare.
zz Suggested Reading
zz Suggested Reading 55 Alduligan MS, Pearl
55 Lee HW, Khoshbin S. “Seizure Disorders” PL. Electroencephalography in the
Chapter 76, page 1045, Mass General metabolic epilepsies. In: Pearl P, editor.
Comprehensive Clinical Psychiatry. Inherited Metabolic Epilepsies. New York:
Demos Medical; 2013. p. 39–68.
?? 111. A 15-year-old girl had the onset of 55 Trescher WH, Lesser RP. The epilepsies.
seizures, which manifested as tran- In: Bradley WG, Daroff RB, Fenichel GM,
sient blindness. She later developed Jankovic J, editors. Neurology in clinical
progressive resting and action myoc- practice. 4th ed. Elsevier Inc.; 2004.
lonus, with cognitive decline. p. 1953–92.
What is the most likely diagnosis is?
A. Unverricht-Lundborg disease ?? 112. Which of the following is most likely
B. Myoclonic epilepsy with ragged to be associated with benign idio-
red fibers pathic neonatal seizures?
Epilepsy 2
305 9
A. An APGAR score of less than 5 Mutations affecting potassium
B. Birth prior to 35 weeks channel, voltage gated, subfamily Q,
C. Clonic or apneic seizures member 2 (KCNQ2) may underlie benign
D. Seizures continuing after infancy familial neonatal convulsions. Potassium
E. Mutations affecting K potassium channel, voltage gated, subfamily Q,
channel, voltage gated, subfam- member 3 (KCNQ3) is also implicated in
ily Q, member 2 (KCNQ2) this condition. Benign familial neonatal
convulsions may present within days of
vv Correct Answer is: C birth and usually present within the first
Criteria for benign idiopathic neona- 2 months. Seizures observed in
tal seizures include child born after patients with this condition include
39 weeks gestation, Apgar score of at tonic or clonic seizures which may be
least 9 at 5 min, no apparent etiology, multifocal.
and lack of seizures beyond the neonatal
period, normal neurological develop- zz Suggested Reading
ment, the presence of a seizure free 55 Millichap JJ, Millichap JG. Clinical features
interval between the time of birth and and evaluation of febrile seizures.
the onset of seizures, and the presence of UpToDate, Nordli DR, Eichler AF (Ed),
apneic or clonic seizures. These are also UpToDate, Waltham. Accessed on 1 Dec
known as “fifth day fits” and prognosis 2015.
is typically good. Apneic and clonic sei- 55 Abram HS. Febrile seizure. In: Tatum WO,
zures, the latter occurring bilaterally or Sirven JI, Cascino GD, ed. Epilepsy Case
unilaterally, are typical and the presence Studies. New York: Springer Science and
of one or the other is suggested for a Business Media; 2014. p. 13–16.
diagnosis of benign idiopathic neonatal 55 Deivasumathy M, Hartman AL. Pediatric
seizures. Despite the fact that prognosis epilepsy syndromes. Neurologist.
is good ultimately, up to 82% of infants 2010;16(4):223–37.
may go into status epilepticus. 55 Park JT, Shahid AM, Jamoul A. Common
Per the proposed diagnostic criteria, pediatric epilepsy syndromes. Pediatr Ann.
an Apgar score of at least 9 at 5 min of life 2015;44(2):e30–5.
is required. Regardless of official diagnos- 55 Korff CM. Juvenile myoclonic epilepsy.
tic criteria, these seizures occur in healthy UpToDate, Nordli DR, Eichler AF (Ed),
infants with a history of uncomplicated UpToDate, Waltham. Accessed on 1 Dec
pregnancy and delivery. 2015.
Per the proposed criteria, birth after 55 Korff CM. Childhood absence epilepsy.
39 weeks gestation is required for a UpToDate, NordIi DR, Eichler AF (Ed),
diagnosis of benign idiopathic neonatal UpToDate, Waltham. Accessed on 1 Dec
seizures. Again, even if the criteria are not 2015.
strictly followed, benign idiopathic
neonatal seizures occur after an uncompli- ?? 113. Which of the following mutations
cated pregnancy and delivery. has not been associated with gener-
Cessation of seizures after the neona- alized epilepsy with febrile seizures
tal period is required for a diagnosis of plus (GEFS+)?
idiopathic neonatal seizures per the A. SCN1D
proposed criteria. Even if the criteria are B. SCN1A
not followed strictly, continuation of C. SCN1 B
seizures past the neonatal period is not a D. SCN2A
characteristic of this condition. E. GABRD
306 Chapter 9 · Epilepsy 2

vv Correct Answer is: A ?? 114. A 3-month-old boy brought to the


Generalized epilepsy with febrile clinic by his mother. She noticed that
seizures plus (GEFS+) is a familial syn- sometimes when he is picked up, he
drome and SCN1 D mutations are not stiffens up for several seconds. When
a recognized etiology. In contrast to he is put back down, he loosens up
febrile seizures (FS), which occur most again. This happens all the time and
commonly between 6 months and she is concerned. He is “jumpy”: even
6 years of age, the phenotype of “febrile with slightly loud or unexpected
seizures plus” includes patients in whom noises. You suspected the diagnosis
FS continue past the defined upper limit of hyperekplexia.
of age. GEFS+ may also be associated Which of the following statements is
with afebrile generalized tonic-clonic incorrect regarding hyperekplexia?
(GTC) seizures. One-third of patients A. Some patients respond to benzo-
have other seizure types as well. It diazepines or sodium valproate
has a complex pattern of inheritance, B. Mutations in the glycine receptor
although initial genetic discoveries and presynaptic glycine trans-
first identified an autosomal dominant porter have been identified in
familial pattern. Mutations of a number familial hyperekplexia
of ion channel genes have been identi- C. Glycine is the inhibitory neu-
fied in GEFS+ kindreds. These include rotransmitter at spinal interneu-
9 sodium channel (SCN) subunits (SCN1A, rons, including Renshaw cells
SCN1 B, and SCN2A) and GABAA recep- and la inhibitory interneurons
tor subunit genes (GABRD and GABRG2). D. Hyperekplexia results from
The result is increased sodium chan- abnormal spinal la inhibitory
nel activity or impaired GABA activity, interneuron reciprocal inhibition
ultimately leading to increased cortical E. Hyperekplexia is a startle-evoked
hyperexcitability. The most frequently epileptic seizure
reported mutation is SCN1A, which
encodes the pore-forming A-subunit of vv Correct Answer is: E
the sodium channel and comprises four Hyperekplexia is not epileptic, but it
transmembrane domains. The EEG usu- must be distinguished from startle­
ally shows generalized spike-wave or evoked seizures. Hyperekplexia, or
polyspikes. exaggerated startle, can manifest with
sudden brief exaggerated startle reac-
zz Suggested Reading tions (blinking, flexion of the neck and
55 Scheffer IE, Zhang YH, Jansen FE, et al. trunk, abduction and flexion of the
Dravet syndrome or genetic (generalized) arms) or more prolonged tonic startle
epilepsy with febrile seizures plus? Brain spasms. These sometimes occur sec-
Dev. 2009;31(5):394–400. ondary to minor stimuli and do not
55 Zucca C, Redaelli F, Epifanio R, et al. habituate. There are primary familial
Cryptogenic epileptic syndromes related to forms of hyperekplexia in which muta-
SCN1A: twelve novel mutations identified. tions in the glycine receptor and pre-
Arch Neurol. 2008;65(4):489–94. synaptic glycine transporter have been
55 Comprehensive Review in Clinical identified. Glycine is the inhibitory
Neurology: A Multiple-Choice Question neurotransmitter at spinal interneurons
Book for the Wards and Boards ©2011 including Renshaw cells and la inhibi-
Wolters Kluwer Health Lippincott tory interneurons, and abnormal spinal
Williams & Wilkins. All rights reserved. la inhibitory interneuron reciprocal
Epilepsy 2
307 9
inhibition is thought to be the cause of at the beginning, but progressively
startle in these cases. Secondary forms over months or years, the background
of exaggerated startle occur in a vari- deteriorates and multifocal epileptiform
ety of brainstem disorders as well as in abnormalities appear, mainly in the
Creutzfeldt­Jakob disease and stiff per- occipital regions, in addition to general-
son syndrome. ized bursts. Lafora bodies are PAS­posi-
tive intracellular polyglucosan inclusion
zz Suggested Reading bodies found in neurons, cardiac
55 Fahn S, Jankovic j. Principles and practice muscle, skeletal muscle, hepatocytes,
of movement disorders. Philadelphia: and sweat gland duct cells, making it
Elsevier; 2007. possible to detect these bodies in skin
biopsy specimens. Most patients die
?? 115. A 15-year-old male with progressive within 10 years of onset, and the treat-
cognitive decline and ataxia has a ment remains supportive.
history of myoclonic epilepsy and Mutation in cystatin B and EPM1
multiple seizure types. EEG shows mutation are seen in Unverricht Lundborg
occipital region spikes and waves. syndrome. Cherry red spot in a patient
You have done a skin biopsy, which with progressive myoclonic epilepsy (PME)
showed periodic-acid-­Schiff (PAS)- suggests sialidosis. Ragged red fibers on
positive intracellular inclusions. muscle biopsy are seen in myoclonic
Which of the following is the most epilepsy with ragged red fibers.
likely finding in this patient?
A. Mutation affecting cystatin zz Suggested Reading
B. Cherry red spot on fundoscopic 55 Delgado-Escueta AV, Ganesh S, Yamakawa
examination K. Advances in the genetics of progressive
C. Ragged red fibers on muscle myoclonus epilepsy. Am J Med Genet.
biopsy 2001;106:129–38.
D. EPM1 mutation 55 Shahwan A, Farrel M, Delanty
E. EPM2 A mutation N. Progressive myoclonic epilepsies: a
review of genetic and therapeutic aspects.
vv Correct Answer is: E Lancet Neural. 2005;4:239–48.
The clinical presentation is indicative 55 Comprehensive Review in Clinical
of Lafora body disease, which is an Neurology: A Multiple-Choice Question
autosomal recessive disorder associated Book for the Wards and Boards ©2011
with a mutation in the gene EPM2A on Wolters Kluwer Health Lippincott
chromosome 6q, encoding laforin, a Williams & Wilkins. All rights reserved.
ribosomal protein with undetermined
function. It presents between 12 and ?? 116. A 5-year-old girl has moderate
17 years of age. Patients usually have intellectual disability came for her
different seizures types, including follow-up. She has history of intrac-
myoclonus, atypical absences, atonic, table seizures and is on 4 different
complex partial, and occipital seizures antiepileptic medications. On exami-
with transient blindness and visual nation, she very small eyes with very
hallucinations. Patients can have dys- poor vision. She has small philtrum,
arthria, ataxia, as well as emotional dis- a flat nose with an upturned tip,
turbance, and cognitive decline leading large ears, and sparse eyebrows. Her
to dementia. EEG shows an evolution, brain MRI shows agenesis of corpus
with multiple spike-wave discharges callosum.
308 Chapter 9 · Epilepsy 2

Which one of the following is true teacher reports that the student
regarding this condition? has episodes “several times a day”
A. This condition is inherited in when she is unable to respond to
autosomal dominant pattern her name for a few seconds when
B. This condition is lethal in boys the teacher calls on her, but then
and occurs almost exclusively in “snaps out of it” and can answer
females whatever questions are posed a few
C. Patient will slowly improve in seconds later. The child claims that
her mental development to have she does not heard the teacher call-
normal IQ ing her name initially. The teacher
D. Patient will have syringomyelia has a concern that the child might
along with above features have ADHD. Also, her family have
E. This condition is inherited in seen similar behavior at home, but
autosomal recessive pattern felt that she was “going through a
phase”, as her parents have recently
vv Correct Answer is: B separated.
The clinical scenario is Aicardi syn- If this is a seizure, how would you
drome. Aicardi syndrome is an X-linked classify it?
dominant disorder and occurs almost A. absence seizure, partial with
exclusively in females. It has a triad. (1) impairment of consciousness
9 Agenesis or dysgenesis of the corpus only
callosum. (2) Infantile spasms and epi- B. absence seizure, general, with
lepsy, and (3) Chorioretinal lacunae. impairment of consciousness
Additional brain abnormalities include - and mild clonic components
asymmetry between hemispheres, C. absence seizure, general, non-
polymicrogyria, cysts, microcephaly convulsive, with atonic compo-
and hydrocephalus. Other ophthalmic nents
abnormalities include - microphthalmia D. absence seizure, general, with
or coloboma, which can cause blind- autonomic components
ness. Facial features may include - small E. none of the above adequately
philtrum, a flat nose with an upturned describes this seizure, there is
tip, large ears, and sparse eyebrows. insufficient information
Other features include - small hands,
scoliosis. They often have gastrointes- vv Correct Answer is: E
tinal problems such as constipation or We do not have enough descriptions
diarrhea, gastroesophageal reflux, and adequately describe this as a seizure.
difficulty feeding. Further testing, and more history are
needed. It might got confused with an
zz Suggested Reading absence seizure, but it could be due to
55 Mowzoon N, Flemming KD. Neurology other causes. However, absence seizures
Board Review: an illustrated study guide. are generalized. While there is descrip-
Rochester: Mayo Clinic Scientific, 2007. tion of impairment of consciousness
Print. only, there is also no description of
other symptoms. This question is not
?? 117. A third grade student was referred meant to be confusing, but to expose
to you by her teacher with concerns the student to the idea that seizures
that the student seems to “space need to be diagnosed by a medical
out” frequently in her class. The professional on the basis of observed
Epilepsy 2
309 9
phenomena and concrete evidence. a history of absence seizures (occurring
In order to diagnose a seizure certain in between 20% and 40% of patients)
parameter must be reported, and not all and myoclonic jerks may be elicited.
of the details needed are mentioned in Myoclonic jerks, often occurring in the
this case. A description of the eye move- morning, may be misinterpreted as
ments, if any, a description of any clonic, clumsiness. Patients may fling items and
atonic, or tonic movements present, any drop things. The EEG shows 4 to 6 Hz
automatisms or autonomic phenomena polyspike and slow waves bilaterally.
would be needed in order to fully clas- This condition is treated with valpro-
sify/describe an absence seizure. While ate as a first line although women of
this sounds like an absence seizure, child bearing age may choose to take
with impairment of consciousness only lamotrigine. Levetiracetam and topi-
we cannot be sure until more history is ramate can be used as a second line
gathered and an episode observed by a treatment. Phenytoin, carbamazepine,
medical professional. Other helpful tests and oxcarbazepine may cause increases
would be an EEG, a brain MR, as well as in absence seizures and increased
basic blood work. myoclonic jerks. Pregabalin, tiagabine,
vigabatrin, and gabapentin may also
zz Suggested Reading increase seizure frequency in those with
55 Lee HW, Khoshbin S. “Seizure Disorders” juvenile myoclonic epilepsy.
Chapter 76, page 1042, Mass General Patients with juvenile myoclonic
Comprehensive Clinical Psychiatry. epilepsy may have absence seizures;
however, childhood absence epilepsy is a
?? 118. A 14-year-old female had a general- separate entity. Childhood absence
ized tonic clonic seizure. She had no epilepsy starts at 4 to10 years of age, they
alarming preceding aura and was have a normal neurological development,
associated with urinary incontinence and the presence of generalized rhythmic
and tongue biting. It lasted for 3 min spikes or double spike wave discharges
and was followed by several hours of around 3 Hz. In addition, seizures should
lethargy and confusion. Her mother be brief, lasting between 4 and 20 s, and
reported that the patient has persis- should occur frequently. The presence of
tent difficulties in school. Her teachers other seizure types is one of the exclusion
have reported that she doesn’t pay criteria.
attention during class. The patient Late onset childhood occipital epilepsy
is also clumsy and frequently drops has also been referred to as late onset
items, especially in the morning. childhood epilepsy with occipital spikes or
What is the most likely diagnosis? Gastaut type. Onset is often between 8 or
A. juvenile absence epilepsy 9 years of age and seizures are character-
B. juvenile myoclonic epilepsy ized by visual loss or hallucinations,
C. Late onset childhood occipital sometimes with post ictal headache.
epilepsy Hemiclonic activity or generalized
D. Panayiotopoulos syndrome convulsions may also occur. EEG may show
bursts of spike wave complexes in the
vv Correct Answer is: B occipital region.
This is a clinical presentation of juvenile Panayiotopoulos syndrome is also
myoclonic epilepsy. Usually, patients referred to as early onset benign occipital
present with generalized tonic clonic epilepsy. Age of onset is often between
seizures, but, on taking detailed history, the ages of 3 and 6 years. Seizures are
310 Chapter 9 · Epilepsy 2

characterized by emesis in up to 80% of 55 Nordi DR. Focal and multifocal seizures.


patients. In about half of cases, these In: Swaiman K, Ashwal S, Ferriera OM,
seizures may last up to a half hour. Other Ferriero D, editors. Swaiman’s pediatric
seizure manifestations include pallor, neurology: principles and practice. 5th ed.
hypersalivation, and urinary incontinence Elsevier; 2012. p. 751–66.
all with preservation of consciousness. 55 Nickels KC, Wirrell EC. Benign and
Gaze deviation may occur after episodes malignant childhood epilepsies. In: Miller
of emesis. Generalized convulsions, hemi JW, Goodkin HP, editors. Epilepsy.
convulsions, and visual hallucinations may Hoboken: Wiley; 2013. p. 147–58.
also be observed. 55 Pandolfo M. Pediatric epilepsy genetics.
Curr Opin Neurol. 2013;26:137–45.
zz Suggested Reading 55 Hani AJ, Mikati HM, Mikati MA. Genetics
55 Millichap JJ, Millichap JG. Clinical features of pediatric epilepsy. Pediatr Clin North
and evaluation of febrile seizures. Am. 2015;62(3):703–22.
UpToDate, NordIi DR, Eichler AF (Ed),
UpToDate, Waltham. Accessed on 1 Dec ?? 119. A baby with epileptic encepha-
2015. lopathy has been evaluated for
55 Abram HS. Febrile seizure. In: Tatum WO, microcephaly and prominent devel-
SirvenJI, Cascino GD, editors. Epilepsy opmental delay. His CSF analysis
case studies. New York: Springer Science shows a glucose level of 30 mg/dl,
9 and Business Media; 2014. p. 13–6. whereas the level of serum glucose
55 Deivasumathy M, Hartman AL. Pediatric was 112 mg/dl. Other CSF tests are
epilepsy syndromes. Neurologist. normal.
2010;16(4):223–37. Which of the following is correct
55 ParkJT, Shahid AM, Jamoul A. Common regarding the most likely condition in
pediatric epilepsy syndromes. Pediatr Ann. this patient?
2015; 44(2):e30–e35. A. Brain MRI usually suggests the
55 Korff CM. Juvenile myoclonic epilepsy. diagnosis
UpToDate, Nordli DR, Eichler AF, editors. B. It is caused by deficiency in the
UpToDate, Waltham. Accessed on 1 Dec glucose transporter type 1
2015. C. Phenobarbital is the treatment of
55 Korff CM. Childhood absence epilepsy. choice
UpToDate, Nordli DR, Eichler AF, editors. D. Ketogenic diet has been shown to
UpToDate Waltham. Accessed on 1 Dec be ineffective for seizure control
2015. E. It is inherited in an X-­linked fashion
55 Moller RS, Heron SE, Larsen LH, Lim CX,
Ricos, MG, Bayly MA, van Kempen MJ, vv Correct Answer is: B
Klinkenberg S, Andrews I, Kelley, K, This is a case of a glucose transporter
Ronen GM, Callen D, McMahon JC, type 1 (GLUT-1) deficiency. The brain
Yendle SC, Carvill GL, Mefford HC, utilizes glucose as its primary source of
Nabbout R, Nabbout R, Poduri A, Striano energy. In fasting conditions, glycogen
P, Baglietto MG, Zara F, Smith is depleted within minutes, and since
NJ. Pridmore C, Gardella E, Nikanorova amino acids and fat cannot be used for
M, Dahl HA, Gellert P, Scheffer IE, the production of energy in the brain,
Guning, Kragh-Olsen B, Dibbens ketones become the alternative fuel.
LM. Mutations in KCNT1 cause a spec- Glucose crosses the blood brain barrier
trum of focal epilepsies. Epilepsia. facilitated by GLUT-1, which is a mem-
2015;56(9):e114–20. brane-bound protein encoded by the
Epilepsy 2
311 9
SLC2A 1 gene on chromosome 1p35– C. Aicardi syndrome
31.3. It is inherited as an autosomal D. Ohtahara syndrome
dominant fashion and causes a defect E. Benign myoclonic epilepsy of
in glucose transport across the blood- infancy
brain barrier and into brain cells, mani-
festing as an epileptic encephalopathy vv Correct Answer is: B
with infantile-onset seizures, develop- This is a presentation of benign neonatal
mental delay, microcephaly, and com- seizures. In this syndrome, full-term, other-
plex movement disorders. CSF glucose wise healthy, newborns develop seizures
level is low with a normal serum glucose around day 5 of life (also referred to as
level, and other CSF studies are normal, “fifth day fits”), which are partial clonic sei-
excluding other causes of hypoglycor- zures that may be unilateral and/or sym-
rhachia (such as CNS infection). EEG metric and may migrate to other regions
may show 2.5 to 4 Hz spikes and waves of the body. They are frequently associated
and the interictal EEG findings may with apneic spells. The EEG is normal, but
improve with glucose. Neuroimaging may demonstrate the “pointu alternant”
shows no specific abnormalities. Later- pattern, characterized by discontinuous,
onset forms with episodic movement asynchronous, unreactive activity with
disorders and ataxia, paroxysmal exer- intermixed sharp waves. Patients are neu-
tional dyskinesia, or early-onset atypical rologically normal. No seizure treatment is
absence epilepsy have been described. need, and seizures resolve spontaneously
A ketogenic diet should be started as by 4–6 weeks of age. Benign neonatal
soon as the diagnosis is suspected, since seizures and benign familial neonatal
this treatment option improves seizure seizures should be diagnoses of exclu-
control and the abnormal movements; sion, and workup to rule out symptomatic
however, it is less effective for the psy- seizures is recommended. Benign familial
chomotor impairment. neonatal seizures is an autosomal domi-
nant disorder, characterized by seizures
zz Suggested Reading in the first few days of life, which resolve
55 Comprehensive review in clinical neurol- spontaneously within few weeks. Genetic
ogy: a multiple-choice question book for linkage studies have mapped two disease
the Wards and Boards ©2011 Wolters loci, both associated with mutations in
Kluwer Health Lippincott Williams & voltage-­gated potassium channels, in the
Wilkins. All rights reserved. genes KCNQ2 on chromosome 20 and
KCNQ3 on chromosome 8.
?? 120. You are evaluating a newborn for
seizures. His seizures are character- zz Suggested Reading
ized by apneic spells associated with 55 Bradley WG, Daroff RB, Fenichel GM,
unilateral or bilateral clonic move- et al. Neurology in Clinical Practice, 5th
ments. Starting on day 5 of life he ed. Philadelphia: Elsevier; 2008.
has been having multiple spells per 55 Browne TR, Holmes GL. Handbook of
day. He has a normal neurological Epilepsy, 4th ed. Philadelphia: Lippincott
exam in between seizures. His inter- Williams & Wilkins; 2008.
ictal EEG is normal. 55 Comprehensive review in clinical neurol-
Which of the following is the most ogy: a multiple-choice question book for
likely diagnosis? the Wards and Boards ©2011 Wolters
A. West syndrome Kluwer Health Lippincott Williams &
B. Benign neonatal seizures Wilkins. All rights reserved.
312 Chapter 9 · Epilepsy 2

?? 121. A first-time mother brings her or lethargy. Neurologic manifestations


9-day-old infant in with a chief com- rapidly develop, such as alternating
plaint of seizures. The infant had an hypotonia and hypertonia, dystonia,
uncomplicated term delivery and seizures, and encephalopathy.
has been well. Her infant has been
and remains afebrile. ?? 122. Myoclonic seizures (sudden, brief
Which of the following is the most muscular contractions that occur
likely cause of this infant’s seizures? singly or repetitively) are characteris-
A. Hypokalemia tically seen in what setting?
B. Hyponatremia A. West syndrome
C. Hypocalcemia B. adolescents with gray matter
D. Hypomagnesemia ­disease
E. Maple syrup urine disease C. adults with viral encephalitis
D. adults with prion disease
vv Correct Answer is: B E. all of the above
Hyponatremia is the most common F. none of the above
cause of seizures in a well newborn
without fever. Water intoxication is the vv Correct Answer is: E
most common cause of hyponatremia All of the previous seizure disorders are
during infancy. Infants are unable to characterized by a tendency of having
9 adequately concentrate urine so parents myoclonic seizures. West Syndrome is
who dilute formula or give their infants characterized by childhood spasms,
tap water put their infants at risk. hypsarrhythmia on interictal EEG, and
Hypocalcemia is also a very common intellectual disability. The spasms may
cause of seizures in the neonate, so occur up to hundreds times per day.
serum calcium levels should be checked. The cause of West syndrome is theo-
In fact, all infants have a slight decline in rized to be organic brain dysfunction
serum calcium levels with a nadir at but the precise etiology is unknown.
24–48 h. Gray matter diseases usually manifest
Symptomatic hypocalcemia is more as a neuronal dysfunction, and may
common in infants of diabetic mothers, occur from metabolic storage disease,
preterm infants, or infants with a history such as (Tay-Sachs disease, Nieman-
of anoxic encephalopathy. Pick disease, etc). Gray matter is where
Hypomagnesemia is not as common as the neuronal dendrites and cell bodies
hypocalcemia, but symptoms of hypo- are, so it makes sense that diseases
magnesemia mimic those of hypocalcemia which affect the neuron would cause
and it is difficult to correct hypocalcemia if seizures. Adults with viral encephalitis,
the serum magnesium is also low. such as that caused by HSV, HIV, and
Therefore, infants with seizures should other viruses, can cause seizures. It
have a comprehensive evaluation of their is postulated that the viral infection
electrolytes. makes changes to the neuron, and the
Hypokalemia is uncommon in infancy neurons are unable to heal the changes
and does not typically cause seizures. caused, which results in abnormal func-
Maple syrup urine disease is a rare tioning. Prion diseases, such as BSE,
disease resulting from the inability to “mad cow disease”, or Kuru, affect the
catabolize branched chain amino acids. proteins in the brain, but at present the
Infants typically present between 4 and exact mechanisms by which they oper-
7 days of life with poor feeding, vomiting, ate is still under debate.
Epilepsy 2
313 9
zz Suggested Reading vv Correct Answer is: B
55 Lee, HW, Khoshbin, S. “Seizure Disorders” Ethosuximde controls absence seizes
Chapter 76, page 1044, Mass General in about 80% of cases. In addition, the
Comprehensive Clinical Psychiatry. drug has a very safe side effect profile
and is therefore the drug of choice in
?? 123. An 18-year-old female was admitted this seizure type. Valproic acid and
to the emergency room after a gener- lamotrigine are effective in about 60%
alized tonic­clonic seizure that began of cases but have more serious potential
focally in her right lower extremity. side effects. Carbamazepine has been
Although the seizure stopped within shown to aggregate absence seizures
1 min, there was persistent weakness and worsen the condition. This can
of the right lower extremity lasting occur when the clinical presentation of
several hours. Further testing revealed absence seizures is mistaken for com-
a small arteriovenous malformation plex partial seizures, in which carbam-
near the motor cortex. Focal weak- azepine is chosen.
ness lasting for 24 h following a motor
seizure is most likely attributable to zz Suggested Reading
which of the following? 55 Camfield P, Camfield C. Epileptic syn-
A. lntracerebral hemorrhage dromes in childhood: clinical features,
B. Subarachnoid hemorrhage outcomes, and treatment. Epilepsia.
C. Encephalitis 2002;43(Suppl 3):27–32.
D. Todd paralysis
E. Hyponatremia ?? 125. Which of the following is the best
treatment option for simple febrile
vv Correct Answer is: D seizures?
Weakness after seizure activity is evi- A. IV lorazepam
dence of a postictal paralysis, or Todd B. Rectal diazepam
paralysis. Postictal weakness does not C. Supportive management
suggest bleeding or new areas of cere- D. Phenobarbital
brocortical damage, but imaging with E. Intranasal midazolam
CT scan is appropriate to exclude these
possibilities. Postictal paralysis may vv Correct Answer is: C
last for many hours, or even days. The Supportive care is the best management
precise cause is unknown, but it appears choice for a simple febrile seizure (FS). It
to be caused by some kind of neuronal is estimated that about 3–5% of children
exhaustion occurring after frequent aged 5 months to 5 years have simple FS.
repetitive discharges. It may reflect 90% occurs in the first 3 years of life. 1/3
depletion of glucose in the neurons in of patients have at least one additional
the epileptic focus. seizure. Risk factors of simple FS are fam-
ily history of FS, prolonged neonatal ICU
?? 124. What is the drug of choice for a 6-year- stay, developmental delay, and day care.
old girl that is symptomatic and is Incidence does not increase in propor-
found to have 3 per second spike and tion to increase in temperature. No risk
wave complexes on her EEG? factors are found in 50% of children with
A. Valproic acid an FS. The risk of afebrile epilepsy after FS
B. Ethosuximde is increased in children with developmen-
C. Carbamazepine tal delay, abnormal neurologic examina-
D. Lamotrigine tion, complex FS, and a family history of
314 Chapter 9 · Epilepsy 2

afebrile seizures. There is a < 5% risk that 55 Steering Committee on Quality


patients with a simple FS will develop Improvement and Management,
epilepsy. It is estimated that approxi- Subcommittee on Febrile Seizures
mately 15% of patients with epilepsy have American Academy of Pediatrics.
a history of FS. Simple FS are characterized 55 Febrile seizures: clinical practice guideline
by the following: <15 min in duration, for the long-term management of the child
generalized seizure, and no focality, with simple febrile seizures. Pediatrics.
normal neurologically examination, no 2008;121:1281–6.
persistent deficits, and negative family 55 Comprehensive review in clinical neurol-
history for seizures. Complex FS occur in ogy: a multiple-choice question book for
approximately 20% of FS and are charac- the Wards and Boards ©2011 Wolters
terized by the following: >15 min in Kluwer Health Lippincott Williams &
duration, focal features, abnormal Wilkins. All rights reserved.
neurologic examination, and seizure
recurrence in < 24 h, postictal signs (Todd’s ?? 126. Which infant with infantile spasms
paralysis), and are more likely to be due to has the best prognosis?
meningitis, encephalitis, or an underlying A. An infant with tuberous sclerosis
seizure disorder. No need for prophylaxis B. An infant with cryptogenic infan-
medications, but can be considered for tile spasms
recurrent or prolonged seizures, afebrile C. An infant who was premature
9 seizures, after complex FS, and with an D. An infant with Sturge-­Webber
abnormal neurologic examination and Syndrome
developmental delay. Chronic prophylaxis E. An infant with Maple-­Syrup
commonly includes phenobarbital and Urine Disease
valproic acid, whereas short-term prophy-
laxis could include diazepam and anti- vv Correct Answer is: B
pyretics, though definitive data on the use Infantile spasms, or “drop attacks” are a
of antipyretics for the prevention of FS are form of seizure in which there is a sud-
lacking. Furthermore, the potential den tonic contraction of the trunk and
toxicities associated with available limbs that lasts a few seconds. Typically,
antiepileptic agents outweigh the there is developmental regression. They
relatively minor risks associated with occur more common just before sleep or
simple FS. After reviewing the potential upon awakening, but they can occur at
risks and benefits of available effective any time. They tend to occur in clusters.
therapies for short- and long-term An EEG shows a hypsarrhythmia pat-
prophylaxis, the AAP concluded (in its tern, which is pathognomonic for this
clinical practice guideline on long-term condition. Infants who develop infantile
management of children with FS) that spasms often have an underlying condi-
neither continuous nor intermittent tion, which may or may not have been
anticonvulsant therapy is recommended previously diagnosed. Therefore, part of
for children with one or more simple FS. the workup includes an evaluation for
an underlying issue; this would include
zz Suggested Reading a brain MRI and CT, genetic studies, a
55 Berg AT, Shinnar S. Complex febrile metabolic work-­up, and a lumbar punc-
seizures. Epilepsia. 1996;37:126–33. ture. Tuberous sclerosis is the most com-
55 Knudsen FU. Febrile seizures: treatment mon disorder associated with infantile
and prognosis. Epilepsia. 2000;41(1):2–9. spasms.
Epilepsy 2
315 9
Treatment is with ACTH and antiepilep- vv Correct Answer is: A
tics, and the prognosis is poor, with a high Diagnosis of primary generalized epi-
likelihood of developmental delay and/or lepsy (PGE) is a contraindication in
autism. Prognosis is often related to the consideration of epilepsy surgery for
underlying condition. Prognosis is best in refractory epilepsy. Best candidates will
the cryptogenic (or idiopathic) form, where have a focal non-dominant hemisphere
there is no underlying disorder. Other seizure focus who failed at least 2 or 3
positive prognostic indicators is age of antiepileptic agents. Other listed condi-
diagnosis over the age of 4 months, short tions are not contraindications for epi-
time from onset to treatment, and early lepsy surgery.
response to treatment. Early intervention is
imperative. All of the other answers are ?? 129. Which of the following anticonvul-
conditions that are found in patients with sants binds to synaptic vesicle pro-
infantile spasms, and all carry a poor tein SV2A?
prognosis. A. Levetiracetam
B. Topiramate
?? 127. Levels of which of the following hor- C. Lamotrigine
mones is evidently used to evaluate D. Ethosuximide
recent seizure? E. Perampanel
A. TSH
B. Prolactin vv Correct Answer is: A
C. ACTH Levetiracetam binds to synaptic vesicle
D. GH protein SV2A, which decreases calcium
influx into the presynaptic terminal.
vv Correct Answer is: B The drug also blocks high voltage
Serum prolactin levels are increased for activated calcium channels and affects
5–30 times, if measured within 20 min gamma­aminobutyric acid (GABA) func-
of generalized tonic clonic seizures. This tion. It is unclear, however, how these
level helps in excluding pseudoseizures actions relate to its anticonvulsant
in few patients. effect.
Topiramate has multiple mechanisms
zz Suggested Reading of action, blocking sodium channels,
55 Sadock BJ, Sadock VA, Kaplan HI. Kaplan calcium channels, kainate receptors, and
& Sadock’s comprehensive textbook of a-amino-­3-hydroxyl-5-methyl-4-
psychiatry. Philadelphia: Lippincott isoxazole-proprionate (AMPA) glutamate
Williams & Wilkins. Chapters 2.4;2005. receptors, while enhancing GABA
activity.
?? 128. An 18-year-old male has refractory Lamotrigine also has multiple mecha-
epilepsy. Which of the following nisms of action, blocking sodium channels
conditions is a likely contradiction to and high voltage activated calcium
epilepsy surgery in this patient? channels.
A. Grade 1 Astrocytoma Ethosuximide blocks thalamic T-type
B. Autism calcium channels, and likely inhibits
C. Primary generalized epilepsy sodium currents. Perampanel is a new
D. Prior pacemaker placement for anticonvulsant with a novel mechanism of
arrhythmia action, acting as a non-competitive AMPA
E. Frontal lobe seizures glutamate receptor antagonist.
316 Chapter 9 · Epilepsy 2

zz Suggested Reading known seizures. Also, in patients who do


55 Stafstrom CE. Mechanisms of action of have clinical and EEG findings characteris-
antiepileptic drugs: the search for synergy. tic of this form of epilepsy, the EEG
Curr Opin Neurol. 23:157–63. abnormalities may persist even after the
seizures have stopped. Thus, decisions on
?? 130. Which of the following is most likely whether to start or stop anti-epileptic
to be true of benign focal epilepsy drugs are not based on EEG findings. As
with central mid temporal spikes? mentioned above, anti-epileptic medica-
A. Lifelong treatment with anti- tions are often not indicated at all.
epileptic medications is often
necessary zz Suggested Reading
B. Seizures usually occur during the 55 Millichap JJ, Millichap JG. Clinical features
daytime while the child is awake and evaluation of febrile seizures. UpToDate,
C. Children with this disorder may Nordli DR, Eichler AF editors. UpToDate,
complain of facial numbness or Waltham. Accessed on 1 Dec 2015.
may be observed drooling 55 Abram HS. Febrile seizure. In: Tatum WO,
D. EEG findings are used to deter- Sirven JI, Cascino GD, editors. Epilepsy
mine whether to continue, dis- case studies. New York: Springer Science
continue, or start anti-epileptic and Business Media; 2014. p. 13–6.
medications 55 Deivasumathy M, Hartman AL. Pediatric
9 epilepsy syndromes. Neurologist.
vv Correct Answer is: C 2010;16(4):223–74.
Benign focal epilepsy with centrotem- 55 Park JT, Shahid AM, Jamoul A. Common
poral spikes (benign rolandic epilepsy), pediatric epilepsy syndromes. Pediatr Ann.
accounts for 15% of childhood epilepsy. 2015;44(2):e30–5.
Usually, it starts around age of 7 and 55 Korff CM. Juvenile myoclonic epilepsy.
9 years. Its name is based on typical EEG UpToDate, Nordli DR, Eichler AF editors.
findings. There are diphasic or triphasic UpToDate, Waltham. Accessed on 1 Dec
sharp waves occur in the mid temporal 2015.
and central areas. Clinical manifesta- 55 Korff CM. Childhood absence epilepsy.
tions include: speech arrest, hypersali- UpToDate, Nordli DR, Eichler AF editors.
vation, unilateral facial twitching, and UpToDate, Waltham. Accessed on 1 Dec
paresthesias inside the mouth. 2015.
Treatment with anti-epileptic medica- 55 Hani AJ, Mikati HM, Mikati MA. Genetics
tions is often not necessary. Seizures of pediatric epilepsy. Pediatr Clin North
usually occur at night and most patients Am. 2015;62(3):703–722. 12. Mizrah
have under 10 seizures total. Patients may EM. Neonatal epileptic syndromes.
enter remission after 2 to 4 years of onset of 55 Anderson GD. Pharmacopeia. In: Miller
seizures. Most are in remission by age 16. JW, Goodkin HP, editors. Epilepsy.
Treatment is indicated in patients with Hoboken: Wiley; 2013. p. 126–38.
frequent seizures, or in those with general- 55 Glaze DG. Clinical features and diagnosis
ized seizures. 75% of seizures occur either of infantile spasms. UpToDate, Nordli DR,
during the night, or when the patient Eichler AF, editors. UpToDate, Waltham.
wakes. On EEG studies, epileptiform activity Accessed on 5 Dec 2015.
is more prominent during Non-REM sleep. 55 Glaze DG. Management and prognosis of
EEG activity typical of benign focal infantile spasms. UpToDate, Nordli DR,
epilepsy with centrotemporal spikes may Eichler AF editors, UpToDate, Waltham.
occur in individuals who do not have Accessed on 5 Dec 2015.
Epilepsy 2
317 9
55 Glaze DG. Etiology and pathogenesis of management of absence seizures and is
infantile spasms. UpToDate, Nordli DR, thus not a first line therapy. Similarly,
Eichler AF editors. UpToDate, Waltham. phenobarbital is not effective in the
Accessed on 5 December 2015. management of this seizure type.
Tiagabine may worsen absence seizures,
?? 131. A 6-year-old female came to the and is thus contraindicated.
office with her mother who com- Carbamazepine, gabapentin, and
plains about her daughter declining vigabatrin are also contraindicated.
school performance. Her teachers It is worthwhile to note that carbam-
have complained to the mother that azepine may worsen absence seizures, and
the child doesn’t listen and tends to it is contraindicated.
“stare off.” You ordered an EEG which
showed 3 Hertz spike and slow wave ?? 132. West’s syndrome consists of the triad of:
complexes. A. infantile spasms, hydrocephalus,
What is the best next step in 3 Hz spike-wave
­management? B. absence seizures, myoclonic sei-
A. The child should be started on zures, 3 Hz spike-wave
phenytoin C. myoclonic seizures, agenesis of
B. The child should be started on the corpus callosum, hypsar-
tiagabine rhythmi
C. The child should be started on D. infantile spasms, developmental
carbamazepine arrest, hypsarrhythmia
D. The child should be started on E. absence seizures, developmental
ethosuximide arrest, 4-6 Hz spike-­wave
E. The child should be started on
phenobarbital vv Correct Answer is: D
West’s syndrome consists of: (1) Infan-
vv Correct Answer is: D tile spasms. Flexor spasms include
This is a clinical presentation of child- flexion of the head, trunk, and lower
hood absence epilepsy. Exclusion cri- extremities, with adduction of the
teria include (1) the presence of other upper extremities. Extensor spasms
seizure types (2) eyelid, perioral, head, involve extension of the head and
or limb myoclonias that are massive and trunk, with abduction of the upper
sustained (3) sensory or visual seizure extremities. The infantile spasms typi-
precipitants, and (4) either no impair- cally occur in clusters during morning
ment, or mild impairment, of conscious- feedings. (2) Developmental arrest,
ness during spike wave discharges, and beginning with or prior to the onset of
(5) spike wave discharges lasting under infantile spasms. (3) Hypsarrhythmia on
4 s on EEG. Inclusion criteria include. EEG. Up to 40% of cases have no identi-
normal neurological exam and fiable etiology (cryptogenic), while the
normal development, (2) seizures lasting remaining cases may be due to cerebral
between 4 and 20 s occurring many times dysgenesis, genetic or metabolic dis-
per day and (3) the presence of 3 Hz orders, infections, anoxic injury, and
spike wave discharges. Ethosuximide is head trauma. Onset is typically within
the first line medication. Valproate, the first year, most often between 3
lamotrigine, topiramate, benzodiaz- and 7 months of age. Corticosteroids
epines, and acetazolamide are also are first-line agents, but the prognosis
efficacious. Phenytoin is not effective in remains poor.
318 Chapter 9 · Epilepsy 2

zz Suggested Reading lepsy. Interestingly, a prior history of a


55 Trescher WH, Lesser RP. The epilepsies. complex FS has not been found to be
In: Bradley WG, Daroff RB, Fenichel GM, associated with a higher risk for recur-
Jankovicj, editors. Neurology in clinical rence of FS. Risk factors to develop a
practice. 4th ed. Elsevier, Inc.; 2004. first FS include a first- or second-degree
p. 1953–92. relative with a history of FS, develop-
mental delay, neonatal nursery stay for
?? 133. A 20-month-old girl has a general- more than 30 days, and attendance of
ized seizure lasting 5 min with fever day care. Most patients will have only
of 39C. She has no significant past one FS without recurrence and without
medical or family history. The patient subsequent development of epilepsy.
recovered without any residual neu- Recurrent FS may occur, especially in the
rologic deficit. The family would like presence of recognizable risk factors,
to know the risk of recurrence. which include a family history of FS, age
Which of the following is not a pre- younger than 18 months at the time of
dictor of recurrence of febrile sei- the first FS, lower peak temperature, and
zures (FS)? shorter duration of fever prior to the
A. Family history of FS FS. Simple or complex FS carry similar
B. Age younger than 18 months at risk of recurrence. The risk of develop-
the time of FS ing epilepsy following a single simple
9 C. Shorter duration of fever prior to FS FS is not substantially different than the
D. Lower peak temperature at the risk in the general population. On the
time of FS other hand, patients with complex FS
E. Complex FS may be at risk of developing epilepsy in
the future. The risk is even higher when
vv Correct Answer is: E the complex FS was very prolonged in
This patient had a simple febrile sei- duration (i.e., febrile status epilepticus).
zure (FS). FS is defined as a seizure that Other risk factors to develop epilepsy
occurs in association with a febrile ill- include the presence of a neurodevel-
ness in the absence of CNS infection or opmental abnormality and/or family
acute electrolyte imbalance in children history of epilepsy. Importantly, a family
without prior afebrile seizures. These history of FS does not predispose the
occur commonly between 6 months and patient to subsequent development of
5 years of age, with a peak incidence at epilepsy. The evaluation of patients with
18 months. FS can be simple or complex. FS should be targeted at assessing the
Most FS represent simple FS, that is, cause of the seizure and ruling out CNS
generalized, usually isolated, seizures infections or abnormalities. The exact
of relatively brief duration lasting less pathophysiology of FS remains unclear.
than 15 min. Complex FS on the other A relationship between prolonged FS
hand are prolonged, lasting more than and mesial temporal lobe sclerosis has
15 min, have focal features, and may been proposed; however, this associa-
occur multiple times in the course of tion is controversial and has not con-
the same febrile illness, or within the clusively been confirmed in prospective
same 24-h period. A prior history of a and population-­based studies, which
complex FS is a known risk factor for are ongoing. Long-term prophylaxis and
subsequent development of epilepsy, AEDs are not indicated for FS, but can be
whereas a family history of FS does not considered in some cases, especially if
increase the risk of subsequent epi- a high risk of recurrence exists. In these
Epilepsy 2
319 9
cases, phenobarbital or valproic acid Mutations in potassium channel,
can be considered on a case-by­case voltage gated, subfamily Q, member 2
basis. Short-term prophylaxis may also (KCNQ2) are implicated in benign familial
be needed in some cases, and should be neonatal convulsions. A related gene-
focused on temperature control during potassium channel, voltage gated,
febrile illness and use of benzodiaz- subfamily Q, member 3 (KCNQ3) is also
epines, such as rectal diazepam gel. implicated in this disorder. Benign familial
neonatal convulsions are an autosomal
zz Suggested Reading dominant condition characterized by
55 Shinnar S, Glauser TA. Febrile Seizures. seizures which may start a few days after
J Child Neural. 2002;17:S44–52. birth, often persisting for 6 weeks. If they
do not occur within a few days of birth,
?? 134. Which of the following is most seizures are usually present by 8 weeks of
likely to be associated with epilepsy age, and may persist for up to 6 months.
of infancy with migrating focal Seizures may be either generalized or
­seizures? focal and development is usually normal.
A. Potassium channel, voltage Mutations in sodium channel, voltage
gated, subfamily Q, member 2 gated, type II, alpha subunit (SCN2A) are
(KCNQ2) implicated in benign familial infantile
B. Sodium channel, voltage gated, seizures 3 and early infantile epileptic
type II, alpha subunit (SCN2A) encephalopathy 11. Less commonly, this
C. Potassium channel, subfamily T, gene is implicated in severe myoclonic
member 1 (KCNT1) epilepsy of infancy, and generalized
D. Aristaless related Homeobox, epilepsy with febrile seizures plus.
X linked (ARX) Mutations in aristaless related homeo-
E. Cholinergic receptor, neuronal box (ARX), which codes for a homeobox
nicotinic, alpha polypeptide 4 transcription factor, are implicated in early
(CHRNA4) infantile epileptic encephalopathy 1. Cyclin
dependent kinase-like 5 (CDKL5), solute
vv Correct Answer is: C carrier family 25, member 22 (SLC25A22},
Epilepsy of infancy with migrating focal syntaxin binding protein 1 (STXBP1),
seizures is characterized by onset prior spectrin,alpha, nonerythrocytic 1 (SPTAN1},
to 6 months of age (with normal devel- sodium channel, voltage gated, type I,
opment until seizure onset) and the alpha subunit (SCN1A),sodium
presence of focal motor seizures which channel,voltage gated, type IX, alpha
ultimately become intractable and unre- subunit (SCN9A),potassium
sponsive to conventional anti­epileptic channel,voltage gated, subfamily Q,
medications. Psychomotor delay usually member 2 (KCNQ2),cdc42 guanine
follows seizure onset. nucleotide exchange factor 9 (ARHGEF9),
Mutations in the potassium channel, protocadherin 19 (PCDH19), polynucleo-
subfamily T, member 1 (KCNT1) gene tide kinase 3-prime phosphatase (PNKP},
result in persistent channel activity, sodium channel,voltage gated, type
thereby lowering the seizure threshold. II,alpha subunit (SCN2A}, phospholipase C,
Other genes implicated in this disorder beta 1 (PLCB1),sodium channel,voltage
include solute carrier family 25 (mitochon- gated, type VIII, alpha subunit (SCN8A),
drial carrier: glutamate), member 22 potassium channel, subfamily T, member 1
(SLC25A22) and phospholipase C, beta 1 (KCNT1), ST3 beta-galacstoside alpha-­2,3-­
(phosphoinositide-specific) (PLCB1). sialyltransferase 3 (ST3GAL3),tre2- bub2-­
320 Chapter 9 · Epilepsy 2

cdc16- domain family, member 24 in the ER is 38 C. No family history of


(TBC1024), guanine nucleotide­binding seizures. His CSF and brain MRI are
protein,alpha-activating polypeptide O normal.
gene (GNA01),mouse seizure threshold 2 Which of the following is associated
gene (SZT2),gamma-aminobutyri c acid with increased risk of future seizures?
(GABA) A receptor, alpha 1 (GABRA 1), A. No family history of epilepsy or
phosphatidylinositol glycan,class A (PIGA), febrile seizure
NECAP endocytosis-associated protein 1 B. Age over 2 years
(NECAP1), solute carrier family 35, member C. Long interval between onsets of
2 (SLC35A2), dedicator of cytokinesis 7 fever to onset of seizure
(DOCK7), hyperpolarization­activated cyclic D. Seizure duration more than
nucleotide-gated potassium channel 1 15 min
(HCN1), solute carrier family 13, member 5
(SLC13A5), SLIT-ROBO rho GTP-ase vv Correct Answer is: D
activation protein 2 (SRGAP2), and MADs Febrile seizures are generally associated
Box transcription enhancer factor 2, with temperature more than 38C in a
polypeptide C (MEF2C) are implicated in child between 3 months and 6 years of
early infantile epileptic encephalopathy. age with (1) no central nervous system
This typically starts in infancy; patients may infection (2) no acute metabolic or sys-
have tonic spasms, and burst suppression temic abnormalities and (3) no history
9 on EEG. of prior afebrile seizures. The child in
Cholinergic receptor, neuronal this case has a simple febrile seizure.
nicotinic, alpha polypeptide 4 (CHRNA4) is Simple febrile seizures last under 15 min
implicated in autosomal dominant with no seizure recurrence within 24 h
nocturnal frontal lobe epilepsy 1. The are typically generalized and may be
product of this gene regulates nicotinic clonic, atonic, or tonic. Complex febrile
acetylcholine receptor GABAergic seizures are characterized by duration
inhibition. Other genes implicated in more than 15 min, focal onset, recur-
autosomal dominant frontal lobe epilepsy rence within 24 h, or the presence of
include cholinergic receptor, nicotinic, baseline neurological deficits. Risk
beta 2 (Neuronal) (CHRNB2), cholinergic factors for recurrent febrile seizures
receptor, nicotinic, alpha 2 (neuronal) include age under 15 months, devel-
(CHRNA2), and potassium channel, opmental delay, time spent in daycare,
subfamily T, member 1 (KCNT1). shorter time between fever onset and
Autosomal dominant nocturnal frontal seizure, low grade fever at onset, and
lobe epilepsy typically starts in adoles- first degree relative with febrile seizures
cence or late childhood and is character- or another epilepsy type. In the absence
ized by predominately night time frontal of known risk factors, there is a 2.5%
lobe seizures. risk of epilepsy by the third decade. If
there are multiple risk factors, the risk
?? 135. A The mother of a 13-month-old boy may be as high as 17%.
brought him to the ER after she wit- Family history of epilepsy or febrile
ness a jerky movement in his arms seizures is associated with increased risk
and legs. The event lasts 1 min and of future febrile seizure. A child with
afterwards, the child seems tired and either a parent or sibling with febrile
uninterested in his surroundings. seizures has a risk 2 to 3 times higher of
The child has an upper respiratory febrile seizure than that of a child with no
tract infection and his temperature family history.
Epilepsy 2
321 9
Age under 15 months is associated addition to tonic clonic seizures. Genes
with increased risk of future febrile implicated in this disorder include
seizures. sodium channel, voltage-­gated, type I,
Shorter fever duration prior to onset of alpha subunit (SCN1A), sodium chan-
seizure is associated with increased risk of nel, voltage gated, type I, beta (SCN1
future seizures. B), gamma-aminobutyric acid (GABA) A
receptor, gamma 2 (GABRG2), gamma-
?? 136. A mother of a 12-year-­old girl with aminobutyric acid (GABA) A receptor,
a history of febrile seizures brought delta (GABRD), and sodium channel,
her to your office, with a concern voltage gated, type IX, alpha subunit
that her daughter had a generalized (SCN9A). Mutation affecting voltage
tonic clonic seizure in the absence gated KQT-like subfamily Q, member
of elevated temperature. The fam- 2 are implicated in benign familial
ily history is remarkable of several neonatal seizures and in early infan-
family members including the child’s tile epileptic encephalopathy. Muta-
biological father and sister have had tions affecting voltage gated KQT-­like
febrile and afebrile seizures. subfamily Q, member 3 (KCNQ3) may
Which of the following is most likely also cause benign familial neonatal
true? seizures.
A. The patient has a mutation Mutations in aristaless related
affecting voltage gated K ­ QT-­like homeobox (ARX) may be seen in early
subfamily Q, member 2 (KCNQ2) infantile epileptic encephalopathy, and X
B. The patient has a mutation linked lissencephaly.
affecting aristaless related Mutations affecting Gamma-­
homeobox (ARX) Aminobutyric Acid (GABA) A receptor,
C. The patient has a mutation alpha 1 (GABRA1) are thought to cause
affecting sodium channel, volt- some forms of early infantile epileptic
age-gated, type I, alpha subunit encephalopathy. Additionally, variations
(SCN1A) in this gene may increase one’s suscepti-
D. The patient has a mutation bility to juvenile myoclonic epilepsy.
affecting Gamma-Aminobutyri c Mutations affecting chloride channel,
Acid (GABA) A receptor, alpha 1 voltage-sensitive 2 (CLCN2) may increase
(GABRA1). one’s susceptibility to juvenile absence
E. The patient has a mutation epilepsy or juvenile myoclonic epilepsy.
affecting chloride channel, volt-
age-sensitive 2) CLCN2. ?? 137. A 5-month-old baby boy is brought
to your office by his mother who is
vv Correct Answer is: C concerned about abnormal move-
The clinical presentation is indicative ments. She reports that the child
of generalized epilepsy febrile seizures seems to “jerk” briefly. She states
plus (GEFS+). Patients with this condi- that these episodes are becoming
tion may have febrile seizures initially, more frequent and she is con-
but both febrile and afebrile seizures cerned that the child is uncomfort-
persist past age 6. It is an autosomal able. You ordered an EEG which
dominant disorder, so there is often showed hypsarrhythmia. Brain MRI
a known family history of febrile and brain is normal. The mother wants
afebrile seizures. Myoclonic, atonic, to know what she can do to treat
or absence seizures may be present in the condition.
322 Chapter 9 · Epilepsy 2

Which of the following is the best C oxidase deficiency, pyridoxine defi-


­initial therapy? ciency, dihydropteridine reductase
A. Vigabatrin deficiency, histidinemia, and a number
B. Valproic acid of disorders affecting the urea cycle.
C. Zonisamide Syphilis, cytomegalovirus and toxoplas-
D. Adrenocorticotropic hormone mosis are potential infectious etiologies
(ACTH) and hypoxic ischemic encephalopathy is
E. Topiramate another potential underlying cause.
Cryptogenic IS may be diagnosed in a
vv Correct Answer is: D child with no other seizure type, a normal
This a case of infantile spasms (IS). neurological exam, normal brain imaging,
IS generally starts between 3 and and normal development prior to symp-
7 months of age. It is characterized by tom onset. The recommended first line
a brief (often lasting under 2 s) muscle therapy for this condition is ACTH, often
contractions which may involve the given IM. Vigabatrin may also be given
extremities, trunk, and/or neck. Those although ACTH is deemed preferential as
contractions are followed by a tonic per American Academy of Neurology
component lasting up to 10 s. There is (AAN) Guidelines.
considerable variation between patients Vigabatrin is not recommended as first
and between seizures in a single line therapy for cryptogenic infantile
9 patient. Extensor spasms are character- spasms. However, in those with tuberous
ized by extension of the trunk and neck sclerosis this medication is particularly
with either adduction or abduction of efficacious. Unfortunately, vigabatrin can
the legs and/ or arms. Flexer spasms cause visual loss, something difficult to
may involve the neck, arms, legs, and/ monitor in infants. An initial ophthalmo-
or trunk. Mixed flexor extensor spasms logical exam is recommended with follow
may also occur as well. IS may occur up every 3 months.
with other conditions including Miller- The benefits afforded by valproic acid
Dieker syndrome, Aicardi syndrome, are unclear although this has been used.
hemimegalencephaly, tuberous sclero- Adequate clinical trials are lacking.
sis complex, nevus linearis sebaceous, Zonisamide may be effective,
incontinentia pigmenti, Ito syndrome, although there is insufficient evidence to
and neurofibromatosis type I. Addi- recommend this. Topiramate is also not
tionally, chromosomal abnormalities recommended as first line. Nitrazepam,
including trisomy 21 and 18q duplica- pyridoxine, IVlg, thyrotropin releasing
tion may occur in association with IS. A hormone, levetiracetam and the keto-
number of genes have been identified genic diet have also been considered as
which may play a causative role. Muta- possible treatments. Again, there is
tions in calcium/calmodulin-dependent insufficient evidence to recommend any
serine protein kinase (CASK), ALG13, of these treatments. ACTH remains first
pyridoxamine-5′-phosphate oxidase line.
(PNPO), adenylosuccinate lyase (ADSL),
aristaless-related homeobox (ARX) gene ?? 138. Which of the following is typical of
syntaxin-binding protein 1 (STXBP1), juvenile absence seizures?
and forkhead protein G1 (FOXG1) A. Patients often have more than
are associated with infantile spasms. 100 seizures per day
Metabolic conditions associated with B. The average age of onset is
IS include Menkes disease, cytochrome between 7 and 10 years
Epilepsy 2
323 9
C. Patients usually do not need medications that through inhibition
lifelong anti-epileptic drug treat- of sodium channels include phenytoin
ment and carbamazepine; the use of either
D. Most patients eventually have medication blocks repetitive action
generalized tonic clonic seizure potential firing. Rufinamide works by
prolonging voltage dependent sodium
vv Correct Answer is: D channel inactivation. Similarly, lacos-
Juvenile absence epilepsy tends to pres- amide works on voltage gated sodium
ent later than childhood absence epi- channels, enhancing slow inactiva-
lepsy- starting between 10 and 16 years tion. Felbamate inhibits voltage gated
of age, as opposed to between 3 and sodium channels and it blocks sodium
10 years of age. Up to 80% of patients and calcium conduction by binding to
with juvenile absence epilepsy may N-methyl-­0-aspartate (NMDA) gluta-
have a generalized tonic clonic seizure mate receptors. Lamotrigine works both
within 2 years of diagnosis. Variations in through blockage of sodium channels
the chloride channel, ­voltage-­sensitive and by inhibition of high threshold acti-
2 gene (CLCN2) and the G protein-cou- vated calcium channels.
pled receptor kinase 1 gene (GRK1) are Topiramate works through blockage of
implicated in this disorder. voltage activated sodium channels; this
People with juvenile absence epilepsy medication also acts at the kainate/
may have up to 3 absence seizures per alpha-­amino-­3 -hydroxy-5- methylisoxa-
day, while those with child absence zole-4-proprioni c acid (AMPA) glutamate
epilepsy may have between 25 and 50 receptor subtype, decreasing glutamate
seizures per day. mediated excitation. Topiramate also
Most patients with juvenile absence activates hyperpolarizing potassium
epilepsy do require lifelong treatment conduction, enhances chloride current at
with anti-­epileptic medications. GABAA receptors, and reduces high
Medications used for this disorder include voltage activated calcium current ampli-
lamotrigine, valproate, topiramate, tude.
zonisamide, and levetiracetam. This medication is also a carbonic
anhydrase inhibitor. Oxcarbazepine
?? 139. Which of the following anti-epileptic stabilizes cell membranes through
medications works primarily through blockage of sodium, potassium, and
sodium channel inhibition? calcium conduction. Zonisamide inhibits
A. Eslicarbazepine slow sodium channels, blocks T type
B. Ethosuximide calcium channels, and inhibits carbonic
C. Ezogabine anhydrase. This also works on GABAA
D. Phenobarbital receptors through alteration of ligand
E. Diazepam biding. Valproate works through sodium
channel inhibition and through attenua-
vv Correct Answer is: A tion of voltage sensitive calcium channels.
Eslicarbazepine is a voltage gated This is also an NMDA receptor antagonist.
sodium channel blocker. This is Ethosuximide works through attenua-
approved for management of partial tion of voltage sensitive calcium channels.
seizures (either with or without sec- This medication is indicating for the
ondary generalization) in patients at management of absence seizures in
least 6 months of age, although it is patients at least 3 years of age and is
not approved as a monotherapy. Other approved as a monotherapy.
324 Chapter 9 · Epilepsy 2

Ezogabine works through activation of with “swimming” movements of the


voltage gated potassium channels and upper extremities and thrashing
also potentiates GABA evoked currents. of the body. The spells last 30 s to
This is indicated for management of 5 min in duration and are followed
partial seizures in patients at least by somnolence and lack of recall
18 years of age and it is not approved as a for the event. There are no obvious
monotherapy. medical precipitants, and serum lev-
Phenobarbital works through els of lamotrigine have been in the
enhancement of chloride currents mid therapeutic range. An extended
mediated by GABAA receptors. This is video-EEG is obtained, demonstrat-
approved as a monotherapy in patients of ing occasional generalized poly-
all ages. phasic spike/wave discharges. One
Clonazepam works by enhancing of the episodes described above
chloride currents mediated by GABAA occurs during the recording and is
receptors. Clobazam, diazepam, and associated with muscle artifact, with
lorazepam have the same mechanism. resumption of normal waking pat-
Gabapentin decreases GABA turnover, terns after the event ends.
thereby increasing GABA levels. This Which of the following is the most
medication also inhibits monoamine appropriate next step in manage-
neurotransmitter release and binds to L ment?
9 type calcium channels at the a2o subunit. A. Replace lamotrigine with val-
Similarly, pregabalin inhibits voltage proic acid
gated calcium channels at the a2o B. Obtain psychiatry consultation
subunit. Vigabatrin increases GABA levels C. Increase dose of lamotrigine
in the brain through inhibiting GABA D. Start diphenhydramine
transaminase irreversibly. Tiagabine also E. Obtain brain MRI
increases GABA levels, by blocking
neuronal and glial GABA transporters. vv Correct Answer is: B
Stiripentol increases GABA release and There are a constellation of evidence
enhances the activation of GABAA that strongly suggest psychogenic non-
receptors. Perampanel works as an epileptic seizures, which is a form of
antagonist of the ionotropic AMPA conversion disorder. They are: Episodes
glutamate receptor on post synaptic are induced by stress, in certain settings,
neurons, thereby reducing neuronal or by specific individuals. Seizures recur
excitation. Levetiracetam binds to despite therapeutic antiepileptic drug
synaptic vesicle protein 2A and decreases levels. The ictus is of gradual onset and
influx of calcium through N type voltage of unusually prolonged duration. There
sensitive calcium channels. are thrashing/writhing movements, cry-
ing and/or screaming, pelvic thrusting,
?? 140. An 18-year-old female with idio- and rolling side-to-side. Movements
pathic generalized epilepsy well are intermittent, arrhythmic, out-of-
controlled with moderate-dose phase jerking, or with multiple periods
lamotrigine has a sudden onset of of starting and stopping.There is no
multiple paroxysmal episodes unlike loss of consciousness despite bilateral
those previously experienced. Her movements, and a lack of incontinence
family described a sudden startled or cyanosis. There is a lack of post-ictal
look, followed by whole-body stiff- confusion or lethargy. Presence of cry-
ening and arching in conjunction ing or shouting of obscenities. Eyes
Epilepsy 2
325 9
will be closed, and patients will tend to to repeat short phrases. Otherwise
resist having their eyes forced open. the rest of her neurologic examina-
The diagnosis of psychogenic seizures tion is normal. Formal audiometric
rests primarily on video-EEG documenta- testing is unremarkable. Her parents
tion of the non-epileptic nature of the state that she occasionally stares off
clinical episodes of concern. While into space for 30 s to 1 min. She has
considered the definitive diagnostic test, never had a convulsive seizure, nor
video-EEG testing does not have perfect does she have a family history of epi-
sensitivity and specificity. Some non- lepsy.
lesional focal epilepsies may be missed. An EEG is most likely to show which
Also, there is a high co-­morbidity between of the following patterns?
true epileptic seizures and psychogenic A. Bilateral periodic epileptiform
non-epileptic seizures (up to 30–40%), discharges (BiPEDs)
making it extremely difficult to ascertain B. Continuous, diffuse, polymorphic
what are or aren’t true epileptic events in theta and delta slowing
the usual clinical settings, and even in the C. Hypsarrhythmia with periods of
context of an emergency department visit. electrodecrement
The presentation of the diagnosis of D. Electrical status epilepticus in
psychogenic non-epileptic seizures must slow wave sleep
be made with great care and sensitivity. E. Normal waking and sleep
The possibility of sexual abuse should
always be considered. vv Correct Answer is: D
In 1957, William L. Landau and Frank
zz Suggested Reading R. Kleffner reported a “syndrome of
55 Bodde NM, Brooks JL, Baker GA, Boon acquired aphasia with convulsive dis-
PA, Hendriksen JG, Aldenkamp order in children” and postulated that
AP. Psychogenic non-epileptic seizures – persistent convulsive discharges were
diagnostic issues: a critical review. Clin causing a functional ablation of brain
Neurol Neurosurg. 2009;111(1):1–9. areas concerned with language. Since
55 LaFrance WC Jr. Psychogenic nonepileptic that time, it has become apparent that
seizures. Curr Opin Neurol. there are several epileptic encepha-
2008;21(2):195–201. lopathy syndromes of childhood that
are characterized by deterioration of
?? 141. You are evaluating a 7-year-old girl one or more cognitive functions with
for language difficulties. She was or without motor, behavioral, and/or
a product of a full-term uncompli- psychomotor decline in association with
cated pregnancy by normal sponta- an EEG that shows strong activation of
neous vaginal delivery. She walked epileptic activity during sleep. At this
at 10 months, said single words at point, electrical status epilepticus of
12 months, and combined words slow wave sleep (ESES) and continu-
at 22 months. At age 5, her speech ous spikes and waves during slow wave
became progressively more tele- sleep (CSWS) are considered synony-
graphic, and she began indicating mous terms, and Landau-Kleffner syn-
her needs by pointing or by using drome (LKS) a distinct presentation of
sign language. She can follow simple ESES/CSWS in which acquired aphasia is
commands. She has word-­finding the core symptom.
difficulties in spontaneous speech LKS typically presents with language
and on naming tasks and is unable disturbance in normally developing
326 Chapter 9 · Epilepsy 2

children aged 3 and 8 years. Some have B. Her child’s IQ may be lower with
argued that the language disturbance is in utero exposure to levetirace-
actually a verbal auditory agnosia (i.e. tam monotherapy
inability to comprehend spoken language C. Her child’s IQ may be lower with
in the absence of hearing difficulties). in utero exposure to lamotrigine
Over time, affected children experience a monotherapy
gradual deterioration in verbal compre- D. Her child’s IQ may be lower with
hension and production, eventually in utero exposure to carbamaze-
leading to failure to respond to non-verbal pine monotherapy
sounds and mutism. E. Her child IQ may be lower with in
Seizure semiology varies, and 20–30% utero exposure to all anti-epilep-
of patients do not exhibit clinical seizure tic drugs
activity at all. There is no clear relationship
between the severity of the seizures and vv Correct Answer is: A
the severity of the language deficits. EEG While divalproex’s association with physi-
shows predominantly bilateral posterior cal malformations following in utero expo-
temporal spikes or spike-wave discharges sure, the NEAD study’s 6 year follow up
that are activated by sleep. The clinical report also demonstrated that children
seizures are usually easy to control, with exposed to divalproex in utero are at risk
spontaneous resolution occurring in for cognitive differences compared to chil-
9 mid-adolescence. There are several dren exposed to other anti-epileptic drugs.
treatment options, including corticoste- The difference ranged from 8–11 IQ points.
roids, levetiracetam, benzodiazepines, and
valproic acid. Language function improves zz Suggested Reading
when the active phase of spike-wave 55 Meador KJ, Baker GA, Browning N,
discharges resolves; however, if the aphasia Cohen MJ, Bromley RL, et al. Fetal antiepi-
begins in early childhood and/or persists leptic drug exposure and cognitive
for more than 1–2 years, long-term outcomes at age 6 years (NEAD study): a
language deficits are almost universal. prospective observational study. Lancet
Neurol. 2013;12(3):244–52.
zz Suggested Reading
55 Smith MC, Hoeppner TJ. Epileptic ?? 143. You are seeing a 17-year-old boy
encephalopathy of late childhood. J Clin in the ED with a prolonged seizure
Neurophysiol. 2003;20(6):462–72. lasting 45 min. ED team could not
55 Van Bogaert P, Paquier PF. Landau-­ obtain an IV access.
Kleffner syndrome: 50 years after. What is the next best step in manage-
Epilepsia. 2009;50(S7):1–2. ment?
A. Phenytoin 1000 mg intramuscular
?? 142. An 18-year-old pregnant female B. Intubate the patient to secure
comes for evaluation of new onset the airway
epilepsy. She is concerned about the C. Insert an internal jugular central
possibility of cognitive difficulties line
in her child if she is taking seizure D. Midazolam 10 mg intramuscular
medications. E. Initiate a video EEG study
You counsel her:
A. Her child’s IQ may be lower with vv Correct Answer is: D
in utero exposure to divalproex A recent randomized controlled trial
monotherapy indicated that IM midazolam is equal to
Epilepsy 2
327 9
intravenous lorazepam for the treatment of ?? 145. You are seeing a 7-month-old baby
status epilepticus. Phenytoin should not be in the ED with new onset of sei-
given intramuscularly because the risk of zures. You ordered an EEG which
purple glove syndrome. The other choices showed hypsarrhythmia.
are important considerations in the man- What is your next best step in man-
agement of this patient, but treatment with agement?
a benzodiazepine should not be delayed. A. Routine pediatric neurology con-
sultation
zz Suggested Reading B. Treat with adrenocorticotropic
55 Silbergleit R, Durkalski V, Lowenstein D, hormone (ACTH)
Conwit R, Pancioli A, Palesch Y, Barsan C. Lumbar puncture
W. NETT Investigators. Intramuscular D. Referral for corpus callosotomy
versus intravenous therapy for prehospital E. Levetiracetam 10 mg/Kg/dose
status epilepticus. N Engl J Med.
2012;366(7):591–600. vv Correct Answer is: B
Hypsarrhythmia on the EEG is part of the
?? 144. You are evaluating a 16-month-old diagnosis for infantile spasms. Prompt
girl in the ER after her first seizure. The treatment with ACTH is recommended.
mother describes this event with the
girl is stiffening up and then rhythmi- zz Suggested Reading
cally shaking for 4 min. On arrival she 55 Wilmshurst JM, Gaillard WD, Vinayan KP,
has a temperature of 39 °C (102.2 °F). Tsuchida TN, Plouin P, et al. Summary of
After treatment with antipyretics, the recommendations for the management of
patient is now awake and alert and infantile seizures: Task Force Report for
routine laboratory results are normal. ILAE Commission of Pediatrics. Epilepsia.
What is your next best step in man- 2015;56(8):1185–97.
agement?
A. Discharge patient with pediatric ?? 146. You are seeing a 16-month-old boy for
neurology follow up his an unprovoked generalized tonic
B. Lumbar puncture and empiric clonic seizure (GTC). Previously, he had
antibiotics 2 complex febrile seizures. They were
C. Video EEG monitoring for 24 h prolonged alternating unilateral con-
D. MRI brain with and without vulsions. His mother reported recent
gadolinium some jerking movements of his arms.
E. Admit for observation for 23 h This jerking occurred as i­ncreased clus-
ters prior to onset of the GTC. Recently
vv Correct Answer is: A he has also increased falls. While he is
This is a classic presentation of a child in the hospital, he underwent an EEG
with a simple febrile seizure. Febrile sei- and MRI of the brain, both of which are
zures occur in 2–5% of the population. normal. One day later, he is awake, but
Work up is typically only be initiated in has some mild truncal ataxia on exam.
febrile seizures with focal seizure or in Which gene is responsible for his
the setting of febrile status epilepticus. condition?
A. CACNA1A
zz Suggested Reading B. DMPK gene
55 Gupta A. Febrile Seizures. Continuum C. KCNQ2
(Minneap Minn). 2016;22(1 D. SCN1A
Epilepsy):51–9. E. TSC1
328 Chapter 9 · Epilepsy 2

vv Correct Answer is: D with phenobarbital, levetiracetam


The child has Dravet syndrome which and oxcarbazepine. He has micro-
is one of the genetic epileptic encepha- cephaly and developmental delay.
lopathies. Usually, unprovoked seizures CSF reveals a cell count of 3, protein
start in the first or second year of life in of 25 ug/dL and glucose of 20 mg/
an otherwise healthy infant. However, dL. An extensive metabolic work-up
recurrent spells of hemiconvulsive was done immediately prior to the
febrile status epilepticus in the first procedure was normal including a
year of life may delay the diagnosis. glucose level of 85 mg/dL. His brain
The term “severe myoclonic epilepsy of MRI is normal.
infancy (SMEI)”, has been not used any What would be the most appropriate
more because there are too many cases treatment for his intractable seizures?
are being identified without obvious A. Corpus callosotomy
myoclonic events. Classical, the child B. High dose nightly benzodiaz-
has normal development at the onset epine usage
of seizures, though most patients will C. Ketogenic diet
have very obvious delays in motor, lan- D. Topiramate
guage and cognitive development by E. Vagal nerve stimulation
3–4 years of age. Progressive ataxia has
is a common finding in this syndrome. vv Correct Answer is: C
9 Up to 80–90% of cases meeting clinical The most likely diagnosis of this child
criteria of Dravet syndrome will have a is glucose transporter type 1 (GLUT-1)
mutation identified in the SCN1A gene deficiency. The diagnosis is based on
region. the combination of intractable seizures
Mutations in the CACNA1A region lead and developmental delays in infancy or
to multiple phenotypes including sporadic childhood with hypoglycorrhachia (low
and familial hemiplegic migraine, episodic glucose in the CSF) in the absence of any
ataxia and spinocerebellar ataxia, type 6. other abnormality. Almost all patients
KCNQ2 mutations typically lead to benign have a CSF glucose of less than 40 mg/
familial neonatal convulsions, though a dL, and a ratio of glucose in the CSF/
more aggressive phenotype with a severe blood ratio less than 0.33 (normal range
epileptic encephalopathy has been is 0.5–0.67). The above phenotypic pre-
described (though without myoclonic sentation is the most common, though
seizures or prolonged hemiconvlusive milder cases may present with complex
febrile status). TSC1 mutations lead to movement disorders and other parox-
tuberous sclerosis, unlikely given the ysmal symptoms triggered by fasting
normal MRI scan in this patient and DMPK or exercise. A majority of the patients
mutations are seen in myotonic dystrophy. have a de-novo mutation in the SLC2A1
gene, leading to impaired active trans-
zz Suggested Reading port of glucose into the CSF through
55 Dravet C. The core Dravet syndrome the glucose transporter. A variety of
phenotype. Epilepsia. 2011;52(s2):3–9. seizure types are seen and although
EEG is frequently abnormal, no specific
?? 147. You are seeing a 6-month-old baby pattern is diagnostic. Similarly, brain
boy for his intractable seizures and imaging is either normal or non-specific.
encephalopathy. He started to have The ketogenic diet is highly effective in
seizure at 2 months of age and both control of seizures and prevention
have been unresponsive to therapy of progressive mental decline, and early
Epilepsy 2
329 9
institution of the diet is recommended habituate with repetition of the stimu-
as soon as the diagnosis is established. lus. Symptoms include massive flexion
Commercial testing of the SLC2A1 gene of the neck (severe enough to at times
is available for confirmation. compromise the airway of the infant)
and trunk with abduction and flexion
zz Suggested Reading of the arms. It can also result in tonic
55 DeGiorgis, V, Veggiotti, P. GLUT1 muscle spasms. They may resemble
deficiency syndrome 2013: Current state of tonic seizures, but the consistent ability
the art. Seizure. 2013;(22):803. to be provoked by a stimulus is atypical
55 Klepper, J. GLUT1 deficiency syndrome in for seizures. An EEG would be expected
clinical practice. Epilepsy Research. to show only movement artifact during
2012;(100):272. the spells. The disorder most commonly
is due to mutations in the glycine recep-
?? 148. You are seeing a 5-month-old boy tor alpha1-subunit (GLRA1) gene and
with history of spasms. His mother inherited as an autosomal dominant
reports that he is very sensitive to trait (though autosomal recessive and
any noise and reacts with an exag- x-linked inheritance has been reported.)
gerated startle response with muscle Mutations typically result in deficient
spasms involving the whole body. production and activity of glycine (a
The symptoms will recur if the stim- major inhibitory neurotransmitter) in
ulation has been repeated. It was the central nervous system. Anti-GAD
becoming increasingly difficult for antibodies are not seen in this disor-
her to take care of him as he would der, rather being best known for their
become very stiff every time she presence in “stiff-person syndrome.”
picks him up for change of clothes or CSF exam is typically normal in these
to feed him. His muscle tone appears patients. The condition can improve
normal when lying in bed by himself. over time, but benzodiazepines, such
What is the correct answer? as clonazepam, provide effective symp-
A. Serum anti-glutamic acid decar- tomatic treatment for this condition.
boxylase (GAD) antibodies will
be positive. zz Suggested Reading
B. EEG will show generalized 3 Hz 55 Espay A, Chen R. Myoclonus. Continuum
spike and wave activity. Lifelong Learn Neurol. 2013;19(5):1264.
C. CSF will be positive for 14-3-3 55 Bode A, Lynch J. The impact of human
protein. hyperekplexia mutations on glycine
D. Autosomal recessive inheritance receptor structure and function. Molecular
is most common. Brain. 2014;7:2.
E. It involves mutation in the gene
for alpha-1 subunit of the glycine ?? 149. You are evaluating a 15-year-old
receptor. boy for his epilepsy. His seizures
stared 5 months ago and he is on
vv Correct Answer is: E carbamazepine. He has a total of 8
This is the clinical presentation of exag- seizures. His seizure occurs about
gerated startle disease or hyperek- 1–2 h after waking from sleep, either
plexia. The main feature to differentiate early in the morning and or after
from a normal infantile startle response awakening from a midday nap. He
is that the event with hyperekplexia is has no loss of consciousness and no
markedly exaggerated and does not jerky movements of the hands have
330 Chapter 9 · Epilepsy 2

been reported. No family history of discharges can be present on EEG,


epilepsy. His exam and brain MRI are photosensitivity is rare. Without a
normal. EEG shows 3–4 Hz general- suggestive family history, a diagnosis of
ized spike wave discharges and pho- ADNFLE would be hard to confidently
tosensitivity. make. The patient has not been identified
What is the most accurate diagnosis? to have myoclonic seizures, and has not
A. Autosomal dominant nocturnal have the typical progressive cognitive
frontal lobe epilepsy (ADNFLE) decline seen with the disorders leading to
B. Epilepsy with grand mal seizures a progressive myoclonic epilepsy.
on awakening
C. Left frontal lobe epilepsy zz Suggested Reading
D. Juvenile myoclonic epilepsy 55 Janz D. Epilepsy with grand mal on
E. Progressive myoclonic epilepsy awakening and sleep-waking cycle. Clin
Neurophysiol. 2000;111(S2):S103–10.
vv Correct Answer is: B 55 Rudzinski LA, Shih JJ. The classification of
Epilepsy with grand mal seizures upon Seizures and epilepsy syndromes.
awakening is an idiopathic generalized Continuum 2010;16(3):15–35.
epilepsy (IGE) syndrome characterized
by generalized tonic clonic seizures ?? 150. You are seeing a 6-month-old baby
occurring predominantly on awakening in the ED for his left arm shaking sei-
9 (independent of the time of day.) The zure which became generalized and
typical EEG findings of an IGE including lasted for 15 min. Birth and delivery
predominantly generalized spike and were normal. He has reached all
slow waves (though focal abnormalities developmental milestones on time.
can also be noted) and photosensitiv- There is a family history of febrile
ity. The syndrome shares too many seizures in his father. On examina-
manifestations with the more common tion, he smiles and coos, but does
juvenile myoclonic epilepsy, including not reach with his left hand. On
similar provoking features (sleep depri- examination, he has a right otitis
vation, alcohol use and bright flickering media and is febrile.
lights), but the diagnosis of JME would Which of the following is consistent
typically require either clinical absence with a simple febrile seizure in this
or myoclonic events in addition to patient?
generalized tonic-clonic seizures. Treat- A. Age of the patient
ment for both conditions would B. Family history
involve broad spectrum agents such as C. Length of seizure
lamotrigine, levetiracetam, sodium val- D. Neurologic examination
proate, topiramate or zonisamide. Car- E. Seizure semiology
bamazepine (narrow spectrum sodium
channel blocking agent) is relatively vv Correct Answer is: A
contraindicated. Febrile seizure is very common in
Though frontal lobe seizures can occur the young child, with an age onset of
during sleep-wake transitions, they most onset 6 months to 5 years of age (other
commonly occur out of sleep, and not author’s reported onset as young as
1–2 h after awakening. Additionally, 3 months and as late as 6 years). Simple
although very rapidly generalizing febrile seizures are characterized by
Epilepsy 2
331 9
seizures accompanied by fever that seizure in his 20s after diving into
are generalized in onset, last less than cold water. A paternal uncle had
15 min, and do not occur more than childhood seizures and died while
once in 24 h. Febrile seizures are com- swimming as a teenager. She has
plex due to either longer duration, focal a normal general and neurological
signs at onset, or recurrence within examinations.
24 h. Achievement of normal neurode- What is the most urgent diagnostic
velopmental milestones and a normal test?
neurologic exam are required to classify A. 12-lead electrocardiogram
the episode as a febrile seizure. Promi- B. Transthoracic echocardiogram
nent encephalopathy prior to or after C. Routine EEG
the seizure should raise suspicion of a D. MRI brain
secondary cause such as a central ner- E. Tilt table test
vous system infection and focal onset
seizures with post-ictal hemiparesis vv Correct Answer is: A
may suggest a structural cause such as The patient has a spell of exercise-
tumor, stroke or abscess. Although the induced syncope with convulsions. Her
genetics of febrile seizures is still not family history is suggestive of autosomal
well known (other than for syndromes dominant pattern of convulsions possibly
such as generalized epilepsy febrile induced by cold water immersion. This is
seizures plus- GEFS+), a family history most concerning for a congenital long QT
of such events is very common, though syndrome and a 12-lead electrocardio-
not effective in differentiating between gram (EKG) is necessary for diagnosis. If
simple and complex febrile seizures. normal, the patient should have exercise
testing or be referred to pediatric cardiol-
zz Suggested Reading ogy, as occasionally the initial resting EKG
55 Waruiru C, Appleton R. Febrile seizures: a can be normal. Majority of congenital
consensus of their significance, evaluation, long QT syndrome patients have a normal
and treatment. Consensus development structural heart exam on echocardiog-
conference of febrile seizures. 1980. raphy so this is not a high priority test
National Institute of Health. in this population. A routine EEG may
Pediatrics.1980;66:1009–12. be warranted if the EKG is normal or the
55 Shinnar S, Glauser TA. Febrile Seizures. patient has recurrent episodes outside
J Child Neuro. 2002:17(1):s44–52. the setting of exercise. Typically, however,
convulsions associated with long QT
?? 151. You are seeing a 10-year-old girl syndrome are due to anoxic seizures, and
for her loss of consciousness and would not be likely to reveal any interictal
convulsions. She was exercising and abnormalities on EEG. Likewise, brain MRI
had sudden loss of consciousness is not indicated for this condition but may
with witnessed convulsions and be indicated if the patient is felt to have
urinary incontinence. Few minutes epilepsy rather than anoxic seizures. A tilt
later, she regaines her consciousness table test may be helpful is cases neuro-
and reports no recall of any warn- cardiogenic syncope due to orthostatic
ing symptoms. Her developmental hypotension. Patients with this etiology
history is normal. Her family history for syncope do not typically have syncope
is remarkable of her father had a during active phases of exercise.
332 Chapter 9 · Epilepsy 2

zz Suggested Reading phenobarbital possibly reduces cogni-


55 McLeod KA. Syncope in childhood. Arch tive outcomes. The AAN Practice Guide-
Dis Child. 2003; 88(4):350–3. line recommends, if possible, avoidance
of valproate during the first trimester
?? 152. You are following an 18-year-old of pregnancy to decrease the risk of
woman with juvenile myoclonic major congenital malformations (Level
epilepsy for preconception plan- B) and throughout pregnancy to prevent
ning. When she has her menarche, reduced cognitive outcomes (Level B).
she had her first seizure. She has If possible, avoidance of phenytoin and
not had a seizure in 3 years. She is phenobarbital during pregnancy may
on valproic acid 500 mg twice a day be considered to prevent reduced cog-
and folate 1 mg daily. She is using nitive outcomes (Level C). There is also
a barrier method of contraception. evidence of potentially less risk with
She has no side effects. Her neuro- other broad-spectrum AEDs, lamotrigine
logic examination, EEG, and MRI are and levetiracetam, making one of these
­normal. agents a better choice for this patient.
Based on evidence-­based medicine, There is no evidence that seizure con-
what is appropriate advice to give trol is likely to worsen during preg-
her at this point regarding pregnancy nancy although it may for a minority
planning? of women. Although increased folate is
9 A. increase dose of valproic acid as recommended during pregnancy, there
her seizure control will worsen is a paucity of evidence-­based medicine
during pregnancy to recommend a specific dose although
B. increase folate dose to 5 mg 4 mg a day is commonly used.
daily and proceed with plans for
pregnancy zz Suggested Reading
C. she should not have children due 55 Harden CL, Meador KJ, Pennell PB, et al.
to risk of teratogenicity Practice parameter update: Management
D. substitute extended release val- issues for women with epilepsy–focus on
proic acid, which will decrease pregnancy (an evidence-based review):
risk of teratogenicity Teratogenesis and perinatal outcomes:
E. transition to another broad-spec- report of the Quality Standards
trum antiepileptic drug prior to Subcommittee and Therapeutics and
pregnancy Technology Assessment Subcommittee of
the American Academy of Neurology and
vv Correct Answer is: E American Epilepsy Society. Neurology.
Valproate (VPA) exposure in the first 2009;73:133–41.
trimester has a higher risk of major 55 Hernández-Díaz S, Smith CR, Shen A,
congenital malformations and likely et al. Comparative safety of antiepileptic
reduced cognitive outcomes. In addition, drugs during pregnancy. Neurology.
monotherapy exposure to phenytoin or 2012;78(21):1692–9.
333 10

Ethics and Professionalism

© Springer International Publishing AG, part of Springer Nature 2018


Y. M. Awaad, Absolute Pediatric Neurology, https://doi.org/10.1007/978-3-319-78801-2_10
334 Chapter 10 · Ethics and Professionalism

?? 1. A 7-year-old boy is admitted to the approach to migraine would have


ICU after an out-of-­hospital cardiac likely resulted in resolution of his
arrest. He was being treated for symptoms prior to discharge mak-
vomiting and headache that became ing only rescue medications nec-
daily after initially being intermit- essary. You suggest they withdraw
tent, paroxysmal and focal on the support since he is brain dead on
right side. There was a family history your exam.
of migraine and he reported visual B. “Your child is dead according to
blurring at the onset of vomiting my tests…” You explain that after
and headache in the left visual field. cardiac arrest loss of pupillary
He had a non-contrast CT scan that light response is associated with
was unremarkable and no sign of death or vegetative state 95% of
intercurrent illness. He was initially the time and he has not improved
treated with sumatriptan but as this so he cannot recover. The EEG
produced no real relief he was hospi- confirms your opinion.
talized and given ondansetron with C. “Let me summarize what I know
analgesia. After 2 days in the hospital about your son…” You describe
he was discharged slightly improved the history as you know it and
on scheduled ondansetron and acet- then your exam and test results
aminophen. The following morning describing how you arrive at the
he was discovered pulseless in his conclusion the child is brain dead.
bed. His father began CPR and emer- After asking for questions you
10 gency medical technicians arrived give your opinion that support
within 5 min. He was intubated and should be withdrawn.
ECG showed ventricular fibrillation. D. To be sure we understand each
By the time he was transported to the other, could you tell me what you
ER at 30 min he had a sinus rhythm understand is going on with your
and blood pressure. His exam in the son…” You listen to the family’s
ER showed no pupillary light reflex ideas and answer any questions
and absent corneal reflex, extraocular that arise from your opening
movements, and gag. CT scan was statement and then explain your
normal except for indistinct gray- exam and the EEG results. After
white differentiation. An EEG at 10 h allowing questions and discus-
post-arrest showed electrocerebral sions with the family you state
silence. Evaluation for cause of his your recommendation that sup-
arrest was negative except for the port be withdrawn.
presence of a prolonged QT inter- E. “The studies and my exam show
val of 0.48 suggesting he may have there is no hope…” You present
acquired long QT syndrome second- the data from the CT scan, EEG
ary to ondansetron. A repeat EEG and your exam and state that the
shows electrocerebral silence and his child is brain dead. You wait for
exam is unchanged. questions and when there are
What is your approach with the none excuse yourself.
­family?
A. “If only I had seen this child…” You vv Correct Answer is: D
explain that you never use ondan- The approach in “d” addresses many
setron for nausea and vomiting in of the issues that have been identified
the setting of migraine and your in the literature on end-of-life care. In
Ethics and Professionalism
335 10
the case of children the parents’ views zz Suggested Reading
of their and their child’s rights to act 55 Schwani H, Wenrich MD, Tonelli MR,
autonomously is the foremost principle Curtis JR. Meeting physician’s responsibili-
of contemporary medical ethics, but the ties in providing end-of-life care. Chest.
importance of autonomous choice to 2008;133:775–86.
individual parents varies greatly, with
some preferring to delegate or share ?? 2. A 16-year-old girl whose parents are
decision making with family members or friends of yours is evaluated for possi-
physicians. Physicians need to determine ble seizure activity. She was observed
each family’s preferences in this regard. to have episodes of staring, morning
This is one of many areas where commu- jerks, and had one convulsion. A diag-
nication with the family is important. The nosis of juvenile myoclonic epilepsy
list shows the areas identified in reports (JME) was made and confirmed on
(see reference) that are crucial to good EEG. Additional testing revealed that
ICU communication. she tested positive for marijuana and
Identify a private place for commu- cocaine. She was also found to have
nication with family members genital herpes. The patient asks that
Increase proportion of time spent her parents not be told of her drug or
listening to family rather than talking sexual activities.
Identify commonly missed opportu- Which of the following is the most
nities appropriate management?
55 Listen and respond to family members A. Begin treatment for JME with val-
55 Acknowledge and address family proate and inform her parents of
emotions the social problems described.
55 Explore and focus on patient values B. Begin treatment for JME with
and treatment preferences valproate and consult your state
55 Explain the principle of surrogate law on disclosing information to
decision making to the family parents.
55 Affirm no abandonment of patient and C. Begin treatment for JME with
family lamotrigine and inform her
parents of the social problems
Assure family that the patient will not described.
suffer D. Begin treatment for JME with
Provide explicit support for deci- lamotrigine and consult your state
sions made by the family law on disclosing information to
Use of the VALUE mnemonic during parents.
family conferences: E. Refrain from treatment of her JME
55 V = value statements made by family until social problems described
members are adequately resolved.
55 A = acknowledge emotions
55 L = l isten to family members vv Correct Answer is: D
55 U = u  nderstand who the patient is as a Juvenile myoclonic epilepsy and patient
person confidentiality Discussion: The patient
55 E = elicit questions from family presents with a relatively obvious
members epileptic syndrome. Given her sexual
proclivities, the use of valproate should
Provide consistent communication from be avoided because of its potential tera-
different team members togenic effect. The main issue has to do
336 Chapter 10 · Ethics and Professionalism

with what to tell the parents. Child neu- t­ esting and screening of children. A sum-
rologists might disagree as to whether we mary of the statement:
think the patient’s welfare is best served 1. Decisions about whether to offer
by discussing the problems with the par- genetic testing and screening should be
ents or abiding by the secrecy requested driven by the best interest of the child.
by the patient. However, we need to be 2. Genetic testing is best offered in the
aware laws vary from state to state and context of genetic counseling.
need to be carefully addressed before 3. In a child with symptoms of a genetic
decisions are applied. condition, the rational for genetic
testing is similar to that of other
zz Suggested Reading medical diagnostic evaluations.
55 Weddle M, Kokotailo P. Adolescent 4. When performed for therapeutic
substance abuse: confidentiality and purposes, pharmacogenetics testing
consent. Pediatr Clin North Am. of children is acceptable, with
2002;49(2). permission of parents or guardians
and, when appropriate, the child’s
?? 3. A 7 year old presents to your office assent.
with headaches. The symptoms are 5. The AAP and ACMG support the
consistent with migraines. Family mandatory offering of newborn
history is negative for migraines but screen for all children.
mom states that the child’s father was 6. The AAP and ACMG do not support
recently diagnosed with Huntington’s the routine carrier testing in minors
10 disease. Although the child is asymp- when such testing does not provide
tomatic, mom would like genetic health benefits in childhood.
­testing. 7. For pregnant adolescents or for
The best response should be: adolescents considering reproductive,
A. The child is not at risk for Hunting- genetic testing and screening should
ton’s disease be offered as clinically indicated.
B. If testing is done, the child should 8. Parents or guardians may authorize
not be informed of the results predictive genetic testing for asymp-
until he/she reaches adulthood tomatic children at risk for childhood-­
C. Since there is no cure for Hun- onset conditions.
tington’s disease, the testing 9. Predictive genetic testing for adult-
should never be done until a child onset conditions generally should be
reaches adulthood deferred unless an intervention
D. The testing should not be done until initiated in childhood may reduce
the child becomes symptomatic morbidity and mortality. An exception
E. The recommendations are that might be made if diagnostic uncer-
predictive genetic testing for tainty poses a significant psychosocial
adult onset conditions gener- burden, particularly when an adoles-
ally should be deferred unless an cent and his or her parents concur in
intervention initiated in childhood their interest in predictive testing.
may reduce morbidity or mortality 10. For ethical and legal reasons, health
care providers should be cautious
vv Correct Answer is: E about providing predictive genetic
The American Academy of Pediatrics testing to minors without the involve-
published a policy statement in 2013 on ment of their parents or guardians,
the Ethical and Policy Issues in Genetic even if a minor is mature.
Ethics and Professionalism
337 10
11. Tissue compatibility testing of minors a full and tense anterior fontanelle.
of all ages is permissible to benefit Pupils are small and reactive; corneal
immediate family members but and gag reflexes are present. Tone is
should be conducted only after a decreased, with little spontaneous
thorough exploration of the psycho- movement, but withdrawal to pain.
social, emotional and physical Reflexes are brisk throughout with
­implications. 4–6 beats of clonus at both ankles.
12. The rationale for genetic testing of CT of the head reveals acute on
children in biological families should chronic subdural hemorrhages bilat-
apply for adopted children awaiting erally, with patchy areas of intrapa-
placement for adoption. renchymal edema. Chest x-ray shows
13. At the time of genetic testing, parents multiple rib fractures in various stages
and guardians should be encouraged of healing. The child is admitted to
to inform their child of the test results the pediatric intensive care unit. After
at an appropriate age. Under most receiving a brief update on the child’s
circumstances, a request by a mature condition, the father asks to be alone
adolescent for test results should be with his son.
honored. Which of the following would be the
14. Results from genetic testing of a child most appropriate response to his
may have implications for the parents request?
and other family members. A. advise the father to retain legal
15. The risk of misattributed paternity, counsel
use of donor games, adoption, or B. further testing, including MRI of
other questions about family relation- the brain, need to be performed
ships may be uncovered “incidentally” to exclude non-accidental head
whenever genetic testing is trauma (i.e., child abuse) prior to
­performed. allowing his visitation
C. given the nature of the injuries
zz Suggested Reading noted, specialists in child protec-
55 Ethical and policy issues in genetic testing tion will need to further assess
and screening of children. 2013. AAP. the situation to determine who is
eligible to be left alone with the
?? 4. A 6-month-old male infant is brought child
to the emergency room by his father D. inform the father he is suspected
after suffering a seizure. The father of child abuse and will not be
states he has been more irritable than allowed to see his son
usual since waking from his nap, then E. inform the father that his son
suddenly became unresponsive and can have no visitors until his sei-
stiffened all over, followed by 2 min of zures have been controlled for at
generalized clonic jerking. He has not least 24 h
been awake since the event. While in
the emergency room, he has 2 more vv Correct Answer is: C
short seizures, and is given 20 mg/ When treating young children with
kg of phenobarbital. Per the father, injuries suspected to be secondary to
he has had no prior seizures and only nonaccidental trauma (i.e., child abuse),
a history of reflux, for which he takes the main goal of the treating physician
ranitidine. On examination, he is som- is to stabilize the patient and focus on
nolent, with little responsiveness, has treatment of the affected organ systems.
338 Chapter 10 · Ethics and Professionalism

It is not the job of the clinician to assign D. proceed with the EMG, explain-
blame to caregivers, or make primary ing the boy is not of age to give
determinations over safety of caregiver informed consent
contact, either during the ongoing hos- E. proceed with the EMG, explain-
pitalization or regarding discharge plan- ing that the risks of the test are
ning. It is also imperative to remember ­minimal
that there are rare cases of metabolic or
genetic disorders that can mimic injury vv Correct Answer is: A
produced by nonaccidental injury. Physi- Electromyography, which is a painful
cians with expertise in the field of child test, should be performed only when
abuse are often employed to assist with the patient consents to the procedure.
reviewing the history, physical exam At issue is not whether family members
findings and response to treatment to disagree, but who has the capacity and
determine if one of these rare masquer- the right to make a medical decision.
ading conditions exists. Although a his- The need for a diagnosis does not take
tory needs to be obtained by the treating precedence over informed consent, and
physicians, detailed questioning of the competent patients have a moral and a
caregiver(s) should be left to law enforce- legal right to refuse diagnostic (or thera-
ment and child protective agents. peutic) interventions. Although the boy
is not of legal age (usually 18, except for
zz Suggested Reading emancipated minors) to give informed
55 Swaiman K, Ashwal S, Ferriero DM, consent, at 16 he is likely developmen-
10 Schor N. Swaiman’s pediatric neurology. tally mature enough to have the capacity
5th ed. Philadelphia: Elsevier; 2012. to participate in medical decision mak-
p. 1126–48. ing and is capable of giving, or refusing,
assent. Assent is the capacity to agree to
?? 5. A 16-year-old boy presents with a medical procedure. A minor’s refusal to
weakness in his left arm, following a assent should be weighed against how
motorcycle accident 6 weeks ago. He essential the test result is to the minor’s
is referred for an EMG due to concern welfare. Honoring the boys wish, in this
that he may have a brachial plexopathy. case, is likely to foster trust and a better
He is with his mother, who is anxious to partnership with his physicians.
see what findings the EMG will reveal.
After explaining the procedure to the zz Suggested Reading
boy, he refuses to undergo the test. 55 Evans P. Assent and refusal by children
Which of the following choices is and adolescents. In: Williams MA,
ethically advisable? McGuire D, Rizzo M, editors. Practical
A. cancel the EMG, explaining ethics in clinical neurology: a case-based
that the boy’s wishes should be learning approach. Philadelphia:
­honored Lippincott Williams & Wilkins; 2012.
B. cancel the EMG, explaining that p. 149–50.
diagnostic testing cannot be
provided when family members ?? 6. A 12-year-old girl is evaluated in the
disagree ED and diagnosed with bacterial men-
C. proceed with the EMG, explain- ingitis. Her parents are divorced; her
ing that what is most important is mother, who has sole custody, refuses
ascertaining the diagnosis treatment.
Ethics and Professionalism
339 10
Which of the following is the best manual/manual6th.htm (7 http://www.
course of action in this case? acponline.org/runni ng_practice/ethics/
A. A court order should be sought manua l/manual6th.htm) 7 http://pediat-
for treatment rics.aappublications.org/content/128/
B. The mother’s wishes should be Supplement_4/5167.full (7 http://pediat-
respected, but the child should rics.aappublications.org/content/128/
be admitted for observation and Supplement_4/S167.full).
placed in isolation
C. Treat the patient without regard ?? 7. A 17-year-old male patient arrives at
for the mother’s wishes the ED in police custody, suspected
D. The health care provider can- of ingesting illicit medications to
not treat the patient in this case avoid arrest. The patient was arrested
unless consent is obtained from in a raid on an apartment where he
the father has lived for several months with 2
E. An ethics consult should be friends. The police request a naso-
obtained before treating the gastric aspirate for evidence, but the
patient patient refuses.
Which of the following is correct?
vv Correct Answer is: C A. The health care provider should
If there is a clear and imminent danger comply with the request because
to the life of a child, with a risk of death ingestion could result in death for
as a result of a delay in treatment, the the patient
health care provider can proceed with B. The health care provider is legally
treatment without judicial review against obligated to comply with the
the wishes of the custodial parent. In evidentiary request in a criminal
divorce, only parents with custody of the investigation
child are able to consent to treatment. If C. The health care provider should
both parents have custody, only one par- honor the patient’s wishes
ent must give consent. The health care D. The health care provider should
provider’s decision should be based upon seek a court order to pass a naso-
the best interest of the patient, so if treat- gastric tube
ment is required urgently, the health care E. Parental permission should be
­provider can proceed. obtained to treat the patient, but
evidence should not be supplied
zz Suggested Reading to the police against the patient’s
55 7 http://www.educus.com/ wishes
Journals/12612264.
55 7 http://pediatrics.aappublications.org/ vv Correct Answer is: C
content/111/3n03.long 7 http://pediatrics. Minors who live apart from their parents
aappublications.org/content/92/2/290. who are older than age 13 are considered
Abstract key=66ab600cb23cd0b00b7b3505 emancipated minors and are authorized
0f371bcfff895a50&keytype2=tf _ipsecsha to consent to medical care. This includes
(7 http://pediatrics.aappublications.org/ refusal of care. The health care provider
content/92/2/290. abstract? ij key=66ab600 should inform the patient of the risks of
cb23cd0b00b7b35050f371bcfff895aSO&ke refusal of care and should also tell the
ytype2=tf ipsecsha) 7 https://www. patient that evidence will be collected for
acponline.org/running_practice/ethics/ the police if the patient consents to care.
340 Chapter 10 · Ethics and Professionalism

A health care provider cannot perform a 7 http://pediatrics.aappublications.org/


medical procedure at the direction of the content/128/Supplement_4/5167.full.
police without a patient’s consent. 55 Fischer C, Oneto C. USMLE medical
ethics: The 100 cases you are most likely to
zz Suggested Reading see on the exam. New York: Kaplan; 2009.
55 7 https://www.acponline.org/running_prac-
tice/ethics/manual/manua16th.htm

10
341 11

Headache

© Springer International Publishing AG, part of Springer Nature 2018


Y. M. Awaad, Absolute Pediatric Neurology, https://doi.org/10.1007/978-3-319-78801-2_11
342 Chapter 11 · Headache

?? 1. Cyclic vomiting in children is a syn- reads in the car back seat. On exami-
drome in which of the following nation, she has two 0.5-cm café au
groups? lait spots, one on the left leg and
A. Migraine with aura one on the right back. Her MRI scan
B. Classic Migraine showed a 4 mm Chiari I malforma-
C. Migraine without aura tion without abnormalities of the
D. Migraine equivalent ventricles, brain parenchyma, or
extra-axial spaces.
vv Correct Answer is: D What is your explanation to the girl
Children can have three types of and her family?
migraine; migraine with aura (classic A. The anatomic abnormality of her
migraine) migraine without aura (com- brain seen on MRI scan.
mon migraine) and migraine equivalent B. Her mother’s chronic sinusitis.
syndromes. Migraine equivalent syn- C. The family’s predisposition to
dromes is characterized by the transitory migraine and its equivalents.
disturbance in neurological function. D. Her café au lait spots and likely
Abdominal migraine, benign paroxys- neurofibromatosis.
mal vertigo, cyclic vomiting syndrome, E. A brain tumor too small to be
benign paroxysmal torticollis, alternating seen on the MRI scan.
hemiplegia of childhood, and vestibular
migraine are different presentations of vv Correct Answer is: C
migraine equivalents. The child has nor- Benign paroxysmal vertigo is a “migraine
mal neurological examination between equivalent” which could happen with
attacks, there typically a family history migraine in families. The mother’s head-
of migraine and a clinical evolution over aches are a migraine headache and it is
11 the years to the more classic types of not a sinus headache, and her brother’s
migraine. motion sickness is also a migraine equiva-
lent. Her café au lait spots are neither
zz Suggested Reading large nor numerous enough to be of
55 Lagman-Bartolome A, Lay C. Pediatric consequence. Her Chiari I malformation
migraine variants: a review of epidemiol- is also likely a variant of no medical sig-
ogy, diagnosis, treatment, and outcome. nificance.

?? 2. You are evaluating an 18-year-old ?? 3. A PCP referred a 14 years old female


female for her episodic vertigo, to you because she missed 3 months
which not associated with headache of school because of her headaches.
and lasting hours to days for the last She has been getting “homebound
6 months. She always complains of schooling” and has not been doing
occasional bi-frontal, pulsatile head- well; the school wants her to repeat
aches without photo- or phono- the year. She has a daily headache
phobia or vertigo. Those headaches and it is all day but not pulsatile. The
got relived by ibuprofen. No family headaches do not have a specific
history of migraine, but the mother location and it gets worse by the end
gets sinus headaches not associated of each day. She has mild asthma for
with fever or nasal drainage, but which she takes a prn inhaler. Fam-
she gets relieve when she lies down ily history is remarkable for depres-
in a quiet, dark room. Her younger sion in his mother, sleepwalking in
brother gets motion sickness if he his younger brother, and a maternal
Headache
343 11
grandmother had menstrual head- ?? 4. The EMS brought an 8-year-old girl
aches that required her to sleep accompanied with her teacher after
when they occurred. The parents are she was seen wandering around the
divorced and there is a restraining playground, and then vomited at
order against the father because he school. The teacher reported that
physically abused the mother. She is she is clumsy, which was unusual for
always having a normal general and her. You examined her and found
neurological on multiple occasions. that, she is pale. She complains of
She is on amitriptyline at night and nausea, headache in the back of her
sumatriptan twice a week for the head, which started after arrival in
worst of the headaches, but still she the ED. When the mother arrives,
has no relief. she reports that the girl had a similar
What is your recommendations? episode twice before in the last few
A. Obtain a CT scan of the child’s months, and she will go back to base-
head. line after an extended nap.
B. Switch from amitriptyline to nor- What is your most likely diagnosis?
triptylline. A. Abdominal migraine
C. Tell the mother to have the B. Acute confusional migraine
restraining order against the C. Acute post-traumatic migraine,
father lifted. after unwitnessed fall on the play-
D. Get toxic screens on the child’s ground
blood and urine in case the father D. Basilar migraine
poisoned him. E. Cyclic vomiting syndrome
E. Arrange for psychiatric evaluation
and counseling for the mother vv Correct Answer is: D
and children. This is a case of basilar migraine, which
is one of the most common complicated
vv Correct Answer is: E migraine variants, represents 3–19% of all
The family needs psychiatric evalua- migraine. The onset is usually by 7–8 years.
tion and counseling. She has chronic You have to have two or more clear signs
daily headache with many psychosocial and symptoms of posterior fossa involve-
stressors causing it. There is a family ment to diagnose it. The signs and symp-
history of migraine and also migraine- toms include: vertigo, nausea or vomiting,
equivalent (sleepwalking), but her ataxia, visual field deficits, diplopia, tinnitus,
headaches do not have the character or hearing loss, confusion, dysarthria, weak-
temporal features of common migraine. ness, syncope, with the first 4 being the
Her school absence requires urgent most common. The typical child will begin
action and return to school may be with the posterior fossa symptoms such
therapeutic for her but will likely require as intense dizziness, vertigo, ataxia and
considerable psychological and scholas- diplopia, which can last for minutes to an
tic support. hour, and then are followed by the head-
ache phase. The headache is often occipital
zz Suggested Reading in location.
55 Waldie KE, Thompson JM, Mia Y, Murphy
R, Wall C, Mitchell EA. Risk factors for ?? 5. A 16-year-old female with known
migraine and tension-­type headache in history of intermittent migraines
11-year-old children. J Headache Pain. treated with sumatriptan nasal spray
2014;15(1):60. who has a 4-year history for the last
344 Chapter 11 · Headache

5 years, she is complaining of a new often triggered by chewing, swallow-


type of pain. It is a burning pain with ing, or rotating the head. It is due to
numbness on the left side of her face. an elongated stylohyoid process that
She has no fever or other sickness. irritates or compresses the nerve, and
She recently had four wisdom teeth radiograph shows a calcified stylohyoid
extracted. She has unremarkable past ligament; treatment is resection of this
medical history, but she has a family ligament with microvascular decom-
history of migraines in her mother pression of the nerve.
and grandmother. She has subjective Nervus intermedius neuralgia is
decreased sensation to touch over the localized to the middle ear, other-
right cheek on examination. wise described similarly to trigeminal
What is the most likely diagnosis? ­neuralgia.
A. Glossopharyngeal neuralgia
B. Nervus intermedius neuralgia zz Suggested Reading
C. Periodic migrainous neuralgia 55 David Dodick, et al, editor. Wolff ’s
D. Trigeminal dysesthesia headache and other head pain. 8th ed.
E. Trigeminal neuralgia Oxford University Press; 2008. p. 431–2.

vv Correct Answer is: D ?? 6. Which of the following can be seen


Trigeminal dysesthesia is a continuous in patients with idiopathic increased
pain following complete or partial dam- intracranial pressure?
age to a peripheral nerve. Wisdom tooth A. Constant continuous headache
extraction or root canal is a frequent B. 6th nerve palsy
cause. The pain can be self-limiting, C. Neck pain
depending on nerve regeneration. It can D. All of the above
11 be treated with local anesthetic block-
ade, topical application of capsaicin, or vv Correct Answer is: D
oral tricyclic antidepressants. The idiopathic increased intracranial
Periodic migrainous neuralgia is a pressure can present with different clini-
type of cluster headache that presents cal symptoms in different patients. Usu-
as orofacial pain, usually in the maxilla, ally, patient will complain of a severe
and is not precipitated by oral surgery, headache, most commonly in the morn-
but is a primary headache disorder. ing, and usually in the occipital region.
Trigeminal neuralgia is a painful, Diplopia, transient visual obscurations,
unilateral occurrence of brief, electric, and blurred vision may be also present.
shock-like pain limited to one or more Some patients report pulsatile tinnitus,
divisions of the trigeminal nerve. The and many can describe severe neck pain.
pain is often evoked by minor stimuli, This can at times be the presenting symp-
such as washing the face or brushing tom and can be mistaken for signs of
the teeth. It is unrelated to oral surgery, ­meningitis.
and is usually due to a tumor, such as
acoustic neuroma, or a demyelinating zz Suggested Reading
lesion. 55 Tibussek D, Distelmaier F, von Kries R,
Glossopharyngeal neuralgia Mayatepek E. Pseudotumor cerebri in
similar in quality to that of trigeminal childhood and adolescence – results of a
neuralgia, but occurs in the distribu- Germany-wide ESPED-survey. Klin
tion of the glossopharyngeal nerve, Padiatr. 2013;225(2):81–5.
Headache
345 11
55 Per H, Canpolat M. Clinical spectrum of B. Headache present for >15 days a
the pseudotumor cerebri in children: month
etiological, clinical features, treatment C. Headache has markedly worsened
and prognosis. Brain Dev. 2013;35(6): during the time of medication
561–8. overuse
D. All of the above
?? 7. Familial Hemiplegic Migraine (FHM)
includes all of the following except: vv Correct Answer is: D
A. 50% of those with FHM type 1 Medication overuse is a reason for chronic
develop a progressive cerebellar daily headache. To diagnose medication
ataxia overuse headache, medications have
B. Autosomal recessive inheritance to be used more than 3 months, and
C. Although sensory auras and apha- for > 15 days a month. Treatment includes
sia can be present, the diagnosis starting the patient on a preventative
requires the presence of a motor medication, significantly decreasing the
aura use of the acute medications, and possibly
D. Some attacks can be associated an admission to the hospital to break the
with a CSF pleocytosis headache cycle.

vv Correct Answer is: B zz Suggested Reading


Familial hemiplegic migraine is an auto- 55 Cuvellier JC, Nasser H, Vallée medication
somal dominant inheritance. Patients headache. 2013;53(2):387–8.
typically have hemiparesis along with 55 Pakalnis A, Kring D. Chronic daily
other focal neurological signs or symp- headache, medication overuse, and obesity
toms. The focal neurological deficit can in children and adolescents. J Child
be prolonged, at times lasting days. It Neurol. 2012; 27(5):577–80.
is typically occurring in the first or sec- 55 ICHD II Revision of criteria for 8.2
ond decade with attacks decreasing in Medication overuse headache. Silberstein,
frequency over time. Patients can have et al. Cephalalgia. 2005;25:460–5.
cerebellar signs or deficits with time.
Mutations in the CACNA1A, ATP1A2, ?? 9. Which of the following are considered
SCN1A, and PRRT2 genes have been contraindications to triptan use?
found to be associated with FHM. A. Hemiplegic migraine
B. Migraine with aura
zz Suggested Reading C. Hypothyroidism
55 Russel M, Ducros A. Sporadic and familial D. Previous history of benign parox-
hemiplegic migraine: pathophysiological ysmal torticollis
mechanisms, clinical characteristics,
diagnosis, and management. Lancet vv Correct Answer is: A
Neurol. 2011;10(5):457–70. You cannot use the triptans in patients
with a hemiplegic migraine because their
vasoconstrictive action.
?? 8. Medication overuse headache is
defined by which of the following? zz Suggested Reading
A. Overuse of particular medications 55 Gelfand AA. A neurologist’s guide to acute
used to treat an acute headache migraine therapy in the emergency room.
for greater than 3 months Neurohospitalist. 2012; 2(2):51–9.
346 Chapter 11 · Headache

55 Hershey A. Current approaches to the ?? 11. In the pediatric headache patient,
diagnosis and management of paediatric neuroimaging is recommended in the
migraine. Lancet Neurol;2010;9. following circumstances:
A. Adolescent patients with chronic
?? 10. Headaches attributed to Chiari I mal- daily headache
formations include all of the following B. Patients with headaches that
except: are predominantly occipital in
A. Typically, occipital in location ­location
B. Exacerbated by valsalva maneu- C. Patients with chronic recurrent
vers headaches
C. Signify that the Chiari is symptom- D. Patients with >10 missed school
atic and needs decompression days a month
D. May be associated with bilateral
sensory symptoms if a syrinx is vv Correct Answer is: B
present Routine neuroimaging is not recom-
mended for headaches that do not have
vv Correct Answer is: C any atypical features. Morning headache
To diagnose Chiari I malformation, the cer- or h
­ eadache waking up patient out of
ebellar tonsils has to be extended down- sleep requires imaging. If patient has any
ward into the foramen magnum, at least focal neurological signs or symptoms
5 mm. Headaches associated with Chiari associated with the headache, neuroimag-
I malformations are typically occipital ing is indicated. Studies have shown that
in location, associated with neck pain, and headaches that are predominantly occipi-
are exacerbated by Valsalva maneuvers. tal in location may have a higher associa-
Usually, Chiari I malformations are discov- tion with structural abnormalities and
11 ered accidently when the patient is under- imaging should be considered.
going neuroimaging for an unrelated
problem. The presence of headache does zz Suggested Reading
not always signify the Chiari is symptom- 55 Sheridan DC, Meckler GD, Spiro DM,
atic. A trial of medication for headaches Koch TK, Hansen ML. Diagnostic testing
should be done first to see if the headache and treatment of pediatric headache in the
is unrelated to the Chiari malformation. emergency department. J Pediatr.
Chiari malformations can be associated 2013;7 https://doi.org/10.1016/j.
with spinal cord syrinx. If this is the case, jpeds.2013.07.006.
then they are considered “symptomatic” 55 Lewis DW, Ashwal S, Dahl G, Dorbad D,
and typically require decompression. Hirtz D, Prensky A, Jarjour I. Practice
parameter: evaluation of children and
zz Suggested Reading adolescents with recurrent headaches:
55 Aitken LA, *Lindan CE, et al. Chiari type I Report of the Quality Standards
malformation in a pediatric population. Subcommittee of the American Academy
Pediatr Neurol. 2009;40(6):449–54. of Neurology and the Practice Committee
55 Benglis D Jr., Covington D, et al. of the Child Neurology Society. Neurology.
Outcomes in pediatric patients with Chiari 2002;59:490–8.
malformation type I followed up without 55 Hershey A. Current approaches to the
surgery. J Neurosurg Pediatr. diagnosis and management of pediatric
2011;7(4):375–9. migraine. Lancet Neurol. 2010;9:190–204.
Headache
347 11
?? 12. Which medication is typically used to vv Correct Answer is: A
treat hemicrania continua? Pediatric cyclic vomiting syndrome (CVS) is
A. Amitriptyline a periodic syndrome associated with attacks
B. Topiramate of vomiting occurring at least 4 times per
C. Propranolol hour for at least 1 h. The vomiting cannot
D. Indomethacin be caused by any other gastrointestinal dis-
order. Patients are typically symptom free
vv Correct Answer is: D between attacks so should not have fre-
Hemicrania continua is included in the quent single episodes of vomiting. Nystag-
trigeminal autonomic cephalgias along mus is not a frequent accompanying sign.
with chronic paroxysmal hemicrania. This Reports of prevalence of CVS are approxi-
syndrome is associated with a moderate mately 2% of children in the US. Attacks are
to severe unilateral headache that can be strikingly similar in a patient, beginning
associated with ptosis, tearing and swell- around the same time of day and lasting
ing of the eyelid. These disorders are felt typically a certain number of days. CVS is
to part of the indomethacin responsive considered a childhood “migraine variant”
headache syndromes. Although there are syndrome. Various reports suggest 40–60%
some reports of response to the migraine of children will go on to develop typical
prophylactic agents such as topiramate migraine in adolescence.
and amitriptyline, indomethacin is still
the drug of choice for this headache syn- zz Suggested Reading
drome. 55 Cuvellier JC. Childhood periodic syn-
dromes. Pediatr Neurol. 2010;42(1):1–11.
zz Suggested Reading 55 Stickler GB. Relationship between cyclic
55 Rozen TD. Indomethacin-­responsive vomiting syndrome and migraine. Clin
TACs (proxysmal hemicrania, hemicrania Pediatr (Phila). 2005; 44(6):505–8.
continua, and LASH): further proof of a
distinct spectrum of headache disorders. ?? 14. Considering basilar migraine: All of
Headache. 2012. the following are true except
55 Moorjani BI, Rothner AD. Indomethacin-­ A. Motor weakness is a frequent
responsive headaches in children and ­finding
adolescents. Semin Pediatr Neurol. B. Ataxia is common
2001;8(1):40–5. C. The aura commonly involves
­diplopia
?? 13. The cyclic vomiting syndrome D. Dysarthria is common
includes which of the following:
A. At least 5 attacks of vomiting, vv Correct Answer is: A
which occurs at least 4 times per Basilar migraine is a migraine syndrome
hour for at least 1 h that typically includes ataxia, slurred
B. Often associated with a diagnosis speech, diplopia, vertigo, and bilateral
of delayed gastric emptying visual symptoms. Weakness is uncommon.
C. Frequent single episodes of
­vomiting are noted in-­between zz Suggested Reading
attacks 55 The international classification of head-
D. Nystagmus is a frequent accom- ache disorders. 2nd ed. Cephalalgia.
panying sign 2004;24 suppl 1:1–160.
348 Chapter 11 · Headache

?? 15. Which of the following medications D. Have the mother bring her other
used to treat pediatric headache children in for neurological evalu-
might not be the best choice for ation.
obese patients? E. Refer the child to a psychiatrist or
A. Topiramate psychologist.
B. Verapamil
C. Cyproheptadine vv Correct Answer is: C
D. Gabapentin Alice in Wonderland syndrome is a
migraine variant that is unaccompanied
vv Correct Answer is: C by pain, nausea, or photo-/phonopho-
Cyproheptadine is a medication fre- bia and that serves as a marker for the
quently used as a migraine prophylactic migraine-­susceptible genotype. It gener-
agent. Its side effects include sedation ally occurs in children who have a family
and a significant stimulation of appetite history of migraine and associated phe-
that limit its use. nomena and who are otherwise neuro-
logically normal.
zz Suggested Reading
55 Lewis DW, Yonker M, Winner P, Sowell zz Suggested Reading
M. The treatment of pediatric migraine. 55 Ilik F, Ilik K. Alice in Wonderland syn-
Pediatr Ann. 2005;34(6):448–60. drome as aura of migraine. Neurocase.
55 Termine C, Ozge A, Antonaci F, 2014;20:474–5.
Natriashvili S, Guidetti V, Wöber-Bingöl
C. Overview of diagnosis and management ?? 17. You are seeing a 16-year-old female
of paediatric headache. Part II: therapeutic with daily headache after she has a
management. J Headache Pain. mild head trauma a week ago. She
11 2011;12(1):25–34. had no loss of consciousness and the
pain of the impact was gone within a
?? 16. You are seeing a 7-year-old boy who minute. She described the headache
told his family that he has a cool as bilateral, pulsatile, and diffuse with
“superpower”. He said, “I can sit in my a frontal predominance. She has no
seat in class and make the teacher get headache when she wakes up in the
bigger and smaller and bigger and morning, but she will have a moder-
smaller just by really concentrating ate headache by lunch time. She had
hard.” His mother is concerned and difficulty concentrating at school.
she took him to an optometrist to When she comes home from school
have his eyes checked. By taking the every day, she is irritable, which was
history, you find that there is a family very unusual for her. She will take a
history of migraine in the boy’s mater- nap for couple of hours before doing
nal grandmother and aunt. The boy’s her homework. Taking short naps
father was a sleepwalker as a child and make the headache tolerable enough
his mother has to sit in the front seat to finish her work. She has a past
of the car when she is a passenger, medical history of headache before
otherwise she will vomit. You should: her period every month. Her family
A. Order an MRI scan for the child. history is remarkable for a younger
B. Tell the child to stop scaring his brother gets motion sickness and a
parents. “sinus headaches” that are preceded
C. Reassure the family by telling by zigzag lines in front of her mother
them what the episodes likely are. eyes and associated with nausea. She
Headache
349 11
has a normal general and neurologi- and it was treated by amoxicillin, but
cal examinations. it has not helped. He developed a
What is your next step? bony swelling behind his left ear, but
A. Order an MRI scan of the head he did not tell his mother about it.
B. Begin a therapeutic trial of ami- Finally, he told his mother this morn-
triptyline ing. On examination in the ED, he
C. Have a school psychologist evalu- was sick looking with 38.5 °C fever,
ate the girl for school phobia tenderness over the left mastoid, and
D. Do a lumbar puncture bilateral papilledema. The rest of his
E. Refer the girl to an otolaryngolo- general and neurological examination
gist for sinus evaluation is normal. A CT scan has been ordered
and showed, lack of aeration of the
vv Correct Answer is: B left mastoid air cells and a white area
Migraine like headache that occurs inside of the left lateral venous sinus
within days-to-weeks of a minor head posteriorly.
injury in a child with the genetic suscep- You decide to obtain:
tibility to migraine is called post-trau- A. An EEG with sphenoidal leads
matic migraine. The mother’s headaches now and a CT angiogram later
are not sinus headaches, as they are B. A lumbar puncture with opening
preceded by a scotoma, accompanied pressure now and an MRI/MRV later
by nausea, related to motion sickness, C. An MRI of the orbits now and an
and unrelated to fever, cough, or nasal MRI of the spine late
discharge. The girl herself has had D. A Humphrey’s visual field test now
menstrual ­headache, also a migraine-­ and a serum vitamin A level later
associated syndrome. Amitriptylline is E. A CBC and ESR now and a blood
among the most commonly used and culture later
­efficacious agents for post-traumatic
migraine. vv Correct Answer is: B
Mastoiditis and lateral cerebral venous
zz Suggested Reading sinus thrombosis became much less com-
55 Kuczynski A, Crawford S, Bodell L, Dewey mon after the antibiotic era. They do
D, Barlow KM. Characteristics of post- occur, however, and are often associated
traumatic headaches in children following with an acute, antibiotic-resistant exacer-
mild traumatic brain injury and their bation of chronic otitis media. Occlusion
response to treatment: a prospective of the dural venous flow can cause pseu-
cohort. Dev Med Child Neurol. dotumor cerebri and a lumbar puncture
2013;55:636–41. is both diagnostic and therapeutic. His
CT scan demonstrates left dural venous
sinuses thrombosis, and neither the CT
?? 18. A mother of 10-year-­old boy brought scan nor his examination demonstrates
him to the ED after he awakened this evidence of focal differential intracranial
morning with a bad frontal headache pressure, so there is no need to obtain an
and blurry vision. His past medical MRI/MRV urgently. Visual fields should
history is remarkable of chronic ear be obtained, but are not of emergent
infections for years requiring bilateral importance. Neither an EEG nor an MRI of
PE tube insertion. The tubes fell out the spine would be likely to be useful in
1 year ago, and never been replaced. this child. In a febrile child with systemic
He had ear pain and fever 1 week ago spread of a presumed bacterial infection,
350 Chapter 11 · Headache

a blood culture should be obtained expe- vv Correct Answer is: E


diently. While vitaminosis A can cause This boy sustained an intracranial hemor-
pseudotumor cerebri, that is unlikely to be rhage with a subarachnoid component
the cause of this youngster’s difficulties. after several episodes during the prior
month of sentinel bleeding. His family
zz Suggested Reading history is remarkable for one elderly indi-
55 Kuczkowski J, Dubaniewicz-­Wybieralska vidual with a glioblastoma, generally not
M, Przewoźny T, Narozny W, Mikaszewski a hereditary condition, and two relatives
B. Otitic hydrocephalus associated with with polycystic kidney disease, a disorder
lateral sinus thrombosis and acute which is genetically linked to the occur-
Mastoiditis in children. Int J Pediatr rence of cerebral aneurysms.
Otorhinolaryngol. 2006;70:1817–23.
zz Suggested Reading
55 Ruggieri PM, Poulos N, Masaryk TJ, Ross
?? 19. You are seeing a 14-year-old boy in
JS, Obuchowski NA, Awad IA, Braun WE,
the PICU after he lost his conscious-
Nally J, Lewin JS, Modic MT. Occult
ness while he was playing a game
intracranial aneurysms in polycystic
at school. His father told you that
kidney disease: screening with MR
the boy was fine before the game
angiography. Radiology. 1994;191:33–9.
although he had headed the ball
55 Fehlings MG, Gentili F. The association
once or twice during the game,
between polycystic kidney disease and
nothing unusual seemed to have
cerebral aneurysms. Can J Neurol Sci.
happened until he suddenly lost
1991;18:505–9.
his consciousness. The boy was run-
ning down the field and he did not ?? 20. A 17-year-old female with a long history
make contact with the ball, suddenly of childhood migraine which occurs
11 stopped, grimaced, held his head once or twice per month and well
with both hands crying out, and fell controlled by ibuprofen. Recently, she
to the ground. He had generalized increases in her headache frequency,
jerky movement for couple of min- headaches come most days of each
utes and then lay unconscious, inter- week, and she has to take ibuprofen 2
mittently extending both arms and or 3 times a day almost every day. In the
arching his back. He has been uncon- last 2 weeks, her headaches became
scious ever since. His father also more severe and less responsive to ibu-
reports that 3 weeks ago, he woke profen. There is no change in her head-
up with headache and neck stiff- ache character. She told you that her
ness in 3 different occasions, but the mother is taking a medicine every night
symptoms went away shortly after at bedtime to prevent the mother’s
he got out of bed. His family history headache from coming. She has a nor-
is remarkable of a maternal grand- mal general exam except her BMI is 30.
mother who died from glioblastoma. You instructed her to stop the ibuprofen
His father and paternal grandfather all of a sudden and prescript her a trip-
have polycystic kidney disease tan to use no more than twice a week.
The boy is most likely had an intra- You might best decide to also give
cranial hemorrhage because of: her daily preventive treatment with:
A. A brain tumor A. Naproxen sodium
B. Head trauma B. Valproic acid
C. An arteriovenous malformation C. Botulinum toxin
D. An infection D. Verapamil
E. An aneurysm E. Topiramate
Headache
351 11
vv Correct Answer is: E Treatment with acetaminophen, ibu-
The girl has an increase of her headaches profen, naproxen, and indomethicin
after a long history of migraine for years have been ineffective.
during adolescence. This is a very com- What is your likely diagnosis?
mon phenomenon in young women. This A. Chronic daily headache
led her to take more ibuprofen daily most B. Chronic paroxysmal hemicrania
of the days. She has therefore developed C. Cluster headache
medication overuse headaches. Interest- D. New daily persistent headache
ingly, NSAIDs is very effective and even E. Occipital neuralgia
protective in patients with infrequent
headaches, but likely to trigger medica- vv Correct Answer is: E
tion overuse headaches in those with International Headache Society criteria
frequent or chronic headaches as in her for ICHD-II criteria for occipital neuralgia
case. Treatment involves discontinuation includes paroxysmal stabbing pain, with
of NSAID, symptomatic use of triptans or without persistent aching between
no more than twice a day for no more paroxysms, in the distribution(s) of the
than 2 days in any week, and starting the greater, lesser and/or third occipital
patient on a daily preventive medication nerves; tenderness over the affected
for migraine. Of those listed, verapamil is nerve; and pain is eased temporarily by
not particularly efficacious and valproic local anesthetic block of the nerve. It is
acid is likely to cause weight gain and, very important to differentiate between,
possibly, ovarian cysts in this already Chronic daily headache occurs when
overweight youngster. On the other hand, a patient has > 15 headaches days per
topiramate is both efficacious and likely to month, for more than 4 h per day for more
cause weight loss, so it is a good first-line than 3 months, and a new daily persistent
preventive drug for this adolescent. headache occurs in a patient without a
previous significant headache history,
zz Suggested Reading and the features of the headache are simi-
55 Starling AJ, Hoffman-­Snyder C, Halker lar to a chronic tension-type headache.
RB, Wellik KE, Vargas BB, Dodick DW, Cluster headaches are severe head-
Demaerschalk BM, Wingerchuk DM. Risk aches that are typically located behind
of development of medication overuse the eye or over the eye, with autonomic
headache with nonsteroidal anti-inflam- features such as tearing, conjunctival
matory drug therapy for migraine: a injection, nasal congestion or ptosis.
critically appraised topic. Neurologist. The patient often appears agitated dur-
2011;17:297–9. ing a cluster headache, in contrast to
55 Carod-Artal FJ. Tackling chronic migraine: the typical migraine headache patient
current perspectives. J Pain Res. who wants to go to sleep in a quiet, dark
2014;7:185–94. room. Cluster headaches are also unilat-
eral side-­locked headaches, and never
?? 21. A 13-year-old girl stared to have daily switch between sides.
episodic lancinating pains in the back Chronic paroxysmal hemicrania
of her head for the last 3 months after (CPH), like cluster, belongs to a group of
she hit her left occiput on the floor. headaches referred to as the trigeminal
She is not complaining of photopho- autonomic cephalgias. However, in CPH
bia, nausea, tearing, and conjunctival the headaches occur multiple (1–40)
injection with the headaches. On times a day and lasts for a shorter dura-
examination, she has tenderness tion (minutes. CPH is, by definition,
over her right greater occipital nerve. responsive to indomethicin.
352 Chapter 11 · Headache

zz Suggested Reading but would not make the diagnosis in this


55 International classification of headache child. The inciting event can be mild or sig-
disorders (ICHD-II). Cephalalgia. 2004;24 nificant, and in a gymnast, can be a com-
(s1):23–136. bination of minor injuries. The diagnosis is
primarily clinical, and usually requires at
?? 22. A 10-year-old girl who practice gym- least 2 of the following symptoms: sensory
nastic is complaining of pain in her (hyperesthesia), vasomotor (temperature
right foot. She describes the pain as or skin color abnormalities), sudomotor/
constant and aching, but sometimes, fluid balance (edema, sweating abnormali-
there is a burning sensation that ties), or motor (decreased range of move-
makes her stop practicing. When her ment, weakness, tremor, or neglect), and
mother tries to rub her foot, the girl at least 2 of the following signs: sensory
cries and pulls her foot away. She has (allodynia or hyperalgesia), vasomo-
unremarkable past medical history. tor (objective temperature or skin color
You examined her and you found abnormalities), sudomotor/fluid balance
that, the skin appears slightly cooler (objective edema or sweating abnormali-
on the right foot than on the left, and ties), or motor (objective decreased range
somewhat mottled. She guards the of motion, weakness, tremor, or neglect).
foot on examination, wincing even
with light touch. She has a normal zz Suggested Reading
DTRs. She is able to walk indepen- 55 Johnson RT, et al., editors. Current therapy
dently, and currently describes the in neurologic disease. 6th ed. St. Louis:
pain as a 3 on a scale of 10. She states Mosby Press;2002. p. 71–4.
that she can practice when it is this
level, but when it increases, she has to ?? 23. A 16-year-old girl with a long history
11 stop. of migraines came to the ED after
What are you going to order to 2 day history of unrelenting head pain
establish her diagnosis? associated with severe photophobia
A. MRA of ankle for blood flow aber- and phonophobia, anorexia, nausea
ration and vomiting. Her Previous head-
B. Radiograph of ankle to evaluate aches occurred 3–4 times per month,
for subtle fracture lasting 3–5 h. She is on amitriptyline
C. Somatosensory evoked potential 30 mg daily at night, and has been
to assess for spinal plexus injury taking sumatriptan intranasal twice
D. Venous doppler to assess for deep a day for the last 2 days. Her past
venous thrombosis medical history is remarkable for
E. Sudomotor function test to assess asthma, treated with Singulair, and
for greater sweat output acne treated with doxycycline. She is
pale, tired appearing and in moder-
vv Correct Answer is: E ate distress. Neurological examina-
Sudomotor axon reflex test (QSART), tion is normal. ED physician gave her
which is a quantitative teat can be used 20 mg of metaclopromide orally and
to show sympathetic hyperactivity which 15 mg of ketorolac intramuscularly.
is associated with altered sweat output. It Her headache has not resolved after
helps to confirm clinical impressions of a 30 min, and she is given 0.2 mg of
complex regional pain disorder. It usually dihydroergotamine intravenously.
occurs after an initiating noxious event. A 15 min later, she develops repeated
specific fracture on x-ray might be seen, forced head turning episodes with
Headache
353 11
eye deviation to either side. She is time” headache. He has normal exam
very anxious during the episodes and and has no papilledema. His MRI is
cries repeatedly. normal.
What do you have to do? What is the most appropriate next
A. Administer fosphenytoin 1000 mg step in management?
intravenously A. Continue current treatment and
B. Administer diphenhydramine add corticosteroid
50 mg intravenously B. Discontinue current medications
C. Administer repeat doses of dihy- and observe
droergotamine, metaclopramide, C. Perform lumbar puncture
and ketoralac D. Replace acetaminophen/codeine
D. Start baclofen 10 mg every 8 h with oxycodone
E. Start gabapentin 300 mg every 6 h E. Taper opiate and barbiturate and
start topiramate
vv Correct Answer is: B
Acute dystonic reaction can occur in vv Correct Answer is: E
some patients immediately after the This presentation is for a chronic daily
administration of the offending medica- headache syndrome, which meets the
tion, such as metaclopromide, or slightly International Headache Society Criteria
delayed. Recognition of the disorder is for Chronic Migraine, with the exception
the key to appropriate treatment and pre- of his overuse of both opiate and barbi-
vention of further episodes in the future. turate compounds. These compounds
Diphenhydramine is the treatment of should be withdrawn in the patient.
choice. Care should be taken, depending on the
patient’s dose and duration of use, not to
zz Suggested Reading trigger an abrupt withdrawal syndrome.
55 Johnson RT, et al., editors. Current therapy Recent evidence suggests that starting a
in neurologic disease. 6th ed. St. Louis: preventative medication during this with-
Mosby Press; 2002. p. 339–40. drawal phase is helpful.

?? 24. You are seeing a 16-year-old male zz Suggested Reading


with a long history of once a week 55 Revised International Headache Society
migraine headache, who has an criteria for chronic migraine. Appendix
increase in his headache frequency 1.5.1 chronic migraine
for the last year. He was on acetomi- 55Headache (tension-type and/or
nophen with codeine, and a combi- migraine) on ≥15 days per month for at
nation medication with butalbital least 3 months*
(Acetominophen/butalbital/caffeine; 55Occurring in a patient who has had at
Fioricet) for abortive therapy. These least 5 attacks fulfilling criteria for 1.1
medications had initially helped Migraine without aura
with the headache pain, but no
longer seem to be effective, and his ?? 25. A 16-year-old girl came to your office
severe headaches have continued to with a complaint of sharp pain which
increase in frequency. He uses one varies in location headaches. Each
of these medications at least four headache lasts less than 1 min, but it
times a week. He currently has severe is quite severe. When the headache
migraine headaches four times a is over, she feels tired, but the pain
week, as well as a lower grade “all the is gone. The headache has been
354 Chapter 11 · Headache

increased in frequency lately, occur- ?? 26. You are seeing a 14-year-old girl
ring multiple times each day. She with a history of intermittent once a
is worried and she is looking for an month headache for the last 3 years.
answer to her worries. She does not She describes the headaches as a
have any other type of headaches, visual aura of flashing lights, followed
and she has a normal neurological by severe throbbing headache (9 out
exam. of 10 on a visual analog scale) 10 min
What is the most appropriate treat- later. The headache is associated with
ment? phonophobia and photophobia, and
A. Indomethacin 50 mg three times a she has to go to sleep in order to
day for 2 weeks relieve the headaches. The headaches
B. Naproxen sodium 500 mg twice a last for several hours. She has tried
day for 10 days acetaminophen and ibuprofen with-
C. Propranolol twice a day for out success.
2–4 months What is the best medication for her
D. Sumatriptan nasal spray at onset pain control at the onset of a head-
of headache ache?
E. Topiramate 25 mg at night for A. Almotriptan
1 week, increasing weekly to B. Aspirin/butalbital/caffeine
100 mg at night C. Oxycodone
D. Prochlorperazine
vv Correct Answer is: A E. Valproic acid
The clinical presentation of an icepick
headache, which is a type of primary vv Correct Answer is: A
stabbing headache and is responsive to Migraine is a moderate to severe headache
11 indomethacin. The age of onset ranges of pulsating quality and is made worse
from 12 to 70 years, and it is more com- with activity according to the International
mon in females. The pain is experienced Headache Society. Headache usually is
as a sharp stab, exclusively or predomi- associated with either nausea and vomiting
nantly felt in the distribution of the first or photophobia and phonophobia. Chil-
division of the trigeminal nerve, lasting dren tend to have shorter duration head-
seconds to a minute, recurring irregularly. aches than adults, usually between 1–72 h
A response to indomethacin is often sup- in duration, and (in contrast to adults) may
portive of the diagnosis, and doses of have bilateral pain.
25–150 mg are usually effective. Primary Acetominophen and ibuprofen
stabbing headaches can also co-occur have been shown to be effective in
with migraine. Other diagnostic consid- patients with migraine. These are good
erations include trigeminal autonomic first choices for children and adoles-
cephalgias, such as hemicrania cicrania cents to use in the treatment of epi-
or chronic paroxysmal hemicrania. These sodic migraine. On June 2009, the FDA
conditions are also responsive to Indo- extended the indication for almotriptan
methicin, but are side locked to one side to include treatment of migraines in
of the head. adolescents between 12 and 17 years
of age, making it the first triptan to
zz Suggested Reading be approved for pediatric patients.
55 Dodick D, et al., editors. Wolff ’s headache Published data also support a role for
and other head pain. 8th ed. Oxford the other triptans in the treatment of
University Press; 2008:p. 431–2. headaches in children under the age of
Headache
355 11
18, although many of these compounds 55 Linder SL, Mathew NT, Cady RK,
have not yet approved by the FDA for Finlayson G, Ishkanian G, Lewis
children under the age of 18. Opiates, DW. Efficacy and tolerability of almotrip-
such as oxycodone, and barbituate tan in adolescents: a randomized, double-
containing compounds such as Aspirin/ blind, placebo-controlled trial. Headache.
butalbital/caffeine (Fiorinal), can make 2008;48(9):1326–36.
a headache patient prone to rebound,
or medication overuse, headaches and ?? 27. A 15-year-old right handed girl
should be avoided in this situation. Pro- presented to the ED with a sudden
chlorperazine alone or in combination onset of a right sided paresis, lasting
with ketorolac can be useful in an emer- for 15 min, followed by an intense
gency room setting for the treatment of throbbing left sided headache. Previ-
childhood migraine. Oral valproic acid ously she has migraine with visual
is not used for the acute treatment of auras. There is a family history of her
migraine, although some suggestive evi- mother and aunt have had similar
dence exists that intravenous valproic episodes of weakness and head-
acid may be helpful. aches as teenagers. On exam, she is
uncomfortable with headache pain,
zz Suggested Reading is photophobic, but her weakness
55 Bigal ME, Lipton RB. Excessive acute has resolved.
migraine medication uses and migraine What diagnostic tests are you going
progression. Neurology. 2008;71(22):1821– to order to reveal the etiology of
8; Brousseau DC, Duffy SJ, Anderson AC, her condition?
Linakis JG. Treatment of pediatric A. Carotid ultrasound
migraine headaches: a randomized, B. Diffusion weighted imaging on a
double-blind trial of prochlorperazine Head MRI
versus ketorolac. Ann Emerg Med. C. EEG
2004;43(2):256–62. D. Genetic testing for the Calcium
55 Damen L, Bruijn JK, Verhagen AP, Berger channel gene CACNA1A
MY, Passchier J, Koes BW. Symptomatic E. Thematic Apperception Test
treatment of migraine in children: a
systematic review of medication trials. In vv Correct Answer is: D
Pediatrics. 2005;116(2):e295–302. A sudden onset of hemiplegia in an ado-
55 International classification of headache lescent is of great concern. MRI study
disorders (ICHD-II). Cephalalgia. with diffusion weighted imaging should
2004;24:(s1):23–136; Lewis D, Ashwal S, be strongly considered. However, in this
Hershey A, Hirtz D, Yonker M, particular scenario, this young woman
Silberstein S; American Academy of most likely has familial hemiplegic
Neurology Quality Standards migraine. This can be associated with
Subcommittee; Practice Committee of the three different genes (CACNA1A, ATP1A2
Child Neurology Society. Practice and SCNA1), with calcium channel
parameter: pharmacological treatment of mutations (CACNA1A) being commonly
migraine headache in children and detected, particularly if the symptoms
adolescents: report of the American first appear before 16 years of age. The
Academy of Neurology Quality Standards hemiplegia may occur before the head-
Subcommittee and the Practice ache, as is typical for most migraines
Committee of the Child Neurology auras. A Todd’s paralysis, acute stroke,
Society. Neurology. 2004;63(12):2215–24. or functional disorder could also be
356 Chapter 11 · Headache

considered in this scenario. Given the to indomethacin. Not all patients with
strong family and personal history of hemicrania continua with resolve with
migraine, this is most likely familial hemi- indomethacin, but all patients with uni-
plegic migraine. lateral headache should be given a trial of
indomethacin.
zz Suggested Reading
55 Lewis DW. Pediatric migraine. Neurol zz Suggested Reading
Clin. 2009;27(2):481–501. 55 Marmura MJ, Silberstein SD, Gupta
55 International classification of headache M. Hemicrania continua: who responds to
disorders (ICHD-II). Cephalalgia. indomethacin? Cephalalgia.
2004;24(s1):23–136. 2009;29(3):300–7.
55 Roach ES, Golomb MR, Adams R, Biller J, 55 Ji T, Mack KJ. Unilateral chronic daily
Daniels S, Deveber G, Ferriero D, Jones headache in children. Headache.
BV, Kirkham FJ, Scott RM, Smith 2009;49(7):1062–5. Epub 2009 Jun 1.
ER. Management of stroke in infants and
children: a scientific statement from a ?? 29. A 14-year-old girl is complaining of
Special Writing Group of the American headaches. She has 2–3 headache
Heart Association Stroke Council and the per week of pounding pain in her
Council on Cardiovascular Disease in the forehead which occasionally cause
Young. Stroke. 2008;39(9):2644–91. her to miss school. They are 8/10 in
severity on a scale of 0/10. They occur
?? 28. A 16-year-old girl came with a 1-year more often in the afternoons, and
history of daily headaches. She never awaken her at night. She has
reports that her daily continuous light and sound sensitivity with her
headache does not prevent her from headache. When she takes Ibupro-
11 attending her school, but it does not fen, the headache goes away within
improve with sleep. She has a consis- 30 min but occasionally she has to
tently right-side head headache. She sleep to get relief. She takes accutane
has no family history of headache. for acne. She has a family h­ istory of
She has unremarkable examination sinus headaches in her mother and
and a normal brain MRI. grandmother. She has a normal exam.
What medication should be used Venous pulsations are present on
initially? ophthalmoscopic exam.
A. Ibuprofen Which of the following is the most
B. Prednisone likely diagnosis?
C. Sumatriptan A. Tension headache
D. Valproic acid B. Cluster headache
E. Indomethacin C. Migraine
D. Sinus headache
vv Correct Answer is: E E. Pseudo tumor cerebri
Classification of headaches is essential to
their optimal treatment. Unilateral head- vv Correct Answer is: C
aches can represent ominous conditions, Classification of headaches is important
such as arteriovenous malformations, in assisting with treatment and progno-
or can represent hemicranias continua sis issues. Migraine diagnosis requires
which is a primary headache disorder specific criteria to be met, including
which is partly defined by its response photophobia, nausea, and relief by
Headache
357 11
sleep, and exacerbation with exercise. What is your next step in her man-
Family history is important during this agement?
assessment, and further delineation A. Repeat the lumbar puncture
of the specific nature of headache in and remove a larger volume
her family members would be advised, of CSF
as “sinus” headaches are occasion- B. Increase fluids, administer IV caf-
ally actually misdiagnosed migraines feine, and consider an epidural
Rebound headaches or transformed blood patch
migraines can be misdiagnosed if the C. Administer intravenous antibiotics
medication history is not adequately and antiviral medications
assessed. Age of onset of headaches is D. Administer intranasal sumatriptan
not relevant in diagnosis of migraines, followed by intravenous dihydro-
though there is a typical age of onset ergotamine
around puberty. E. Administer intravenous cortico-
steroids
zz Suggested Reading
55 Winner P. Classification of pediatric vv Correct Answer is: B
headache. Curr Pain Headache Rep. She has worsening chronic recurrent
2008;12(5):357–60. Review. headaches, but her headache does not
have any migraines features described.
?? 30. You are seeing a 16-year-old female Most properly she has a rebound or
in the ED because off her headache chronic daily headache due to daily
for 2 months. Her headache was over-the-counter analgesic use. The
intermittent, but now it became lumbar puncture was p ­ erformed to rule
worse, daily for the last 2 weeks, and out pseudo tumor cerebri, and did show
it is 5/10. She describes it as being elevated opening pressure. This case is
generalized. She has tried over the dealing with a low-pressure headache
counter medications, but never which is the sequelae of the LP. Headache
been on any prescribed medications. after a lumbar puncture is common, but
Her body mass index is 30, she is in is usually resolved within a day or two. If
moderate distress but the rest of there is no relief after 48 h, strong con-
her general and neurological exams sideration should be made to diagnosing
are normal. She does not have any and treating the low-pressure headache
signs and symptoms meningitis. You with bedrest, hydration, and caffeine and,
ordered a LP, which demonstrates an if needed, an epidural blood patch.
opening pressure of 29 cm. but the
rest of the CSF parameters are nor- zz Suggested Reading
mal. She has relief of her headache 55 Candido KD, Stevens RA. Post-dural
shortly after the LP but the headache puncture headache: pathophysiology,
came back after 24 h and she came prevention and treatment.
back to the ED because her headache
worsens. She is lying flat on bed and ?? 31. You are seeing a 17-year-old female
every time she gets up her headache in the ED because off her sudden,
will worse. You admitted her to the acute, and severe (10/10) headache.
hospital and put her on strict bedrest, It is bi-temporal in location, pulsatile,
but her headache fails to improve and associated with photophobia and
over the next 24 h. phonophobia. She has vomited once
358 Chapter 11 · Headache

in the emergency room. She had three pain needs to be treated with pain medi-
similar episodes in the last 6 months, all cation and the severe nausea needs to be
requiring emergency room visits due to treated with anti-emetics and intravenous
dehydration from repeated vomiting. rehydration. Specific migraine medica-
What is the most appropriate treat- tions such as triptans can often treat the
ment recommendation? pain as well as the associated symptoms,
A. Acute treatment with intrave- though most are not FDA-approved in
nous fluids, non-steroidal anti-­ adolescents. The use of daily medications
inflammatory agent and anti-emetic, should be reserved for those patients who
followed by rescue medication only have headaches unresponsive to rescue
B. Acute treatment with narcotic, medications or whose headaches are so
followed by 6-month course of frequent as to cause disability, such as
prophylactic medication missed school days.
C. Acute treatment with non-
steroidal oral anti-inflammatory, zz Suggested Reading
followed by daily use of non- 55 Lewis DW. Pediatric migraine. Neurol
steroidal anti-inflammatory agent Clin. 2009;27(2):481–501.
for 2 weeks
D. Acute treatment with intravenous ?? 32. You are evaluating an 8-year-old boy
valproic acid followed by pre- for his headaches which started 1 year
ventative treatment with sodium ago, but they became more frequent
valproate for 1 year and severe over time. They occur 2–3
E. Acute treatment with intravenous times/ week, are 8/10 in severity, and
steroids, followed by daily use of last for an hour after he takes acet-
indomethacin for 6 weeks aminophen. On one occasion, the pain
11 was so severe that he passed out for a
vv Correct Answer is: A few minutes. The headache is located
Acute treatment of migraines in a child is in the mid-­forehead and he cannot
dictated by the severity of the headache characterize its nature. He has normal
and the concomitant symptoms. Migraine exam. A midsagittal MRI is shown here.

a b

..      Fig. 11.1 a Coronal MRI. b Midsagittal MRI showing Sephenoid mucocele. Sphenoid mucocele. Permission by
Lui YW, Dasari SB, Young RJ. Am J Neuroradiol. 2011;32(4):617–26. 7 https://doi.org/10.3174/ajnr.A2144
Headache
359 11
What is the most likely diagnosis
regarding the area identified by the
arrow?
A. Hypothalamic ­hamartoma
B. Pituitary tumor
C. Sphenoid mucocele
D. Cribriform plate lipoma
E. Chondrosarcoma of the skull base

vv Correct Answer is: C


Child neurologist has to be fully aware
with neuroanatomy as seen on MRI
imaging and all films should be viewed
personally, without relying on the
neuro-radiologist report. This image
shows a sphenoid mucocele of fairly sig-
nificant size. These are rare in children,
but can cause significant headache that
..      Fig. 11.2 Image of a Type 1 Arnold-Chiari
is often medically refractory. Treatment
Malformation. The cerebellum has descended 7 mm
is with surgical resection of the muco- and there are herniated cerebellar tonsils into the
cele, usually with complete resolution of foramen magnum. (Basket of Puppies)
the ­headaches.
What is on the image could explain
zz Suggested Reading his headache?
55 Uren BA, Berkowitz RG. Isolated inflam- A. Thrombosis of the torcula
matory sphenoid sinus disease in children. B. Cerebellar tonsillar ectopia
Ann Otol Rhinol Laryngol. C. Sphenoid mucocele
2003;112(4):370–2. D. Empty sella
E. Frontal sinus air-fluid levels
?? 33. A 12-year-old boy is complaining of
headache every time he is doing a vv Correct Answer is: B
valsalva maneuver which have been Many structural lesions can cause head-
increased in frequency and sever- aches in children, including sinus disease,
ity in the last 2 months, now rated pituitary lesions, pineal lesions, venous
8/10. They are occipital in location, sinus thrombosis, and AVM. Chiari I
and non-radiating. He denies any malformation with ectopia or excessive
photophobia or phonophobia. Sleep descent of the cerebellar tonsils can cause
does not improve the pain. He has a increased intracranial pressure leading to
blurred optic disc margins on fundu- headaches that can be refractory to medi-
scopic examination. His brain MRI is cation. Cine flow studies at the level of the
shown. foramen magnum can help in determining
the extent of obstruction of the Chiari.
360 Chapter 11 · Headache

?? 34. A 16-year-old female has had several A. Ioratadine


spells of severe (rated 8/10) unilateral B. Albuterol
headache associated with contralateral C. Minocycline
weakness and numbness, each last- D. Fluticasone
ing for several hours. She is normal in E. Salicylic acid
between the attacks. Her family history
is remarkable of her mother had similar vv Correct Answer is: C
spells as a ­teenager. You have to be fully aware of medication
What is the most likely underlying triggers of pseudo-tumor cerebri to be
pathophysiology regarding these able to treat it rapidly. Minocycline and
episodes? other antibiotics such as tetracycline are
A. Mitochondrial oxidative phos- known to promote increased intracranial
phorylation disorder pressure in some patients, precipitating
B. Glutamate neurotoxicity pseudo- tumor cerebri. Other agents can
C. Ion channelopathy include lithium and retinoids.
D. Vasoconstriction
E. Acetylcholine receptor blockade zz Suggested Reading
55 Friedman DI. Medication-­induced intracra-
vv Correct Answer is: C nial hypertension in dermatology. Am J
Hemiplegic migraine is a relatively rare Clin Dermatol. 2005;6(1):29–37. Review.
form of migraine with aura, and has
both familial and non-familial (sporadic) ?? 36. A 10-year-old girl is complaining of fre-
forms. Genetic tests are available, and the quent headaches which have caused
known genes encode ion-transporters. her to miss many school days this year.
The pathophysiologic mechanisms are The lights and noise of the classroom
11 complex, but are best explained by abnor- are increasing her headache. She goes
malities in the trigeminal-vascular system. to the office 4–5 times a week to get
ibuprofen 200 mg orally. She reports
zz Suggested Reading that she also has headache at home
55 Cady R, Biondi DM. An update on migraine as well. Acetaminophen or ibuprofen
pathophysiology and mechanism-based does help to relieve her headache, and
pharmacotherapeutics for migraine. estimates she goes through a bottle
Postgrad Med. 2006; Spec No: 5–13. Review. per month of each medication. Patient
and mother state the ­medication is
?? 35. You are evaluating a 15-year-old boy for helpful in relieving the pain, and deny
his worsening in frequency and duration any associated side effects.
headache which stared 3 weeks ago. They Treatment of which of the following
are as 6/10 rate. They are diffuse in nature, conditions would most likely lead to
no nausea or vomiting, and no change in improvement in her headache fre-
appetite. He complains of photophobia, quency?
but no phonophobia. He has seasonal A. Migraine
allergies, exercise-induced asthma, and B. Cluster
moderately severe inflammatory acne. C. Tension-type
His is taking loratadine, albuterol, flutica- D. Analgesia overuse
sone, minocycline, and salicylic acid. His E. Hemicranias continua
examination is remarkable for absence of
venous pulsations and mild papilledema. vv Correct Answer is: D
Which of the following medications is Chronic daily headaches in children and
the most likely cause of his headaches? adolescents can cause significant disability,
Headache
361 11
but they are not as common as intermit- What is the most likely explanation
tent headaches. The diagnosis of analgesic for the abnormality demonstrated?
overuse syndrome, and/or transformed A. Hypothalamic hamartoma
migraine, is necessary for effective treat- B. Simple pineal cyst
ment. The use of a prophylactic agent com- C. Pineal teratoma
bined with removal of the overused rescue D. Choroid plexus cyst
medication, and educating the patient and E. Optic nerve schwannoma
family as to the cause of the change in fre-
quency of the headaches, can be dramatic vv Correct Answer is: B
in returning the child to a functional state Glial cysts of the pineal are often
in terms of school attendance. found incidentally on neuroimaging
or at autopsy, and are usually benign.
zz Suggested Reading The finding can be seen in 20–25% of
55 Moore AJ, Shevell M. Chronic daily healthy individuals on MRI so are usu-
headaches in pediatric neurology practice. ally incidental and do not usually need
J Child Neurol. 2004;19(12):925–9. repeat imaging, unless they have wor-
risome characteristics such as large size
?? 37. A 14-year-old girl is brought to the or m
­ ulti-locuated cyst. They very rarely
ED after a syncope spell at school. become symptomatic, causing headache,
She has a history of chronic, recurring syncope, insomnia, or sudden death.
headaches and mild attention-deficit
disorder for which she takes methyl- zz Suggested Reading
phenidate. Her MRI is shown. 55 Pu Y, Mahankali S, Hou J, Li J, Lancaster
JL, Gao JH, Appelbaum DE, Fox PT High
prevalence of pineal cysts in healthy adults
demonstrated by high-­resolution, noncon-
trast brain MR imaging. AJNR Am J
Neuroradiol. 2007;28(9):1706–9. Epub
2007 Sep 20.

?? 38. A 14-year-old female fell in the school


yard. She did not have any skin abra-
sion anywhere, but she twisted her
left ankle so that it was mildly painful
all day. At home that night, her ankle
was mildly swollen and she held an
ice compress against it on and off all
evening. Her symptoms disappeared
over the next couple of days. A week
later, she noticed the onset of episodes
of redness, heat, and intense burning
pain over the whole lower 1/3 of her
left leg and foot that last for a few min-
utes to a few hours, and then
remit completely until the next
..      Fig. 11.3 Tornwaldt’s cyst in sagittal MRI (FLAIR).
The cyst appears hyperintense in the midline of the ­episode.
nasopharynx. In this case there is also a cyst of the What is your explanation?
pinealis gland showing a signal intensity slightly A. Cellulitis caused by MRSA
higher than the CSF. (Hellerhoff Wikipedia) B. Erythema multiforme
362 Chapter 11 · Headache

C. Pellagra vv Correct Answer is: C


D. Complex regional pain syndrome Complex regional pain syndrome (for-
E. Erythromelalgia merly known as reflex sympathetic dys-
trophy) is well-described in children but
vv Correct Answer is: D poorly understood in any age group. Mul-
This is a case of a complex regional pain tifactorial etiology and predisposition are
syndrome because the predated history suspected and therapy is almost always
of minor trauma and episodic nature of multidisciplinary, involving graded physi-
the color and temperature change and cal therapy, individual and family psy-
pain. Erythromelalgia usually occurs in chotherapy, agents aimed at decreasing
older individuals and is not preceded by neurogenic pain, and agents that affect
injury. Pellagra involves desquamation the autonomic nervous system.
and usually there is an identified risk
factor for vitamin deficiency. Erythema zz Suggested Reading
multiforme has a characteristic pattern 55 Katholi B, Daghstani SS, Banez GA, Brady
on the skin and is usually not this focal or KK. Non-invasive treatments for pediatric
lateralized. Finally, cellulitis does not pro- complex regional pain syndrome – a
duce episodic symptoms. focused review. PM R. 2014. pii: S1934-
1482(14)00175-0. 7 https://doi.
zz Suggested Reading org/10.1016/j.pmrj.2014.04.007. [Epub
55 Fukushima FB, Bezerra DM, Villas Boas ahead of print] PMID 24780851.
PJ, Valle AP, Vidal EI. Complex regional 55 Goldschneider KR. Complex regional pain
pain syndrome. Brit Med J. syndrome in children: asking the right
2014;348:g3683. questions. Pain Res Manag. 2012;17:
386–90.
11 ?? 39. An 8-year-old girl fell off her Bick on
the sidewalk. The child’s right leg hit ?? 40. A 9-year-old boy who was involved
the sidewalk, scraping the skin and in a car accident presents to the ED,
bruising it, and the Bick toppled on is complaining of worsened progres-
top of her, but did not cause serious sively for the last 2 weeks. There is no
injury. She cried briefly but walked history of loos of consciousness and
back to her house. Two weeks later, was not seen immediately after the
she began refusing to walk on that accident. He has a history of intermit-
leg. Her parents noticed that the leg tent headaches, typically in the back
would become bright red, hot, and of his head and neck for 1 year. Now,
swollen from the knee down and stay he has daily headache, and over the
that way for hours at a time. counter medication does not of any
The child might benefit from: help. He cannot characterize the head-
A. Bed rest and pampering. ache quality, but denies any has pho-
B. Antibiotics and topical vasocon- tophobia or phonophobia. Yesterday,
strictors. his headache has intensified after hav-
C. Physical therapy ing a bout of coughing secondary to a
D. Oxycodone and prednisone. flare up of his asthma. His past medical
E. Casting and avoidance of weight history is unremarkable beside his
bearing. asthma. Family history is remarkable of
Headache
363 11
migraine in his father (mainly occipital mother to pick him up and he missed
headaches) and the paternal side of a lot of school days. When he has the
the family. He has a normal neurologic headaches, he prefers to lie in a dark,
exam, including funduscopic exam. quite room. Also, vomiting relieves
What is your next step in the evalua- the headache. He used to use the
tion and management of this case? over-the-counter medications such as
A. amitriptyline 10 mg nightly acetaminophen or ibuprofen, but they
B. indomethacin every 8 h as needed are not helping any more. Because his
C. MRI of the brain frequent absences, his school grades
D. reassurance, no intervention have been dropped. There is a family
needed unless headache fre- history of migraines in his mother. He
quency persists for another month has a normal neurologic exam.
E. spinal tap for opening pressure What is your next step in his evalu-
and cerebrospinal fluid analysis ation?
A. CT scan of the sinuses
vv Correct Answer is: C B. EEG
Occipitally located migraine in adults C. MRI of the brain with and without
is common, but in children, occipital gadolinium
headache always increase concern to D. perform lumbar puncture
find a potential secondary etiology. E. reassure mother and child and
Headache increases with cough (Valsalva- begin treatment with a triptan
like maneuver) can indicate either an
increased intracranial pressure, or head- vv Correct Answer is: E
aches associated with a Chiari I malforma- This is a case of migraine headaches with-
tion. Posterior fossa tumors (the most out aura. He has intermittent headaches
common location for primary brain tumors that meet the IHS criteria for migraine.
in children) often present with pain in the He prefers to lie in a quiet dark room,
occipital region as well. Cough headache because movement can trigger the head-
in adults can be treated with daily indo- ache. Also, he has vomiting associated
methacin, though this primary headache with the headaches.
disorder is also unusual, and up to 25% of According to the AAN Guidelines for
adults with headache induced with cough- the evaluation of episodic headaches
ing have been found to have a Chiari in adolescents an imaging procedure
malformation on imaging. Neither acute would only be indicated in patients
treatment with sumatriptan, nor chronic with a recent change in the type of
treatment with amitryptline would be headache; associated features of neu-
indicated without a more clear history of rological dysfunction; or if there are
migraine and exclusion of a more obvious neurologic abnormalities noted on
cause of headache in this patient. examination.
In addition, the Guidelines did not
?? 41. You are evaluating a 14-year-old recommend an EEG as part of the evalu-
boy with 1-year history of headache. ation of any child with episodic head-
The headache has been increased in aches.
frequency and intensity in the last According to the AAN Guidelines
3 months. The school has to call his of the Pharmacological Treatment of
364 Chapter 11 · Headache

Migraines in Children and Adolescents, nausea and vomiting near the onset.
sumatriptan nasal spray was found to be This morning, it was difficult to arouse
effective in the treatment of migraines her from a nap so her family brought
in adolescents and children. A recent her for evaluation. Her medical history
study also found that almotriptan was is remarkable for small stature. Fam-
effective in treating migraines in chil- ily history is remarkable for acquired
dren. His history is not suggestive of hearing loss, type II diabetes and early
sinus disease nor CNS infection. onset heart disease on her maternal
side. Her general examination is sig-
zz Suggested Reading nificant for weight and height in the
55 Linder SL, Mathew NT, Cady RK, et al. 5rd percentile, with head size at the
Efficacy and tolerability of almotriptan in 20th percentile. Blood pressure and
adolescents: a randomized, double-blind, pulse are normal. On your exam, she
placebo-controlled trial. Headache. was drowsy, though with stimulation
2008;48:1326–36. she awakens, has fluent speech and
55 Lewis, DW, Ashwal S, Dahl G, et al. can follow limited directions. Cranial
Practice parameter: evaluation of children nerve examination show evidence of
and adolescents with recurrent headache. a left homonymous hemianopsia on
Neurology. 2002;59(4):490–8. confrontation testing, but is otherwise
normal. The rest of her exam was nor-
mal. Laboratory studies show a normal
?? 42. An 8-year-old girl is brought to the ED CBC; a comprehensive metabolic pro-
2 days after having an acute onset of file with serum bicarbonate of 14; and
headache that has not relived by over- an increased anion gap. Serum lactate
the-counter medications use. Her PCP returns at 4.5 mmol/L (normal less
11 suspected migraine because of the than 2.3). Her CT and MRI are shown.

..      Fig. 11.4 Imaging revealed an apparent stroke in the right posterior cerebral artery distribution. American
Academy of Neurology Institute, produced by permission
Headache
365 11
Which of the following results are was felt to be a PCA distribution infarc-
most likely given her presentation tion.) Recovery from the “stroke” is often
and imaging findings? quite good. Diagnosis relies initially on
A. cerebral angiography showing elevated lactate and pyruvate in the blood
diffuse irregular narrowing and or cerebrospinal fluid. Muscle biopsy com-
beading monly reveals ragged-red fibers (even if
B. hemoglobin electrophoresis there is not clear evidence of a myopathy
showing 90% hemoglobin S on exam), and specific mutations can
C. magnetic resonance angiogra- be searched for in plasma or muscle in
phy showing abrupt tapering of multiple mitochondrial genes. There is no
the right carotid artery in specific treatment for this mitochondrial
the neck disorder, though supplementation with
D. muscle biopsy showing ragged- vitamins (to include coenzyme Q10, thia-
red fibers mine and carnitine) and l-arginine may be
E. skin biopsy showing electron- somewhat protective in decreasing the
dense granules in the media of severity of the stroke-like events. There
arterioles on electron microscopy is some evidence that intravenous infu-
sion of L-argining early in the “stroke” can
vv Correct Answer is: D reverse the process and result in complete
This patient, though initially thought to recovery. Ischemic stroke is very rare in
have primary headache, has focal neu- children outside of the immediate perina-
rologic findings several days into the tal time frame. The placenta is a wonderful
headache prompting imaging. Imaging clot former and due to the patent foramen
revealed an apparent stroke in the right ovale in normal prenatal cardiac physiol-
posterior cerebral artery distribution. Her ogy, there is a natural right to left shunt
CT scan also shows subtle hyperdense for clots to enter the arterial circulation. It
lesions of the basal ganglia, likely calcifica- is very uncommon (less than 2–5% likely)
tions. Given the lactic acidosis noted on that an additional stroke risk factor will
presentation, short stature of the patient be identified in such perinatal infarctions.
and family history of cardiomyopathy, Conversely, stroke occurring outside the
hearing loss and type II diabetes in mater- neonatal time frame is often secondary to
nal relatives, a mitochondrial disorder another disorder including coagulopathies
would be the most likely diagnosis, in and hemoglobinopathies (sickle cell dis-
particular mitochondrial encephalopa- ease in particular), congenital or acquired
thy, lactic acidosis and stroke (MELAS). heart disease, neck or head trauma, vas-
This multisystem disorder typically has culitis (primary CNS or secondary) and
onset between 2 and 10 years of age, with multiple vasculopathies to include Fabry
normal early developmental milestones. disease, homocystinuria, MELAS and moy-
Initially seizures, recurrent headaches, amoya disease or syndrome. The workup
intermittent ataxia and focal neurologic needs to be extensive, unless particular
deficits arising from stroke-like episodes clues from the personal or family history
predominate. Eventually there is progres- suggest a specific ­diagnosis.
sive loss of function due to the after-
effects of the metabolic disturbance of the zz Suggested Reading
brain. The initial attacks often involve the 55 Tarnopolsky MA. Mitochondrial cytopa-
occipital lobe, with infarctions not always thies in children and adults. Continuum
conforming to definitive vascular territo- Lifelong Learning Neurol. 2009;5(6):
ries (though this patient did have what 98–125.
366 Chapter 11 · Headache

55 Fenichel GM. Clinical pediatric neurology. zz Suggested Reading


6th ed. Philadelphia: Saunders Elsevier; 55 Liu AM, Liu JG, Liu GW, Liu GT. Alice in
2009;256–63. wonderland syndrome: presenting and
follow-up characteristics. Pediatr Neurol.
2014;51(3):317–20.
?? 43. A 10-year-old girl is referred to you 55 Illik F, Illik K. Alice in Wonderland
because of several spells of altered syndrome as aura of migraine. Neurocase.
perception. She has three spells 2014;20(4):474–5.
which she experienced nausea and 55 Hamed SA. A migraine variant with
headache and perceived people in abdominal colic and Alice and
her immediate environment as being Wonderland syndrome: a case report and
very near, but she had no loos of review. BMC Neurol. 2010;10:2. 7 https://
consciousness. She is doing excellent doi.org/10.1186/1471-2377-10-2.
at school. She denies the use of recre-
ational drug. Her general, neurologic ?? 44. Which of the following can be caused
examinations are normal, as well as by defects in a calcium channel gene?
her brain MRI and EEG. A. Familial hemiplegic migraine
What is the most likely diagnosis? B. Myotonia congenital
A. Alice in wonderland syndrome C. Hyperkalemia periodic paralysis
B. Conversion disorder D. Paramyotonia congenital
C. Delusional disorder
D. Focal seizures with dyscognitive vv Correct Answer is: A
features Mutations in calcium channel can cause
E. Schizophrenia familial hemiplegic migraine. It is a
voltage dependent, P/Q type, alpha 1A
11 vv Correct Answer is: A subunit (CACNA1A) which codes for a
Alice in Wonderland syndrome, is a calcium channel. Mutations in the same
benign syndrome usually related to gene can cause alternating hemiplegia
migraine and typically affecting young of childhood, episodic ataxia type 2, and
children. During spells, patients may spinocerebellar ataxia type 6. Familial
perceive objects or other people as being hemiplegic migraine can also be caused
larger, smaller, broader, thinner, nearer or by mutations in Sodium Channel, Voltage
farther away than they really are. Altered Gated, Type I Alpha Subunit) (SCN1A1)
perception of body image may also occur. or ATPase, Na+/K+ Transporting, Alpha 2
Migraine headaches may occur before, Polypeptide (ATP1A2).
during, or after the perceptual altera- Mutations in the gene CLCN1 which
tion. Evaluation for hallucinogenic drugs, codes for a chloride channel can cause
even if denied, is very important. The mytonia congenita. A condition which is
patient is quite young to be presenting characterized by a “warm up phenome-
with schizophrenia or delusional disorder, non” where repeated motions improves
which would also require indications of a symptoms. This can be inherited in an
thought disorder, which is not specified autosomal dominant or in an autoso-
in this case. Focal epilepsy, while impor- mal recessive manner; in each case, the
tant to exclude, is less likely based on the same gene is implicated.
normal EEG and absence of altered con- Hyperkalemic periodic paralysis is
sciousness. Conversion disorder would be caused by mutations in Sodium channel,
a diagnosis of exclusion. Voltage Gated, Type IV Alpha Subunit
Headache
367 11
(SCN4A) are implicated in up to 70% of Looking out the window while driving in
cases of. This is characterized by epi- the car can minimize this affect. Younger
sodes of weakness exacerbated by high children in the back seat typically have
potassium food, cold exposure, and visual input only of the car interior. A
moderate exercise followed by rest. higher set seat with greater window expo-
Mutations in (Calcium Channel, sure, such as in a minivan, would reduce
Voltage­Dependent, L Type, Alpha 1S the incidence of motion sickness as the
Subunit) CACNA 1S may underlie up to child can watch the environment pass in
70% of cases of hypokalemic periodic the direction opposite of their body.
paralysis. Weakness can be triggered by An eye mask would eliminate the
carbohydrate rich meals or by exercise visual input of the motion in the direc-
after a period of rest. tion opposed to the body movement and
Paramyotonia congenita is similar increases motion sickness symptoms.
to myotonia congenita; however, the Focusing on a book is providing
warm up phenomenon is not present stationary input compared to the move-
and symptoms may actually be exac- ment of the body and will increase
erbated by repeated movement. Muta- motion sickness symptoms.
tions in Sodium Channel, Voltage Gated, Focusing on a video player again is
and Type IV Alpha Subunit (SCN4A) are providing stationary input compared
implicated. to the movement of the body and will
increase motion sickness symptoms.
zz Suggested Reading
55 Knupp K, Parsons JA, Brooks-Kayal zz Suggested Reading
AR. Channelopathies. In: Swaiman K, 55 Golding JF, Gresty MA. Pathophysiology
Ashwal S, Ferriero OM, Ferriero D, and treatment of motion sickness. Curr
editors. 5th ed. Edinburgh: Elsevier Opin Neurol. 2015; 28(1):83–8.
Saunders; 2012: 544–53.
?? 46. What is the recommendation of the
?? 45. An 8-year-old girl complains of dizzi- American Academy of Neurology and
ness and frequently vomiting while The Child Neurology Society recom-
riding in back seat of her mother car. mendations in the evaluation of a
She has normal development, general 12-year-old girl with a 15-month his-
and neurological exams. tory of frontal headaches and a non-
What is your r­ ecommendation to the focal neurological exam?
parents? A. Non-contrast CT
A. She should wear an eye mask B. MRI with venogram
while the car is moving C. EEG
B. Make sure she can see forward out D. No study required
the window
C. She should read in the car vv Correct Answer is: D
D. Provide DVD video player for her Brain imaging in children with a history
to watch of chronic headaches is not cost effective
because there is minimal chance of iden-
vv Correct Answer is: B tifying a treatable intracranial pathology
Motion sickness can be induced by motion if a thorough history, general physical
in the visual filed opposed to the actual and neurological examinations are nor-
forward movement of the body in the car. mal and non-­focal.
368 Chapter 11 · Headache

Non-contract CT is rarely helpful in 2. Disrupted ion homeostasis, release of


the evaluation of chronic headaches, neurochemicals, and transient
with the exception of headaches sec- dysfunction of neuronal function.
ondary to chronic sinusitis. In this case 3. Meningeal blood vessel dilation and
the general physical examination and activation of trigeminovascular
history would be suggestive of sinusitis- system.
associated symptoms. In addition, non- 4. Release of vasoactive neuropeptides
contrast CT is also preferred in cases of (calcitonin gene-related peptide
acute onset headache when intracranial (CGRP), neurokinins, prostaglandins,
bleeding is suspected. substance P, etc.) from activated
MRI with MRV is utilized in suspected trigeminal sensory nerves leads to
venous sinus thrombosis when increased sterile neurogenic inflammation.
intracranial pressure is present. 5. Worsening vasodilation, increasing
EEG changes are not common, are firing of trigeminal afferents causing
non-­specific and play no role in the pain intensification.
diagnosis or management of headaches. 6. Trigeminal nociceptive afferents carry
pain signals to trigeminal nucleus
zz Suggested Reading caudalids (TNC) for processing and
55 Hershey AD. Pediatric headache: update ascent through thalamus to cortex
on recent research. Headache: J Head Face 7. Continuous ascending pain signals
Pain. 2012;52(2):327–32. activate more neurons leading to
associated symptoms such as photo/
?? 47. Which of the following would be the phonophobia, nausea, and vomiting.
earliest step in the proposed develop- 8. Continuous TNC firing, leads to central
ment of migraine? sensitization if activated pathways are
11 A. Release of vasoactive neuropep- not stopped.
tides from trigeminal sensory
nerves zz Suggested Reading
B. Meningeal blood vessel dilation 55 Silberstein SD, Lipton RB, Dodick O, et al.
C. Activation of ­trigeminal sensory Wolffs headache and other head pain. 8th
afferents in the meningeal vessels ed. New York: Oxford University Press;
D. Cortical spreading depression 2008.
E. Meningeal blood vessel
­constriction ?? 48. An 18-year-old girl complains of
headaches, blurred vision, nausea,
vv Correct Answer is: D and vomiting. She has no previous
Cortical spreading depression is felt history of migraines, and had no
to occur first. However, it is uncertain known thrombotic disorders. Fundus
whether the trigger for cortical spreading examination showed papilledema.
depression occurs in the brain stem or MRI was normal, and she has elevated
within the cortex. cerebrospinal fluid pressure.
The pathophysiology of migraine is This disorder may be associated with:
as follows: A. Oral contraceptive use
1. Activation of hypersensitive “central B. Hypervitaminosis A
generator” (it is debated whether the C. Hypoparathyroidism
initiating trigger for migraine occurs in D. Obesity
the cortex or in the brain stem). E. All of the above
Headache
369 11
vv Correct Answer is: E exam is normal. M RI and MRV of the
The patient has pseudo tumor cerebri, brain are normal as well. You suggest
also called idiopathic intracranial hyper- initiating this patient on a preventive
tension or benign intracranial hyper- medication.
tension. It is a disorder of chronically What is the safest medication to be
elevated intracranial pressure, resulting used?
in papilledema, which may progress to A. Amitryptiline
sub-retinal hemorrhages, optic atrophy, B. Topiramate
and blindness. Symptoms include head- C. Valproic acid
ache, diplopia, visual loss and distor- D. Atenolol
tion (typically peripheral), nausea, and E. Lithium
vomiting. The disorder is often idiopathic
and commonly occurs in women of child- vv Correct Answer is: A
bearing age with obesity. Pseudo tumor Amitryptiline is a pregnancy category
cerebri is also associated with a num- C. It has a very low level in the breast
ber of conditions, including endocrine milk. There is a conflicting data regard-
dysfunction (e.g. hypoparathyroidism, ing limb deformities on high doses of
Addison’s disease, and Cushing’s disease), amitryptline, but there is no association
pregnancy, oral contraceptive use, ste- reported with doses of 10–50 mg a day,
roid withdrawal, hypervitaminosis A, and which is what is used as preventive dos-
tetracycline therapy. The goal of treat- ing. It is recommended that the dose be
ment is to preserve vision and decrease tapered 3–4 weeks prior to the delivery.
intracranial pressure. Otherwise, babies should be monitored
Medical management may include for drowsiness, hyper excitability, and
acetazolamide and steroids. Surgical suckling problems.
approaches include optic nerve sheath Topiramate is associated with cleft
fenestration and ventricular shunting. palates and cleft lips and also hypospa-
Historically, patients were treated with dias.
repeated lumbar punctures. Valproic acid is associated with neu-
ral tube defects as well as lower IQ in
zz Suggested Reading children exposed in utero.
55 Gans MS. Idiopathic intracranial hyper- Atenolol is a pregnancy category
tension. 7 http://emedicine.medscape. D. It is associated with babies with low
com/article/1214410-overview. Updated weight for gestational age.
5/2/14 accessed 5/29/14. Lithium is teratogenic and is a cat-
55 Simon RP, et al. Clinical neurology. 4th ed. egory D in pregnancy.
Stamford, CT: Appleton & Lange. p. 91, 152.
zz Suggested Reading
?? 49. An 18-year-old 34 weeks pregnant 55 MacGregor, A. Headache in pregnancy.
female is complaining of daily head- Continuum. Am Acad Neurol. Review
aches that are unilateral, throbbing, Article. 2014;20(1):128–47.
associated with nausea, vomiting,
photophobia and phonophobia, last-
ing for about 4–5 h every day. These ?? 50. Psychosocial factors that differ in
headaches are not different from adolescents with migraine headaches
the headaches that she used to have compared to age matched controls,
before her pregnancy. Her neurologic include which of the following?
370 Chapter 11 · Headache

A. A More often have divorced Approximately 10% of children with


­parents SLE and mixed connective tissue disease
B. Have social skill deficient will experience severe headaches and
C. Experience more school problems may be the presenting feature.
D. Families are more likely to be in Treatment includes corticosteroids.
the lower socioeconomic class Asthma, type I diabetes and osteo-
E. There are no significant psychoso- genesis imperfecta are seen in children
cial differences but are not associated with vasculitis.
Cerebellar astrocytoma represents
vv Correct Answer is: E approximately 20% of all brain tumors
Headaches are common in children and up in children, much less common in
to 80% of adolescent’s report experiencing adults, are typically low grade with a
a headache before the age of 14. Psycho- good prognosis and rarely associated
social factors in children with migraine do with vasculitis.
not differ significantly from headache-free
controls, but are absent more days from zz Suggested Reading
school on average. In contrast, children with 55 Malattia C, Martini A. Pediatric-onset
tension-type are more likely to have prob- systemic lupus erythematosus. Best Pract
lems with family (primarily divorce) and Res Clin Rheumatol. 2013;27(3):351–62.
housing conditions, school and relations in
peer groups. ?? 52. Which of the following medications
is specifically FDA-­approved for the
zz Suggested Reading treatment of acute migraine attacks in
55 Karwautz A. Psychosocial factors in pediatric patients?
children and adolescents with migraine A. Codeine
11 and tension-type headache: a controlled B. Frovatriptan
study and review of the literature. C. Frovatriptan with acetaminophen
Cephalalgia; 1999;(19)1:32–43. D. Diazepam
E. Sumatriptan and naproxen in a
?? 51. Headache from cerebral vasculitis is single oral formulation
uncommon in children, but occurs in
children with which disease? vv Correct Answer is: E
A. Type I diabetes Treximet is a combination of sumatriptan
B. Asthma and naproxen and was approved for pedi-
C. Cerebellar astrocytoma atric use in 2015, 7 years after approval
D. Systemic lupus erythematosus for adults. The recommend dose for
E. Osteogenesis imperfecta children 12 years and older is one tablet
with1 0 mg of Sumatriptan and 60 mg
vv Correct Answer is: D of naproxen sodium (10/60 mg) and the
Headaches are frequently seen in the maximum dose is 85/500 mg.
course of a child with a connective tissue Zolmitripan (Zomig) and almo-
disease, such as systemic lupus erythe- triptan (Axert) are triptans also FDA-
matosus (SLE). approved for children 12 and older.
Connective tissue disease related Frovatriptan is used in adults and
vasculitis occurs as a result of hypersen- off-­label in children. Narcotics are com-
sitivity of cerebra l vessels to an autoim- monly used in emergency department
mune process. settings for the treatment of severe
Headache
371 11
childhood migraines, but are not spe- zz Suggested Reading
cifically FDA-approved for this indica- 55 Mowzoon N, Flemming KD. Neurology
tion. board review: an illustrated study guide.
Benzodiazepines are not effective in Rochester: Mayo Clinic Scientific; 2007.
migraine headaches. Print.

zz Suggested Reading ?? 54. Which of the following symptoms is


55 Freitag FG, Shumate D. Current and a “red flag” in the diagnosis of child-
investigational drugs for the prevention of hood migraine that may suggest
migraine in adults and children. CNS more serious intracranial pathology?
Drugs. 2014;28(10):921–7. A. Head ache in a child less than
4 year of age
?? 53. An 18-year-old obese female came B. A consistent headache pattern
with a 5-month history of intermit- over a one-year period
tent headaches. The pain is character- C. Headaches associated with vomit-
ized as pressure-like and throbbing, ing
but without photophobia. Headaches D. Headache associated with dis-
are not associated with nausea or torted vision
vomiting. She reports that she is E. Pain correlates with episode of
able to “hear my pulse in my head” vertigo
and occasionally has blurred vision,
especially in the morning when she vv Correct Answer is: A
awakens. Her past medical history is Headaches, head pain and migraine
significant only for severe menstrual are rare in children under the age of 4
cramping, for which she was recently and warrant neuroimaging to rule-out
started on an oral contraceptive. causes of increased intracranial pressure.
She has no papilledema on fundus Headaches occurring at an increasing
exam. Rest of the neurological exam frequency or increasing severity over
is normal. several weeks in a child under the age
What is most likely the diagnosis? of 12 are not characteristic of a migraine
A. Classic migraine pattern. Headaches with vomiting upon
B. Migraine without aura awakening suggest increased intracranial
C. Benign intracranial hypertension pressure. Transient visual auras occur in
D. Subarachnoid hemorrhage classic migraines and benign paroxysmal
E. Tension headache vertigo is a migraine variant in young
children and is associated with nausea,
vv Correct Answer is: C vomiting and nystagmus.
The transient visual loss is most com-
monly seen in patients with benign intra- zz Suggested Reading
cranial hypertension than migraines, and 55 Raieli V, et al. Migraine in a pediatric
results from changes in intracranial pres- population: a clinical study in children
sure that can relate to position. Retinoids, younger than 7 years of age. Dev Med
vitamin A toxicity, tetracycline, lithium, Child Neurol. 2015;57(6):585–8.
and oral contraceptives use is associated
with pseudo tumor cerebri. Obesity is ?? 55. Which of the following statements
also a strong risk factor for benign intra- regarding migraine headaches in chil-
cranial hypertension. dren is true
372 Chapter 11 · Headache

A. The prevalence of migraine in chil- month, but in the last year they have
dren 7 and younger is 2.5% occurred twice a week. The head-
B. It is a hereditary disorder trans- aches are severe enough to prevent
mitted by autosomal recessive her from attending school, and
inheritance engaging in social activities. Her past
C. A history of migraine in at least medical history was significant for an
one parent is 50% allergy to sulfa, and she has a family
D. Boys and girls are affected equally history of kidney stones. Her neuro-
in all pediatric age groups logic examination is normal, and she
E. About one-quarter will continue has no papilledema.
to have migraines into What is the appropriate next step in
adulthood management?
A. lumbar puncture
vv Correct Answer is: A B. Start amitriptyline
In pediatric neurology practice 75% of C. Head MRI and MRA
headaches in young children referred for D. Start Topiramate
neurological consultation are Migraines E. Start Valproic acid
headaches. Muscle tension and psycho-
genic headaches are less likely to be vv Correct Answer is: B
experience in children. Under the age This is a female with frequent migraine
of 7 is the only age category in children headaches, without recent change, and
that both sexes are affected equally. a normal examination. An MRI scan is
The incidence of migraines in ages 7 to unlikely to reveal a treatable cause in
puberty is 5%, with a female predomi- patients with chronic (>6 months) head-
nance of 3:2. In post-pubertal boys the aches, with no focal neurologic deficits,
11 incidence is 5% and in girls it is 10%. papilledema or a history of seizures.
There is a strong autosomal dominant Similarly, the MRA or CSF studies would
hereditary pattern and there is a history unlikely be of value.
of migraines in at least 90% of parents. Preventative or prophylactic therapy
Approximately three­quarters will con- are daily medications to be taken by the
tinue to experience migraines by the patient to decrease the headache fre-
age of 25 and half will continue until the quency. More than 20 medications have
age of 50. been used for migraine prevention, but
we have no studies to clearly indicate
zz Suggested Reading which would be the optimal to use in
55 Lewis DW. Pediatric migraine. Neurol children or teenager.
Clin. 2009;27(2):481–501. However, in this patient one would
want to avoid the use of Valproic acid
?? 56. An 18-year-old female with a 5-year because of potential teratogenic side
history of severe migraine head- effects. Topiramate is chemically related
aches. She has a visual aura, she to sulfa compounds, and occasional
has a unilateral temporal headache patients will show an allergic cross-reac-
occurring either on her left or right tivity. Topiramate also predisposes to
side, nausea and photophobia are renal stones. Amitriptyline is frequently
associated with her headache, fall- used in children and teens. With ami-
ing asleep is relieving the headache. triptyline, EKG has to be done to detect
Her headaches occurred once a a prolonged QT interval.
Headache
373 11
zz Suggested Reading C. naproxen 750 mg extended
55 Hershey AD, Powers SW, Bentti AL, release once daily
Degrauw TJ. Effectiveness of amitriptyline D. oxycodone 30 mg twice daily
in the prophylactic management of E. rizatriptan 10 mg twice daily
childhood headaches. Headache.
2000;40:539–49. vv Correct Answer is: C
55 Lewis DW, Ashwal S, Dahl G, Dorbad D, Naproxen 750 mg extended release once
Hirtz D, Prensky A, Jarjour I; Quality daily can be used in mediation-overuse
Standards Subcommittee of the headache require bridge therapy to
American Academy of Neurology; decrease the transient increase in head-
Practice Committee of the Child aches that occurs during discontinuation
Neurology Society. Practice parameter: of the overused agent. The choice of
evaluation of children and adolescents bridge therapy is based on the specific
with recurrent headaches: report of the overused ­treatment and the tolerabil-
Quality Standards Subcommittee of the ity of the acute treatment alternative.
American Academy of Neurology and For patients who overuse sumatriptan,
the Practice Committee of the Child long-acting NSAIDs are recommended
Neurology Society. Neurology. as bridge therapy. Opioids are not
2002;59(4):490–8. ­recommended for this purpose because
55 Lewis D, Ashwal S, Hershey A, et al. of the potential for d
­ ependence. Riza-
Practice parameter: pharmacological triptan would not be an appropriate
treatment of migraine headache in substitute since it, like sumatriptan, is
children and adolescents: report of the a short-acting triptan, although long-
American Academy of Neurology acting triptans such as naratriptan or
Quality Standards Subcommittee and frovatriptan may be appropriate as res-
the Practice Committee of the Child cue therapy.
Neurology Society. Neurology.
2004;63:2215–24. zz Suggested Reading
55 Lewis D, Winner P, Saper J, Ness S, 55 Giamberardino, Mitsikostas, Martelletti.
Polverejan E, Wang S, Kurland CL, Nye J, Update on medication overuse headache
Yuen E, Eerdekens M, Ford and its treatment. Curr Treat Options
L. Randomized, double-blind, placebo- Neurol. 2014;17(8):368.
controlled study to evaluate the efficacy 55 Lipton. Risk factors for and management
and safety of topiramate for migraine of medication overuse headache.
prevention in pediatric subjects 12 to Continuum Lifelong Learning in
17 years of age. Pediatrics. Neurology. 2015;21(4):1118–31.
2009;123(3):924–34.

?? 58. You are evaluating a 17-year-old


?? 57. What drug can be used as substitute obese girl with worsening headache.
for sumatriptan in a patient with She has borderline hypertension. Her
medication-overuse headache who headaches bifrontal and bioccipital
requires bridge therapy? band-like pressure and pain. Occa-
A. butorphanol 2 mg intranasally sionally, she experiences brief visual
every 4 h loss or graying, especially with strain-
B. hydromorphone 4 mg four times ing, which lasts only seconds, but no
daily photophobia or phonophobia. When
374 Chapter 11 · Headache

the pain is severe, she feels her vision condition. Various mechanisms have
is becoming increasingly blurred and been proposed to explain its pathogen-
she will have nausea and vomiting. esis, including decreased CSF absorption,
Which of the following is not a pro- increased CSF production, increased
posed mechanism in the pathogen- intravascular volume, and increased intra-
esis of this condition? cranial venous pressure. Of the possible
A. Decreased CSF absorption etiologies listed, aqueductal stenosis
B. Increased venous pressure is not a cause of idiopathic intracranial
C. Increased CSF ­formation hypertension (llH), although it would be a
D. Congenital stenosis of venous secondary cause of hydrocephalus.
sinuses
E. Aqueduct stenosis zz Suggested Reading
55 Acheson JF, Green WT, Sanders
vv Correct Answer is: E MD. Optic nerve sheath decompression
This is a case of Idiopathic intracranial for the treatment of visual failure in
hypertension (llH), also known as benign chronic raised intracranial pressure. J
intracranial hypertension or pseudotu- Neurol Neurosurg Psychiatry.1994;
mour cerebri, which is a syndrome with 57:1426–29.
signs and symptoms of increased intra- 55 Burgett RA, Purvin VA, Kawasaki
cranial pressure but where a causative A. Lumboperitoneal shunting for pseudo-
mass or hydrocephalus is not identified. tumor cerebri. Neurology. 1997;49:734–9.
The typical demographic are middle-aged 55 Headache Classification Subcommittee of
obese females. Presentation is usually the International Headache Society. The
with headaches, visual problems and international classification of headache
papilledema. Papilledema may not always disorders: 2nd ed. Cephalalgia. 2004;24
11 be present, or may be unilateral, making (Suppl 1):9–160.
the clinical diagnosis less than straight-
forward. Neurological examination is ?? 59. You are asked to see a 9-year-old
usually normal, with a sixth cranial nerve female because she has several spells
palsy sometimes encountered. Additional of altered perception. In three of
clinical findings include normal CSF com- her episodes, she had nausea and
position with elevated opening pressure headache and perceived people in
(opening pressure varies however, with her immediate environment as being
a mean pressure of less than 35 mmHg, larger than they are. She is doing very
with pressure waves as high as 80 mmHg good at school and denies any loss
which last 5–20 min). of consciousness or recreational drug
Treatment options include CSF let- use. Her neurologic examination, MRI
ting, acetazolamide and lumboperitoneal of the brain and EEG are normal.
shunts. In patients with progressive visual What is the most likely diagnosis?
deterioration, optic nerve fenestration A. Alice in wonderland syndrome
may be required to preserve vision. Topi- B. Conversion disorder
ramate can be used if there is a contrain- C. Delusional disorder
dication to Acetazolamide (such as a sulfa D. Focal seizures with dyscognitive
allergy) because of its carbonic anhydrase features
inhibitor effect. llH is a poorly understood E. Schizophrenia
Headache
375 11
vv Correct Answer is: A no response to the oral combination.
Alice in Wonderland syndrome is a The current headache started 5 days
benign syndrome often related to ago and there was no response to the
migraine and typically affecting young oral combination. She ran out of her
children. During spells patients may per- injectable Sumatriptan, so she kept
ceive objects or other people as being using oral Sumatriptan and Ibuprofen
larger, smaller, broader, thinner, nearer or for the last 5 days, but the headache
farther away than they really are. Altered did not improve, so she decide to go
perception of body image may also occur. to emergency department.
Migraine headaches may occur at any She has a stable vital sign and her
time during or after the perceptual altera- neurological exam is normal.
tion. It is important in the evaluation of After initiating IV fluids, what treat-
perceptual alterations to consider the use ment is the most appropriate first
of hallucinogenic drugs even if denied. line therapy in this patient?
The patient is quite young to be present- A. IV prochlorperazine
ing with schizophrenia or delusional B. IV magnesium sulfate
disorder, which would also require indica- C. IV sodium valproate
tions of a thought disorder, which is not D. IV methylprednisolone
mentioned in the vignette. Focal epilepsy, E. IV hydromorphone
while important to exclude, is less likely
based on the normal EEG and absence of vv Correct Answer is: A
altered consciousness. Conversion disor- This is a clinical presentation of status
der would be a diagnosis of exclusion. migrainosus. The proper treatment of
status migrainosus is IV therapy. Dihydro-
zz Suggested Reading ergotamine (DHE) is the gold standard
55 Liu AM, Liu JG, Liu GW, Liu GT. Alice in therapy. Between the choices given here,
wonderland syndrome: presenting and IV prochlorperazine would be the most
follow-up characteristics. Pediatr Neurol. appropriate next step. Anti-emetics
2014;51(3):317–20. parentally are considered first-line phar-
55 Illik F, Illik K. Alice in Wonderland macologic treatment for status migraino-
syndrome as aura of migraine. Neurocase. sus in the ED with high-quality evidence
2014;20(4):474–5. supporting their use. Between antiemet-
55 Hamed SA. A migraine variant with ics, the strongest data exists for prochlor-
abdominal colic and Alice and perazine and metoclopramide.
Wonderland syndrome: a case report and Steroids, including methylpredni-
review. BMC Neurol. 2010;10:2. 7 https:// sone, can be used in status migrainosus,
doi.org/10.1186/1471-2377-10-2. but data does not support its use in the
reduction of headaches.
?? 60. You are evaluating an 18-year-old girl Hydromorphone and other opioids
in the ED with severe headache for should not be used in almost all cases,
the last 5 days. She has a history of as they may lead to dependence and
migraine since she was 9 years of age. increase the headache relapse rate.
Her headaches usually respond to a Magnesium sulfate can be used in
combination of Ibuprofen with oral status but not as a first line treatment.
Sumatriptan. Sometimes she needs to Studies have shown mixed evidence of
use injectable Sumatriptan if she has effectiveness for acute migraine in ED.
376 Chapter 11 · Headache

No placebo-controlled studies have zz Suggested Reading


evaluated the efficacy of valproate 55 O’Sullivan M, Jarosz JM, Martin RJ, et al.
sodium in patients with status migraino- Neurology. 2001;56:628–34.
sus. So it cannot be used as a first line
agent. Although it can be tried if other ?? 62. What is the most reasonable
first line agents (DHE, NSAIDs, antiemet- substitute for sumatriptan in an
ics) have failed. 18 years-old-female patient with
medication-overuse headache who
zz Suggested Reading requires bridge therapy?
55 Continuum. Headache. 2015;21(4): A. butorphanol 2 mg intranasally
1004–1010. every 4 h
B. hydromorphone 4 mg four times
?? 61. You are seeing an 18-year-old male daily
with history of recurrent, severe C. naproxen 750 mg extended
migraine headaches and some pro- release once daily
gressive mild cognitive impairment D. oxycodone 30 mg twice daily
brings. He brought a recent brain MRI E. rizatriptan 10 mg twice daily
which shows multiple white matter
lesions. His mother recently died at 59 vv Correct Answer is: C
from dementia but the autopsy sug- Some patients with mediation-overuse
gested cerebral autosomal dominant headache require bridge therapy to ease
arteriopathy with subcortical infarcts the transient increase in headaches that
and leukoencephalopathy (CADA- occurs during discontinuation of the
SIL). He would like to know if his MRI overused agent. The choice of bridge
shows that he has the disease. therapy is based on the specific overused
11 Affection of what brain area is most treatment and the tolerability of the
supportive of the diagnosis of acute treatment alternative. For patients
(CADASIL)”? who overuse sumatriptan, long-acting
A. Corona radiata NSAIDs are recommended as bridge
B. Posterior limb of internal capsule therapy. Opioids are not recommended
C. External capsule for this purpose because of the potential
D. Anterior limb of internal capsule for dependence. Rizatriptan would not
E. Centrum semiovale be an appropriate substitute since it, like
sumatriptan, is a short-acting triptan,
vv Correct Answer is: C although long-acting triptans such as
Cerebral autosomal dominant arteriopa- naratriptan or frovatriptan may be appro-
thy with subcortical infarcts and leukoen- priate as rescue therapy.
cephalopathy (CADASIL) is an inherited,
autosomal dominant condition caused by zz Suggested Reading
mutations of the Notch3 gene. Affected 55 Giamberardino, Mitsikostas, Martelletti.
individuals usually have migraine, mood Update on medication overuse headache
disturbance, and recurrent strokes, often and its treatment. Curr Treat Options
progressing to subcortical dementia. Neurol. 2014;17(8):368.
T2-weighted hyper-intensity of the tem- 55 Lipton. Risk factors for and management
poral white matter and external capsule of medication overuse headache.
are characteristic findings that may help Continuum Lifelong Learning in
to distinguish the disease. Neurology. 2015;21(4):1118–31.
377 12

Infections

© Springer International Publishing AG, part of Springer Nature 2018


Y. M. Awaad, Absolute Pediatric Neurology, https://doi.org/10.1007/978-3-319-78801-2_12
378 Chapter 12 · Infections

?? 1. A 14-year-old female is brought to nosis. EEG, CT, and MRI are supportive
the ED because she has altered con- studies in the diagnosis. Treatment con-
sciousness, headache and fever of sists of intravenous acyclovir 10 mg/kg
(102 F.) for 2 days. You examined her every 8 h (30 mg/kg/d) for 14–21 days.
and you found no meningeal signs, Idoxuridine was the first antiviral drug
but she requires repeated tactile used to treat HSE; however, it proved to
stimulation to maintain arousal, she is be both ineffective and toxic. Acyclovir
not oriented except to person, makes was demonstrated to be superior to
frequent paraphasic errors, and can vidarabine in a head-to-head trial.
recall just one of three objects after a Neither ceftriaxone, nor solumedrol
distracting task. The rest of the neu- would be appropriate treatments, as
rologic examination is unremarkable. both the clinical presentation and CSF
Lumbar puncture was done and CSF studies suggest viral encephalitis.
analysis shows a red blood cell count
of 25 cells/hpf; white blood cell count zz Suggested Reading
of 120 cells/hpf with a lymphocytic 55 Whitley RJ, Kimberlin DW. Herpes
predominance; glucose of 70 mg/dL; simplex: encephalitis children and
and protein of 95 mg/dL. adolescents. Semin Pediatr Infect Dis.
What is the best treatment to be 2005;16:17–23.
­initiated at this time?
A. Iduroxidine ?? 2. A 2 days old full-term infant has
B. Ceftriaxone been evaluated for irritability and
C. Vidarabine microcephaly. He is a product of
D. Solumedrol uncomplicated pregnancy, labor and
E. Acyclovir delivery for a healthy mother. Head
ultrasonography and CT scan showed
vv Correct Answer is: E enlarged posterior horns of the lateral
12 the clinical presentation is indicative of ventricles and punctuate calcifica-
herpes simplex encephalitis (HSE), which tions which are both periventricular
affects 1 in 250,000 to 1 in 500,000 indi- and diffuse. On neurological exami-
viduals per year in the United States, nation, he has increased tone and
with approximately one-third of cases hyperreflexia. A dilated funduscopic
occurring in patients between the ages exam shows a large area of absent
of 6 months and 20 years. HSE can occur retina with hyperpigmented margins.  
as a primary infection or a reactivation of What is the most likely diagnosis?
the herpes simplex virus (HSV). Patients A. Congenital cytomegalovirus infec-
present with a variety of non-specific tion
findings, including fever, headache, B. Congenital ­toxoplasmosis
altered consciousness, dysphasia, sei- C. Congenital herpes simplex type 1
zures, and/or focal neurologic deficits. D. Congenital rubella
CSF analysis could be normal in 5–10%. E. Congenital syphilis
In majority of cases, CSF analysis shows
an elevated white blood cell count with a vv Correct Answer is: B
lymphocytic predominance and an ele- Congenital toxoplasmosis (CT) is a rare
vated protein. Red blood cells are often disorder despite the ubiquitous pres-
present but are non-­diagnostic. PCR ence of the parasite in the environment
form the CSF has a sensitivity of 94% and and high incidence of sero-positivity in
specificity of 98% in confirming the diag- the population. Domestic cats are the
Infections
379 12
reservoir and human transmission she has been sick with intermittent
occurs from oral ingestion of ova in fecal respiratory symptoms for the last few
contaminated dirt or litter boxes. months. Today, he starts to have dou-
Infected mothers during pregnancy will ble vision and he becomes dysarthric.
transmit the congenital infection to His brain CT shows enlarged ventri-
their babies in 50–80%, via hemaotgen- cles without any obvious mass lesion.
ous spread of the parasite to the pla- On examination, he has bilateral pap-
centa and fetus. Only 5% of children will illedema, an incomplete bilateral 6th
have neurological sequelae, approxi- CN paresis and mild right facial weak-
mately 35% develop chorioretinitis by ness. He has trouble swallowing liq-
age 12 with many lesions being subclini- uids. You performed a LP and shows
cal. In the US, Massachusetts routinely 250 WBC’s (lymphocyte predominant),
screens newborns for the condition with protein of 350 mg/dL and glucose of
a yearly incidence of new cases of 0.8 30. Opening pressure was 300 mm.
per 10,000. Treatment is not known to Routine gram stain and bacterial
prevent chorioretinitis or neurologic culture of the cerebrospinal fluid are
injury but this is debated. The case pre- negative at 48 h. His brain MRI shows
sented represents the minority of CT enhancement of the basilar meninges
cases with only 1 of 22 cases reported in and two nodular enhancing lesions in
a 2-year period in the UK having this the temporal lobes.  
constellation of signs and symptoms. What are you going to order on the
The calcifications in CT is diffuse remaining spinal fluid?
involving the cortex and may also be A. Enterovirus PCR
periventricular with hydrocephalus, but B. Herpes simplex virus PCR
calcification in congenital CMV involve C. Human herpesvirus PCR
the periventricular region only. Mater- D. Lymphocytic choriomeningitis
nal prenatal screening has been sug- virus PCR
gested and is carried out in France for E. Mycobacterium tuberculosis PCR
example, however, opponents point out
that it is expensive, raises anxiety in vv Correct Answer is: E
positives and there is no evidence that this is a case of basilar meningitis con-
maternal treatment prevents CT. The firmed by the neurologic and radio-
prognosis for a child affected as above is graphic findings. The cranial nerve
poor. palsies typically develop from a thick-
ened exudate at the base of the brain
zz Suggested Reading that either entraps the cranial nerve
55 Gilbert R, Tan H, Cliffe S, Guy E, and itself or leads to a vasculitis and isch-
Stanford M. Symptomatic toxoplasma emia of the nerve. This exudative pro-
infection due to congenital and postnatally cess suggests a more localized
acquired infection. Arch Dis Child inflammatory process, seen more com-
2006;91:495–8. monly with tubercular, Lyme, fungal
and parasitic meningitis. This process is
also seen in cases of sarcoidosis, neo-
?? 3. You have been consulted on a 4-year- plastic meningitis and some forms of
old boy for his headache, progres- CNS vasculitis. Basilar meningitis is
sive vomiting and fever for 10 days. extremely rare with viral meningitis.
His mother reports that he has no Lymphocytic choriomeningitis virus can
current respiratory symptoms, but cause the intrauterine infection with
380 Chapter 12 · Infections

clinical and radiologic features that can ?? 4. You are evaluating a 5-year-old boy
be confused with the congenital CMV for his behavior changes. He has a
infection, but would not present in the history of mild developmental, but
above fashion. Central nervous system recently he became more subdued
tuberculosis follows the dissemination and less interested to play with his
of tuberculosis after primary infection brother. He used to talk in full sen-
(most commonly the lung). This process tences, now he is speaking in 2-word
can occur at the same time as the pri- phrases. He has no other focal neuro-
mary disease or several years later. logical abnormality except bilateral
Basilar meningitis and at times paren- mild spastic Para paresis. His past
chymal infections called tuberculomas medical history is unremarkable.
(noted on this patient’s MRI scan) can be Recently, his mother was tested posi-
caused by multiplying organisms in the tive for HIV. You ordered ELISA and
subarachnoid space. Hydrocephalus is Western blot studies on the child
also seen frequently. Tuberculosis can which came back positive for HIV. His
also involve the spinal cord, either from CSF studies shows 10 WBCs/hpf,
a vertebral body abscess or an arach- protein 40 mg/dL, glucose 65 mg/
noiditis. Diagnosis of central nervous dL. India ink and AFB stains are
system tuberculosis can be quite chal- negative. PCR and serology for CMV
lenging. Only 5–25% of patients have a is negative. Toxoplasmosis serology
positive acid-­fast stain. Only in 20–40% (IgM and IgG) are negative. Serum
of patients, bacteria by routine single and CSF cryptococcal antigen test-
lumbar puncture (AFB culture) can be ing is normal. PCR of the spinal fluid
isolated. Increasing the volume col- for JC virus DNA is sent and pend-
lected to 10 mL (20 mL or more in adult ing. Brain MRI shows global cerebral
patients) and performing repeated sam- atrophy and bilaterally symmetric
pling of the spinal fluid yield a positive hyperintense signal in the white mat-
12 result in up to 50–80%. PCR assay has a ter on T2-weighted sequences; these
sensitivity on a single sample of same regions appear isointense on
50–75%. Treatment of CNS tuberculosis T1-weighted images.  
is quite challenging. Initial therapy What treatment is most likely to
involves four drugs (combination of iso- improve him clinically?
niazid, rifampin, pyrazinamide and eth- A. Amphotericin
ambutol or streptomycin) for 2 months. B. Gangcyclovir
As sensitivity tests allow, two agents C. Anti-retroviral therapy
can be used as a treatment. Response to D. Isoniazid and rifampin
treatment is usually seen within E. Pyrimethamine and sulfadiazine
2 weeks, though therapy should con-
tinue for 9–12 months. Treatment of the vv Correct Answer is: C
primary TB infection reduces the inci- this is a HIV positive child patient who
dence of CNS infection. A shunting pro- develop signs and symptoms of pro-
cedure of the CSF is occasionally gressive encephalopathy and spastic
required and corticosteroids may diparesis. This is acquired prenatally
reduce the mortality and morbidity of form his mother. The differential diag-
the disease, especially in children with nosis includes many different infectious
altered level of consciousness, papill- agents, as well as primary CNS lym-
edema, focal deficits or elevated intra- phoma. Toxoplasmosis is ruled out
cranial pressure. because of the negative serology and
Infections
381 12
lack of mass lesions on MRI. Tuberculous puted tomography.) Aggressive use of
infection usually produces a striking anti-­retroviral therapy has been shown
basilar meningitis. Although a negative to either slow or reverse the features of
acid-fast stain of the CSF does not progressive HIV encephalopathy in both
exclude this possibility, the child’s clini- pediatric and adult patients.
cal and radiology presentation is not
consistent with CNS tuberculosis. The zz Suggested Reading
laboratory markers for both cryptococ- 55 Barton, et al. The neurologic manifesta-
cal and CMV meningitis were negative. tions of pediatric infectious diseases and
In this case 2 entities have to be con- immunodeficiency syndromes. 2008:157–
sider, progressive multifocal leukoen- 72 AAN Continuum, April 2006, 111–32.
cephalopathy (PML) and HIV
encephalopathy. Both can cause a sub- ?? 5. You are seeing an 8-year-old boy in
acute encephalopthy, but PML is the PICU for his new onset status epi-
unlikely to produce isolated cognitive lepticus. He has a history of a large,
decline, without other focal neurologic tender lymph node in the right neck
deficits (especially visual field loss). for 3 days, and he was on a 14-day
Neuroimaging of PML also shows more course of Augmentin as a treatment.
focal abnormalities, extending into the His mother had difficulty getting him
subcortical white matter, and tend to be to arouse this morning and called
hypointense on T1-weighted images. the EMS. While he was in his way to
The incidence of congenital HIV the ER, he developed right face and
encephalopathy has decreased greatly arm clonic activity, and was given 3
in developed countries since the use of doses of lorazepam and a loading
highly active anti-­retroviral therapy dose of fosphenytoin which stopped
(HAART) in affected women. There are the seizure. You ordered an EEG
multiple presentations for this disorder, showed intermittent non-­convulsive
including a progressive, static or indo- electrographic seizures from the left
lent course. Pre-existing global develop- hemisphere. His brain CT was normal.
mental delay is not unusual in the His CSF showed a white blood count
progressive course. After a varied period of 18 cells/mm3, with normal glucose
of time, the child usually develops a and protein levels. You started him on
combination of behavioral deteriora- vancomycin, ceftriaxone and acyclo-
tion, cognitive regression and motor vir. Herpes simplex PCR testing was
impairment (with both pyramidal and negative.  
extra-pyramidal findings). Diagnosis is Which of the following laboratory
one of exclusion, with neuroimaging tests is most likely to be abnormal?
and spinal fluid examination necessary A. Acid fast bacilli culture of the cere-
to rule out the many other infectious brospinal fluid
and neoplastic processes contributing B. Cerebrospinal fluid for cryptococ-
to encephalopathy in children with HIV cal antigen
infection. The CSF in HIV encephalopa- C. Repeat cerebrospinal fluid PCR for
thy is often normal, though can have a HSV I/II
mildly increased cell count and protein D. Serum Bartonella henselae titers
level. The most common radiographic
findings include cortical atrophy, non- vv Correct Answer is: D
specific white matter changes and basal Skin abrasion or scratch from a young
ganglia calcifications (best seen on com- kitten can cause Bartonella infections.
382 Chapter 12 · Infections

The most striking manifestations the tuberculosis infections of the central


prominent lymphadenopathy in the cer- nervous system would typically present
vical, axillary or inguinal region. Other in an immune compromised host.
systemic symptoms such as fever, Although atypical mycobacterial infec-
anorexia, fatigue and headache may tions can lead to prominent cervical
also be present. Atypical cases with neu- lymphadenopathy, this feature is not
rologic features can occur. present in M. tuberculosis infection.
Encephalopathy (presenting with men-
tal status changes or new-onset sei- zz Suggested Reading
zures, often status epilepticus) is the 55 Barton, et al. The neurological manifesta-
most common neurologic presentation, tions of pediatric infectious diseases and
representing about 2% of cases of cat immunodeficiency syndrome. 2008:243–50.
scratch disease in the largest case series
reviewed. Less likely neurologic presen- ?? 6. You are seeing a 10-year-old boy in
tations include acute or subacute mon- the ED with new-­onset seizure activ-
ocular blindness due to retinitis, a ity and 1 month of headaches. He is
peripheral neuritis resembling Guillain- not febrile. He moved with his family
Barre syndrome, meningitis and a trans- recently form South America. His neu-
verse myelitis. Fever is not necessarily rologic examination is unremarkable.
present during the encephalopathy and EEG is characterized by continuous,
CSF may not show pleocytosis. Imaging polymorphic delta slowing in the left
studies are typically normal but hyperin- frontal region. His brain MRI is shown.  
tense lesions on T2-weighted and FLAIR
MRI in the deep grey structures and
rarely the cortex can be seen. A consid-
eration of the disease should be main-
tained in anyone with acute or subacute
12 encephalopathy and either lymphade-
nopathy and/or recent kitten exposure.
The benefits of multiple antibiotic in
patients with focal brain lesions or
encephalitis from bartonella infections
are uncertain. Spontaneous resolution is
the rule for this disease, though rarely
chronic epilepsy may develop. Seizures
and status epilepticus are not common
in Lyme disease. Subacute or chronic
meningitis and unilateral or bilateral
facial nerve palsies are the most com-
mon neurologic presentations in Lyme
disease. Rarely, initial PCR testing for
..      Fig. 12.1 Neuro-cysticercosis (NCC). American
herpes simplex encephalitis can have a Academy of Neurology Institute, ­produced by
false negative result. In cases with a permission
high index of suspicion, especially with
focal seizures and hemorrhagic infarc- Which test are you going to order to
tions in the temporal or orbital frontal confirm his diagnosis?
cortices, repeat PCR testing should be A. Blood culture
considered. Both cryptococcal and M. B. HIV test
Infections
383 12
C. ELISA for serum IgG antibodies to 55 Matthaiou DK, Panos G, Adamidi ES,
T. solium Falagas ME. Albendazole versus
D. Sedimentation rate Praziquantel in the treatment of neurocys-
E. Stool analysis ticercosis: a meta-analysis of comparative
trials. PLOS N`egl Trop Dis.
vv Correct Answer is: C 2008;2(3):e194.
Neuro-cysticercosis (NCC) is a parasitic
disease caused by the larval form of the ?? 7. You have been asked to see a 5-day-
pork tapeworm, Taenia solium. It is the old baby girl in the NICU for possible
most common parasitic disease of the seizures. She was small for gestational
nervous system and the main cause of age and born at 32 weeks gestation.
acquired epilepsy in developing coun- On day 2 of life, she has had feeding
tries. There are four subtypes of NCC difficulties and has received photo-
based on where the larvae lodge— therapy for jaundice. On your exam,
parenchymal, subarachnoid/cisternal, she is micro cephalic, has hepato-
intraventricular, and spinal. You have to splenomegaly, and a purpuric rash.
consider the following in your differen- You ordered head ultrasound which
tial diagnosis, tuberculoma, pyogenic shows ventriculomegaly. Her blood
brain abscesses, fungal abscess, toxo- labs elevated aminotransferases,
plasmosis, a primary or metastatic direct and indirect hyperbilirubine-
brain tumor and infectious vasculitis. mia, thrombocytopenia, and hemo-
When there is a clinical suspicion for lytic anemia.  
NCC, the next step is to obtain an Which test is most likely to confirm
enzyme-linked immunosorbent assay this patient’s diagnosis?
(ELISA) for serum IgG antibodies to T. A. Bone marrow biopsy
solium. Since there is cross-reactivity B. Head CT scan
between cysticercosis and echinococ- C. Ophthalmologic examination
cus antibodies (approximately 23%) a D. Rapid plasma reagin (RPR) test
positive result by ELISA should be con- E. Urine CMV culture
firmed by Western blot. Calcified lesions
in the chronic stage can be seen by the vv Correct Answer is: E
CT scan. Albendazole and praziquantel Cytomegalovirus is one of the herpesviri-
have both been shown to be cysticidal; dae family. Infected immunocompetent
however, a recent meta-analysis sug- adults are asymptomatic; on the other
gests that albendazole is superior to hand, congenital infection is the most
praziquantel in terms of its ability to prevalent infection-related cause of con-
control seizures in affected patients and genital neurologic impairment. Vertical
to completely eradicate cysts. transmission—during pregnancy
Treatment is not required in patients through the placenta, during delivery
without significant symptomatology. through cervical secretions or blood, or
Hydrocephalus is a common complica- postnatally through breast milk—occurs
tion, often necessitating placement of in 30% of cases; however, not all infected
an intraventricular shunt. fetuses are affected. Symptomatic
Infants at birth (10%) are typically small
zz Suggested Reading for gestational age with feeding prob-
55 Nash TE, et al. Treatment of neuro-cysti- lems, ocular problems (chorioretinitis,
cercosis: current status and future research retinal scars, optic atrophy, and central
needs. Neurology. 2006;67:1120–27. vision loss), sensorineural hearing loss,
384 Chapter 12 · Infections

organomegaly, thrombocytopenia, jaun- of her body, with apparent apnea


dice, microcephaly, seizures, and abnor- and cyanosis of the lips. The seizure
mal neurologic examinations. stopped spontaneously after few
Neuroimaging may reveal periventricu- minutes, but on exam, she is lethar-
lar cysts and calcifications, ventriculo- gic and has a rectal temperature of
megaly, vasculitis, hydranencephaly, 40.6 °C. She has a full and open ante-
and/or neuronal migration abnormali- rior fontanelle. She has an uncompli-
ties. Perinatal mortality for symptomatic cated pregnancy, labor, and delivery
newborns ranges from 2–30%, with sur- at full term.  
vivors having poor neurodevelopmental Which of the following is the most
prognoses. Infected infants who are likely cause of her condition?
asymptomatic at birth are still at risk for A. Coccidiomycosis systemic infection
neurodevelopmental issues, such as B. Congenital CMV
learning disabilities and attention deficit C. Congenital toxoplasmosis
hyperactivity disorder. In addition, D. Focal cortical dysplasia
5–15% of these infants will develop sen- E. Neonatal herpes simplex encepha-
sorineural hearing loss, which is typically litis
bilateral and progressive. If CMV infec-
tion is suspected in utero, an amniocen- vv Correct Answer is: E
tesis should be performed to confirm Vertical transmission (from infected
vertical transmission. Fetal ultrasound or mother to neonate at the time of birth)
MRI can help to predict the risk that the is common in neonatal HSV, though
fetus will be affected at birth. Urine cul- congenital infection can occur, and
ture or CMV-DNA testing by PCR can help postpartum virus exposure similar to
in confirming congenital infection within that found in older children can occur in
3 weeks of birth. Infected newborns the young infants depending on their
should be further evaluated with a com- exposure to the appropriate source.
12 plete blood count, platelet count, trans- Patient age, level of consciousness at
aminase level, and bilirubin levels (direct presentation, and duration of symptoms
and indirect) and should have both an before the initiation of therapy are the
ophthalmologic examination and audi- most three important outcome mea-
tory brainstem evoked responses. sures are. The last one of the three is the
Symptomatic newborns should be only one that can be controlled, so
treated with gancyclovir 6 mg/kg/dose treatment for HSV encephalitis with
twicw a day for 6 weeks, as this improves intravenous acyclovir must be initiated
hearing outcomes. Treatment of asymp- immediately for there to be optimal
tomatic newborns is not currently rec- chance at neurological recovery. The
ommended. therapy should not be delayed too long
for evaluation purposes, because diag-
zz Suggested Reading nostic information can still be obtained
55 Coll O, et al. Guidelines on CMV congenital 24–48 h after the initiation of treatment.
infection. J Perinat Med. 2009;37:433–45.
zz Suggested Reading
?? 8. You are seen a one-­week-­old baby 55 Current therapy in neurologic disease. In:
in the ED for her focal seizure which Johnson RT, et al., editors 6th ed. St. Louis:
consisted of shaking of the left side Mosby Press: 2002 123–24.
Infections
385 12
?? 9. The following scenario is part of a C. Doxycycline
two-question series, Discussion & Ref- D. Azithromycin
erences included at the end of series.   E. Fluconazole
A 16-year-old female came to
the office with 2 days history of right vv Correct Answer is: C
facial weakness and she is unable to The skin manifestation is the target sign
close her right eye. She has trouble of the Lyme disease which is caused by
sucking from a straw and also, she has the tick-transmitted spirochete, B. burg-
drooling from her right side of the dorferi. Clinical manifestations may
mouth. She denies paresthesias, ear include erythema migrans, lymphocytic
pain or hyperacusis. On skin examina- meningitis, cranial neuritis, and polyra-
tion you note the following:   diculitis. Diagnosis includes an ELISA on
The most likely infectious agent serum for anti-B. burgdorferi antibodies.
causing the above symptoms is: If serum is positive, then Western blot is
completed. Treatment options include
oral doxycycline, ampicillin, and cefu-
roxime axetil.

zz Suggested Reading
55 Halperin JJ. Lyme disease: a multisystem
infection that affects the nervous system.
Continuum. 2012; 18(6)

?? 11. You are evaluating a 7-year-old girl for


her new-onset headache and nausea.
She had a slight headache last night
and was given children Motrin. Early
morning, she woke up complaining of a
..      Fig. 12.2 Erythematous rash in the pattern of a “really bad headache,” as well as nausea.
“bull’s-eye” from Lyme disease. 7 en:User:Jongarrison. She felt warm by her mother. Her past
Hannah Garrison
medical history is unremarkable. All
of her vaccinations are up to date. On
A. Herpes Simplex Virus Type 1 examination, temperature is 39 °C. She
B. Borrelia burgdorferi is uncomfortable and prefers the lights
C. Rickettsia rickettsii to be dimmed. She refuses to flex her
D. Mycoplasma pneumonia neck and resists passive flexion of the
neck. She has a positive Kernig sign.
vv Correct Answer is: B She is alert and oriented in all three
spheres. Although in significant pain,
?? 10. The following scenario is part 2 of a she is cooperative in the rest of the neu-
2-question series, Discussion & Refer- rological exam which is normal. There
ences included at the end of series.   is no papilledema. She has a vesiculo-
The best treatment option for this papular rash on her palms and soles;
patient is: and small ulcerations on her lips. CSF
A. Acyclovir examination reveals normal opening
B. Forscarnet pressure; 2 RBC/mm3; 156 WBC/mm3
386 Chapter 12 · Infections

(80% lymphocytes, 20% PMN); protein fever. Anterior fontanelle is full. He is


51 mg/dL; and glucose 70 mg/dL.   lethargic, but does become more alert
What is your likely diagnosis? when his neck is moved and moves
A. acute disseminated encephalomy- all four extremities symmetrically. His
elitis reflexes are symmetric, 2+ and the
B. cerebral toxoplasmosis plantar responses are extensor. CSF
C. encephalitis due to herpes sim- examination reveals cell count 290
plex virus-1 WBC/mm3 with 98% polymorphonu-
D. meningitis due to enterovirus clear cells; 4 RBC/mm3; protein 45 mg/
E. neuro-cysticercosis dL; glucose 10 mg/dL. MRI with gado-
linium is obtained with imaging as
vv Correct Answer is: D shown.  
This is a clinical presentation of menin- What is your likely diagnosis?
gitis. She has fever, headache, and
nuchal rigidity. The CSF profile (lympho-
cytic pleocytosis, normal glucose) is typ-
ical of a viral process. The rash on the
palms, soles, and mouth are characteris-
tic for enterovirus. Enteroviruses cause
about 80% of all cases of viral meningi-
tis and are more likely to cause infection
in the autumn. Acute disseminated
encephalolmyelitis (ADEM) is unlikely to
be associated with a rash, and children
usually appear more ill with focal neuro-
logic deficits and seizures. Cerebral
toxoplasmosis and neuro-­cysticercosis
12 would be unlikely to present with men-
ingitis symptoms. HSV encephalitis may
have a similar CSF profile, but the clini-
cal picture would not be compatible as
there are no behavioral or cognitive
symptoms.
..      Fig. 12.3 MRI, bacterial meningitis; T1. Own work
zz Suggested Reading (Original text: selbst erstellt. MBq Disk 12:01, 26. Okt.
55 DeBiasi RL, Tyler K. Viral meningitis and 2010 (CEST). Wikipedia
encephalitis. Continuum Lifelong Learing
Neurol. 2006;12(2):58–94. A. acute disseminated encephalomy-
elitis
?? 12. A 5-week-old baby infant is brought B. bacterial meningitis
to the ED with lethargy, irritable and C. cerebral toxoplasmosis
refusal to eat. He is a full-term infant D. fungal meningitis
who was born via uncomplicated E. viral encephalitis
vaginal delivery and had been doing
very well until 1 day before admission. vv Correct Answer is: B
He has a low-grade fever this morn- He has fever and lethargy and has men-
ing. On examination, he has a 39.5 °C ingeal, signs on examination. CNS infec-
Infections
387 12
tion has to be number one on your list. D. Miller Fisher syndrome
The CSF showed, polymorphonuclear E. Neurosarcoidosis
pleocytosis and depressed glucose, so
bacterial meningitis is the most likely vv Correct Answer is: C
diagnosis. The MRI adds further infor- The correct answer is Lyme disease. The
mation by demonstrating gadolinium clinical scenario is consistent with Lyme
enhancement in the walls of the lateral disease because of the subacute to
ventricles, suggesting that this child chronic meningitis and bifacial weak-
also has ventriculitis as a complication ness, especially she has a recent camping
of meningitis. The MRI is not consistent trip with possible recent tick exposure.
with acute disseminated encephalomy- Not all patients recall having the typical
elitis (ADEM), which would demonstrate rash, as it is generally painless and may
T2 lesions in the white matter. Cerebral be in a region of the body the patient
toxoplasmosis would demonstrate ring- cannot easily see. Acute HIV infection,
enhancing lesions. Fungal meningitis is neuro-sarcoidosis, and carcinomatous
possible, but less likely than bacterial meningitis can cause a similar presenta-
meningitis and the CSF profile would tion, but is less likely given the travel his-
usually demonstrate a lymphocytic, or tory. Miller Fisher syndrome can cause
mixed pleocytosis. Viral encephalitis is bilateral facial weakness but the CSF pro-
also unlikely due to the depressed CSF file would not demonstrate a pleocytosis.
glucose and polymorphonuclear cells.
zz Suggested Reading
zz Suggested Reading 55 Halperin J. Lyme disease: a multisystem
55 Bartt R. Acute bacterial and viral meningi- infection that affects the nervous system.
tis. Continuum Lifelong Learning Neurol. Continuum Lifelong Learning Neurol.
2012;18(6):1255–89. 2012;18(6):1338–50.

?? 13. A 16-year-old healthy female came ?? 14. The mother of a 3-year-old girl
back from a summer camping trip, is brought her for her ongoing poorly
complaining of headache and facial controlled seizures despite the use of
weakness which started upon awak- multiple anti-epileptic medications.
ening this morning. She had been sick Her mother brought a recent CT per-
with generalized headaches for the formed 1 week ago, but the rest of her
past 2 weeks. She does not recall hav- medical records is not available. On
ing a rash. She is afebrile, but has a examination, her head circumference
mild nuchal rigidity. She has bilateral is below 5th percentile, she has no
lower motor neuron facial weakness visual fixation, and spasticity bilater-
otherwise the rest of her neurological ally in the lower extremities more
exam is unremarkable. CSF shows 110 than the upper. Reflexes are brisk with
white blood cells (90% lymphocytes), sustained clonus at both ankles. No
protein of 95 mg/dL, and glucose of family history of similar condition,
60 mg/dL.   and she has two older siblings who
What is your diagnosis? are both healthy. She is taken leveti-
A. Acute HIV infection racetam and phenobarbital.  
B. Carcinomatous meningitis What abnormal results would you be
C. Lyme disease looking for as a neonate?
388 Chapter 12 · Infections

or aqueductal stenosis are at risk for


macrocephaly due to non-communicat-
ing hydrocephalus. Tuberous sclerosis
can lead to calcified sub-­ependymal
nodules and rarely regions of pachygyria
or polymicrogyria, though not lissen-
cephaly. Tuberous sclerosis (TS) does not
have such extreme microcephaly. CMV is
the most common congenital viral infec-
tion, occurring in 1–2% of live births.
Fortunately, less than 10% of infants are
symptomatic. Severe migrational
defects are a consequence of infection
early (in the first trimester). Clinical man-
ifestations include intrauterine growth
retardation, jaundice, petechiae and
purpura (the so-called blueberry muffin
spots), severe microcephaly, cerebral
..      Fig. 12.4 CT, congenital cytomegalovirus (CMV)
calcifications (peri-ventricularly), glau-
infection. American Academy of Neurology Institute,
produced by permission coma and chorioretinitis. Due to the
ubiquitous nature of this virus it is
essential that the virus be isolated from
A. CT scan revealing a Dandy-Walker one of many different locations in the
malformation newborn (most commonly urine and
B. genetic testing revealing a muta- saliva) within the first 2–3 weeks of life
tion in the TSC1 gene region to confirm that infection was acquired
C. karyotype showing three copies congenitally. Antiviral therapy may
12 of the 21st chromosome modify the degree of retinal involve-
D. MRI showing aqueductal stenosis ment but does not modify the CNS fea-
E. urine from the infant positive for tures of the infection. Symptomatic
cytomegalovirus treatment of associated seizures and
spasticity is needed for most patients.
vv Correct Answer is: E The outcome can be predicted by the
This girl has uncontrolled epilepsy, spas- severity of the associated migrational
ticity, and prominent microcephaly. Her defect. Children with lissencephaly
head CT shows very large ventricles with (reported in severe cases of CMV which
hyperdense signal in the immediate can impair neuronal migration) have
periventricular region, and a very simple profound delays in development,
appearing cerebral cortex, without any whereas those with only mild micro-
apparent sulcation consistent with lis- cephaly and intracranial calcifications
sencephaly. This is a presentation of con- may have a milder degree of develop-
genital cytomegalovirus (CMV) mental delay.
infection. Children with Down syndrome
(trisomy 21) do not develop a prominent ?? 15. You are seeing a previously healthy
microcephaly and are usually hypotonic 16-year-old female with 1 day of left
on examination, and intracranial calcifi- facial droop 14 days after camping
cations would not be expected. Children trip up north. She was seen by her
with either Dandy-Walker malformation PCP, who prescribed oral prednisone.
Infections
389 12
3 days later she has right face weak- this case especially in the absence of
ness. On examination, she has moder- other systemic clues and the clear risk
ate to severe bilateral facial weakness factors for Lyme disease. Acute HIV
but the rest of her general and neuro- seroconversion has rarely been associ-
logic examination are unremarkable, ated with unilateral or bilateral facial
and she has no rash.   palsy. Because this is an early manifes-
What test are you going to order to tation, both serum HIV antibody and
find out the cause of her facial weak- antigen testing should be obtained as
ness? the patient may not have seroconver-
A. Serum angiotensin converting ted. While unlikely to be causative in
enzyme level this case due to the demographics, HIV
B. Serum HIV antibody and antigen testing should still be considered based
testing on multiple guidelines recommending
C. Serum Lyme antibody routine screening in patients aged
D. CSF cell count and protein 13–65 years of age. Another relatively
E. CSF varicella zoster virus PCR common cause of simultaneous bilat-
eral facial palsy is the Miller-Fisher vari-
vv Correct Answer is: C ant of Guillain-­Barré syndrome. In these
A patient has bilateral facial nerve pal- cases CSF cell count and total protein
sies and coming back from an area of showing cytoalbuminologic dissocia-
endemic Lyme disease, most likely the tion is supportive. With the lack of are-
cause of her presentation either idio- flexia or ataxia, however, this is an
pathic (Bell’s) palsy or Lyme disease. unlikely diagnosis. There are cases
Lyme disease is the more likely diagno- reported with preserved reflexes, how-
sis because the simultaneous facial ever, and a spinal fluid exam might be
nerves involvement. Bilateral facial considered if Lyme testing is unreveal-
nerves involvement can happen in Bell’s ing. Varicella zoster reactivation in the
palsy but less common. As this neuro- geniculate ganglion may lead to vesicu-
logical complication typically occurs lar eruption in the ear canal and ipsilat-
within weeks of initial infection of the eral facial palsy (Ramsay Hunt
borrelia burgdorferi organism, serum syndrome). It may rarely occur in the
Lyme antibodies are usually positive (if absence of shingles. This is a clinical
not, they should be rechecked within diagnosis and it may occur without CSF
1–2 weeks and the patient treated spread of virus, as detected in CSF VZV
empirically for Lyme disease). In cases PCR. Given the young age of the
of facial palsy without meningitis, CSF patient, absence of shingles, and the
testing is not required to make the diag- more common occurrence of Lyme dis-
nosis, though should be performed if ease in this setting, VZV is unlikely.
there are any signs of meningitis
(nuchal rigidity or persistent headache.) zz Suggested Reading
Cranial neuropathy is the most common 55 Halperin JJ, Shapiro ED, Logigian E,
neurological manifestation of sarcoid- Belman AL, Dotevall L, Wormser GP,
osis, and the facial nerve is the most fre- et al. Practice parameter: treatment of
quently involved. Multiple cranial nerve nervous system Lyme disease (an evi-
involvement is not rare. The serum ACE dence-based review): report of the Quality
level may be elevated in neuro-sarcoid- Standards Subcommittee of the American
osis but is not sensitive or specific. Academy of Neurology. Neurology.
Sarcoidosis is a less likely diagnosis in 2007;69(1):91–102. 125.
390 Chapter 12 · Infections

?? 16. A 16-year-old boy is complaining of a Peripheral nerve disease is character-


rapid onset lower extremity weakness ized with symmetrical, distal paresthe-
and facial paralysis, more on the right sias with sensory loss on examination.
side than the left. Two days after a The presenting features of acute dis-
He was on a camping trip and 2 days seminated encephalomyelitis include a
later he started to have congestion, rapid onset encephalopathy with multi-
cough and muscle aches. Over the focal neurologic signs and deficits.
course of the next 48 h he began hav- Peripheral nerve deficient are not char-
ing trouble walking. He has no rash acteristic of this condition.
and but he has remarkable weakness Poliovirus is the most devastating
in right leg extensors and flexors, enterovirus infection and causes severe
accompanied by flaccid muscle paralysis. Coxackie virus and echovirus
tone, right facial drooping with sym- are enterovirus, but more likely to cause
metrical forehead wrinkling. He has aseptic meningitis. Initial symptoms are
normal mental status. EMG indicates fever, sore throat, and malaise and gas-
severe denervation in affected muscle trointestinal illness.
regions.  
What is your diagnosis? zz Suggested Reading
A. Lyme disease 55 Solomon T, Ooi MH, Beasley DW,
B. West Nile virus infection Mallewa M. West Nile encephalitis. BMJ:
C. Acute demyelinating encephalo- Br Med J. 2003;326(7394):865.
myelitis
D. Enterovirus infection ?? 17. A 14-year-old female symptoms were
not improved after been on amoxicil-
vv Correct answer is: B lin for 2 days for her otitis media and
Since 1999, West Nile virus has become low-grade fever and she developed
the most important cause of arboviral episodes of severe vertigo.  
12 ­meningitis, encephalitis and acute flac- What is the cause of her vertigo?
cid paralysis in the U.S. The virus is A. Hypervolemia secondary to illness
transmitted by an infected mosquito. A B. Meningitis
flu-like illness develops 3–14 days after C. Amoxicillin toxicity
the mosquito bite. Sensory symptoms D. Acute serous labyrinthitis
are minimal and encephalopathy does
not occur. MRI of the brain can be nor- vv Correct answer is: D
mal or show white matter demyelin- Viral or bacterial ear infection can be
ation. In this case, the facial weakness is followed by rapid onset of vertigo.
lower motor neuron and there is sparing Labyrinth mechanisms can be irritated
of the forehead muscle. In West Nile and stimulate vestibular actions can
infections, MRI of the spine and electro occur secondary to direct extension of
diagnostic studies are consistent with the infection to the inner ear, as well as
lower motor neuron and ventral horn local effects of bacterial toxins.
pathology. Hypervolemia, orthostatic hypoten-
Lyme meningitis is presented ini- sion and syncope can occur with ill-
tially by intermittent attacks of severe nesses due to reduced intake and fever
headache and mild meningismus. Some but the symptom is mainly light-head-
will develop signs of mild encephalitis edness.
and cranial neuritis, most commonly Meningitis is a major cause of vestib-
lower motor neuron facial palsy. ular and auditory dysfunction in chil-
Infections
391 12
dren, but meningismus must be present. Before the MRI era, periodic lateral-
Amoxicillin toxicity is not associated izing epileptiform discharges (PLEDS)
with vertigo, but aminoglycosides, anti- was the most the predictive study.
convulsants, streptomyci n and minocy- Although background slowing is
cline toxicity can cause vestibular seen with encephalitis, slowing itself is
symptoms. not specific to herpes encephalitis. Focal
slowing is seen commonly in areas of
zz Suggested Reading cortical strokes or large tumors. Non-
55 Kishore A. Assessment and management contrast CT is most useful in acute
of pediatric vestibular disorders. ECAB trauma or suspected arterial hemor-
Dizziness and Vertigo across Age Groups, rhage, as it can rapidly be utilized to
2014. identify intracranial blood.

?? 18. A 16-year-old female was admitted to zz Suggested Reading


the hospital with a two-day history of 55 Songmen S, Panta OB, Maharjan S, Paudel
fever, headache, nausea and vomit- S, Ansari MA, Ghimire RK. A
ing. This morning her mother found Retrospective Study of Magnetic
her confused and lethargic. On your Resonance Imaging Findings in Acute
examination, you found that she had Encephalitis Syndrome. J Inst Med.
a moderate left hemiparesis. Her CSF 2015;38(2).
showed: 250 white blood cells/mm3
(79% leukocytes), 350 red blood cells/ ?? 19. You are evaluating a 17-year-old male
mm3, protein of 95 mg/dl and normal in the ER for his headache, nausea,
glucose levels.   vomiting, photophobia, neck pain,
What test findings will provide suffi- and fever of 39C. On examination, he
cient presumptive evidence to make has right VI CN palsy and right upper
a diagnosis? and lower facial droop. Funduscopic
A. Periodic lateralizing epileptiform examination revealed no papilledema
discharges and spontaneous venous pulsations
B. Right cerebral slowing on EEG are present. The boy cries out in pain
C. Non-contrast CT and lifts his knees on passive neck
D. Right temporal increased signal flexion during his exam. He has no
intensity on T2 MRI other focal neurological signs.  
What is your next step in manage-
vv Correct answer is: D ment?
Temporal regions are significantly have A. Head CT followed by administra-
been affected more than other brain tion of vancomycin and ceftriaxone
regions in herpes encephalitis. T2 IV followed by a lumbar puncture
weighted imaging identifies the earliest B. Lumbar puncture followed by head
stage of herpes encephalitis that most CT followed by administration of
commonly involves the cortex and white vancomycin and ceftriaxone IV
matter in the temporal and inferior fron- C. Lumbar puncture followed by
tal lobes. On subsequent imaging, inde- immediate administration of van-
pendent foci enlarge then coalesce and comycin and ceftriaxone IV
in severe cases result in encephalomala- D. Immediate administration of van-
cia of the temporal lobe. The confirma- comycin and ceftriaxone IV with
tory test is spinal fluid PCR for the no further action because lumbar
herpes virus. puncture is contraindicated
392 Chapter 12 · Infections

vv Correct answer is: C 55 Kaplan SL. Bacterial meningitis in


Head CT is recommended prior to lum- children older than one month: treatment
bar puncture based on the recommen- and prognosis. In: Up-to-date, Edwards
dations of the Infectious Diseases MS, Nordli DR, Armsby C, editors
Society of America if the patient has Up-to-date. Waltham. Accessed on 20 Dec
papilledema, focal neurological deficits, 2015.
recent neurosurgery or trauma, hydro-
cephalus, or a CSF shunt, or immuno- ?? 20. You are seeing a 12-year-old male in
compromised. In this case, the patient the ED for his fever, headache, nausea,
does have focal neurological deficits on vomiting, lethargy, and a seizure just
exam. However, cranial nerve VI and VII after he arrived to the ED. You have
palsies are specifically excluded and done a lumbar puncture and you will
thus, this patient does not need a head send CSF for analysis.  
CT prior to lumbar puncture. When the What set of orders will you send for?
lumbar puncture can be done immedi- A. Opening pressure, protein, glu-
ately, this should be performed fol- cose cell count and differential,
lowed by empiric therapy which in this PCR for Herpes Simplex 1 and 2,
case includes vancomycin and a third- PCR for enterovirus, PCR for vari-
generation cephalosporin. cella zoster virus, and bacterial
If the patient needs a CT scan before culture
the LP based on the criteria described B. Opening pressure, protein, glu-
above, treatment should not wait until cose, cell count and differential,
head CT is done. Poorer outcomes are and bacterial culture
associated with delays in therapy. C. Opening pressure, protein, glu-
Typically, the head CT would precede cose, cell count and differential,
the lumbar puncture if it is even neces- bacterial culture, and herpes sim-
sary. Again, empiric treatment should plex 1 and 2 viral culture.
12 not be delayed. D. Opening pressure, protein, glu-
Empiric treatment with antibiotics in cose, cell count and differential,
this age group vancomycin and a third-­ and west
generation cephalosporin like ceftriax- E. Nile virus PCR
one, should be started prior to gram
stain and culture results. The major vv Correct answer is: A
pathogens are Neisseria meningitidis If you are ordering CSF analysis as part
and Streptococcus pneumoniae in of the work up for suspected viral
patients between the ages of 24 months encephalitis, the minimum you have to
and 50 years old. order includes, opening pressure, pro-
tein, glucose, cell count and differential,
zz Suggested Reading and PCR for herpes virus 1 and 2, vari-
55 Tunkel AR, Hartman BJ, Kaplan SL, et al. cella zoster, and enterovirus. Other viral
Practice guidelines for the management of testing can be performed as well based
bacterial meningitis. Clin Infect Dis. on clinical presentation or geographic
2004;39:1267. area, but HSV1, HSV2, VZV, and entero-
55 Kaplan SL. Bacterial meningitis in children virus are the highest yield.
older than one month: clinical features and Viral culture is not recommended.
diagnosis. In: Up-to-date, Edwards MS, PCR for HSV 1 and 2 should be per-
Nordli DR, Armsby C, editors. Up-to-date. formed, not culture.
Waltham. Accessed on 20 Dec 2015.
Infections
393 12
HSV, VZV, and enterovirus PCR are 55 Solomon T, Hart IJ, Beeching NJ. Viral
recommended in the initial examination encephalitis: a clinician’s guide. Pract
of cerebrospinal fluid. While in some Neural. 2007;7(5):288–305.
cases, it is appropriate to order testing 55 Petersen LR. Clinical manifestations and
for West Nile virus, this is not the best diagnosis of West Nile virus infection. In:
answer as HSV and enterovirus testing Up-to-date, Hirsch MS, Mitty J, editors.
are not included in this order set. And Up-to-date. Waltham. Accessed on 20
actually, in those with West Nile Virus Dec 2015.
which is causing central nervous system
disease, viral PCR of the cerebrospinal ?? 21. A 6-month-old baby boy is brought
fluid is only 55% sensitive. A diagnosis to the ED with severe hypotonia
can be made based on lgM antibodies and respiratory distress. His mother
specific to west Nile virus in the cerebro- started him on different solid foods
spinal fluid. Thus, even if west Nile virus in the last 2 weeks. 3 days ago, he
is suspected, PCR may be negative and had constipation and “seemed weak”.
thus insufficient for diagnosis. One day ago, the infant had a poor
suck, droopy eyelids and was unable
zz Suggested Reading to feed normally. On neurologi-
55 Di Pentima C. Viral meningitis: clinical cal exam, he has severe hypotonia,
features and diagnosis in children. In: absent tendon reflexes, limited facial
Up-to-date, Edwards MS, Nardi DR, movement, and ptosis and dilated
Armsby C, editors. Up-to-date. Waltham. pupils.  
Accessed on 20 Dec 2015. What test will you order to make the
55 Tunkel AR, Glaser CA, Bloch KC, Sejvarl J, diagnosis?
Marra CM, Roos KL, Hartman BJ, Kaplan A. CT of the brain
SL, Scheid WM, Whitley RJ. The manage- B. MRI of the cervical spinal cord
ment of encephalitis: clinical practice C. Edrophonium test
guidelines by the infectious diseases D. Stool culture
society of America. Clin Infect Dis.
2008;47:303–27. vv Correct answer is: D
55 Hardarson HS. Acute viral encephalitis in Clostridium botulinum infects the
children and adolescents: clinical manifes- infants from contaminated formula,
tations and diagnosis. In: Up-to-date, ingestion of contaminated honey or
Kaplan SL, Fleisher GR, Nordli DR, inhaled from dust from nearby con-
Armsby C, editors. Up-to-date. Waltham. struction. The solid foods in this
Accessed on 20 Dec 2015. vignette may have included honey.
55 Hardarson HS. Acute viral encephalitis in The growth of clostridium botuli-
children and adolescents: pathogenesis num can be tested by Stool culture. The
and etiology. In: Up-to-date, Edwards MS, typical prodromal symptoms are consti-
Armsby C, editors. Up-to-date. Waltham. pation and poor feeding. The patho-
Accessed on 20 Dec 2015. physiology is a cholinergic blockade of
55 Keen R, Michael BO, Menson E, Mehta B, skeletal muscle and autonomic nerves
Easton A, Hemingway C, Klapper PE, caused by the exotoxin. Progressive
Vincent A, Lim M, Carrol E, Solomon facial, pharyngeal weakness, ptosis,
T. Management of suspected viral encepha- dilated pupils, absent reflexes and con-
litis in children e Association of British stipation differentiate infantile botu-
Neurologists and British Paediatric Allergy lism from other inflammatory
Immunology and Infection Group National demyelinating and neuromuscular
Guidelines. J Infect. 2012;64:449–77. junction disorders.
394 Chapter 12 · Infections

Brain MRI is of no value in the diag- vv Correct answer is: B


nosis, because it is a neuromuscular B is most consistent with viral meningi-
junction disease. tis. Neutrophils may be present, typi-
The spinal cord is not affected in cally early in the infection, but typically
infantile botulism. do not account for more than 50% of
Familial infantile myasthenia is a the white blood cells present. White
rare form of myasthenia. Administration blood cells range from 100 to 500 cells
of edrophonium will rapidly reverse the per mm3. Protein may be elevated but is
symptoms in myasthenia related condi- typically under 100 mg/dl. Glucose is
tions. The condition occurs: in the usually normal although may be
absence of myasthenia in the mother, decreased in patients with mumps or
with respiratory depression at birth, herpes simplex.
episodic weakness and apnea during In normal adults, up to 5 cells per
the first 2 years of life. Familial infantile mm3 leukocyte is acceptable with no
myasthenia typically improves with age. neutrophils. Normal leukocyte count
Since the condition responds to anti- may be higher in preterm infants; in this
cholinesterase medication, early diag- case normal cerebrospinal fluid has less
nosis is important. Infantile myasthenia than 20 cells per mm3. In term infants,
syndromes does not present with up to 2 months of age, cerebrospinal
dilated pupils, absent reflexes or consti- fluid may contain up to 15 cells per
pation. mm3, and still be considered normal
Protein should be between 15 and
zz Suggested Reading 50 mg/dl.
55 Mitchell WG, Tseng-­Ong L. Catastrophic In acute bacterial meningitis, WBC
presentation of infant botulism may cells are more than 100 to 500 cells per
obscure or delay diagnosis. Pediatrics. mm3, mostly neutrophils, and protein is
2005;116(3):e436–8. 100–500 mg/dl. Glucose is often low,
12 under 40 mg/dl; if the patient is hyper-
?? 22. A 13-year-old male came to the ED glycemic, glucose may be corrected to
with headache, nuchal rigidity, and under 60% of the total serum value.
fever. On examination, he has no focal In tuberculous meningitis, WBC may
neurological signs. A lumbar puncture range from 10 to 500 cells per mm3, glu-
was performed, and showed that the cose is often decreased, and protein may
patient has viral meningitis.   range from 100 to 500 mg/dl. Neutrophils
Which CSF results from below are most may be present in greater numbers ear-
consistent with viral meningitis? lier in the infection. In 3% of children and
A. WBC < 5 cells/mm3, no neutro- 6% of adults with tuberculous meningi-
phils, protein of 20 mg/dl, glucose tis, cell count is normal. 16% of children,
of 60 mg/dl and 6% of adults may have normal pro-
B. WBC 100 cells/mm3, 30% neutro- tein and 77% of children and 72% of
phils, protein of 50 mg/dl, glucose adults have glucose of under 45 mg/dL in
of 60 mg/dl the cerebrospinal fluid.
C. WBC 200 cells/mm3, 70% neutro-
phils, protein of 110 mg/dl, glu- zz Suggested Reading
cose of 30 mg/dl 55 Pomar V, Domingo P. Acute viral meningi-
D. WBC 200 cells/mm3, 40% neutro- tis. In: Garda-Monc JC, editors. CNS
phils, protein of 209 mg/dl, glu- infections: a clinical approach. London:
cose of 30 mg/dl Springer;2014:45–55.
Infections
395 12
55 Garcfa-Monc JC, Rodriguez-Sainz A. CNS antibodies in serum determined by
tuberculosis and other mycobacterial enzyme-linked immunoelectrotransfer
infections. In Garda-Monc, JC, editors. blot assay (EITB), (3) intracranial lesion
CNS Infections: a clinical approach. resolution with appropriate treatment
London: Springer;2014:139–61. (albendazole or praziquantel), and (4)
55 Gomez-Beldarrain M, Garcfa-Monc the presence of one enhancing lesion
MDJC. Lumbar puncture and CSF analysis that resolves spontaneously.
and interpretation. In Garcfa-Monc, JC, Minor criteria include (1) lesions that
editors. CNS infections: a clinical are compatible with the diagnosis, (2)
approach. London: Springer;2014:1–16. clinical findings that are suggestive of
this diagnosis, (3) anticysticercal anti-
?? 23. A 16-year-old boy from Cuba came bodies or antigens within cerebrospinal
to the ED after having a focal right fluid determined by ELISA, and (4) cysti-
arm shaking seizure for 5 min and cercosis documented outside the cen-
resolved spontaneously. Head CT was tral nervous system.
ordered and shows a round 10 mm Epidemiologic criteria include (1)
hypodense cystic lesion in the left known household member who has
frontal lobe. You ordered MRI which Taenia solium infection, (2) frequent
showed a small, hyperintense spot travel to areas where the disease is
within the cystic lesion. The radiology endemic, and (3) history of living in an
attending reads as a scolex.   area where the disease is endemic.
Which of the following is most likely The presence of one absolute crite-
true? rion- in this case the presence of a sco-
A. The patient has definitive neuro- lex on neuro imaging- allows for a
cysticercosis definitive diagnosis of neuro-cysticerco-
B. A diagnosis of definitive neuro- sis. A definitive diagnosis can be made
cysticercosis can be made in in the presence of 1 epidemiologic crite-
this case in the absence of brain rion, 1 minor criterion, and 2 major cri-
biopsy. teria. If a patient has 1 major criterion
C. Again, the patient has definitive and 2 minor criteria, a diagnosis of
neuro-cysticercosis based on probable neuro-cysticercosis can be
imaging findings and does not made. If a patient has 1 major criterion,
need serological testing 1 minor criterion, and 1 epidemiologic
D. The patient does not need cere- criterion OR if a patient has 3 minor cri-
brospinal fluid testing in order to teria and 1 epidemiologic criterion, a
confirm this diagnosis. diagnosis of probable neuro-cysticerco-
sis can be made.
vv Correct answer is: A A diagnosis of definitive neuro-­
Absolute criteria include (1) demonstra- cysticercosis can be made in this case in
tion of parasite histologically on brain the absence of brain biopsy.
biopsy, (2) the presence of a cystic Again, the patient has definitive
lesion with scolex on neuro imaging, neuro-­cysticercosis based on imaging
and (3) the presence of sub-retinal para- findings and does not need serological
sites, seen on funduscopic exam. testing.
Major criteria include (1) Lesions The patient does not need cerebro-
that are highly suggestive of the diag- spinal fluid testing in order to confirm
nosis on imaging, (2) anticysticercal this diagnosis.
396 Chapter 12 · Infections

zz Suggested Reading What is your next best step in man-


55 White AC. Epidemiology, transmission, agement?
and prevention of cysticercosis. In: A. Gentamicin should be discontin-
Up-to-date, Weller PF, Baron EL, editors. ued immediately
Up-to-date. Waltham. Accessed on 20 Dec B. Repeat lumbar puncture should
2015. be performed in a week; if cells,
55 White AC. Clinical manifestations and protein, and glucose are normal,
diagnosis of cysticercosis. In: Up-to-date, all antibiotics can be discontinued.
Weller PF, Baron EL, editors. Up-to-date. C. Repeat lumbar puncture should
Waltham. Accessed on 20 Dec 2015. be performed in 24–48 h; if cells
55 White AC. Treatment of cysticercosis. In: protein, and glucose are normal,
Up-to-date, Weller PF, Baron EL, editors. gentamicin and ampicillin can be
Up-to-date. Waltham. Accessed on 20 Dec discontinued, although penicil-
2015. lin G should be started for a total
55 Dhawan VK. Pediatric neuro-cysticer- treatment period of 14–21 days
cosis. In: Kumar A, Barton LL, Kossoff D. Repeat lumbar puncture should be
EH, editors. Medscape Reference. performed in 24–48 h; if cells pro-
7 http://emedicine.medscape.com/ tein, and glucose are normal, ampi-
article/999053-overview (7 http:// cillin can be discontinued, although
emedicine.medscape.com/article/999053- treatment with gentamicin should
overview ). Updated March 3 2015, continue for 14 to 21 days.
Accessed 21 Dec 2015.
55 Rodriguez S, Wilkins P, Dorny vv Correct answer is: C
P. Immunological and molecular diagnosis Premature rupture of membranes, cho-
of cysticercosis. Path Glob Health. rioamnionitis, and GBS bacteriuria dur-
2012;106:286–98. ing pregnancy, prior delivery of an
55 Baird RA, Wiebe S, ZuntJR, HalperinJJ, infected infant, the presence of fever
12 Gronseth G, Roos KL. Evidence-based during labor and delivery, or delivery at
guidelines: Treatment of parenchymal under 37 weeks are the risks factors for
neuro-cysticercosis: Report of the guide- early onset group B streptococcal dis-
line development subcommittee of the ease (GBS). Gentamicin and ampicillin is
American Academy of Neurology. recommended as an empiric treatment
Neurology. 2013;80:1424–29. for infants with suspected meningitis at
55 Garcia Prof HH, Nash TE, Del Brutto 2 days of age. Penicillin G is the treat-
OH. Lancet Neurol. 2014;13(1):1202–15. ment of choice if GBS meningitis has
55 Lucato LT, Guedes MS, Sato JR, Bacheschi been confirmed. However, it is recom-
LA, Machado LR, Leite CC. The role of mended that CSF to be obtained
conventional MR imaging sequences in 24–48 h after the onset of therapy. If
the evaluation of neuro-cysticercosis: sterilization is confirmed, then gentami-
impact on characterization of the scolex cin and ampicillin can be discontinued;
and lesion burden. Am J Neuroradiol. penicillin G should be started and anti-
2007;36(12):1501–04. biotics should continue for 14–21 days.

?? 24. A 2-day old baby was treated empiri- zz Suggested Reading


cally with gentamicin and ampicillin 55 Tunkel AR, Hartman BJ, Kaplan SL, et al.
for suspected meningitis. His CSF Practice guidelines for the management of
results showed an infection with bacterial meningitis. Clin Infect Dis.
group B streptococcus.   2004;39:1267.
Infections
397 12
55 Edwards MS, Baker CJ. Bacterial meningi- may need to be prolonged. IV ganciclovir
tis in the neonate: clinical features and is recommended as a substitution if acy-
diagnosis. In: Up-to-date, Kaplan SL, clovir becomes unavailable, given every
Weisman LE, Armsby C, editors. Up-to-­ 12 h at a dose of 5 mg/kg.
date. Waltham. Accessed on 20 Dec 2015. You have to monitor the patient for
55 Edwards MS, Baker CJ. Bacterial meningi- thrombocytopenia, leukopenia, or anemia.
tis in the neonate: treatment and outcome. If ganciclovir is not available, foscar-
In: Up-to-date, Kaplan SL, Weisman LE, net can be used. Although foscarnet can
Armsby C, editors. Up-to-date. Waltham. also be used in place of acyclovir during
Accessed on 20 Dec 2015. drug shortages, ganciclovir is recom-
mended as a first choice. Foscarnet can
?? 25. A 14-year-old boy has been admit- cause renal insufficiency and electro-
ted to hospital with headache and lytes should be monitored.
altered mental status after he had a The patient needs to be treated. He
generalized seizure. A lumbar punc- cannot wait for acyclovir to become
ture is performed, opening pressure available.
is normal, and there is a white cell
count of 50 mainly neutrophil, protein zz Suggested Reading
is slightly elevated at 55 and glucose 55 Hardarson HS. Acute viral encephalitis in
is normal. He was treated with acyclo- children and adolescents: Treatment and
vir empirically for suspected herpes prevention. In: Up-to-date, Edwards MS,
simplex encephalitis. Finally, the Fleisher GR, NordIi DR, Armsby C,
diagnosis is confirmed by HSV1 and editors. Up-To- Date. Waltham. Accessed
2 PCR. After a few days of acyclovir on 20 Dec 2015.
treatment, the drug becomes unavail- 55 Zachary KC. Acyclovir: an overview. In:
able nationwide and becomes expen- Up-to-date, Hirsch MS. Mitty J, editors.
sive to the patients and the hospital. Up-to-date. Waltham. Accessed on 20 Dec
The patient has confirmed herpes 2015.
simplex encephalitis.  
What should the worried you do? ?? 26. A 22-month-old boy was seen with
A. You should do nothing- if there is his family in your office for a second
no acyclovir, he cannot treat her- opinion. 2 weeks ago, he was admit-
pes simplex encephalitis ted to the PICU with a 101.5 F tem-
B. You should start ganciclovir immedi- perature, irritability, and lethargy and
ately at a dose of 5 mg/kg every 12 h. neck pain because of the suspicion
C. You should start foscarnet IV of meningitis. You reviewed the lab
immediately at a dose of 60 results provided to his family upon
D. Mg/kg every 12 h discharge. The CSF showed 25 leuko-
E. You should wait for acyclovir to cytes/mm3, 95% lymphocytes, pro-
become available- surely the per- tein concentration of 65 mg/dl and
son responsible will end up in jail glucose concentration of 70. He was
for securities fraud soon treated with antibiotics and his blood
and spinal fluid cultures were nega-
vv Correct answer is: B tive at discharge 5 days later.  
A dose of 10 mg/kg has to be used to What is the likely diagnosis?
treat HSV encephalitis in a child of at A. Migraine headache
least 12 years of age for 3 weeks. B. Asceptic meningitis
Although if the CSF is still positive for C. Self-limiting mild encephalitis
infection on repeat testing, the treatment D. Lyme disease
398 Chapter 12 · Infections

vv Correct answer is: B What is your next step in manage-


Aseptic meningitis and bacterial menin- ment?
gitis can have the same clinical presen- A. Obtain blood and cerebrospinal
tation. But, bacterial meningitis can fluid samples, send for cultures,
cause more aggressive and severe and treat based on the results
symptoms and sequela. The relatively B. Obtain blood and cerebrospinal
low spinal fluid protein level, normal fluid samples but immediately
glucose and the moderately elevated start ampicillin and meropenem
cell count with lymphocytic predomi- while waiting for results
nance are highly suggestive of viral or C. Obtain blood and cerebrospinal
aseptic meningitis. fluid samples but immediately
In older children and adults, start dexamethasone, vancomycin
migraine headaches can present with and, cefotaxime while waiting for
stiff neck and focal neurological distur- results
bances. Migraine headaches are uncom- D. Obtain blood and cerebrospinal
mon in children under 2, and typically fluid samples but immediately
do not present with a fever. start vancomycin; dexamethasone
By definition, persons with encepha- can be given an hour after the
litis present with an altered mental sta- antibiotics are started.
tus and most progress to seizures with a
prolonged recovery phase. The spinal vv Correct answer is: C
fluid analysis may be similar to those If you suspected bacterial meningitis in
described in this case. children over 1 year of age, and in adults
Although Lyme disease can cause up to 50 years of age, vancomycin and a
meningitis, it is rarely a presenting man- third-­generation cephalosporin should
ifestation and is rarely occurs in children be started for empiric coverage.
this age. Potential pathogens include
12 Streptococcus pneumonia, Neisseria
zz Suggested Reading meningitidis, Streptococcus agalactiae,
55 Cohn KA, Thompson AO, Shah SS, Hines Haemophilus influenza, and Escherichia
EM,Lyons TW, Welsh EJ, Nigrovic LE. coli. In patients between 1 and 3 months
Validation of a clinical prediction rule to of age, Streptococcus agalactiae is most
distinguish Lyme meningitis from aseptic common followed by Streptococcus
meningitis. Pediatrics. 2012;129(1):e46–53. pneumonia and then Neisseria menin-
gitidis. In those at least 3 months of age
?? 27. You are examining a 16-month-old but under 3 years of age, Streptococcus
boy comes with vomiting, irritability, pneumonia is most common, followed
and appetite loss. His immunizations by Neisseria meningitidis, Streptococcus
are not up to date because his mother agalactiae and then gram-negative
has a concern over the side effects. bacilli.
On exam, he is febrile with tempera- Patients at least 3 years of age but
ture of 39C and appears lethargic. under 10 years of age tend to have
When you attempted to check his Streptococcus pneumonia and Neisseria
pupils, he cried, and closed his eyes. meningitidis. Overall, the most common
He resisted attempts to bend his head pathogens in fully vaccinated children
forward. There are no focal deficits on are Streptococcus pneumonia and
neurological exam.   Neisseria meningitidis although in this
Infections
399 12
case, the child is not fully vaccinated. gentamicin, to cover for Streptococcus
Regardless, vancomycin and a third- agalactiae, Escherichia coli, Listeria
generation cephalosporin are used monocytogenes, and Klebsiella. In those
empirically to cover the most common over age 50, ampicillin is added empiri-
pathogens, accounting for antibiotic cally to vancomycin and a third-genera-
resistance. Dexamethasone is recom- tion cephalosporin to cover Listeria
mended in those with meningitis due to monocytogenes. Meropenem may be
Haemophilus influenza type b to miti- used empirically with vancomycin in
gate hearing loss. Use in other cases is suspected cases of meningitis occurring
controversial; clinicians can consider post neurosurgery. In this case, cover-
giving this drug in those older than age should include Pseudomonas aeru-
6 weeks who have meningitis due to ginosa and other aerobic gram-negative
Pneumococcal bacteria. This is the best bacilli, Staphylococcus aureus, and
answer only because there are no obvi- Staphylococcus epidermidis. Similarly,
ous contraindications to dexametha- vancomycin and meropenem may be
sone use, and also, it is the only answer used in patients with suspected menin-
that includes immediate use of the cor- gitis occurring after penetrating head
rect empiric therapy. If antibiotic ther- trauma. The same regimen may be used
apy is given immediately, empirically in a patient with a CSF shunt
dexamethasone is not recommended if in place, when bacterial meningitis is
over an hour has passed. suspected. In this case, coverage needs
Anti-microbial therapy should be to include Staphylococcus epidermidis,
started at once in an individual with sus- Staphylococcus aureus, Pseudomonas
pected bacterial meningitis. It is not aeruginosa, and Propionibacterium
prudent to wait for cultures, gram stain, acnes.
cell count, protein level, or glucose If dexamethasone is given, it should
within the spinal fluid prior to treating. be given within 20 min of the first dose
Ideally, cerebrospinal fluid is obtained of antibiotics, if it is not given just prior
prior to the start of antibiotics; however, to the first dose. Dexamethasone may
one should not wait for any results. actually interfere with the ability of
Treatment should start at once. If cere- anti-microbials to cross the blood brain
brospinal fluid cannot be obtained barrier. Dexamethasone is not recom-
immediately, treatment should start mended an hour after antibiotics have
regardless and cerebrospinal fluid been given.
obtained later if possible.
Meropenem coverage includes zz Suggested Reading
Listeria monocytogenes, 55 Kaplan SL. Bacterial meningitis in
Staphylococcus aureus, Escherichia coli, children: dexamethasone and other
Streptococcus pneumoniae, measures to prevent neurological compli-
Haemophilus influenza, and cations. In: up-to-date, Edwards MS, Nordi
Pseudomonas aeruginosa.4 Ampicillin DR, Armsby C, editors. Up-To –Date.
coverage includes, among other patho- Waltham. Accessed on 20 Dec 2015.
gens, Listeria monocytogenes and 55 Kaplan SL. Bacterial meningitis in children
Streptococcus agalactiae. In a child older than one month: clinical features and
under 1 month old, empiric treatment diagnosis. In: up-to-date, Edwards MS,
includes ampicillin and cefotaxime or Nordli DR, Armsby C, editors. Up-to-date.
ampicillin and an aminoglycoside like Waltham. Accessed on 20 Dec 2015.
400 Chapter 12 · Infections

55 Kaplan SL. Bacterial meningitis in children Patients who do NOT have CSF gram
older than one month: treatment and stain positivity and who did NOT pres-
prognosis. In: up-to-date, Edwards MS, ent with seizure are at lower risk of bac-
Nordli DR, Armsby c, editors. Update. terial meningitis.
Waltham. Accessed on 20 Dec 2015. Patients who do NOT have an abso-
55 Tunkel AR, Hartman BJ, Kaplan SL, et al. lute neutrophil count of at least 10,000
Practice guidelines for the management of cells per microliter in the peripheral
bacterial meningitis. Clin Infect Dis. blood are at lower risk of having bacte-
2004;39:1267. rial meningitis. Patients who do NOT
have an absolute neutrophil count in
?? 28. A 7-year-old female is seen in the ED the cerebrospinal fluid of at least 1000
with suspected bacterial meningi- cells per microliter are at lower risk of
tis. After you done with her history having bacterial meningitis.
and physical and reviewed her lab
tests, you determined that she has a zz Suggested Reading
very low risk of bacterial meningitis, 55 Nigrovic LE, Kuppermann N, Macias CG,
despite having 15 white blood cells et al. Clinical prediction rule for identify-
per microliter in her cerebrospinal ing children with cerebrospinal fluid
fluid.   pleocytosis at very low risk of bacterial
Which of the following is most likely meningitis. JAMA. 2007;297:52.
correct?
A. She has a positive CSF gram stain ?? 29. Which of the following is most true
and presented with a seizure about pediatric neuro-­cysticercosis?
B. She has a peripheral blood abso- A. Most pediatric cases involve mul-
lute neutrophil count of 15,000 tiple cysts.
cells per microliter B. Enzyme linked immunoelectro-
C. Her CSF protein is 40 mg/dl transfer blot assay (EITB) can be
12 D. She has an absolute neutrophil used to confirm a diagnosis in
count of 1500 cells per microliter a child with a single intracranial
in the cerebrospinal fluid lesion given specificity of 100%
and sensitivity of 90%
vv Correct answer is: C C. The most common initial clinical
The Bacterial Meningitis Score can be manifestation in children is a seizure
used to determine whether a child has a D. This condition is most often
lower risk of bacterial meningitis in the diagnosed in children in the first
setting of cerebrospinal fluid with at 24 months of life
least 10 cells per microliter. Those who
do NOT have (1) a positive cerebrospinal vv Correct answer is: C
fluid gram stain (2) history of seizure (3) Seizures are the most common presenta-
peripheral blood absolute neutrophil tions in children up to 65–80% of cases.
count (ANC) of at least 10,000 cells per Majority have a single lesion at diagnosis
microliter (4) cerebrospinal fluid abso- as well. Focal sensory or motor deficits,
lute neutrophil count (ANC) of at least headache, vomiting, or mental status
1000 cells per microliter, or (5) a cere- changes are less common presentations.
brospinal fluid protein of at least 80 mg/ ¾ of children will have single intra-
dl are at low risk of having bacterial cranial lesion at diagnosis. About 50%
meningitis. The patient in this case has a may have a scolex visible as a small
protein of under 80 mg/dL. focus of hyperdensity within the lesion.
Infections
401 12
ELISA (enzyme linked immunoelec- of cysticercosis. Path Glob Health.
trotransfer) can be used in order to 2012;106:286–98.
determine whether antibodies to Taenia 55 White AC. Clinical manifestations and
solium are present in the CSF or blood. diagnosis of cysticercosis. In: Up-to-date,
The sensitivity and specificity of this test Weller PF, Baron EL, editors. Up-to-date.
may vary, however. In those with at least Waltham. Accessed on 20 Dec 2015.
2 known viable central nervous system
cysts, sensitivity is as high as 98%. ?? 30. A 16-year-old female was admitted to
However, in those with 1 cyst, sensitivity the ICU after she has a sudden onset
may be between 50 and 60%. In chil- of altered mental status and seizures.
dren with 3 or more lesions, sensitivity Her mother reported that the girl had
is 90% while specificity is up to 100%. a temperature of 102 F 2 days ago
In children below 24 months of age, accompanied with, vomiting, diar-
the diagnosis cannot be made because rhea, headache, myalgia, hypotension
the life cycle and incubation period of and desquamating rash. She started
the parasite. Taenia solium is ingested to be agitated and confused in the
by pigs; following this, oncospheres, last 24 ours. She experienced a gen-
which have passed through the intesti- eralized seizure 2 h before. She has a
nal wall during digestion, travel to the negative toxicology screens. A lumbar
muscles where cysticerci form. Humans, puncture showed normal opening
upon ingestion of the cysticerci through pressure and protein level with no
improper cooking and/ or handling of cells. Brain MRI was normal. She has
pork, are not immediately infected with a normal past medical history and
cysticercosis. Adult worms grow in the developmental history is normal.  
small intestine until eggs (via the fecal What is the likely etiology of this
oral route) are ingested. After this, onco- condition?
spheres travel through the body, reach- A. Staphylococcus aureus
ing the liver, central nervous system, B. Measles virus
musculature, and other tissues. Thus, C. Rhabdovirus
infection is not instantaneous and tends D. Ecstacy (MOMA) overdose
not to manifest in babies or children
under 24 months of age. vv Correct answer is: A
The presentation is most likely of toxic
zz Suggested Reading shock syndrome, which can be a fatal ill-
55 White AC. Epidemiology, transmission, ness that is caused by infection or colo-
and prevention of cysticercosis. In: nization of staphylococcus. This can
up-to-date, Weller PF, Baron EL, editors. result from tampon use in menstruating
Up-to-date. Waltham. Accessed on 20 Dec females, but has also been reported to
2015. occur in children requiring occlusive
55 Dhawan VK. Pediatric neuro-cysticercosis. surgical dressings. The onset is sudden,
In: Kumar A. Barton LL, Kossoff EH, rapid and presents as described above.
editors. Medscape Reference. 7 http:// Signs of lethargy, obtundation, enceph-
emedicine.medscape.Com/article/999053- alopathy and generalized seizures can
overview (7 http://emedicine.medscape. have happened rapidly. Complications
com/article/999053-overview). Updated can include cardiac arrhythmias, pulmo-
March 3 2015, Accessed 21 Dec 2015. nary edema and renal failure. Clinical
55 Rodriguez S, Wilkins P, Dorny course and cultures of the infected areas
P. Immunological and molecular diagnosis are diagnostic.
402 Chapter 12 · Infections

Measles presents with a morbilform D. There are early and late neuro-
rash and the clinical presentation does logic manifestations of the disease
not progress to the described severity. E. Approximately 10% of asymptom-
Rabies occurs from a bite or scratch atic people in endemic areas may
from an infected animal and presents be seropositive for this organism
with fever, myalgia and paresthesias.
Neurological symptoms include hyper- vv Correct answer is: C
excitability, autonomic dysfunction. The The clinical presentation is of Lyme dis-
paralytic manifestations include flaccid ease. Lyme disease has a wide variety of
paralysis in the bitten limb that progres- neurologic presentations; but bacterial
sively ascends to involve the remainder meningitis with neutrophilic predomi-
of the body. nance is unlikely to occur. Subdural
Ecstacy overdose presents in a simi- empyema is not seen in Lyme disease. It
lar fashion to other stimulant class is caused by Borrelia burgdorferi. It is
drugs. High does have been reported to more commonly acquired in the
cause hyperpyrexia, rhabdomyolysis, Northeastern United States and is trans-
intravascular ­coagulopathy, hepatic mitted by a tick bite, more specifically
necrosis, cardiac arrhythmia and stroke. the lxodes tick. Neurologic complications
These symptoms can occur in reported can occur in both early phase and a late
doses associated with “recreational” phase of the disease. A rash called ery-
doses. thema migrans, which is characteristic
and has a “bull’s eye” appearance occurs
zz Suggested Reading in the early phase. Other non- neurologic
55 Chan BC, Maurice P. Staphylococcal toxic manifestations include carditis, arthral-
shock syndrome. New Eng J Med. gias, lymphadenopathy, and fever.
2013;369(9):852. Neurologic manifestations in the
early phase include the following: -
12 ?? 31. A 16-year-old male presents with Aseptic meningitis: these patients have
headache and severe pain radiating a mononuclear pleocytosis and mild ele-
down his low back to the right leg vation of protein levels in the CSF, as
4 weeks after he came back from a well as intrathecal production of anti-
camping trip. He complains also from bodies against Borreliaburgdorferi. -
paresthesias in his left hand and right Encephalitis. - Cranial neuropathy:
foot. He reports having a rash that commonly cranial nerve VII, and this can
started 2 weeks ago, which seems to be uni- or bilateral. - Peripheral nervous
be migrating and looks round and system: including mononeuritis multi-
erythematous with a paler center.   plex, peripheral neuropathy, polyradicu-
Which of the following is incorrect lopathy, or even a Guillain-Barre type
regarding this condition? presentation (which is not common).
A. Ceftriaxone is used for the treat- Neurologic presentations in the late
ment of this condition phase are encephalopathy, encephalo-
B. Doxycycline can be used for the myelitis, and peripheral neuropathy.
treatment in the setting of normal CNS Lyme disease diagnosis is based on
CSF clinical and epidemiological suspicion,
C. Bacterial meningitis with predom- supported by the presence of positive
inance of neutrophils is a common serology, CSF abnormalities, and intra-
manifestation thecal production of antibodies.
Infections
403 12
Serology is positive in 10% of people tions may occur later in life in normal
living in endemic areas, and this should newborns. The treatment of choice in
be taken into account. Intravenous cef- newborn includes pyrimethamine, sulfa-
triaxone, up to 2 g daily for 2–4 weeks is diazine, and leukovorin for 1 year.
the treatment of choice. For late presen- Prednisone may be added in the setting
tations, a longer course may be of elevated cerebrospinal fluid protein
required. Oral antibiotic therapy with levels or chorioretinitis.
doxycycline can be used in patients
without CSF abnormalities. zz Suggested Reading
55 Fenichel GM. Clinical pediatric neurology.
zz Suggested Reading 5th ed. Philadelphia: Elsevier Saunders.
55 Comprehensive Review in Clinical p. 122.
Neurology: A Multiple-Choice Question 55 Jones J, et al. Congenital toxoplasmosis.
Book for the Wards and Boards ©2011 Am Fam Physician. 2003;67(10):2131–8.
Wolters Kluwer Health Lippincott
Williams & Wilkins. All rights reserved. ?? 33. A 3-day old full term with no prenatal
care. You suspected meningitis and
?? 32. The classic triad of symptoms in con- ordered cerebrospinal fluid analysis.
genital toxoplasmosis includes: Pending the CSF results.  
A. Seizures, psychomotor retarda- what should be started empirically?
tion, hydrocephalus A. Vancomycin n and ceftriaxone
B. Seizures, rash, intracranial calcifi- should be started
cations B. Ampicillin and vancomycin should
C. Hydrocephalus, chorioretinitis, be started
intracranial calcifications C. Ampicillin and gentamicin should
D. Chorioretinitis, rash, spastic para- be started
paresis D. Vancomycin should be started
E. Psychomotor retardation, hypoto-
nia, chorioretinitis vv Correct answer is: C
Ampicillin and gentamicin can be
vv Correct answer is: C started empirically; the most common
Congenital toxoplasmosis is secondary pathogens in infant under 3 days of age
to placental transmission of the proto- will be Streptococcus agalactiae,
zoan toxoplasmosis gondii, most often Escherichia coli, Listeria monocytogenes
in the third trimester. Hydrocephalus, and other enteric bacilli.
chorioretinitis, and intracranial calcifica- On the other hand, the most com-
tions are the classic triad of symptoms mon pathogens after 4 days of life are,
in congenital toxoplasmosis. Other Serratia marcescens, Pseudomonas
manifestations may include fever, rash, aeruginosa, and Citrobacter koseri.
hepatosplenomegaly, jaundice, throm- In infants under the age of 1-month,
bocytopenia, seizures, and psychomotor common pathogens include
retardation. Approximately 25% of Streptococcus agalactiae, Escherichia
affected children will have systemic coli, Listeria monocytogenes, and
symptoms at birth. Premature infants Klebsiella. Ampicillin and cefotaxime
are typically more severely affected may also be used in patients under 1
than babies born at full term. Develop month of age, but this is not listed as an
learning disabilities or ocular manifesta- answer choice.
404 Chapter 12 · Infections

In children over 1 month of age, and neurological evaluation including


in adults up to the age of 50, Vancomycin EMG/NCS indicates a motor neuron
and a third-generation cephalosporin disease.  
such as ceftriaxone can be started What is the most likely cause of his
empirically because the coverage for symptoms?
Listeria monocytogenes is required. A. Poliomyelitis
Streptococcus pneumoniae, B. Subacute sclerosing panencepha-
Streptococcus agalactiae, Haemophilus litis (SSPE)
influenza, Neisseria meningitidis, and C. Progressive multifocal leukoen-
Escherichia coli are common between cephalopathy (PML)
1 month and 23 months of age. D. Subacute HIV encephalomyelitis
Vancomycin and third generation ceph- E. Kuru
alosporin are usually sufficient.
Streptococcus pneumoniae and vv Correct answer is: A
Neisseria meningitidis are common Poliomyelitis is the only choice that can
between 2 and 50 years of age. cause a pure motor neuron disease.
Ampicillin and vancomycin are not Only1% of cases, the virus enters the
recommended for empiric coverage in central nervous system, preferentially
neonates with suspected bacterial infecting and destroying motor neu-
meningitis. rons, leading to muscle weakness and
Vancomycin n is not recommended acute flaccid paralysis but the majority
for empiric treatment of suspected bac- of cases, infection cause no symptoms
terial meningitis in infants under at all. Different types of paralysis may
1 month of age. occur, depending on the nerves
involved. Spinal polio is the most com-
zz Suggested Reading mon form, characterized by asymmetric
55 Edwards MS, Baker CJ. Bacterial meningi- paralysis that most often involves the
12 tis in the neonate: clinical features and legs. Between 25% and 50% of patients
diagnosis is. In: Up-to-date, Kaplan SL, who have recovered from paralytic
Weisman LE, Armsby C, editors. Up-to- polio in childhood can develop addi-
date. Waltham. Accessed on 20 Dec 2015. tional symptoms decades after recover-
55 Kaplan SL. Bacterial meningitis in children ing from the acute infection, notably
older than one month: Clinical features new muscle weakness and extreme
and diagnosis. In: Up-to-date, Edwards fatigue. This condition is known as post-
MS, Nordli DR, Armsby C, editors. polio syndrome (PPS). The symptoms
Up-to-date. Waltham. Accessed on 20 Dec are thought to involve a failure of the
2015. over-sized motor units created during
55 Tunkel AR, Hartman BJ,Kaplan SL, et al. the recovery phase of the paralytic dis-
Practice guidelines for the management of ease. PPS is a slow, progressive disease,
bacterial meningitis. Clin Infect Dis. and there is no specific treatment for it.
2004;39:1267. Post-polio syndrome is not an infec-
tious process, and patients experienc-
?? 34. A 16-year-old boy has fever and ing the syndrome do not shed
progressive right leg weakness. His poliovirus.
Infections
405 12
zz Suggested Reading zz Suggested Reading
55 Bradley WG, Daroff RB, Fenichel GM, 55 Fenichel GM. Clinical pediatric neurology.
Jankovic J. Neurology in clinical practice. 5th ed. Philadelphia: Elsevier Saunders;
5th ed. Philadelphia: Elsevier; 2008. 2005. p. 124–5.

?? 35. Which of the following statements ?? 36. An 8-year-old boy became unsteady
regarding congenital AIDS is correct? and unable to walk all of a sudden,
A. 80% of infants born to mothers this afternoon. Despite of being
with AIDS will show evidence of scared, he is very cooperative and
infection at birth well-spoken. On examination, he has
B. Macrocephaly is a common com- mild bilateral lateral gaze nystagmus,
plication of the disorder past-pointing on finger-nose-­finger
C. Basal ganglia calcifications are test, and normal reflexes. He has a nor-
more common in children than mal strength, but he cannot balance
adults with AIDS while standing, and he has a lurching
D. HIV encephalopathy results in gait. He had chicken pox 14 days ago,
hypotonia but he is otherwise healthy. Develop-
E. None of the above mental milestones are normal.  
What is your most likely diagnosis?
vv Correct answer is: C A. Acute postinfectious cerebellitis
Infants of HIV mothers can be infected via B. Friedreich ataxia (FRDA)
placental transmission, perinatal expo- C. Medulloblastoma
sure, or breastfeeding. The incidence of D. Miller Fisher syndrome
HIV transmission to newborns is approxi- E. Multiple sclerosis
mately 30% with the use of retroviral
therapies. Children disease progression is vv Correct answer is: A
the reflection of the disease severity in Acute post-infectious cerebellitis can
the mother. 20% will have a rapidly pro- present as acute onset of cerebellar
gressive course with death in infancy if symptoms (maximal within a few hours)
the HIV is positive. Hepatosplenomegaly, following any viral infection by
bone marrow failure, lymphocytic inter- 10–14 days. Usually, the sensorium
stitial pneumonia, chronic diarrhea, fail- should remain clear. MRI of the brain
ure to thrive, microcephaly, and basal may show increased T2 intensities in the
ganglia calcifications, are more common cerebellum. Improvement should begin
symptoms in children than adult. HIV within a few days of onset, and gait
encephalopathy may be evident 2 should normalize within 1–5 months.
months to 5 years post-exposure, with
developmental regression, microcephaly, zz Suggested Reading
dementia, spasticity, ataxia, pseudobul- 55 Fenichel GM. Clinical pediatric neurology:
bar palsy, movement disorders, myoclo- a signs and symptoms approach.
nus, and seizures. Death typically occurs Philadelphia: WB Saunders; 1997.
within a few months of onset. Treatment Montenegro MA, Santos SL, Li LM,
of HIV in children includes zidovudine, Cendes F. Neuroimaging of acute cerebel-
didanosine, and nevirapine. litis. J Neuroimaging 2002;12:72–4.
406 Chapter 12 · Infections

?? 37. An 8-year-old female is admitted with was associated with seizure freedom in
continued shaking of her left arm. Her 63% of patients treated with surgery.
brain CT scan is shown.   Despite the aggressive nature of
Rasmussen’s encephalitis, hemispherec-
tomy does offer a chance of seizure free-
dom in this population.

zz Suggested Reading
55 Hoffman CE, Ochi A, Snead OC 3rd,
Widjaja E, Hawkins C, Tisdal M, Rutka
JT. Rasumussen’s encephalitis: advances in
management and patient outcomes. Childs
Nerv Syst. 2016;32(4):629–40.

?? 38. You are evaluating an 18-year-old


female for her diplopia and right facial
and retro-orbital pain. Her symptoms
were preceded by a severe ear infec-
tion for the last few weeks. Examina-
tion reveals a right sixth (abducens)
nerve palsy and diminished sensations
on the right face. Motor strength is nor-
mal in all extremities. A CT scan reveals
..      Fig. 12.5 Brain CT scan without contrast enhance- opacity of the right mastoid air cells
ment of a patient, female, 8 years old, with Rasmusen’s
and petrous part of temporal bone.  
encephalitis. (Own work. Mikhail Kalinin. Wikipedia)
What is your diagnosis?
A. Bell palsy
12 Which of the following is true B. cavernous sinus thrombosis
regarding treatment? C. Gradenigo syndrome
A. Immunotherapy represents the D. Millard-Gubler syndrome
best chance for seizure remission E. Ramsay-Hunt syndrome
B. Anti-epileptic drug polytherapy
represents the best chance for sei- vv Correct Answer is: C
zure remission The clinical presentation is suggestive
C. Hemispherotomy represents the of Gradenigo syndrome. It is character-
highest chance for seizure remis- ized by the triad of suppurative otitis
sion media, pain in the distribution of the tri-
D. Vagus nerve stimulator represents geminal nerve, and abducens nerve
the highest chance for seizure palsy. Cranial nerve (CN) VII can also be
remission involved. The cranial nerves involve-
E. No treatment offers any sizable ment occurs when otitis media infection
chance for seizure remission. spreads to involve the petrous apex.
Diabetics or immunosuppressed
vv Correct Answer is: C patients are at higher risk. Awareness of
This patient has Rasmussen’s encephali- the symptom complex is important for
tis. A recent case series of 13 patients early diagnosis and intervention with
some patients do have transient benefit high dose antibiotics or surgery as indi-
from immunotherapy. However, surgery cated to avoid the morbidity and poten-
Infections
407 12
tially life-threatening complications vv Correct Answer is: E
associated with this condition. This is a presentation of a vestibular neu-
Ramsay-Hunt syndrome is second- ritis which results from viral inflammation
ary to herpes zoster infection which can of the vestibular nerve. The main features
involve cranial nerves V (ophthalmic of unilateral vestibular neuritis include
branch), VI and VII palsy with vesicular severe and prolonged vertigo, nausea,
eruption in the auditory canal. vomiting, and nystagmus with fast com-
Bell palsy involves cranial nerve VII ponent away from the affected side, nor-
causing ipsilateral lower motor type mal hearing and otherwise normal
weakness of the facial muscles. neurologic exam. Incidence is highest in
Cavernous sinus thrombosis can pres- middle age and there is frequently a
ent with chemosis, proptosis and painful recent history of viral illness. Patients with
ophthalmoplegia and multiple cranial bilateral vestibulopathy typically present
nerve deficits. Depending upon the extent with imbalance and oscillopsia and do
of the involvement this can include CN III, not have vertigo or spontaneous nystag-
IV, V (divisions I and II), and VI and post- mus. Acute labyrinthitis can also result
ganglionic sympathetic axons to the eye. from similar viral etiology but the involve-
Millard-Gubler syndrome is caused ment of the labyrinth causes hearing loss
by a pontine lesion affecting cranial in addition to the vestibular symptoms.
nerves VI and VII and corticospinal tract Benign paroxysmal positional vertigo
with ipsilateral facial and abducens is characterized by recurrent episodes of
palsy and contralateral hemiplegia. vertigo brought on by a sudden change
in head position which is thought to be
zz Suggested Reading caused by abnormal stimulation of the
55 Motamed M, Kalan A. Gradenigo syn- ampulla of one of the semicircular
drome. Postgrad Med J. 2000;7(899):559–60. canals, by gravity induced movement of
55 Bradley’s neurology in clinical practice. free floating otoliths during changes in
Daroff R, Fenichel G, Jankovic J, Mazziotta head position. Typically, there are no
J, editors. 6th ed. Philadelphia: Elsevier, hearing abnormalities and other neuro-
2012; 1754. logic deficits. Ménière disease is charac-
terized by attacks of vertigo lasting from
?? 39. A 16-year-old boy complains of sud- minutes to hours associated with nausea
den onset vertigo and nausea which and vomiting, sensation of fullness or
is continuous and severe for 3 days. pressure in the ears, fluctuating hearing
He had an upper respiratory symptom loss and tinnitus. Symptoms can be uni-
1 week ago which resolved. He has no lateral or bilateral.
history of hearing loss, tinnitus or aural
fullness. On examination, he has hori- zz Suggested Reading
zontal nystagmus with fast component 55 Ropper A, Samuels M. Adams and Victor’s
towards the right as well as decreased principles of neurology. 8th ed.
caloric response in the left ear.   New York:Mcgraw Hill, 2005; 261.
What is your diagnoses? 55 Baloh R. Acute unilateral and bilateral
A. acute labyrinthitis peripheral vestibular loss. Continuum
B. benign paroxysmal positional ver- Lifelong Learning Neurol 2006;12(4):46–64.
tigo 55 Zee DS. Pathophysiology of vestibular
C. bilateral vestibular neuritis symptoms and signs: the clinical examina-
D. Ménière disease tion. Continuum Lifelong Learning Neurol
E. unilateral vestibular neuritis 2006;12(4):13–32.
409 13

Mental Retardation/
Cognitive Disorders

© Springer International Publishing AG, part of Springer Nature 2018


Y. M. Awaad, Absolute Pediatric Neurology, https://doi.org/10.1007/978-3-319-78801-2_13
410 Chapter 13 · Mental Retardation/Cognitive Disorders

?? 1. A 5-year-old boy with a history of dysmorphic features with prominent


speech delay. He still does not know ears. He has a sister with learning
his colors or shapes. He has no signifi- disabilities and a maternal uncle
cant past medical history. He is not with Parkinsonism.  
any treatment. During your exam, he What is the most appropriate test to
is friendly and loquacious. You heard obtain at this time?
a systolic murmur that radiates to the A. MRI of the Head
neck. He has several dysmorphic fea- B. FMR1 trinucleotide repeat
tures, including a long philtrum and C. Serum ceruloplasmin
prominent lips with an open mouth.   D. EEG
What is the most likely diagnosis? E. High resolution chromosomes
A. Fragile X syndrome
B. William’s syndrome vv Correct Answer is: B
C. Velocardiofacial defect The boy has mild mental retardation,
D. Mitochondrial disorder behavioral difficulties, a family history
E. Down’s syndrome of learning difficulties in a sister and
movement disorder in his maternal
vv Correct Answer is: B uncle. In boys, Fragile X is the most com-
William’s syndrome is associated with mon inherited cause of mental retarda-
a deletion at 7q11.23. It is associated tion. Usually, they have large protruding
with mental delay, unusual elfin facies, ears, macro-orchidism, moderate to
and cardiovascular anomalies, includ- severe mental retardation, hyperexten-
ing supra-­valvular aortic stenosis, sible finger joints, hypotonia and autism
mild prenatal growth deficiency and or autistic like behavior. Also it might
occasionally, hypercalcemia. They are be associated with ADHD and social
very friendly and have an outgoing and anxiety. It is secondary to an unstable
loquacious personality. However, they triplet repeat amplification of the CGG
may also have mood lability. Although within the FMR1 gene on chromosome
they have good verbal skills, they may Xq27.3. More than 200 triplet repeats
13 have delayed adaptive skills. are needed to have the full phenotypic
Other cardiac anomalies include expression. A triplet repeats above
peripheral pulmonic stenosis and mitral 55–200 is considered a pre-mutation
valve prolapse (13%). state. The maternal uncle is the pre-
mutation carrier manifesting with
zz Suggested Reading the Fragile X tremor/ataxia syndrome
55 Jones KL. Smith recognizable patterns of (FXTAS). It is a late onset, neurodegen-
human malformation. 6th ed. erative condition that is characterized
Philadelphia: Elsevier; 2006. p. 122–4. by an intention tremor, ­cerebellar
55 Menon Deepa. Williams syndrome. In: ataxia and executive dysfunction. This
Accardo PJ, editor. Developmental is usually seen in pre-mutation carriers.
disabilities in infancy and childhood. vol. Patients may have weakness, fatigue,
2. 3rd ed. Baltimore: Brooke’s Publishing; anxiety and irritability.
2008. p. 353–61.
zz Suggested Reading
?? 2. A 9-year-old boy has been evalu- 55 Bodensteiner J, Ellis C, Schaeffer
ated for deterioration of his behavior GB. Mental retardation. In: Maria B,
and school performance. His psy- editor. Current management in child
chological testing reveals IQ of 65. neurology. 4th ed. Helton: BC Decker;
On examination, he has significant 2009. 346–57, 285–95.
Mental Retardation/Cognitive Disorders
411 13
?? 3. A 7-year-old girl was referred by her Her mother also reported that for
school teacher because school dif- the last 3 months she has had diffi-
ficulties. Her teacher mentioned in culty following directions and seems
her referral that the child does not irritable because the family does
seem to understand spoken instruc- not understand her. She had one
tions. She witnessed a violent crime seizure 2 weeks ago. Her sleep EEG
in her neighborhood 3 months ago. is shown.  

..      Fig. 13.1 EEG findings of Landau-Kleffner syndrome. American Academy of Neurology Institute, produced by
permission

What is your diagnosis? terized by verbal auditory agnosia (loss


A. Rett syndrome of understanding of spoken words). The
B. Landau-Kleffner syndrome patient has normal psychomotor devel-
C. Angelman syndrome opment initially followed by loss of
D. Lennox-Gastaut syndrome language milestones. Seventy to eighty
E. Benign Rolandic Epilepsy percent of patients have epilepsy.
Behavioral abnormalities characterized
vv Correct Answer is: B by hyperactivity, inattention, aggres-
Landau-Kleffner syndrome is one of the siveness and social withdrawal are very
epileptic encephalopathies. It is charac- common. EEG findings are continuous
412 Chapter 13 · Mental Retardation/Cognitive Disorders

generalized slow spike and wave in increases with age have to be moni-
sleep or bilateral independent temporal tored and a substantial number of chil-
spike and wave discharge. dren have high myopia. Astigmatism
and hyperopia are also seen but are less
zz Suggested Reading common.
55 Nickel K, Wirrell E. Electrical status Cervical spine X - Rays are routinely
epilepticus in sleep. Semin Pediatr Neurol. recommended in children with Down’s
15:50–60. syndrome because of atlantoaxial sub-
luxation. The boy is presenting with
?? 4. A 12-year-old girl with Down’s syn- cognitive decline rather than the typical
drome came to the office with her gait changes, loss of bladder or bowel
family. Her family is complaining of control or quadriparesis that is seen in
her self-help skills regression and she patients with cord disorders.
became very irritable lately. She is
attending special education classes zz Suggested Reading
but her school grades have been 55 Roger P, Roizen N, Capone GT. Down
dropping. She has dysmorphic fea- syndrome. In: Accardo PJ, editor.
tures consisting of a flat facial profile Developmental disabilities in infancy and
with almond shaped palpebral fis- childhood. vol 1. 3rd ed. Baltimore:
sures.   Brookes Publishing; 2008. p. 285–303.
Which of the following is the most
appropriate next step in her evalu- ?? 5. A 12-month-old female brought to
ation? the office by her family because they
A. CT of the brain have a concern about her develop-
B. Thyroid function tests ment. She has a normal development
C. Cervical spine films in her first sixth month of life. Few
D. Electroencephalogram months ago she stopped playing
E. Spinal fluid for neurotransmitters with her toys, and wrings her hands
together constantly. On exam, her
13 vv Correct Answer is: B head circumference at 10th per-
Down’s syndrome is one of the most centile, which had been at the 25th
common chromosomal disorder that percentile at her 6-month checkup.
results from complete trisomy of the She is hypotonic and unable to walk
chromosome 21 due to non-disjunc- independently. She does not babble,
tions during gamete formation. There point, nor gesture.  
are a complex medical issues that may What test is most likely to identify
vary from birth to adulthood in children the correct diagnosis?
with Down’s syndrome. A. Brain magnetic resonance imag-
There are certain considerations ing
have to be addressed in children with B. Electrocencephalogram
Down’s syndrome between the ages of 1 C. MecP2 sequencing
to puberty. There may be delayed erup- D. Muscle biopsy
tion of the first tooth. Hypo-thyrodism E. Serum lactate
is on the rise in children with Down’s
syndrome and the signs and symptoms vv Correct Answer is: C
maybe subtle. Rett syndrome is one of a neurodevel-
Autoimmune thyroiditis may pres- opmental disorders characterized by
ent at any age. Refractive errors which loss of communication skills. Usually,
Mental Retardation/Cognitive Disorders
413 13
children have a normal prenatal and has a strong correlation with the clini-
perinatal period. It affects mainly cal symptoms, and cognitive deficits.
females because it is X-linked domi- Although blood levels have dramatically
nant disorder. It is a genetic disorder dropped in the past decades (from 15
on chromosome Xq28. Genetic testing ug/dl to 2.7 in 1991 to 1994), there are
with sequencing abnormalities of the still a significant number of children
MeCP2 gene can be used to confirm the who may have blood levels equal or
diagnosis. The mecp2 gene is a DNA greater than 10ug/dl. The consensus
binding protein that acts as a global guidelines indicated targeted screening
transcriptional repressor. Nonsense of all children with identifiable risk fac-
mutations patients present with a tors including children living in housing
sever phenotypes, while patients with before 1950, children of ethnic or racial
missense mutations present with mild minority groups who may be exposed
phenotypes. Clinical manifestations to lead containing food remedies, chil-
include acquired microcephaly, seizures, dren who have emigrated (or adopted)
respiratory abnormalities, and psycho- from countries where lead poisoning is
motor retardation with loss of manual prevalent, children with iron deficiency,
praxis and stereotypic hand wringing children exposed to contaminated dust
movements. or soil, children with developmental
delay whose oral behavior places them
zz Suggested Reading at risk for lead exposure, children whose
55 Menkes JH. Heredodegenerative disease. parents are expose to lead (vocationally,
In: Menkes JH, Sarnat H, Maria B, editors. avocationally, and during home renova-
Child neurology. 7th ed. Philadelphia: tion), children of low income families
Lippincott Williams and Wilkins; 2000. and victims of abuse and neglect.
p. 210–11.
55 Bibat G, Naidu S. Rett syndrome. In: zz Suggested Reading
Accardo PJ, ed. Developmental disabilities 55 Shevell M, Ashwal S. Practice parameter:
in infancy and childhood. vol. 2. 3rd ed. evaluation of the child with global
Baltimore: Paul Brookes Publishing; 2008. developmental delay: report of the Quality
p. 545–50. Standards subcommittee of the American
Academy of Neurology and the practice
?? 6. A 4-year-old foster boy presented for committee of the Child Neurology Society.
a developmental delay evaluation. Neurology. 2003;60:367–80.
He has been with his foster parents 55 Davioli CT and Chisolm J.J. Childhood
for three, but they do not know his lead poisoning. In: Capute AJ, Accardo PJ,
previous medical history. He eats toys, editors. Developmental disabilities in
paper, wood and clay.    infancy and childhood. vol 1. 2nd ed.
What test are you going to order? Baltimore: Brooke’s Publishing.1996.
A. ­Electroencdephalogram p. 232–3.
B. Lead level
C. Liver function tests ?? 7. A 4–year-old female with develop-
D. Cranial MRI mental delay has seen by you. Her
E. Thyroid function tests family describes her as a very slow
feeder, who often choked while tak-
vv Correct Answer is: B ing her bottle. She started to stand at
Lead is the most common environ- 18 months of age and started walk-
mental toxin. High serum lead levels ing at 24 months. At 4-years of age,
414 Chapter 13 · Mental Retardation/Cognitive Disorders

she started to combine two words vv Correct Answer is: D


together. She has moderate hypoto- Prenatal exposure to alcohol is well
nia and has a voracious appetite.   recognized as a teratogen to the devel-
What test would most likely confirm oping fetus. The condition is known as
the diagnosis? Fetal alcohol spectrum (FAS) disorder.
A. High resolution chromosome The clinical features include dysmorphic
studies facial characteristics (microcephaly,
B. Muscle biopsy facial anomalies: short palpebral fis-
C. Fish testing of 15q11.2-13 sures, long smooth nasal philtrum
D. MECP2 sequencing and thin upper vermillion border). The
E. Brain magnetic resonance imaging behavioral profile in children with FAS is
consistent with Attention deficit hyper-
vv Correct Answer is: C activity disorder.
Clinical manifestations of Prader Willi
syndrome are hypotonia (usually dur- zz Suggested Reading
ing infancy), failure to thrive, obesity 55 Lipkin P. Attention-­deficit hyperactivity
(childhood onset), hypogonadism, disorders mimic disorder. In: Accardo PJ,
small hands and feet, short stature, editors. Developmental disabilities in
hypopigmentation and almond shaped infancy and childhood. Vol. 2. 3rd ed.
eyes. Methylation analysis is the test of Baltimore: Paul Brookes Publishing; 2008.
choice to diagnose the syndrome. 75% p. 684.
of affected individuals have a deletion
of the long arm of chromosome 15 at q ?? 9. A 52-year-old woman with the his-
11-q13. The paternally derived chromo- tory of progressive tremor for the last
some has been deleted. Maternal unipa- 1–2 years. She has both resting and
rental disomy accounts for 20% of cases. action tremors on exam. Her action
tremors are worst at end point. She
zz Suggested Reading has a past medical history of primary
55 Jones KL. Smith recognizable patterns of ovarian failure. Her grandson has a
13 human malformation. 6th ed. severe developmental delay, but no
Philadelphia: Elsevier; 2006. p 223–7. other family history of neurologic dis-
ease. Brain MRI shows hyper intense
?? 8. A couple is in the process of adopt- signal changes in the middle cerebel-
ing a 4-year-old boy who has a his- lar peduncles and periventricular
tory of hyperactivity and behavioral white matter on T2-weighted images.  
problems. On examination, he has What is the diagnosis?
a remarkable short stature, micro- A. Spinocerebellar ataxia type 1
cephaly, short palpebral fissures, a B. Pantothenate kinase-­associated
long, smooth nasal philtrum, and thin neurodegeneration (PKAN)
upper lip.   C. Huntington disease
Exposure to which agent during the D. LRRK2 mutation
pregnancy is most likely to account E. Fragile X tremor ataxia syndrome
for his clinical presentation?
A. Methamphetamine vv Correct Answer is: E
B. Cocaine Fragile X syndrome is one of the most
C. Valproic acid common identifiable causes of intel-
D. Alcohol lectual disability in children (mainly
E. Phenytoin male). It is a result of an abnormal
Mental Retardation/Cognitive Disorders
415 13
trinucleotide triplet repeat in the FMR1 The cause of The well described MRI
gene region on the X chromosome. findings listed above in this syndrome
Female patients have a highly variable have no well-­known cause. Females with
phenotype, with a different neuropsy- these permutations may also develop
chiatric presentation, but typically the primary ovarian failure.
IQ scores are not in the intellectual SCA-1, Huntington disease and
disabled range. Patients who have only PKAN all lead to progressive neurologic
a per-mutation (and not the full muta- decline, though with different clinical
tion in the FMR-1 gene region) develop phenotypes, and not simply progressive
two other clinical phenotypes. Fragile X tremor. All of them do not have the MRI
tremor ataxia syndrome (FXTAS) occurs findings described. LRRK2 mutations
in men and women, but the neurologic result in a classic Parkinson’s phenotype.
manifestations typically developing
after the age of 50. The tremor can be zz Suggested Reading
variable, having essential, cerebellar or 55 Pandolfo M, Manto M. Cerebellar and
mixed features. Mild cognitive changes afferent ataxias. Continuum. 2013; 19(5
in executive function are also common. Movement Disorders):1312–43.
417 14

Metabolic

© Springer International Publishing AG, part of Springer Nature 2018


Y. M. Awaad, Absolute Pediatric Neurology, https://doi.org/10.1007/978-3-319-78801-2_14
418 Chapter 14 · Metabolic

?? 1. You are evaluating a 5-year-old boy 2 and 4 years of age, though is not uni-
for his development and speech formly noted. Motor skills like sitting
delay. He was born full term. He rolled and walking are lost (if they were ever
on time, but he sat very late and has acquired) and 50% of patients develop
never walked. Excessive movements seizures. Speech is much delayed, and
of his hands started at 13 months of mental retardation is common.
age and he never was able to feed Elevated levels of uric acid are
himself and had no hand coordina- noted in both urine and serum and
tion. He has very limited speech. are very helpful in screening for
His family history is unremarkable the disease. Confirmatory enzyme
of developmental or speech delay. activity of hypoxanthine-guanine-­
On examination, he has significant phosphoribosyltransferase (HGPRT),
areas of injury to his lips and cheeks in erythrocytes or culture fibroblasts,
and healing wounds on his tongue, provides a definitive diagnosis. Treat-
but skin exam is normal. His mother ment with allopurinol may prevent the
reported that he has orange crystals renal complications of the disorder;
in the urine sediment in his diaper. including kidney stones, obstructive
Prominent choreoathetoid move- uropathy, and eventual renal failure,
ments are noted. He has decrease but is not effective in treating the neu-
truncal tone, with mild spasticity in rologic m­ anifestations. Typically, drastic
his lower extremities. ­methods—such as tooth extraction,
Which of the following is most likely or continuous restraint of limbs— are
to be abnormal? needed to prevent self-mutilation.
A. glycine
B. ammonia zz Suggested Reading
C. lactate 55 Pagon RA, Adam MP, Bird TD, et al., editors.
D. uric acid Gene reviews [Internet]. Seattle: University
E. very long-chain fatty acids of Washington, Seattle; 1193–2013.

vv Correct Answer is: D ?? 2. You are evaluating a 5-year-old


This is a classic clinical presentation boy for his development delay. He
of Lesch-Nyhan disease, which is an achieved his milestones as follows:
14 X-linked disorder of purine metabolism sat late at 11 months, began pulling
leads to an overproduction of uric acid, to stand at 17 months, but has never
affecting multiple organ systems. The walked independently. He starts
underlying neurological pathophysiol- losing his ability to stand and is not
ogy is unclear, but it could be second- supporting his weight in a seated
ary to abnormality in neurotransmitter position over the last year. He has
metabolism, including dopamine, rather sever speech delay, only some grunts
than direct toxicity of uric acid itself. and no babbling. He cannot reach or
Affected children have normal develop- grasp for objects. Head circumference
ment initially, followed by motor devel- is in the 50th percentile. On examina-
opment delay at 3–6 months of age. tion, he has a very large tongue with
Involuntary movements of the limbs and prominent supraorbital ridges and
face appear in the second year of life, thick eyebrows. The liver edge is pal-
and also progressive changes in tone pated 5 cm below the costal margin.
(axial hypotonia and limb spasticity). On neurologic exam, he has poor
Oral auto-mutilation, a very classic fea- visual attention with limited tracking;
ture of the disorder, develops between axial tone is decreased with normal
Metabolic
419 14
extremity tone and normal reflexes. (and more specifically one of the muco-
His brain MRI shows prominent peri- polysaccharidoses). The specific form of
vascular spaces, without other obvi- MPS can be suggested by the abnormal
ous abnormality.   excretion of urinary glycosaminoglycans,
What is most likely diagnosis? and confirmatory testing involves assess-
A. adrenoleukodystrophy ing and quantifying the specific enzyme
B. fragile X syndrome deficiency. A spectrum of severity is pres-
C. metachromatic leukodystrophy ent with patients with some residual activ-
D. mucopolysaccharidosis ity, having in general a milder and less
E. Rett disease rapidly progressive form of the disease.
On the other hand, the oligosac-
vv Correct Answer is: D charidoses, present in a similar fashion
This boy has sever global developmen- with global developmental delay (and/
tal delay. Majority of those children will or regression) with evidence of extra
never have a definitive etiology for their neuronal storage and must be screened
delays. The AAN guidelines have found for separately by assessing for abnormal
brain MRI and chromosome microar- urinary oligosaccharide excretion.
ray testing to be of the highest yield Another way to divide global delay
in patients with non-specific and non- children with or without regression, is by
syndromic global delay. In other cases, signs indicating a white matter (leuko-
a full medical history and physical exam dystrophy) versus grey matter (poliodys-
can often assist the child neurologist in trophy) disorder. Both metachromatic
reaching the appropriate diagnosis. This and adrenoleukodystrophy are disorders
is especially true if there is progressive affecting the white matter primarily
loss of previously acquired developmen- leading mostly to motor regression early
tal skills (neuro-regression.) Certain exam in the course, with evidence of spastic-
findings are very helpful; evaluation of ity, brisk reflexes and optic atrophy.
growth parameters including head cir- Neither of these disorders would be
cumference, assessment for dysmorphic expected to have prominent early cogni-
features or neuro-cutaneous stigmata tive delays nor signs of storage on exam.
and other signs of storage which include Fragile X syndrome presents with
macro-glossia and hepatosplenomegaly global developmental delay and in par-
can be of great importance to recognize. ticular, prominent intellectual disability,
The neurologic exam should document though would not be expected to lead
whether there is presence or absence of to regression in skills nor signs of pro-
concomitant white matter (spasticity, gressive storage with time (though mild
brisk reflexes, optic atrophy), deep grey dysmorphic features are present).
nuclear (abnormal movements or rigid- Rett syndrome, (identified by MECP2
ity) or cerebellar (truncal or limb ataxia) mutational analysis) is X-linked domi-
involvement. When particular clues do nant and thus typically affects females
arise from the history or exam more (most often lethal before birth in males).
focused laboratory testing can often Surviving males with this disorder have
improve the diagnostic yield. This patient duplications of the genetic region,
has evidence of enlarged non-neuronal rather than deletion, and have promi-
tissue, including bone (supraorbital nent difficulties early on in life with
ridging), tongue and liver. Sometimes feeding due to low muscle tone; and
referred to as the Hurler phenotype although they may have dysmorphic
­presence of such enlargement is highly features, they do not develop progres-
indicative of lysosomal storage disorder sive coarsening with time.
420 Chapter 14 · Metabolic

zz Suggested Reading What is the most likely diagnoses?


55 Fenichel GM. Clinical pediatric neurology. A. Wilson disease
6th ed. Philadelphia: Saunders Elsevier; B. acute disseminated encephalomy-
2009. p. 119–51. elitis
55 Michelson DJ, Shevell MI, Sherr Eh, et al. C. hereditary spastic paraplegia
Evidence report: genetic and metabolic D. metachromatic leukodystrophy
testing on children with global develop- E. pantothenate kinase-­associated
mental delay: Report of the Quality neurodegeneration
Standards Subcommittee of the American
Academy of Neurology and the Practice vv Correct Answer is: E
Committee of the Child Neurology The clinical and radiological findings
Society. Neurology. 2011;77:1629–35. are classic of Pantothenate kinase-
55 Muenzer J. Overview of the mucopolysac- associated neurodegeneration (PKAN),
charidoses. Rheumatology (Oxford). formerly Hallervorden-Spatz syndrome.
2011;50 Suppl 5:v4–12. It is one of brain iron accumulation
(NBIA) neurodegenerative disorders.
?? 3. You are seeing a 9-year-old female Infantile neuroaxonal dystrophy, neuro-
with a history of progressively wors- ferritinopathy and aceruloplasminemia
ening dystonia, choreoathetosis, dys- are other types of NBIA. It is an autoso-
arthria and intellectual deterioration mal recessive disorder and associated
for the last 3 years. On your examina- with mutations in the PANK2 gene. MRI
tion for noted, pigmentary retinopa- shows the “eye of the tiger” sign, with
thy and extensor plantar responses. hyper-­intensity in the medial globus
MRI is shown.   pallidus surrounded by hypo-intensity.

zz Suggested Reading
55 Gregory A, Hayflick SJ. Pantothenate
kinase-associated neurodegeneration.
In: Gene Reviews at Gene Tests: Medical
Genetics Resource (online resource).
Copyright University of Washington,
Seattle. 1997–2012. Available at 7 www.
14 ncbi.nlm.nih.gov/books/NBK1490/. 2002.
Accessed 9-4-12.

?? 4. You are evaluating a 6-year-old boy


for his gait. He was born prematurely
at 33 weeks gestation, stayed in the
regular nursery for 1 week because he
had feeding and temperature control
problems, he was discharged home
without other complication. His early
milestones were on target, including
walking at 13 months of age. Over the
last few months, he has had difficulty
with his walking and running, start
..      Fig. 14.1 MRI image shows iron deposits in the
basal ganglia, the so-called eye-of-the-tiger sign to trip and fall more often. Simul-
(T2w GRASE sequence). (Enro2002. Wikipedia) taneously, he started to have some
Metabolic
421 14
behavioral regression with increased vv Correct Answer is: A
outbursts and difficulty understand- The boy has clinical and radiographic
ing his speech. You examined him and manifestations of a white matter func-
noticed that, he has poor eye contact, tion disorder. With the A diagnosis
some dysfluent speech, and is not of cerebral palsy (a static disorder of
interested in following most direc- motor control by definition) is incorrect
tions. He has mild spasticity in both because of the progressive nature of the
legs, with brisk reflexes. Gait is spastic disease.
in nature, with a mild crouch and some Lesch-Nyhan syndrome and Menkes
scissoring. His brain MRI is shown. disease are primarily grey matter dis-
orders with progressive dementia and
seizures. Additional deep grey matter
dysfunction is seen with the former as
well.
Dopa-responsive dystonia can
have misdiagnosed as a cerebral palsy,
though a careful history will have
demonstrated its diurnal nature (with
patients often unaffected in the morn-
ing upon awakening) and the exam will
disclose dystonia rather than spasticity.
Adrenoleukodystrophy (ALD) is an
X-linked disorder of oxidation of very
long-­chain fatty acids (by peroxisomes)
leading to accumulation in tissues
and plasma. The classic presentation
occurs in young boys aged 4–8 years,
with concomitant change in behavioral
regulation, cognition and gait. Corti-
cotropin stimulation test can detect
early subclinical adrenal dysfunction.
The disease is relentless, leading to
..      Fig. 14.2 Adrenoleukodystrophy and MRI showing death within 3 years of presentation
T2 weighted axial scan at the level of the caudate in most children. Different treatments
heads demonstrates marked loss of posterior white
modalities have been tried in the form
matter, with reduced volume and increased signal
intensity. The anterior white matter is spared. Features of vitamin or enzyme therapy without
are consistent with X-linked adrenoleukodystrophy. much success. Bone marrow transplan-
(Sources: Radiopediacase. 7 http://radiopaedia.org/ tation is useful if initiated early in the
images/318699 and 7 http://radiopaedia.org/ disease. Unfortunately, by the time the
uploads/radio/0031/8699/T2.jpg. Frank Gaillard)
diagnosis is made it is often too late to
affect the disease process. Diagnosis is
What is the most likely diagnosis for often suggested by the pattern of white
this boy? matter hyper-­intensity on MRI. Posterior
A. adrenoleukodystrophy predominant changes, sometimes also
B. cerebral palsy involving the splenium of the corpus
C. dopa-responsive dystonia callosum and not uncommonly contrast-­
D. Lesch-Nyhan syndrome enhancing along the leading edge of
E. Menkes disease the actively demyelinating areas, which
422 Chapter 14 · Metabolic

should suggest the diagnosis and dif- vv Correct Answer is: E


ferentiate ALD from other primary leu- Myophosphorylase deficiency (McArdle
kodystrophies. Elevated levels of very disease) is a common disorder of
long-chain fatty acids (24 and 26 carbon carbohydrate metabolism caused by
length molecules) are found in the mutations in the PYGM gene encoding
plasma, as is found in most peroxisomal myophosphorylase. It is inherited as
disorders. an autosomal recessive disorder, and
presents with episodes of myoglobin-
zz Suggested Reading uria occurring as a result of brief intense
55 Fenichel GM. Clinical pediatric neurology. or prolonged low-level exercise. The
6th ed. Philadelphia: Saunders Elsevier; classic manifestations in childhood are
2009. p. 150–51. exercise intolerance, fatigue, myalgias,
poor endurance and cramping. “Second-
?? 5. You are evaluating a 14-year-old wind phenomenon” in which brief rest
male who was referred by his coach allows for dissipation of muscle pain is
because the boy has had difficulties an associated finding. Although fixed
during training. His father reports weakness can occur later in the course
that the coach has noticed that the of the disease, its early presentation is
boy’s endurance is poor and that he with episodic muscle pain and exercise
complains frequently of cramps and intolerance.
muscle pain. The boy told you that Unlike patients with adult-onset car-
he is able to continue his training as nitine palmitoyltransferase II deficiency,
long as he is able to rest briefly as this CK remains elevated even between
allows the muscle pain to lessen. Even episodes of myoglobinuria. Needle EMG
resuming exercise after rest, he must is usually normal, although occasional
resume at a slightly reduced level to myotonic discharges, muscle irritability
prevent recurrence of muscle pain and myopathic potentials with rapid
and cramping. Also, he told you that recruitment may be seen.
his urine becomes very dark colored Cori or Forbes disease (glycogen
after football practices. You suspect storage disease, type III) is secondary of
a common disorder of carbohydrate debranching enzyme deficiency. Muscle
metabolism resulting in a metabolic weakness may present late in life, with a
14 myopathy. distal predominant weakness. Systemic
Which statement is true regarding symptoms are much more common in
myophosphorylase deficiency? childhood and include hepatomegaly,
A. Needle EMG typically demon- short stature, intermittent hypoglyce-
strates myotonic discharges. mia and rarely seizures.
B. It is inherited in an autosomal
dominant fashion. zz Suggested Reading
C. It is characterized by progressive 55 Tobon, Alejandro. Metabolic myopathies.
proximal weakness in a limb- CONTINUUM: Lifelong Learn Neurol.
girdle pattern. 2013;19(6):1571–97.
D. It occurs as a result of deficiency
of debranching enzyme. ?? 6. Mutations of the FBN1 gene encoding
E. CK remains persistently elevated fibrillin are associated with which of
even in between episodes of myo- the following disorders?
globinuria. A. Ehlers-Danlos syndrome
Metabolic
423 14
B. Hereditary hemorrhagic telan- 55 Cañadas V, Vilacosta I, Bruna I, Fuster
giectasia (Rendu-Osler-Weber V. Marfan syndrome. Part 1: pathophysiol-
­syndrome) ogy and diagnosis. Nat Rev Cardiol.
C. Marfan syndrome 2010;7(5):256–65. Epub 2010 Mar 30.
D. Sturge-Weber syndrome
E. Susac syndrome ?? 7. Which of the following is X linked?
A. Ornithine transcarbamylase (OTC)
vv Correct Answer is: C deficiency (OTC)
Marfan syndrome is a connective-tissue B. Carbamoyl phosphate synthetase
disorder inherited in an autosomal deficiency (CPS1)
dominant manner and caused mainly C. Argininosuccinate acid lyase defi-
by mutations in the gene FBN1. This ciency (ASL)
gene encodes fibrillin-1, a glycopro- D. Argininosuccinate acid synthetase
tein that is the main constituent of the deficiency (ASS)
micro-fibrils of the extracellular matrix.
Most mutations are unique and affect vv Correct answer is: A
a single amino acid of the protein. Ornithine transcarbamylase deficiency
Reduced or abnormal fibrillin-1 leads to is the only urea cycle disorders is inher-
tissue weakness, increased transform- ited as an X linked disorder; the rest are
ing growth factor beta signaling, loss autosomal recessive. Elevated ammonia,
of cell-­matrix interactions, and finally, respiratory alkalosis, vomiting, and
to the different phenotypic manifesta- encephalopathy are the characteristic
tions of Marfan syndrome. Since the findings in urea cycle disorders. If there
description of FBN1 as the gene affected is a suspicion for a urea cycle disorder
in patients with this disorder, great if ammonia is elevated, suggested test-
advances have been made in the under- ing includes arterial blood gas, serum
standing of its pathogenesis. glucose, electrolytes, lactate, and amino
The development of several mouse acids in addition to urine orotic acid and
models has helped our understanding amino acids. OTC deficiency is marked
of this disease, and it would change the by low or absent serum citrulline, and
treatment and the prognosis of patients high urine orotidine/ orotic acid.
in the coming years. Cardiovascular Carbamoyl phosphate synthetase
involvement which is one of many dif- deficiency, is inherited in an autosomal
ferent clinical manifestations of Marfan recessive fashion like most other urea
syndrome, deserves special consider- cycle disorders. In OTC deficiency, there
ation, due to its impact on prognosis. is elevated plasma ammonia and low
Ghent criteria which requires a compre- or absent citrulline. In this case, urine
hensive clinical assessment of multiple orotidine/ orotic acid is low or normal.
organ systems helps in the diagnosis of Lethargy, encephalopathy, and eye
Marfan syndrome. Genetic testing can movement abnormalities may be pres-
be useful in the diagnosis of selected ent at birth.
cases. Argininosuccinic aciduria is second-
ary to argininosuccinate lyase defi-
zz Suggested Reading ciency, and inherited as an autosomal
55 Cunha L. Marfan’s syndrome. In: recessive manner. Laboratory tests are
Uncommon cause of stroke. Caplan L, significant for elevated ammonia and
editor. 2nd ed. New York: Cambridge citrulline. Urine, plasma, and cerebro-
University Press; 2008. p. 131–8. spinal fluid may have increased levels of
424 Chapter 14 · Metabolic

argininosuccinic acid. Urine orotic acid Mutations in the quinoid dihydrop-


is also elevated. teridine reductase (QDPR) gene lead to
Citrullinemia type I is caused by dysfunction of quinoid dihydropteridine
defects in argininosuccinate synthetase. reductase which is involved in the cre-
It affect neonates who will have hypo- ation of tetrahydrobiopterin (BH4) from
tonia and seizures. Ammonia and citrul- quinoid dihydropterin. The resultant
line levels are usually increased. deficiency of BH4, which is necessary
for hydroxylation of phenylalanine and
zz Suggested Reading processing of tyrosine and tryptophan,
55 Haberle J, Boddaert M, Burlina A, leads to defects in serotonin, epineph-
Chakrapani A, Dixon M, Huemer M, rine, and norepinephrine and dopamine
Karail D, Martinelli D, Sanjurjo Crespo P, synthesis. This is associated with hypo-
Santer R, Servais A, Valayannopoulos V, tonia, oculogyric crisis, seizures, and
Lindner M, Rubio V, Dionisi-Vici involuntary movements.
C. Suggested guidelines for the diagnosis Mutations in sepiapterin reductase
and management of urea cycle disorders. (7, 8-dihydrobiopterin: NADP+ oxido-
Orphanet J Rare Dis. 2012;7(32):1–30. reductase) (SPR) lead to altered func-
55 Lee B. Urea cycle disorders; clinical tion of sepiapterin reductase, another
features and diagnosis. In: Hahn S, TePas molecule involved in the production
E, editors. Up-to-date. Waltham. Accessed of tetrahydrobiopterin. Dystonia,
on 12 Dec 2015. spasticity, and mental retardation are
55 Menkes JH, Wilcox WR. Inherited common with Sepiapterin reductase
metabolic diseases of the nervous system. deficiency. Phenylalanine levels in
In: Menkes JH, Sarnat HB, Maria BL, the blood may be normal, although
editors. Child neurology. 7th ed. elevated levels sepiapterin, biopterin,
Philadelphia: Lippincott Williams and and dihydropterin may be present in the
Wilkins; 2006L. p. 29–142. ­cerebrospinal fluid.
Mutations in GTP cyclohydrolase 1
?? 8. Which of the following protein is (GCH1) lead to alterations in in the pro-
defective in phenylketonuria? tein cyclohydrolase. This is also required
A. Dihydropteridine reductase for tetrahydrobiopterin synthesis and
B. Sepiapterin reductase dysfunction of this protein results in
14 C. Phenylalanine hydroxylase hypotonia, hyperthermia, and seizures.
D. GTP cyclohydrolase Mutations in this gene also cause dopa
E. 6-pyruvoyl tetrahydrobiopterin responsive dystonia. Mutations in
synthase 6-pyruvoyltetrahydropteri n synthase
(PTS) lead to dysfunction in the process-
vv Correct answer is: C ing of tetrahydrobiopterin. In this case,
Phenylketonuria is caused by muta- phenylalanine hydroxylase activity is
tions in the phenylalanine hydroxylase normal although phenylalanine levels
(PAH) gene. Seizures, developmental are high. Patients may have hypotonia,
delay, and mental retardation will occur seizures, and involuntary movements.
because of the inability to process phe-
nylalanine. Rough dry skin, musty odor, zz Suggested Reading
microcephalus, and tremor are other 55 Menkes JH, Wilcox WR. Inherited
clinical features. Much of this can be metabolic diseases of the neNous system.
avoided if a phenylalanine restricted In: Menkes JH, Sarnat HB, Maria BL,
diet is followed. editors. Child neurology. 7th ed.
Metabolic
425 14
Philadelphia: Lippincott Williams and Scheie syndrome is the least severe-
Wilkins; 2006L. p. 29–142. with life expectancy in the decades.
55 National Library of Medicine (US). Hunter disease refers to mucopoly-
Genetics Home Reference [Internet]. saccharidoses type 2, a deficiency of
Bethesda: The Library; 2015 Dec 7. iduorate sulfatase. Presentation is simi-
Tetrahydrobiopterin deficiency; [reviewed lar to that of Hurler syndrome although
2011 July; cited 2015 Dec 13]; Available corneal clouding is not present.
from: 7 http://ghr.nlm.nih.gov/condition/ Mucopolysaccharidoses type 3, which
tetrahydrobiopterin-deficiency. encompasses Sanfilippo A, B, C, and D,
55 National Library of Medicine (US). is another subtype. Type A is associated
Genetics Home Reference [Internet]. with sulfamidase deficiency, type B is
Bethesda: The Library; 2015 Dec 7. SPR; associated with alpha N acetylglucosami
[reviewed 2012 May; cited 2015 Dec 13]; nidase deficiency, type C is associated
Available from: 7 http://ghr.nlm.nih.gov/ with acetyl-­CoA: alpha­glucosaminide
gene/SPR. n-acetyl transferase deficiency, and
55 National Library of Medicine (US). type D is associated with N acetylglu-
Genetics Home Reference [Internet]. cosami ne 6 sulfatase deficiency. All
Bethesda: The Library; 2015 Dec 7. GCH1; lead to build up of heparan sulfate.
[reviewed 2012 May; cited 2015 Dec 13]; Presentation includes behavioral prob-
Available from: 7 http://ghr.nlm.nih.gov/ lems, developmental delay, and sleep
gene/GCH1 disorders. Sanfilippo syndrome type B
is associated with alpha-N-acetylglu-
?? 9. Which of the following is associated cosaminidase deficiency with buildup
with deficiency of alpha L lduronidase? of heparan sulfate, aggressive behavior,
A. Hurler syndrome and neurological decline.
B. Hunter syndrome Mucopolysaccharidoses type 4
C. Sanfilippo syndrome type A encompasses Morquio syndrome
D. Sanfilippo syndrome type B type A and Morquio syndrome type
E. Morquio syndrome type A B. Morquio type A is caused by defects
in galactosamine-6-­sulfatase, while
vv Correct answer is: A Morquio type B is caused by dysfunc-
Mucopolysaccharidoses type1 is a lyso- tion of beta galactosidase. Mucopoly-
somal storage disease caused by muta- saccharidoses type 4 is characterized
tions in the lduronidase, alpha-L (IDUA) by short stature, lax ligaments, genu
gene which codes for alpha L lduroni- valgum, C1-2 subluxation, hearing loss,
dase. Mucopolysaccharidoses type1 coarse facial features, and valvular heart
encompasses Hurler syndrome, Hurler disease.
Scheie syndrome, and Scheie syndrome.
Hurler is the most severe with hepa- zz Suggested Reading
tosplenomegaly, breathing difficul- 55 Wynn R. Mucopolysaccharidoses: clinical
ties, dystosis multiplex, valvular heart features and diagnosis. In: Hahn, Patterson
disease, hearing loss, hydrocephalus, MC, TePas E, editors. Up-to-date,
myelopathy, and coarse facial features. Waltham. Accessed on 12 Dec 2015.
Death usually occurs in the first decade. 55 Murali HR, Renaud DL. Inborn errors of
Hurler Scheie is less severe with death metabolism. In: Mowzoon N, Flemming
occurring by age 20. Micrognathia, toe KO, editors. Neurology board review: an
walking, and coarse facies may be pres- illustrated study guide. Rochester: Mayo
ent. Intelligence may be normal. Clinic Scientific Press; 2007. p. 933–80.
426 Chapter 14 · Metabolic

?? 10. Which of the following is caused by zz Suggested Reading


defects in mitochondrial DNA? 55 National Library of Medicine (US).
A. Carnitine palmitoyltransferase I Genetics Home Reference [Internet].
deficiency Bethesda: The Library; 2015 Dec 7.
B. Carnitine palmitoyltransferase II Complex III deficiency; [reviewed 2014
deficiency Apr; cited 2015 Dec 13]; Available from:
C. Carnitine-acylcarnitine translo- 7 http://ghr.nlm.nih.gov/condition/
case deficiency mitochondrial-complex-iii-deficiency.
D. Cytochrome b ­deficiency 55 O’Farrell E. Mitochondrial myopathies
caused by disorders of lipid and purine
vv Correct answer is: D metabolism. In: Patterson MC, Dashe JF,
Defects in the mitochondrial gene editors. Up-to-date. Waltham. Accessed on
MT-­CYB (mitochondrial encoded cyto- 12 Dec 2015.
chrome b) can cause the cytochrome 55 Darras BT. Mitochondrial myopathies:
b deficiency. This leads to dysfunction clinical features and diagnosis. In: Shefner
of mitochondrial complex Ill. include JM, Dashe JF, editors. Up-to-date.
Fatigue, myopathy, exercise intolerance, Waltham. Accessed on 12 Dec 2015.
encephalopathy, cardiomyopathy, and
renal dysfunction are the classic mani- ?? 11. You are evaluating a suspected cere-
festations. bral palsy 3-year-old male with a his-
Defects in the gene carnitine palmi- tory of psychomotor delay, retinitis
toyltransferase 1A (liver) (CPT1A) can pigmentosa, failure to thrive with
cause the carnitine palmitoyltransferase persistent large volume loose stools.
I deficiency. This is nuclear DNA, not His past laboratory results include a
mitochondrial DNA. Hepatic encepha- hemoglobin level of 6.8 g/dl, plasma
lopathy, myopathy of adult onset, or cholesterol level of 85 mg/dl and tri-
acute fatty liver of pregnancy are the glyceride level of 22 mg/dl.  
clinical findings. What blood tests are going to order
Carnitine palmitoyltransferase II to reach the diagnosis?
deficiency is caused by mutations in A. Apolipoprotein B
nuclear DNA; CPT2 (carnitine palmi- B. Carnitine
toyltransferase 2) is the specific gene C. Creatine kinase
14 implicated. This has a wide variation in D. Plasma phenylalanine
phenotypes. In its more severe form,
patients die in the neonatal period after vv Correct answer is: A
presenting with liver failure, seizures, This is a clinical presentation of Abetali-
encephalopathy, cardiomyopathy, and poproteinemia. It results in fat malab-
hypoglycemia. sorption from birth and is characterized
Defects in nuclear, not mitochon- by low density lipoprotein deficiency.
drial, DNA can cause the carnitine- This results in insufficient cholesterol,
acylcarnitine translocase deficiency. The triglyceride and vitamin absorption for
gene implicated is solute carrier family normal development. Chronic vitamin
25 (carnitine/acylcarnitine translocase), E deficiency is the reason for the neuro-
member 20 (SLC25A20). It appears logical complications. Retinitis pigmen-
in the neonatal period with elevated tosa is present in almost all cases.
ammonia, hypoglycemia, and encepha- Carnitine is a cofactor that aids
lopathy. Dilated cardiomyopathy may in the transfer of long­chain fatty
also be present. acids across the inner mitochondrial
Metabolic
427 14
membrane. Carnitine deficiency pres- manifestations like peripheral neu-
ents with progressive proximal muscle ropathy, tremor, ataxia, blindness and
weakness. seizures are common with high blood
Creatine kinase levels are elevated in levels.
myopathies and muscular dystrophies Lead intoxication in children at low
that typically present with muscle weak- levels causes slowed developmental
ness in children and gait disturbance in progress with greatest effect on lan-
older children. guage development. However, acute
Phenylketonuria presents with lead encephalopathy can be rapidly
similar developmental delays but is not fatal.
associated with anemia and abnormal Gauchers type Ill presents with hep-
lipid profile. atosplenomegaly, impaired bone mar-
row production of red blood cells and
zz Suggested Reading platelets and progressive neurological
55 Burnett JR, Hooper AJ. Vitamin E and symptoms due to an accumulation of
oxidative stress in abetalipoproteinemia glycolipids in reticuloendothelial cells.
and familial hypobetalipoproteinemia.
Free Radical Biol Med. 2015;88:59–62. zz Suggested Reading
55 Lorincz MT. Recognition and treatment of
?? 12. What is the most likely diagnosis is neurologic Wilson’s disease. Semin Neurol.
a 16-year-old boy with a 6-month 2012; 32(5):538.
history of progressive dysarthria, dys-
tonia and Kayser-Fleischer rings on ?? 13. You are seeing a 4 days old newborn
examination? who developed ketoacidosis, hypo-
A. Mercury poisoning glycemia, hypotonia, and lethargy. His
B. Hepatolenticular degeneration urine has a characteristic sweet odor.
C. Lead encephalopathy Which statement regarding this dis-
D. Gauchers Type Ill disease order is true?
A. It is a disorder of lysosoma l storage
vv Correct answer is: B B. It results in an accumulation of
Hepatolenticular degeneration (Wilson alanine
disease), is a disorder of copper metab- C. A variant is responsive to vitamin
olism and can present with hepatic, B6 (pyridoxine
neurological or psychiatric symptoms. D. There is a deficiency of branched-
Excess copper accumulation in the liver, chain ketoacid dehydrogenase
brain and cornea damage tissue. The E. MRI demonstrates basal ganglia
most common neurological symptoms calcifications
in children are dysarthria and dysto-
nia. Children under the 10 year of age vv Correct Answer is: D
present with liver failure. The Kayser-­ The clinical scenario is indicative of
Fleischer ring is a prominent predictor maple syrup urine disease. It is a result
of the disease and is evident when cop- of a deficiency of branched-chain keto-
per deposits along the limbus of the acid dehydrogenase. The enzyme decar-
cornea as a yellow-­brown pigment. boxylates branched chain amino acids,
Exposed to mercury is common and its deficiency results in the accumu-
in general population from food with lation of isoleucine, leucine, and valine.
fish being the major source of methyl Dysmyelination, heterotopias, diffuse
mercury exposure. Neurological spongy degeneration, and a f­ ailure
428 Chapter 14 · Metabolic

of cortical lamination are the main C. Acute inflammatory demyelinat-


manifestations. Severe deficiency has ing polyneuropathy
a neonatal onset, and occurs in babies D. Acute disseminated encephalo-
who are normal at birth, but develop myelitis
ketoacidosis, hypoglycemia, hypotonia, E. Acute intermittent porphyria
lethargy, seizures, and cerebra l edema
after beginning feedings containing vv Correct Answer is: E
protein. If untreated, coma and respira- This is a case of an acute hepatic por-
tory failure will result in death. A milder phyria. It is an autosomal dominant and
form has later onset, with a moderate involve multiple areas of the nervous
deficiency of the enzyme. Ketoacidosis, system. They are secondary to enzy-
ataxia, and intellectual disability are matic dysfunction in porphyrin metabo-
the main symptoms. The characteristic lism with subsequent abnormalities in
feature is a sweet smell of the urine. heme synthesis and metabolism. These
Subcortical white matter changes and disorders include acute intermittent
cerebral edema, particularly after a porphyria (AIP), which results from
protein-containing meal are seen on various mutations that lead to reduced
MRI. Hemodialysis and diets low in activity of the enzyme porphobilino-
branched-chain amino acids are the gen deaminase, variegate porphyria,
treatment of choice. A thiamine respon- which results from reduced activity of
sive variant exists; hence a trial of thia- the enzyme protoporphinogen IX oxi-
mine is warranted. dase, hereditary coproporphyria which
results from reduced activity in the
zz Suggested Reading enzyme coprophorphinogen oxidase,
55 Fenichel GM. The disorder has autosomal and plumboporphyria, which results
recessive inheritance. Clin Pediatr Neurol. from reduced enzymatic activity of Li-
5th edition. Philadelphia: Elsevier aminolevulinic acid (ALA) dehydratase.
Saunders; 2005. p. 4–5. Symptoms often do not appear until
adolescence or later because the dys-
?? 14. You are evaluating a 15-year-old functional enzymes have some residual
female in the ED with a history of enzyme activity, with some symptoms
severe abdominal pain, nausea, and occurring in episodes with exposure to
14 vomiting for 2 days. All her labs includ- certain triggers. Medications, menstrua-
ing liver enzymes are normal. She tion, and alcohol exposure are common
received intravenous fluids in the triggers. With exposure to these trig-
ED. Today, she has a generalized tonic- gers, activity of the hepatic enzyme ALA
clonic seizure followed by hallucina- synthase increases, leading to overpro-
tions and delusions. Brain MRI with duction of heme precursors that can-
contrast and CSF analysis are normal. not be sufficiently metabolized by the
On day 4 of her admission to the hospi- involved downstream enzymes.
tal, she is noted to have mild weakness In spite the different porphyries dif-
of wrist extension; 1 week later, exami- fer in several biochemical aspects, their
nation shows bilateral wrist drop.   neurologic manifestations are similar.
What is your diagnosis in this Symptoms typically manifest in early
patient? adulthood, with females being more
A. Wilson’s disease commonly and more severely affected as
B. Systemic lupus erythematosuis compared to males. The early symptoms
Metabolic
429 14
typically is abdominal pain and other The history in this vignette is not
gastrointestinal symptoms, followed by consistent with acute disseminated
neurologic symptoms, including most encephalomyelitis, in which the MRI
prominently manifestations of auto- would be abnormal, or acute inflamma-
nomic instability (tachycardia, labile tory demyelinating polyneuropathy.
hypertension, orthostasis, and urinary Although systemic lupus erythemato-
retention). Neuropsychiatric symptoms suis can lead to a similar clinical picture,
including psychosis occur, and seizures normal MRI of the brain and CSF analy-
may be present as well. In some patients, sis make this less likely.
a subacute predominantly motor neu-
ropathy occurs; the arms may be affected zz Suggested Reading
prior to the legs, and proximal muscles 55 Bradley WG, Daroff RB, Fenichel GM,
are involved more than distal muscles. et al. Neurology in clinical practice, 5th ed.
Involvement of the radial nerve in Philadelphia: Elsevier; 2008.
isolation may occur. The neuropathy is 55 Ropper AH, Samuels MA. Adams and
both axonal and demyelinating. Cranial Victor’s principles of neurology, 9th ed.
nerve involvement, and even respira- New York: McGraw-Hill; 2009.
tory muscle involvement leading to 55 Comprehensive review in clinical neurol-
respiratory failure, may occur. ogy: a multiple-choice question book for
In AIP, photosensitivity is not present, the wards and boards ©2011 Wolters
whereas with variegate porphyria and Kluwer Health Lippincott Williams &
hereditary coproporphyria, photosen- Wilkins. All rights reserved.
sitivity with skin blistering and hyper-
pigmentation occur. During an attack, ?? 15. You were asked to see a newborn,
elevated levels of porphobilinogen and who is lethargic with poor feeding
aminolevulinic acid are detectable in the and rapidly becomes encephalo-
urine and serum. Elevated protoporphy- pathic and develops seizures. His lab
rinogen and coproporphyrinogen in the tests form the nursery shows hyper-
stool distinguish variegate porphyria ammonemia with respiratory alkalosis
and hereditary coproporphyria from AIP. and normal anion gap. The NICU
Treatment of the porphyrias involves team has to intubate him and start
prevention of attacks by avoidance of mechanical ventilation. You suspected
drugs that may precipitate the attack a urea cycle disorder.
(including anti-­epileptics, particularly Which of the following is correct
barbiturates) and other triggers. Treat- regarding this group of disorders?
ment of attacks includes supportive A. This is a group of disorders that
care, a high-carbohydrate diet, and in are all inherited in an X-linked
some cases hematin, which suppresses fashion
delta ALA synthase activity. B. Ornithine transcarbamylase defi-
Although Wilson’s disease is on the ciency is the most common cause
differential diagnosis of patients with C. Hyperammonemia and organic
neuropsychiatric symptoms, it does acidemias are commonly found in
not typically lead to peripheral motor patients with these disorders
mono-neuropathies or abdominal pain, D. Arginase deficiency commonly
except in the setting of hepatic impair- presents in the newborn period
ment, and there is no evidence of liver E. High protein diet is indicated for
disease in the case presented. their treatment
430 Chapter 14 · Metabolic

vv Correct Answer is: B and fat. During acute episodes, sodium


Urea cycle disorders are a group of con- benzoate and sodium phenylacetic acid
ditions caused by deficiency of enzymes are used, and sometimes dialysis may
responsible for urea synthesis and more be required. Mannitol has been used for
importantly ammonia and ammonia-­ brain edema and increased intracranial
containing compound removal. The pressure. Long-term treatment with
most common disorder is Ornithine low-protein diet and essential amino
transcarbamylase (OTC) deficiency acids, as well as arginine supplementa-
which is X-linked. Carbamyl phosphate tion (except in arginase deficiency), may
synthetase deficiency, argininosucci nic stabilize the neurologic deterioration.
acid synthetase deficiency, argininosuc-
cinic acid lyase deficiency, and arginase ?? 16. You are seeing a newborn baby
deficiency are an autosomal recessive who developed encephalopathy
disorder. These enzyme defects will lead secondary to metabolic acidosis and
to hyper-ammonemia, which is the main hyper-ammonemia. There is also keto-
cause of the clinical manifestations. acidosis and elevated methylmalonic
Ammonia induces glutamine accumula- acid levels. You suspect methylmalo-
tion, which leads to astrocyte swelling nic acidemia.  
and brain edema. Hyper-ammonemia, Which of the following is incorrect
encephalopathy, and respiratory alka- regarding this condition?
losis is the triad suggesting of those A. The deficient enzyme is D-methyl-
disorders. This infant has very high con- malonyl-CoA mutase
centrations of ammonia, no evidence of B. Propionyl-CoA levels are not
organic acidemias, normal anion gaps, altered in this enzymatic disorder
and normal serum glucose level. C. Adenosylcobalamin acts as cofac-
Liver biopsy specimens can help in tor in the involved enzymatic step
the amino acid analyses and enzyme D. Protein restriction in the diet is
activity for specific urea cycle disorders. part of the treatment plan
The clinical presentation often E. The enzyme affected participates in
begins in the newborn period with pro- the conversion of L-­methylmalonyl­
gressive lethargy, vomiting, hypotonia, CoA to succinyl-CoA
and seizures. Higher levels of ammonia
14 may be associated with coma and even- vv Correct Answer is: B
tually death. In methylmalonic acidemia, the level
Females with OTC deficiency, and of propionyl-CoA increases because its
some patients with partial deficien- accumulation. It is an autosomal reces-
cies, may have late-onset presentations sive condition caused by the deficiency
and become symptomatic after large of D-methylmalonyl-CoA mutase. This
amounts of protein ingestion or inter- enzyme normally catalyzes the isom-
current sickness. erization of L-methylmalonyl-CoA to
Arginase deficiency does not cause succinyl­CoA, which then enters the
symptoms in the newborn, whereas Krebs cycle. Adenosylcobalamin is a
newborn presentation is common in the required cofactor. A defect in this meta-
rest of urea cycle disorders. bolic pathway leads to accumulation
Treatment includes limitation of of propionyl- CoA, propionic acid, and
nitrogen intake in the diet and adminis- methylmalonic acid, causing metabolic
tration of essential amino acids. Calories acidosis, hyperglycinemia, and hyper-­
can be supplied with carbohydrates ammonemia. At birth those children are
Metabolic
431 14
normal, becoming symptomatic within failure of lamination. It begins between
the first week of life, manifesting with 2–3 months of age with a musty odor
lethargy, failure to thrive, vomiting, of the skin and urine, intellectual dis-
dehydration, hypotonia, and respiratory ability, hyperactivity, irritability, tremor,
distress. Hematologic abnormalities, spasticity, dry and hypo-­pigmented
including bleeding disorders leading skin, microcephaly, and infantile spasms
to intracranial hemorrhage, may also that later evolve to a generalized tonic-
occur. Survivors are left with intellectual clonic seizure disorder.
disability, developmental delay, and Hartnup disease results from an
recurrent acidosis. abnormality of the neutral amino acid
The diagnosis should be suspected transport protein, such that it impairs
in newborn patients with metabolic intestinal and renal absorption of neu-
acidosis, ketosis, hyperglycinemia, and tral amino acids (i.e. tryptophan). Symp-
hyper-­ammonemia. Methylmalonic acid toms are unrelated to diet, and include
is elevated in plasma and urine, and photosensitive pellagra-­like dermatitis
the enzyme activity can be analyzed in (due to nicotinamide deficiency), devel-
fibroblasts. opmental delays, headaches, psychiatric
The treatment is a protein restricted changes, hypotonia, and ataxia. Clinical
diet and supplementation of hydroxy- presentations may be due in part to
cobalamin and carnitine. Antibiotics are intestinal absorption of toxins result-
also helpful to reduce the production ing from amino acid breakdown. Stress
of propionic acid. In the acute setting, or infection can trigger symptomatic
these patients need hydration and episodes, and take up to 1 month for
glucose administration with discontinu- resolution. The disorder is of autosomal
ation of protein intake. This condition recessive inheritance, with a mutation
is screened for by extended newborn on chromosome 5.
screening, thus allowing for pre-symp- Canavan disease is an autosomal
tomatic treatment in most patients with recessive deficiency of aspartoacyl-
the classic or severe form of the disease. ase, which results in accumulation of
N-acetylaspartate (NAA). NAA serves as
?? 17. You are evaluating a 1-year old boy an osmolyte, causing edema and spon-
for his axial hypotonia, limb hyperto- giform changes of white matter. Symp-
nia, myoclonus, intellectual disabil- toms include developmental regression,
ity, and episodic hypoglycemia. His hypotonia that evolves to spasticity,
symptoms are worse throughout the macrocephaly, dysphagia, optic nerve
day, and improve after sleep.   atrophy, seizures, and posturing with
The most likely diagnosis is: stimulation (extension of the lower
A. Phenylketonuria extremities and head, with arm flexion).
B. Hartnup disease DOPA decarboxylase deficiency
C. Canavan disease prevents conversion of L-dopa to dopa-
D. DOPA decarboxylase deficiency mine, and 5-hydroxytryptophan to sero-
E. Homocysteinuria tonin. Its symptoms begin within the
first two years of life, and consist of axial
vv Correct Answer is: D hypotonia, limb hypertonia, dystonia,
Phenylketonuria is secondary to a defi- startle myoclonus, intellectual disability,
ciency of phenylalanine hydroxylase, and autonomic dysfunction (e.g. sweats,
which causes dysmyelination, heteroto- temperature instability, ptosis, ­miosis,
pias, diffuse spongy degeneration, and a and episodic hypoglycemia). A main
432 Chapter 14 · Metabolic

feature is the worsening of symptoms manifestations such as pancytopenia


throughout the day, with improvement and bleeding disorders may occur.
after sleep. It is an autosomal recessive disorder
In homocysteinuria, heterozygous caused by a deficiency of propionyl-
mutations lead to accelerated athero- CoA carboxylase. This enzyme normally
sclerosis, while homozygous mutations participates in the carboxylation of pro-
lead to increased platelet aggregation. pionyl-CoA to D-methylmalonyl-CoA, a
Homocysteinuria consists of three sub- step that requires the coenzyme biotin.
types: (1) Cystathionine 13-synthase Children with propionic acidemia are
deficiency. Homozygous mutations normal at birth but develop symptoms
may result in strokes, myocardial infarc- either in the early neonatal period, in
tions, intellectual disability, glaucoma, infancy, or later in childhood. The clas-
lens dislocations, a marfanoid body sic presentation is feeding difficulty,
type, and osteoporosis. Symptom onset lethargy, hypotonia, dehydration, and
occurs prior to age five. (2) Methylene- attacks of metabolic acidosis and hyper-
tetrahydrofolate reductase (MTHFR) ammonemia. They may progress to have
deficiency. This form leads to a more seizures and coma. Hepatomegaly, pancy-
severe phenotype. (3) Deficiency of B12 topenia, and bleeding disorders including
coenzyme synthesis and transport. intracranial hemorrhage are also other
findings. Survivors have intellectual dis-
zz Suggested Reading ability and basal ganglia abnormalities.
55 Fenichel GM. Clinical pediatric neurology. Newborn babies with ketoacidosis,
5th ed. Philadelphia: Elsevier Saunders; with an anion gap, and elevated propi-
2005. p. 125–8,138, 222. onic acid levels in the blood and some-
times in the urine, diagnosis should be
?? 18. You are evaluating a newborn baby suspected. Elevation of glycine levels in
for encephalopathy in association of plasma and urine and of methylcitrate
metabolic acidosis and hyper-ammo- and B-hydroxypropionate in urine are
nemia. There is also ketoacidosis, and also observed. Enzyme activity in leuko-
elevated blood propionic acid and cytes or fibroblasts is reduced.
glycine levels with normal methylma- Treatment is restricted protein diet,
lonic acid levels. You suspect propi- IV fluids for hydration, as well as carni-
14 onic acidemia.   tine and biotin supplementation. Dialy-
Which of the following is correct sis may be required in some patients.
regarding this condition? Antibiotics such as metronidazole may
A. It is caused by deficiency of decrease the production of propionate
D-methylmalonyl-CoA mutase by enteric bacteria. D-methylmalonyl-
B. It is X-linked CoA mutase deficiency causes methyl-
C. High protein diet is recommended malonic academia. Propionic acidemia
for these patients is screened for by extended newborn
D. Hematologic disorders occur in screening, thus allowing for pre-­
this condition symptomatic treatment in most patients
E. Propionyl-CoA carboxylase activ- with the classic or severe form of the
ity is increased disease.

vv Correct Answer is: D ?? 19. You are evaluating a 12- month-old


This is a clinical presentation of propi- boy for his hypotonia, alopecia, and
onic acidemia, in which hematologic seizures. His Lab shows ketoacidosis,
Metabolic
433 14
hyperammonemia, and elevated zz Suggested Reading
urine organic acid levels. You suspect 55 Comprehensive review in clinical neurol-
Biotinidase deficiency.   ogy: a multiple-choice question book for
Which statement below is incorrect? the wards and boards © 2011 Wolters
A. Biotin levels are elevated in bio- Kluwer Health Lippincott Williams &
tinidase deficiency and should not Wilkins. All rights reserved.
be supplemented
B. Hearing and vision loss can occur ?? 20. You saw a 2 days old newborn for his
C. Biotinidase cleaves biocytin and seizures. The nurse told you that the
participates in biotin recycling baby refused feeding and seemed irri-
D. Biotinidase participates in pro- table in the first few hours of life. The
cessing of dietary protein-bound infant subsequently developed respi-
biotin ratory failure and required intubation.
E. Patients have ketoacidosis, hyper- You have ordered an extensive workup
ammonemia, and organic aciduria including a brain MRI (shown below).
His CSF showed a high glycine level.
vv Correct Answer is: A Which of the following is incorrect
Biotin is a water-soluble B-complex regarding this condition?
vitamin that is necessary in multiple
metabolic reactions including glu-
coneogenesis, fatty acid synthesis,
catabolism of amino acids, and gene
expression. Biotinidase deficiency can
cause biotin deficiency. This enzyme
normally cleaves biocytin, thereby
recycling biotin, and also participates
in processing of dietary protein-bound
biotin, making it available to the free
biotin pool. Biotinidase deficiency has
previously been known as late-­onset
multiple carboxylase deficiency, and it
is inherited in an autosomal recessive
fashion. Children with this enzymatic
defect manifest with seizures, hypoto-
nia, ataxia, developmental delay, hear- ..      Fig. 14.3 Corpus callosum, agenesis. Sagittal
ing and visual loss, spastic para-paresis, T1-weighted MRI of the brain shows partial agenesis of
the corpus callosum. The genu and anterior body of
and cutaneous abnormalities, including
the corpus callosum are visualized, whereas the
alopecia. Laboratory studies demon- posterior body, splenium, and rostrum are absent.
strate ketoacidosis, hyper-­ammonemia, Wikipedia
and organic aciduria. The enzyme activ-
ity can be analyzed in serum. Treatment A. This patient has a disturbance of
is biotin supplementation, which pre- glycine degradation
vents intellectual disability and reverses B. Patients who survive the acute
most of the symptoms. This condition phase are left with intellectual dis-
is screened for by extended newborn ability, spasticity, and epilepsy
screening, thus allowing for pre-symp- C. Treatment with sodium benzoate
tomatic treatment in most patients with is effective and prevents progres-
the classic or severe form of the disease. sion of the disease
434 Chapter 14 · Metabolic

D. It is autosomal recessive ?? 21. What is the treatment of a 12-month-


E. EEG may show burst suppression old boy that presents with a history
of acute episodes of ataxia, irritabil-
vv Correct Answer is: C ity and lethargy accompanied by
There is a partial agenesis of the corpus the appearance of dark blood in the
callosum on the MRI. A diagnosis of urine?
glycine encephalopathy (non-ketotic A. Maintenance anti-convulsive
hyperglycinemia) is based on the medication
MRI finding, the clinical picture and B. Carnitine and Coenzyme Q
the high CSF glycine levels. Available supplementation
treatment options are limited and not C. A phenylalanine free diet
effective. Glycine encephalopathy is D. Protein restricted diet
an autosomal recessive. The onset is
in newborns who, within a few hours vv Correct Answer is: D
after birth, become irritable with poor Maple syrup urine disease is a meta-
feeding and hiccups. Subsequently, bolic disorder caused by defect in the
they develop a progressive encepha- enzyme branched-chain keto acid
lopathy with hypotonia, myoclonic sei- dehydrogenase. It is a recessively inher-
zures, and respiratory failure requiring ited disorder. Trigger factors are minor
mechanical ventilation. Survivors will infections, surgery or a protein rich
have intellectual disability, spasticity, diet. During an episode, the urine has a
and epilepsy. Brain MRI may reveal a maple syrup smell and is made dark by
hypoplastic or absent corpus callosum blood and urine containing high levels
and gyral malformations and cerebellar of branched-chained amino acids and
hypoplasia. In the acute phase, the EEG ketoacids.
shows burst suppression and hypsar- Carnitine and coenzyme Q1O are
rhythmia. Non-ketotic hyperglycinemia commonly administered to maximize
or glycine encephalopathy is caused function in mitochondrial disorders.
by a defect in the P-protein (glycine Phenylketonuria is an autosomal
decarboxylase) gene, which encodes a recessive disorder caused by a deficiency
component of the mitochondrial gly- of the hepatic enzyme phenylalanine
cine cleavage system, therefore affect- hydroxylase and presents at birth. The
14 ing the degradation of glycine, with treatment is a phenylalanine free diet.
accumulation of this substance. Serum
and CSF glycine levels are elevated, and zz Suggested Reading
the ratio of CSF to plasma glycine con- 55 Strauss KA, Puffenberger EG, Morton DH.
centration is more than 0.6, whereas 2013. Maple syrup urine disease.
normally, it is less than 0.4. No effective
treatments are available; sodium ben- ?? 22. You are seeing a 17-year-old male in
zoate decreases plasma glycine con- the PICU with left-sided weakness and
centrations, but the CSF level does not aphasia. His head CT showed an acute
normalize with this therapy, and neuro- ischemic infarct in the territory of the
logic dysfunction is not reversible. Ben- right middle cerebral artery. On exami-
zodiazepines can be used for seizures. nation, he is tall, thin, has long fingers,
Dextromethorphan and ketamine can and high arched feet. Past medical his-
be used to inhibit NMDA receptor exci- tory is remarkable of lens dislocation,
tation by glycine. pigeon chest and osteoporosis.
Metabolic
435 14
What single blood test is likely to be develops seizures. He has hyperam-
of diagnostic use in this patient? monemia with respiratory alkalosis
A. Peripheral blood smear and normal anion gap. Eventually,
B. Thyroid-stimulating hormone the baby requires intubation and
C. Homocysteine mechanical ventilation. A urea cycle
D. Branched chain amino acids disorder is suspected.  
E. Lactic acid Which statement below is correct?
A. This is a group of disorders that
vv Correct Answer is: C are all inherited in an X-linked
This is a classic manifestation of homo- fashion
cystinuria. It is an AR disorder; caused by B. Ornithine transcarbamylase defi-
Cystathion ine-13-synthase deficiency ciency is the most common cause
-- > elevation of blood and urine levels of C. Hyperammonemia and organic
homocysteine and methionine. Usually, acidemias are commonly found in
patients are normal at birth; but they patients with these disorders
will have developmental delay and intel- D. Arginase deficiency commonly
lectual disability later. They may have presents in the newborn period
seizures and psychiatric manifestations. E. High protein diet is indicated for
They have a collagen metabolism their treatment
abnormality -- > eye, bones, and the
vascular system -- > Ectopia lentis and vv Correct Answer is: B
lens dislocation. Other clinical mani- Urea cycle disorders are a group of con-
festations include, marfanoid habitus ditions caused by deficiency of enzymes
with tall and thin stature, pectus cari- responsible for urea synthesis and more
natum, pes cavus, genu valgum, and importantly ammonia and ammonia-­
osteoporosis. Intimal thickening of the containing compound removal.
blood vessel walls and high incidence of Ornithine transcarbamylase (OTC)
thromboembolism, including strokes. deficiency is the most common and
Elevation of plasma and urine homo- is X-linked. All others are inherited in
cysteine, methionine in the plasma and an autosomal recessive fashion and
CSF can confirm the diagnosis. include carbamyl phosphate synthe-
Majority of patient respond to pyri- tase deficiency, argininosuccinic acid
doxine, so give this first. Betaine, folate synthetase deficiency, argininosuccinic
and vitamin 812 supplements. Low-pro- acid lyase deficiency, and arginase defi-
tein diet, specifically low in methionine ciency. These enzyme defects will lead
with cysteine supplementation. to hyperammonemia, which is the main
cause of the clinical manifestations.
zz Suggested Reading Ammonia induces glutamine accumula-
55 Mowzoon N, Flemming KD. Neurology tion, which leads to astrocyte swelling
board review: an illustrated study guide. and brain edema. Hyperammonemia,
Rochester: Mayo Clinic Scientific; 2007. encephalopathy, and respiratory alkalo-
Print. sis are the triad suggestive of this group
of disorders. These patients have very
?? 23. A mother of a newborn reported high concentrations of ammonia, no
that the baby is lethargic with poor evidence of organic acidemias, normal
feeding. At the hospital he rapidly anion gaps, and normal serum glucose
becomes encephalopathic and level. Amino acid analyses help to
436 Chapter 14 · Metabolic

distinguish specific urea cycle disorders, Which of the following is not true
and enzyme activity can be evaluated regarding this disorder?
in liver biopsy specimens. Clinical mani- A. Galactose-1-phosphate urid-
festations often begin in the newborn yltransferase deficiency is the
period with progressive lethargy, vomit- only defect associated with
ing, hypotonia, and seizures. Higher lev- ­galactosemia
els of ammonia may be associated with B. Galactitol accumulation produces
coma and eventually death. Females cataracts that are reversible
with OTC deficiency, and some patients C. It is autosomal recessive
with partial deficiencies, may have late- D. The treatment is removal of lac-
onset presentations and become symp- tose and ga lactose from the diet
tomatic after large amounts of protein E. Patients with this disorder may
ingestion or inter-current illnesses. develop ataxia and tremor despite
Arginase deficiency does not cause treatment
symptoms in the newborn, whereas
newborn presentation is common in the vv Correct Answer is: A
rest of urea cycle disorders. Galactosemia occurs in newborns and
Treatment includes limitation of nitro- it is an autosomal recessive disorder.
gen intake in the diet and administration There are three enzymatic defects that
of essential amino acids. Calories can be account for galactosemia: galactose-
supplied with carbohydrates and fat. Dur- 1-phosphate uridyltransferase defi-
ing acute episodes, sodium benzoate and ciency, galactokinase deficiency, and
sodium phenylacetic acid are used, and uridine diphosphate galactose 4_ epim-
sometimes dialysis may be required. erase deficiency.
Mannitol has been used for brain Galactose-1-phosphate uridyltrans-
edema and increased intracranial pres- ferase deficiency causes classic galac-
sure. Long-­term treatment with low- tosemia and is the only type associated
protein diet and essential amino acids, with intellectual disability. Patients
as well as arginine supplementation present in the first days of life with feed-
(except in arginase deficiency), may sta- ing difficulties, vomiting, diarrhea, and
bilize the neurologic deterioration. jaundice.
Hepatomegaly, failure to thrive, leth-
14 zz Suggested Reading argy, and hypotonia are also a clinical
55 Comprehensive review in clinical neurol- manifestation. Cataracts also occur and
ogy: a multiple-choice question book for are caused by an accumulation of galac-
the wards and boards ©2011 Wolters titol. Developmental delay, cognitive
Kluwer Health Lippincott Williams & impairment, ataxia, and tremor are late
Wilkins. All rights reserved. neurologic sequelae. B rain MRI shows
white matter changes and cortical and
?? 24. You are asked to see a 5 -day-old cerebellar atrophy. Prenatal ­diagnosis of
baby boy at the hospital because galactosemia is available, and newborn
of persistent vomiting and diar- screening is available for this condition,
rhea, predominantly after being thus allowing for treatment prior to
fed. He has jaundice, hypotonia and the onset of symptoms. The diagnosis
hepatosplenomegaly on examination. can be presumed in patients present-
Reducing substances are detected in ing with the clinical manifestations
the urine. A presumptive diagnosis of described, and the detection of reduc-
galactosemia is made.   ing substances in the urine, especially
Metabolic
437 14
after feeding. The enzymatic defect can The gene is localized to chromosome
be detected in plasma and/or erythro- 4, and there is accumulation of both
cytes. Lactose and galactose should be dermatan and heparan sulfate. These
immediately restricted from the diet. patients are normal at birth, but within
This intervention may reverse cataracts the first 2 years of life, they will develop
and hepatomegaly and may prevent coarsening of facial features, with pro-
progression of neurologic disease. gressive skeletal dysplasia with dysos-
However, despite this intervention, tosis multiplex and growth impairment
these patients may develop long-term (dwarfism). There is a restricted range
neurologic sequelae, including learning of motion of the joints, hearing loss,
disability, cognitive impairment, ataxia, corneal clouding, macroglossia, hernias,
and tremor. visceromegaly, valvular heart disease,
and prominent intellectual disability.
?? 25. Which of the following is not true Scheie syndrome is a mild MPS
regarding mucopolysaccharidoses type I, and the intermediate form is
(MPS)? known as Hurler-Scheie syndrome.
A. There is accumulation of glu- Most recently, MPS type I has been
cosaminog lycans, which are classified as severe MPS type I and
detected in urine attenuated MPS type I. The diagnosis of
B. Hurler syndrome is caused by MPS type I is based on elevated urinary
A-L-iduronidase deficiency excretion of dermatan and heparan
C. Sanfilippo syndrome is a group of sulfate and confirmed with enzyme
MPS with accumulation of hepa- analysis in leukocytes and fibroblasts.
ran but not dermatan sulfate Pathologically, there are cells with
D. Morquio syndrome manifests with vacuolated appearance, expansion of
prominent intellectual disability perivascular spaces in the CNS, and
E. Zebra bodies can be seen on elec- neuronal lipidosis. Electron microscopy
tron microscopy demonstrates reticulogranular material
in epithelial and mesenchymal cells, and
vv Correct Answer is: D lamellar material in neurons, some of
There is no intellectual disability in which adopt a layered appearance and
patients with Morquio syndrome. Muco- are called zebra bodies. Enzyme replace-
polysaccharidoses (MPS) are caused ment therapy can be used to treat non-
by impaired lysosomal degradation central nervous system manifestations
of glucosaminoglycans, which are of the disease. Stem cell transplantation
long unbranched molecules of repeat- can be potentially helpful.
ing disaccharides. Various enzymatic Hunter syndrome or MPS type II is
defects lead to the accumulation of glu- caused by a defect in iduronate sulfa-
cosaminoglycans in lysosomes and the tase, with accumulation of dermatan
extracellular matrix. All MPS are auto- sulfate and heparan sulfate. These
somal recessive except Hunter disease, patients have the Hurler phenotype but
which is X-linked. In general, MPS are a lack the corneal clouding and have char-
group of progressive multisystem dis- acteristic nodular i­ vory-­colored lesions
orders that affect the cornea, cartilage, on the back, shoulders, and upper
bone, connective tissue, reticuloendo- arms. These children have short stature,
thelial system, and nervous system. macrocephaly, macroglossia, hoarse
Hurler syndrome is type I MPS and voice, hearing loss, visceromegaly, dys-
is caused by A-L-iduronidase deficiency. ostosis multiplex, joint contractures,
438 Chapter 14 · Metabolic

entrapment neuropathies such as carpal the number of repeats. The number of


tunnel syndrome, and intellectual dis- repeats is unstable and increases with
ability. The diagnosis is based on muco- each subsequent generation a phenom-
polysacchari duria and confirmed with enon known as a dynamic mutation.
analysis of the enzyme activity. Treat- Encephalitis is an infectious disorder.
ment is symptomatic.
Other MPS include types Ill, IV, VI, zz Suggested Reading
VII, and IX as follows: - MPS type Ill or 55 Sadock BJ, Sadock VA, Kaplan HI.
Sanfilippo syndrome has several sub- Kaplan & Sadock’s synopsis of psychiatry:
types (A, B, C, and D) and is associated behavioral sciences/clinical psychiatry.
with accumulation of heparan sulfate Philadelphia: Lippincott Williams &
only. The main manifestation is intellec- Wilkins; 2013.
tual disability. - MPS type IV or Morquio
syndrome has two subtypes (A and B), ?? 27. You are evaluating a 14- year-old boy
and they manifest with corneal cloud- with a history of severe generalized
ing, dysostosis multiplex, and heart myoclonus, ataxia, and loss of color
disease, with normal intelligence. - MPS vision. On examination, he has a
type VI is also known as Maroteaux- cherry red spots. His EEG showed slow
Lamy syndrome and manifests with background, and photosensitive bilat-
normal intelligence, dysostosis multi- eral fast spike and wave activity.  
plex, corneal clouding, heart disease, What finding would establish the
and other features similar to Hurler diagnosis?
syndrome. - MPS type VII is also known A. An alpha-N-acetylgalactosamini-
as Sly syndrome, and these patients dase deficiency
have hydrops fetalis, intellectual dis- B. A decrease in cultured skin fibro-
ability, dysostosis multiplex, corneal blast alpha-neuraminidase
clouding, and other features of Hurler C. A response to oral biotin
syndrome. D. A hexoaminidase A deficiency
E. A muscle biopsy with ragged red
?? 26. Which of the following conditions has fibers
mutation in gene ATP7B?
A. Wilson’s disease vv Correct Answer is: B
14 B. Hemochromatosis Schindler disease is caused by an alpha-­
C. Parkinson’s disease N-­acetylgalactosaminidase deficiency.
D. Huntington’s disease It is a rare autosomal recessive disorder
E. All of the above with acute infant and chronic adult-
onset forms. The main manifestations
vv Correct Answer is: A in infants are severe psychomotor dete-
Wilson’s disease has a common gene rioration with spasticity, cortical blind-
defect in ATP7B. Hemochromatosis has ness, and myoclonic epilepsy with an
mutations in HFE gene. Parkinsonism exaggerated startle response. The adult
and Huntington’s disease are neuro- form is characterized by mild cognitive
degenerative disorders. Huntington’s deficits.
disease is autosomal dominant and is Sialidosis type 1A occurs in ado-
a trinucleotide repeat disease whose lescent and is characterized by severe
severity and onset is determined by the generalized myoclonus, night blindness,
number of CAG repeats. The disease and loss of color vision, ataxia, cherry-
has variable penetrance depending on red spots, and generalized convulsions.
Metabolic
439 14
The EEG usually shows progressive D. Coarse facial features are almost
slowing and bilateral, photosensitive never seen in sialidosis type II
fast spike-wave discharges. A decrease E. These patients have myoclonic
in cultured skin fibroblast alpha-­ epilepsy
neuraminidase is diagnostic.
Biotinidase deficiency responses to vv Correct Answer is: E
oral biotin. Its clinical manifestations Sialidosis is one of the glycoproteinoses.
include hypotonia, generalized tonic- Myoclonic epilepsy is one of its features.
clonic and myoclonic seizures, rash, and Glycoproteinoses are a lysosomal stor-
alopecia, and occurs in infancy. The EEG age disorders, and they are inheritance
demonstrates multifocal spikes and as autosomal recessive disorders, in
slow waves. which the enzymatic defect leads to
Tay-Sachs disease is secondary to a accumulation of oligosaccharides,
hexoaminidase A deficiency, with the glycopeptides, and glycolipids. Vacuol-
onset of development delay, cherry-red ization of multiple cell types occur sec-
spots, and “startle seizures” in infancy. ondary to accumulation in the brain and
A muscle biopsy with ragged red viscera. There are multiple phenotypes
fibers is indicative of myoclonic epilepsy depending on the enzyme affected.
with ragged red fibers (MERRF), which Usually, patients have coarse facial
is a mitochondrial disorder. The clinical features, skeletal abnormalities, and
features include myoclonus, epilepsy, psychomotor retardation.
myopathy, intellectual disability, and Sialidosis is caused by deficiency of
cerebellar dysfunction. Patients may lysosomal A-neuraminidase (sialidase),
also have short stature, deafness, optic leading to increased urinary excretion of
atrophy, cutaneous lipomas, and neu- sialic acid-­containing oligosaccharides.
ropathy. The onset is typically in child- Type I sialidosis (cherry red spot myoc-
hood or early adulthood. lonus syndrome) occurs in adolescence
to adulthood and manifests with myo-
zz Suggested Reading clonic epilepsy, visual deterioration, and
55 Varma A, Moraes CT. Mitochondrial cherry-red spots without dysmorphism.
disorders. In: Bradley WG, Daroff RB, Type II occurs in childhood, and these
Fenichel GM, Jankovic J, editors. patients have not only myoclonic epi-
Neurology in clinical practice, 4th ed. lepsy and cherry-red spots in the retina
Elsevier, Inc.; 2004. p. 781–808. but also severe neurologic abnormali-
55 Pastores GM, Kolodny EH. Inborn errors ties, coarse facial features, severe dysos-
of metabolism of the neNous system. In: tosis, and psychomotor retardation.
Bradley WG, Daroff RB, Fenichel GM, The neonatal form is character-
Jankovic J, editors. Neurology in clinical ized by hydrops fetalis, nephrotic
practice. 4th ed. Elsevier, Inc.; 2004. syndrome, and early death. Other
p. 1811–32. glycoproteinoses include A-mannosi-
dosis caused by A-­mannosidosidase
?? 28. Which of the following is correct deficiency, B-­mannosidosis caused by
regarding sialidosis? B-­mannosidosidase deficiency, fucosi-
A. Sialidosis type I has infantile dosis caused by fucosidase deficiency,
onset aspartylglucosami nuria caused by
B. Sialidosis type II has adult onset aspartylglucosaminidase deficiency,
C. A cherry-red spot is rarely seen in and Schindler disease caused by A-N­
this condition acetylgalactosaminidase deficiency.
440 Chapter 14 · Metabolic

Abnormal urinary excretion of oli- 55 Type 2 has its onset before the age of
gosaccharides and glycopeptides, as 2 years, with psychomotor involve-
well as vacuolated lymphocytes with ment, spasticity, choreoathetosis,
membrane-­bound vacuoles, are pres- oculomotor abnormalities, and
ent. Analysis of enzyme activity can give progresses to death by 2–4 years of
you the definitive diagnosis. age. These patients may also have
hepatosplenomegaly, hydrops fetalis,
?? 29. You are evaluating an 11-year-old boy and cutaneous changes.
for his developmental delay and psy- 55 Type 3 begins after the age of 2 years
chomotor retardation. You examined and progresses slowly, with hepato-
him and noticed, there is limited hori- splenomegaly, psychomotor deteriora-
zontal gaze, ataxia, and generalized tion, spasticity, ataxia, and oculomotor
spasticity. On general examination, he involvement.
has hepatosplenomegaly. Enzymatic
activity of glucocerebrosidase in leu- The diagnosis can be made with an
kocytes is depressed.   analysis of the enzyme B-glucosylce-
What is correct regarding this ramidase or glucocerebrosidase in leu-
­disorder? kocytes. This enzyme can also be tested
A. There is accumulation of lyso- for in amniocytes and through chorionic
somal glucocerebrosides villous sampling, allowing for prenatal
B. Patients with type 1 of this disease diagnosis.
have severe neurologic impair- Gaucher cells are caused by the
ment by age 2 lysosomal storage of glucocerebroside
C. Type 3 has its onset before age 2 in macrophages. These cells are found in
and progresses rapidly to death the liver, spleen, lymph nodes, and bone
between age 2 and 4 marrow, and have large a cytoplasm
D. Enzyme replacement therapy is with striated appearance, what has
not available for this condition been likened to “wrinkled tissue paper.”
E. It is more prevalent in Asian popu- In the CNS, the brain stem and deep
lations than in Ashkenazi Jews nuclei are most severely affected, and
neuronal degeneration is seen, likely
vv Correct Answer is: A from neurotoxic action of glucosyl-
14 This is a clinical presentation of Gaucher sphingosine.
disease. It is an autosomal recessive Enzyme replacement with imiglu-
disorder, and is caused by deficiency of cerase can be effective for liver and
the enzyme glucocerebrosidase (acid spleen involvement and for the hemato-
8-­glucosylceramidase) leading to lyso- logic abnormalities.
somal accumulation of glucocerebro- Type 3 may benefit from bone mar-
sides (glucosylceramide). It is caused by row transplantation.
mutations in the gene GBA on chromo-
some 1q21 and is more common in Ash- ?? 30. You were asked to see a 15-year-
kenazi Jews. old girl in the ED who came in with
There are three phenotypes: severe headache and vomiting that
55 Type 1 is the most common and does was consistent with migraine. She
not involve the CNS. It is characterized was treated with IV fluids, ketorolac
by hepatosplenomegaly with anemia and diphenhydramine with no relief.
and thrombocytopenia, skeletal She was admitted to the hospital and
involvement, and pulmonary i­ nfiltrates. was given intravenous valproic acid
Metabolic
441 14
and her headache completely has acid sensitivity. In males with OTC
been subsided. Later that evening deficiency, sodium valproate may pre-
she felt better and drank two protein cipitate acute liver failure (Tripp et al.
shakes. The following morning, the 1981). Hjelm et al. (1986) concluded
nurse could not arouse her. All tests that the vulnerability of toxic effects of
including a toxicology screen and MRI valproate extends to heterozygotes as
which were negative. An ammonia well. They described a family in which 2
level was elevated at 250 mmol/l. The daughters and a son died in childhood,
valproic acid was stopped. She was all with clinical features suggesting a
treated with intravenous fluids and metabolic disorder; in one, valproate
the ammonia level fell to 50 mmol/l. seemed to have accelerated death.
She was discharged home. One They concluded that the mother was
month later, she came back with a heterozygote for OTC deficiency.
severe headache, vomiting and con- Honeycutt et al. (1992) reported a
fusion. Her mother mentioned that previously undiagnosed heterozygous
she could not eat for a couple of days woman who had symptomatic hyper-
due to headaches so she substituted ammonemia during initiation of valpro-
meals with several protein shakes. ate therapy.
The mother reported that she often
drank protein shakes and had sub- zz Suggested Reading
sequent irritability but she thought 55 Ornithine Transcarbamylase Deficiency.
that was due to the headaches. On OMIM Entry # 31125.
examination she was lethargic and
had myoclonic jerks. An ammonia ?? 31. A 16-year-old boy presents to your
concentration was ordered and was office with facial dystonia and dysar-
400 mmol/l. Repeat ammonia was thria. On ophthalmologic exam, you
450 mmol/l.   have seen picture show below.  
What is the most likely diagnosis is?
A. Hyperammonemia due to Valproic
acid
B. Ornithine transcarbamylase
­deficiency
C. Lafora disease
D. Status epilepticus
E. Viral Encephalitis

vv Correct Answer is: B


The patient is heterozygous for Orni-
thine Transcarbamylase (OTC) Defi-
ciency. Female heterozygotes can
..      Fig. 14.4 Kayser-Fleischer ring: copper deposition
present as early as the first week of life
in Descemet’s membrane of the cornea. These rings
or as late as 30 years of age. Symptoms can be either dark brown, golden, or reddish-green,
before diagnosis can be nonspecific: are 1–3 mm wide, and appear at the corneal limbus.
episodic extreme irritability, head- With rare exceptions, they are diagnostic of inherited
aches, episodic vomiting and lethargy, hepatolenticular degeneration—Wilson’s disease. This
32-year-old patient complained of longstanding
ataxia, stage II coma, delayed growth,
difficulty speaking. He also had a tremor. (7 http://
developmental delay, and seizure. cnx.org/content/m15007/latest/. Herbert L. Fred, MD,
There have been cases of valproic Hendrik A. van Dijk. Wikipedia)
442 Chapter 14 · Metabolic

What is your first choice for treat- in this condition, though several other
ment of this condition? agents, such as dimercaprol, may be
A. Deferoxamine used. Deferoxamine chelates iron. Ede-
B. Hemodialysis tate salts chelate heavy metals, but is
C. Lactulose not used for copper chelation. Lactu-
D. D-penicillamine lose helps with clearance of ammo-
E. Disodium edetate nia. Hemodialysis is not used for this
­condition.
vv Correct Answer is: D
The picture shows the Kayser-Fleischer zz Suggested Reading
rings which are classic presenting symp- 55 White H. Neurologic manifestations of
toms of Wilson disease, a disorder of acute and chronic liver disease.
copper metabolism. D-penicillamine, a Continuum. 2014;20(3 Neurology of
chelation therapy is first choice therapy Systemic Disease):670–80.

14
443 15

Movement Disorders

© Springer International Publishing AG, part of Springer Nature 2018


Y. M. Awaad, Absolute Pediatric Neurology, https://doi.org/10.1007/978-3-319-78801-2_15
444 Chapter 15 · Movement Disorders

?? 1. A 12-year-old female came for an Neuropharmacology. 2013;68:143–9. Epub


evaluation for her complex motor 20 Jun 2012.
tics including excessive eye blinking, 55 Singer HS. Treatment of tics and Tourette
facial movements and neck stretch- syndrome. Curr Treat Options Neurol.
ing. She has daily tics which are 2010;12:539–61.
annoying to her, but no social embar-
rassment for her and her family. She ?? 2. The diagnosis of benign paroxysmal
is an excellent student, socially well- torticollis of childhood includes all of
adjusted, and has no obsessive-com- the following except:
pulsive, anxiety or ADHD symptoms. A. Typical age of onset is 5–7 years of
What is your first-­line treatment age.
options? B. Episodic attacks of head tilting
A. Atomoxetine 40 mg daily lasting minutes to days
B. Risperdone 1 mg daily C. Attacks remit spontaneously and
C. Comprehensive Behavioral Inter- tend to reoccur monthly
vention for Tics (CBIT) D. Associated signs and symptoms
D. Clonidine 0.1 mg tid may include ataxia, pallor, irritabil-
E. Neurofeedback ity and vomiting

vv Correct Answer is: C vv Correct Answer is: A


Treatment should be individualized in Benign paroxysmal torticollis is an epi-
tics and Tourette syndrome patients. CBIT sodic torticollis syndrome, classically
or other behavioral interventions and starts before age 12 months and resolving
treatments should be considered before by age 3–5 years of age. Family history of
pharmacological treatments especially migraine is very common. Episodes can
in cooperative children. CBIT is a “pack- last hours to days, typically associated
age” of approaches with “habit reversal” with pallor and occasionally vomiting.
techniques as a major component. If
unsuccessful, and if tics are causing zz Suggested Reading
loss of self-esteem, social dysfunction, 55 Rosman NP, Douglass LM, Sharif UM,
provoking anxiety, limiting participation Paolini J. The neurology of benign
in activities, physical discomfort, school paroxysmal torticollis of infancy: report of
issues or other functional difficulties, 10 new cases and review of the literature. J
adjunctive pharmacological therapies can Child Neurol. 2009;24(2):155–60.
15 be considered. Medications used for tic-
suppression include alpha-2 adrenergic ?? 3. A 6-year-old boy came with his fam-
agonists as first-­line consideration, and ily with complain of in-­toeing of his
typical and atypical neuroleptics as sec- left foot with walking. His dystonic
ond-tier medications. Co-­morbid ADHD, posturing generalizes to involve his
obsessive-compulsive disorder, anxiety or left arm, and the right side of his body
depression should be treated with specific as well over the following year. He has
pharmacological therapies. no cognitive problems, and he has
had no seizures. There is no change
zz Suggested Reading in his dystonia during the day. When
55 Roessner V, Schoenefeld K, Buse J, et al. you examined him, he has a general-
Pharmacological treatment of tic disorders ized dytsonia involving all extremities.
and Tourette syndrome. Brain MRI is normal.
Movement Disorders
445 15
Which gene is involved in this zz Suggested Reading
patient genetic disease? 55 Please see 7 http://www.ncbi.nlm.nih.gov/
A. Epsilon-sarcoglycan (SCGE) sites/GeneTests/?db=GeneTests for up to
B. GTP cyclohydrolase 1 (GTPCH 1) date reviews on these topics.
C. Pantothenate kinase (PANK2)
D. Thyroid transcription factor 1 ?? 4. A 10-year-old boy with the history
(TITF1) of frequent eye blinking for the last
E. Torsion A (DYT1) 6 months. The eye blinking is bother-
ing him, his teachers and family. He
vv Correct Answer is: E has a normal physical examination
Torsion dystonia (DYT1 dystonia) or with the exception of frequent eye
also referred as dystonia musulorum blinking and facial grimacing.
deformans, usually presents with dys- What is your the most likely
tonic posturing of a foot, leg, or arm. It ­diagnosis?
appears first with specific actions; e.g., A. Chronic Vocal Tic Disorder
writing or walking. Finally, the contrac- B. Chronic Motor Tic Disorder
tions frequently become evident with C. Tourette’s Disorder
less specific actions and spread to other D. PANDAS
body regions. E. Transient Tic Disorder
Myoclonus-dystonia (M-D) is a
movement disorder characterized by a vv Correct Answer is: E
combination of myoclonus and dystonia. Tic disorders occur in nearly 1% of
Depression, anxiety, and obsessive-com- children. Tics usually start between
pulsive disorder (OCD) are common 7–12 years of age. Motor tics typically
co-morbid features. It is secondary to appear first, followed by the onset of
mutations in the epsilon-­sarcoglycan vocal tics in some individuals. “Transient
gene. Tic Disorder” should be considered if tics
GTP cyclohydrolase 1-deficient are longer than 4 weeks, but less than a
dopa-­responsive dystonia (GTPCH1) is year. On the other hand, Tourette’s will
usually respond to low doses of levodopa. have multiple motor and at least one
The typical presentation is a gait distur- vocal tic for more than 1 year in dura-
bance caused by foot dystonia, later tion. Chronic motor or Chronic Vocal tic
development of Parkinsonism, and diurnal disorder may be used to define other
fluctuation of symptoms. children. (DSM-­IV TR for the tic disorders
Pantothenate kinase-associated DSM-IV-TR Criteria 307.23 (Tourette’s
neurodegeneration (PKAN) is a neurode- Disorder). Both multiple motor and one
generative disease characterized by brain or more vocal tics have been present at
iron accumulation and was formerly called some time during the illness, although
Hallervorden-­Spatz syndrome. PKAN is not necessarily concurrently. (A tic is a
characterized by progressive dystonia and sudden, rapid, recurrent, non-rhythmic,
basal ganglia iron deposition that results stereotyped motor movement or vocal-
in an “eye of the tiger” sign on MRI. ization). The tics occur multiple times a
Benign hereditary chorea is an day, (usually in bouts,) nearly every day
autosomal dominant disorder with non- or intermittently throughout a period of
progressive chorea. It is a result of more than 1 year, and during this period
mutations in the thyroid transcription there was never a tic-free period of more
factor (TITF-1) gene. than 3 consecutive months. The onset is
446 Chapter 15 · Movement Disorders

before age 18 years. The disturbance is What tests are you going to order at
not due to the direct physiological effects this time?
of a substance (e.g. stimulants) or a gen- A. Erythrocyte sedimentation rate
eral medical condition (e.g. Huntington’s B. Anti-streptolysin antibody
disease or post viral encephalitis) DSM- C. Cholesterol and triglyceride panels
IV-TR Criteria 307.22 (Chronic Motor or D. Anti-phospholipid antibody
Vocal Tic Disorder). Single or multiple E. Liver function tests
motor or vocal tics (i.e. sudden, rapid,
recurrent, non-rhythmic, stereotyped vv Correct Answer is: B
motor movements or vocalizations), but Sydenham chorea is one of the complica-
not both, have been present at some tions of acute rheumatic fever which has
time during the illness. Criteria have to be considered in any child presenting
never been met for Tourette’s disorder with a movement disorder, especially
DSM-­IV-­TR Criteria 307.21 (Transient Tic due to the long-term treatment require-
Disorder). Single or multiple motor and/ ments. It is a complication of untreated
or vocal tics (i.e. sudden, rapid, recurrent, streptococcal infections. The onset can
non- rhythmic, stereotyped motor move- be insidious, and therefore the diagnosis
ments or vocalizations). Criteria have can be delayed. The cardinal features are
never been met for Tourette’s disorder chorea, emotional lability, and hypotonia;
or chronic motor or vocal tic disorder trouble at school may be the reason for
Diagnostic and Statistical Manual of seeking medical attention. In addition to
Mental Disorders. 4th ed., Text Revision treating the chorea, these patients should
DSM-IV-TR. Washington, DC: American have treatment of the active streptococcal
Psychiatric Association; 2000. Walkup infection for 10 days and then prophylac-
JT, Ferrão Y, Leckman JF, Stein DJ, Singer tic penicillin therapy until age 21.
H. Tic disorders: some key issues for
DSM-V. Depress Anxiety. 2010 Jun; zz Suggested Reading
27(6):600–10. 55 Fenichel GM (ed.). A Signs and Symptoms
Approach. In: Clinical Pediatric
?? 5. A 12-year-old girl came to your office Neurology. 2nd ed. Philiadelphia:
for an evaluation for her repetitive W.B. Saunders Company; 1993.
jerky movements of her hands and
arms that have appeared over the ?? 6. At 6 years of age, a boy was diagnosed
last 2 weeks. She cannot control with ADHD, but no medication was
15 them and they are interrupting her prescribed. He started having frequent
handwriting. Her school teacher com- twitching of his face and eye blinking
plained of her behavioral outbursts behavior at 7 years of age. By 8 years of
in class which is not in character. age, he was disruptive in class by mak-
On examination, she is thin, and no ing frequent whistling noises.
dysmorphic features. She has a soft Which of the following is the most
systolic murmur noted anteriorly, and likely additional symptom that this
has no hepatosplenomegaly noted. patient may experience?
She has moderate facial acne, and an A. Myoclonus
elevated body mass index of 35. She B. Dystonia
was diagnosed with attention deficit C. Learning disorder
disorder, but she is not taking any D. Coprolalia
medications. E. Wing beating tremor
Movement Disorders
447 15
vv Correct Answer is: C An abnormal/positive result on
This is a classic presentation for Tourette’s which of the following tests would
Disorder, which is defined as having alter the patient’s treatment?
frequent multiple motor tics and at least A. Anti-DNase B
one verbal tic for 1 year in duration. The B. C-reactive protein
initial presentation can include ADD or C. Toxicology screen
ADHD, which often is present before the D. Echocardiogram
onset of the tics. Anxiety, depression, E. ASO titer
obsessions, compulsions, and learn-
ing disabilities are common. Coprolalia vv Correct Answer is: D
or copropraxia are uncommon, but 10% to 20% of patients with acute rheu-
may occur. The presence of additional matic fever (ARF) will have Sydenham’s
movement disorders, such as dystonia, chorea, usually weeks to months after
a wing-beating tremor, or myoclonus a group A β-hemolytic streptococcal
should alert the clinician to additional (GABHS) infection. The classical presen-
potential diagnoses including myoclonus tations include chorea, hypotonia, and
dystonia, Wilson’s disease, or neuro-­ emotional lability. The mechanism of
acanthocythosis. the disease is not fully understood, the
pathological explanation is that molecu-
zz Suggested Reading lar mimicry between streptococcal and
55 Jankovic J. Tourette’s syndrome. N Engl J basal ganglia peptides leads to immune-­
Med. 2001;345(16):1184–92. modulated damage of the basal ganglia.
The diagnosis is based on elevated anti-
?? 7. A 14-year-old female has one-month streptolysin O and/or anti-DNase B titers
history of rapidly worsening invol- may corroborate the diagnosis. Negative
untary movements causing her to titers do not exclude the diagnosis. Throat
be unable to write and walk. She culture to determine whether the patient
had slurred speech 1 week before has an active GABHS infection should be
presentation. In spite of she is doing done; if positive, the patient should be
well at school, teachers reported that treated with antibiotics. Other labora-
she is crying more easily than usual. tory and imaging studies may be neces-
She has unremarkable past medical sary to exclude other causes of chorea,
and birth histories. She is not taking such as drug reactions, systemic lupus
any medications. She has a normal erythematosus, and Wilson’s disease.
general exam. Neurologic examina- Echocardiogram has to be done on all
tion shows, severe dysarthria and patients to assess for carditis and should
motor impersistence with tongue be started on long-term antibiotic prophy-
protrusion. She has mild, diffuse laxis to prevent rheumatic recurrences.
hypotonia and marked generalized If chorea is disabling, treatment options
chorea, which causes her to bump include valproate, carbamazepine, and/or
into things while ambulating. DTRs dopamine receptor antagonists.
are 2+ but are hung-up at the knees.
Her routine blood work, including ?? 8. An 8-year-old female with one-year
a basic metabolic panel, complete history of eye rolling, facial grimacing,
blood count, sedimentation rate, and and head jerking that occur multiple
pregnancy test, all of which were times per day. She has a tingling sen-
normal/negative. sation in her neck that is temporarily
448 Chapter 15 · Movement Disorders

relieved by jerking her head. She α-adrenergic drugs (clonidine, guanfa-


reported that she can stop the move- cine). Although, haloperidol, pimozide,
ments if she wants for several seconds and risperidone can be used for patients
at a time. She has been on loratadine who have failed an α-adrenergic agent,
10 mg/day for the last 2 years for her they have significant side effects, includ-
repetitive sniffing and throat clearing, ing extrapyramidal symptoms, weight
and ibuprofen 400 mg as needed for gain, and electrocardiographic changes.
neck pain related to her head jerking
movements. Sniffing has stopped zz Suggested Reading
over the past year but she has started 55 Swain JE, Scahill L, Lombroso PJ, King
audibly inhaling. She has no school RA, Leckman JF. Tourette syndrome and
troubles. Her family and teachers tic disorders: a decade of progress. J Am
have no concerns about her mood or Acad Child Adolesc Psychiatry.
behavior. She has a family history of 2007;46(8):947–68.
anxiety in her mother and ADHD in
her brother. Her general and neuro- ?? 9. An 8-year-old boy is presented to your
logic examinations are normal, with office with a gradual worsening of his
the exception of the aforementioned gait for the past 2 years. His family
adventitious movements. reports a tendency for his right foot to
What is your choice as a fi­ rst-line turn down and in. He has no problem
treatment for her abnormal in the morning, but his foot begins to
­movement? turn by late afternoon each day, even
A. Guanfacine if he has not been physically active.
B. Sertraline He has no significant past medical his-
C. Pimozide tory, and he is doing fine at school. No
D. Risperidone medications were prescribed. His fam-
E. Haloperidol ily history is remarkable of a father
has similar but less severe symptoms
vv Correct Answer is: A which began in his early 20’s. He has
Tourette syndrome (TS) affects an a normal general and neurological
estimated 0.1 to 3% of the population. examination except his tone and ten-
Diagnosis of Tourette syndrome is based don reflexes are increased in the distal
on, patient will have multiple motor right lower extremity. He has a flexor
and one or more vocal tics, which occur plantar response. Sensation is intact.
15 many times a day nearly every day for What test are you going to order to
more than 1 year, though not necessarily reach the diagnosis?
concurrently. Onset must be prior to age A. MRI brain
18 years, and the tics cannot be due to the B. Genetic study for DYT1
effects of a substance or general medical C. CSF neurotransmitters
condition. Obsessive-compulsive disorder D. Ceruloplasmin
(OCD), attention deficit hyperactivity E. Genetic study for hereditary spas-
disorder (ADHD), anxiety, depression, tic paraplegia
learning disorders, and/or oppositional
behavior/conduct disorder are very vv Correct Answer is: C
common co-morbidity. You do not have Segawa disease (Autosomal dominant
to treat it unless the tics are causing GTP cyclohydrolase deficiency) is a
morbidity. In this case, pharmacotherapy dopa-­responsive dystonia caused by
should be initiated with low doses of a heterozygous mutation of the GCH1
Movement Disorders
449 15
gene on chromosome 14q22.1–2. The bilateral putamen and caudate nuclei
term “hereditary progressive dystonia on T2 weighted images. He has a large
with marked diurnal fluctuation” is not head size since early infancy. Family
in use any more. Onset is usually around history is unremarkable. On examina-
6 years of age and starts with postural tion, he has severe diffuse rigidity, dys-
dystonia of a lower extremity, mostly tonia, and tongue thrusting.
with talipes equinovarus. Action dystonia Which of the following is the most
(especially retrocollis or oculogyric crisis) likely diagnosis?
can occur in some patients. Postural high- A. Glutaric acidemia type 1
frequency (8–10 Hz) tremor develops in B. MELAS
an upper extremity at around 10 years C. Wilson’s disease
of age. Usually, it starts asymmetric with D. Carbon monoxide poisoning
marked diurnal fluctuation (aggravation E. Methylmalonic aciduria
towards the evening and near complete
if not complete recovery after overnight vv Correct Answer is: A
sleep). Growth decelerates such that Glutaric acidemia type 1 is an autosomal
patients are 2 standard deviations below recessive inborn error of lysine, hydroxyli-
age norms for height by the late teens. sine, and tryptophan metabolism caused
Eventually, the dystonia becomes more by deficiency of glutaryl-CoA dehydroge-
generalized and tremor becomes more nase. This enzyme is located in the mito-
pronounced and constant rather than chondrial matrix. More than 60 mutations
fluctuating. Anticipation occurs in some of the gene encoding this enzyme have
families with a wide variation in age of been identified, including missense, non-
onset. Neuroimaging is usually normal. sense, and intronic variants. There is a cor-
Low levels of homovanillic acid, biopterin, relation between residual enzyme activity
and neopterin can be detected on CSF and biochemical phenotype, but not
examination. It is recommended to start between phenotype and severity of the
patients on levodopa/carbidopa genetic lesion, as siblings within the same
4–5 mg/kg/day to alleviate the symptoms. family and with the same mutation have
discordant phenotypes. Fever, infections,
zz Suggested Reading and fasting play can precipitate neuronal
55 Segawa M. Chang Gung Med J. 2009; damage. It is rare (worldwide incidence
32:1–11. 1:100,000) except in certain populations
such as the Old-Order Amish community
?? 10. A 10-month-old boy, 3 months ago he of Pennsylvania and North American
was acutely sick then he developed Ojibway-Cree in Canada.
severe dystonia and rigidity which Patients have macrocephaly at birth or
worsen over time. He was admitted shortly thereafter, with atrophy of the
to the PICU after he had a fever and frontoparietal cortical areas creating a
vomiting accompanied by hypogly- “bat-wing” appearance (widening of the
cemia and metabolic acidosis and he Sylvian fissures), creating a “micrence-
was treated with IV fluids. During his phalic macrocephaly”. Subdural hemor-
PICU admission he became acutely rhages due to stretching of veins from
hypotonic and lost head control and large CSF extra-axial collections can occur
had seizures. An EEG was normal. Brain in those patients. Bouts of acute decom-
MRI showed atrophy of the frontopari- pensation usually occur between 6 and
etal areas with widening of the Sylvian 18 months and if not present by 5 years of
fissures and increased signal of the age are unlikely to occur. These bouts are
450 Chapter 15 · Movement Disorders

associated with edema of the putamen self-limited. Chronic treatment with neu-
and caudate nuclei and present clinically roleptics can also cause tardive chorea.
with hypotonia and loss of head control. Anticonvulsants ­(particularly phenytoin),
Seizure-like episodes are common, but oral contraceptives, and stimulants can
EEG does not show epileptiform features. induce chorea.
Hypotonia disappear after few weeks and
is replaced by rigidity and dystonia. zz Suggested Reading
Patients will end up with severe motor 55 Shannon KM. Treatment of chorea.
disability but they have normal cognition. Continuum Lifelong Learning Neurol.
Usually, urine organic acid analysis shows 2007;13(1):72–93.
elevated levels of 3-OH-glutaric acid
which is diagnostic. Urine glutarylcarni- ?? 12. A 10-year-old girl with the diagnosis
tine is also typically elevated (acylcarni- of cerebral palsy came for an abnor-
tine urine testing). Early detection by mal gait, her symptoms are signifi-
newborn screen helps to prevent the cantly worse at the end of the day
acute decompensation episodes by use of compared to the morning. She has
a special diet with protein restriction, hyperreflexia and equinovarus postur-
supplemented with lysine-free special ing of the feet on her examination.
formula and carnitine. What is your first step in
­management?
zz Suggested Reading A. Implantation of an intrathecal
55 Hedlund GL, Longo N, Pasquali baclofen pump
M. Glutaric acidemia type 1. Am J Med B. Injections of botulinum toxin into
Genet Part C Semin Med Genet. calf muscles
2006;142C:86–94. C. Oral carbidopa/levodopa
D. Oral tetrabenazine
?? 11. A 10-year-old male came to the office E. Referral to a pediatric orthopedic
with his foster mother for evaluation surgeon
of random, rapid, involuntary flowing
movements after sudden stopping vv Correct Answer is: C
his medication he was on after he was This is a clinical presentation of hereditary
removed from his biological parents dopa-responsive dystonia; the diurnal
due to concerns of neglect. variation is characteristic of this disorder.
Which of the following types of Low-dose levodopa provides a dramatic
15 medication has most likely been dis- benefit that usually is sustained for
continued? years. Recently, this syndrome has been
A. Antibiotic agent renamed as autosomal dominant GTP
B. Antiepiletpic agent cyclohydrolase 1 deficiency or Segawa
C. Bronchodilator agent disease. The reason is a mutation of the
D. Neuroleptic agent GCH-1 gene located on 14q22.1 to q22.2.
E. Nonsteroidal anti-­inflammatory Because of heterozygous mutation, partial
agent deficiency of tetrahydrobiopterin affects
tyrosine hydroxylase (TH) rather selec-
vv Correct Answer is: D tively and causes decrease of TH in the
Sudden discontinuation of neuroleptics terminals of the nigrostriatal dopamine
can cause chorea in children and ado- (NS DA) neurons, projecting to the D1
lescents. This is known as withdrawal- receptors on the striosome, the striatal
emergent syndrome and is generally direct pathways and the sub-thalamic
Movement Disorders
451 15
nucleus (STN) and the D4 receptors of the ?? 13. A 10-year-old boy came with his fam-
tuberoinfundibular tract. ily to your office because his family
The activities of TH in the terminal are suspecting him to have Tourette
high in early childhood decrease expo- syndrome because off his occasional
nentially to the stational level around blinking, sniffing, or grunting. The
early 20s, and show circadian oscillatron. movements do not bother him; even
TH in HPD follows these variations with him and his teachers do not notice
around 20% of normal levels and with them. He does not have any hyperac-
development of the downstream tivity, attentional difficulty, or psychi-
structures show appears characteristic atric dysfunction, and the neurologic
clinical symptoms age dependently. In examination is normal.
late fetus period to early infancy, through What is the most reasonable treat-
the striosome-substantia nigra pars ment at this time?
compacta pathway failure in morphogen- A. Botulinum toxin injections
esis of the DA neurons in substantia B. Clonidine patch
nigra, in childhood around 6 years C. Oral methylphenidate
postural dystonia through the D1 direct D. Oral pimozide
pathways and the descending output of E. Reassurance and education of the
the basal ganglia. Diurnal fluctuation is parents
apparent in childhood but decrease its
grade with age. TH deficiency at the vv Correct Answer is: E
terminal on the STN causes action The child has tics, but they are mild
dystonia from around 8 years and and you do not have to use any phar-
postural tremor from around 10 years, macotherapy. Because the prognosis is
focal dystonia in adulthood. good the family should be reassured.
Adult onset cases in the family with Clondine, or neuroleptics like pimozide,
action dystonia start with writer’s cramp, can be used if the tics were more severe.
torticollis or generalized rigid hypertonus The role of botulinum toxin in treating
with tremor but do not show postural tics is promising, but certainly would
dystonia. TH deficiency on the D4 recep- not be first-line therapy in a mildly
tors causes stagnation of the body length affected patient of this age. If the child
in childhood. With or without action has attention deficit hyperactivity disor-
dystonia depends on the locus of muta- der as a comorbid with the tic disorder
tion. Postural dystonia is inhibitory Stimulants like methylphenidate can be
disorder, while action dystonia is excit- useful; despite a longstanding concern
atory disorder. The TH deficiency at the that stimulants can exacerbate tic, the
terminal does not cause morphological evidence for this association is not very
changes or degenerative process. Thus, strong.
levodopa shows favorable effects without
any relation to the duration of illness. zz Suggested Reading
55 Sanger TD. Tic disorders and Tourette
zz Suggested Reading syndrome in children. Continuum Lifelong
55 Geyer HL, Bressman, SB. Treatment of Learning Neurol. 2007;13(1):139–53.
dystonia. Continuum Lifelong Learning
Neurol. 2007;13(1):94–121. ?? 14. Four months after a 9-year-old boy
55 Segawa M. Hereditary progressive has frequent blinking, grimacing, and
dystonia with marked diurnal fluctuation. grunting, these completely stopped
Brain Dev. 2011;33(3):195–2. completely without any intervention.
452 Chapter 15 · Movement Disorders

What is your diagnosis? treated with antibiotics. On examina-


A. Sydenham’s chorea tion, she has rapid muscle contractions
B. Tourette syndrome flowing randomly from one body part
C. Meige syndrome to another, as well as mild dysarthria,
D. Chronic motor tic disorder hyperactivity, and inattentiveness. Her
E. Transient tic disorder Antistreptolysin O titer is elevated.
What is the most important next
vv Correct Answer is: E investigation?
Tics are stereotyped and repetitive move- A. MRI of the brain
ments or vocalizations that are quite B. Genetic testing
common in childhood with age of onset C. Lumbar puncture
typically between 6 and 12, and peak D. Echocardiogram
intensity often between 10 and 14 years of E. Slit lamp examination
age. If symptoms persisted for more than
1 month, but less than 1 year, a transient tic vv Correct Answer is: D
disorders could be considered in the major- The patient has Sydenham chorea, which
ity of children. If tics are present for more is one of the major Jones criteria for rheu-
than 1 year, it is a chronic tic disorders by matic heart disease. Carditis, another of
definition, and may involve motor or vocal the major Jones criteria, is present in up to
(phonic) tics. On the other hand, Tourette 80% of patients with Sydenham chorea.
syndrome has at least 1 year of multiple
motor tics and at least one phonic tic. zz Suggested Reading
Additional neuropsychiatric disorders are 55 Cardoso F, Silva CE, Mota CC. Sydenham’s
commonly seen with Tourette syndrome, chorea in 50 consecutive patients with
with up to 50% of patients meeting criteria rheumatic fever. Mov Disord.
for an attentional disorder (ADD or ADHD) 1997;12:701–3.
and 15–20% an anxiety or obsessive-­
compulsive disorder. Sydenham’s chorea ?? 16. A 9-year-old girl has gait disturbance
is the movement disorder associated with in the morning but it is worse later
rheumatic fever, secondary to an immune in the day. You decided to exam her
reaction to a streptococcal infection. Meige in the afternoon. On exam, she has
syndrome is a form of oro-facial or cervical reduced facial expression, increased
idiopathic dystonia, commonly affecting tone in the limbs, and hyperreflexia.
patients in their 50–70 years of life. Both When you asked her to walk, there is
15 disorders have no tics as part of their symp- plantar flexion and inversion of the
tomology. ankles.
What medication is likely to dramati-
zz Suggested Reading cally improve her symptoms?
55 Martino D, Mink J. Tic disorders. A. Acetazolamide
Continuum (Minneap Minn). B. Carbidopa/levodopa
2013;19(5):1287–311. 7 pediatrics. C. Mexiletine
aappublications.org/content/121/6/1281. D. Tetrabenazine
full.pdf. E. Zinc

?? 15. A 10-year-old girl with 2 weeks his- vv Correct Answer is: B


tory of involuntary ­movements in Dopa-responsive dystonia is characterized
her limbs. 5 weeks earlier she had by diurnal fluctuation of Parkinsonism
streptococcal pharyngitis and she was and dystonia in a child. Treatment with
Movement Disorders
453 15
low-­dose levodopa is both diagnostic and grimacing and rapid, jerky move-
therapeutic, as it is expected to confer ments in both arms and legs that she
dramatic and sustained improvement. cannot suppress.
Which of the following findings is
zz Suggested Reading most likely to be found on further
55 Geyer HL, Bressman SB. Treatment of evaluation?
dystonia. Continuum Lifelong Learning A. positive urine toxicology screen
Neurol. 2007;13(1):94–121. B. Koplik spots
C. expanded CAG repeats in the
?? 17. 15-year-old boy has Tourette syn- gene coding for the huntingtin
drome for the last 4 years, asking you protein
for a pharmacologic treatment for his D. low serum ceruloplasmin
bothered tics. E. carditis
What medications from below list
can cause tardive dyskinesia? vv Correct Answer is: E
A. guanfacine Sydenham chorea is a complication of
B. clonidine infection with group A beta Hemolytic.
C. pimozide Streptococci that cause pharyngitis
D. methylphenidate few weeks before onset of the neuro-
E. tetrabenazine logic.
Symptoms. Patients usually have
vv Correct Answer is: C emotional or behavioral disturbances.
Pimozide blocks dopamine receptors Chorea is one of the major Jones
in the striatum like other neuroleptic criteria for rheumatic fever. Carditis is
medications and its use can cause tardive another major criteria and can affect as
syndromes. Stimulants like methylphe- many as 80% of patients with Sydenhams
nidate traditionally can worsen tics, chorea. Cardiac Work-up is should be done
although more recent controlled trials in children with Sydenham chorea. This
have not supported this association. patient’s presentation is not typical of
Guanfacine and clonidine are used to Wilson disease (low ceruloplasmin),
treat tics and attention deficit hyperactiv- Huntington disease (expanded CAG
ity disorders but do not cause tardive repeats), measles (Koplik spots) or acute
syndromes. Tetrabenazine is a dopamine intoxication.
depletor rather than a dopamine receptor-­
blocking agent and does not cause tardive zz Suggested Reading
­dyskinesias. 55 Leffler J. Sydenham chorea. Movement
Disorder Virtual University. Accessed 27
zz Suggested Reading June 2012.
55 Kenney C, Hunter C, Mejia N, Jankovic
J. Tetrabenazine in the treatment of ?? 19. A 7-year-old girl is complaining of
Tourette syndrome. J Pediatr Neurol. difficulty walking secondary to right
2007;5:11–6. leg stiffness and involuntary inversion
of her ankle. Her symptoms begin
?? 18. An 8-year-old girl with a history of by late morning and worsen as the
restlessness, irritability and inat- day progresses, although she is nor-
tentiveness in school for the last mal upon waking up in the morning
3 weeks. 5 weeks ago, she had a sore or after a nap. She has generalized
throat for few days. Now she has facial hyperreflexia.
454 Chapter 15 · Movement Disorders

Which of the following medications onset cases who start with writer’s cramp,
is most likely to be effective? torticollis or generalized rigid hypertonus
A. acetazolamide with postural tremor, but do not show
B. baclofen dystonic posture or apparent progres-
C. carbamazepine sion. Symptoms show marked diurnal
D. haloperidol fluctuation in childhood, which becomes
E. levodopa progressively less apparent over time.
In adult onset cases it is not observed.
vv Correct Answer is: E Childhood onset cases show stagnation of
The patient presented with a dopa-­ body length with onset of dystonia. This
responsive dystonia (DRD) which is suc- is not observed in cases with onset after
cessfully treated with low-dose levodopa. adolescence.
Trihexyphenidyl and benztropine
(Anticholinergic medications) can also zz Suggested Reading
be effective but are associated with more 55 Nygaard TG, Marsden CD, Fahn S. Dopa-
side effects. Carbamazepine has been responsive dystonia: long-term treatment
effective in a few patients, but this ben- response and prognosis. Neurology.
efit is mild and inconsistent. Hereditary 1991;41:174–81.
progressive dystonia with marked diurnal 55 Segawa M. Hereditary progressive
fluctuation (HPD) is a dopa-­responsive dystonia with marked diurnal fluctuation.
dystonia described by Segawa et al. in Brain Dev. 2011;33(3):195–201.
1976. After discovery of the causative
gene, the gene of GTP cyclohydrolase ?? 20. A 9-year-old boy has been evaluated
1 (GCH-1) located on 14q22.1 to q22.2, for being off balance. His parents
this is called autosomal dominant GCH-1 reported that his symptoms come
deficiency or Segawa disease. Clinically on in attacks and in between attacks
there are two types: postural dystonia he appears completely normal. In
and action dystonia type. In most cases the office, he had his symptoms for
the symptoms at onset is dystonic pos- a few hours. On examination, he has
ture, with rigidity in one lower extremity, dysarthria, ataxia and nystagmus.
pes equinovarus, around 6 years. Postural His symptoms gradually resolved
dystonia expands to other limbs, and and he complained of a throbbing
all limbs and trunk muscles are affected headache.
by late teens, the rigidity aggravates What statement below is correct?
15 progressively until around 20 years of A. This condition is associated with
age, but the progression subsides in mutation in the voltage-­gated
the 20s and becomes stationary in the potassium channel gene KCNA1.
30s. Postural tremor with high cycles of B. This condition can be associ-
8 Hz to 10 Hz appears after 10 years in ated with progressive cerebellar
an upper extremity and expands to all ­atrophy.
limbs by thirties. However, locomotion C. This condition is associated with
is preserved throughout the course. In mutation in the glial glutamate
action dystonia type, besides postural transporter gene SLC1A3.
dystonia, dystonic movements of an D. It is caused by a triple repeat
upper extremity or action retrocollis mutation in the calcium channel
appear around 8 years. The latter may gene CACNA1A.
associate oculogyric crises. In families E. It has autosomal recessive mode
with action dystonia type, there are adult of inheritance.
Movement Disorders
455 15
vv Correct Answer is: B or fewer, without any accompany-
The clinical presentation is consistent with ing alteration in consciousness. He
episodic ataxia type 2 (EA2). There are may have up to 20 episodes per day.
8 different of this ataxia. It is caused by He is started on carbamazepine by
mutations of ion channel genes and pres- his pediatrician for possible epilepsy
ent as attacks of ataxia with or without which leads to a significant reduction
additional neurologic symptoms. They are in the frequency of episodes. His par-
autosomal dominant. ents bring him to your office for a sec-
Episodes of ataxia in type 1 (EA1) can ond opinion. On further history, his
be associated with brief facial myokymia father and paternal uncle had similar
or neuromyotonia, lasting seconds to but milder symptoms as children,
minutes. EA2 has longer lasting episodes which eventually subsided in middle
of ataxia and can be associated with age. His examination is normal.
nystagmus and dysarthria. Progressive Which of the following is the most
cerebellar atrophy on neuroimaging can likely explanation for his symptoms?
present over time in these patients. A. Paroxysmal kinesogenic dyskinesia
The causative gene is known in four of B. Paroxysmal nonkinesogenic
the episodic ataxias; types 1, 2, 5, and 6. EA1 ­dyskinesia
is due to a mutation in the voltage-gated C. Paroxysmal exertional dyskinesia
potassium channel gene KCNA1 on chromo- D. Paroxysmal hypnogenic dyskinesia
some 12. EA2 results from a non-sense E. Episodic ataxia type 1
mutation and not a triple repeat mutation in
the calcium channel CACNA1A. This is the vv Correct Answer is: A
same gene mutation causing familial The patient has episodes of chorea
hemiplegic migraine and hence patients can (form of dyskinesia) triggered by sudden
have positive history of migraines. EA6 is movement and lasting only seconds,
associated with mutation in the glial with an autosomal dominant pattern of
glutamate transporter gene SLC1A3. inheritance. There is brisk response to
Stress, exertion, caffeine, alcohol and carbamazepine. This is most consistent
heat can trigger EA2 attacks, which can be with paroxysmal kinesogenic dyskine-
treated by acetazolamide. Over time, ataxia sia, which is more common in males
and nystagmus may become more perva- than females with median age of onset
sive symptoms. The pathophysiology for of 12 years. For unknown reasons, it
the progressive nature of symptoms and responds well to carbamazepine and
atrophy on imaging is unknown at this time. other anticonvulsants although EEG
reveals no seizures. Paroxysmal nonkine-
zz Suggested Reading sogenic dyskinesia also manifests with
55 Pandolfo M, Manto M. Cerebellar and intermittent dyskinesias, but the episodes
afferent ataxias. Continuum Lifelong last minutes to hours rather than seconds
Learning Neurol. 2013;19(5):1312. to minutes. They are not triggered by
55 Ropper A, Samuels M, Klien J. Adams and movement (although alcohol or caffeine
Victor’s principles of neurology. 10th ed. may precipitate) and do not respond to
New York: Mcgraw Hill; 2014. p. 946. anticonvulsants. Paroxysmal exertional
dyskinesia is triggered by prolonged
?? 21. A 14-year-old boy develops episodes exertion and may last 5–30 min. Episodes
of left-sided writhing movements, of paroxysmal hypnogenic dyskinesia
especially when getting up quickly are short in duration and may respond
from his desk at school. They last 10 s to anticonvulsants, but are often due to
456 Chapter 15 · Movement Disorders

frontal lobe (supplementary motor) sei- ?? 23. What is the safest treatment for a
zures and by definition occur out of sleep 7-month-old boy with a 12-days his-
rather than occurring with movements tory of binocular dysconjugate, rotary
while awake. Patients with episodic ataxia nystagmus that was preceded by
type 1 have brief episodes lasting sec- ­torticollis?
onds and that may be triggered by move- A. Focused radiation treatments
ment and relieved with anticonvulsants. B. Vascular embolization
However, the episodes are characterized C. Monitor with no intervention
by ataxia and myokimia is present during D. ACTH treatment
and between episodes.
vv Correct answer is: C
zz Suggested Reading Spasmus nutans happens between
55 Fahn S. The paroxysmal dyskinesias. In: 6–12 months of age. First symptom is
Principles and practice of movement either a head tilt or torticollis, sometimes
disorders. Elsevier; 2011. p. 476–95. accompanied with titubation. It lasts for
1–2 years and spontaneously resolves.
?? 22. A 17-year-old male presents with Rarely these symptoms can occur in
facial dystonia and dysarthria. On children with anterior visual pathway
ophthalmologic exam, he has brown glioma. Glioma should be considered if
discoloration around the periphery of nystagmus is monocular, and the optic nerve
his iris. is pale and the onset after 1 year of age.
What is the treatment of choice in There is no vascular location that
his case? would present with these symptoms as a
A. Lactulose result of an aneurysm.
B. Deferoxamine Infantile spasms can be treated with
C. D-penicillamine ACTH, but spams are episodes of repeti-
D. Hemodialysis tive whole body flexion.
E. Disodium edetate
zz Suggested Reading
vv Correct Answer is: D 55 Halachmi-Eyal O, Kowal L. Assessing
This is the classic presentation of Wilson abnormal head posture: a new paradigm.
disease, a disorder of copper metabolism. Curr Opin Ophthalmol. 2013;24(5):432–37.
The Kayser-Fleischer rings are a cardinal
symptom of the disease. Chelation therapy ?? 24. A 16-year-old girl with a history of
15 is the treatment of choice. D-penicillamine persistent abnormal mouth and facial
is first choice, though several other movements for the last 4 months.
agents, such as dimercaprol, may be used. She has severe mental retardation
Deferoxamine chelates iron. Edetate salts and has been on different medica-
chelate heavy metals, but is not used for tions for severe behavioral outbursts.
copper chelation. Lactulose helps with Currently she is taking valproate and
clearance of ammonia. Hemodialysis is not clonazepam for her behavior, but past
used for this condition. medications history is not available.
The movements are seen all day, but
zz Suggested Reading they disappear in sleep.
55 White H. Neurologic manifestations of You observed several periods of
acute and chronic liver disease. ­masticatory movements with lip
Continuum. 2014;20(3 Neurology of smacking and episodic tongue
Systemic Disease):670–80. ­protrusion.
Movement Disorders
457 15
What is the most likely etiology of tilt with titubation and normal optic
these movements? discs on examination?
A. Seizures A. ACTH or high dose steroids for
B. Behavioral disorder 2 weeks
C. Lesch-Nyhan syndrome B. Specific amino acid diet restriction
D. Neuroleptic medication C. Chemotherapy
D. Observation only
vv Correct answer is: D
Tardive dyskinesia can be caused by vv Correct answer is: D
Dopamine receptor targeting drugs such Spasmus nutans appears between ages
as neuroleptics and antimimetics. In all 6–12 months and lasts up to 2 years. It
children with abnormal facial or limb consists of high frequency, low amplitude
movements with a current or recently nystagmus (bi- or unilateral) that can be
history of taking neuroleptics drug- horizontal, vertical or rotary and is often
induced dyskinesia should be suspected. dysconjugate. The reason of the syndrome
Neuroleptic medications are administered is unclear, but it is benign and transitory.
to the pediatric population for mania Therefore, no medical or surgical interven-
and severe behavioral disorders with tion is required.
increasing frequency. Tardive dyskine- Glioma of the anterior visual pathway
sia symptoms can manifest long after can present similarly in some children, but
the medication has been discontinued. characteristically the nystagmus is
Neuroleptics are also an effective treat- monocular with a pale optic nerve.
ment for refractory, problematic motor Progressive cognitive and motor
tic and phonic tics. Focal motor seizures impairment occur gradually in
would rarely manifest as isolated oral- Phenylketonuria if a diet containing the
buccal movements. Subtle movements amino acid, phenylalanine was continued
can be seemed in in partial seizures and in the child diet. Infantile spams consist of
associated with absence seizures. These generalized episodes of body flexion and
oral-buccal movements are not consistent the first line treatment is ACTH or steroids
with a behavioral disorder or an inten- until the spasms resolve.
tional habitual behavior.
Lesch-Nyhan occurs almost exclusively zz Suggested Reading
in males and is characterized by severe 55 Sanger TD, Delgado MR, Gaebler-Spira D,
neurological and behavioral abnormali- Hallett M, Mink, JW. Classification and
ties. Abnormal movements include definition of disorders causing hypertonia
dystonia, chorea and Ballismus but not in childhood. Pediatrics. 2003;111(1):
symptoms similar to tardive dyskinesia. e89–97.

zz Suggested Reading ?? 26. What is your diagnosis of a develop-


55 Correll CU, Leucht S, Kane JM. Lower risk mentally appropriate 10-year-old boy
for tardive dyskinesia associated with with a one-year history of chorea,
second­generation antipsychotics: a dysarthria, emotional liability and a
systematic review of 1-year studies. Am J recent finding of mitral valve insuf-
Psychiatry. 2014. ficiency?
A. Glutaric acidemia type 1
?? 25. What is your treatment for a 9-month- B. Fahr disease
old boy that presents with increasing C. Neuroacathocytosis
binocular nystagmus, persistent head D. Sydenham chorea
458 Chapter 15 · Movement Disorders

vv Correct answer is: D in the evening or when he is tired. At


Sydenham chorea is the most common age 4, he had 3 generalized seizures,
cause of chorea in childhood. The etiology Carbamazepine was prescribed and
is an underlying untreated streptococcal after one-year seizure free, the medi-
infection and chorea is a cardinal feature. cation was discontinued. His EEG at
The onset is approximately 4 months after the time showed “seizure” activity.
the streptococcal infection is contracted. Which of the below questions would
­Anti-­streptococcal anti-bodies cross react help distinguish between seizures
with basal ganglia cells to activate the and movement disorder?
abnormal movements and obsessive- A. Can you suppress the movements
compulsive symptoms. The chorea typi- if you try?
cally begins unilaterally by will generalize B. Have the movements ever
in most cases. occurred during sleep?
Glutaric academia type 1 occurs in C. Are the movements identical each
infancy and include cognitive regression, time?
excessive sweating and irritability D. Did the current movements occur
followed by progressive choreoathetosis during the time the child was
and dystonia. monitored for seizures?
Fahr disease (idiopathic basal ganglia
calcification), occurs secondary to vv Correct answer is: B
infection, metabolic disturbance or Involuntary movements, such as chorea,
genetic disorders. Age of onset is in 3-4th characteristically abate or completely dis-
decade, but may occur in children and appear during sleep, spinal myoclonus is an
comprises of neuropsychiatric symptoms exception. Periodic movements of sleep are
and a variety of abnormal movements. one movement disorder that occurs during
The features of neuroacanthocytosis, sleep, but it not problematic during waking
or choreoacanthocytosis, include motor hours. Seizures do occur during sleep and
tics, oromandibular dyskinesia, and some forms occur more often.
dystonia and occasionally self-mut ilation Most movement disorder cannot be
of the lips. Onset may occur in the teens, suppressed, with the exception of
but most commonly presents in adult- temporary suppression of vocal and motor
hood. tics of childhood. Seizures cannot be
willfully suppressed.
zz Suggested Reading Focal motor seizures will manifest in
15 55 Swedo SE. Sydenham’s chorea: a model for similar motor patterns with each seizure.
childhood autoimmune neuropsychiatric Most movement disorders motor compo-
disorders. Jama. 1994;272(22):1788–91. nent vary and will not provide diagnostic
benefit to differentiate between seizure
?? 27. A 10-year-old boy was evaluated for and movement disorders.
a history of paroxysmal right arm Frontal lobe or temporal lobe focal
and shoulder movements in the last motor seizures may manifest with
4 months. The movements consist of movements described in this case.
sudden flexion of his arm with exten- Nocturnal paroxysmal dystonia was once
sion of his shoulder of 1–2 s duration. believed to be a movement disorder, but
There is no change in his mental subsequent research has proven this
status. The patient cannot anticipate condition to be frontal lobe epilepsy. In
when the movements will occur. The addition, children can have both epilepsy
movements happened more often and a movement disorder.
Movement Disorders
459 15
zz Suggested Reading ?? 29. A 10-year-old boy with a history of
55 Silveira-Moriyama L, Lin JP. A field guide renal failure who was on dialysis and
to current advances in pediatric movement he had encephalopathy, developed
disorders. Curr Opin Neurol. rapid muscle jerks of his arms and
2015;28(4):437–46. shoulders. The movements are sym-
metrical of about 3 cycles of rapid con-
?? 28. A 7-year-old boy with normal behav- traction and relaxation. Sometimes,
ior and he has a good academic the movements in the arms are asym-
performance. He has a three-month metric and even just unilateral. He has
history of rapid facial contractions no change in mental status and he
with forced eye blinking that occur cannot suppress the movements.
more frequently under stress and are What is the classification of those
not present in sleep. movement?
What is your diagnosis? A. Motor tics
A. Tourette syndrome B. Myoclonus
B. Chronic motor tics of childhood C. Chorea
C. Transit motor tics of childhood D. Tremor
D. Nervous child
vv Correct answer is: B
vv Correct answer is: C Myoclonus is classified as several invol-
Transient motor tics of childhood is an untary movements manifested as rapid
isolated motor tic of childhood with a dura- muscle jerks that can be rhythmic or non-
tion of less than 6 months. Motor tics are rhythmic, focal, multifocal or generalized,
abrupt, uncontrollable movements that spontaneous or activated.
most commonly involve head, neck and Myoclonus occurs during sleep tend to
facial muscles. 10% of all children can have have greater amplitude. There are many
tics at some time during the school years. causes of myoclonus, including metabolic
Transient tics may also include phonic or disturbances associated with renal failure.
vocal tics. Motor tics are typically including
A Tourette’s syndrome diagnosis motor complex, stereotyped movements
requires the presence of both motor and that can be at least temporarily sup-
phonic tis, Attention Deficit Disorder and pressed.
Obsessive­Compulsive Disorder. Chronic Choreaform movements are usually
motor tics require at least a 6-month more random, single larger movements
duration of tics. Individuals with chronic that are often incorporated into a volun-
motor tics are at risk for maintaining tary action.
longer-term tic presence and for develop- Rhythmic myoclonus can look very
ing Tourette’s syndrome. Tics could occur in similar to a tremor, but a tremor is a
a nervous child is an outdated and incorrect continuous to and fro movement where
statement. This originates from the fact that rhythmic myoclonus has a characteristic
stress and anxiety include the presence of pause between contraction and relaxation
tics in a child predisposed to the condition of the muscle groups involved.
or activates tics in a child with a tic disorder.
zz Suggested Reading
zz Suggested Reading 55 Silveira-Moriyama L, Lin JP. A field guide
55 Swedo SE, Leonard HL. Childhood to current advances in pediatric movement
movement disorders and obsessive compul- disorders. Curr Opin Neurol.
sive disorder. J Clin Psychiatry. 1994. 2015;28(4):437–46.
460 Chapter 15 · Movement Disorders

?? 30. A 7-year-old girl is diagnosed at 5 years zz Suggested Reading


of age with moderate quadriplegic 55 Sanger D, Delgado MR, Gaebler-Spira D,
cerebral palsy because of significant Hallett M, Mink W. Classification and defi-
variability of her spasticity and restric- nition of disorders causing hypertonia in
tion of movement. The symptoms are childhood. Pediatrics. 2003;111(1):
much better when she awakens in the e89–97.
morning, then her muscle tone gradu-
ally increases as the day progresses. ?? 31. A parents of a 10-year-­old boy have a
The gait disturbance was noticed by concern regarding their sons “unusual
her family at 4 years of age, with her behavior”? He has a rapid, random
right lower leg being most affected. involuntary movements of his arms
The abnormal tone spread to her arms, that he appears to end in a voluntary
and then became generalized. On action. His arms move simultaneously
examination, she has no cerebellar, down to his sides, and then he slips
sensory or autonomic disturbances them into his pockets. He will also jerk
and her intelligence is age appropriate. his leg up, and then act as if he is re-
What treatment is most likely to tying his shoe.
benefit this child the most? What is your classification of this
A. Baclofen movement?
B. Rhizotomy A. Ballismus
C. Levodopa B. Athetosis
D. Dantrolene sodium C. Chorea
D. Motor tics
vv Correct answer is: C
Her symptoms with diurnal variation indi- vv Correct answer is: C
cates Dopa-responsive dystonia. Usually, Chorea is a rapid, involuntary movement
it starts at 4–8 years of age and the initial that can affect any body that culminates
features can be foot dystonia or abnormal with the patient attempting to integrate
hand posturing. The increased tone and the end of the movement into a purposeful
posturing progress to include arms and action. The rate of movements can be spo-
legs and is commonly diagnosed as cere- radic or almost continuous.
bral palsy. A trial of carbadopa-levodopa Ballismus is a high-amplitude rapid
is recommended if generalized dystonia violent movement of a limb, shoulder or
is suspected in children. Baclofen is pre- pelvis. In children is occurs with cerebral
15 scribed for the treatment of spasticity asso- palsy, kernicterus, Sydenham chorea and
ciated with cerebral palsy and spinal cord lupus.
injury. The clinical course symptoms onset Athetosis are low-amplitude writing
at age 4 years with continued progres- movements that occur alone or associated
sion is not characteristic of cerebral palsy. with chorea in choreoathetosis. The most
Rhizotomy is a neurosurgical procedure for common cause of independent Athetosis is
the treatment of medically refractory spas- perinatal brain injury and is seen in
ticity mainly in diplegic amulatory patients. children with cerebral palsy.
Dantrolene sodium reduces the Motor tics usually involve head, neck
excitation contraction in spasticity and facial movements in transient or
associated with cerebral palsy and spinal chronic motor tic of childhood or
cord injury. Tourette’s syndrome.
Movement Disorders
461 15
zz Suggested Reading frequently get treated for ischemic stroke,
55 Boyd KN, Mailman RB. Dopamine however, MRI is always normal. Symptoms
receptor signaling and current and future resolve spontaneously within several
antipsychotic drugs. In Current hours. FHM comes in several genetic
Antipsychotics. Springer Berlin: variants, the most common being FHM1,
Heidelberg; 2012. pp. 53–86. which is due to defect in the CACNA 1A
gene on Chromosome 19, encoding a P/Q-
?? 32. An 18-year-old boy presents for evalu- type calcium channel. B: Mutations in the
ation of recurrent episodes of weak- CACNA1A gene also cause Episodic Ataxia
ness and headaches. He states that he 2, and spinocerebellar ataxia type 6. All
typically sees bright flickering lines, other choices are channellopathies as well,
accompanied by difficulty speaking however, none of them involve calcium
and right-sided weakness. Shortly channels. A and C: Episodic Ataxia 1 and
afterwards he gets a unilateral pound- benign familial neonatal convulsions are
ing debilitating headache, associated both potassium chanellopathies. D and E:
with nausea, photo- and phonopho- Myotonia congenital is a chloride chanel-
bia. He has been hospitalized 3 times lopathy, and generalized epilepsy with
for these episodes, and has been febrile seizures (GEF+) is a sodium chanel-
treated twice for stroke with a “clot- lopathy. It should be noted that these
buster”. All episodes resolve spon- characteristics may be seen in several
taneously within 24 h. His mother other neurological disorders. Specifically,
who passed away in a car accident Generalized epilepsy with febrile seizures
last year had similar symptoms. His plus (GEFS+) is related to familial hemiple-
current neurological examination is gic migraine type 3 (FHM3), yet also
unremarkable. MRI brain is normal. comes up in relation to severe myoclonic
Subsequent work-up reveals a gene epilepsy of infancy (SMEI), temporal lobe
defect involving a calcium channel. epilepsy (TLE),and benign familial neona-
The gene defect responsible for this tal- infantile seizures (BFNIS). In addition,
patient’s condition is also the cause CACNA1A is involved in FHM1, SCA6,
for which of the following? childhood absence epilepsy (CAE), and
A. Episodic Ataxia 1 episodic ataxia type 2 (EA2).
B. Episodic Ataxia 2
C. Benign familial neonatal convul- zz Suggested Reading
sions 55 Barrett CF, van den Maagdenberg AM,
D. Myotonia congenital Frants RR, Ferrari MD. Advances in
E. Generalized epilepsy with febrile Genetics. 2008;63:57–83.
seizures (GEF+)
?? 33. A 14-year-old boy complaining of
vv Correct answer is: B multiple attacks of dystonic postur-
This patient has familial hemiplegic ing of his arm and leg per day. These
migraine (FHM). FMH is an autosomal attacks usually occur when he is play-
dominant chanellopathy resulting in ing with his friends, but can occur at
migraines with auras associated with any time. He is an excellent baseball
unilateral hemiparesis that can last up player, but has not been participating
to several hours. Occasionally cerebellar as much in baseball lately because
sign and symptoms are present. Patients the attacks occur frequently during
462 Chapter 15 · Movement Disorders

such activities, and he has fallen or several hours, and there are sometimes
dropped the ball several times dur- no clear triggers, although episodes can
ing these attacks. They last about be aggravated by alcohol, caffeine, and
15–30 s, and can occur multiple times fatigue. Episodes are less frequent than
a day, depending on his level of activ- in PKD. PNKD does not typically respond
ity. They also occur if he is startled. to anticonvulsants. A third form of par-
In between episodes. His exam is oxysmal dyskinesias is referred to as par-
entirely normal. oxysmal exertional dyskinesias, in which
Which of the following statements episodes are triggered by prolonged
is correct regarding this patient’s exercise and last typically 5–30 min
condition? but sometimes up to 2 h. Paroxysmal
A. This disorder results from a muta- hypnogenic dyskinesias were previously
tion in the nicotinic acetylcholine thought to be non-epileptic dyskinesias
receptor but are now known to be frontal lobe sei-
B. This patient has paroxysmal non- zures as part of the syndrome autosomal
kinesigenic dyskinesia dominant nocturnal frontal lobe epilepsy,
C. A good response to anticonvul- which results from mutations in the
sants is seen in this disorder nicotinic acetylcholine receptor, among
D. A good response to acetazol- other genetic mutations. The paroxysmal
amide is seen in this disorder dyskinesias are not epileptic, but there is
E. This is an epileptic disorder an association between PKO or PNKD and
epilepsy; families have been reported that
vv Correct answer is: C have both PNKD and infantile convulsions
The clinical presentation is consistent with choreoathetosis, and a mutation
with paroxysmal kinesigenic dyskinesias in the gene encoding a sodium/glucose
(PKOs). There are several categories transporter on chromosome 16 has been
of paroxysmal dyskinesias, all sharing identified.
in common episodes of hyperkinetic
abnormal movements with intervening zz Suggested Reading
normalcy. The abnormal movements 55 Fahn S, Jankovic J. Principles and practice
may include dystonia, chorea or cho- of movement disorders. Philadelphia:
reoathetosis, ballism, or dysarthria. Elsevier; 2007.
These disorders differ in the length of
the paroxysm, triggers for the episodes, ?? 34. Which of the following statements is
15 pharmacologic therapy, and genetics. incorrect regarding acquired causes
PKO is characterized by episodes that last of cerebellar ataxia?
seconds to at most 5 min, triggered by A. Hypothyroidism can lead to gait
sudden movement as well as by startle ataxia, and ­thyroid-­stimulating
and hyperventilation. PKO may be either hormone should be checked in
familial or sporadic, and secondary forms patients with gait ataxia
occur in multiple sclerosis, following B. Celiac autoantibodies should
trauma, in patients with a history of peri- be checked in patients with gait
natal hypoxic encephalopathy, and in the ataxia only if there are gastro-
setting of other underlying neurologic intestinal symptoms to suggest
disorders. The primary form responds gluten intolerance
well to anticonvulsant such as carbam- C. The chemotherapeutic agent
azepine. In paroxysmal non-kinesigenic cytarabine can lead to irreversible
dyskinesia (PNKD), attacks last 2 min to cerebellar ataxia
Movement Disorders
463 15
D. Mercury and bismuth both can zz Suggested Reading
lead to cerebellar ataxia in toxic 55 Bradley WG, Oaroff RB, Fenichel GM,
amounts et al. Neurology in clinical practice, 5th ed.
E. Chronic phenytoin can lead to Philadelphia: Elsevier; 2008.
cerebellar atrophy due to Purkinje 55 Ropper AH, Samuels MA. Adams and
cell loss Victor’s principles of neurology, 9th ed.
New York: McGraw-Hill; 2009.
vv Correct answer is: B
Celiac disease can cause isolated cerebel- ?? 35. A 6-year-old boy came to the office
lar dysfunction without gastrointestinal with his parents for concern of sei-
symptoms. In a patient with ataxia zures. His family reported that he
of unclear etiology, celiac antibodies has episodes of flapping his arms
should be checked as gluten free diet repeatedly against his sides and then
can improve the ataxia. There are several against his head, sometimes for sev-
causes of acquired cerebellar ataxia. eral minutes. He ignores everyone
Hypothyroidism can cause gait ataxia, around him during those episodes.
and checking serum thyroid­stimulat- Episodes happens when he is upset,
ing hormone is indicated in an adult but also in the evening around the
presenting with ataxia. Supplementation time his father gets back home from
with thyroid hormone can lead to work, particularly when his father
improvement of the gait disorder. brings home ice cream or other treats.
Chemotherapeutic agents including He has a motor and language delay,
5-fluorouracil and cytarabine can cause and has been in special education
significant cerebellar toxicity. In cyta- classes at school. He does not have
rabine toxicity, Purkinje cell loss and many friends and spends most of his
gliosis occur, and there is loss of dentate free time playing with the same train
neurons as well; the cerebellar dysfunc- set, assembling and disassembling it
tion is typically irreversible. Metals such repeatedly.
as mercury can cause cerebellar toxicity What is your impression?
as well as visual cortex toxicity, leading to A. He likely has seizures and he
a syndrome of ataxia, visual field deficits, should undergo video-EEG moni-
and paresthesias. Bismuth salicylate can toring
cause cerebellar toxicity if ingested in B. He likely has a complex motor tic
high amounts. Other cerebellar toxins disorder
include the solvent toluene. Chronic C. This patient’s history is consistent
intake of phenytoin can cause cerebellar with Tourette’s syndrome
atrophy due to damage to Purkinje cells. D. He has paroxysmal dyskinesias
Acute phenytoin toxicity can cause a E. This patient likely has stereotypies
reversible cerebellar ataxia. Other causes
of acquired cerebellar ataxia include vv Correct answer is: E
infection (as in HIV infection, Creutzfeldt­ From the history, the episodes of arm flap-
Jakob disease, and Whipple’s disease) ping are likely stereotypies. Stereotypies
or post infection (such as is seen after are patterned, repetitive, stereotyped
varicella zoster infection in children). movements, or vocalizations that occur in
The Miller-Fisher variant of Guillain­Barre response to an external or internal stimu-
leads to ataxia in addition to areflexia, lus. Common stereotypies are head nod-
ophthalmoplegia, and involvement of ding, arm flapping, body rocking, head
other cranial nerves. banging, grunting, humming, or moaning.
464 Chapter 15 · Movement Disorders

They may occur in otherwise normal vv Correct answer is: A


children during times of excitement or Treatment of psychogenic movement
boredom, but more often occur in children disorders is a process that often requires
with developmental delay and autism. multiple steps and interventions. The
When associated with the latter, self- most important first step is the diagnos-
injurious stereotypies may be present. In tic debriefing, in which the diagnosis is
Rett’s syndrome, several stereotypies are disclosed in an empathetic and nonjudg-
often seen, including hand wringing, body mental way. The most important predictor
rocking, and others. Stereotypies have no of favorable outcome after treatment for
urge with relief after executing the move- these patients is acceptance of their diag-
ment or vocalization like complex motor nosis. The 3 important points of emphasis
tics. This patient’s history is not consistent should be that the movements are real
with Tourette’s syndrome or paroxysmal and disabling, that they are not deliberate,
dyskinesias. and that they are not necessarily indica-
tive of psychiatric disease. It is crucial to
zz Suggested Reading emphasize the certainty of the diagnosis,
55 Fahn S, Jankovic J. Principles and practice and further neurological tests are not
of movement disorders. Philadelphia: necessary or recommended. Ultimately,
Elsevier; 2007. psychological intervention may be appro-
priate for many of these patients, and
?? 36. An 18-year-old girl presents for evalu- ­multidisciplinary approaches with physical
ation of new abnormal movements of therapy and rehab medicine may be also
the right arm. The movements began useful.
shortly after she got into an argument
with her mother. The movements have zz Suggested Reading
persisted since then. The movement 55 Morgante F, Edwards M, Espay A.
disappears during sleep. On examina- Psychogenic movement disorders.
tion, she has continued tremor of the CONTINUUM: Lifelong Learning in
right hand that is equal at rest and Neurol. 2013;19(5):1383–96.
with posture. The frequency of the
tremor varies during your exam, and it ?? 37. You are evaluating a 9-year-old boy
increases when talking about the argu- with symptoms of being off balance.
ment that triggered her symptoms. The symptoms come on in attacks and
The frequency also entrains to a dif- he is completely normal in between
15 ferent examiner-provided frequency, attacks. In the office, he had been
and disappears during a motor exami- having symptoms for a few hours. On
nation of the left arm. You suspect examination, he has dysarthria, ataxia
she has a psychogenic movement and nystagmus. While still in the clinic,
disorder. his symptoms gradually resolved and
What is the most important initial he complained of a margin like head-
intervention for this condition? ache.
A. Diagnostic debriefing What is the correct statement
B. Cognitive behavioral therapy regarding his condition?
C. Dynamic psychotherapy A. This condition is associated with
D. Physical therapy mutation in the voltage-­gated
E. Start an SSRI for depression potassium channel gene KCNA1.
Movement Disorders
465 15
B. This condition can be associ- There are 4 known causative genes for
ated with progressive cerebellar the episodic ataxias; types 1, 2, 5, and 6.
­atrophy. EA1 is due to a mutation in the voltage-
C. This condition is associated with gated potassium channel gene KCNA1 on
mutation in the glial glutamate chromosome 12. EA2 results from a
transporter gene SLC1A3. non-sense mutation and not a triple
D. It is caused by a triple repeat repeat mutation in the calcium channel
mutation in the calcium channel CACNA1A. This is the same gene mutation
gene CACNA1A. causing familial hemiplegic migraine and
E. It has autosomal recessive mode hence patients can have positive history
of inheritance. of migraines. EA6 is associated with
mutation in the glial glutamate trans-
vv Correct answer is: B porter gene SLC1A3.
This is a clinical presentation of episodic Stress, exertion, caffeine, alcohol and
ataxia type 2 (EA2). There are 8 subtypes heat can trigger the attacks of EA2, and
of these ataxias group, caused by muta- can be ameliorated by use of acetazol-
tions of ion channel genes and present amide. Over time, ataxia and nystagmus
as attacks of ataxia with or without addi- may become more pervasive symptoms.
tional neurologic symptoms. They have The pathophysiology for the progressive
autosomal dominant mode of inheritance. nature of symptoms and atrophy on
Episodes of ataxia in type 1 (EA1) can imaging is unknown at this time.
be associated with brief facial myokymia
or neuromyotonia, lasting seconds to zz Suggested Reading
minutes. EA2, as in this case, has longer 55 Pandolfo M, Manto M. Cerebellar and
lasting episodes of ataxia and can be afferent ataxias. Continuum Lifelong
associated with nystagmus and dysarthria. Learning Neurol. 2013;19(5):1312.
Additionally, progressive cerebellar 55 Ropper A, Samuels M, Klien J. Adams and
atrophy can be seen over time in these Victor’s principles of neurology. 10th ed.
patients. New York: Mcgraw Hill; 2014. p. 946.
467 16

Neuro-cutaneous
­Syndromes

© Springer International Publishing AG, part of Springer Nature 2018


Y. M. Awaad, Absolute Pediatric Neurology, https://doi.org/10.1007/978-3-319-78801-2_16
468 Chapter 16 · Neuro-cutaneous Syndromes

?? 1. Which of the following statements is callosum, Dandy-Walker syndrome, and


incorrect regarding epidermal nevus neural tube defects. Cerebral vascular
syndrome (ENS)? abnormalities including leptomeningeal
A. This syndrome is a manifestation hemangiomas and arteriovenous mal-
of a heterogeneous group of dis- formations may also occur. Patients with
orders that share in common the ENS are at increased risk of malignancy;
presence of epidermal nevi the nevus itself may undergo malignant
B. Hemimegalencephaly may occur transformation into basal cell carcinoma
ipsilateral to a facial nevus or other skin malignancies, and there
C. The epidermal nevi in this disor- is an increased risk of astrocytoma and
der may undergo malignant trans- other, systemic, malignancies. Various
formation skeletal abnormalities may occur in
D. There is increased risk of astrocy- ENS, including kyphoscoliosis. Ocular
toma and other tumors in patients abnormalities include iris colobomas
with ENS (most common), retinal lesions, and
E. Lisch nodules are a feature of ENS strabismus. Cardiac and genitourinary
abnormalities also occur.
vv Correct Answer is: E
E. Lisch nodules are a feature of neu- ?? 2. Regarding the finding in a patient
rofibromatosis, not epidermal nevus with neurofibromatosis type 1 shown
syndrome (ENS). Iris colobomas are the in the Figure, which of the following
most frequent ocular abnormality in statements is incorrect?
patients with ENS. ENS includes several
disorders that are characterized by the
presence of epidermal nevi and neuro-
logic manifestations. These disorders
include Proteus syndrome, which is
characterized by asymmetric and often
marked hypertrophy of soft tissues and
bones, and other rare disorders such as
sebaceous nevus syndrome and Becker
nevus syndrome. Epidermal nevi are
slightly raised patches of hyperpig-
mentation that are present at birth or
appear in childhood. They enlarge over ..      Fig. 16.1 Genitourinary Plexiform Neurofibroma
time. Not all patients have neurologic Mimicking Sacrococcygeal Teratoma. (7 http://openi.
nlm.nih.gov/imgs/512/348/3461780/3461780_JSTCR-
16 manifestations; occurrence of nevi over
the face and scalp predict neurologic
4-50-g001.png. Nasir AA, Abdur-Rahman LO, Ibrahim
KO, Adegoke MA, Afolabi JK, Adeniran JO – J Surg Tech
involvement. Neurologic manifesta- Case Rep (2012) Wikipedia)
tions may include intellectual disability,
seizures, and cranial neuropathies. In
patients with hemimegalencephaly A. It is a plexiform neurofibroma
(which often occurs ipsilateral to a facial B. These types of neurofibromas can
nevus), contralateral hemiparesis may invade the skin, causing thicken-
be seen. Other brain malformations ing and skin hypertrophy
seen in patients with ENS include focal C. Plexiform neurofibromas consist of
pachygyria, agenesis of the corpus mainly Schwann cells and fibroblasts
Neuro-cutaneous Syndromes
469 16
D. In a minority of cases, plexiform ?? 3. You are seeing a 9-year-old boy with
neurofibromas undergo malig- a history of headache. His head-
nant degeneration into malignant aches increase when he lie down
peripheral nerve tumors and accompanied with nausea and
E. Plexiform neurofibromas typi- vomiting. You ordered an MRI which
cally regress with age, particularly showed a 1 centimeter enhancing
when they first form at younger mass close to the foramen of Monro.
age. As a baby he had a cardiac rhabdo-
myoma.
vv Correct Answer is: E What gene is responsible for his
Figure shows a plexiform neurofibroma. condition?
Plexiform neurofibromas enlarge in A. The patient has a mutation in the
approximately half of patients, particu- NF1 gene
larly when they are present prior to the B. The patient has a mutation in the
age of 10 years. Neurofibromas, seen in TSC1 gene
neurofibromatosis type 1, occur in two C. The patient has a mutation in the
types: cutaneous neurofibromas, which PTCH1 gene
originate in the dermis or adjacent lay- D. The patient has a mutation in the
ers, and plexiform neurofibromas, which PTEN gene
originate in peripheral nerves. They E. The patient has a mutation in the
consist predominantly of Schwann cells NF2 gene
and fibroblasts but also contain mast
cells. Plexiform neurofibromas may vv Correct Answer is: B
occur in the face and can be disfiguring; This is a clinical presentation of tuber-
invasion into the overlying skin causes ous sclerosis complex. TS is caused by
hypertrophy, hyperpigmentation, and mutations in the TSC1 or TSC2 genes,
thickening of the skin. Neurofibromas coding for hamartin and tuberin,
that arise from dorsal root ganglia can respectively. Major criteria include
grow in a dumbbell shape, invading the facial angiofibromas, forehead plaques,
spinal canal and leading to nerve root ungula or peri-ungual fibroma, 3
and even spinal cord compression. Neu- hypomelanotic macules, shagreen
rofibromas can arise also in the GI tract, patch, retinal nodular hamartomas,
leading to intestinal obstruction or gas- cortical tubers, sub-ependymal nodules,
trointestinal bleeding. As mentioned, in sub-ependymal giant cell astrocytoma,
almost 50%, plexiform neurofibromas cardiac rhabdomyoma and lymphan-
enlarge and can become disfiguring giomyomatosis/ renal angiomyolipoma.
when superficial, particularly when Minor criteria include hamartomatous
present from an early age (before the rectal polyps, multiple pits in dental
age of 10 years). These lesions should enamel that are randomly distributed,
be monitored closely, since in minority bone cysts, cerebral white matter radial
they can undergo malignant. migration lines, gingival fibromas, non-
renal hamartomas, retinal achromatic
zz Suggested Reading patch, confetti skin lesions, and renal
55 Degeneration into a malignant periph- cysts. The diagnosis of definite tuberous
eral nerve sheath tumor. ©2011 Wolters sclerosis complex can be made in the
Kluwer Health Lippincott Williams & presence of either 2 major features or 1
Wilkins N2013E.All rights reserved. major feature and 2 minor features. The
470 Chapter 16 · Neuro-cutaneous Syndromes

presence of 1 major feature and 1 minor zz Suggested Reading


feature is sufficient for a diagnosis of 55 Jozwiak S, Migone N, Ruggieri M. The
probable tuberous sclerosis complex. tuberous sclerosis complexes. In: Ruggieri
The presence of 1 major feature in isola- M, Pascual­Castroviejo I, Di Rocco C, eds.
tion, or the presence of at least 2 minor Neurocutaneous Disorders Phakomatoses
features is suggestive of possible tuber- and Hamartoneoplastic Syndromes.
ous sclerosis complex. The patient’s Morlenbach: Springer-Verlag/Wien;
enhancing brain mass, located near the 2008:181–227.
foramen of Monro, is consistent with 55 Tabori U, Laberge A-M, Ellezam B, Carret
sub-ependymal giant cell astrocytoma A-S. Cancer predisposition in children with
(SEGA). This, and the patient’s cardiac brain tumors. In: Scheinemann K, Bouffet
rhabdomyoma, allow for a diagnosis of E, editors. Pediatric Neuro-­Oncology.
definite tuberous sclerosis complex. New York: Springer; 2015. p. 69–89.
Mutations in the NF1 gene, encod- 55 Gorlin RJ. Nevoid basal cell carcinoma.
ing for the protein neurofibromin, In: Ruggieri M, Pascual-Castroviejo I, Di
cause neurofibromatosis type I. This is Rocco C, editors. Neurocutaneous Dis-
diagnosed in the presence of 2 or more orders Phakomatoses and Hamartoneo-
of the following:(1) the presence of > 6 plastic Syndromes. Morlenbach: Springer/
cafe au lait macules (5 mm in diameter Wien; 2008. p. 669–94.
pre-pubertal, 15 mm in diameter post 55 Chan TSY, Wang X, Spence T, Taylor MD,
pubertal) (2); 2 neurofibroma or 1 plexi- Huang A. Embryonal brain cell tumors.
form neurofibroma (3) freckling of the In: Scheinemann K, Bouffet E, editors.
inguinal or axillary region (4) an optic Pediatric Neuro-Oncology. New York:
glioma, (5) iris hamartomas (6) osseous Springer; 2015. p. 127–13.
lesion (thinning of long bone cortex,
sphenoid wing dysplasia) or (7) first ?? 4. Which of the following is not correct
degree relative with NF. regarding neurofibromatosis type 1
Mutations in the PTCH1 gene encod- (NF1)?
ing for Patched 1 are associated with A. Renal artery stenosis and pheo-
nevoid basal cell carcinoma, or Gorlin chromocytoma are on the differ-
syndrome. Major features include > 2 ential diagnosis of hypertension
basal cell carcinomas (12: is sufficient occurring in NF1 patients
for people under 20 years of age), odon- B. There is an association between
togenic keratosis of the jaw, 3 > palmar NF1 and Moyamoya disease
or plantar pits, bilamellar calcification C. The majority of patients with NF1
of falx cerebri, rib abnormalities (bifid, have severe developmental delay
16 fused, or markedly splayed), and a first
degree relative with the condition.
D. Macrocephaly is the most com-
mon head size abnormality seen in
Minor features include (1) frontal boss- NF1 Patients and is independent
ing/ hypertelorism/ cleft lip or palate (2) of the extent of hydrocephalus
sprengel deformity/ pectus/ syndactyly E. NF1 patients have an increased
of digits, (3) bridging of sella turcica/ risk of intracranial aneurysms
hemivertebrae flame shaped radio-
lucensies (4) ovarian fibroma and (5) vv Correct Answer is: C
medulloblastoma. A diagnosis of Nevoid The majority of patients with neurofibro-
basal cell carcinoma can be made in the matosis type 1 (NF1) have normal cogni-
presence of either 2 major criteria or 1 tion or mild developmental delay. Other
major and 2 minor criteria. neuropsychiatric manifestations in NF1
Neuro-cutaneous Syndromes
471 16
include behavioral problems and learn- atrophy. SMN2 produces a protein prod-
ing disabilities, which may be present in uct that appears to improve the pheno-
approximately half of NF1 patients. type in patients with SMN1. The larger the
Renal artery stenosis due to renal copy of SMN2, the milder the disease.
artery dysplasia occurs in some patients PMP2 is peripheral myelin protein gene
with NF1 and can lead to hypertension. that is duplicated on chromosome 17 in per-
Pheochromocytoma has also been sons with Charcot-Marie-Tooth type 1.
associated with NF1 and the latter two
causes of hypertension should be consid- zz Suggested Reading
ered in an NF1 patient with hypertension. 55 Northrup H, Koenig MK, Au KS. Tuberous
Moyamoya disease and other cere- sclerosis complex. 2011.
bral artery abnormalities including
intracranial aneurysms may occur in ?? 6. You diagnosed a 12-year-old boy
NF1 patients. recently with tuberous sclerosis complex.
Macrocephaly is the most common You ordered an MRI of the brain as part
head size abnormality seen in NF1 of his work-­up. His MRI is shown below.
patients and occurs independent of Which of the following statements is
hydrocephalus, although aqueductal incorrect regarding the findings on
stenosis may occur in NF1. this MRI
Thinning of the cortex of long A. These are known as cortical tubers
bones and other long bone dysplasias B. These lesions are hamartomas
may lead to pathologic fractures and C. The burden of such lesions corre-
pseudoarthrosis. Other skeletal abnor- lates with cognitive function
malities in NF1 include scoliosis and D. The burden of such lesions cor-
sphenoid wing dysplasia. relates with the occurrence and
severity of seizures
?? 5. You are evaluating a 5-month-old E. These lesions are premalignant
child with developmental delay,
infantile spasms, a renal tumor and
sub ependymal nodules on MRI most
likely has the presence of which of the
following genes?
A. TSC1
B. TSC2
C. SMN1
D. SMN2
E. PMP2

vv Correct Answer is: A


The tuberous sclerosis complex (TSC) is an
autosomal dominant disorder with vari-
able phenotypic transmission. Two genes
responsible for are TSC1 on chromosome
9 and TSC2 on chromosome 16. Children
with TSC1 are more likely to have more
severe symptoms than those with TSC2.
SMN1 is responsible for approximately ..      Fig. 16.2 ILAE.org. EpilepsyDiagnosis.org,
95% of the cases of spinal muscular Diagnostic Manual
472 Chapter 16 · Neuro-cutaneous Syndromes

vv Correct Answer is: E vv Correct Answer is: A


There are multiple hyper-intense lesions in Menkes disease, also known as kinky
the cortex and gray­white junction. These hair syndrome, is a neuro-cutaneous
are cortical tubers, also known as cortical disorder characterized by the presence
hamartomas, findings seen in tuberous of brittle hair (pili torti), hyper-elastic
sclerosis complex (TSC). Cortical tubers skin, and thin or absent eyebrows.
contain large bizarre neurons, abnormal Abnormal fullness of the cheeks, high­
glia, predominantly astrocytes, and hypo- arched palate, micro-gnathia, osteopo-
myelinated axons. These lesions are not rosis, and metaphyseal dysplasia are
premalignant. CNS manifestations of TSC also seen. Neurologic manifestations
include intellectual disability, although up include cerebral vasculopathy and pro-
50% of the patients with TSC have normal gressive cerebral atrophy leading to
intelligence. Seizures occur in the majority subdural hematomas and/or hygromas.
of TSC patients; the majority of patients Seizures and severe developmental
with TSC and intellectual disability will delay occur. Other organ systems are
have seizures, but not all TSC patients with involved, including the skeletal, gas-
seizures will have intellectual disability. trointestinal, and genitourinary tract.
A variety of seizure types may occur in Menkes disease is an X-linked recessive
patients with TSC; TSC is the most com- disorder resulting from a mutation in
mon cause of infantile spasms and, in such ATP7A, a copper transporter, resulting
cases, treatment with vigabatrin is very in defective copper transport across
effective. Other common neuropsychiatric the intestines and widespread cop-
manifestations of TSC include behavioral per deficiency in the brain and other
problems including attention deficit/ organs. Various enzymes require cop-
hyperactivity disorder and learning dis- per as a cofactor, including cytochrome
abilities. c oxidase, dopamine 8-hydroxylase,
The burden of cortical tubers cor- and lysyl oxidase among others. Histo-
relates to some extent with cognitive pathologically, the cortex, thalamus,
function and presence, frequency, and and subcortical nuclei demonstrate loss
severity of seizures. of neurons and gliosis. The cerebellum
shows loss of granular neurons and Pur-
?? 7. You are evaluating a 3-month-old baby kinje cells. Laboratory studies demon-
for developmental delay. His brain MRI strate low serum levels of ceruloplasmin
shows atrophy with bilateral subdural and copper. The presence of subdural
hematomas. He is hypotonic. He has a hematomas in an infant may raise ques-
coarse, brittle hair, hyperelastic skin, and tion of non-accidental injury, although
16 absent eyebrows.
What is the most likely diagnosis?
the cutaneous findings and other
manifestations allow the distinction to
A. Menkes disease be made. Menkes disease is a disorder
B. Wilson disease of copper, deficiency, unlike Wilson
C. Ehlers-Danlos syndrome disease, a disorder of copper toxicity,
D. No accidental injury (child abuse) which is due to a mutation in the ATP7B
E. Hypomelanosis of Ito enzyme.
Neuro-cutaneous Syndromes
473 16
?? 8. What medications would be helpful in generalized seizures, and even worsening
the treatment of this patient? cognitive function and hemiparesis.

?? 9. You got a call from the radiologist


regarding a 9-year-old female, who
you referred her for an MRI as part of
her seizure work-up. Her neurological
examination was non-focal and on
her skin exam, there was a unilateral
cutaneous angioma over the left max-
illary region of her face.
What is the brain abnormality and
where is it is most likely located?
A. Right temporal grey matter-lined
cleft extending from the epen-
..      Fig. 16.3 Courtesy of Dr. David Rothner. Shown dyma to the pia mater
also in color plates. Comprehensive Review in Clini- B. Left hemispheric cyst
cal Neurology: A Multiple-Choice Question Book for C. Venous angioma of the pia matter
the Wards and Boards ©2011 Wolters Kluwer Health D. Cortical tubers and sub-ependy-
Lippincott Williams & Wilkins. All rights reserve
mal nodules
A. Famotidine 25 mg PO BID
B. High-dose vitamin D daily vv Correct Answer is: C
C. Dorzolamide 2% (Trusopt) drops Sturge-Weber syndrome is a sporadic
to affected eye q8h neuro-cutaneous disorder consisting of a
D. Amitriptylene 10 mg qHS port-wine stain (cutaneous angioma) of
E. Minocycline 50 mg BID the face and ipsilateral venous angioma
of the pia matter. The clinical presentation
vv Correct Answer is: C might include cognitive impairments, con-
This is a case of Sturge-Weber Syndrome tralateral hemiparesis with hemiatrophy
(also called also called encephalotrigemi- and homonymous hemianopia. A 15–50%
nal angiomatosis) and his likely medica- of newborn with a facial port-wine birth-
tions include AED such as phenytoin and mark in the upper face region has a risk of
glaucoma drops such as Dorzolamide developing Sturge-Weber syndrome with
or Brinzolamide. Sturge-Weber (SWS) is the accompanied brain and ocular involve-
caused by residual embryonal blood ves- ment, and up to 80% will have seizures.
sels. These vessels develop around the Grey matter-lined cleft extending
cephalic portion of the neural tube part from the ependyma to the pia mater is
of which is in the ectoderm destined to characteristic of a schizencephalic cleft
become facial skin. Normally, these ves- and is not associated with a neuro-cuta-
sels regress around the ninth week of neous manifestation.
gestation but failure of regression results A porencephalic cyst is a broad cat-
in residual vascular tissue, which forms egorization of the presence of a cerebral
the abnormal vascular collections which cyst and most are formed as a result of
can be in the leptomeninges, face, and cell loss. The most common etiologies
eye. Besides the Port-Wine stain usually include prenatal or antenatal infarction
seen on the face, SWS patients often have or trauma. There is no associated neuro-
developmental delay, ADHD, seizures, and cutaneous findings.
glaucoma. Neurologic damage can prog- Cerebral cortical tubers and sub-­
ress from small focal seizures to frequent ependymal nodules are seen in individuals
474 Chapter 16 · Neuro-cutaneous Syndromes

with Tuberous Sclerosis. Associated der- What is incorrect regarding the dis-
matologic features include hypomelanotic order depicted?
macules and raised plaques of thickened A. It is autosomal dominant in inheri-
skin, referred to as Shagreen patches. tance
B. It results from a mutation in the
zz Suggested Reading merlin gene on chromosome 22
55 Comi AM. Sturge-­Weber syndrome. C. Various CNS tumors including
Handbook of clinical neurology. schwannomas, meningiomas,
2014;132:157–68. astrocytomas, and ependymomas
can occur in this disorder
?? 10. Neuro-cutaneous syndromes (phako- D. Cutaneous findings such as neuro-
matoses) represent a group of central fibromas and axillary freckling are
nervous system disorders that can be common in this disorder
associated with lesions in the skin, E. Sub-capsular cataracts are a fea-
eye, brain and other visceral organs. ture of this disorder
The neuro-­cutaneous syndrome
least likely to have comorbid epi- vv Correct Answer is: D
lepsy is: Neurofibromatosis type 2(NF2) has less
A. Sturge Weber cutaneous findings than (NF1). NF2 is less
B. Tuberous Sclerosis common than NF1 and has distinct diag-
C. Neurofibromatosis Type 1 nostic criteria, clinical manifestations, and
D. Hypomelanosis of Ito pathophysiology. Diagnostic criteria for
E. Incontinentia Pigmenti NF2 include one of the following:
55 Bilateral schwannomas of cranial nerve
vv Correct Answer is: C (CN) VIII (although other CNs can be
Epilepsy can be seen as a feature in affected, most commonly CN V).
many of the Neuro-cutaneous syn- 55 A unilateral CN VIII schwannoma with
dromes. The occurrence rate of seizures a first-degree relative with NF2.
varies. Seizures occur in approximately 55 A family history of a first-degree
80% of patients with Sturge Weber, 80% relative with NF2 combined with any
of patients with Tuberous Sclerosis, 50% two of the following lesions: neuro-
of children with Hypomelanosis of Ito, fibroma, meningioma, glioma, sub-
13% of patients with Incontinentia Pige- capsular (pre-senile) cataracts.
menti and 1% of patients with NF 1.
In contrast to NF1, cutaneous lesions
zz Suggested Reading such as cafe au lait spots and neuro-
55 Cross JH. Neuro-­cutaneous Syndromes
16 and Epilepsy –Issues in Diagnosis and
fibromas are uncommon in NF2, but
they may occur. The main cutane-
Management. Epilepsia. 2005;46(Supp ous findings seen in NF2 are cafe au
10);17–23. lait spots and plexiform cutaneous
schwannomas. On the other hand, vari-
?? 11. An 18-year-old female has been eval- ous CNS tumors occur more commonly
uated for left ear hearing loss with in NF2, often in the same patient,
vertigo. You ordered a MRI, which including schwannomas, meningiomas,
shows bilateral vestibular schwanno- ependymomas, and astrocytomas of
mas. Her family history is remarkable the brain and spine. Lisch nodules do
of bilateral vestibular schwannomas not occur in NF2, but sub-­capsular cat-
and multiple meningioma in her aracts and epi-retinal folds may occur.
mother. NF2 is one of the neuro-cutaneous
Neuro-cutaneous Syndromes
475 16
syndromes that is often not diagnosed Which of the following statements is
until adulthood. NF2 is autosomal correct regarding these lesions?
dominant, with variable expression
and complete penetrance. It results
from a mutation in the merlin (also
known as schwannomin) gene on chro-
mosome 22.
Merlin is a tumor suppressor gene,
and mutations in this gene account for the
various neoplasms seen in NF2. A variety
of mutations in the NF2 gene have been
identified, and the type of mutation cor-
relates with clinical severity, with missense
mutations leading to some functional pro-
tein production and milder clinical pheno-
type, and frame shift mutations leading to
more severe disease.
Gorlin syndrome and Rubinstein-
Taybi syndrome are rare syndromes
associated with multiple meningiomas,
but this patient’s history of bilateral ..      Fig. 16.4 Tuberous sclerosis in MRI: Sub-ependy-
schwannomas combined with the family mal nodules; T2 axial. Hellerhoff. Wikipedia
history is suggestive of NF2.
Tuberous sclerosis complex has dis- A. These are all malignant and
tinct cutaneous and CNS findings. should be resected
Schwannomatosis is a distinct disor- B. This patient likely has, in addition to
der characterized by the occurrence of TSC, benign hereditary calcification
multiple schwannomas affecting various of the basal ganglia (Fahr disease)
CNs but not CN VIII. It may be segmen- C. The burden of these lesions cor-
tal, and a pure spinal form also exists. It relates with the severity of mental
may be familial and has in some cases retardation in TSC patients
been associated with a mutation on D. These are calcified sub-­ependymal
chromosome 22 in a gene near, but dif- nodules and are one of the major
ferent from, the gene mutated in NF2; diagnostic criteria for TSC
less than 15% of cases of schwannoma- E. These are malignant sub-ependy-
tosis are hereditary. mal giant cell astrocytomas

zz Suggested Reading vv Correct Answer is: D


55 Comprehensive Review in Clinical There are multiple periventricular hyper-­
Neurology: A Multiple-Choice Question dense lesions. Those are sub-ependymal
Book for the Wards and Boards © 2011 nodules, and their presence is one of the
Wolters Kluwer Health Lippincott major diagnostic criteria of tuberous scle-
Williams & Wilkins. All rights reserved. rosis complex. Sub-ependymal nodules
mainly occur in the periventricular region,
?? 12. You ordered a brain CT scan for a often at the caudo-thalamic groove. They
patient with tuberous sclerosis com- are thought to arise from remnants of the
plex (TSC), and it shows hyperdensi- germinal matrix. They have the potential to
ties, as shown in the figure. grow over time and, in a minority, transform
476 Chapter 16 · Neuro-cutaneous Syndromes

into sub-­ependymal giant cell astrocytoma. hematomas are typically darker in color
Unlike cortical tubers, the presence and and resolve with time.
number of sub-ependymal nodules are not Subungual fibromas occur under the nail
thought to correlate with cognitive function and are also one of the major criteria for TSC.
or seizures. Benign hereditary calcification Angiokeratomas are purplish
of the basal ganglia is on the differential lesions seen in intertriginous regions in
diagnosis of subcortical calcifications, but patients with Fabry disease.
the distribution of these calcifications is
typically within the striatum or thalamus ?? 14. You are evaluating a 12-year-old boy
and they are not as nodular as these sub- for his seizures. On examination, he
ependymal nodules are. There is no associa- has multiple hypo-­pigmented lesions
tion between TSC and Fahr disease. on his trunk, and also lesions on his
face as shown in the figure.
?? 13. The shown skin lesion is characteristic What term best describes the face
in a neuro-cutaneous disorder associ- lesions?
ated with seizures and hamartomas in
multiple body parts.
What is this lesion?

..      Fig. 16.5 Is Courtesy of Dr. David Rothner. Shown


also in color plates. Comprehensive Review in Clinical
Neurology: A Multiple-Choice Question Book for the
Wards and Boards ©2011 Wolters Kluwer Health Lip-
pincott Williams & Wilkins. All rights reserve

16 A. Periungual hematoma
B. Cutaneous neuro-­fibromas
C. Subungual fibroma
D. Periungual fibroma
E. Angiokeratoma

vv Correct Answer is: D


Periungual fibroma is one of the major ..      Figs. 16.6, 16.7 and 16.8 Is Courtesy of Dr. David
Rothner. Shown also in color plates. Comprehensive
criteria the cutaneous findings of
Review in Clinical Neurology: A Multiple-Choice Question
tuberous sclerosis complex (TSC). On Book for the Wards and Boards ©2011 Wolters Kluwer
the other hand, traumatic periungual Health Lippincott Williams & Wilkins. All rights reserve
Neuro-cutaneous Syndromes
477 16
A. Ashleaf spots often become apparent in early child-
B. Cutaneous neurofibromas hood as papules in the malar region and
C. Angiofibromas become more apparent and numerous
D. Shagreen patche with time: they characteristically prog-
E. Severe acne ress to involve the nasolabial folds and
sometimes the chin. Facial angiofibro-
vv Correct Answer is: C mas may resemble acne if not examined
This boy has tuberous sclerosis complex closely and if other historical and clini-
(TSC), because of the combination of cal features are not taken into consid-
the ash leaf spots and the facial angiofi- eration.
bromas. TSC is a neuro-cutaneous disor- Shagreen patches are cutaneous
der that affects multiple organ systems hamartomas that have irregular borders
including the skin, brain, heart, lungs, and are raised. Shagreen patches most
and kidneys and others. The diagnosis often occur on the trunk (back or flank).
of TSC can be made when two of the fol- These lesions may not be present in
lowing major criteria or one major and childhood but may appear later in life.
two minor criteria are present. Hamartomas can occur in a variety
Major criteria: facial angiofibroma of body parts in TSC including the retina
or forehead plaque, periungual, ungual, and gastrointestinal tract.
or subungual fibroma; shagreen patch;
more than three hypomelanotic mac- ?? 15. You are seeing an infant in the ED
ules (including ash leaf spots); retinal with a history of seizures manifest by
hamartomas; cortical tubers; sub-epen- flexor and extensor spasms. On exam-
dymal nodules; sub-­ependymal giant ination, there is a hypo-melanotic
cell astrocytoma, cardiac rhabdomy- macule on his back.
oma, lymphangiomyomatosis, or renal Which of the following syndromes
angiomyolipoma. should be considered?
Minor criteria: dental pits, rectal A. Hypomelanosis of Ito
hamartomatous polyps, bone cysts, B. NF1
radial migration lines in the cerebral C. NF2
white matter, gingival fibromas, non- D. Tuberous sclerosis
renal hamartomas, retinal achromic
patches, confetti skin lesions (hypo-pig- vv Correct Answer is: D
mented, stippled lesions on the extremi- This child has Tuberous Sclerosis (TSC).
ties), and multiple renal cysts. Ash leaf TSC is associated with seizures that
spots are not specific for TSC but occur typically begin in infancy. The flexor/
in the majority of patients with TSC and extensor myoclonus may indicate infan-
are often present at birth but become tile spasms, which are associated with
more obvious with age; in newborns, hypsarrhythmia on EEG. The hypo-mel-
examination under ultraviolet light anotic macule may be shaped like a leaf
makes them more apparent. Tinea cor- and thus has been deemed an “ash leaf
poris, a fungal infection, also leads to spot”; this is the most common lesion
circular lesions on the trunk, but tinea associated with this disease.
has a different appearance.
Facial angiofibromas, also known zz Suggested Reading
as adenoma sebaceum, which are 55 Fenichel GM. Clinical pediatric neurol-
hamartomatous lesions consisting of ogy: a signs and symptoms approach.
vascular and connective tissue. They Philadelphia: WB Saunders; 1997.
478 Chapter 16 · Neuro-cutaneous Syndromes

?? 16. Regarding the phakomatoses, which a protein involved in the nuclear factor
of the following finding is not associ- K-B pathway. Neuro-­cutaneous melano-
ated with the specified disorder? sis is characterized by the presence of
A. Hyper-pigmented cutaneous various types of congenital cutaneous
lesions and leptomeningeal lesions that are abnormally pigmented
melanoma­ neuro-cutaneous (such as giant hair pigmented nevi and
melanosis congenital melanocytic nevi) in associa-
B. Hemi-facial atrophy-­Parry-­ tion with leptomeningeal melanoma.
Romberg syndrome The leptomeningeal areas most often
C. Multiple endochondromas and affected include those around the
secondary hemangiomas-­Maffucci base of the brain, brain stem, and cer-
syndrome ebellum. The pathophysiology of this
D. Hypo-pigmented streaks or ­disorder is not well defined; the cells of
patches that follow skin lines- origin of the leptomeningeal melano-
incontinentia pigmenti mas are thought to be melanoblasts,
E. Retinal, cerebellar, and spinal pigmented cells normally found in the
hemangioblastomas-von Hippel- pia mater.
Lindau disease Parry-Romberg syndrome is marked
by the occurrence of facial atrophy,
vv Correct Answer is: D which involves atrophy of facial bone,
Hypo-pigmented streaks or patches cartilage, and soft tissue, often with
that follow skin lines occur in Hypomel- ipsilateral loss of eyelashes, eyebrows,
anosis of Ito, not in incontinentia pig- and scalp hair. This begins typically
menti. The phakomatoses are a group after birth or in early childhood and the
of disorders that share in common the atrophy ceases by the third decade of
occurrence of dysplastic lesions and life. Neurologic manifestations include
the tendency for tumor formation. They headaches, Horner syndrome, seizures,
include neurofibromatosis, tuberous and hemiparesis. Patients with Parry-
sclerosis, Sturge-Weber syndrome, epi- Romberg syndrome are at increased risk
dermal nevus syndrome, in addition to a for a variety of benign tumors.
variety of other rare disorders. In incon- In Maffucci syndrome, multiple
tinentia pigmenti, skin involvement endochondromas (tumors of cartilage)
occurs in stages including vesiculobul- occur, in association with secondary
lous lesions present at birth, verrucous hemangioma formation, and various
lesions that appear at approximately skin findings including vitiligo and cafe
6 weeks of age, then hyper pigmented au lait spots. These endochondromas
16 lesions that appear “splashed-on.” Some
patients have normal cognition and
grow over time, leading to disfigure-
ment and skeletal abnormalities.
no evidence of neurologic dysfunc- Neurologic manifestations result from
tion; neurologic manifestations include the association of this syndrome with
intellectual disability, pyramidal tract various CNS tumors, including CNS tera-
findings, and ocular abnormalities. It is tomas and pituitary adenomas, as well
X-linked dominant in inheritance and as compression of nervous system struc-
affects only females; it is thought to be tures by the endochondromas, such as
lethal in males. It results from a muta- cerebral compression by calvarial endo-
tion in the NEMO gene, which encodes chondromas.
Neuro-cutaneous Syndromes
479 16
In von Hippel-Lindau disease, angiomas are termed “port wine stains,”
multiple retinal, cerebellar, and spinal and typically occur in a V1 distribution.
hemangioblastomas occur. Benign Glaucoma is a possible complication
hemangiomas and cysts in various when angiomas involve the choroid.
body parts can also occur. Cutaneous Intellectual disability is evident in 70%
manifestations are not a feature of this of patients, and seizures are noted in
disorder. This disorder is autosomal the majority of cases. Seizures are most
dominant and results from a mutation in often focal motor or generalized tonic-
a gene on chromosome 3 that encodes clonic initially, although other types
for a tumor suppressor protein. may develop. In addition to angiomas,
imaging may reveal cerebral atrophy,
zz Suggested Reading hydrocephalus, and tram tracking gyral
55 Comprehensive Review in Clinical calcifications. Sturge-Weber syndrome
Neurology: A Multiple-Choice Question was believed to occur sporadically,
Book for the Wards and Boards © 2011 although a responsible gene mutation
Wolters Kluwer Health Lippincott has recently been identified on chro-
Williams & Wilkins. All rights reserved. mosome 9 (GNAQ). The sudden onset
of weakness may reflect hemorrhage
?? 17. A 14-year-old female is brought to the of an angioma (a rare occurrence) or a
ED because a sudden onset of weak- post-­ictal Todd’s paralysis. Weakness
ness of the right face and arm. She often develops in the setting of the ini-
has a history of a focal motor seizures tial flurry of seizures, although unlike
and intellectual disability. On exam, a typical Todd’s paralysis, it may be
she has a port wine stain in the left long-lasting or even permanent. Other
V1 distribution, and she has weak- patients have TIA or stroke-­like episodes
ness of the right face and arm. CT was unrelated to seizure activity. Although
ordered. initially progressive, the condition typi-
This disorder may be associated with cally stabilizes over time.
which finding(s)?
A. Meningeal angiomas zz Suggested Reading
B. Hydrocephalus 55 Santos CC, et al. Neuro-cutaneous
C. Tram tracking calcifications syndromes. In: Bradley WG, Daroff
D. lntracerebral parenchymal angio- RB, Fenichel GM, Jankovic J, editors.
mas Neurology in Clinical Practice. 4th ed.
E. All of the above Elsevier Inc.; 2004. p. 1867–900.

vv Correct Answer is: E ?? 18. A 10-year-old boy with a history of


This is a presentation of Sturge-Weber seizures and headache came to your
syndrome. Findings are variable, and office for an evaluation. You ordered a
may include angiomas of the face, brain MRI, which is shown. The lesion
meninges and brain (typically ipsilat- shown in the MRI is resected and
eral to the facial lesion, often involving pathologic analysis is consistent with
the occipital region), with contralateral sub-ependymal giant cell astrocy-
weakness and atrophy, and hom- toma.
onymous hemianopia. The cutaneous What is the most likely diagnosis?
480 Chapter 16 · Neuro-cutaneous Syndromes

?? 19. Which of the following statements is


incorrect regarding the genetics of
tuberous sclerosis complex (TSC)?
A. It is autosomal recessive with
complete penetrance
B. It is autosomal dominant with
variable penetrance
C. It can be caused by a mutation in
the TSC1 gene that encodes for
the protein hamartin
D. It can be caused by a mutation in
the TSC2 gene that encodes for
the protein tuberin
E. The presence of specific clinical
features does not reliably dis-
tinguish between the different
genetic mutations that can cause
this condition

vv Correct Answer is: A


Tuberous sclerosis complex (TSC) is an
..      Fig. 16.9 ILAE.org. EpilepsyDiagnosis.org,
Diagnostic Manual autosomal dominant disorder with vari-
able penetrance. TSC may be either inher-
A. Tuberous sclerosis complex ited or more commonly sporadic. TSC is
B. Neurofibromatosis type 1 (NF1) caused either by a mutation in the TSC1
C. Neurofibromatosis type 2 (NF2) gene on chromosome 9 that encodes for
D. Sturge-Weber syndrome the protein hamartin or by a mutation in
E. He probably does not have a the TSC2 gene on chromosome 16 that
neuro-cutaneous disorder; this encodes for the protein tuberin.
tumor is most often seen sporadi- Tuberin and hamartin interact with
cally each other and normally function as
tumor suppressor genes, and abnormali-
vv Correct Answer is: A ties in these proteins lead to unregulated
MRI shows a sub-ependymal giant cell cell growth and proliferation. Although
astrocytoma (SEGA), which is an uncom- there are some genotype phenotype
mon tumor, but it is seen almost exclu- correlations, the clinical manifestations
16 sively in patients with tuberous sclerosis of each mutation overlap significantly,
so the two mutations cannot be distin-
complex (TSC) and is a major diagnostic
criterion for TSC. It is a benign, low- guished between reliably on the basis of
grade astrocytoma but leads to symp- clinical features alone.
toms due to mass effect and ventricular
obstruction. Surgery is usually curative. zz Suggested Reading
Rapamycin may be of benefit in the 55 Comprehensive Review in Clinical
treatment of SEGA. SEGA does not typi- Neurology: A Multiple-Choice Question
cally occur in the other neuro-cutane- Book for the Wards and Boards ©2011
ous disorders described. Wolters Kluwer Health Lippincott
Williams & Wilkins. All rights reserved.
Neuro-cutaneous Syndromes
481 16
?? 20. The findings shown in the figures of the leptomeninges and the brain.
are consistent with which neuro- The manifestation of Sturge-Weber syn-
cutaneous syndrome? Fig. (A) Cour- drome includes cutaneous angioma of
tesy of Dr. David Rothner; (B) Coronal the face (port-wine nevus), which often
T1-weighted pre-contrast MRI. occurs in a trigeminal distribution, but
What is your diagnosis? may involve any part of the body. Asso-
ciated features in some cases include
angiomatosis of the ipsilateral and less
commonly bilateral leptomeninges
and the cortex. Some patients have
only cutaneous findings without CNS
involvement; this is most often the case
if cutaneous angiomas are present only
in the limbs (without facial involve-
ment). Neurologic manifestations are
variable; some patients may have no
neurologic signs or symptoms, whereas
others may have seizures, contralateral
hemiparesis, and/or developmental
delay. CNS involvement is most com-
mon in those with cutaneous angiomas
involving the face. Cobb syndrome, or
cutaneomeningospinal angiomatosis,
is a variant of Sturge­Weber syndrome
(SWS) in which cutaneous angiomas
occur in a dermatome corresponding
to spinal dural angioma. Glaucoma may
be a complication of Sturge-Weber syn-
drome but pre-senile cataracts are not.
The pathophysiology of SWS is thought
to relate to persistence of embryonal
..      Figs. 16.10 and 16.11 Coronal T1-weighted pre- blood vessels that normally regress
contrast MRI. (Courtesy of Dr. David Rothner) during gestation; it is considered a
congenital malformation rather than
a genetic disorder and is not heredi-
A. Neuro cutaneous melanosis
tary. The classic radiographic finding
B. lncontinentia pigmenti
is one of gyral calcifications giving a
C. Hypomelanosis of Ito
tram-track appearance that may be ini-
D. Sturge-Weber syndrome
tially best seen on a CT scan. Cerebral
E. Epidermal nevus syndrome
hemiatrophy, rather than hemimega-
lencephaly, is also seen. MRA is useful
vv Correct Answer is: D
in assessing the extent of intracranial
The findings shown are indicative of
involvement. Cerebral venous throm-
Sturge-Weber syndrome, in which gyral
bosis may occur uncommonly.
calcifications result from angiomatosis
482 Chapter 16 · Neuro-cutaneous Syndromes

?? 21. You are examining a 7-year-old boy,


you found out that he has more than
six hyper pigmented lesions.
What term best describes these
lesions?

..      Fig. 16.14 The patient was a 14-year-old male


with pulsating exophthalmos of the left eye and poor
vision of this highly myopic left eye. There was also
some ptosis of the left eye with mild puffiness. Upward
gaze of the left eye was limited. Slit lamp examination
showed multiple, slightly raised nodules of the right iris.
National Eye Institute. National Eye Institute. Wikipedia

..      Fig. 16.12 Patient with multiple small cutaneous A. Ash leaf spots
neurofibromas and a “café au lait spot” (bottom of B. Cutaneous neuro-­fibroma
photo, to the right of centre). A biopsy has been taken C. Plexiform neuro- fibroma
of one of the neurofibromas. Haymanj. Wikipedia
D. Shagreen patches
E. Cafe au lait spots

vv Correct Answer is: E


The cutaneous findings shown in Figures
are seen in neurofibromatosis type 1 (NF1),
or von Recklinghausen disease. NF1 is a
neuro-cutaneous disorder that involves
multiple organs including the skin, brain,
eyes, and bones. The hyperpigmented
16 macules seen are known as cafe au lait
spots. Axillary freckling is seen in Fig. 1,
and cutaneous neuro-fibromas are seen in
Fig. 2. To make a diagnosis of NF1, two or
more of the following must be present:
55 Six or more cafe au lait macules mea-
suring more than 5 mm in diameter
in pre-pubertal children or more than
15 mm in diameter post-puberty.
..      Fig. 16.13 Neurofibromi cutanei sulla schiena di
un individuo affetto da neurofibromatosi di tipo 1. 55 Two or more cutaneous neuro-fibro-
Article of MedMedicine.it. Unknown. Wikipedia mas or one plexiform neuro-fibroma.
Neuro-cutaneous Syndromes
483 16
55 Inguinal or axillary freckling vv Correct Answer is: C
55 Optic nerve gliomas. Hypo-melanosis of Ito (HI) is a neuro-­
55 Two or more Lisch nodules. cutaneous disorder that involves
55 NF1 diagnosed in a first-degree relative. multiple organ systems including
55 Sphenoid wing dysplasia, pseudo- the skin, eyes, brain, and skeleton.
arthrosis, or cortical thinning of long The cutaneous features of HI include
bones. multiple hypo-pigmented streaks or
patches that are present at birth and
Cafe au lait spots may be seen in local- in some follow Blaschke lines (skin
ized, segmental form in isolation in the lines that form specific patterns over
setting of post-somatic mutations in the the trunk and extremities, such as a V
NF1 gene. In contrast to cafe au lait spots, shape over the back and linear lines
ash leaf spots are hypo-pigmented and over the limbs). Skin lesions are pres-
are seen in tuberous sclerosis complex. ent at birth or appear in infancy and
Shagreen patches are connective tissue are best detected by ultraviolet light
hamartomas also seen in tuberous scle- in light-­skinned children. The severity
rosis. The other disorders listed are also of the skin lesions does not correlate
neuro-cutaneous disorders, collectively strongly with neurologic involvement.
known as the phakomatoses. Intellectual disability as neurologi-
Unlike NF1, neurofibromatosis type cal finding is seen in some but not all
2 (NF2) is not diagnosed by the pres- patients, and may be severe in some
ence of specific cutaneous findings but cases. Seizures are another major neu-
rather by other criteria. rologic manifestation. Macrocephaly
Sturge-Weber syndrome is marked or microcephaly may be seen, with the
by the presence of hemangiomas. former being more common. Cerebral
TSC and epidermal nevus syndrome and cerebellar hypoplasia are also
have specific cutaneous findings that often seen, although various malfor-
differ from NF1. mations of cortical development may
occur in patients with HI, including
?? 22. A 5-month-old boy is referred to hemimegalencephaly, lissencephaly,
you for an evaluation of his hypo- and polymicrogyria. Eye involvement
pigmented streaks on his skin that (microphthalmia, cataracts, optic atro-
occurred in a V-shape on his back, phy, and retinal detachment) are com-
and in a linear pattern on his legs, fol- mon findings. Other manifestations
lowing the skin lines since birth. He include, skeletal hemi-hypertrophy,
has history of seizures. On ophthal- cleft lip and palate, and congenital
mologic exam, he has cataracts. Head heart disease such as tetralogy of Fal-
circumference was 2 standard devia- lot. Various karyotype abnormalities
tions above the mean. have been found in patients with HI,
What is your most likely diagnosis? including autosomal mosaicism for
A. Neuro-cutaneous melanosis aneuploidy or unbalanced transloca-
B. lncontinentia pigmenti tions, mosaic trisomy 18, ring chromo-
C. Hypomelanosis of Ito some 22, and translocations involving
D. Sturge-Weber syndrome the X chromosome. There is not a clear
E. Epidermal nevus syndrome genotypic-phenotypic correlation.
484 Chapter 16 · Neuro-cutaneous Syndromes

?? 23. A brain MRI of a 17-year-old male with vv Correct Answer is: C


neurofibromatosis type complaining Schwannomas and ependymomas are
of diplopia, shows abnormalities in rare in neurofibromatosis type 1 (NF1);
the left optic nerve and in the pons. they more commonly occur in neurofi-
Which of the following statements is bromatosis type 2 (NF2). Thickening of
incorrect regarding these abnormalities? the left optic nerve represents an optic
nerve glioma. Optic nerve gliomas are
the most common tumor of the CNS
seen in NF1 patients and may be uni-
lateral or bilateral. These are often low-
grade lesions but may cause symptoms
due to mass effect, including diplopia,
pain, and proptosis. Optic pathway
gliomas can occur anywhere along the
optic pathways, from the optic nerve
to the optic radiations. Optic chiasm
gliomas may present with precocious
puberty when they lead to pressure on
diencephalic structures. Because these
lesions are often benign, serial imaging
over time is often used to monitor these
tumors, with chemotherapy, radiation,
or surgery instituted as necessary.
Nonspecific subcortical hyper-inten-
sities (in the basal ganglia, thalamus,
and other regions) are common in NF1
patients and are of unclear etiology and
significance. Figure shows enlargement
of the pons, which likely represents a
low-grade glioma. NF1 patients are at an
increased risk of cerebral, cerebellar, and
brain stem astrocytomas. NF1 patients
are also at an increased risk for leukemia.
..      Figs. 16.15 and 16.16 Optic nerve sheath menin-
gioma (thick arrows) and optic nerve (thin arrow). ?? 24. Which of the f­ ollowing statements is
Glioma of optic nerve. 7 https://openi.nlm.nih.gov/
incorrect regarding the genetics and
detailedresult.php?img=2765171_IJRI-19-135-g010&
16 query=optic+nerve+glioma&it=xg&lic=by&req=4&n
pos=1. Chavhan G. B., Shroff M. M.W.
molecular biology of neurofibromatosis
type 1 (NF1, van Recklinghausen disease)?
A. NF1 results from a mutation in the
A. He likely has an optic nerve glioma neurofibromin gene on chromo-
B. Optic nerve gliomas are common in some 17
NF1 B. Neurofibromin activates a GTPase
C. Schwannomas and ependymomas that inhibits the ras proto-onco-
are the most common tumors gene, a protein involved in cell
occurring in NF1 patients proliferation.
D. Optic nerve gliomas in patients C. Several mutations in the NF1 gene
with NF1 typically are low grade have been identified, but in gen-
and often can be monitored for eral, strong phenotype genotype
years without intervention correlations do not occur
Neuro-cutaneous Syndromes
485 16
D. NF1 is an autosomal recessive dis- TS, although may not be evident in
order childhood. Koenen tumors are ungual
E. NF1 has complete penetrance but fibromas, which are nodular or fleshy
variable expressivity lesions under or next to the nails. They
commonly appear in patients with TS
vv Correct Answer is: D during puberty, and are seen in 15–20%
Neurofibromatosis type 1 (NF1) is an of patients.
autosomal dominant disorder with
markedly variable expression but zz Suggested Reading
complete penetrance. It results from a 55 Santos CC, et al. Neurocutaneous
mutation in the neurofibromin gene on syndromes. In: Bradley WG, Daroff
chromosome 17. Approximately half of RB, Fenichel GM, Jankovic J, editors.
cases are sporadic. Neurofibromin is a Neurology in Clinical Practice. 4th ed.
tumor suppressor protein that normally Elsevier Inc.; 2004. p. 1867–900.
activates a GTPase that inhibits ras, a
proto-­oncogene involved in cell prolif- ?? 26. You are seeing a 16-year-old male
eration. More than 100 mutations in the with a history of moderate intellectual
NF1 gene have been identified, but no disability and new-onset generalized
specific genotype-­phenotype correla- tonic-clonic seizures. You ordered an
tions occur. MRI, which is shown.
What are you expecting to see on his
?? 25. Which is not a common cutaneous physical examination?
presentation of tuberous sclerosis A. adenoma sebaceum
(TS)? B. bilateral hearing loss
A. Ash-leaf hypo-­pigmented macules C. café au lait spots
B. 1-3 mm stippled hypo-­pigmented D. facial port wine stain
areas on the extremities E. Lisch nodules
C. iris hamartomas
D. sub-epidermal fibrous patches
E. ungual fibromas

vv Correct Answer is: C


Up to 90% of patients with TS have Ash-­
leaf hypo-pigmented macules. They are
typically present at birth, but difficult to
visualize, requiring the use of ultraviolet
light. Confetti macules are evident in
patients with TS, and consist of 1-3 mm
stippled hypo-pigmented areas on
the extremities. Lisch nodules are iris
hamartomas, common in neuro-fibro-
matosis type 1. Shagreen patches are
sub-epidermal fibrous patches, which
have an “orange-peel” appearance.
They are typically 1–10 cm, irregularly
shaped, flat or slightly raised, flesh­col-
ored plaques, most often located on the
back or flank in patients with TS. They ..      Fig. 16.17 ILAE.org. EpilepsyDiagnosis.org, Diag-
are present in 20–30% of patients with nostic Manual
486 Chapter 16 · Neuro-cutaneous Syndromes

vv Correct Answer is: A and others are age-related manifesta-


There are multiple areas of T2 signal tions. A National Institutes of Health
abnormality that are most consistent (NIH) Consensus Development Confer-
with cortical tubers. Also, there is a ence (2) regarding NF1 demarcated the
sub-­ependymal giant cell astrocytoma following 7 features, of which 2 or more
(SEGA) on the right side periventricular. are required to establish the diagnosis
The MRI findings suggest a diagnosis of NF1:
of tuberous sclerosis. Adenoma seba- 1. Six or more cafe-au-lait spots (CLSs)
ceum is a common cutaneous feature equal to or greater than 5 mm in
associated with tuberous sclerosis. longest diameter in pre-pubertal
Other physical exam features of tuber- patients and 15 mm in longest
ous sclerosis include periungual fibro- diameter in post- pubertal patients.
mas, hypomelanotic macules, gingival 2. Two or more neurofibromas of any
fibromas, shagreen patches, randomly type or 1 plexiform neurofibroma.
distributed pits in dental enamel, achro- 3. Freckling in the axillary or inguinal
matic retinal patches, and “confetti” skin regions.
lesions. 4. Optic glioma (optic pathway glioma).
Café au lait spots and Lisch nodules 5. Two or more Lisch nodules (iris
are features of neurofibromatosis type I. hamartomas).
Facial port wine stains are a feature 6. A distinctive osseous lesion, such as
of Sturge-Weber syndrome. sphenoid wing dysplasia or cortical
Bilateral hearing loss may be seen in thinning of the cortex of long bones,
many conditions, but is often a feature with or without pseudoarthrosis; and
of neurofibromatosis type II. 7. A first-degree relative (parent, sibling,
or child) with NF1 according to the
zz Suggested Reading aforementioned criteria.
55 Roach ES, Sparagana SP. Diagnosis of
tuberous sclerosis complex. J Child Neurol In addition, although areas of increased
2004; 19(9):643–49. T2 signal intensity are commonly
identified on MRI of the brain, they do
?? 27. Which of the following are not included not represent an obligatory feature
in the required features of neurofibro- of NF. Therefore, the NIH Consensus
matosis type 1 (NF1) as established by Development Conference did not rec-
the NIH, of which two are needed to ommend routine neuroimaging of the
make a clinical diagnosis: brain as a means of establishing a diag-
A. Optic pathway glioma nosis of NF1.”
16 B. Distinctive osseous lesion such as
sphenoid wing dysplasia or corti- zz Suggested Reading
cal thinning of the cortex of long 55 Joseph H. Hersh and Committee on
bones Genetics. Health Supervision for Children
C. First degree relative with NF1 with Neurofibromatosis. Pediatrics.
D. Increased T2 signal intensity on 2008;121:633. 7 https://doi.org/10.1542/
MRI of the brain peds.2007-3364.

vv Correct Answer is: D ?? 28. What would be the most other abnor-
“NF1 is a multisystem disorder in which mal finding in the 16-year-old male
some features may be present at birth whose brain MRI is shown?
Neuro-cutaneous Syndromes
487 16

..      Fig. 16.18 American Academy of Neurology Institute, produced by permission

A. Axillary freckling
B. Café-au-lait spots
C. Cataracts
D. Iris hamartomas
E. Scoliosis

vv Correct Answer is: C


The brain MRI shows vestibular schwan-
nomas, which are the hallmark of
neurofibromatosis type 2. Another
common feature of neurofibromatosis
type 2 are juvenile cataracts. The other
options are typical of neurofibromatosis
type 1 but are rarely present in neurofi-
bromatosis type 2.

zz Suggested Reading
55 Hilton DA, Hanemann CO. Schwannomas
and their pathogenesis. Brain Pathol
..      Fig. 16.19 STURGE WEBER SYNDROME. ILAE.org.
2014;24(3):205–20. EpilepsyDiagnosis.org, Diagnostic Manual
55 Lloyd SK, Evans DG. Neurofibromatosis
type 2 (NF2): diagnosis and management. What feature, if present, would be
Handb Clin Neurol 2013;115:957–67. most specific in making a definitive
55 Evans DG. Neurofibromatosis type 2 diagnosis?
(NF2): a clinical and molecular review. A. Autosomal recessive inheritance
Orphanet J Rare Dis. 2009;4:16. 7 https:// pattern
doi.org/10.1186/1750-1172-4-16. B. Facial capillary malformation
C. Failure to meet developmental mile-
?? 29. An 8-year-old boy came with his fam- stones in the first few months of life
ily to your office for evaluation. The D. Retinal astrocytoma on dilated
family brought a previous MRI to the eye exam
visit, shown below. E. Seizures in the first year of life
488 Chapter 16 · Neuro-cutaneous Syndromes

vv Correct Answer is: B 30–70% of patients with SWS. Glaucoma


The MRI shows right parieto-occipital may be congenital or develop later in
pial enhancement, gyral calcifications, life. In the rare patient without facial
and atrophy consistent with leptomen- capillary malformation, the presence
ingeal capillary-­venous malformation. of glaucoma and typical MRI pattern is
The presence of facial capillary malfor- highly suggestive of SWS. Retinal astro-
mation (a port wine stain) in the 1st or cytoma is seen in patients with tuberous
2nd division of the trigeminal nerve sclerosis, not SWS.
in this setting would be diagnostic of Though seizures and developmental
Sturge-Weber syndrome (SWS). This delay occur commonly in patients with
neuro-cutaneous finding is typically SWS, (with 95% having developed sei-
ipsilateral to the cerebral vascular mal- zures by age five), neither are specific
formation. An autosomal dominant to SWS and may occur in a multitude of
inheritance pattern would be atypical. congenital and developmental diseases.
SWS, although congenital, is a sporadic
disease and so unlikely to occur in sib- zz Suggested Reading
lings or offspring of affected individuals. 55 Thomas-Sohl KA, Vaslow DF, Maria
Glaucoma is a common but incon- BL. Sturge-Weber syndrome: a review.
sistent feature of SWS, occurring in Pediatr Neurol. 2004;30(5):303–10.

16
489 17

Neurologic Complications
of Systemic Disease

© Springer International Publishing AG, part of Springer Nature 2018


Y. M. Awaad, Absolute Pediatric Neurology, https://doi.org/10.1007/978-3-319-78801-2_17
490 Chapter 17 · Neurologic Complications of Systemic Disease

?? 1. The following scenario is part of a with focal neurological changes, facial


two-question series, Discussion & Ref- weakness, incontinence and focal sensory
erences included at the end of series. changes, but if present should trigger
You are evaluating a 16-year-old further evaluation. It is very important
female with a history of increasing to consider the possibility of elevated
headache for the last 2 months. The increased intracranial pressure in patients
headache is episodic pounding but has with abnormal thyroid status, although
become daily. She reported increasing the ICP typically returns to normal after
of her weight by 5 Ib., and she is abnor- treatment of the hyperthyroidism.
mally sweaty. On exam she has mild
proximal muscle weakness, hyperre- zz Suggested Reading
flexia and a new intention tremor. 55 Author, Stephen La Franchi, MD, Section
What lab test are you going to order Editor, Mitchell Geffner, MD. Deputy
to reach the diagnosis? Editor, Alison G. Hoppin, MD.Up-To-Date
A. Creatine Kinase Clinical manifestations and diagnosis of
B. Electrolytes hyperthyroidism in children and adoles-
C. Tissue Transglutaminase IgA cents Accessed 12/8/2014.
D. Thyroid Hormone
E. Bun/Cr ?? 3. You are seeing a 16-year-old female
in the ICU because of her unrespon-
vv Correct Answer is: D siveness for 5 h after having a seizure.
One week ago, she had her first
?? 2. The following scenario is question generalized tonic-clonic seizure, she
two of the two question series, Dis- was seen in the ER, had a normal CT
cussion & References included at the scan of the brain, and was discharged
end of this series. home. Her family reported that she
What other finding on your exam can has increasing unsteadiness in her
be associated with this diagnosis? gait for 2 days before admission. Her
A. Urinary Incontinence medical history is unremarkable and
B. Papilledema she is not taken any medications.
C. Hemiplegia Family history is remarkable for type
D. Distal Sensory Loss I diabetes in her mother and hypo-
E. Facial Weakness thyroidism in a maternal aunt. On
examination, she is minimally respon-
vv Correct Answer is: B sive to sternal rub. Vital signs were
Increasing headache without other temperature 98.8 F, pulse 88/min., BP
manifestations may be associated 110/70, respiratory rate 12/min. and
with hyperthyroidism, but may also be oxygen saturation 99% on 2 L nasal
associated with intracranial hyperten- cannula. Pupils were 4 mm and reac-
17 sion or rarely sinus venous thrombosis. tive, no gaze preference was noted.
Hyperthyroidism or Grave’s Disease Corneal and gag reflexes were pres-
patients can compline of acceleration ent. DTRs are hyperactive throughout.
of linear growth, weight loss or poor Basic laboratory studies, including a
weight gain, tachycardia, feeling warm, CBC, comprehensive metabolic pro-
and increased sweating. Neurological file, serum lactate and ammonia were
symptoms can include headache, tremor, normal. An EEG showed diffuse back-
behavior or mood changes and rarely ground slowing with no epileptiform
ataxia. Hyperthyroidism is not associated discharges. Brain MRI was normal.
Neurologic Complications of Systemic Disease
491 17
Lumbar puncture showed 6 WBC/ cause is Hashimoto’s encephalopathy. In
mm3, normal glucose and protein spite it is an adult disorder, many cases of
60 mg/dL. Thyroid function tests were Hashimoto’s encephalopathy have been
normal. Anti-microsomal antibodies reported in children, usually in females
were markedly elevated. over the age of 10 years. Although many
What is the best treatment interven- neurologic symptoms may be present,
tions being most likely to produce alterations in consciousness, seizures and
improvement in her case? status epilepticus have been the most
A. IV acyclovir commonly reported in pediatric patients.
B. IV fosphenytoin Patients themselves are usually euthyroid
C. IV solumedrol or mildly hypothyroid but have markedly
D. IV thiamine elevated titers of anti-microsomal (and
E. Oral thyroid replacement less commonly anti-thyroglobulin) anti-
bodies. The exact pathophysiologic role
vv Correct Answer is: C of these antibodies has not been clearly
The patient has subacute encephalopathy. understood, most likely they represent an
The list of differential diagnosis is long immunologic marker for an as yet uniden-
and has many disorders. Toxic or traumatic tified autoimmune mechanism. Although
etiologies have to be considered from her replacement of thyroid hormones is
history. Treatable or modifiable condi- indicated for long term treatment of clini-
tions should be considered, including cally hypothyroid individuals, this thyroid
hypoglycemia, hyponatremia, uremia and replacement has no impact on the neu-
hyper-­ammonia, which respond quickly to rologic features of the disorder. Immune
treatment. Structural lesions of the brain, suppressive therapy, most commonly with
such as tumors or vascular insults should oral or intravenous steroids has resulted
be considered, though would be more in rapid clinical improvement. The dura-
likely to present with focal neurologic tion of immune suppressive therapy is
findings in addition to the encephalopa- variable, although most patients are able
thy. Infectious causes should always be to taper off medication within months.
considered, though would be unlikely The diagnosis requires a high index of
without fever and CSF pleocytosis. Acute suspicion. Ancillary tests other than the
disseminated encephalomyelitis is a fairly antibodies mentioned above are nonspe-
common cause of subacute encepha- cific. EEG typically reveals only slowing
lopathy in childhood, but it usually has of the background, a majority of patients
radiological findings in the cerebral white have normal neuroimaging studies and
and/or deep gray matter. Non-­convulsive cerebrospinal fluid shows a normal cell
status epilepticus should be considered in count with a modestly elevated protein in
a patient with a history of seizure, though most patients.
there was no evidence in this case based
on the EEG findings. Nutritional deficien- zz Suggested Reading
cies would be a rare cause in a previously 55 Watemberg, et al. Encephalopathy
healthy child, though thiamine deficiency Associated with Hashimoto Thyroiditis: a
should be considered with a history of pediatric perspective In J. Child Neurol.
ophthalmoparesis, ataxia and alteration in 2006;21(1):1–5.
mental status. Rarer diagnostic possibili-
ties should be considered if the etiology ?? 4. You were asked to see a 16-month-
remains undefined despite an in-depth old girl with a history of 4 weeks of
search for the above conditions. One such fussiness and repeated falling. Her
492 Chapter 17 · Neurologic Complications of Systemic Disease

past medical history is remarkable of ?? 5. A 6-year-old boy came to the ED


a viral illness 2 weeks ago and her PCP after having 2 clonic seizures involv-
diagnosed her with acute cerebel- ing his left arm and leg, 15 min each.
lar ataxia. Her exam at that time was His past medical history is unremark-
significant for mild ataxia but she able. Many of his family members
was able to walk on discharge. Her were ill with a gastrointestinal illness
brain MRI was normal. Today, she has few weeks ago. On examination,
had worsening of her symptoms; she his blood pressure is 140/94, he is
is not able to stand and cannot sit afebrile and pulse is 140/min. He
without support. Her family noted a has weakness and decreased tone
tremor when she reaches for objects of his left arm greater than leg and
and jerky eye movements. an extensor plantar response on the
What test from below list would left side. His labs are, hemoglobin
change her management? of 7.6, hematocrit of 25 and plate-
A. Serum VMA/HVA let count of 25,000. The peripheral
B. Scan of her torso smear shows multiple Schistocytes.
C. EMG/Nerve Conduction study Serum sodium is 134, bicarbonate
D. Testing for autoantibodies 20, glucose 165, BUN 40, creatinine
E. CSF Neurotransmitters 3.9. A non-­contrast head CT scan is
unremarkable.
vv Correct Answer is: B What is the most likely diagnosis for
This is the clinical presentation of opsoc- this patient?
lonus myoclonus syndrome (OMS) and A. Hemolytic-uremic syndrome
search for underlying neuroblastoma is B. Hypertensive ­encephalopathy
essential. Neither serum VMA/HVA nor C. Hyponatremia
urine VMA/HVA are sensitive enough to D. Kawasaki syndrome
rule out or detect an underlying neuro- E. Sickle cell vasculopathy
blastoma in patients with OMS. Finding
an underlying neuroblastoma is critical vv Correct Answer is: A
as 48% of children with OMS. If there is This patient has symptomatic focal
no neuroblastoma on CT or MRI of the seizures secondary to a recent ischemic
torso, one could consider MIBG scan, and/ stroke. Hyponatremia is a cause of symp-
or follow up scan several months after tomatic seizures, though not secondary
the initial normal scan. If neuroblastoma to stroke. Typically, serum sodium level
is detected it should be removed and need to be less than 120–125 meq/L to
treated appropriately but treatment of the produce seizures. The remainder of the
OMS with immunomodulatory therapy choices listed above are potential causes
(ACTH, IVIG etc.) is also critical. of seizures and ischemic stroke, though
hypertensive encephalopathy usually
17 zz Suggested Reading produces bilaterally symmetric abnormali-
55 Gorman MP. Update on diagnosis, ties on imaging, especially in the more
treatment, and prognosis in opsoclonus- posterior brain regions. Focal infarctions
myoclonus-ataxia syndrome. Curr Opin respecting vascular distributions are not
Pediatr. 2010;22(6):745–50. commonly seen. The constellation of
Neurologic Complications of Systemic Disease
493 17
hemolytic anemia, acute renal failure and and anorexia, and was diagnosed
thrombocytopenia is most suggestive of with acute hepatitis secondary to
hemolytic-uremic syndrome. Although non-A, non-B hepatitis. She has not
sickle cell disease can cause anemia with responded well to supportive care,
seizures secondary to ischemic stroke, and has developed a significant coag-
sickle cells and not schistocytes (red ulopathy, requiring treatment with
blood cells that have been fragmented by cryoprecipitate and recombinant Fac-
passage through damaged small blood tor VII (rFVII). You examined her and
vessels) would be noted on the peripheral she is able to open her eyes to voice,
smear. Hemolytic-­uremic syndrome is but cries in response to questions,
most commonly triggered by a diarrheal and will not respond verbally. Her
associated illness, often shiga-toxin pro- neck is supple and her optic disc mar-
ducing strains of E. coli. The toxins have gins are blurred. There is no tremor or
a direct effect on the kidney, causing asterixis. Her parents stated that she
injury to the sub-endothelial capillary has been confused for the last 2 days.
spaces, leading to acute renal failure. What is the most likely neurologi-
Several other infectious, immunologic, cal complication from her systemic
familial and endocrine causes have been disease?
reported. Neurologic manifestations are A. Obstructive hydrocephalus
seen in 30–40% of cases, and are most B. Large-vessel ischemic stroke
commonly a combination of behavioral C. Cerebral edema with compro-
changes, motor seizures, stroke and mised cerebral perfusion
varying degrees of encephalopathy. D. Coma secondary to elevated
Treatment is mainly supportive, with serum carnitine levels
renal dialysis required in 30–50% of cases. E. Hyperglycemia
Plasmapheresis has been utilized in some
cases. vv Correct Answer is: C
Acute hepatic failure in children is
zz Suggested Reading uncommon. In infants, it is more likely
55 Ariceta G, Besbas N, Johnson S, Karpman to be associated with an inborn error
D, Landau D, Licht C, Loirat C, Pecoraro of metabolism (such as tyrosinemia),
C, Taylor CM, Van de Kar N, Vandewalle J, whereas in older children and ado-
Zimmerhackl LB; European Paediatric lescents it is more often due to viral
Study Group for HUS. Guideline for the hepatitis or drug exposure (such as
investigation and initial therapy of acetaminophen). Wilson’s disease should
diarrhea-negative hemolytic uremic also be considered in older children.
syndrome. Pediatr Nephrol. Children with encephalopathy associ-
2009;24(4):687–96. Epub 2008 Sep 18. ated with acute hepatic failure may not
have asterixis or tremor. The presence of
?? 6. You are asked to see a 9-year-old severe coagulopathy or coma is associ-
female at the hospital because she ated with an increased risk of death. Liver
is significantly obtunded. She was transplantation is required for those chil-
admitted 2 days ago by a pediatric dren who do not recover spontaneously
gastroenterologist for vomiting with supportive care.
494 Chapter 17 · Neurologic Complications of Systemic Disease

zz Suggested Reading encephalopathy, or atypical psychosis,


55 Cochran JB, Losek JD. Acute liver failure since it can be successfully treated with
in children. Pediatr Emerg Care corticosteroid administration. It is likely
2007;23:129–135. under-­diagnosed in children and ado-
lescents due to the failure to consider
?? 7. You are evaluating a 14-year-old HE on the differential diagnosis. The
female after her tonic-­clonic seizure, diagnosis is usually made by identify-
preceded by unresponsiveness with ing the presence of high serum levels of
head and gaze deviation to the right anti-thyroglobulin and anti-microsomal
side. She has a change in her behavior antibodies. A relapsing-remitting course
for the last 2 days, mainly agitation may be observed.
and possible visual hallucinations.
She has a similar event 6 months ago zz Suggested Reading
which has been resolved over 1 week, 55 Watemberg N, Greenstein D, Levine
but no diagnosis has been made. A. Encephalopathy associated with
She is afebrile, awake, but agitated, Hashimoto thyroiditis: pediatric perspec-
unable to settle in bed. Her gait is tive. J Child Neurol. 2006;21:1–5.
ataxic. Brain CT and MR imaging 55 Alink J, de Vries TW. Unexplained
are normal. You ordered a LP, shows seizures, confusion or hallucinations:
normal opening pressure and CSF think Hashimoto encephalopathy. Acta
profile. Her EEG shows generalized Paediatr. 2008;97:451–453.
slowing with multifocal epileptiform 55 Castillo P, Woodruff B, Caselli R, Vernino
discharges. Her complete blood count S, Lucchinetti C, Swanson J, Noseworthy J,
and serum chemistries are normal, Aksamit A, Carter J, Sirven J, Hunder G,
including serum thyroid levels. How- Fatourechi V, Mokri B, Drubach D, Pittock
ever, she has markedly elevated anti- S, Lennon V, Boeve B. Steroid-responsive
thyroglobulin serum antibodies. encephalopathy associated with autoim-
What is the most likely diagnosis? mune thyroiditis. Arch Neurol.
A. Herpes simplex encephalitis 2006;63(2):197–202.
B. Schizophrenia
C. Idiopathic frontal lobe epilepsy ?? 8. You are evaluating a 6 -year-old
D. Hashimoto encephalopathy boy presents for his uncontrollable
E. Rasmussen chronic encephalitis seizures despite he was treated
with several different anticonvul-
vv Correct Answer is: D sant medications. He is currently on
The pathogenesis of Hashimoto encepha- oxcarbazepine and topiramate, but
lopathy (HE) is unknown, but likely continues to have complex partial
relates to the activity of autoantibodies seizures monthly. His medical history
against CNS antigens. Serum levels of is remarkable for poor weight gain,
17 thyroid hormone may be either normal abdominal cramping and intermit-
or low for patients with this disorder. It tent diarrhea. His development is on
is an important diagnosis to consider target. He has a normal examination.
in children and adolescents with unex- He has no neuro-cutaneous stigmata.
plained seizures, ataxia, myoclonus, Brian CT scan is shown:
Neurologic Complications of Systemic Disease
495 17

..      Figs. 17.1 and 17.2 A CT scan of the patient revealed calcification (white) in the back of his brain, caused by
celiac disease. (Permission: The New England Journal of Medicine ©2014)

What test are you going to order to is refractory to traditional anticonvulsant


reach the diagnosis of his epilepsy? agents are common in those children. Gluten
A. Cerebrospinal lactate and pyru- free diet improve the seizures control. It is
vate levels felt chronic folic acid deficiency may lead
B. Genetic testing for TSC1 and TSC 2 to the development of the calcifications. A
C. Serum parathyroid hormone majority of affected children will have some
D. Serum toxoplasmosis titers form of gastrointestinal involvement, such
E. Anti-endomysial antibodies fol- as chronic diarrhea, constipation or failure to
lowed by a small bowel biopsy thrive, though neurologic dysfunction can be
the sole presenting feature of the disorder.
vv Correct Answer is: E Once celiac disease is suspected, a
Many assumptions have been made to link practical approach to diagnosis starts with
celiac disease to many neurologic disorders. screening serum laboratory testing.
Celiac disease results secondary to a perma- Traditionally this was done via anti-gliadin
nent sensitivity in genetically susceptible antibodies (IgA mediated), though these
patients to gluten products in wheat, barley antibodies are not specific for celiac
and rye, and presents with features of an disease and have been replaced with
immune mediated enteropathy. The most anti-endomysial antibodies and/or assay
common neurologic symptoms include of tissue transglutaminase. Although a
chronic progressive ataxia and/or neuropa- positive antibody test is indicative of
thy in adults. Recent studies have shown an celiac disease, a formal diagnosis can be
increased rate of celiac disease in children confirmed by small bowel biopsy before
with epilepsy. A specific syndrome of epi- starting a gluten-free diet. A false-nega-
lepsy and occipital calcifications has been tive biopsy can happened if the biopsy
reported. Complex partial seizures, which has been done after initiation of the diet.
496 Chapter 17 · Neurologic Complications of Systemic Disease

Calcifications seen in tuberous temporal lobe. He recovered gradu-


sclerosis complex typical are located ally over the following week, and
sub-ependymal. The calcified lesions repeat MR imaging 1 month later
resulting from leptomeningeal angioma- shows complete resolution of the dif-
tosis in Sturge-Weber syndrome often fusion abnormalities.
have a similar appearance to those Which of the following medications
described above. However, the lack of an is most likely to have caused this
associated facial port-wine nevus or patient’s symptoms?
severe developmental delay would make A. Doxorubicin
this diagnosis highly unlikely in this B. Intrathecal hydrocortisone
patient. Those seen in mitochondrial C. Intrathecal methotrexate
disorders traditionally affect the deep D. Methylprednisolone
gray structures. Hyper- or hypoparathy- E. Vincristine
roidism has been associated with bilateral
basal ganglia calcifications in adults (often vv Correct Answer is: C
cases of Fahr’s disease). Neuroimaging of Several central nervous system compli-
patients with congenital toxoplasmosis cations are associated with childhood
often reveals subtle punctate calcifica- leukemia, some secondary to the disease
tions in the parenchyma, but either itself and others secondary to its treat-
microcephaly or significant developmen- ment. Leukemia can cause neurologic
tal delay is usually noted as well. dysfunction through direct invasion of
the nervous system, which generally
zz Suggested Reading occurs through leptomeningeal metasta-
55 Journal of Clinical Neuromuscular sis with dissemination of leukemic cells
Disease. 2004;5:129–37. throughout the subarachnoid space. It can
55 Journal of Child Neurology. 2002;17 also cause dysfunction indirectly, often
(11):800–6. through disturbances of coagulation and
blood flow (e.g. disseminated intravascu-
?? 9. The floor nurse has noted that her lar coagulation, hyper-viscosity/leukosta-
3-year-old boy patient is not using sis, venous sinus thrombosis). Finally, the
his right arm. The boy was admitted CNS is a common site of relapse, as seen in
for induction chemotherapy for his 3–8% of children with either ALL or AML.
precursor B cell acute lymphoblastic The treatments for childhood leuke-
leukemia. He has received intrathecal mia—systemic/intrathecal chemotherapy,
cytarabine; systemic methylpred- cranial irradiation, and bone marrow
nisolone, doxorubicin, vincristine, transplantation—can also cause neuro-
and asparaginase; and intrathe- logic dysfunction. The side effects of
cal triple therapy with cytarabine, chemotherapy vary based on the specific
methotrexate, and hydrocortisone. agents used. Intrathecal and high-dose
17 His last intrathecal treatment was 10 intravenous methotrexate have been
days ago, and he had no complica- shown to cause immediate, acute to
tion at that time. You examined him subacute, and chronic neurologic syn-
and you found that, he has slurred dromes. The acute to subacute syndrome
speech, right lower facial droop, and usually occurs within days to several
right arm paralysis. DW- MR imaging weeks of treatment and is characterized
shows bilateral areas of restricted by a variety of neurologic signs, including
diffusion in the cerebral white mat- seizures, mental status changes, and focal
ter, most prominently within the left neurologic deficits (e.g. aphasia, anesthe-
Neurologic Complications of Systemic Disease
497 17
sia, paresis, ataxia, tremor, blurred vision). count of 52,000/uL. His basic meta-
Brain imaging reveals leukoencephalopa- bolic panel shows a BUN of 110, a cre-
thy, but the lesions are usually reversible. atinine of 2.1 mg/dL and glucose of
Intrathecal chemotherapy—whether with 136 mg/dL. His urine shows moderate
methotrexate or other agents—can also blood and protein, and hyaline casts.
be associated with complications such as What is the most likely diagnosis?
Post-Dural puncture headache, CNS A. Celiac disease
hemorrhage, spinal cord dysfunction, B. Febrile seizure
chemical meningitis, and neuropsycho- C. Hemolytic uremic syndrome
logical dysfunction. Cranial irradiation is a D. Idiopathic thrombocytopenic
very effective form of CNS-­directed ­purpura
treatment, but attempts have been made E. Shigellosis
to eliminate or reduce its use given its
substantial rates of secondary neoplasms, vv Correct Answer is: C
endocrine diseases, growth impairment, Hemolytic uremia is a triad of: microan-
neurocognitive dysfunction, and other giopathic hemolytic anemia, thrombocy-
neurotoxic effects. Patients who undergo topenia, and acute renal failure. More than
bone marrow transplantation are at risk 75% of all cases occur in children younger
for seizures, which occur in about 5% of than 3 years, with few cases after 5 years
patients treated with cyclosporine; CNS of age. In older adults it can occur, usually
infections; and graft-versus host disease. with an identifiable provocation. Thirty-
three percent of patients can have neuro-
zz Suggested Reading logical manifestations. Most of these tend
55 Pui, C-H, Howard SC. Current manage- to be mild and transient, but convulsions
ment and challenges of malignant disease and ischemic cerebral infarctions, espe-
in the CNS in paediatric leukemia. In cially in the basal ganglia, can be severe.
Lancet Oncolo. 2008;9:257–68.
55 Shuper A, et al. Methotrexate treatment zz Suggested Reading
protocols and the central nervous system: 55 Rust R Hemolytic Uremic Syndrome. On
significant cure with significant neurotox- 7 http://emedicine.medscape.com/
icity. In J Child Neurol. 2000;15:573–80. article/1183555-overview, updated
55 Demopoulous A, DeAngelis LM. April 3, 2009.
Neurologic complications of leukemia. In
Curr Opin Neurol. 2002;15:691–99. ?? 11. You are seeing an 8-year-old boy in
the PICU because of progressive con-
?? 10. You are seeing a 4-year-old boy in fusion and drowsiness for 2 days. His
the ED who has his first generalized past medical history is unremarkable
convulsive seizure for 5 min. He has and takes no medications. He has
watery diarrhea for 3 days, which been vomiting several times a day
became bloody today. On examina- for 4 days, and this has continued. He
tion, he has a low-grade fever (38 has normal vital signs. Scleral icterus
degrees Celsius) and a mildly full and right upper quadrant abdominal
abdomen, but no rash or neuro- tenderness are present. He is drowsy
cutaneous stigmata. He is minimally and unable repeat his name, and he
arousable, only moaning with exami- does not know his age or his siblings’
nation. Laboratory studies show a names. His speech is mildly dysarthric.
low hematocrit of 32%, a white blood Cranial nerves are intact. He has low
cell count of 13.5/mm3, and a platelet truncal tone but normal extremity
498 Chapter 17 · Neurologic Complications of Systemic Disease

tone and strength. Reflexes are brisk compounds including short chain fatty
in the upper and lower extremities, acids, false neurotransmitters and gamma
with 3–4 beats of clonus at the ankles. aminobutyric acid have been implicated
He is not able to sit unsupported. as well, helping to explain the few patients
His laboratory studies show a mild with hepatic encephalopathy without
normocytic anemia, with an ALT of hyper-ammonemia. Cerebral edema is a
450 U/L and AST of 680 U/L. Biliru- frequent complication in the later stages
bin is elevated at 6.5 mg/dL. Serum of hepatic encephalopathy, and is often
ammonia is elevated at 120 ug/ the cause of death.
dL. EEG is ordered because of inter- Reducing the ammonia concentration
mittent arms twitching movements, and treating various complications
and showed diffuse background slow- including infections, renal and cardio-
ing with tri-phasic waves. Brain CT vascular dysfunction, and bleeding
scan is normal. secondary to ruptured varices, hyper-
What is most appropriate initial splenism or clotting factor deficiencies
intervention management of this are the most effective treatments for
patient? hepatic encephalopathy. Ammonia
A. Lactulose production can be reduced by lowering
B. Liver transplantation dietary protein intake and increasing
C. Lorazepam osmotic removal from the gut by
D. Manganese chelation non-absorbable disaccharides such as
E. Mannitol lactulose.
Lorazepam would be indicated in the
vv Correct Answer is: A setting of seizures, which are associated in
The boy has an encephalopathy associ- only 10–30% of cases of hepatic encepha-
ated with a hepatic disease. The causes lopathy. In fact, the benzodiazepine recep-
of hepatic encephalopathy in children tor antagonist flumazenil has shown some
differ greatly from those in adults (where evidence in adult studies improving
chronic liver disease and cirrhosis are clinical features of the encephalopathy.
most common. Children with hepatic Mannitol or hypertonic saline adminis-
encephalopathy most often have acute tration is effective when cerebral edema is
fulminant liver failure, either from viral present, though not present in the current
hepatitis, toxic ingestion or inborn errors case.
of metabolism. Although encephalopathy Liver transplantation should be
is the most constant clinical finding, also considered for any severe and progressive
noted are variations in muscle tone, hyper- cases of hepatic encephalopathy, not
reflexia, parkinsonian features, ataxia, and responding to more conservative mea-
dysarthria and action tremor. Asterixis, a sures.
hallmark of adult metabolic encephalopa- Manganese deposition in the basal
17 thies, including those of hepatic origin, ganglia has been postulated to lead to
can be absent in pediatric cases. extrapyramidal features of hepatic
The pathophysiology behind hepatic encephalopathy in adults with chronic
encephalopathy appears to arise from liver cirrhosis. Trials of manganese
harmful effects of compounds toxic to the chelation are underway in adults, since
brain that would typically be filtered by chronic cirrhotic liver disease is rare in
the portal circulation. Although ammonia children, the utility in this age patient is
has been the chief toxin identified, other quite unclear.
Neurologic Complications of Systemic Disease
499 17
zz Suggested Reading perfusion to the penumbra. Primary pre-
55 Swaiman, et al. Pediatric neurology, vention of stroke risk in children with sickle
principles and practice, 4th ed. pp. 2294–7. cell disease remains a very important part
of ongoing medical surveillance of patients
?? 12. A 16-year-old male with history of with this disorder. Screen transcranial
sickle cell disease and previous strokes Doppler studies are recommended start-
(the most recent occurring 4 months ing at age two. Individuals with elevated
prior) came to the ED for his left-sided flow velocities over 200 cm/sec are recom-
weakness which happened 20 min mended to begin month transfusions, with
ago. His NIH stroke scale is 8 because the goal of keeping the hemoglobin S con-
the combination of dysarthria, facial centration under 30%. Data from the major
weakness and left-sided weakness. His trial (the STOP trial) showed a 90% reduc-
brain MRI showed a right frontal-pari- tion in stroke risk using this treatment
etal infarct. His previous 2 transcranial plan. Over time, patients with sickle cell
Doppler (TCDs) revealed velocities disease are at risk for development of moy-
>250 cm/s, though he was lost to fol- amoya syndrome, with progressive nar-
low up and never started receiving rowing of the proximal intracranial carotid
regular monthly transfusions. artery and development of multiple small
What is the most appropriate acute collateral vessels. Consideration of EC-IC
intervention for this patient? bypass procedures may be necessary for
A. Exchange transfusion to reduce such patients, but this therapy would not
the concentration of sickle hemo- be used in the therapy of acute stroke.
globin to less than 30%
B. Hypotonic saline boluses zz Suggested Reading
C. Permissive hyperglycemia 55 Holmstedt C, Adams RJ. Neurologic
D. Tissue plasminogen activator complications of hemoglobinopathies.
E. Urgent extracranial-­intracranial Continuum (Minneap Minn).
bypass procedure 2011;17(1):56–72.

vv Correct Answer is: A ?? 13. You are evaluating an 18-year-old


The use of tissue plasminogen activator in man for his hearing loss. After few
pediatric patients with stroke secondary to hours after he got his first dose of
sickle cell disease has no data to support chemotherapy for testicular cancer,
it, which is a treatment option for adult he developed bilateral tinnitus and
patients. Supportive care measures includ- muffled hearing that progressed over
ing correction of hypoxemia and hypoten- the next 48 h. He denies vertigo. He
sion and maintenance of normoglycemia has a normal neurological examina-
are standard procedures, as in any patient tion except reduced hearing to finger
with stroke. Hypotonic saline should be rub bilaterally. Weber test does not
avoided in patients with cerebral edema lateralize and air conduction is greater
after large infarctions as this could aggra- than bone conduction bilaterally.
vation this condition. The current standard Audiometry demonstrates significant
of care is exchange transfusion to keep high- and middle-frequency hearing
sickle hemoglobin <30% is, and shows a loss bilaterally.
lower recurrent stroke risk than simple Which chemotherapeutic agent
transfusion alone. The combination of likely caused this syndrome?
vigorous hydration and exchange transfu- A. Bleomycin
sion decrease viscosity and allow improved B. Cisplatin
500 Chapter 17 · Neurologic Complications of Systemic Disease

C. Etoposide
D. Ifosfamide
E. Vinblastine

vv Correct Answer is: C


Ototoxicity is a well-known complica-
tion of cisplatin chemotherapy. Tinnitus
is common and hearing loss is often
bilateral, symmetric and permanent. High-
frequency hearing loss is most common,
but medium frequencies may also be
affected. Other drugs that lead to ototox-
icity include aminoglycosides, quinine and
salicylates. Ifosfamide can cause acute
encephalopathy.

zz Suggested Reading
55 Mattox DE. Assessment and management
of tinnitus and hearing loss. Continuum
Lifelong Learning Neurol. 2006:12(4);
135–50.
..      Fig. 17.3 Magnetic resonance image showing
?? 14. You are evaluating a 13-year-old multiple cortico-subcortical areas of hyperintense signal
involving the occipital and parietal lobes bilaterally and
boy with chronic renal disease on
pons in a patient with posterior reversible encephalopa-
monthly cyclophosphamide pulse thy syndrome. Near fatal posterior reversible encepha-
therapy regimen. He is complaining of lopathy syndrome complicating chronic liver failure and
headaches that are not responsive to treated by induced hypothermia and dialysis: a case
acetaminophen. He is brought to the report. (J Medical Case Reports 2009, 3:6623. doi:
7 https://doi.org/10.1186/1752-1947-3-6623. Rashmi
ED after having focal seizure in his left
Chawla, Daniel Smith and Paul E Marik. Wikipedia)
hand and arm which became general-
ized. On your exam, he is still quite
sleepy, but is waking up. His left hand The clinical and MRI findings confirm
is clumsy and his left arm is weak. You your suspicion of:
could not understand his speech he A. ADEM
is frustrated by this. His blood pres- B. A brain abscesses
sure is 180/100 and his temperature C. PRES
is 38 °C. He is complaining of a dif- D. A hypertensive hemorrhage
fuse headache. His MRI scan is shown E. MELAS
below:
17 Three days later, he is feeling better. vv Correct Answer is: C
He has normal speech but his hand Posterior reversible encephalopathy syn-
and arm are weak and clumsy. His drome (PRES) most often occurs in the set-
blood pressure is 150/85 and his tem- ting of renal failure, hypertension, and/or
perature is 37.5 °C. immunosuppression and is characterized
He is discharged from the hospital by headache, confusion, and visual loss.
and you see him 7 months later. His It often reverses completely over weeks
examination is completely normal. His with effective treatment for the underly-
MRI scan is below. ing disease. The rapidity with which this
Neurologic Complications of Systemic Disease
501 17
patient improves back to baseline and the developed both systemic and neurological
reversal of his MRI findings in a patient manifestations of hyponatremia.
with renal disease requiring immunosup- Hypoglycemia and sometime hypocal-
pressive treatment marks this as PRES. cemia can cause mental status change,
but not the delirious type; also, seizures
zz Suggested Reading are less likely to occur.
55 Gera DN, Patil SB, Iyer A, Kute VB, Hypokalemia and hypomagnesemia
Gandhi S, Kumar D, Trivedi HL. Posterior very unlikely to cause mental status
reversible encephalopathy syndrome in change.
children with kidney disease. Indian J
Nephrol. 2014;24(1):28–34. zz Suggested Reading
55 Akın F, Kılıçaslan C, Solak ES, Uzun M, 55 Continuum. Acute symptomatic seizure
Aygün S, Arslan Ş. Posterior reversible and systemic illness. 2014;20(3):601–23.
encephalopathy syndrome in children:
report of three cases. Childs Nerv Syst. ?? 16. You are evaluating an 18-year-old
2014;30(3):535–40. female for her double vision. This is
55 Raj S, Overby P, Erdfarb A, Ushay started 4 months ago mostly in the
HM. Posterior reversible encephalopathy morning upon waking, and then it
syndrome: incidence and associated subsides. She reports double vision as
factors in a pediatric critical care popula- side-by-side double vision (horizontal
tion. Pediatr Neurol. 2013;49(5):335–9. diplopia). She has no associated head-
ache.
?? 15. An 18-years old healthy male has a Her exam is remarkable for proptosis
change in his mental status right after and weak abduction of the left eye.
he finished running a full marathon. Pupils are symmetric and reactive
His coach brought him to the first aid to light equally. MRI of orbit shows
tent at the finish line, as he was unable enlargement and increased enhance-
to walk. He was delirious when the ment of the muscles and soft tissue of
physician was examining him and did the left orbit.
not have orientation to time and place. What is your likely etiology for her
He had generalized weakness without diplopia?
focal findings on neurological exam. A. Tumor of the orbit
Few minutes later, he developed a B. Idiopathic intracranial hyperten-
generalized tonic-colonic seizure. sion
What is the most likely cause for his C. Lambert-Eaton myasthenic syn-
symptoms? drome
A. Hypoglycemia D. Ocular myasthenia gravis
B. Hypokalemia E. Thyroid ophthalmopathy
C. Hyponatremia
D. Hypocalcemia vv Correct Answer is: E
E. Hypomagnesemia Worsening in the morning for her diplopia
is a key feature for her condition. Thyroid
vv Correct Answer is: C ophthalmopathy (which is sometimes
Patient’s symptoms of delirium with men- referred to as thyroid-associated orbi-
tal status change and weakness followed topathy) is associated with diplopia that
by seizure is highly suggestive of hypona- is worse upon awakening due to orbital
tremia. Hyponatremia can occur in about congestion when in the supine position. A
13% of marathon runners and this patient lymphocytic inflammatory infiltration of
502 Chapter 17 · Neurologic Complications of Systemic Disease

the orbital tissues occurs in approximately myasthenic syndrome. Diplopia could be


50% of patients with graves disease and an initial symptom of ocular myasthenia
can be detected in a MRI of the orbit. gravis, but in myasthenia, diplopia is
Horizontal diplopia also could be the conversely better in the morning and
first symptom of Idiopathic intracranial worsens at the end of the day.
hypertension, but she denies headache or
any other feature in the history to support zz Suggested Reading
this diagnosis. Lack of the any other 55 Continuum. Neuro-­ophthalmology.
symptoms rules out the Lambert-Eaton 2014;20(4):952–3.

17
503 18

Neuromuscular

© Springer International Publishing AG, part of Springer Nature 2018


Y. M. Awaad, Absolute Pediatric Neurology, https://doi.org/10.1007/978-3-319-78801-2_18
504 Chapter 18 · Neuromuscular

?? 1. A family of a 2-month-­old baby boy not feeding well. Three days later, her
brought him for evaluation because he mother brings her back to the pediatri-
is floppy compared to its first child. On cian’s office because she. You asked the
examination, he does not lift his head or mother if anything new or different is
roll over, he perspires profusely on his being fed to the baby or is present in
head and face when feeding, and hypo- the baby’s environment. The mother
tonic and weak. He is alert and smiles says she has not fed the baby anything
appropriately but has scant limb move- other than breast milk, but she and
ments and a weak cry. He is areflexic her husband are having an extension
and the muscles are difficult to palpate. put on their home for which they just
What study are going to order to broke ground 3 weeks ago. There is no
establish the diagnosis? family history of neuromuscular dis-
A. Serum CK ease and no one else has been sick at
B. Blood spot for alpha 1–4 glucosi- home.
dase deficiency. On examination, the infant is
C. Determination of CTG repeat hypotonic, areflexic, and has ptosis
number in the PK gene. and dysconjugate gaze although
D. Survival motor neuron gene he seems to be awake and alert. An
mutation analysis EMG d ­ emonstrates an incremental
E. Electromyography response to rapid repetitive nerve
stimulation.
vv Correct Answer is: D The baby’s condition is likely the
Perspiration when feeding is an indication result of:
of the work the infant is doing to breathe A. The Karo syrup being given before
and eat at the same time. The expressive 2 years of age
face would be expected in SMA at this stage B. The recent digging up of soil out-
but not in myotonic dystrophy (type 1 asso- side of the home
ciated with PK gene abnormality). Although C. An occult viral infection
Pompe’s disease would not affect the facial D. A genetic disorder of muscle
expression at this age, the fact that the mus- E. A congenital disorder of the neu-
cles are not firm and easily palpated would romuscular junction
be unexpected. Though the serum CK, EMG,
MRI and muscle biopsy are reasonable tests vv Correct Answer is: B
to consider, the most common cause of Infant botulism often starts with constipa-
hypotonia and weakness in this age group, tion and rapidly progresses to include
particularly given the alert and expressive areflexic hypotonia and paresis of eye
face and flabby muscles would SMA-1. movement. It can be the result of inges-
tion of Clostridium botulinum spores
zz Suggested Readings in honey; more often, it is the result of
55 Bodensteiner JB. The evaluation of the spores in soil that is unearthed during
hypotonic child. Semin Pediatr Neurol. construction. EMG findings include an
18 2008;15:10–20. incremental response to rapid repetitive
nerve stimulation.
?? 2. A pediatrician referred a 5-month-old
infant to you because his eyelids look zz Suggested Readings
droopy and his muscle tone is floppier 55 Domingo RM, Haller JS, Gruenthal
than usual. He saw the infant few days M. Infant botulism: two recent cases and
ago for constipation and puts him literature review. J Child Neurol.
on Karo syrup, and also the infant is 2008;23(11):1336–46.
Neuromuscular
505 18
?? 3. You are following a 14-year-old morbidly musculature and is painful and reversible
obese boy for metabolic syndrome. His with early withdrawal of the statin.
fasting glucose was 150 and his fasting
cholesterol was 250, so he was started zz Suggested Readings
on a sulfonylurea and a statin. Two 55 Scripko PD, Amato AA, Puig A. Mystery
weeks later, he awaked unable to lift case: a 63-year-old man with progressive
either arm sufficiently to comb his hair. proximal pain and weakness. Neurology.
His proximal arm muscles ached as if he 2014;82(4): e26–9.
had done chin-ups in gym class.
He is likely to have: ?? 4. You are evaluating a 3-year-old girl
A. Statin-induced myopathy with congenital hypotonia, severe
B. Deconditioning syndrome intellectual disability, and dysphagia,
C. FSH muscular dystrophy neonatal diagnosis of hydrocephalus
D. Endogenous steroid myopathy and recent onset of seizures. She
E. Hypothyroidism cannot sit independently but does
smile and respond to her environ-
vv Correct Answer is: A ment. She has not had any neuro-
An uncommon side effect of statins exists logical regression. Fundus exam
along a spectrum that includes myoly- revealed optic nerve hypoplasia and
sis, myopathy, and myalgias in various retinal dysplasia. Her MRI at 2-days
patients. It generally involves proximal of life is shown.

a b

c d e

..      Fig. 18.1 Fetal (a, b) and 2-day-old (c–e) brain MRI small cysts. d Axial-T2: retinal detachment (arrow);
showing malformations consistent with muscle-eye-­ microphthalmia. e Axial-T2: abnormal gyral sulcal
brain disease. a Sagittal-T2: brainstem hypoplasia and pattern with a nodular cobblestone-appearing cortex.
“kinked” pontomesencephalic contour. b Coronal-T2: (Teaching NeuroImages: Prenatal MRI of muscle-eye-
absent septum pellucidum and severe ventricular brain disease. Resident and Fellow Section. Neurol-
dilation. c Axial-T2: hypoplastic pons with an abnormal ogy June 1, 2010 vol. 74 no. 22 e101)
posterior indentation; dysplastic cerebellum with
506 Chapter 18 · Neuromuscular

What is your most likely cause of her typically less severe. In addition, the
syndrome? brainstem is typically normal.
A. LMNA-related congenital muscular Merosin positive congenital muscular
dystrophy dystrophy and SEPN1-related congenital
B. Muscle Eye Brain Disease muscular dystrophy are other forms of
C. Merosin Positive Muscular congenital muscular dystrophy, but
Dystrophy typically brain MRI is normal and intel-
D. SEPN1-related congenital muscu- ligence is normal in merosin positive CMD.
lar dystrophy
E. Fukuyama Congenital muscular zz Suggested Readings
dystrophy 55 Bertini E, D’Amico A, Gualandi F, Petrini S.
Congenital muscular dystrophies: a brief
vv Correct Answer is: B review. Semin Pediatr Neurol.
MRIs show poly-microgyria, pontine 2011;18(4):277–88.
hypoplasia, patchy T2/FLAIR subcortical 55 Congenital Muscular Dystrophy Overview,
and periventricular white matter signal Gene Reviews, Susan Sparks, MD, PhD,
abnormality, partial posterior commisural Susana Quijano-Roy, MD, PhD, Amy
dysgenesis, and absent septum pellu- Harper, MD, Anne Rutkowski, MD, Erynn
cidum. Those findings are suggestive of Gordon, MS, CGC, Eric P Hoffman, PhD,
a congenital muscular dystrophy/cobble- and Elena Pegoraro, MD, PhD. January 22,
stone complex malformation. MRI findings 2001; Last Revision: August 23, 2012
together with her clinical symptoms sug- accessed 12/8/2014.
gested that she was mostly likely to have
a more severe form of a dystroglycanopa- ?? 5. You are seeing a newborn infant with
thy, in particular, Muscle Eye Brain (MEB) hypotonia and respiratory failure. The
disease. Her genetic testing confirmed this mother had polyhydramnios but her
with 2 mutations of the POMGNT1 gene, delivery was normal. Mother had a
which is the most common cause of MEB. long narrow face and release of a
LMNA-related CMD (L-CMD) can hand grip is delayed.
present with a severe picture in the first 6 In the first year of life this infant is
months of life (absence of head or trunk most likely to have which of the fol-
support) or with progressive loss of head lowing?
support after acquisition of sitting or A. Cognitive delay, cardiac rhythm
walking ability (dropped head syndrome). disturbances and hepatitis
Often hypotonia and weakness of the B. Myotonia, cognitive delay and
axial-cervical muscles is rapidly progres- dysphagia
sive, followed by more slowly progressive C. Cardiac rhythm disturbance, dys-
weakness of the proximal upper limbs and phagia and myotonia
distal lower limbs. However, facial muscles D. Cognitive delay, facial weakness
are spared. With time, the characteristic and pneumonia
findings are head lag, thoracic and lumbar E. Facial weakness, myotonia and
18 spinal hyperextension (rigidity), lower pneumonia
limb contractures, and talipes equin-
ovarus but no significant upper limb vv Correct Answer is: D
contractures. MRI is typically normal. This is clinical presentation is most likely
Fukuyama Congenital muscular congenital myotonic dystrophy 1. DM1
dystrophy is another dystroglycanopathy, results from an expansion of a CTG tri-
but the neurological and eye findings are nucleotide repeat in the 3′-untranslated
Neuromuscular
507 18
region of the dystrophia myotonica What is the most likely cause of his
protein kinase gene (DMPK gene) on symptoms?
chromosome 19q 13.3. Usually the A. Limb-Girdle Muscular Dystrophy
mother has only mild symptoms and is B. Becker’s Muscular Dystrophy
diagnosed after she has an infant with C. Fascioscapulohumeral Dystrophy
the severe congenital form as there may D. Emery-Dreifuss Muscular
be significant expansion of the trinucleo- Dystrophy
tide repeat when passed from mother E. Merosin-Deficient Congenital
to child. In general, longer CTG repeat Muscular Dystrophy
expansions correlate with an earlier age
of onset and more severe disease. Small vv Correct Answer is: D
but abnormal repeats (50–99) are often The following scenario is part of a two-­
associated with a mild or asymptomatic question series. Please proceed to the next
phenotype. Maternal anticipation of question. Discussion & References included
the DM1 gene is more common than at the end of series.
paternal anticipation. Infants with con-
genital DM1 have hypotonia, and often ?? 7. At 16 years of age, this boy developed
subsequent respiratory failure. They recurrent dizzy spells and fainted once.
are also at significant risk for intellec- What is the most likely cause of his
tual disability (50–60%). However, the dizziness?
myotonia is not typically present in the A. Dilated Ventricular Cardiomyopathy
first year of life and cardiac conduc- B. Vagal Nerve Dysfunction
tion abnormalities, while significant, C. Aortic Stenosis
do not typically p ­ resent until the 2nd D. Cardiac Conduction Deficits
decade. Hepatitis is not typically associ- E. Vertebrobasilar Arterial Disease
ated with DM1.
vv Correct Answer is: D
zz Suggested Readings Emery-Dreifuss muscular dystrophy
55 Myotonic Dystrophy Type 1 in Gene presents in childhood or early adoles-
Reviews, Thomas D Bird, MD, Initial cence with relatively mild weakness and
Posting: September 17, 1999. Last Update: contractures at the elbows and ankles. It
May 16, 2013. is characterized by the triad of early con-
tractures of the Achilles tendons, elbows,
?? 6. T
 he following scenario is part of a and posterior cervical muscles; slowly
two-question series, Discussion & Ref- progressive muscle weakness and wasting
erences included at the end of series. with humeroperoneal distribution in the
You are seeing an 11old boy who early stages; and later cardiomyopathy
has been toe walking since he was with conduction deficits. The disease is
toddler and has difficulty straighten- most commonly x-linked ­(mutations in
ing his arms. On exam he has mild X-EMD at Xq28), less commonly autosomal
weakness and atrophy of biceps and recessive (lamin A/C gene on chromosome
triceps muscles, slight facial weakness 1q21) and rarely autosomal dominant. The
and cannot fully extend his arms at the neuromuscular symptoms may be rela-
elbow. However, his hand strength is tively mild.
normal. CK is mildly elevated. Family his- The cardiac conduction deficits
tory is remarkable for maternal uncle has usually occur later, in the 3rd decade, but
similar symptoms as a child. may present as early as 5 years of
508 Chapter 18 · Neuromuscular

age. While the weakness may be mild and vv Correct Answer is: A
missed in childhood, the cardiac conduc- As stated in the AAN and CNS Practice
tion deficits, which can initially include Parameter on Corticosteroid treatment
first degree AV-Block, sinus bradycardia of Duchenne Dystrophy (DMD), boys
and supraventricular tachycardia and with DMD should be offered treatment
may progress to atrial flutter, atrial with 0.75 mg/kg/d of prednisone or
fibrillation, and atrial stand-still, can be Deflazacort (0.9 mg/kg/day) as these
lethal and increase the risk of ischemic treatments have been shown to have a
stroke. beneficial effect on muscle strength and
function for boys with DMD. Even lower
zz Suggested Readings doses of steroid can have benefit and
55 Deymeer F. Emery-­Dreiffus muscular so if side effects require a decrease in
dystrophy In: Jones H, De Vivo D, Darras B, prednisone, tapering to dosages as low as
editors. Neuromuscular disorders of 0.3 mg/kg/day have been shown to pro-
infancy, childhood and adolescence, A vide a less robust but significant improve-
clinician’s approach. Philadelphia: ment. IVIG does not typically have a role
Butterworth Heinmann; 2003. p. 753–63. in the management of DMD.

?? 8. You are evaluating a 6-year-old boy zz Suggested Readings


with difficulty walking and trouble 55 AAN Practice Parameter: Corticosteroid
going up stairs. For the last year his treatment of Duchenne dystrophy, Report
family noticed that he was falling of the Quality Standards Subcommittee of
more often and was not able to jump the American Academy of Neurology and
well or keep up with his friends on the Practice Committee of the Child
the playground. On examination, Neurology Society, R. T. Moxley III, MD,
he has proximal muscle weakness, S. Ashwal, MD, S. Pandya, MS, PT,
calf hypertrophy and he use Gowers’ A. Connolly, MD, J. Florence, MHS, PT,
maneuver to stand. CK is 23,000. K. Mathews, MD, L. Baumbach, MD,
What is the most appropriate treat- C. McDonald, MD, M. Sussman, MD and
ment advice for this family? C. Wade, PhD, PT, RN, Neurology January
A. Prednisone treatment should be 11, 2005 vol. 64 no. 1 13–20.
offered as it has been demon-
strated to have a beneficial effect ?? 9. You were asked to see a 3-month-old
on muscle strength and function girl in the hospital with a 5-day his-
B. Prednisone can be considered but tory of constipation followed over the
reliable data to support its use in next 3 days by poor feeding with a
this disorder is lacking weak suck, decreased urinary output
C. Prednisone may be used but and irritability. No systemic mani-
should be stopped if their son festations or fever. Her pregnancy
is unable to tolerate the recom- and delivery were unremarkable. On
mended dose as lower doses of examination, her heart rate is 140/
18 prednisone are not effective min, systolic blood pressure 60, res-
D. IVIG may be used to treat crises in pirations 40/min, temperature 36.7
this disorder with or without pred- degrees. She is listless, with a weak
nisone use cry, weak suck, and poor head control
E. Prednisone use in this disorder and decreased DTRs. Eye movements
can hasten deterioration and are decreased and pupillary reactions
should be avoided sluggish.
Neuromuscular
509 18
What study is most likely to estab- related to autonomic dysfunction, bulbar
lish the etiology? weakness manifesting as hypophonia and
A. Acetylcholine receptor antibodies a weak cry, poor suck, and diminished gag,
B. Cerebrospinal fluid examination leading to difficulty swallowing, drooling,
C. CPK level and often dehydration, cranial nerve
D. Stool sample for botulinum toxin palsies such as ptosis, sluggish papillary
E. Thyroid function studies light reflexes and ophthalmoplegia, facial
weakness, and diminished head control
vv Correct Answer is: D with neck and shoulder-girdle weakness,
This is a classic presentation of infantile then progressing to generalized hypotonia
botulism. There are 4 types of human and a descending, symmetric flaccid
disease which may result from botulinum paralysis of the skeletal muscles with
toxin, including food-borne or “classical” reduced spontaneous movement, delayed
botulism, wound botulism, adult intestinal motor developmental milestones, and
toxemia, and infant botulism. diminished or absent muscle stretch
Food-borne botulism occurs via the reflexes. Other findings of autonomic
ingestion of foods contaminated with dysfunction may result, including anhydro-
preformed toxin, whereas the remaining sis, dry mouth and throat, often leading to
forms of human disease occur through the mucosal irritation and an incorrect initial
infection with and growth of clostridial clinical suspicion for pharyngitis, hyperten-
organisms that germinate in situ in sion alternating with postural hypotension,
wounds or in the gastrointestinal tracts of brady- and tachydysrhythmias, flushing,
those afflicted, thereafter producing decreased anal sphincter tone, and bladder
toxins which are absorbed and exert atony. Respiratory compromise, hypoventi-
systemic effects. lation, hypoxia, and the need for mechani-
Iatrogenic botulism has been reported cal ventilation in a majority of cases ensue
to rarely result from the injection of toxin as the diaphragmatic and accessory
for medical and cosmetic purposes. respiratory musculature becomes affected.
Inhalational botulism could result from Treatment is supportive and administra-
aerosolization of botulinum toxin, and has tion of Botulism immune globulin. This
been of interest in recent years as a compound has been found to have a
potential weapon of bioterrorism. Since it half-life of 28 days in vivo, and a single
was first described as a recognizable and infusion has been shown to neutralize
discrete entity in 1976, infant botulism has botulinum toxin for at least 6 months. It
become the most frequently reported immediately neutralizes all circulating
syndrome related to botulinum toxin, botulinum toxin and remains present in
accounting for roughly 75% of all annual neutralizing amounts in the circulation for
cases in the United States. In approxi- several months, allowing for the regenera-
mately 85% of cases, a source of ingestion tion of nerve endings to take place. It
is never found. The clinical syndrome of consists of immune globulin prepared from
infant botulism usually occurs between the plasma of donors immunized with
2 weeks and 1 year of age, with a median pentavalent (AB-­CDE) botulinum toxoid.
age of about 10 weeks. The symptoms
described in infant botulism are related to zz Suggested Readings
peripheral cholinergic blockade due to 55 Domingo RM, Haller JS, Gruenthal M.
botulinum toxin. These include constipa- Infant Botulism: two recent cases and
tion or a change in stool pattern with literature review. J Child Neurol.
decreased frequency of bowel movements 2008;23(11):1336–46.
510 Chapter 18 · Neuromuscular

The approach to this condition in the


absence of family history or inheritance
pattern should be to first determine if the
disease is demyelinating or axonal or
mixed. If the MNCV is normal or near
normal the condition is axonal and the
causes of axonal hereditary motor sensory
neuropathy should be sought. The full CMT
battery is expensive and wasteful. In
considering the axonal forms of disease,
mitofuscin-2 mutations are the most
common but only make up about 20% of
the cases. If the mitofuscin-2 gene is normal
the MPZ protein gene and the gap junction
binding protein (GJB1, Connexin 32) should
be tested.

..      Fig. 18.2 Own work (Original text: Own work,


zz Suggested Readings
originally from en.wikipedia; description page is/
was here.). Benefros at English Wikipedia 55 Bird TD Charcot Marie Tooth Hereditary
Neuropathy Overview. In Genetests.com
?? 10. You are evaluating a 12-year-old girl last revised 1/25/11.
with progressive weakness of the
distal extremities. She has no family ?? 11. You are evaluating a 3-year-old boy
history of weakness. She wears AFOs for developmental delay. He was a
to prevent a foot drop and aid his product of full term pregnancy with
walking. Her hands are nearly non- no complications and acquired nor-
functional due to weakness and atro- mal early milestones, however did
phy of the intrinsic muscles and she not stand until 12 months and walk
cannot oppose the thumb and little until 15 months. Also, he has speech
finger. Her reflexes are absent and she delay. There is negative family history
has normal strength in the proximal of developmental delay, mental retar-
muscles and face. dation or other neurological problems.
What is your next step in her On examination, he is mildly hypotonic
evaluation? with low amplitude tendon reflexes in
A. Genetic CMT evaluation. all limbs. He is not dysmorphic and has
B. Genetic study of CTG repeats in average weight and height. His urine
the PK gene. organic acid and serum amino acid
C. Muscle biopsy with immunohisto- screen, a metabolic panel including
chemical stains. lactate, and acyl-carnitine profile were
D. Nerve conduction velocity. normal. AST and ALT both are 2–3
E. Serum CK times the upper limit of normal. There
18 is no history of jaundice or hepatitis.
vv Correct Answer is: D. What is the next test being you
Although the clinical picture is indicative going to order?
of a peripheral neuropathy, the lack of A. Complete genomic microarray
family history means it is impossible to analysis.
determine the exact type of disease or the B. Gene testing for fragile X syndrome.
inheritance pattern. C. Liver biopsy.
Neuromuscular
511 18
D. MRI of brain, with and without function, you should always check the CK
contrast. before proceeding to a liver biopsy.
E. Serum creatine kinase determination.
zz Suggested Readings
vv Correct Answer is: E 55 Srour M, Bejjani BA, Rorem EA, et al. An
The MDA currently recommends CK instructive case of an 8-year-old boy with
screening in male children with devel- intellectual disability. Sem Ped Neurol.
opmental delay if there are no other 2008;15:154–7.
features which would suggest a direction
for diagnostic evaluation. Children with ?? 12. You are evaluating a 7-month-old
dystrophinopathies (Duchenne muscular infant with delayed acquisition of
dystrophy) on average have a 15-point motor developmental milestones. She
decrease in IQ and may not show weak- seems to be cognitively intact but
ness, calf hypertrophy, absent reflexes or cannot roll, sit, support her weight or
other signs of the muscle disease until crawl. She can sit if placed in the sit-
later in childhood. The so called “liver ting position. Her reflexes are absent
enzymes” AST and ALT are also produced and her muscles are flabby on palpa-
in muscle whereas creatine kinase (CK) is tion. Her mother reports that she has
produced only in muscle (skeletal and car- been affected since birth and does
diac) and brain. In patients with high “liver not appear to be getting worse. Her
enzymes” and apparently normal liver MRI is shown.

..      Fig. 18.3 A 3-year-old girl suffered from merosin-­ abnormal T2 prolongation in cerebral white matter,
deficient congenital muscular dystrophy (MDCMD). without involvement of corpus callosum, internal
Magnetic Resonance Imaging (MRI) brain shows capsule, brain stem and cerebellum or other structural
diffuse and symmetrical increase in T2W signal in abnormalities. (Protocol: Magnet strength 1.5 Tesla, TR
white matter of cerebral hemispheres. Signal 4500, TE 1086/Ef, without contrast, 5 mm slice
characteristic of corpus callosum, internal capsule, thickness). (J Radiol Case Rep. 2012 Aug;6(8):1–7. doi:
basal ganglia and thalami are normal. Brain stem and 7 https://doi.org/10.3941/jrcr. v6i8.997. Epub 2012
cerebellum are structurally normal (not shown). Aug 1. PubMed.gov)
Ventricles are not dilated. The major abnormality is
512 Chapter 18 · Neuromuscular

What is the most likely diagnosis? posture, and a paucity of sponta-


A. Adrenal leukodystrophy neous movements. DTRs are 2+
B. Globoid cell leukodystrophy throughout, but he has a weak suck.
C. Multiple sclerosis What is the most appropriate next
D. Periventricular leukomalacia step in determining this patient’s
E. Merosin deficient muscular diagnosis?
dystrophy A. SMN gene dosage analysis
B. Acetylcholine receptor antibody
vv Correct Answer is: E assay
The absence of encephalopathy makes C. Head MRI
Krabbe and ALD unlikely. The absence of D. Methylation analysis of chromo-
reflexes argues against all of the above some 15q
but E although some cases of ALD will E. Botulinum toxin assay
have depressed reflexes for a time dur-
ing the development of the disease. The vv Correct Answer is: B
extensive white matter changes in the Transient neonatal myasthenia gravis
brain without cognitive impairment, (TNMG) is a disorder of postsynaptic
particularly in the context of a non- neuromuscular transmission that occurs
progressive congenital muscle disease in 21% of infants born to mothers with
is so characteristic of Merosin deficient acquired myasthenia gravis, particu-
congenital muscular dystrophy that some larly mothers with active, acetylcholine
feel muscle biopsy is not necessary in the receptor (AChR) antibody-­positive dis-
diagnostic evaluation of these patients. ease. Although the putative mechanism
involves passive transfer of anti-AChR or
zz Suggested Readings anti-MUSK antibodies across the placenta,
55 Bodensteiner JB. The evaluation of the the actual mechanism may be more
hypotonic child. Semin Pediatr Neurol. complicated. The majority of patients
2008;15:10–20. develop symptoms within a few hours
of birth; onset after 72 h has never been
?? 13. You are evaluating a full-term male reported. Typical findings include general-
infant in the NICU who has devel- ized hypotonia and difficulty sucking/
oped difficulty feeding, generalized swallowing, as well as weak cry, facial
hypotonia, and a weak cry in his first diplegia, and respiratory distress. Atypical
day of life. The mother had “extreme cases may present with arthrogrypo-
fatigue” during her pregnancy to sis. Gavage feeding and mechanical
the point that she had difficulty ventilatory support may be necessary in
keeping her eyes open and chew- severe cases. If the mother is known to
ing her food at dinnertime. She have anti-AChR or anti-­MUSK antibodies,
was not taking any medications or the diagnosis of TNMG may be fairly obvi-
illicit substances. Birthweight was ous. On the other hand, if the mother is
3475 g. APGAR scores were 9 at 1 and undiagnosed or has seronegative MG, it
18 5 min, and a physical examination may be necessary to confirm the diagnosis
performed in the delivery room was by administering an acetylcholinesterase
unremarkable. On repeat next day agent (e.g. neostigmine, edrophonium,
examination, the infant has a weak pyridostigmine) and observing for clinical
cry and intercostal retractions. He is improvement. False-negative pharmaco-
awake and alert, but his face appears logic tests can occur, in which case repeti-
expressionless. He has generalized tive nerve stimulation before and after the
hypotonia, resulting in a frog-legged administration of an acetylcholinesterase
Neuromuscular
513 18
agent improves the accuracy of the quantity of preformed toxin as in the adult
diagnosis. There is conflicting data about form. Thus the onset may be subacute and
whether maternal serum AChR antibody difficult to recognize at first. These chil-
titers are helpful in predicting which new- dren are almost all constipated for several
borns will develop symptoms. Neonatal days before the clinical presentation. They
AChR antibody titers are not helpful in clearly suffer from an acquired motor defi-
predicting the disease course, but their cit rather than one present at birth and
decline correlates with clinical improve- they have prominent involvement of the
ment. Treatment consists of supportive cranial musculature. The diagnosis can be
care and, when necessary, acetylcholines- made by inoculation of mice with serum
terase agents. Recovery generally occurs from the affected individual but electro
within 4 months. diagnosis can be done more quickly and
is quite reliable. Since these patients can
zz Suggested Readings be treated with antiserum it’s important
55 Niks EH, Verrips A, et al. A transient to make the diagnosis and initiate therapy
neonatal myasthenic syndrome with anti- as early as possible to limit the duration
MUSKantibodies. Neurology and severity of the disease. The source of
2008;70:1215–16. the botulism organisms is frequently not
ascertained but classically includes honey
?? 14. You are seeing a 5–month-old baby and inadequately sterilized bottles and
girl with difficulty feeding and inabil- nipples.
ity to handle secretions. She has been
progressively less active for the last zz Suggested Readings
5 days and has been constipated for 55 Bale JF, Infant Botulism in Gilman S,
the last 7 days. On examination she Goldstein GW, Waxman SG (eds.),
has bilateral ptosis, is drooling, and Medlink, San Diego, Arbor Publishing,
is unable to hold her head up. single Medlink 2007. 12th ed. Gutierrez
fiber EMG (S-SFEMG) shows incremen- AR. Electro diagnosis of infant botulism.
tal response with high-rate repetitive J Child Neurol. 1994;9:362–6.
nerve stimulation.
What is the most likely diagnosis? ?? 15. You are seeing a 4-year-old boy
A. Myasthenia Gravis because he has trouble keeping up
B. Congenital Myasthenic Syndrome with his peers when running or jump-
C. Infantile botulism ing. He is a little slower cognitively
D. Hypokalemic periodic paralysis than his sister who is 14 months
E. Myotonia Congenita younger. No family history of neuro-
logic and neuromuscular diseases. On
vv Correct Answer is: C examination his tendon reflexes are
The incremental response at high rates present but uniformly depressed and
of repetitive stimulation is one of the his muscles are firm on palpation. He
major criteria for botulism. Other features has a serum CK 10,200 IU.
would include decreased amplitude of The laboratory examination most
the CMAPs, post tetanic facilitation which likely to yield the definitive diagno-
is prolonged. Infant botulism is a more sis is which of the following?
insidious disease than the adult form as A. Deletion analysis of the SMN gene
the disease results from the colonization B. Deletion analysis of the dystro-
of the upper GI tract with the absorp- phin gene
tion of tiny amounts of the toxin at a C. Analysis of the CTG repeat num-
time rather than the ingestion of a larger ber in the PK gene
514 Chapter 18 · Neuromuscular

D. Electromyogram and nerve con- vv Correct Answer is: C


duction velocity studies This is a case of paramyotonia congenita,
E. Alpha 1–4 glucosidase enzyme caused by mutations in the SCN4A gene
activity determination which encodes the alpha subunit of the
muscle sodium channel. In paramyoto-
vv Correct Answer is: B nia congenita, the myotonia typically
Few diseases are associated with high becomes more pronounced with repeated
CK. In the face of muscle weakness, cogni- muscle contractions. This “paradoxical
tive delay and depressed reflexes and this myotonia” contrasts with that of myotonia
very elevated CK the dystrophinopathies congenita (due to mutations in CLCN1,
are the most obvious concern and would which encodes a chloride channel), asso-
be the first test to do. SMN gene test- ciated with a warm-up phenomenon of
ing for SMA, CTG repeats for myotonic improvement in stiffness with repeated
dystrophy and alpha 1–4 glucosidase use. Other disorders due to SCN4A muta-
activity for Pompe’s disease would be tions include hyperkalemic periodic paral-
considerations if the CK were only slightly ysis and hypokalemic periodic paralysis.
elevated. EMG and MNCV testing will
help to distinguish myopathy from neuro- zz Suggested Readings
pathic disease but will not yield a specific 55 Statland, JM, Barohn, RJ. Muscle
diagnosis. Channelopathies: the Nondystrophic
Myotonias and Periodic Paralyses.
zz Suggested Readings CONTINUUM: Lifelong Learn Neurol.
55 Engel AG, Yamamoto M, Fishback KH. 2013;19(6):1598–614.
Dystrophinopathies. In: Engel AG,
Frenzine-Armstrong C, editors. Myology. ?? 17. You are seeing a 6-year-old boy with a
2nd ed. 1994. p. 1133–1187. history of delay in attaining his motor
milestones, prolonged toe walking,
?? 16. You are evaluating an 18-year-old frequent falls, and difficulties perform-
female who had episodes of gen- ing basic motor tasks, such as running
eralized stiffening of her muscles and jumping. On examination he has
that last for several hours since she symmetric calf hypertrophy, a mildly
was 9 years of age. Exposure to cold lordotic posture, waddling of gait, and
temperature or exercise can provoke poor hip excursion during running. He
those episodes. On examination, has a head lag when you pulled him
she has delayed muscle relaxation forward from a supine position. He
after voluntary contraction and this performs a Gower’s maneuver when
becomes more marked with contin- rising from a seated position on the
ued attempts. Her upper arm muscles floor to a standing position. Serologic
are hypertrophied and the CPK is 850. testing reveals a CK of 31,400 U/L.
A mutation affecting the function of What treatment options should be
which of the following is likely to be offered?
18 present? A. Achilles tendon lengthening
A. Calcium channel B. Coenzyme Q supplementation
B. Chloride channel C. Mexiletine 150 mg orally 3 times
C. Potassium channel per day
D. Sodium channel D. Prednisone 0.75 mg/kg/d
E. Sodium/potassium pump E. Pyridostigmine 30 mg every 6 h
Neuromuscular
515 18
vv Correct Answer is: E ?? 18. You are evaluating a 15-year-old boy
The diagnosis of Duchenne muscular with progressive running difficulties
dystrophy (DMD) should be your first and occasional falls. These tended to
thought in a 3–5-year-old boy with occur when he was at sports prac-
difficulty running/jumping, toe walk- tice; although he was still active, his
ing, large calves, neck weakness, a athletic performance was deterio-
partial Gower’s sign, and a CK level rating and he was having increas-
greater than3000 U/L. Corticosteroids ing difficulty arising from a chair,
should be offered to all boys with climbing stairs or riding a bicycle.
DMD. Corticosteroids prolong walking; His past medical history is unremark-
transiently increase, and then slow the able expect for tremor beginning at
decline in, muscle strength; reduce falls; age 9 years. On examination, he has
improve pulmonary measures; decrease a normal cranial nerves exam but
scoliosis; and may improve cardiac and mildly reduced muscle tone through-
cognitive function. The timing for initia- out. DTRs were reduced. Proximal
tion of steroids, and the specific dosage strength was reduced with some
to be used remain undefined. Alternate resistance preserved (4/5). He has
regimens of weekend-only dosing have Trendelenburg gait and he could walk
shown similar benefits with fewer long on his toes, but not on his heels. He
term side effects of steroid use (growth has Gower’s sign. Electro diagnostic
and adrenal suppression, bone health, studies performed soon thereafter
hypertension). Achilles tendon lengthen- showed normal motor nerve con-
ing is contraindicated in this disorder, duction velocities with a reduced
with likely worsening of pre-existent compound muscle action potential in
weakness. Early institution of stretching multiple muscles. Sensory nerve con-
and orthotic use is standard of care to ductions are also normal. EMG shows
prevent worsening ankle contractures in evidence of acute denervation and re-­
this disorder. Coenzyme Q supplementa- innervation in several muscles of the
tion has shown no benefit for patients proximal and distal extremities. CK is
with DMD, though might provide benefit elevated at 550. A muscle biopsy was
(in addition to other nutritional supple- performed showing grouped atrophy,
ments) in mitochondrial myopathies. consistent with denervation.
Mexilietine would be helpful for patients The clinical, electro diagnostic and
with myotonic disorders and pyridostig- pathologic manifestations of the
mine for disorders of the neuromuscular case suggest which of the choices as
junction (classic autoimmune myasthe- the most likely diagnosis?
nia gravis or for many of the congeni- A. Becker muscular dystrophy
tal myasthenic syndromes.) Neither of B. Early-onset amyotrophic lateral
these disorders should present with sclerosis
such classic features of DMD, including C. Limb-girdle muscular dystrophy
such a markedly elevated creatine kinase D. Paraneoplastic Lambert-­Eaton
level. mysasthenia
E. Spinal muscular atrophy
zz Suggested Readings
55 The Muscular Dsytrophies. Wicklund, vv Correct Answer is: E
Matthew P. CONTINUUM: Lifelong Learn The clinical presentation demonstrates
Neurol. 2013;19(6):1535–70. progressive, primarily proximal muscle
516 Chapter 18 · Neuromuscular

weakness, which would typically be seen 55 Barohn RJ. Clinical spectrum of motor
primarily with disorders of skeletal muscle neuron disorders. CONTINUUM Lifelong
or neuromuscular junction. However, the Learn Neurology. 2009;15(1, Myasthenic
electro diagnostic testing and biopsy are Disorders and ALS):111–131.
more consistent with disease of the anterior
horn cells. There are no concomitant upper ?? 19. You are following a 9-year-old boy in
motor neuron findings on exam, making the DMD clinic. When he was a tod-
any type of ALS unlikely. The most likely dler, he was diagnosed based on a
diagnosis would be late-onset spinal muscu- workup for elevated liver function
lar atrophy (SMA) type 3 or 4 (also known as tests. At that time, his CPK was found
Kugelberg-­Welander syndrome). Late child- to be elevated at 5880 U and muscle
hood/early adolescence is the typical time biopsy confirmed his diagnosis. His
of presentation with proximal neurogenic examination is significant for lumbar
muscular atrophy that may be confused lordosis and thick calves. When walk-
clinically with LGMD. An elevated CK level ing, he toe walks, has a Trendelenburg
can be seen with later-onset SMA, though is gait and demonstrates a Gower sign
more likely to be elevated with early-onset on arising from the floor.
forms. It is distinguished from early onset What is correct regarding therapeu-
ALS but the absence of UMN signs. Grouped tic interventions for this patient?
atrophy on muscle biopsy is a hallmark of A. as an alternative to chronic cor-
the disease. Genetic testing is available, ticosteroid therapy, immunosup-
showing abnormalities in the survival motor pression with azathioprine should
neuron (SMN) region on chromosome 5. The be considered for long-term
SMN1 gene protein product is important in treatment
the formation of spliceosomes, which are B. chronic corticosteroid treatment
important agents in the processing of pre- should be offered now, using
mRNA into mRNA. Homozygous recessive either prednisone or deflazacort
gene deletions are lethal to motor neurons C. exposure to sunshine and a bal-
but the phenotype is predicated by the pres- anced diet rich in vitamin D and
ence of a second gene, SMN2; more copies calcium has no role in the man-
of SMN2 modify the clinical presentation of agement of these patients
SMA. Patients with 0 or 1 copy of SMN2 typi- D. physical therapy should not
cally present in infancy at the more common include passive stretching exer-
SMA, type I (Werdnig-Hoffman disease), cise, which is associated with
never achieving ability to sit, with death increased pain
from respiratory compromise by age 2, if E. prednisone treatment should be
not supported with mechanical ventilation. deferred until he becomes wheel-
At the other end of the spectrum, with 4 or chair confined
more copies of the SMN2 protein product,
types III and IV can have a normal lifespan, vv Correct Answer is: B
though usually undergo some degree of This is a case of Duchenne muscular
18 progressive muscular weakness at varying dystrophy (DMD). This is an X-linked
ages. degenerative muscle disorder which
affects males in early childhood. Patients
zz Suggested Readings have thickened calves and toe walk.
55 Tiryaki E, Horak HA. ALS and other motor Progression occurs over years, becoming
neuron diseases. CONTINUUM Lifelong wheelchair-bound by 12 years of age. The
Learn Neurol. 2014;20(5):1185–207. AAN and the CNS have published national
Neuromuscular
517 18
practice parameters for the use of cortico- normal appearing motor unit poten-
steroid therapy` in DMD/BMD. tials on needle examination of weak
Recommendations include offering muscles.
boys older than 5 years of age treatment What prognoses is most likely for
with prednisone at a dose of 0.5 mg per this child?
kilogram per day. Recommendations also A. good due to the normal appear-
include the use of deflazacort at 0.9 mg ing motor unit potentials
per kilogram per day. Management should B. good in the pediatric population
include also include physical therapy, overall
especially for passive stretching exercises C. poor due to the absent sensory
to prevent contractures of the iliotibial responses
band, Achilles tendons and flexors of the D. poor due to the low CMAPs
hip. E. poor in the pediatric population
To prevent secondary complications overall
patients should receive exposure to
sunshine as well as a balanced diet rich in vv Correct Answer is: B
vitamin D and calcium to improve bone This is a case of pediatric acute inflam-
density and reduce the risk of fractures. matory demyelinating polyneuropathy
Immunosuppression with azathioprine (AIDP), presenting with Guillain-Barre syn-
has not been demonstrated to be benefi- drome (GBS). In children, the spinal fluid
cial and is not recommended as therapy. examination is normal more commonly
than in adults, and can be normal early
zz Suggested Readings in either population. Sensory responses
55 Darras BT. Muscle diseases. Continuum are frequently absent or abnormal, but
Lifelong Learn Neurol. 2006;12(3):33–75. not universally so. F wave responses are
frequently abnormal as an early finding.
?? 20. You are seeing a 6-year-old boy who Indicators of a potentially poor prognosis
refused to walk. 3 weeks ago, he had in adults (low CMAP amplitudes and fibril-
an upper respiratory tract infection lation potentials) are not necessarily pre-
which resolved with supportive mea- dictors of a poor prognosis in the pediatric
sures. Six days prior to presentation population where series have demon-
he complained of back pain and diffi- strated a favorable outcome overall.
culty walking. This rapidly progressed
to complete refusal to walk over the zz Suggested Readings
next few days. On examination, he 55 Bradshaw DY, Jones HR Jr. Guillain-Barré
has severe leg weakness and milder syndrome in children: clinical course,
arm weakness. He had reduced with- electrodiagnosis, and prognosis. Muscle
drawal to pin to the neck. Muscle Nerve. 1992;15:500–506.
stretch reflexes are absent. He has
normal CSF studies. Motor and sen- ?? 21. You are seeing a 9-year-old girl in the
sory nerve conduction studies show ED for muscle stiffness and an inabil-
low compound muscle action poten- ity to walk after receiving a shot of
tials (CMAPs) and sensory responses epinephrine for a presumed allergic
are unobtainable. There is no conduc- reaction. Her family history is remark-
tion block or temporal dispersion. able for her father has “muscle stiff-
Lower limb F wave responses are ness” on awakening in the morning
unobtainable. There are fibrillation that improves when he does “warm
potentials and reduced recruitment of up exercises.” Her older son was
518 Chapter 18 · Neuromuscular

evaluated for frequent falls and “clum- ?? 22. You are seeing a 9-year-old girl, with
siness” by his pediatrician but no a 7-month history of onset of proxi-
diagnosis was rendered. The patient’s mal weakness, presents for further
neurologic examination is remarkable evaluation of muscle weakness. Her
for well-developed musculature in mother mentioned that she fell on
both lower extremities. Needle EMG her left arm 2 months ago, and a
of these affected muscles demon- plain film shows no fracture, but the
strates ongoing waxing and waning PCP mentioned something about
discharges. “extra calcium”. On examination she
Which of the following abnormal ion has erythema over her cheeks and
channels is most likely responsible elbows and proximal limb weakness.
for her condition? Serum creatine kinase (CK) level is
A. calcium normal. EMG demonstrates fibrillation
B. chloride potentials and short duration, low
C. magnesium amplitude motor unit potentials in
D. potassium proximal muscles, without myotonic
E. sodium discharges. She is admitted to the
hospital with severe abdominal pain
vv Correct Answer is: B and bloody stools.
This is a clinical presentation of myotonia What is your likely diagnosis?
congenita. This is an autosomal inherited A. childhood acid maltase deficiency
condition that affects the function of chlo- B. congenital myotonic dystrophy
ride channels. The autosomal dominant type 1
form, known as Thomsen disease, appears C. early onset myotonia congenita
earlier in childhood, but tends to have D. juvenile dermatomyositis
less severe symptoms than the autosomal E. pediatric mitochondrial myopathy
recessive form known as Becker disease.
People with Becker disease present later vv Correct Answer is: D
in childhood and have a more severe form The subacute onset of proximal muscle
of the disease, characterized by temporary weakness and EMG with fibrillation poten-
attacks of muscle weakness, usually in the tials and myopathic motor unit potentials
upper extremities, often brought on by are relatively nonspecific. The associated
activity after periods of rest. The Becker abnormalities assist with the diagnosis in
form is far more common than the auto- this case. Plain films likely revealed subcu-
somal dominant Thomsen form. The myo- taneous calcification, and she ultimately
tonia is often precipitated by release of likely developed gastrointestinal vasculitis
adrenaline during periods of excitement or leading to intestinal hemorrhage or perfo-
with the use of epinephrine. Symptomatic ration. This is all compatible with juvenile
treatment is available with anticonvulsants dermatomyositis. Serum CK values are not
and mexiletine for severe cases. uncommonly normal and do not exclude
the diagnosis. EMG typically shows fibrilla-
18 zz Suggested Readings tion potentials and myopathic motor unit
55 Bertorini TE, editor. Neuromuscular potentials in the affected muscles. Muscle
disorders: treatment and management. biopsy shows peri-­fascicular atrophy and
Woburn:Butterworth-­Heinemann; 2010. muscle fiber structural changes with a peri-
55 Ropper AH, Samuels MA. Adams and mysial and perivascular predominance of
Victor’s principles of neurology. 9th ed. the inflammatory infiltrate.Mitochondrial
New York: McGraw Hill; 2009. myopathy can be associated with pseudo-
Neuromuscular
519 18
obstruction in some cases or gastrointes- ?? 23. You are evaluating a 7-year-old boy
tinal dysmotility or cachexia. Myotonic for gait disturbance. He has normal
congenita is rarely diagnosed in childhood, birth history; he started crawling at
and myotonic discharges are typical on 10 months and walked at 18 months.
EMG in the pediatric population. Childhood Around the age of 3, he was noted
acid maltase deficiency frequently causes to fall. He has not been able to climb
delayed motor development, proximal stairs, and he has difficulty running.
weakness, and respiratory insufficiency and His general examination is remarkable
failure. Myotonic discharges are frequently for a protuberant abdomen, enlarged
seen. Congenital myotonic dystrophy pres- gastrocnemius muscles, and a wad-
ents at birth rather than in childhood. dling gait. The rest of his family is nor-
mal including his two sisters. Family
zz Suggested Readings history is remarkable for a maternal
55 Shah DU, Darras BT, Markowitz JA, et al. uncle who was wheelchair bound
The spectrum of myotonic and myopathic and died of “heart problems.” His CK is
disorders in a pediatric electromyography markedly elevated (10,000). An EMG
laboratory over 12 years. Pediatr Neurol. shows increased insertional activity,
2012;47:97–100. with fibrillation potentials and small
55 Mastaglia FL, Garlepp MJ, Phillips BA, polyphasic motor unit potentials dif-
Zilko PJ. Inflammatory myopathies: fusely, but most prominent in proximal
clinical, diagnostic and therapeutic muscles. A muscle biopsy is performed
aspects. Muscle Nerve. 2003;27:407–25. and the results are shown.

..      Fig. 18.4 Histopathology of gastrocnemius Dr. Edwin P. Ewing, Jr. Permission: None - This image is
muscle from patient who died of pseudohypertrophic in the public domain and thus free of any copyright
muscular dystrophy, Duchenne type. Cross section of restrictions. As a matter of courtesy. we request that
muscle shows extensive replacement of muscle fibers the content provider be credited and notified in any
by adipose cells. (7 http://phil.cdc.gov/phil/home.asp public or private usage of this image. Wikipedia)
ID#: 70 US Department of Health and Human Services.
520 Chapter 18 · Neuromuscular

What is your diagnosis? C. agrin


A. Duchenne muscular dystrophy D. choline acetyltransferase
B. Nemaline rod E. rapsyn
C. Normal muscle
D. Polymyositis vv Correct Answer is: A
E. Spinal muscular atrophy This is a typical presentation of congenital
myasthenic syndrome (CMS). It was initially
vv Correct Answer is: A described in 1977 with the identification
The pathological slide shows the charac- of a case due to acetylcholinesterase
teristic changes of Duchenne muscular deficiency. Since that time, many families
dystrophy. Duchenne muscular dystrophy have been identified, and through analysis
is caused by a defect at the Xp21 site. it has been determined that mutations
The abnormal gene produces a reduced in the acetylcholine receptor account for
muscle content of dystrophin, a structural the majority of cases, well over 50%. Of
muscle protein. The dystrophin content these mutations, the majority are found
is less than 3% of normal. The incidence in the epsilon subunit of the acetylcholine
of Duchenne dystrophy is 1/3500 male receptor, leading to a marked decrease in
births. The initial features of this disease the amount of receptor available in the
are delayed motor milestones and gait postsynaptic cleft. However, mutations
disturbance. Motor function declines which also alter the kinetics of the channel
throughout childhood. The immedi- have also been defined, so called slow and
ate cause of death may be caused by fast channel mutations. These mutations
arrhythmia, aspiration, and inter-current lead to failure of postsynaptic depolariza-
infection. Respiratory insufficiency is a tion.Mutations in choline acetyl transferase
contributing factor in most cases. lead to failure to produce sufficient ace-
tylcholine. Whereas, mutations in agrin
zz Suggested Readings and rapsyn lead to failure of post-­synaptic
55 Heffner RR. Neuromuscular diseases. aggregation of receptors at the synapse.
From Neuropathology review. AFIP course The absence of the acetylcholinesterase
2002. prolongs the lifetime of acetylcholine in
55 Fenichel GML. Clinical pediatric neurol- the synaptic space, which increases the
ogy: a signs and symptoms approach. 4th duration of the end-plate current so that it
ed. Philadelphia: WB Saunders, 2001;176–8. outlasts the refractory period of the muscle
fiber and excites a second compound mus-
?? 24. You are evaluating 2 siblings, ages 8 cle action potential (CMAP). The prolonged
and 10 in the DMD clinic for weak- end-plate currents lead to overload of the
ness. Both had a weak cry as neonates synaptic space with cations and end-plate
and ptosis by age 1. Ocular ductions myopathy, loss of junctional folds, and loss
were noted to be reduced by age 3. of acetylcholine receptors. Transmission of
Both children fatigue easily and were the CMS is autosomal recessive, except in
never able to run. An EMG shows dec- the case of slow channel syndrome, which
18 remental response which repairs with is inherited as an autosomal dominant trait.
exercise and edrophonium.
In this genetic condition, where zz Suggested Readings
would the highest frequency of 55 Engel AG, Ohno K, Sine SM. Sleuthing
mutations be found? molecular targets for neurological diseases
A. acetylcholine receptor at the neuromuscular junction. Nat Rev.
B. acetylcholinesterase Neurosci. 2003;4(5):339–52.
Neuromuscular
521 18
?? 25. You were asked to see a previously This also, however, may be absent in par-
healthy 6-month-old infant presents ticularly severe cases. Needle examination
to the emergency room with a 2-day usually demonstrates fibrillation poten-
history of irritability and constipation; tials and short duration, low amplitude
then hypotonia, weakness, weak cry, motor unit potentials, with or without
and respiratory distress. On exami- polyphasia.
nation she is hypotonic diffusely,
with diffuse limb weakness, ptosis, zz Suggested Readings
ophthalmoplegia, poor cry and suck, 55 Cornblath DR, Sladky JT, Sumner AJ.
and reduced reflexes. Routine nerve Clinical electrophysiology of infantile
conduction studies show very low botulism. Muscle Nerve. 1983;6:448–52.
compound muscle action potential
(CMAPs) amplitudes, less than 20% of ?? 26. You are evaluating an 18-year-old
normal, in the median and peroneal male for episodes of hand numbness.
motor nerves. Motor nerve conduc- He has noticed hand numbness after
tion velocities and sensory nerve playing video games for long periods
responses are normal. 2 Hz repetitive of time for several years. He reports
nerve stimulation of the peroneal that the tingling and diminished
motor nerve reveals no decrement. sensation would persist for several
On 50 Hz repetitive nerve stimulation days then resolve. He also had similar
for 20 s. sensations after sleeping with his
Which of the following diagnostic arm back behind his head. On one
findings would be expected? occasion he had arm weakness after
A. decrement greater than 10% driving a long distance with his arm
B. decrement greater than 100% resting out the car window. This per-
C. facilitation greater than 10% sisted for 10 days before resolving.
D. facilitation greater than 100% On examination he has mildly re-
E. no decrement or facilitation is duced pinprick on the finger pads of the
diagnostic index fingers bilaterally and very mild
left finger abduction weakness. Muscle
vv Correct Answer is: C stretch reflexes are diffusely diminished.
This is a classic case of infantile botulism. Electro diagnostic studies of both arms
The diagnosis is ultimately made via show prolonged median sensory nerve
bacteriological testing on stool samples, peak latencies bilaterally, and focal slow-
but electro diagnostic findings are quite ing of left ulnar motor nerve conduction
specific. Typical findings include very low on short segment incremental stimula-
CMAP amplitudes at rest. The remainder tion (“inching”) studies localizing to the
of routine nerve conduction studies is medial epicondyle. Median and ulnar
normal. Repetitive nerve stimulation at sensory nerve conduction velocities are
slow rates may produce decrement, but mildly slow.
decrement may be absent when CMAP What is the most appropriate next
amplitudes are very low, such as in this step?
case.At higher rates of repetitive stimula- A. alpha-galactosidase enzyme assay
tion (20 Hz or 50 Hz), facilitation is typical, B. genetic testing
but often does not reach the degree that C. lower limb nerve conduction
is expected in Lambert-Eaton myasthenia studies
syndrome. An increment of 30–200% in D. mRI of the cervical spine
the CMAP amplitude can be expected. E. left upper limb needle examination
522 Chapter 18 · Neuromuscular

vv Correct Answer is: C C. burst suppression on electroen-


Hereditary neuropathy with liability to cephalography
pressure palsies (HNPP) must be consid- D. frontal intermittent rhythmic delta
ered in a young patient with multiple on electroencephalography
mono-­neuropathies at common sites E. low-voltage delta activity on elec-
of compression and diffusely reduced troencephalography
reflexes. Classic findings on EMG include
multiple entrapment neuropathies super- vv Correct Answer is: B
imposed on a mild polyneuropathy, often Bilateral absence of the N20 somato-
characterized by sensory nerve conduc- sensory response to median nerve
tion slowing and distal motor latency stimulation constitutes the most reliable
prolongation. This is often most helpful predictor of non-awakening in post-
when found in the lower limbs where anoxic coma. The other indices listed also
fewer nerves are typically compressed. predict a poor prognosis, but with less
Therefore, lower limb nerve conduction specificity. Bilateral absence of the N20
studies would be the most appropriate somatosensory response to median nerve
next step to evaluate for findings typical stimulation is not, however, diagnostic of
for HNPP.Needle examination would be brain death and can also occur in severe
appropriate to perform following that, bilateral supra-tentorial lesions, drug
and genetic testing for a deletion in intoxication, or severe cerebral edema.
the peripheral myelin protein-22 gene
(PMP22) would be confirmatory. Alpha- zz Suggested Readings
galactosidase enzyme assay would be 55 Guérit JM. Neurophysiological testing in
utilized to evaluate for Fabry disease, also neurocritical care. Curr Opin Crit Care.
presenting in young men but with painful 2010;16:98–104.
neuropathy, angiokeratomas, and renal 55 Carter JL, Stevens JC. Somatosensory
insufficiency. MRI of the cervical spine evoked potentials. In: Daube JR, Rubin DI,
would be appropriate if the history or editors. Clinical neurophysiology. 3rd ed.
examination were compatible with radicu- New York: Oxford University Press; 2009.
lopathy or myelopathy. p. 276.

zz Suggested Readings ?? 28. You are seeing a 16-year-old boy who


55 Anderson PB, Yuen E, Parko K, So YT. is referred to you by his school soccer
Electro diagnostic features of hereditary coach after he noticed slowing in his
neuropathy with liability to pressure speed with running and loss of power
palsies. Neurol. 2000;54:40–4. when kicking the ball. His mother also
noticed that there have been changes
?? 27. The most reliable predictor of non- in the appearance of his running and
awakening (death or vegetative state) walking over the past year. In particu-
in post- anoxic coma is provided by lar, she relates his “steps are shorter”
which of the following neurophysiol- when he runs and that he has a slight
18 ogy tests? “waddle” to his walking. On examina-
A. alpha coma on electroencepha- tion he has a mild, limb-girdle pattern
lography of weakness with associated mild calf
B. bilateral absence of the N20 hypertrophy. His facial and extraocu-
somatosensory response to lar muscles are spared. Serum CK was
median nerve stimulation elevated at 1850 U/L. Needle EMG
Neuromuscular
523 18
of a proximal muscle revealed trace CCTG repeat expansion in the zinc
fibrillation potentials and positive finger protein 9 gene results in proximal
sharp waves, most of the motor units myotonic myopathy (DM2). Calf hypertro-
however were myopathic in appear- phy is not expected, and myotonic
ance and recruitment was notably discharges should be noted on needle
increased. A muscle biopsy is deferred EMG. A D4Z4 repeat contraction on
for genetic analysis. chromosome 4q35 results in facioscapulo-
Which of the following genetic humeral muscular dystrophy. Duplications
defects is most likely to be found in in the PMP22 region are the most common
this boy? genetic cause of Charcot-­Marie-­Tooth
A. A PMP22 duplication of chromo- neuropathy, type 1A, which would
some 17 produce neuropathic rather than myo-
B. Out-of-frame mutation in the pathic electro diagnostic abnormalities.
DMD gene
C. CCTG repeat expansion in the zz Suggested Readings
ZNF9 gene on chromosome 55 Wicklund, Matthew P. The Muscular
19q13.3 Dystrophies. CONTINUUM Lifelong
D. D4Z4 repeat contraction on chro- Learn Neurol. 2013:19(6):1535–70.
mosome 4q35
E. In-frame mutation in the DMD ?? 29. You were asked to see an infant with
gene hypotonia. The child is in respiratory
distress and has a very weak suck. He
vv Correct Answer is: E demonstrates no contractures or dys-
A male patient with a limb-girdle pattern morphic features. You noticed that his
of weakness should be evaluated for a mother has bilateral ptosis with mild
dystrophinopathy even in the absence of perioral weakness. His grandmother
family h
­ istory. The elevated CK supports has some mild loss of temporalis
a myopathic etiology as do the needle bulk does not have the features his
EMG findings. Mildly increased abnormal mother demonstrates. His mother has
spontaneous activity with notably myo- difficulty letting go when she shakes
pathic recruitment suggests a dystrophic your hand. His grandmother does not
myopathic process. An out-of-frame but comments that she notices this at
mutation in the DMD gene disrupts the times when she is in the cold.
reading frame, creating a truncated RNA You suspect a trinucleotide expan-
transcript that is degraded (nonsense-­ sion in which gene?
mediated decay). Because the transcript A. ZNF9
is degraded, there is absence of dystro- B. D4Z4
phin protein, resulting in a more classic C. FXN
Duchenne phenotype, which this patient D. DMPK
certainly does not have, given the pre- E. EMD
served independent ambulation at age 16.
A Becker phenotype (which this patient vv Correct Answer is: D
does resemble) typically results from in- The infant’s presentation suggests con-
frame mutations with retained amino- and genital myotonic dystrophy based on the
carboxy-terminus domains and therefore characteristic features in his undiagnosed
retained function, leading to some dystro- mother, and suggestive features in his
phin protein production. grandmother. This is consistent with
524 Chapter 18 · Neuromuscular

anticipation as occurs in the setting of D. Respiratory muscle involvement


a trinucleotide expansion. Myotonic is common in the most common
dystrophy type 1 is caused by a CTG type of CMT
repeat expansion in the DMPK gene. The E. In some subtypes of CMT4, one of
phenotype in infants (due to a markedly the axonal CMTs, vision or hearing
expanded repeat number most classically loss occurs
inherited through the maternal copy of
the gene) is typically marked peripheral vv Correct Answer is: D
hypotonia. Infants do not have clinically The clinical presentation suggests the
myotonia, so it is critical to briefly examine diagnosis of Charcot-Marie-Tooth (CMT)
the mothers of such infants for features of 1. Respiratory compromise is not usually
more classic myotonic dystrophy. seen in the most common forms of CMT,
ZNF9 mutations results in myotonic but occurs in some cases of CMT2C. The
dystrophy type 2, which does not have CMTs, also known as hereditary sen-
infantile presentation. D4Z4 mutations sorimotor neuropathies or peroneal
result in facioscapulohumeral muscular muscular atrophy, are a large, hetero-
dystrophy; and EMD mutations cause geneous group of inherited peripheral
Emery-Dreifuss muscular dystrophy. neuropathies. The CMTs can be divided
Although infantile onset is rarely reported into demyelinating, axonal, and com-
with either, mom nor grandmother bined demyelinating and axonal forms.
possess features of these disorders. They are genetically heterogeneous. The
Repeat expansion in the FXN (Frataxin) demyelinating CMTs include CMT1 and
gene results in Friedreich’s ataxia, which is CMTX.CMT4, involves both demyelination
autosomal recessively inherited and and axon loss. In the demyelinating CMTs,
presents in the second decade of life with NCS generally show diffuse, uniformly
progressive ataxia amongst other slow conduction velocities without con-
systemic features. duction blocks or temporal dispersion,
indicating a hereditary as opposed to
zz Suggested Readings acquired demyelinating process. There
55 Wicklund MP. The muscular Dystrophies. may be evidence of axon loss in the
CONTINUUM Lifelong Learn Neurol. demyelinating forms as well, particularly
2013;19(6):1535–70. in patients with long­standing disease;
this is secondary axon loss, and conduc-
?? 30. Regarding the various manifestations tion velocities as well as CMAP and SNAP
of the different subtypes of Charcot- amplitudes will be reduced. In contrast,
Marie-Tooth (CMT). with the axonal forms, motor and sensory
Which of the following statements is potential amplitudes will be reduced,
not true? but conduction velocities will be normal
A. Hammertoes and spine deformi- or minimally reduced. At the time of this
ties are more prominent in CMT1 publication, there were seven subtypes
than CMT2 of CMT1; they are all inherited in an auto-
18 B. CMT2 typically has a later age of somal dominant fashion. CMT1A is the
onset as compared to CMT1 most common inherited demyelinating
C. CMT3, or Dejerine-­Sottas syn- neuropathy. Clinical manifestations typi-
drome, presents in infancy and cally begin in the first two decades of life
typically leads to disabling and include slowly progressive weakness,
weakness muscle atrophy, kyphosis, and mild (often
Neuromuscular
525 18
asymptomatic) sensory loss. Other signs The CMT2 group are axonal neuropathies;
include hammertoes, high-­arched feet, NCS show normal conduction velocities,
palpably enlarged nerves due to periph- and nerve biopsy shows axon loss with-
eral nerve hypertrophy (which more out evidence of significant demyelination.
commonly occurs in the CMT1 group Compared to CMT1, in CMT2, symptoms
compared to the other CMTs), and pes and signs typically appear later and foot
cavus. Involvement of the upper extremi- and spine deformities are less severe.
ties typically occurs later in life. There Clinical manifestations do not always help
is often a family history of neuropathy, distinguish the different CMTs, but some
though due to variable expression, some clinical features occur more frequently
affected family members may only have in the different subtypes. Optic atrophy
mild features such as hammertoes and occurs more in CMT2A2, foot ulcerations
may remain undiagnosed for a large part in CMT2B, and vocal cord paralysis,
of their life; sporadic cases without a clear intercostal, and diaphragmatic weakness
family history also exist. Other forms of in CMT2C. In CMT2D, unlike in the other
CMT may be congenital and severe. In the CMTs, the hands are involved more than
CMT1 group, many of the genes involved the feet. Peripheral nerve hypertrophy
are related to myelin synthesis. CMT1A does not occur. They are all autosomal
is due to a duplication in the peripheral dominant in inheritance, except for a
myelin protein 22 (PMP22) gene on chro- subtype of CMT2A. The genes implicated
mosome 17, whereas CMT1 B is due to a in the CMT2 group are involved in axonal
mutation in the myelin protein 0 gene. transport and membrane trafficking.
CMT1 B is more severe in terms of clinical CMT2A2 is one of the most common
manifestations as compared to CMT1A.In of this group and is due to mutations
patients with CMT1, CSF shows elevated in mitofusin 2. CMT3, also known as
protein levels in some cases. Roussy-Levy Dejerine-Sottas syndrome or hypertro-
syndrome is phenotypically similar to phic neuropathy of infancy, is one of the
CMT1A but is associated with the pres- more severe forms of the demyelinating
ence of a static tremor and gait ataxia. CMTs. It presents in infancy with proximal
It has been associated with mutations weakness, absent deep tendon reflexes,
in both the PMP22 and myelin protein and hypertrophy of the peripheral nerves.
O genes. On nerve biopsy, pathologic Prominent sensory symptoms including
features of the demyelinating inherited pain and dysesthesias occur. Patients
polyneuropathies include demyelination typically have extensive disability early in
and an onion-bulb appearance due to life. Both autosomal recessive and domi-
Schwann cell proliferation. Onion bulbs nant forms exist. CSF protein is usually
are not specific for CMT, and also occur elevated. Congenital hypomyelination
with chronic inflammatory demyelinating is seen along the spectrum of this disor-
polyneuropathy. CMTX is the second most der. CMT3 is genetically heterogeneous;
common type of CMT. It is demyelinating mutations in several genes including
and is clinically similar to CMT1, but it is PMP22, protein myelin 0, and other genes
X-linked in inheritance; males therefore implicated in defective demyelination
tend to be more severely affected com- are associated with CMT3.The CMTs can
pared to females. It is due to a mutation be misdiagnosed as muscular dystrophy;
in the connexin 32 gene. CMT2 accounts NCS findings, as well as the presence of
for approximately one-­third of the auto- hammertoes and high-arched foot, distin-
somal dominant inherited neuropathies. guish between the two.
526 Chapter 18 · Neuromuscular

zz Suggested Readings progresses, the proximal leg and hands


55 Bradley WG, Daroff RB, Fenichel GM, become affected. The peripheral myelin
et al. Neurology in clinical practice, 5th ed. protein 22 is a critical component of the
Philadelphia: Elsevier; 2008. myelin sheath. In CMT1 this protein pro-
55 Rapper AH, Samuels MA. Adams and duction is limited and results in improper
Victor’s principles of neurology, 9th ed. myelination and decay of peripheral nerve
New York: McGraw-Hill; 2009. function.
55 Comprehensive Review in Clinical Dystrophin is structural muscle protein
Neurology: A Multiple-Choice Question that is absent or present at very low
Book for the Wards and Boards ©2011 amounts in patients with Duchenne
Wolters Kluwer Health Lippincott muscular dystrophy. Proximal muscles are
Williams & Wilkins. All rights reserved. predominately affected.
The survival motor neuron (SMN)
?? 31. You are seeing a 13-year-old boy for protein is deficient in spinal muscular
“trouble walking”. The child has prob- atrophy type I. This protein is found in the
lems walking up stairs and standing spinal cord at high levels, but is present
on his toes when has to reach for throughout the body. This condition
something. In addition, his mother presents in the first 2 years of life.
has noted he has developed a “very Myelin basic protein is a major
high arch in his feet’’. On neurologi- constituent in the sheath of oligodendro-
cal examination, he has weakness in cytes and Schwann cells. This protein is
peroneal and anterior tibial muscles. found in elevated levels in the spinal fluid
The ankle tendon reflex is diminished of patients with central nervous system
compared to the patellar and brachial demyelinating conditions, such as
reflexes. The boys gait is slow and multiple sclerosis, as well as in central
he lifts his leg up high in order to lift nervous system infections.
his foot of the ground as he steps
forward. zz Suggested Readings
What protein is affected in this 55 Suter U. “MOTOR AND SENSORY
condition? NEUROPATHI ES.” Cell biology and
A. Dystrophin pathology of Myelin: evolving biological
B. Survival motor neuron protein concepts and therapeutic approaches. 4
C. Myelin basic protein (2012):51.
D. Peripheral myelin protein 22
?? 32. You are evaluating a 6-year-old boy
vv Correct Answer is: D with a history of proximal muscle
Charcot-Marie-Tooth 1 (CMT1), also weakness, pain and stiffness for the
referred to as hereditary motor and last 2 months. 2 weeks before this he
sensory neuropathy type1 (HMSN1), is a had fever, generalized fatigue and
demyelinating neuropathy that typically anorexia. On examination he has ery-
begins in the second decade of life and thema and mild edema of the upper
18 progresses gradually throughout the eyelids.
patient’s life. The distal nerves in legs are Electromyography (EMG) results will
affected first causing weakness and foot show which of the following findings?
drop. Pes cavus is described as a high A. Normal
instep/arch, or that develops in several B. Fasciculation at rest with
neurological conditions of hereditary increased mean amplitude of
peripheral neuropathies. As the CMT1 motor unit potentials
Neuromuscular
527 18
C. Increased insertional activity, had distal leg weakness with absent
fibrillations and positive sharp tendon reflexes, and appropriate cog-
waves nitive development?
D. Short-duration, low amplitude What is the most likely diagnosis?
motor unit potential A. Late infantile neuronal ceroid lipo-
fuscinosis
vv Correct Answer is: C B. Gauchers type Ill disease
Dermatomyositis is a systemic inflamma- C. Spinal muscular atrophy type II
tory angiopathy resulting from vascular D. Spinal muscular atrophy type Ill
occlusion and infarction of muscle, skin,
connective tissue, gastrointestinal tract vv Correct Answer is: D
and small diameter peripheral nerves. The age of onset of spinal muscular atrophy
Peak incidence is between 5–10 years. The type Ill is after 18 months, compared to type
initial proximal weakness will eventually 1 at 0–6 months and type II, 6–18 months.
generalize and flexion contractures may Type Ill clinical course is less severe and
develop. The muscle biopsy diagnostic initial symptoms include gait instability,
feature is peri-fasicular atrophy. The fine action tremor, and weakness of distal
inflammatory process may remain active leg muscles or proximal arm muscles. Facial
for 2 years and corticosteroids are used to muscle may be weak, extraocular muscles
suppress the inflammatory response and are not affected and hand strength is the
reduce muscle damage. last muscle groups to be involved.
The EMG study would not be normal. Symptoms of late infantile neuronal
EMG aids in distinguishing between ceroid lipofuscinosis begin between 2 and
myopathic and neuropathic disease 4 years of age and include progressive
processes, and provides supportive cognitive impairment and abnormal
evidence of pathophysiology peripheral movements. Visual impairment begins by
neuropathy and motor neuron disease. 4–6 years of age and progresses to
These EMG findings are most consis- blindness.
tent with a denervation process, such as in Gauchers type Ill presents between the
spinal muscular atrophy type I. ages of 2 and 12 with hepatosplenomeg-
These findings would more likely be aly, impaired bone marrow involvement
found in conditions associated with and progressive motor and cognitive
cholinergic blockade at the neuromuscu- impairment.
lar junction, as seen in infantile botulism. The age of onset on of spinal muscular
atrophy type II is 6–18 months and the
zz Suggested Readings initial feature are delayed motor mile-
55 Mills KR. The basics of electromyography. stones with limited achievement. The
J Neurol Neurosurg Psychiatry. children will attain the ability to sit with
2005;76(suppl. 2): ii32–ii35. balance, but will be unable to stand or
55 Dalakas MC. editors. Polymyositis and ambulate.
dermatomyositis. Butterworth­
Heinemann; 2013. zz Suggested Readings
55 Kaufmann P, McDermott MP, Darras BT,
?? 33. You are evaluating a 4-year-old Finkel R, Kang P, Oskoui M,
boy with a history of normal motor Constantinescu A et al. Observational
development until he began to have study of spinal muscular atrophy type 2
frequent falls at the age of 2. On neu- and 3: functional outcomes over 1 year.
rological exam at 2 years of age, he Arch Neurol. 68(6): 779–86.
528 Chapter 18 · Neuromuscular

?? 34. You are evaluating a 17-year-old male Generalized myotonia may be present.
for episodic weakness and fatigue. He Attacks tend to be shorter than those
states that he tends to wake up with associated with hypokalemic periodic
weakness when he eats Chinese food paralysis, lasting minutes to hours. Rarely,
or pizza for dinner in large quanti- attacks may last days. Weakness tends not
ties. Family history is remarkable of to include the face. Triggers include rest
his father also has episodic weak- after exercise, cold, fasting, and potassium
ness and at one point was brought ingestion; symptoms are alleviated by
to the emergency room when he inhaled beta adrenergic agents and
awoke with weakness after eating a carbohydrate ingestion. Patients are
large pasta meal the night prior. The managed by avoiding high potassium
mother recalls that the father’s potas- meals and by avoiding rest following
sium was low on arrival. strenuous activity. Intravenous calcium,
What is the most likely diagnosis? inhaled beta agonists, or thiazide diuretics
A. Hypokalemic periodic paralysis may be used in acute attacks. Prophylaxis
B. Hyperkalemic periodic paralysis involves avoiding potassium rich foods
C. Myotonia congenital and avoiding strenuous exercise.
D. Paramyotonia congenital Additionally, dichlorphenamide, acetazol-
amide, hydrochlorothiazide, and metapro-
vv Correct Answer is: A terenol may be effective in preventing
This a case of hypokalemic periodic attacks.
paralysis. Hypokalemic periodic paralysis Myotonia congenita may be autosomal
type I is caused by a mutation in calcium dominant or recessive; in each case,
channel, voltage-dependent, L type, alpha mutations in chloride channel, voltage
1S subunit (CACNA 1S) while type II is sensitive 1 (CLCN1) may be causative.
caused by sodium channel, voltage gated, Thomsen disease is the autosomal
type IV alpha subunit (SCN4A) mutations. dominant form; in this case, patients may
Typically, weakness follows emotional present in early childhood or in infancy. A
stress, ingestion of carbohydrates or alco- “warm up” phenomenon is present where
hol, rest after exercise, or cold. In particu- stiffness improves with repeated move-
lar, weakness may occur in the morning ment. Becker type myotonia is the
following a large carbohydrate meal the autosomal recessive form. This tends to
night prior. Attacks can be hours to days present later in life although the “warm
and the onset of the disease is usually up” phenomenon is present in this form
in adolescence. Weakness may progress too. Weakness may occur following
to flaccid quadriplegia and deep tendon myotonia in the recessive form. In both
reflexes may be lost. Respiratory muscles forms, stiffness is more prominent in the
may be affected. Preventive treatments cold. Para-myotonia congenita is caused
include potassium sparing diuretics such by mutations in sodium channel, voltage
as spironolactone or daily acetazolamide gated, type IV alpha subunit (SCN4A), as in
or dichlorphenamide acutely, patients hypokalemic periodic paralysis type II and
18 may receive potassium supplementation. hyperkalemic periodic paralysis in
Hyperkalemic periodic paralysis tends individuals with para-myotonia congenita,
to present earlier than hypokalemic the “warm up” phenomenon is not present
periodic paralysis. This is also autosomal and repeated movements may worsen
dominant and typically caused by myotonia and weakness; cold is also a
mutations in sodium channel, voltage potential trigger. Pharyngeal muscles and
gated, type IV alpha subunit (SCN4A). muscles of the hand and face are all
Neuromuscular
529 18
affected. Avoidance of cold, the use of zz Suggested Readings
mexiletine (a lidocaine derivative), and 55 Chiou-Tan FY, Gilchrist JM. Repetitive
hydrochlorothiazide are all preventive. In nerve stimulation and single-fiber
particular, mexiletine is used to prevent electromyography in the evaluation of
cold induced weakness. patients with suspected myasthenia gravis
or Lambert-Eaton myasthenic syndrome:
?? 35. You are seeing an 18-year-old female review of recent literature. Muscle Nerve.
for generalized weakness. On electro 2015;52(3):455–62.
diagnostic testing the initial compound
muscle action potentials (CMAP) were ?? 36. You are seeing an8 month-old girl
normal. After a train of 6 stimuli at a who developed mild to moderate
rate of 3 per second, the amplitude of generalized hypotonia over the last
the CMAP’s decreased by 35%. 3 months and the electromyog-
What is the likely diagnosis? raphy study revealed fibrillations,
A. Dermatomyositis fasciculation, sharp waves on needle
B. Guillain-Barre syndrome ­insertion at rest with large, pro-
C. Myasthenia gravis longed and polyphasic motor unit
D. Hypothyroidism potentials.
What disease classification do these
vv Correct Answer is: C findings support?
The test of the neuromuscular junction A. Neuropathic process
is based on the Jolly principle that first B. Myopathic process
described the electro diagnostic findings C. Mitochondrial disorder
in patients with myasthenia gravis in D. Inconclusive
which the strength of muscle contractions
is progressively diminished during a train vv Correct Answer is: A
of electrical stimuli. This progressive fail- The needle electrode examination is
ure of neurotransmission is similar to the accomplished by recording the firing
ones found with administration of curare response during the muscle insertion of
(or other non-­depolarizing neuromuscular spontaneous and then voluntary muscle
blockers) and can be partially corrected by activity. Denervation disease results in
neostigmine. fibrillations, fasciculation, sharp waves
Electro diagnostic findings in dermato- on needle insertion at rest with large,
myositis include myopathic features of prolonged and polyphasic motor unit
increased insertional activity, fibrillations potentials. Myopathic disease is character-
and positive sharp waves. ized by brief, small­amplitude, polyphasic
Electro diagnostic findings in Guillain-­ potentials.
Barre syndrome include a reduction in Mitochondrial myopathies will have
amplitudes of muscle action potentials myopathic features on electromyography
slowed conduction velocity, or complete studies.
conduction block in motor nerves. The Fibrillations, fasciculation and sharp
initial symptoms in this condition are waves are pathological findings.
most commonly paresthesias and
numbness. zz Suggested Readings
Neuropathy associated with hypothy- 55 Brown WF. The physiological and
roidism is not common and is character- ­technical basis of electromyography.
ized by a significant reduction in nerve Butterworth­ Heinemann;
conduction velocities. 2013.
530 Chapter 18 · Neuromuscular

?? 37. Charcot-Marie-Tooth (CMT) type 1 dif- zz Suggested Readings


fers from CMT type 2 in that: 55 Patzko A, Shy ME. Charcot-Marie-Tooth
A. Type 1 is an axonal neuropathy disease and related genetic neuropathies.
B. Type 1 is associated with upper Continuum. 2012;18(1):39–59.
extremity nerve conduction veloc-
ities >40 mis ?? 38. You are evaluating a 15-year-old male
C. Type 2 has autosomal recessive with intermittent muscle pain and
inheritance weakness when he exercises, with nor-
D. Type 1 is a severe infantile form of mal motor status between events and
CMT age appropriate cognitive abilities?
E. None of the above What test result will support your
diagnosis?
vv Correct Answer is: E A. Serum CK levels
CMT is currently classified as type 1, type B. Impaired lactate response to isch-
2, and type 4. The term “type 3” (Dejerine-­ emic exercise test
Sottas syndrome) is no longer used, with C. Fasciculation and fibrillations on
Dejerine-Sottas syndrome now referring EMG
to severely affected infants with any D. Low ammonia response to fore-
type of CMT. The letter designations (e.g. arm ischemia test
CMT1A, CMT2B, etc.) indicate the different
causative mutations. vv Correct Answer is: D
Type 1 includes hypertrophic demy- Myoadenylate deaminase deficiency pres-
elinating neuropathies with nerve conduc- ents with several different phenotypes;
tion velocities (NCVs) slowed to less than infants with hypotonia, children with
38 mis in the upper extremities. The progressive myopathies and recurrent
disorder is of autosomal dominant rhabdomyolysis and in children and adults
inheritance, with mutations including with exercise intolerance. The forearm isch-
duplications of the peripheral myelin emia test helps in the differential diagnosis
protein gene, PMP22 (in CMT1A) and of muscle pain and weakness associated
mutations of the myelin protein zero with exercise. Patients with myoadenylate
gene, MPZ (in CMT1 B). CMT1X, although deaminase deficiency do not generate
X-linked, is categorized as a CMT type 1 ammonia but have appropriately elevated
disorder as it is a demyelinating process. lactate levels in ischemic conditions. The
CMT1X is caused by mutations of the GJB1 pain associated with the exercised induced
gene, which encodes the protein phenotype, is described as diffuse aching
connexin-32. to more severe localized cramping, along
Type 2 includes axonal neuropathies with muscle swelling and tenderness.
with relatively normal nerve conduction Duchene muscular dystrophy presents
velocities. They are also of autosomal before the age of 5 with a gait disturbance
dominant inheritance, with mutations in with proximal leg weakness.
the mitofusin 2 gene, MFN2 (in CMT2A) McArdle disease is a myophosphorylase
18 and the RAB7A gene of the RAS oncogene deficiency that limits the first step of
family (in CMT2B). glycogenolysis in muscle and therefore
Type 4 includes the CMT subtypes of results in low glucose availability in muscle
autosomal recessive inheritance. cells. Serum CK levels, mutational analysis
Neuromuscular
531 18
and muscle biopsy are used for diagnosis. test (also called an ischemic exercise
This condition may present with exercise test or ischemic forearm test) may aid in
intolerance and weakness but usually is not diagnosis. In this test, the arm muscles
diagnosed until the person is in their 20’s. are contracted in the setting of restricted
Denervation neuropathies are blood flow, with an inflated blood pres-
progress live disorders and will not have sure cuff on the upper arm. Patients will
intermittent symptoms with complete develop pain and contractures. Following
resolution. the exercise, bloodwork demonstrates
that ammonia levels are elevated, while
zz Suggested Readings lactic acid levels do not rise. This contrasts
55 Devanshi J, Kaminski HJ. Rhabdomyolysis with myoadenylate deaminase deficiency,
and Myoglobinuria. In: Neuromuscular in which patients will have elevated lactic
disorders in clinical practice. Springer: acid levels, but fail to increase ammonia.
New York; 2014. p. 1545–59
zz Suggested Readings
?? 39. Myophosphorylase deficiency (McAr- 55 Amato AA and Brooke MH. Disorders of
dle’s disease) is characterized by: skeletal muscle. In: Bradley WG, Daroff RB,
A. Autosomal dominant inheritance Fenichel GM, jankovic j, editors.
B. Prominent distal weakness Neurology in Clinical Practice (4th
C. Lack of a second wind phenom- edition). Elsevier, Inc.; 2004. p. 2463–2510.
enon
D. Elevations of ammonia and lactic ?? 40. You are evaluating a 2-year-old girl is
acid in the exercise forearm test because of she would never sweat no
E. None of the above matter what fluctuations occurred in
her body temperature. She had suf-
vv Correct Answer is: E fered from recurrent fevers since child-
McArdle’s disease results from a disrup- hood, but an infectious etiology to her
tion of the carbohydrate metabolism fevers could not be identified. Her fam-
chain due to a deficiency of myophos- ily noticed that she did not cry when
phorylase. It is caused by mutations of she received her vaccinations, and since
chromosome 11, of autosomal recessive she had started walking, if she bumped
inheritance. Onset is during childhood. into an object or fell, she did not seem
Initially, children tire easily, then prog- to feel pain, even when she sustained a
ress to the classic symptoms of pain significant abrasion or bruise. She has
and contractures with exercise. An EMG also been seen by a psychiatrist for self-
performed during a contracture, however, mutilating behaviors (severe lip biting)
would not demonstrate any muscle activ- and intellectual delay.
ity, differentiating this phenomenon from What is your most likely diagnosis in
a true muscle cramp. Patients experience this patient?
a second wind phenomenon, in which A. Hereditary sensory and auto-
exercise intolerance may be overcome by nomic neuropathy (HSAN) type 1
brief rests. Most patients have no weak- B. Charcot-Marie-Tooth 1
ness. When present, weakness is typically C. HSAN type
mild and affects proximal muscles and D. HSAN type
upper extremities. An exercise forearm E. HSAN type 4
532 Chapter 18 · Neuromuscular

vv Correct Answer is: E What is the most likely diagnosis?


The HSANs are a relatively rare, geneti- A. Hyperkalemic periodic paralysis
cally and phenotypically heterogeneous B. Hypokalemic periodic paralysis
group of hereditary neuropathies. As a C. Andersen-Tawil syndrome
group, they share in common prominent D. Paramyotonia congenital
sensory signs and symptoms, including E. Myotonia congenital
pain, sensory loss, and autonomic features
with little motor involvement. Because of vv Correct Answer is: B
the sensory loss, patients with HSANs are This is a clinical presentation of hypo-
prone to painful calluses, stress fractures, kalemic periodic paralysis, which is
neuropathic (Charcot) joints, skin ulcer- inherited in an autosomal dominant
ations that heal poorly, and infections fashion; however, sporadic cases have
with deep tissue involvement, such as been reported. There are two types: type
osteomyelitis, leading to disfiguring acral 1 caused by a mutation in the calcium
mutilations. Hereditary sensory and auto- channel gene CACNA 1S on chromosome
nomic neuropathy type I (HSAN I) is the 1q31, and type 2 caused by a mutation in
most frequent type of hereditary neuropa- the sodium channel gene SCN4A. These
thy that primarily affects sensory neurons. patients present with episodes of weak-
Hereditary sensory and autonomic neu- ness without myotonia that could be focal
ropathy type II (HSAN2) is characterized or generalized and ranges from mild to
by progressively reduced sensation to severe, associated with hyporeflexia dur-
pain, temperature, and touch. ing the attacks, each of which can last for
Hereditary sensory and autonomic hours, with some persistent mild weak-
neuropathy type Ill features marked ness for a few days. During attacks, the
ataxic gait that progressively worsens creatine kinase level may be elevated and
over time. HSAN 4 is characterized by the serum potassium is usually reduced.
insensitivity to pain, anhidrosis, and These episodes are triggered by exercise,
intellectual disability as well as self-­muti- meals rich in carbohydrates, ethanol,
lation. Sense of touch, vibration, and cold exposure, and emotional stressors.
position are normal. Congenital insensi- Provocative testing can be done with glu-
tivity to pain with anhidrosis is extremely cose administration. The main treatment
rare entity, in which patients are sub- focus is to avoid triggers; carbonic anhy-
jected to repeated injuries which are drase inhibitors such as acetazolamide
often neglected. There is no specific and potassium-sparing diuretics are useful
treatment but patient training and family in treating this condition. Hyperkalemic
education are key to avoid further neglect periodic paralysis, on the other hand,
and damage. presents with episodes of weakness trig-
gered by resting after exercise and fasting.
?? 41. You are seeing an 18-year-old boy This condition is autosomal dominant and
with episodes of weakness and is caused by a mutation in the sodium
inability to move his arms and legs. channel gene SCN4A. Provocative testing
18 Extraocular movements and bulbar is done with administration of potassium.
and respiratory muscles are spared. The treatment is focused on avoiding
The patient says that he gets these triggers. During attacks, glucose can be
episodes with emotional stressors, provided, and as prophylactic therapy,
after exercise, or after eating heavy thiazide diuretics can be used. Andersen-
desserts. His family history is remark- Tawi l syndrome is also a channelopathy
able of his father has similar episodes. characterized by periodic paralysis,
Neuromuscular
533 18
ventricular arrhythmias, and dysmorphic impaired bone marrow involvement
features. It is associated with a mutation and progressive motor and cognitive
in the potassium channel gene KCN J2. impairment.
The age of onset on of spinal muscular
zz Suggested Readings atrophy type II is 6–18 months and the
55 Comprehensive review in clinical neurol- initial feature are delayed motor mile-
ogy: a multiple-choice question book for stones with limited achievement. The
the Wards and Boards © 2011 Wolters children will attain the ability to sit with
Kluwer Health Lippincott Williams & balance, but will be unable to stand or
Wilkins. All rights reserved. ambulate.

?? 42. You are evaluating a 4-year boy with a zz Suggested Readings


history of normal motor development 55 Kaufmann P, McDermott MP, Darras BT,
until he was first noted to begin to Finkel R, Kang P, Oskoui M,
have frequent falls at the age of 2, he Constantinescu A et al. Observational
had distal leg weakness with absent study of spinal muscular atrophy type 2
tendon reflexes, and appropriate cog- and 3: functional outcomes over 1 year.
nitive development on his exam at Arch Neurol. 2011;68:(6):779–86.
this time.
What is the most likely diagnosis? ?? 43. You are seeing a 7-year-old boy with
A. Late infantile neuronal ceroid lipo- a muscular dystrophy was normal
fuscinosis at birth, but at 4 years of age, was
B. Gauchers type Ill diseas unable to jump or run properly. He
C. Spinal muscular atrophy type II developed calf pseudo-hypertrophy,
D. Spinal muscular atrophy type Ill and difficulty standing when seated
on the floor. He now reports difficulty
vv Correct Answer is: D climbing, with frequent falls due to
The age of onset of spinal muscular weakness.
atrophy type Ill is after 18 months, com- What do you expect to see on your
pared to type I at 0–6 months and type II, evaluation?
6–18 months. Type Ill clinical course is less A. An absence of dystrophin staining
severe and initial symptoms include gait on muscle biopsy
instability, fine action tremor, and weak- B. An EMG with myopathic changes
ness of distal leg muscles or proximal arm C. A frameshift mutation at the locus
muscles. Xp21
Facial muscle may be weak, extraocu- D. An electrocardiogram with tachy-
lar muscles are not affected and hand cardia
strength is the last muscle groups to be E. All of the above
involved.
Symptoms of late infantile neuronal vv Correct Answer is: E
ceroid lipofuscinosis begin between 2 and Duchenne’s muscular dystrophy (DMD) is
4 years of age and include progressive the most common form of muscular dys-
cognitive impairment and abnormal trophy, affecting 1 in 3500–5000 newborn
movements. Visual impairment begins by males. DMD is less common in girls, as it is
4–6 years of age and progresses to an X-linked recessive disorder. The muta-
blindness. tion is at the Xp21 locus, and in DMD (in
Gauchers type Ill presents between the contrast to Becker’s muscular dystrophy),
ages of 2 and 12 with hepatosplenomegaly, is most often a frame shift mutation. The
534 Chapter 18 · Neuromuscular

mutation results in absence of dystrophin, What is the most likely diagnosis?


which forms part of the support structure A. Walker-Warburg syndrome
that anchors the contractile proteins to the B. Muscle-Eye-Brain disease
outside of the muscle cell. Affected patients C. Phosphoglycerate mutase
are normal at birth, and like most toddlers, deficiency
are clumsy when learning to walk. The D. Welander myopathy
clumsiness, however, may persist, and chil- E. Phophofructokinase deficiency
dren do not learn to jump or run properly.
Patients have difficulty when rising from vv Correct Answer is: A
the floor, and may need to use their hands Walker-Warburg syndrome consists
to push up against their knees to assist of muscular dystrophy, with cerebral
(Gower’s maneuver). Calf pseudo-hyper- and eye malformations. Retinal and
trophy develops. Patients begin to have eye malformations are severe, and may
difficulty climbing and have frequent falls, include microphthalmia, cataracts, retinal
eventually requiring the use of a wheel- dysplasia, and optic atrophy. Cerebral
chair. Contractures develop at the hips, malformations include aqueductal ste-
knees, and ankles. Cardiac musculature is nosis, pontine hypoplasia, Dandy-Walker
involved, with a dilated cardiomyopathy, malformations, and agyric or pachygyric
degeneration and fibrosis of the postero- cortices. A “cobblestone” lissencephaly
lateral wall of the lateral ventricle, and/or is common, with nodular, disorganized
valvular and wall motion abnormalities. An cortex of variable thickness. MRI may
abnormal EKG is common, often with tachy- also demonstrate white matter changes.
cardia, even in young children. Evaluation Death typically occurs in 2 years. The
reveals an elevated creatine kinase (CK) disorder is caused by a mutation of chro-
level, and an EMG with myopathic changes. mosome 9 and is of autosomal recessive
The diagnosis may be established by DNA inheritance. Muscle-eye-brain disease also
testing or muscle biopsy. Biopsy often dem- presents with muscular dystrophy, but
onstrates muscle necrosis, phagocytosis, the associated cerebral and eye findings
fiber size variation, fibrosis, basophilia, and are of a much less severe phenotype than
hyaline fibers, with the absence of staining evident with Walker-Warburg syndrome.
for dystrophin. Patients may experience Patients may have myopia and a pre-
some benefits to strength and function retinal membrane or gliosis, but there
from administration of prednisone. Death, are no major malformations. The cortical
usually in early adulthood, is typically due changes are also less severe, with focal
to respiratory failure or cardiomyopathy. white matter changes. Muscle-eye-brain
disease is due to a m
­ utation in the gene
zz Suggested Readings for 0-mannose-1, 2 N-­acetylglucosaminyl
55 Amato AA, Brooke MH. Disorders of transferase, located on chromosome 1,
skeletal muscle. In: Bradley WG, Daroff RB, and is of autosomal recessive inheritance.
Fenichel GM, Jankovic J, editors. Phosphoglycerate mutase deficiency
Neurology in Clinical Practice. 4th ed. is a rare autosomal recessive disorder,
18 Elsevier, Inc.; 2004. p. 2463–510. due to mutations on chromosome 7. It
presents with attacks of muscle pain and
?? 44. You are seeing a 4-month-old boy myoglobinuria, of onset in childhood or
was hypotonic at birth, with microph- adolescence. Welander myopathy is a dis-
thalmia and cataracts. MRI reveals order of adult onset (40–60 years of age),
hydrocephalus with cobblestone lis- with weakness of the hands followed by
sencephaly. foot drop. It may also be associated with a
Neuromuscular
535 18
distal small fiber neuropathy. The disorder muscle disease, affecting 1 in 3500 live
is of autosomal dominant inheritance. male births. About a third of the cases
Phophofructokinase deficiency affects are caused by spontaneous mutations in
the conversion of fructose-6- phosphate the dystrophin gene, so there may not be
to fructose-1, 6-diphosphate in the a positive family history. This condition
glycolytic chain. It results in nausea, presents early in life and manifests with
vomiting, and muscle pain elicited by weakness and delayed development of
exercise. An infantile-­onset type is motor milestones. These children have
associated with blindness and psychomo- frequent falls, and difficulty walking, run-
tor retardation, while a late­onset type is ning, and rising from supine and sitting
associated with cardiac dysfunction. positions. Weakness is significant in the
The disorder is due to a mutation on proximal muscles, predominantly in the
chromosome 1, of autosomal recessive iliopsoas, quadriceps, and gluteal muscles,
inheritance. as well as the shoulder girdle and upper
limbs. It also tends to affect the pretibial
zz Suggested Readings muscles. These patients have pseudo-
55 Amato AA, Brooke MH. Disorders of hypertrophy of the calves due to fibrosis.
skeletal muscle. In: Bradley WG, Daroff RB, They also have scapular winging and
Fenichel GM, Jankovic J, editors. contractures.
Neurology in clinical practice. 4th ed. Ocular, facial, and bulbar muscles are
Elsevier, Inc.; 2004. p. 2463–510. usually spared. Cardiac involvement
includes arrhythmias, cardiomyopathy,
?? 45. You are evaluating a 7-year-old boy and heart failure. Mild intellectual
with motor development delay and disability with a subnormal intelligence
difficulty walking. His weakness quotient (IQ) may be seen. These children
is predominantly in the proximal become wheelchair- or bed-bound, and
muscles, with severe weakness of hip may eventually die of respiratory failure
flexors, quadriceps, gluteal, and pre- and pulmonary infections. CPK values
tibial muscles. He also has pectoral range from 10 to 100 times higher than
and shoulder weakness and wing- normal. Needle EMG shows fibrillations,
ing of both scapula. His calves are positive waves, and myopathic motor unit
enlarged, with a “rubbery” texture. potentials. In general, myofiber necrosis,
His intelligence quotient (IQ) is low. degeneration, regeneration, increased
His serum CPK is 11,560 IU/L (normal fiber size variation, endomysial inflamma-
220 IU/L). tion, and fibrosis are collectively called
What is the most likely diagnosis? dystrophic changes, which are characteris-
A. Becker muscular dystrophy tic for Duchenne muscular dystrophy, but
B. Duchenne muscular dystrophy not specific, and can be seen in other
C. Emery-Dreifuss muscular dystrophy dystrophies. Dystrophin is the affected
D. Fascioscapulohumeral muscular gene product and is absent in patients
dystrophy with Duchenne. Dystrophin is a cytoplas-
E. Limb-girdle muscular dystrophy mic protein that binds and interacts with
other intracytoplasmic proteins such as
vv Correct Answer is: B F- actin, and is linked to the trans-sarco-
This is a case of Duchenne muscular lemmal dystroglycan protein complex,
dystrophy. This condition is a dystrophi- providing a structural link between the
nopathy, inherited in an X-linked recessive sub-sarcolemmal cytoskeleton and the
fashion, and is the most common genetic extracellular matrix. If dystrophin is lost or
536 Chapter 18 · Neuromuscular

altered, this sarcolemmal structure is channel gene CLCN1 on chromosome


affected, making it susceptible to rupture. 7q. The main manifestation is myotonia,
The diagnosis of Duchenne muscular which is an impaired muscle relaxation as
dystrophy is made by genetic testing of seen when the patients cannot relax their
the dystrophin gene or by absent dystro- handgrip after grasping an object, and
phin immunostaining on muscle biopsy. also manifested on percussion, leading
to contraction and delayed relaxation.
zz Suggested Readings Myotonic potentials can be detected on
55 Comprehensive review in clinical neurol- EMG. In myotonia congenita, as opposed
ogy: a multiple-choice question Book for to para-­myotonia, there is a “warm-up”
the Wards and Boards ©2011 Wolters phenomenon, in which the myoto-
Kluwer Health Lippincott Williams & nia improves after repetitive muscle
Wilkins. All rights reserved. activation.
Thomsen’s disease is autosomal
?? 46. You are seeing a 7-year-old boy for dominant and begins in the first decade of
stiffness. His legs are stiff, and he life manifesting with painless myotonia,
has difficulty relaxing muscles after but not weakness, and is milder than the
contracting them. He cannot release recessive form. Becker’s disease is the
objects easily once grasped with his recessive form that presents later, usually
hands, and when he closes his eyes, in the second decade of life, is more
it takes a few seconds before he is severe than the dominant form, and may
able to open them completely. If he manifest with weakness after severe
performs the same motor task mul- episodes of myotonia. Propofol and
tiple times, it becomes easier. When depolarizing neuromuscular blocking
his thenar eminence is percussed, agents may aggravate the myotonia, and
there is prolonged contraction and patients may have prolonged recovery.
delayed relaxation. Family history is Mexiletine is the treatment of choice.
remarkable of his father and grandfa-
ther have similar clinical features that ?? 47. You are evaluating a 16-year-old boy
started around the same age. for acute onset of ataxia and dysar-
What is the most likely diagnosis? thria after he recovered from a strep-
A. Paramyotonia congenital tococcal pharyngitis, associated with
B. Myotonia congentia, Becker’ fever, and was successfully treated
disease with penicillin. He had difficulty run-
C. Myotonia congenita, Thomsen’s ning with several sprained ankles
disease since early childhood, but is other-
D. Hyperkalemic periodic paralysis wise healthy. He has a family history
E. Hypokalemic periodic paralysis of Charcot-Marie-­Tooth (CMT) neu-
ropathy. His MRI showed acute white
vv Correct Answer is: C matter changes, particularly within
This boy has myotonia congenita, more the posterior fossa.
18 specifically Thomsen’s disease. There The acute neurological changes are
are two types of myotonia congenita: most likely related to an underlying:
Thomsen’s disease, which is autosomal A. CMT1A
dominant, and Becker’s disease, which B. CMT2A
is autosomal recessive. This group of C. CMT1X
disorders is caused by a channelopathy D. CMT2A
secondary to a mutation in the chloride E. CMT4C
Neuromuscular
537 18
vv Correct Answer is: C What is most likely to cause the
CMT1X neuropathy is associated with muscle weakness in periodic hyper-
acute episodes of ataxia, dysarthria, kalemic paralysis?
and in some cases, asymmetric weak- A. Decreased potassium conduc-
ness. White matter changes are evident, tance in muscle cells
particularly within the posterior fossa, B. Hyperpolarization of muscle cells
during these episodes. The symptoms C. Inactivation of sodium channels in
resolve in weeks to months, and typically muscle cells
do not recur. They are often triggered D. Increased duration of action
by metabolic stressors such as travel to potentials produced by
high altitudes, febrile illnesses, hyper- a-­motoneurons
ventilation, and concussion. These CNS E. Increased release of neurotrans-
changes have been reported in patients mitters from a-­motoneurons
with CMT1X due to TSSI, R75W, E102del,
R142W, R164W and C168Y mutations of vv Correct Answer is: C
the GJB1 gene, which encodes the protein Periodic hyperkalemic paralysis is a dis-
connexin-­32. order of muscle membrane excitability
The presence of CNS symptoms does resulting from a sodium channel disorder.
not correlate with the duration or Inactivation of the sodium channels on the
severity of neuropathy. The neuropathy skeletal muscle membrane prevents action
typically presents as distal lower extrem- potentials from being produced, and there-
ity weakness, with slowed running and fore leads to muscle weakness or paralysis.
frequent sprained ankles, typically Although the exact mechanism of periodic
beginning by 10 years of age in affected hyperkalemic paralysis is not known, it
males. Hands, particularly the thenar appears to be due to a mutation in the gene
muscles, may be affected later in the coding for the sodium inactivation gate.
course. Distal sensory loss with painful Attacks are usually frequent, focal, short
sensory paresthesia may also occur. duration (usually 15 min to 1–2 h, up to
Heterozygous females may be 4–5 h); may occur several times daily. May
asymptomatic, but often have mild be precipitated by exposure to cold, rest
manifestations. after exercise or oral intake of potassium,
emotional stress, and pregnancy. Episodes
zz Suggested Readings more frequent when poor oral intake and
55 Kleopa KA. The role of gap junctions in high potassium diet. Myotonia may or may
Charcot-Marie-Tooth disease. J Neurosci. not be present, serum potassium measured
2011;31(49): 17753–60. during an attack is often normal but may be
55 Patzko A, Shy ME. Charcot-Marie-Tooth high or even low. Myotonic discharges may
disease and related genetic neuropathies. be present on EMG. Treatment - ingestion of
Continuum. 2012;18(1):39–59. carbohydrates and sugars may prevent or
abort attack; prophylaxis with hydrochloro-
?? 48. You are seeing a 9-year-old female thiazide and carbonic anhydrase inhibitors
who experiences 2–4 episodes of (acetazolamide).
proximal muscle weakness per day.
The attacks are precipitated by rest zz Suggested Readings
following exercise and last 1 to 2 h. 55 Mowzoon N, Flemming KD. Neurology
Serum potassium concentration Board Review: an illustrated study guide.
­during the attack of weakness is Rochester: Mayo Clinic Scientific; 2007.
normal. Print.
538 Chapter 18 · Neuromuscular

?? 49. You are seeing an infant with gen- include early onset of hypotonia, followed
eralized hypotonia and weakness, by diffuse limb weakness, with mild weak-
you ordered a muscle biopsy, with a ness of the face and other bulbar muscles.
Gomori trichrome stained, which is The face is typically long and narrow, with
shown. an abnormally long or short jaw. Other
malformations include high arches of the
feet and palate and kyphoscoliosis. An
associated cardiomyopathy may occur. A
fatal infantile form consists of hypotonia
and respiratory failure.

zz Suggested Readings
55 Image adapted from: Lawlor MW, et al.
Novel mutations in NEB cause abnormal
nebulin expression and markedly impaired
muscle force generation in severe nema-
line myopathy. Skelet Muscle.
2011;1(1):23.
55 Amato AA, Brooke MH. Disorders of
skeletal muscle. In: Bradley WG, Daroff RB,
..      Fig. 18.5 Muscle biopsy showing rod-like inclusions Fenichel GM, Jankovic J, editors.
in a myofiber as seen in Nemaline myopathy (Gomori Neurology in Clinical Practice. 4th ed.
trichrome stain). (Own work. Jensflorian. Wikipedia)
Elsevier, Inc.; 2004. p. 2463–510.

What is the most likely diagnosis? ?? 50. Which of the following is correct
A. Central core disease regarding Lambert-Eaton Myasthenic
B. Nemaline myopathy Syndrome (LEMS)?
C. Centronuclear myopathy A. Presynaptic stores of acetylcho-
D. Myofibrillar myopathy line are decreased
E. Congenital fiber type disproportion B. Antibodies are ­typically present
against post-synaptic motor nerves
vv Correct Answer is: B C. Antibodies are most commonly
Gomori trichrome staining of the muscle directed against voltage-­gated
biopsy shows red punctuate inclusions potassium channels
within the skeletal muscle cytoplasm, D. All of the above
representing nemaline rods. Rods may E. None of the above
be visible with electron microscopy, as
well. They are composed of alpha-actin, vv Correct Answer is: E
desmin, and nebulin. Biopsies in nemaline In LEMS, the quantal release of acetylcho-
myopathy also reveal type 1 fiber pre- line is impaired, although the presynaptic
dominance, type 1 atrophy, and type 28 stores of the neurotransmitter and the
18 deficiencies. Both autosomal dominant postsynaptic response remain normal. The
and recessive forms of nemaline myopa- result is a syndrome including proximal
thy have been documented, with various weakness, hyporeflexia, and dysautonomia
mutations affecting alpha-­tropomyosin, (i.e. dry mouth), with particular involve-
beta­tropomyosin, nebulin, troponin T, ment of the lower extremities. Ocular find-
and alpha-actin. Clinical manifestations ings such as ptosis and opthalmoparesis are
Neuromuscular
539 18
less common, and milder, than in myasthe- B. It is due to inhibition of exocytosis
nia gravis. Symptoms may worsen in heat of presynaptic vesicles containing
or with febrile illnesses. A characteristic fea- acetylcholine
ture of LEMS is that rapid repetitive stimula- C. In its most common form, it is
tion or voluntary activity aids in the release autoimmune due to antibod-
of quanta to allow generation of muscle ies against the acetylcholine
action potentials. Clinically, this may be evi- receptor
dent as an increased response of previously D. It results from increased degrada-
hypoactive reflexes after contraction of the tion of acetylcholine at the neuro-
associated muscle, for example. The phe- muscular junction
nomenon may also be evident with facilita- E. In its most common form, it is due
tion of compound muscle action potential to mutations in the acetylcholine
(CMAP) amplitudes with repetitive nerve receptor gene
stimulation on EMG/NCS. In the majority
of patients, LEMS results from antibodies vv Correct Answer is: A
against voltage-­gated calcium channels The pathophysiology of autoimmune
(VGCCs) on the presynaptic motor nerve myasthenia gravis relates to the presence
terminal. The VGCC antibodies are often of circulating anti-acetylcholine receptor
presumed to result from cross-reactivity antibodies, which bind to the acetylcho-
with antigens of an underlying tumor. An line receptor causing immune-mediated
associated cancer is found in approximately destruction of the junctional folds that
40% of patients, most often small cell lung contain dense concentrations of this
cancer (SCLC). LEMS may also be seen in receptor, and a higher rate of internaliza-
the setting of autoimmune disorders, non- tion and destruction of acetylcholine
SCLC, lymphosarcomas, malignant thymo- receptor. In some cases, the antibodies
mas, and cancers of the breast, stomach, might block binding of ­acetylcholine to
colon, prostate, bladder, kidney, and gall- its receptor at the neuromuscular junc-
bladder. LEMS often precedes the diagnosis tion. Antibodies against presynaptic
of cancer. If no cancer is detected within voltage-gated calcium channels occur in
2 years of LEMS onset, an autoimmune Lambert-Eaton myasthenia syndrome.
disorder should be suspected. Prednisone, Botulinum toxin inhibits exocytosis of
plasma exchange, 4-­diaminopyridine, and presynaptic vesicles containing acetyl-
intravenous immunoglobulin are available choline. Rarely, myasthenia results from
treatment options. mutations in the acetylcholine receptor
gene.
zz Suggested Readings
55 Stickler DE. Lambert-­Eaton Myasthenic zz Suggested Readings
Syndrome (LEMS). 7 http://emedicine. 55 Comprehensive Review in Clinical
medscape.com/article/1170810-overview. Neurology: a multiple-choice question
Updated 5/3/13. Accessed 5/16/14. Book for the Wards and Boards ©2011
Wolters Kluwer Health Lippincott
?? 51. Regarding the pathophysiology of Williams & Wilkins. All rights reserved.
myasthenia gravis, which of the fol-
lowing is correct? ?? 52. You were evaluating an infant has
A. In its most common form, it is severe diffuse weakness and hypo-
autoimmune due to antibodies tonia and ordered a muscle biopsy
against presynaptic voltage- with an H&E stained, which is shown
gated calcium channels below.
540 Chapter 18 · Neuromuscular

complex repetitive discharges, and occa-


sionally, myotonic discharges.

zz Suggested Readings
55 Image from: Jungbluth H, et al.
Centronuclear (myotubular) myopathy.
Orphanet J Rare Dis. 2008;25;3:26.
55 Amato AA, Brooke MH. Disorders of
skeletal muscle. In: Bradley WG, Daroff
RB, Fenichel GM, Jankovic J, editors.
Neurology in Clinical Practice. 4th ed.
Elsevier, Inc.; 2004. p. 2463–510.

?? 53. Other physical symptoms of Duch-


enne muscular weakness include:
..      Fig. 18.6 Muscle biopsy of a 70-year old female
showing histological picture of centrunuclear A. Awkward manner of walking,
myopathy, am adult-onset congenital myopathy. stepping, or running
(Own work. Jensflorian. Wikipedia) B. Increased lumbar lordosis
C. Higher risk of neurobehavioral
What is the most likely diagnosis? disorders such ADHD
A. Central core disease D. Skeletal deformities such as scoliosis
B. Nemaline myopathy E. All of the above
C. Centro-nuclear myopathy
D. Myofibrillar myopathy vv Correct Answer is: E
E. Congenital fiber type disproportion Patients tend to walk on their forefeet,
because of an increased calf tonus. Also,
vv Correct Answer is: C toe walking is a compensatory adaptation
Centro-nuclear or myotubular myopathy is to knee extensor weakness. Increased
characterized by muscle fibers with central Lumbar lordosis leads to shortening of the
nuclei, as evident in the above image. The hip-flexor muscles.
nuclei are surrounded by a halo devoid of There is a higher risk of neurobehav-
myofilaments, containing mitochondrial ioral disorders (e.g., ADHD) and learning
and glycogen aggregates. Up to 90% of disorders (dyslexia).
fibers may be affected. Biopsy may also
show fiber size variation, type 1 predomi- ?? 54. You are seeing a 19-month-old girl
nance, and type 1 atrophy. The severe, often who has had weakness since birth,
fatal, infant phenotype consists of extra- and has been unable to learn to walk.
ocular, facial, and limb weakness, hypo- She is developing contractures at the
tonia, and respiratory failure. It is caused knees, but with hypermobility at the
by an X-linked recessive mutation of the wrists and ankles.
MTM1 gene, which codes for myotubularin. What is the most likely diagnosis?
18 The infantile form is most common. Less A. Ullrich’s congenital muscular
severe forms may be evident in adolescents dystrophy
and adults, caused by other mutations B. Laminin a2 deficiency
(e.g. MTMR2, MTMR3), both autosomal C. Emery-Dreifuss dystrophy
dominant and recessive. EMG demonstrates D. Dysferlin deficiency
fibrillation potentials, positive sharp waves, E. Sarcoglycan deficiency
Neuromuscular
541 18
vv Correct Answer is: A zz Suggested Readings
Hypermobility is evident in disorders of 55 Amato AA, Brooke MH. Disorders of
collagen, as seen in the muscular dystro- skeletal muscle. In: Bradley WG, Daroff RB,
phies due to collagen type VI mutations Fenichel GM, Jankovic J, editors.
(i.e. Ullrich’s congenital muscular dystro- Neurology in clinical practice. 4th ed.
phy). It is not a feature of the other disor- Elsevier, Inc.; 2004. p. 2463–510.
ders listed. Ullrich’s congenital muscular
dystrophy is due to a mutation of collagen ?? 55. You were asked to see a 6-year-old
type VI. It presents with neonatal weakness boy because of weakness and a pos-
and multiple contractures. The character- sible muscle problem. You ordered a
istic feature is distal hypermobility with muscle biopsy, is shown below.
protrusion of the calcanei. Which of the following is not true
Laminin a2 (merosin) deficiency regarding this condition?
results from a mutation of chromosome A. It is autosomal dominant, and the
6, of autosomal recessive inheritance. gene involved is RYR1
Laminin is a protein in the basement B. These patients are at risk for
membrane, which attaches to dystrogly- malignant hyperthermia
can. Absence of the protein results in C. There is prominent proximal
hypotonia at birth, severe weakness of weakness, especially in the pelvic
the trunk and limbs, and contractures of girdle
the feet and hips. Seizures occur in up to D. Pathologically, there are small
20% of patients. White matter abnor- cores that lack mitochondria,
malities may be evident on imaging. A occurring only in some segments
partial deficiency causes a milder of the muscle fiber
phenotype. E. Facial, bulbar, and ocular muscles
Emery-Dreifuss muscular dystrophy is are usually spared
caused by a deficiency of emerin, a
structural protein of the inner nuclear vv Correct Answer is: D
membrane coded by the STA gene. It is of The muscle biopsy shown is for a cen-
X-linked inheritance. Slowly progressive tral core myopathy. This nicotinamide
weakness affects the anterior legs, upper adenine dinucleotide-stained specimen
arms, and shoulders. Early contractures demonstrates loss of oxidative activity
are evident. The disorder is associated within the center of muscle fibers, seen
with cardiomyopathies. as paler areas where there is absence of
Dysferlin deficiency (limb girdle mitochondria. These central cores run
muscular dystrophy type 28) is caused by along the length of the muscle fiber, as
a mutation on chromosome 2, of autoso- opposed to multicore or mini-core disease
mal l recessive inheritance. Although the in which the cores are small, extending
function of the protein is unknown, its through only segments of the fiber length.
deficiency leads to limb-girdle weakness. Central core myopathy is an autosomal
Onset is typically in the teenage or early dominant disease, caused by a mutation
adult years. in the ryanodine receptor gene RYR1 on
Sarcoglycan deficiency causes limb chromosome 19q13.1. These patients are
girdle muscular dystrophies of types 2C, at risk for the development of malignant
20, 2E, and 2F. The disorders are of hyperthermia and this should be consid-
autosomal recessive inheritance, and lead ered when general anesthesia is needed.
to trunk and limb weakness with calf The clinical presentation is that of weak-
hypertrophy. ness and hypotonia soon after birth, and
542 Chapter 18 · Neuromuscular

..      Fig. 18.7 Heinz


Jungbluth W Histopa-
thologie der Central-
Core-Myopathie.
(Orphanet J Rare Dis.
2007;2:25 7 https://
doi.org/10.1186/1750-
1172-2-25)

subsequent delay in motor development. with weakness of dorsiflexion of the


The weakness is proximal, and the pelvic feet. After percussion of his thenar
girdle is usually more affected than the eminence, there is prolonged contrac-
shoulder girdle. Facial, bulbar, and ocular tion and slow relaxation. In addition,
muscles are usually spared. CK levels are you observed a significant decline in
slightly elevated. compound motor action potential
(CMAP) amplitude in DM1 with both
?? 56. Both myotonic dystrophy type I and short exercise test (SET) and short
myotonic muscular dystrophy type II exercise test with cooling (SETC)
are inherited as: immediately after effort.
A. Autosomal recessives What is the most likely diagnosis?
B. X-linked recessives A. Becker muscular dystrophy
C. Autosomal dominants B. Dystrophic myotonia type 1 (DM1)
D. Mutations in the DMPK genes myotonic dystrophy
E. None of the above C. Emery-Dreifuss muscular dystrophy
D. Fascioscapulohumeral muscular
vv Correct Answer is: C dystrophy,
Both types of myotonic dystrophy are E. DM2 myotonic dystrophy
inherited in an autosomal dominant pat-
tern, which means one copy of the altered vv Correct Answer is: B
gene in each cell is sufficient to cause the This is a case of dystrophic myotonia type
disorder. In most cases, an affected person 1 (DM1) myotonic dystrophy, which is an
has one parent with the condition. autosomal dominant condition with high
penetrance, caused by a CTG expansion
?? 57. You are evaluating an 18-year-old in the myotonic dystrophy protein kinase
male because of weakness. The gene on chromosome 19q. These patients
present in early adult life with ptosis
18 patient has frontal balding, atrophy of
the temporalis and masseter muscles, and facial weakness, and characteristic
and weakness of the sternocleido- features such as frontal balding, atrophy
mastoids and bilateral ptosis. He has of the masseters and temporalis, and
also upper extremity weakness espe- weakness and atrophy of small muscles of
cially in forearm extensors, as well as the hands and extensors of the forearms
atrophy in the anterior tibial muscles, and peroneal muscles. Pharyngeal and
Neuromuscular
543 18
laryngeal weakness may be present, but D. Pseudo-hypertrophy of the del-
this finding is rare. Patients have myoto- toid and calf muscles
nia, which is a phenomenon of prolonged E. Speech impairment
contraction and slow relaxation. Patients
may also have diaphragmatic weakness, vv Correct Answer is: D
leading to respiratory failure, and cardiac Early signs of Duchenne muscular dys-
abnormalities, especially conduction trophy may include pseudo-hypertrophy
defects. Various other tissues are also (enlargement of calf muscles), low endur-
involved, and patients may exhibit esoph- ance, and difficulties in standing unaided
ageal dilatation, megacolon, lenticular or inability to ascend staircases.
opacities and cataracts, intellectual dis-
ability, testicular atrophy with infertility, ?? 59. Which of the following are associated
and androgen deficiency. CK levels may be with myotonic dystrophy type 1?
slightly elevated. EMG demonstrates myo- A. Christmas tree cataracts
tonic discharges and short, rapidly recruit- B. Cardiac conduction defects
ing motor unit potentials with fibrillations. C. Distal weakness
Histo-pathologically, muscle biopsy D. Endocrine abnormalities
demonstrates marked central nucleation, E. All of the above
type 1 fiber atrophy, peripherally placed
sarcoplasmic masses, ring fibers, and vv Correct Answer is: E
pyknotic nuclear clumps. In addition, a Myotonic dystrophy type 1 is a muscular
significant decline in CMAP amplitude in dystrophy caused by a mutation in the
DM1 with both SET and SETC immediately gene coding for myotonic dystrophy
after effort is consistent with DM1. In protein kinase (DMPK) on chromosome
DM2, there is no marked change in CMAP 19. The disorder results from CTG repeats,
amplitude with either SET or SETC. with anticipation and the size of expan-
sion correlating with severity. The clinical
zz Suggested Readings manifestations include weakness with
55 Comprehensive Review in Clinical muscle wasting and myotonia (involun-
Neurology: a multiple-choice question tary sustained contractions elicited by
Book for the Wards and Boards ©2011 percussion or strong voluntary contrac-
Wolters Kluwer Health Lippincott tion, particularly when cold). The weak-
Williams & Wilkins. All rights reserved. ness is most pronounced distally, and the
55 Value of short exercise and short exercise disease often presents with hand weak-
with cooling tests in the diagnosis of ness and foot drop. The face appears long
myotonic dystrophies {DM1 and DM2). and mournful with ptosis and an open,
55 Gawel M, Szmidt-Salkowska E, slack mouth, due to weakness. A curious
Lusakowska A, Nojszewska M, Sulek A, feature is the denial of illness. Myotonic
Krysa W, Rajkiewicz M, Seroka A, dystrophy type 1 is associated with a num-
Kaminska AM. Muscle Nerve. ber of systemic manifestations, including:
2014;49(2);277–83. Cardiac abnormalities, e.g. conduction
defects, tachy-arrhythmias, fibrosis of the
?? 58. Early signs of Duchenne muscular conduction system. It should be noted
dystrophy may include: that the cardiac effects are not cardiomy-
A. Truncal hypotonia opathies, in contrast to many other muscle
B. Difficulty in sitting diseases. Disturbed sleep with excessive
C. Atrophy of the deltoid and calf daytime sleepiness, due to depression of
muscles the respiratory drive. Christmas tree cata-
544 Chapter 18 · Neuromuscular

racts. Testicular atrophy, impaired glucose extraocular eye movements are


tolerance, and other endocrine abnormali- intact without nystagmus, mucous
ties. Gallbladder disease. membranes are moist with tongue
fasciculation present. Lungs are clear
zz Suggested Readings to auscultation bilaterally, heart has
55 Amato AA, Brooke MH. Disorders of a regular rate and rhythm without
skeletal muscle. In: Bradley WG, Daroff RB, murmurs, and abdomen is soft,
Fenichel GM, Jankovic J, editors. non-tender and non-distended with
Neurology in clinical practice. 4th ed. no hepatosplenomegaly. Patellar
Elsevier, Inc.; 2004. p. 2463–510. reflexes are 0/4, Achilles reflexes are
0/4, and brachioradialis reflexes are
?? 60. Spinal muscular atrophy (SMA) is a 0/4. He is floppy, unable to support
hereditary disease affecting the lower his head when pulled to a seated
motor neurons. position, and does not kick or bat
Which of the following statements at objects although he does move
about SMA are correct? his fingers and toes. In horizontal
A. It is an autosomal dominant disorder suspension he lays limp on the phy-
B. SMA in children is classified into sician’s hand and in vertical suspen-
three types, based on ages of onset, sion he starts to slip through the
severity, and progression of symp- physician’s hands.
toms. All three types are caused by Which of the following is true
defects in the SMN1 gene regarding his likely diagnosis?
C. SMA type I, also called Werdnig- A. This is an autosomal dominant
Hoffmann disease, is evident by trait
the time a child is 6 months old. B. His lower extremities will be
D. (b) and (c) weaker than his upper extremities
E. None of the above. C. His mother has myasthenia gravis
D. There is a vaccine that can prevent
vv Correct Answer is: D infection of this illness
SMA is an autosomal l recessive disorder. E. This child will have impaired
intellect
?? 61. You are seeing a 2-month-old baby
because he is unable to breastfeed vv Correct Answer is: B
due to poor latching and sucking. This is a clinical presentation of spinal
His mother switched him to for- muscular atrophy (SMA) type 1, also
mula but he is still a “lazy eater” and known as Werdnig-Hoffmann disease. The
doesn’t suck much, preferring to let main features in this case that differenti-
it drop into his mouth. He is smiling ate SMA from other conditions that cause
and cooing. His mother is concerned, hypotonia or developmental delay are the
however, because he does not move areflexia and tongue fasciculation exhib-
as much as the other children in his ited by this baby. SMA is one of the most
18 daycare. On examination, he is in frequent autosomal recessive disorders
the 15 percentiles for height and and does not exhibit an ethnic preference.
10 percentiles for weight and 10 Although the phenotype is variable, 95%
percentiles for head circumference. of children with SMA can be diagnosed
There are no dysmorphic features, with gene-mutation testing of the SMN1
his pupils are equal, round, and reac- gene on chromosome 5q. There are 3
tive to light and accommodation, types of SMA.
Neuromuscular
545 18
SMA presents with symmetric muscle 55 Chung B, et al. Spinal muscular atrophy:
weakness however the proximal muscle survival pattern and functional status.
groups are weaker than the distal muscle Pediatrics. 2004;114(5):e548–53.
groups and lower extremities are weaker 55 Kliegman RM, et al. Nelson essentials of
than the upper extremities. Thus, over pediatrics.5th ed. Philadelphia: Elsevier
time, the legs will stop moving but the Saunders; 2006.
child can continue to play with toys that
are placed in their hands. ?? 62. You are evaluating an 18-year-old
Neonates born to mothers with male with weakness of his hands
myasthenia gravis can develop a transitory bilaterally, which he first noticed
myasthenic syndrome as a result of several months prior to the office
anti-acetylcholine receptor antibody. visit. On examination, he has atro-
Symptoms include ptosis, ophthalmoplegia, phy of the thenar muscles, as well as
weak facial movements, poor sucking and impaired vibratory sensation in his
feeding, hypotonia, and variable extremity feet.
weakness. Symptoms will persist from Which of the following subtype of
1–10 weeks. Diagnosis is confirmed by the ­Charcot-Marie-­Tooth (CMT) disease
demonstration of clinical improvement is the most likely diagnosis?
lasting approximately 45 min after intra- A. CMT1 B
muscular administration of neostigmine. B. CMT2D
The infant described in this case does not C. CMT1X
show evidence of ptosis, ophthalmoplegia, D. CMT2A
or weak facial movements, making neonatal E. CMT4C
transitory myasthenia gravis unlikely.
Poliomyelitis is an acute enterovirus vv Correct Answer is: B
illness that presents with a prodromal of In many forms of CMT, the patient pres-
vomiting, diarrhea and aseptic meningitis ents with symmetrical distal limb weak-
followed by evolution of asymmetric ness and sensory loss, pes cavus, and a
flaccid weakness. This weakness is the family history of neuropathy. CMT2D is
result of infection of groups of anterior unique amongst the CMT disorders, in that
horn cells. This is similar to SMA, which the hands are affected early in the course.
also presents with flaccid weakness due to The disorder typically presents in adoles-
anterior horn cell degeneration. The differ- cence or early-adulthood with bilateral
ence between SMA and poliomyelitis is weakness and atrophy of thenar and first
the prodromal illness and acute onset in dorsal interosseus muscles, initially spar-
poliomyelitis in contrast to progressive ing the hypothenar eminence. Patients
symmetric weakness. may also report transient cramping and
Infants with SMA have normal mental, pain in the hands when cold and cramping
social, and language skills and sensation. in calf muscles with exercise. Sensory defi-
For this reason, it is very important that cits may follow, with impaired pinprick,
social, language, and intellectual skills be temperature, touch, and vibration sense
maximized while providing symptomatic in the feet, and less often, the hands, in a
therapy. stocking-glove distribution. CMT2D is due
to a mutation of the GARS gene, which
zz Suggested Readings encodes glycyl-tRNA synthetase. A muta-
55 Schroth M. Special considerations in the tion of the same gene also causes distal
respiratory management of spinal muscu- hereditary motor neuropathy (dHMN)
lar atrophy. Pediatrics. 2009;123(4):245–9. type V. When sensory deficits are present,
546 Chapter 18 · Neuromuscular

the disorder is classified as CMT2D. When peroneal muscular atrophy, are a large,
sensory deficits are absent, the disorder is heterogeneous group of inherited periph-
classified as dHMN type V. eral neuropathies. The CMTs can be divided
into demyelinating, axonal, and combined
zz Suggested Readings demyelinating and axonal forms. They are
55 Goldfarb LG, Sivakumar K. GARS- genetically heterogeneous. The demyelin-
associated axonal neuropathy. 7 http:// ating CMTs include CMT1 and CMTX. CMT4,
www.ncbi.nlm.nih.gov/books/NBK1242/. involves both demyelination and axon loss.
Updated 8/25/11. Accessed 5/16/14. In the demyelinating CMTs, NCS generally
55 Patzke A, Shy ME. Charcot-Marie-Tooth show diffuse, uniformly slow conduction
disease and related genetic neuropathies. velocities without conduction blocks or
Continuum. 2012;18(1):39–59. temporal dispersion, indicating a heredi-
tary as opposed to acquired demyelinating
?? 63. You are evaluating a 14-year-old process. There may be evidence of axon
female with bilateral foot drop. loss in the demyelinating forms as well,
She has been clumsy and seemed particularly in patients with long­standing
to trip on her own feet, and could disease; this is secondary axon loss, and
not participate in ballet classes as a conduction velocities as well as CMAP
young child. In the recent years, her and SNAP amplitudes will be reduced. In
feet became noticeably weak. On contrast, with the axonal forms, motor
examination, she had bilateral sym- and sensory potential amplitudes will be
metric weakness of foot dorsiflexion reduced, but conduction velocities will
and plantarflexion. She had ham- be normal or minimally reduced. At the
mertoes and high-arched feet. Ankle time of this publication, there were seven
and knee-deep tendon reflexes were subtypes of CMT1; they are all inherited in
absent. Examination of the mother an autosomal dominant fashion. CMT1A is
also revealed hammertoes and high-­ the most common inherited demyelinating
arched feet. Conduction velocities of neuropathy. Clinical manifestations typi-
approximately 30 cm/second were cally begin in the first two decades of life
revealed in all NCS of the legs, with no and include slowly progressive weakness,
identifiable conduction block. muscle atrophy, kyphosis, and mild (often
What is the most likely diagnosis in asymptomatic) sensory loss. Other signs
this patient? include hammertoes, high-arched feet,
A. Charcot-Marie-Tooth (CMT) 1 palpably enlarged nerves due to peripheral
B. CMT2 nerve hypertrophy (which more commonly
C. A muscular dystrophy occurs in the CMT1 group compared to the
D. CMT3 other CMTs), and pes cavus. Involvement
E. CMT4 of the upper extremities typically occurs
later in life. There is often a family history
vv Correct Answer is: A of neuropathy, though due to variable
This clinical presentation is consistent expression, some affected family members
18 with Charcot-Marie-Tooth (CMT) 1. The may only have mild features such as ham-
clinical manifestations are discussed fur- mertoes and may remain undiagnosed
ther below, but respiratory compromise for a large part of their life; sporadic cases
is not usually seen in the most common without a clear family history also exist.
forms of CMT, but occurs in some cases Other forms of CMT may be congenital and
of CMT2C. The CMTs, also known as severe. In the CMT1 group, many of the
hereditary sensorimotor neuropathies or genes involved are related to myelin syn-
Neuromuscular
547 18
thesis. CMT1A is due to a duplication in the The genes implicated in the CMT2 group
peripheral myelin protein 22 (PMP22) gene are involved in axonal transport and
on chromosome 17, whereas CMT1 B is due membrane trafficking. CMT2A2 is one of
to a mutation in the myelin protein O gene. the most common of this group and is due
CMT1 B is more severe in terms of clinical to mutations in mitofusin 2. CMT3, also
manifestations as compared to CMT1A. In known as Dejerine-Sottas syndrome or
patients with CMT1, CSF shows elevated hypertrophic neuropathy of infancy, is one
protein levels in some cases. Roussy-Levy of the more severe forms of the demyelin-
syndrome is phenotypically similar to ating CMTs. It presents in infancy with
CMT1A but is associated with the presence proximal weakness, absent deep tendon
of a static tremor and gait ataxia. It has reflexes, and hypertrophy of the periph-
been associated with mutations in both eral nerves. Prominent sensory symptoms
the PMP22 and myelin protein 0 genes. On including pain and dysesthesias occur.
nerve biopsy, pathologic features of the Patients typically have extensive disability
demyelinating inherited polyneuropathies early in life. Both autosomal recessive and
include demyelination and an onion-bulb dominant forms exist. CSF protein is
appearance due to Schwann cell prolifera- usually elevated. Congenital hypo-­
tion. Onion bulbs are not specific for CMT, myelination is seen along the spectrum of
and also occur with chronic inflammatory this disorder. CMT3 is genetically hetero-
demyelinating polyneuropathy. CMTX is geneous; mutations in several genes
the second most common type of CMT. It including PMP22, protein myelin 0, and
is demyelinating and is clinically similar other genes implicated in defective
to CMT1, but it is X-linked in inheritance; demyelination are associated with CMT3.
males therefore tend to be more severely The CMTs can be misdiagnosed as
affected compared to females. It is due to muscular dystrophy; NCS findings, as well
a mutation in the connexin 32 gene. CMT2 as the presence of hammertoes and
accounts for approximately one-­third of high-arched foot, distinguish between
the autosomal dominant inherited neu- the two.
ropathies. The CMT2 group are axonal neu-
ropathies; NCS show normal conduction zz Suggested Readings
velocities, and nerve biopsy shows axon 55 Bradley WG, Daroff RB, Fenichel GM,
loss without evidence of significant demy- et al. Neurology in Clinical Practice, 5th
elination. Compared to CMT1, in CMT2, ed. Philadelphia: Elsevier; 2008.
symptoms and signs typically appear 55 Ropper AH, Samuels MA. Adams and
later and foot and spine deformities are Victor’s Principles of Neurology, 9th ed.
less severe. Clinical manifestations do not New York: McGraw-Hill; 2009.
always help distinguish the different CMTs, 55 Comprehensive Review in Clinical
but some clinical features occur more Neurology: a multiple-choice question
frequently in the different subtypes. Optic Book for the Wards and Boards © 2011
atrophy occurs more in CMT2A2, foot ulcer- Wolters Kluwer Health Lippincott
ations in CMT28, and vocal cord paralysis, Williams & Wilkins. All rights reserved.
intercostal, and diaphragmatic weakness
in CMT2C. In CMT2D, unlike in the other ?? 64. You are seeing a 20-week-old infant
CMTs, the hands are involved more than girl with a one-­week onset of diffuse
the feet. hypotonia, ptosis, dysphagia, weak
Peripheral nerve hypertrophy does not cry and dilated, sluggish pupils. The
occur. They are all autosomal dominant in infant has been constipated and
inheritance, except for a subtype of CMT2A. has a poor suck when feeding. EMG
548 Chapter 18 · Neuromuscular

showed, after 20–50 Hz stimulation, a ?? 65. You are evaluating a floppy baby. You
reversal of a presynaptic block, then diagnosed him with Centro nuclear
a gradual increase in the size of the myopathy based on a muscle biopsy.
motor unit potentials. Which of the following character-
What is the likely diagnosis? istics are not true regarding this
A. Infantile myasthenia condition?
B. Infantile botulism A. Respiratory failure occurs in
C. Infantile spinal atrophy severe forms
D. Infantile myotubular myopathy B. The muscle fibers occur show cen-
tral nucleation
vv Correct Answer is: B C. Pharyngeal and l­ aryngeal muscles
Clostridium botulinum infects the infants may be affected
from contaminated formula, ingestion of D. Extraocular movements are rarely
contaminated honey or inhaled from dust affected
from nearby construction. The exotoxin E. Could be autosomal dominant,
produced causes a cholinergic blockade autosomal recessive, or X-linked
of skeletal muscle and autonomic nerves.
Progressive facial, pharyngeal weakness, vv Correct Answer is: D
ptosis, dilated pupils, absent reflexes and Centro-nuclear myopathy (also known
constipation differentiate infantile botu- as myotubular myopathy) is a congenital
lism from other inflammatory demyelinat- myopathy, manifesting characteristically
ing and neuromuscular junction disorders. with ptosis and ocular palsies, as well as
The peak incidence is between March and weakness of facial, pharyngeal, laryngeal,
October with prodromal symptoms of and neck muscles. It typically presents
constipation and poor feeding. with hypotonia and weakness at birth,
Infantile myasthenia syndromes do or in early childhood. Proximal and distal
not present with dilated pupils, absent weakness and hypo-­reflexia can be seen.
reflexes or constipation. Severe forms may be fatal due to respira-
Infantile spinal muscular atrophy tory failure in the first few months of life.
symptoms do not include bulbar manifes- CK levels are mildly elevated. Needle EMG
tations (facial and pharyngeal weakness, shows a myopathic pattern with positive
ptosis, dilated pupils). waves and fibrillations. Pathologically,
Infantile myotubular myopathy (mild/ there are small muscle fibers and central
moderate form) typically manifests with nucleation, as well as predominance of
low tone at birth, subsequent motor delay type I fibers, which are small and hypo-
and may include ophthalmoplegia and trophic. This condition is inherited in an
cognitive impairment. X-linked or autosomal dominant or reces-
sive fashion. The X-linked form is severe
zz Suggested Readings and presents in the neonatal period, the
55 Brown N, Desai S. Infantile Botulism: a autosomal dominant form is an adult-
case report and review. J Emerg Med. onset milder form, and the autosomal
18 2013;45(6):842–5. recessive form is intermediate in severity.
Neuromuscular
549 18
?? 66. Which of the following statements abnormalities and include subcortical
is not true regarding the disorder band heterotopia, or double cortex, in
depicted in the figure? which there is relatively normal cortex with
an underlying band of white matter, under-
neath which is a band of gray matter. This
disorder results from a mutation in the DCX
gene on chromosome X, which encodes
for the protein doublecortin, which is
involved in microtubule organization and
stabilization. The same mutation can lead
to classic lissencephaly (smooth brain,
agyria, or pachygyria) when occurring in
males. This difference in manifestations in
females as compared with males is thought
to result from lyonization (random X inac-
tivation) in females, such that in neurons
in which the mutated gene is inactivated,
normal migration occurs. Clinical features
include intractable seizures, microcephaly,
hypotonia, spastic quadriparesis, recurrent
aspirations necessitating feeding tube, and
shortened life expectancy. Lissencephaly
..      Fig. 18.8 Subcortical band heterotopia, or double
type I characterized by agyria or pachygy-
cortex. ILAE.org. EpilepsyDiagnosis.org, Diagnostic
Manual ria rather than the presence of two bands
of gray matter separated by a band of
white matter. Cobblestone lissencephaly
A. It is a disorder of neuronal migration
(rather than subcortical band heterotopia),
B. It results from abnormalities in a
or lissencephaly type II, is seen in several
protein involved in microtubule
disorders, including Walker-Warburg syn-
organization and stabilization
drome, Fukuyama muscular dystrophy, and
C. It is an X-linked disorder
muscle-eye-brain disease of Santavuori. In
D. Mutations in the DCX gene lead to
polymicrogyria, there are excess, abnormal
this disorder in females and lissen-
gyri.
cephaly in males
E. This disorder is classically associ-
?? 67. Symptoms of Duchenne muscular
ated with muscular dystrophy
dystrophy usually occur in:
A. Female children before age 6
vv Correct Answer is: E
B. Male children before age 6
The MRI shows subcortical band hetero-
C. Male children between ages 6
topia. The neuronal migration disorder
and 12
associated with muscular dystrophy is cob-
D. Adolescent males
blestone lissencephaly. The lissencephaly
E. Adolescent females
syndromes result from neuronal migration
550 Chapter 18 · Neuromuscular

vv Correct Answer is: B cortical changes are milder and the white
Symptoms usually appear in male children matter changes are more focal. The eyes
before age 6 and may be visible in early are also affected to a lesser degree than
infancy. Progressive proximal muscle Walker-Marburg syndrome.
weakness of the legs and pelvis associated
with a loss of muscle mass is observed ?? 69. What is the most common sign or
first. Eventually this weakness spreads to symptom at presentation of McArdle’s
the arms, neck, and other areas. Disease {Type V Glycogen Storage
Disease)?
?? 68. You are seeing a baby who is diag- A. Hypoglycemia
nosed with a congenital muscular B. Leg cramps
dystrophy. Clinically, he is weak and C. Hepatomegaly
has multiple contractures with distal D. Cardiomegaly
hyperlaxity. There are protrusions of E. Feeding difficulties
the calcanei in the feet.
What is the most likely diagnosis? vv Correct Answer is: B
A. Fukuyama-type congenital mus- McArdle’s disease is caused by
cular dystrophy Myophosphorylase deficiency and occurs
B. Laminin alpha 2 deficiency in 1/100,000 live births. It was the first
C. Walker-Marburg syndrome Muscle- metabolic myopathies to be recognized.
eye-brain disease Though its onset is at birth, it often goes
D. Muscle-eye-brain disease undiagnosed until adulthood well into the
E. Ullrich’s congenital muscular 3rd and 4th decades. Signs and symptoms
dystrophy include exercise-induced muscle cramps,
myalgias, and myoglobinuria. A classic
vv Correct Answer is: E finding is the “second wind phenomenon”,
This patient has Ullrich’s congenital where the muscular symptoms abate
muscular dystrophy, which presents with somewhat around 10 min into activity.
neonatal weakness, contractures and Patients eventually present to the ED or
distal hyperlaxity, as well as protrusion of physician with severe pain, muscle con-
the calcanei. It is associated with muta- tractures, or rhabdomyolysis.
tions of collagen type VI and is thought to There are more than a dozen types of
be related to Bethlem myopathy. Walker- GSDs, but the four that cause the most
Marburg syndrome is an autosomal reces- clinically significant neurologic and
sive condition characterized by muscular muscle problems are: (1) Pompe disease
dystrophy and brain and ocular abnormal- (Type II, acid maltase deficiency) --> car-
ities. Patients are hypotonic at birth with diomegaly, cardiomyopathy, (2) Cori
elevated CK levels. The ocular malforma- disease (Type Ill, debranching enzyme
tions include microphthalmia, colobomas, deficiency) --> Hypoglycemia, hepato-
cataracts, glaucoma, corneal opacity, megaly, (3) McArdle disease (Type V,
retinal dysplasia, and optic atrophy. There myophosphorylase deficiency), (4) Tarui
18 are multiple brain CNS malformations disease (Type VII, phosphofructokinase
including hydrocephalus, aqueductal ste- deficiency).
nosis, cerebellar hypoplasia, and cortical
abnormalities. Muscle-eye-brain disease is ?? 70. You are evaluating a 4-year-old boy
an autosomal recessive condition in which for delayed gross motor develop-
there is also muscular dystrophy and brain ment. He is a very engaging child who
and ocular abnormalities; however, the is a great eater but always appears
Neuromuscular
551 18
to be fatigued and has difficulty with experience more falling than is expected
standing. When he tries to walk he for age. Gait abnormalities often become
falls frequently and has difficulty get- apparent at 3–4 years of age.
ting back up. Past medical history is Muscle fiber necrosis can cause
remarkable for two liver biopsies for elevation in purported liver enzymes.
elevated liver enzymes. Serum alanine transaminase (ALT) and
What is the most appropriate next aspartate transaminase (AST) levels are
step in evaluation? commonly used as biochemical indicators
A. EMG with nerve conduction of hepatocellular injury but can also
B. Plasma lactate detect occult muscle disease. High
C. Ammonia concomitant serum CK levels can point to
D. Creatine kinase muscle as the source of high transaminase
E. MRI of the brain levels.

vv Correct Answer is: D zz Suggested Readings


This is a case of Duchenne muscular dys- 55 McMillan et al. Serum transaminase levels
trophy. Clinical manifestations of muscle in boys with Duchenne and Becker
weakness and calf pseudohypertrophy muscular dystrophy. Pediatrics.
are typically not recognized until 3 years 2011;127(1).
of age or later. Walking often begins later 55 Swaiman. Pediatric Neurology Principals&
than in normal children and affected boys Practice.
553 19

Neuro-Oncology

© Springer International Publishing AG, part of Springer Nature 2018


Y. M. Awaad, Absolute Pediatric Neurology, https://doi.org/10.1007/978-3-319-78801-2_19
554 Chapter 19 · Neuro-Oncology

?? 1. You are evaluating a 6-year-old boy Which of the following tumors is most
who fell down in his school playground likely?
and he has a head trauma. EMS has A. Ependymal
been called because he vomited sev- B. Juvenile pilocytic astrocytoma
eral times. In the ER, the attending phy- C. Medulloblastoma
sician ordered a head CT scan which D. Oligodendroglia
showed large mass filling the fourth E. Sub ependymal giant cell
ventricle, and hydrocephalus. His par- astrocytoma
ents reported that he was clumsy in his
walking and he had mild headaches vv Correct Answer is: C
for 1 week. His past medical history is In the first decade of life, posterior fossa
unremarkable. On exam, he has bilat- region are the most common brain
eral papilledema, mild truncal instabil- tumors in children. Clinical manifesta-
ity while seated, a broad-based gait, tions include: a progressive subacute
and no skin stigmata. His brain MRI ataxia, truncal if located in the vermis
sample is shown. Contrast administra- and appendicular if located primarily in
tion showed patchy enhancement of one cerebellar hemisphere. Signs of
the lesion. Tissue biopsy reveals small increased intracranial pressure, such as
round undifferentiated blue cells with vomiting, headache and occasionally a
hyperchromatic nuclei. No rosettes or sixth nerve palsy are due to the com-
pseudo rosettes are identified. pressive effects these tumors have on
the outflow of cerebrospinal fluid
through the cerebral aqueduct or fourth
ventricle itself. The most common pos-
terior fossa tumors in childhood include
medulloblastoma, juvenile pilocytic
astrocytoma and ependymoma.
Oligodendroglioma is a rare tumor in
children, accounting for 1% of child-
hood brain tumors, and is chiefly supra-
tentorial in location. Sub-ependymal
giant cell astrocytomas are juxtaventric-
ular tumors, usually located at the
region of the foramen of Monro, and
seen exclusively in patients with tuber-
ous sclerosis.
Medulloblastoma among the above
mentioned posterior fossa tumors is
notable for its midline location, fairly
diffuse contrast enhancement on MRI
and histologic presence of small round
blue cells with hyperchromatic nuclei.
..      Fig. 19.1 MEDULLOBLASTOMA. As seen in this MRI Medulloblastomas are currently felt to
be histologically similar to primitive
19 study of a vermian lesion, medulloblastomas are
contrast-enhancing masses. Diffuse enhancement in neuro-ectodermal tumors, differenti-
the cerebral subarachnoid space (arrows) indicates ating along several cell lines, not dis-
leptomeningeal dissemination. (7 http://peir2.path.
similar to tumors of stem cell origin.
uab.edu/scripts/acdis.dll?cmd=see&fp=/dbih/
AFIP/00405851.tif&fmt=jpg&q=100&h=512. The Subarachnoid seeding of the tumor can
Armed Forces Institute of Pathology. Wikipedia) occur, with “drop metastases” present
Neuro-Oncology
555 19
along the spinal axis. With the advent of extremities. Now, she is not able
radiation treatment and more aggres- to ambulate. Two days ago, her
sive neurosurgical resection, the sur- mother noted that her eyes were
vival rate has increased markedly over moving abnormally. No history any
the past 30 years. The overall best sur- recent viral infections or immuni-
vival rate is for children older than age zations. On examination, she has
3, with less than 1.5 cm3 residual tumor significant involuntary, chaotic,
after resection, and no evidence of sub- multi-directional saccadic eye move-
arachnoid seeding, with 5-year survival ments, action myoclonus, and both
rates greater than 80%. Adjuvant che- appendicular and truncal ataxia.
motherapeutic regimens are added for Her head MRI is normal, but CSF
higher risk groups and those with recur- analysis shows a mild pleocytosis
rent disease. and a mildly elevated protein. Urine
Pilocytic astrocytoma can occur catechol amines, shows a homova-
anywhere in the cerebellum. About 80% nillic acid of 61.5 mg/g (­ normal
are cystic with a large mural nodule that 0–32.6) and vanillyl mandelic acid of
contrast enhances on MRI. Pathology 20 mg/g (normal 0–16.7).  
reveals compact, strongly fibrillated What is your next step in management?
cells alternating with loose, spongy A. CT scan of the chest and abdomen
areas composed of micro cysts. B. Anti-Hu antibody titers
Leptomeningeal spread is quite rare. C. Ceruloplasmin level
Treatment for well demarcated astro- D. CSF measles antibody titers
cytomas is total surgical resection, E. ENT consultation
with survival rates approaching 90% at
10 years. vv Correct Answer is: A
Ependymomas arise in relation to This is a clinical presentation of an
any part of the ventricular system, with opsoclonus-­myoclonus-ataxia (OMA)
infratentorial location being most com- syndrome, also referred to as “dancing
mon in children younger than three. eye and dancing feet syndrome.” The
Older patients more commonly have a term opsoclonus refers to involuntary,
supra-tentorial location. MRI appear- chaotic, multidirectional saccades,
ance can mimic that seen with medul- which are present during fixation and
loblastoma, and metastatic spread smooth pursuit and persist during eye-
through the spinal neuro-axis can occur lid closure and sleep. Opsoclonus often
as well. Histologically, they appear quite occurs in association of appendicular
different, with ependymal rosettes or myoclonus and cerebellar ataxia.
pseudo-rosettes lining a small central Behavioral disturbances, postural
lumen. Treatment regimens utilize a tremor, and/or encephalopathy can be
combination of surgical resection, radia- exhibited. Opsoclonus has different eti-
tion and chemotherapy. ologies: paraneoplastic, para-infectious,
toxic-metabolic, or idiopathic phenome-
zz Suggested Reading non. In children, more than half of para-
55 Swaiman et al. Pediatric neurology, neoplastic opsoclonus is secondary to
principles and practice. 4th ed. p. 1707–16. neuroblastoma. Although a variety of
anti-neuronal antibodies have been
?? 2. You were asked to see an 18-month- identified in this disorder, the majority
old girl with one-week history of of patients are seronegative. All children
“twitching” that started in her legs presenting with OMA should be evalu-
but eventually affected all four ated for occult neuroblastoma with
556 Chapter 19 · Neuro-Oncology

urine catechol amines, including What is your most likely diagnosis?


homovanillic acid and vanillyl mandelic A. Hemangioblastoma
acid, imaging of the chest and abdo- B. Medullary glioma
men, and, if negative, C. Teratoma
123I-­metaiodobenzylguanidine (MIBG) D. Medulloblastoma
scan. When negative, the evaluation E. Juvenile pilocytic astrocytoma
should be repeated after several
months. The prognosis for survival of vv Correct Answer is: D
neuroblastoma patients with opsoclo- Different posterior fossa tumors can be
nus is better than for those without ops- presented with the same symptoms, but
oclonus. Management involves both neuroimaging suggest a medullo-
­treatment of the underlying process, as blastoma. Astrocytomas are more com-
well as symptomatic treatment with cor- mon than medulloblastomas but are
ticosteroids, intravenous immunoglobu- predominantly cystic. Medullary gliomas,
lin (IVIG), or adrenocorticotrophic teratomas, and hemangioblastomas have
hormone (ACTH). Low-dose cyclophos- different radiological characteristics and
phamide may be beneficial in refractory are less common. Several different radio-
cases. With or without treatment, chil- logical techniques are used in the medul-
dren with opsoclonus and neuroblas- loblastoms diagnosis, but the defined
toma usually experience resolution of diagnosis is based on WHO-­defined histo-
the opsoclonus, though most experi- logical criteria. Classic medulloblastoma
ence relapses. is characterized by sheet-like areas of
small, round, blue cells with scant cyto-
zz Suggested Reading plasm and dense hyperchromatic nuclei.
55 Wong A. An update on opsoclonus. Curr Homer-Wright rosettes, which are circular
Opin Neurol. 2007;20:25–31. groupings of tumor cells, are present in
less than half of cases. Other subtypes of
?? 3. You are seeing a 9-month-old boy in medulloblastoma include desmoplastic,
the hospital, who has been vomit- large cell anaplastic, medullomyoblastic,
ing (non-bloody, non-­bilious) for and melanotic variants. Staging requires
4 weeks and progressive off balance a gadolinium-­enhanced MRI of the brain
for 2 weeks. He achieved his mile- and entire spine and a lumbar puncture
stones on target like sitting indepen- to assess for disseminated disease, which
dently at 7 months but he cannot do is present in 30% of children at presenta-
so for the last week. You examined tion; however, drop metastases are read-
him, he is alert but irritable, with ily apparent on imaging may obviate the
impaired up gaze and significant need for a lumbar puncture. CSF should
truncal ataxia. You ordered head CT, be obtained 2 weeks post-­operatively to
shows a high density, enhancing avoid false positive results.
posterior fossa mass with significant
hydrocephalus. Because the find- zz Suggested Reading
ing on the CT, you ordered an MRI, 55 Crawford JR, MacDonald TJ, Packer
shows a round, slightly lobulated RJ. Medulloblastoma in childhood: new
mass within the cerebellar vermis biological advances. Lancet Neurol.
19 and fourth ventricle, which is isoin- 2007;6:1073–85.
tense to gray matter on T1- and
T2-weighted images and is homog- ?? 4. An 18-year-old girl came for evalu-
enously enhancing.   ation, she is complaining of blurry
Neuro-Oncology
557 19
vision, headache and difficulty ambu- 55 Maher ER. Von Hippel-Lindau disease.
lating for 4 months. On examina- Curr Mol Med. 2004;4(8):833–42.
tion, she has bilateral papilledema, 55 Lonser RR, Glenn GM, Walther M, Chew
multiple retinal angiomas, dysmetria, EY, Libutti SK, Linehan WM, Oldfield
ataxia, brisk deep tendon reflexes, EH. Von Hippel-Lindau disease. Lancet.
extensor plantar responses, and ankle 2003; 361(9374):2059–67.
clonus. You ordered a brain MRI,
shows a large enhancing m ­ idline cer- ?? 5. You are seeing a 19-month-old girl
ebellar lesion compressing the fourth in the hospital who has normal walk-
ventricle and brainstem. A follow –up ing and understandable two-word
cervical MRI shows multiple nodular phrases for the last few months, sud-
enhancing lesions. denly she became unsteady walking
What is the most likely diagnosis? and wobble irregularly when sitting.
A. Medulloblastoma One day ago, she starts to have
B. Cerebellar astrocytoma irregular, jerky, conjugate movements
C. Arteriovenous malformation of her eyes and is unable to bring a
D. Von Hippel Lindau disease spoon smoothly to her mouth while
E. Ependymoma eating.  
What is your expectations?
vv Correct Answer is: D A. Good prognosis medulloblastoma
In von Hippel Lindau disease patients, B. Bad prognosis medulloblastoma
diagnosis can be made by the presence C. Good prognosis neuroblastoma
of a single retinal angioma or cerebellar D. Bad prognosis neuroblastoma
hemangioblastoma, renal cell carcinoma, E. Bad prognosis glioblastoma
or pheochromocytoma. If there is no fam-
ily history, patients have to have 2 or vv Correct Answer is: A
more retinal angiomas, hemangioblasto- Opsoclonus-myoclonus-ataxia (OMA)
mas within the central nervous system, or syndrome is a paraneoplastic syn-
one such lesion and a characteristic vis- drome most often associated with
ceral tumor. Hemangioblastomas can ­neuroblastoma. The neuroblastomas
affect not only the retina and cerebellum, with which it occurs tend to be the
but the brainstem and spinal cord as well, more differentiated, less malignant
thus resulting in heterogeneous neuro- tumors, and often, the neuroblastoma
logical findings. Von Hippel Lindau dis- can be cured, but the progressive
ease patient’s prognosis is based on the course of OMA syndrome and its cogni-
brain and spinal cord localization, the tive sequelae are the cause of long-
underlying biology of the central nervous term disability. Both anti-T-­cell and
system tumors, and comorbid entities anti-B-cell therapies are useful in this
such as carcinomas. Management of von disorder.
Hippel Lindau patients is best by a multi-
disciplinary approach because of the zz Suggested Reading
potential for multi-organ involvement. 55 Stefanowicz J, Izycka-­Swieszewska E,
Drozyńska E, Pienczk J, Połczyńska K,
zz Suggested Reading Czauderna P, Sierota D, Bień E,
55 Clark PE, Cookson MS. The von Hippel- Stachowicz-Stencel T, Kosiak W,
Lindau gene: turning discovery into therapy. Balcerska A. Neuroblastoma and
Cancer. 2008 Oct 1;113(7 Suppl):1768–78. opsoclonus-­myoclonus-ataxia ­syndrome–
Kaelin WG. Von Hippel-Lindau disease. clinical and pathological characteristics.
Annu Rev. Pathol. 2007;2:145–73. Folia Neuropathol. 2008;46(3):176–85.
558 Chapter 19 · Neuro-Oncology

55 Aydin GB, Kutluk MT, Buyukpamukcu M, nausea, vomiting and double-vision


Akyuz C, Yalcin B, Varan A. Neurological for the last month. On exam, he has
complications of neuroblastic tumors: weakness in right eye abduction, mild
experience of a single center. Childs Nerv right facial weakness, and a decreased
Syst. 2010;26(3):359–65. gag. He also has dysmetria more in
the right upper extremity than left. His
?? 6. You are evaluating a previously DTRs are increased on the left more
6-years old boy who has been having than the right. His brain MRI is shown.

..      Fig. 19.2 The single


voxel spectrum is
derived from a glioma
within the brain stem.
The T2-weighted image
on the left side shows
the extension of the
tumor. The spectrum
exhibits a large peak of
choline compounds at
about 3.2 ppm. (Own
work. Mschocke. W)

What is the most likely prognosis?


A. Good long-term survival following
surgical resection
B. Full recovery following IV antibi-
otic treatment
C. Good long-term survival following
radiation therapy
D. Good long-term survival following
chemotherapy
E. Poor prognosis despite any therapy

vv Correct Answer is: E


Diffuse intrinsic brainstem gliomas typi-
cally has the triad of cranial neuropa-
thies, ataxia, and long tract signs. The
pontine gliomas have the worst progno-
sis of all brainstem gliomas. Less than
10% of patients are alive and free of
19 ..      Fig. 19.3 Brainstem Glioma in 4-year-old. MRI
progressive disease 18 months follow-
sagittal, without contrast. (Own work. Tdvorak. W)
ing diagnosis, even after treatment with
radiotherapy.
Neuro-Oncology
559 19
zz Suggested Reading
55 Recinos PF, Sciubba DM, Jallo GI.
Brainstem tumors: where are we today?.
Pediatr Neurosurg. 2007;43(3):192–201.
55 Korones DN. Treatment of newly diag-
nosed diffuse brain stem gliomas in
children: in search of the Holy Grail.
Expert Rev. Anticancer Ther.
2007;7(5):663–74.

?? 7. A 10-year-old female is complain-


ing of early morning headaches
with vomiting for 1 month. She has
a history of mild ASD, seizures and
multiple hypo-pigmented spots. A T2
weighted axial MRI image is shown
below. Microscopic pathology showed
numerous pleomorphic giant cells.
What is your diagnosis?
A. Segmental neurofibromatosis
B. Sturge Weber syndrome
C. Linear nevus sebaceous syndrome
..      Fig. 19.4 Subependymal Giant Cell Astrocytoma
D. Tuberous sclerosis complex
(Tuberous Sclerosis) s of the type associated with
E. None of the above en:tuberous sclerosis are typically bulky, contrast-­
enhancing masses in the region of the en:foramen of
vv Correct Answer is: D Monro. Most overlie the head of the en:caudate
The MRI showed a large intraventricular nucleus. Foramen obstruction has produced
en:hydrocephalus. Image and description are from the
mass in a child with epilepsy, mild ASD
AFIP Atlas of Tumor Pathology, according to entry
and hypo-pigmented macules. The #405677 in Pathology Education Instructional
pathology of the mass revealed numer- Resource. The Armed Forces Institute of Pathology
ous pleomorphic giant cells consistent Electronic Fascicles (CD-ROM Version of the Atlas of
with a sub-ependymal giant cell astro- Tumor Pathology) contains U.S. Government work
which may be used without restriction [1]. (The Armed
cytoma or SEGA. This consolation is con-
Forces Institute of Pathology. W)
sistent with a tuberous sclerosis
complex diagnosis. SEGAs can occur in
10–15% of patients with tuberous scle- associated with tuberous sclerosis complex
rosis. It is thought that they develop (TSC): clinical recommendations. Eur J
from the sub ependymal nodules near Paediatr Neurol. 2013;17(4):348–52.
the foramen of Monro. 55 Roth J, Roach ES, Bartels U, Jóźwiak S,
Koenig MK, Weiner HL, Franz DN, Wang
zz Suggested Reading HZ Sub-ependymal giant cell astrocytoma:
55 Jóźwiak S1, Nabbout R, Curatolo diagnosis, screening, and treatment.
P. Participants of the TSC Consensus Recommendations from the International
Meeting for SEGA and Epilepsy Tuberous Sclerosis Complex Consensus
Management. Management of sub-­ Conference 2012. Pediatr Neurol.
ependymal giant cell astrocytoma (SEGA) 2013;49(6):439–44.
560 Chapter 19 · Neuro-Oncology

?? 8. You are seeing a 6-year-old girl in D. Despite treatment the median


the ED, who complains of dizziness survival for this lesion less than
and ataxia for the last 3 weeks. She is 1 year from the time of diagnosis
very scary to walk by herself and she E. None of the above
has to hold on her parents. Her PCP
diagnosed her with acute labrynthitis vv Correct Answer is: D
because she had an upper respira- The sagittal image provided The MRI
tory infection at the time of onset. On shows large and abnormal signal of the
examination she has a left 7th nerve pons as well as compression of the cere-
palsy affecting her eye and face. In bellum and obstructive hydrocephalus.
addition, she has inability to abduct This is consistent with a diagnosis of dif-
her right eye. She has a wide based fuse intrinsic pontine glioma. They are typ-
gait and hyperreflexia with clonus. Her ically high-grade tumors with a very poor
sagittal T1 imaging is shown below. prognosis. The typical age at diagnosis is
between 5 and 9 years of age. Patients
typically present with ataxia, cranial nerve
abnormalities or signs of increased intra-
cranial pressure. These lesions are very
hard to be surgical resected. Their
response to radiation and chemotherapy
is variable and short lived. The median
survival is less than 1 year from diagnosis.

zz Suggested Reading
55 Warren K. Diffuse intrinsic pontine
glioma: poised for progress. Front Oncol.
2012;2:205.

?? 9. A 12-year-old girl is brought by her


family because her teacher reported
to them that the girl is not listen, does
not pay attention, is very distractible
presented, and has a slow decline
in her school performance. She has
..      Fig. 19.5 Brainstem Glioma in 4-year-old. MRI decreased hearing on examination.
sagittal, without contrast. (Own work. Tdvorak. W) Her axial MRI imaging is shown below.  
What is your most likely diagnosis?
A. Inflammatory pseudo tumor
Which of the following statements are B. Sarcoidosis
true? C. Neurofibromatosis type 2
A. Surgery followed by radiation is D. Tuberous sclerosis
the conventional approach to E. None of the above
treatment of this lesion
B. This lesion is highly radiosensitive vv Correct Answer is: C
19 and carries a 55% 5-year survival The MRI shows 2 enhancing vestibular
C. This lesion has a high rate of met- schwannomas and an extra axial mass in
astatic spread the right cerebellar hemisphere, most
Neuro-Oncology
561 19
55 Plotkin SR1, Merker VL, Halpin C,
Jennings D, McKenna MJ, Harris GJ, Barker
FG 2nd. Bevacizumab for progressive
vestibular schwannoma in neurofibromato-
sis type 2: a retrospective review of 31
patients. Otol Neurotol. 2012;33(6):1046–52.

?? 10. A 7-year-old girl has a history of pro-


gressive ataxia for the last 2 months.
She has been having morning vomit-
ing for the past 2 weeks. Her MRI is
shown. Treatment consisted of surgi-
cal resection. The child remains well
8 years post-­operative.  

..      Fig. 19.6 Vestibular schwannoma. (RadsWiki.


RadsWiki. W)

likely a meningioma. This is consistent


with a probable diagnosis of neurofibro-
matosis type 2. Although NF2 is mainly
an adult disorder, it can occur in the
pediatric age group. The average age of
onset is from 18 to 24 years. In addition
to the vestibular schwannomas, patients
also develop meningioma and ependy-
momas. This is an autosomal dominant
disorder, with 50% of cases a result of a ..      Fig. 19.7 Axial CSF suppressed FLAIR image showing
a circumscribe hyperintense mass in the tectum of the
de novo mutation. Treatment has been
brainstem presumed to be a pilocytic astrocytoma. In the
mainly symptomatic. Bevacuzumab course of my work as a radiologist. (Jto410. W)
(Avast in) has been used to treat pro-
gressive vestibular schwannoma. What is the most likely diagnosis?
Studies have shown improved hearing A. Choroid plexus papilloma
and tumor shrinkage. B. Juvenile pilocytic astrocytoma
C. Glioblastoma multiform
zz Suggested Reading D. Oligodendroglia
55 Evans, Gareth, D, Neurofibromatosis 2, E. None of the above
Gene Reviews: August 18, 2011 (update).
55 D Gareth R Evans Neurofibromatosis type vv Correct Answer is: B
2 (NF2): a clinical and molecular review. There is a multi-cystic enhancing lesion
Orphanet J Rare Dis. 2009;4:16. involving the cerebellar hemisphere on
562 Chapter 19 · Neuro-Oncology

the MRI. This is the radiological presen-


tation of a juvenile pilocytic astrocy-
toma. This is the most common
cerebellar tumor in children. Typical age
of presentation is between 5 and
13 years old. Surgical resection is typi-
cally the only treatment that is neces-
sary. The other choices listed may
require chemotherapy and/or radiation.

zz Suggested Reading
55 Chourmouzi D, Papadopoulou E,
Konstantinidis M, Syrris V, Kouskouras K,
Haritanti A, Karkavelas G, Drevelegas A.
Manifestations of pilocytic astrocytoma: a
pictorial review. Insights Imaging.
2014;5:387–402.

?? 11. You are evaluating a 16-month-old


girl in the ED for her 2-week history of ..      Fig. 19.8 Brain stem glioma. MRI axial, with contrast.
(Own work. Tdvorak. W)
left head tilt and decreased left facial
movement. By history, she has been
very irritable and clingy for the last brainstem tumors, with approximately
month. In addition, her right leg has 200–300 cases diagnosed in the USA
been “stiff”, and she is not able to take each year. The most clinical presenta-
steps or pulling to stand as before. She tions of those tumors are signs of cranial
has normal language and fine motor nerve, long tract, and cerebellar dys-
skills. Her birth and past medical histo- function. The mean age at diagnosis is
ries are unremarkable. On examination, 7–9 years, and there is no sex predilec-
she is alert but irritable. She has limited tion. Diagnosis is generally based on MR
abduction, decreased blinking, and imaging with and without gadolinium.
incomplete closure of her right eye; Classically, pontine gliomas are associ-
left lower facial droop; limited eleva- ated with diffuse infiltration and hyper-
tion of the left palate; leftward tongue trophy of the pons. They are typically
deviation; and right lower extremity >2 cm at the time of presentation. On
spasticity. Knee jerks are asymmetric, T1-weighted sequences, they appear
with a crossed adductor response on hypo-intense with indistinct margins;
the right only. She is unable to pull to on T2-weight images, they appear
stand. Her brain MRI is shown.   hyper-intense. Administration of gado-
What is your most likely diagnosis? linium results in little to no contrast
A. Ependymal enhancement. In some cases, newer
B. Hemangioblastoma imaging techniques, such as FDG-PET
C. Medulloblastoma and diffusion tensor imaging, may aid in
D. Pontine glioma diagnosis and/or treatment planning.
19 E. Schwannoma The role of stereotactic biopsy remains
controversial, though it may be of bene-
vv Correct Answer is: D fit in cases with atypical imaging charac-
Diffuse intrinsic pontine gliomas teristics. Standard treatment remains
account for 60–75% of all pediatric conventional radiotherapy to a total
Neuro-Oncology
563 19
dose of 54–59.4 Gy, as studies con- nosis of NF-1. In fact, optic pathway
ducted to assess the efficacy of hyper- glioma is one of the diagnostic criteria
fractionated radiotherapy and various for the disorder. As part of the initial
chemotherapeutic regimens have failed evaluation of a child with a unilateral or
to demonstrate improved outcomes. bilateral optic pathway glioma, a skin
The prognosis for pontine gliomas is examination looking for other cutane-
dismal, with a median onset of disease ous manifestations of neurofibromatosis
progression following irradiation of (café-au-lait macules, axillary freckling
<6 months, median survival of and neuro-fibromas) should be done. In
~10 months, and prolonged survival the setting of neurofibromatosis, most
(>/=24 months) in <10% of all patients. optic nerve gliomas are identified on
routing surveillance screening, many will
zz Suggested Reading not even manifest visual symptoms.
55 Frazier JL, Lee J, Thomale UW, Noggle JC, Slowly progressive visual loss is the most
Cohen KJ, Jallo GI. Treatment of diffuse common presenting sign in children
intrinsic brainstem gliomas: failed without NF-1. Additional findings may
approaches and future strategies. A review. include proptosis, nystagmus or an
In J Neurosurg. 2009;3:259–69. acquired strabismus. Examination might
reveal any combination of afferent papil-
?? 12. You are evaluating a 4-year-old lary defect, visual field loss and either
boy with a history of strabismus for optic atrophy or papilledema (especially
3 weeks ago. The strabismus is just on in cases of extension to the hypothala-
the right eye, and occurs mainly when mus which can lead to obstructive
he is tired. There is a family history of hydrocephalus.) In general, optic chiasm
brain tumors in 2 of his mother aunts. lesions and those with extension to the
His brain and orbits MRI, which was hypothalamus have the highest morbid-
ordered by an ophthalmology 1 week ity and mortality. Lesions confined to the
ago, showed enlargement and mild optic nerve tend to be stable over time,
contrast enhancement of the right with even a small percentage undergo-
optic nerve consistent with a glioma. ing spontaneous resolution amongst the
No involvement of the chiasm or patients with neurofibromatosis.
hypothalamus is noted.   Treatment is therefore reserved for
What is your next step in management? patients with progression of disease,
A. Detailed skin examination which can either be clinical (progressive
B. Lumbar puncture visual field loss) or radiologic.
C. MRI of the spinal cord Radiotherapy with chemotherapy is indi-
D. Surgical debulking of the left cated for children over the age of 5. In
optic glioma younger children, or in those with neuro-
E. Radiation therapy fibromatosis, chemotherapy alone may
be utilized. Patients with NF have a much
vv Correct Answer is: A higher incidence of secondary malignan-
Optic pathway gliomas are about 3–5% cies after cranial irradiation due to the
of all childhood tumors. There is an defective tumor suppression inherent
extremely high incidence with neurofi- with the disorder. There is no metastasis
bromatosis, type 1. Up to 5–15% of chil- through the neuroaxis with optic nerve
dren with NF-1 will show evidence of an glioma, therefore imaging of the spine is
optic pathway glioma by the age of 6. not indicated. Lumbar puncture has no
Conversely, of all children with optic gli- place in the evaluation or management
omas, 50% will meet criteria for the diag- of optic pathway gliomas.
564 Chapter 19 · Neuro-Oncology

zz Suggested Reading The most common manifestations of the


55 Willhelm H. Primary optic nerve tumors sellar and supra-sellar region are, head-
In Curr Opin Neurol. 2009;22(1):11–8. ache, visual field defects (classically bi-­
55 Swaiman et al. Pediatric neurology, temporal hemianopia) and endocrine
principles and practice. 4th ed. p. 1702–3. abnormalities. Craniopharygioma, pitu-
itary adenoma, optic pathway glioma,
?? 13. You are evaluating a 14-year-old meningioma, epidermoid/dermoid, ter-
girl for her chronic headaches for atoma and hypothalamic hamartoma
3 years. Her headaches are bi-frontal, are the most common sellar and supra-
squeezing in nature. She has been sellar region. Aneurysms/ arteriovenous
taken ibuprofen and lay down to malformation, granulomatous lesions
relief the headaches. She has no associated with sarcoidosis or tubercu-
aura, phono-phobia but sometimes losis and congenital arachnoid cysts
have photophobia. Sometimes she should be included in the differential
will have nausea. When she has the diagnosis. Imaging, sometimes includ-
headaches, she will be a little more ing magnetic resonance arteriography,
irritable. She has normal social inter- should help distinguish tumor from
action and cognitive performance. other lesions.
Her family history is remarkable for Craniopharyngiomas account for
migraine, her mother and several 5% of all childhood brain tumors. These
other females on the maternal side of very slow-­growing tumors are believed
the family. On examination, she is at to originate from embryonic rest cells
the 3rd percentile for height and the near Rathke’s pouch. Due to the indolent
75th percentile for weight. Neurologic nature, symptoms are often present for
examination, shows bilateral pap- 1–2 years prior to the identification of the
illedema, and her visual fields show tumor. Headache can result from direct
inconsistent finger counting in the effects of the tumor itself or as a result
left temporal field. Basic laboratory of increased intracranial pressure due to
studies, sodium is 158 mEq/L, potas- obstruction of either the third ventricle or
sium 4.2 mEq/L. Urine is dilute, with foramen of Monro. Visual disturbance are
a specific gravity of 1.003. Her head quite common, frank visual field abnor-
CT shows a 5×4 cm partially calcified malities depend on whether the tumor
mass in the sellar region.   affects pre-chiamal, chiasmal or post chi-
What is the most likely diagnosis? asmal pathways. In the past, endocrine
A. Craniopharyngioma abnormalities contributed to a majority
B. Dysembryoplastic neuroepithelial of the morbidity and mortality of the
tumor (DNET) tumor. Thyroid and growth hormone
C. Medulloblastoma deficiencies, as well as diabetes insipidus,
D. Oligodendroglioma tend to be more consistent concomitant
E. Pleomorphic Xanthroastrocytoma disorders, though any aspect of the hypo-
(PXA) thalamic/pituitary axis may be affected.
Post-operatively, morbid obesity due to
vv Correct Answer is: A hyperphagia, is a striking complication.
The typical location for craniopharyngi- Radiologic appearance is varied though
19 oma is the sellar region. classically reveals a sellar/supra-­sellar
Medulloblastoma occurs in the poste- mass that is partially solid, with cystic
rior fossa, and DNET, oligodendroglioma components. Calcification is observed in
and PXA occur in the cerebral cortex. up to 80% of the patients.
Neuro-Oncology
565 19
Treatment is depending on the C. Craniopharyngioma
degree of infiltration of the hypothala- D. Neuroblastoma
mus, optic nerve/chiasm and carotid E. Ovarian teratoma
arteries. If gross total resection is pos-
sible, this provides the least chance of vv Correct Answer is: D
tumor recurrence. On the other hand, This is a classic presentation of
partial resection with adjunctive radia- opsoclonus-­myoclonus syndrome (OMS).
tion therapy (currently using stereotactic Most cases occur in preschool age chil-
or gamma knife technology) is the treat- dren, but it can occur throughout the life
ment of choice if total resection is not span. The usual presentation is either
possible. In younger children, radiation acute or subacute and progressive
therapy may be delayed until recurrence ataxia, raising the suspicious of acute
or progression is noted. Systemic chemo- cerebellar disease due to toxins, post-
therapy has no role in management of infectious inflammation, infection or
craniopharyngioma, though for tumors tumor. The characteristic finding of ops-
of primarily cystic nature, intra-­cystic oclonus (involuntary multidirectional
Bleomycin has been utilized. Overall, the saccades without inter-saccadic inter-
tumor itself tends to be fairly benign, vals) may not be identified for days or
with only rare reports of metastasis, but even weeks into the course of the dis-
the endocrinologic, visual and cognitive ease. Truncal and extremity myoclonus
effects of both the tumor and the late typically presents early on, though can
effects of treatment can cause significant be subtle, and often confused for sei-
quality of life issues. zures. Behavioral changes, sleep dys-
function, and psychomotor retardation
zz Suggested Reading are common in affected children. About
55 Garnett et al. Craniopharyngioma. In 50% of cases to be paraneoplastic in
Orphaned J Rare Dis. 2007;2:18. nature, secondary to a neuroblastoma,
with the remainder of patients felt to
?? 14. You are evaluating a 3-year-old girl have a post-infectious etiology. A well-
for her progressive loss of balance defined antibody has not been identified
control for 1 week. Two days ago, she in pediatric cases. In adults, the most
developed jerking movements in commonly associated cancers are small-
the arms and legs. She started walk- cell lung cancer and gynecologic and
ing independently at 12 months of breast cancers. Rarely, acute infectious
age, but last week she became too encephalitis can produce opsoclonus,
unsteady to walk and even struggled though not typically the entire spectrum
to sit up independently. You ordered of neurologic disease seen in
a brain MRI with contrast and a lum- OMS. Treatment of this syndrome can be
bar puncture, both were normal. On very challenging. In paraneoplastic
examination, she has some multidi- cases, removal of the tumor (and at times
rectional rapid eye oscillation move- adjuvant chemotherapy) is indicated,
ments, spontaneously and at times but unlikely to produce neurologic
gaze-evoked. You strongly suspect a remission. A combined immunomodula-
paraneoplastic syndrome.   tory regimen, including high-dose corti-
What is the most likely neoplasm costeroids (or ACTH), intravenous
to be found in this girl? immune globulin, and rituximab is often
A. Acute myeloid leukemia applied, with varying degrees of success.
B. Angiomyolipoma Though the motor manifestations will
566 Chapter 19 · Neuro-Oncology

improve with time with effective treat- zz Suggested Reading


ment, a chronic static encephalopathy 55 Pandolfo M, Mando M. Cerebellar and
often results. Ovarian teratoma has been Afferent Ataxias. Continuum (Minneap
associated with anti-NMDA receptor Minn) 2013;19(5):1312–134. Arning
encephalitis in children, there has not Neurol 2012;18(2):366–83.
been a well-established para-neoplastic
syndrome associated with leukemia, cra- ?? 16. The gadolinium-­enhanced T-1
niopharyngioma or angiomyolipoma. weighted sagittal brain MRIs for a
patient complaining of headache and
zz Suggested Reading ataxia. It has a posterior fossa mass.
55 Rosenfeld MR, Dalmau JO. Paraneoplastic Surgical resections have been done.  
Disorders of the CNS and Autoimmune
Synaptic Encephalitis. Continuum Lifelong.

?? 15. A 14-year-old boy has been evaluated


for gait ataxia. On neurologic exami-
nation, he has truncal and extremity
ataxia, oculomotor apraxia, intermit-
tent extremity dystonia and chorea,
and multiple scleral telangiectasias.
His mother stated that his eyes have
seemed progressively more “blood-
shot” over the past year. His past medi-
cal history is remarkable for frequent
pulmonary infections. Family history is
remarkable of a 10-year-old brother is
similarly affected.
What serum screening tests is most
likely to be abnormal in this case?
A. alpha-fetoprotein
B. ammonia
C. creatine kinase
D. lactate
..      Fig. 19.9 Anaplastisches Ependymom. MRT in
E. uric acid verschiedenen Wichtungen und Ebenen. (Own work.
Hellerhoff. W)
vv Correct Answer is: A
Patients in this case have ataxia-­ What is your most likely diagnosis?
telangiectasia, an autosomal recessive A. Choroid plexus neoplasm
condition with childhood onset causing B. Ependymoma
ataxia, oculomotor apraxia, choreoathe- C. Meningioma
tosis and dystonia, and telangiectasias on D. Metastatic neoplasm
skin and conjunctivae. Immunodeficiency E. Sub-ependymoma
and susceptibility to cancer (especially
lymphoma and leukemia) are significant vv Correct Answer is: B
19 causes of morbidity and mortality. An Ependymomas account for 3–5% of intra-
elevated alpha-­fetoprotein level (over cranial neoplasms. They arise from the
90%) can help in diagnosis, and genetic ventricular wall, and 60% occur in the
testing of the ATM gene is available for posterior fossa, approximately half of
confirmatory diagnosis. which are intraventricular. The radiologi-
Neuro-Oncology
567 19
cal features consist of iso- to hypo-inten- occur rarely in the fourth ventricle, tend
sity on T1-weighted images and iso- to to be more globular in shape, and are iso-
hyper-intensity on T2-weighted images to hypo-intense on T1-weighted images.
with heterogeneous enhancement. Cystic
components, calcification, or hemor- zz Suggested Reading
rhage may be present. The lesions are 55 Smith AB, Smirniotopoulos JG,
less likely to be sub-­ependymomas, Horkanyne-Szakaly I. From the radiologic
which, even though many occur in the pathology archieves: intraventricular
fourth ventricle, are much rarer, account- neoplasms: radiologic-pathologic correla-
ing for only 0.2–0.7% of intracranial neo- tion. Radiographics. 2013;33:21–43.
plasms. Ependymomas also tend to be
well-circumscribed and have minimal ?? 17. A 14-year-old female came to your
enhancement. Whereas 40% of choroid office complaining of right periorbital
plexus neoplasms occur in the fourth headache with progressive visual loss.
ventricle, unlike Ependymomas, they On examination, she has a slight right
tend to be highly vascular with intense proptosis, visual acuity 20/70 in the
contrast enhancement. Metastatic neo- right eye and 20/20 in the left eye, a
plasms occur rarely in the 4th ventricle right sided afferent pupillary defect.
and are more likely to manifest surround- On her funduscopic examination, she
ing vasogenic edema. Meningiomas also has a few tiny pigmented nodules

..      Figs. 19.10, 19.11, 19.12 and 19.13 Gliomas of PAMJ-17-256-g001&query=optic+nerve+glioma&it=x


both optic nerve in boy with NF type 1. 7 https:// g&lic=by&req=4&npos=28. (Handor H., Laghmari M.,
openi.nlm.nih.gov/detailedresult.php?img=4189867_ Hafidi Z., Daoudi R. W)
568 Chapter 19 · Neuro-Oncology

involving the iris. On her skin exam, Orbital Pseudo-tumor is an inflam-


she has multiple 2 cm, coffee-colored matory disorder affecting the extraocu-
pigmented nevi are present on the lar muscles, not the optic nerve.
trunk and lower extremities. Brain MRI Pseudo-tumor cerebri can cause
is performed shown. pressure on the optic nerves and place
What is your most likely diagnosis? vision at risk secondary to increased
A. Optic nerve glioma intracranial pressure; MRI findings may
B. Orbital Pseudo-tumor include optic nerve sheath edema; but
C. Orbital rhabdomyosarcoma skin and iris abnormalities are not seen.
D. Orbital sarcoidosis Rhabdomyosarcoma is a rare soft
E. Pseudo-tumor cerebri tissue tumor that typically arises in the
head and neck region or genitourinary
vv Correct Answer is: A tract, with 8% of tumors involving
The MRI shows optic nerve glioma. It is the orbit, but this tumor is not associ-
the most common CNS tumor associ- ated with the skin and iris findings as
ated with neurofibromatosis type 1 described, and most cases occur in
(NF1). ­children.
Diagnostic criteria for NF1, or von
Recklinghausen disease, include having zz Suggested Reading
any two or more of: 55 Islam MP, Roach S. Neurocutaneous
(1) 6 or more café-au-lait spots, (2) syndromes. In: RB Daroff, GM Fenichel, J
two or more Lisch nodules (pigmented Jankovic, JC Mazziotta, editors. Bradley’s
nodules on the iris), (3) optic glioma, neurology in clinical practice. 6th ed.
(4) two or more neuro-fibromas or one Elsevier; 2012. p. 1514–7.
plexiform neuro-­fibroma, (5) a first 55 Elomrani F, Touri S, Ouziane I, et al.
degree relative with NF1, or (6) sphe- Orbital rhabdomyosarcoma with skin
noid dysplasia. metastasis: a case report. BMC Res Notes.
Majority of cases occurring in the 2014;7:670.
first decade of life (peak incidence age 55 Van Stavern G. Metabolic, Hereditary,
1–3), with over 90% of cases in the first Traumatic, and Neoplastic Optic
two decades of life. 25% optic gliomas Neuropathies. Continuum (Minneap
are associated with neurofibromato- Minn). 2014;20(4):877–906.
sis, and 15% of patients with NF1 will
develop optic nerve and/or chiasmal ?? 18. A 17-year-old female is complain-
tumors. ing of progressive hearing loss and
Manifestations of progressive visual gait instability for the last 2 years.
loss in young children may be difficult On examination, she has significant
to detect, and yearly routine ophthal- bilateral sensorineural hearing loss,
mologic surveillance is recommended. moderate bilateral lower motor
Acquired strabismus or nystagmus in neuron facial weakness, and a mildly
young children with NF1 should prompt wide-based gait with a positive Rom-
immediate investigation for such a lesion. berg testing with her eyes closed.
Orbital sarcoidosis is one of the dif- She had an audiometry testing few
ferential diagnosis of an optic nerve years ago revealed significant bilat-
19 mass, but there is no skin and iris find- eral hearing loss and hearing aids
ings are found; patients may have chest did not significantly help her. MRI
x-ray abnormalities and elevated angio- is demonstrated (contrast and non-
tensin converting enzyme. contrast).  
Neuro-Oncology
569 19

R R

..      Figs. 19.14, 19.15 and 19.16 American Academy of Neurology Institute, produced by permission

What is your diagnosis? vv Correct Answer is: B


A. Ataxia Telangiectasia NF2 is an autosomal dominant condition
B. Neurofibromatosis type 2 caused by mutation in the Merlin gene.
C. Neurofibromatosis type I Affected patients are at very high risk for
D. Sturge-Weber syndrome acoustic neuromas (usually bilateral).
E. Tuberous Sclerosis Complex Other features include juvenile cataracts
570 Chapter 19 · Neuro-Oncology

and increased risk for other tumors such


as meningiomas and gliomas. On the
other hand, NF2 cutaneous manifesta-
tions are not characteristic like NF1. The
MRI shows large bilateral cerebellopon-
tine angle lesions, which were consistent
with acoustic neuromas. All of the other
conditions listed are well-characterized
neuro-cutaneous syndromes. Tuberous
Sclerosis complex is associated with sub-
ependymal giant cell astrocytomas, but
not acoustic neuromas.

zz Suggested Reading
55 Mattox DE. Assessment and management
of tinnitus and hearing loss. Continuum
Lifelong Learning Neurol. 2006;12(4):
135–50.
55 Nandhagopal R, et al. Teaching
NeuroImages: vestibular schwannomas in ..      Fig. 19.17 American Academy of Neurology
neurofibromatosis type 2. Neurology. Institute, produced by permission
2010;75.14.
vv Correct Answer is: C
?? 19. You are seeing a 6-year-old boy in the MRI shows signs of both midline and
ED with a history of progressive vomit- hemispheric cerebellar compromise and
ing and headache for 3 days. He has a diffusely enhancing mass in the region
no history of fever and has no previous of the fourth ventricle. Medulloblastoma
history of headaches. Medical history is the only tumor from the above list
is unremarkable. On examination, would occur in the posterior fossa.
he is lethargic, he can speak but in a Posterior fossa is the most common area
slow and staccato quality. He has gaze in almost 85% of primary brain tumors
evoked nystagmus with attempts at in children between the ages of 2 and
horizontal gaze. Fundus exam is nor- 12. Medulloblastoma, juvenile pilocytic
mal. Motor exam shows appropriate. He astrocytoma (JPA), and ependymoma
has normal tone and strength while in and brainstem glioma are the four most
bed, but he is too unsteady to sit on his common tumors in this region in child-
own. He has bilateral finger-nose-­finger hood. The rest of tumors mentioned are
dysmetria. Brain MRI with a gadolinium- supra-tentorial region tumors, mainly in
enhanced T1-weighted image is shown. the temporal lobes, and present with
Which of the following tumors is most complex partial seizures or progressive
likely given the patient’s age, neuro- focal motor deficits. Medulloblastoma is
logic findings and imaging results? a primary neuroectodermal tumor
A. Dysembryoplastic Neuroepithelial (PNET) of the posterior fossa, usually in
tumor (DNET) the region of the cerebellar vermis or
19 B. Ganglioglioma 4th ventricle. 90% occur in children
C. Medulloblastoma under the age of 10. These are rapidly
D. Oligodendroglioma growing tumors, usually with a short
E. Pleomorphic Xanthoastrocytoma period of time between symptom onset
Neuro-Oncology
571 19
and diagnosis. The most presenting aspect of the optic chiasm and hypo-
symptoms are headache, vomiting, thalamus. Peak age is 6–14 years, and
unsteady gait and torticollis or stiff the symptom onset is insidious and pro-
neck. Limb dysmetria is also noted. gression is slow. Progressive visual
Papilledema occurs in 2/3 of patients impairment (bi-temporal hemianopia in
due to hydrocephalus secondary to 50%), delayed sexual maturation,
obstruction of outflow through the com- growth delay and diabetes insipidus are
pressed 4th ventricle or cerebral aque- the most common symptoms. Headache
duct. In this setting, limb spasticity and/ and papilledema occur in 25% second-
or brisk reflexes may also be present. ary to hydrocephalus.
The overall 5-year survival rate is Hypothalamic hamartoma is a
approximately 40%, though this number non-­progressive, benign tumor that
is higher in children who undergo a results in a syndrome characterized by
gross total resection. Craniospinal irradi- medically refractory seizures (including
ation and chemotherapy are commonly gelastic seizures), developmental delays
utilized after surgery. Dissemination and psychiatric symptoms.
through the neuro axis is possible, Pituitary apoplexy is caused by a
including drop metastases to the spinal hemorrhage into the pituitary gland
cord and nerve roots. causing a sudden onset of headache,
ophthalmoplegia, altered mental status
zz Suggested Reading and hormonal dysfunction. Eye pain and
55 Fenichel GM. Clinical pediatric neurology. blurred vision that can progress to par-
6th ed. Philadelphia: Saunders Elsevier; tial or total blindness in hours to days.
2009. p. 237–40. In adults it is monocular, but in children
55 Dhall G. Medulloblastoma. J Child Neurol. up to 50% of the cases are binocular.
2009;24(11):1418–30.
zz Suggested Reading
?? 20. You are evaluating a 7-year-old boy 55 Zada G, Lopes BS, Mukundan S Jr, Laws E
with 3-week history of headaches. He Jr. Craniopharyngiomas. In Atlas of Sellar
has generalized upon awaking in the and Parasellar Lesions. Springer
morning headache. He complains of International Publishing; 2016.
increased difficulty seeing the black- p. 197–210.
board in school and while reading. His
physical exam is remarkable for short ?? 21. This is a histopathologic specimen
stature. On neurological examination from an 8-year-old boy with intrac-
he has restricted peripheral vision, table epilepsy underwent temporal
visual acuity in the right is 20/30 and lobectomy.  
left eye is 20/40 with signs of optic Which statement below is incorrect
atrophy bilaterally.   regarding this tumor?
What is your most likely diagnosis? A. Most common location is the tem-
A. Hypothalamic hamartoma poral lobe
B. Pituitary apoplexy B. This lesion often has a cortical or
C. Craniopharyngioma juxta-cortical location
D. Optic neuritis C. Floating neurons are characteristic
of this lesion
vv Correct Answer is: C D. It is a WHO grade I tumor
Craniopharyngioma is a supra-sellar E. on MRI, it has a heterogeneous
tumor that compresses the medial contrast-enhancing pattern
572 Chapter 19 · Neuro-Oncology

..      Fig. 19.18 Dyembryoplastic Neuroepithelial tumour (DNET). (Own work. Sbrandner. W)

vv Correct Answer is: E WHO grade I tumor, and surgical resec-


This is typical pathologic features of tion is helpful for the cure or control of
dysembryoplastic neuroepithelial tumor the ­seizures.
(DNET), showing prominent clear spaces
that seem to contain ganglion cells. In zz Suggested Reading
between these clear spaces, there are 55 Prayson RA, GoldblumJR.
multiple cells and a glial component Neuropathology. 1st ed. Philadelphia:
that resembles an oligodendroglioma. Elsevier; 2005.
DNET is a benign superficial tumor that 55 Ropper AH, Samuels MA. Adams and
is an important cause of refractory sei- Victor’s principles of neurology. 9th ed.
zures in children. These tumors are New York: McGraw-Hill; 2009.
located in the cortical or juxta-cortical 55 Comprehensive Review in Clinical
region, most commonly in the temporal Neurology: A Multiple-Choice Question
lobe. The MRI shows a nodular or cystic Book for the Wards and Boards © 2011
lesion that is hyperintense on Wolters Kluwer Health Lippincott
T2-weighted images and does not Williams & Wilkins. All rights reserve.
enhance with contrast. It grows very
slowly, and macroscopically, it has a ?? 22. Which of the following pineal region
multinodular architecture, with muci- tumors is most likely to have an excel-
nous cysts. Histologically, it is seen as lent prognosis?
multiple areas in which ganglion cells A. Embryonal carcinoma
appear to float within mucin-filled B. Mixed germ cell tumor
spaces (“floating neurons”). In between C. Yolk sac tumor
these nodules, there is a component D. Germinoma
19 with more glial characteristics, some-
times resembling an oligodendrogli- vv Correct Answer is: D
oma. At the edges of these lesions, there Germinoma has the best prognosis of all
may be cortical dysplasia. DNET is a the choices. Non-germinomatous germ cell
Neuro-Oncology
573 19
tumors include teratoma, embryonal carci- 55 Thompson YY, Ramaswamy V, Diamandis P,
noma, yolk sac tumor, choriocarcinoma, Daniels C, Taylor MD. Posterior fossa
and mixed germ cell tumor. Mature tera- ependymoma: current insights. Childs Nerv
toma may also have an excellent prognosis, Syst. 2015;31(10):1699–706.
but this is not one of the answer choices. 55 Korshunov A, Witt H,Hielscher T, Benner A,
Choriocarcinoma and mixed germ cell Remke M, Ryzhova M, Milde T, Bender S,
tumors both have intermediate prognoses. Wittmann A, Schettler A, Kulozik AE,
Embryonal carcinoma and Yolk sac Witt 0, Deimling AV, Lichter P, Pfister S.
tumor are a non-germinomatous germ cell Molecular staging of intracranial ependy-
tumor with a poor and worst ­prognosis. moma in children and adults. J Clin Oncol.
Mixed germ cell tumor is a non-­ 2010;28(19):3182–90.
germinomatous germ cell tumor with an 55 Collins P, Jones DTW, Giannini C.
intermediate prognosis. Pilocytic astrocytoma: pathology, molecu-
lar mechanisms, and markers. Acta
zz Suggested Reading Neuropathol. 2015;129(6):775–88.
55 Dhall G. Medulloblastoma. J Child Neurol.
2009;24(11):1418–30. ?? 23. Corticosteroids are used for the treat-
55 Wells EM, Packer RJ. Pediatric brain ment of intracranial hypertension
tumors. Continuum: Lifelong Learning associated with which of the follow-
Neurol. 2015;21(2):373–96. ing etiologies?
55 Cohen KJ, Heideman RL, Zhou T, Holmes A. Traumatic brain injury
EJ, Lavey RS, Bouffet E, Pollack B. Intracerebral hemorrhage
IF. Temozolomide in the treatment of C. Ischemic stroke
children with newly diagnosed diffuse D. Acute obstructive hydrocephalus
intrinsic pontine gliomas: a report from E. Brain tumors
the Children’s Oncology Group. Neuro-
Oncology. 2011;13(4):410–6. vv Correct Answer is: E
55 Fujimaki T. Central nervous system germ Corticosteroids are used for the treat-
cell tumors: classification, clinical features, ment of intracranial hypertension
and treatment with a historical overview. J associated with primary brain tumors
Child Neurol. 2009;24(11):1439–45. and metastasis to the brain.
55 Frappaz D, Center CF, Szathmari A, Corticosteroids are beneficial in vaso-
Valsijevic A, Mottolese C. The management genic cerebral edema as is seen with
of pineal tumors as a model for a multidis- intracranial tumors, either primary or
ciplinary approach in neuro-oncology. metastatic. However, the exact mecha-
Neurochirurgie. 2015;61(2):208–11. nism of action of steroids in vasogenic
55 Faure-Conter C. Tumoral markers in edema is not well understood.
tumors of the pineal region. Available evidence suggests that corti-
Neurochirurgie 2015;61(2):143–5. costeroids are not useful in the man-
55 Daras M, Kaley TJ. Benign brain tumors agement of other conditions
and tumors associated with phakomatoses. commonly associated with cerebral
Continuum: Lifelong Learning Neurol. edema such as traumatic brain injury,
2015;21(2):397–414. intracerebral hemorrhage, or ischemic
55 Pajtler KW, Witt H, Sill M, et al. Molecular stroke. This is explained in part by the
classification of ependymal tumors across difference in the type of edema seen in
All CNS Compartments, Histopathological the latter conditions. Acute obstructive
Grades, and Age Groups. Cancer Cell. hydrocephalus requires a neurosurgi-
2015;27:728–43. cal intervention.
574 Chapter 19 · Neuro-Oncology

zz Suggested Reading 55 Comprehensive Review in Clinical


55 Suarez JI. Critical care neurology and Neurology: A Multiple-Choice Question
neurosurgery. In: Totowa, NJ editor. 1st ed. Book for the Wards and Boards ©2011
Humana Press; 2004. Wolters Kluwer Health Lippincott
55 Comprehensive Review in Clinical Williams & Wilkins. All rights reserved.
Neurology: A Multiple-Choice Question
Book for the Wards and Boards ©2011 ?? 25. You are evaluating an 11-year-old
Wolters Kluwer Health Lippincott boy with a history of headaches. On
Williams & Wilkins. All rights reserved. examination, he has restriction of
voluntary up gaze and his pupils are
?? 24. Which of the following tumors does more responsive to accommodation
not typically present with seizures? than to light. Hyper-intense enhanc-
A. Ganglioglioma ing mass in the pineal region is noted
B. Dysembryoplastic neuroepithelial on T2 flair MRI.  
tumor What is your next step in manage-
C. Oligodendroglioma ment?
D. Ependymoma A. Biopsy of the mass
E. Astrocytoma B. Surgical resection followed by
chemotherapy
vv Correct Answer is: D C. Surgical resection followed by
Seizures are not common with chemotherapy and radiation
Ependymoma. Ependymoma is the third D. Serum levels of beta human cho-
most common CNS tumor in children, rionic gonadotropin (beta-­HCG)
and 90% occur intracranially, most com- and alpha fetoprotein (AFP) levels
monly in the infra-­tentorial region, typi- in serum and cerebrospinal fluid
cally in the fourth ventricle. E. Recheck MRI in 1 month
Hydrocephalus is the most common
manifestations because those tumors vv Correct Answer is: D
obstruct the CSF flow because of their Pineoblastoma, pineal parenchymal
location and cause increased intracra- tumor, germ cell tumor, and astrocy-
nial pressure. Ependymomas in the toma are common pineal region tumors
supra-tentorial region occur in the peri- in the pediatric age group. Beta -HCG
ventricular region, but they may uncom- and AFP levels in the serum and CSF
monly be more superficial. should be checked before performing a
Ependymomas can also occur in the biopsy on some suspected germ cell
spinal cord, more commonly in adults. tumors, because biopsy or surgery can
Ganglioglioma, dysembryoplastic be avoided.
neuroepithelial tumors, oligodendro- Germinomas are often associated
gliomas, and astrocytomas frequently with slight elevation of beta-HCG, usually
present with seizures. under 25 international units (IU) per liter.
Choriocarcinoma (nongerminomatous
zz Suggested Reading germ cell tumor) is typically associated
55 Bradley WG, Daroff RB, Fenichel GM, with elevations in beta-HCG while yolk sac
et al. Neurology in clinical practice. 5th ed.
19 Philadelphia: Elsevier; 2008.
tumors are associated with AFP elevation.
Mixed germ cell tumors may have
55 Prayson RA, Goldblum JR. elevations in both AFP and beta-HCG
Neuropathology. 1st ed. Philadelphia: while teratomas, particularly mature
Elsevier; 2005. teratomas, are often non- secreting.
Neuro-Oncology
575 19
When a pineal region tumor is neuro-oncology. Neurochirurgie.
noted, and a germ cell tumor suspected, 2015;61(2):208–11.
the first step in management is mea- 55 Faure-Conter C. Tumoral Markers in
surement of beta-HCG and AFP levels in Tumors of the Pineal Region
CSF and serum. If beta-­HCG are elevated Neurochirurgie. 2015;61(2):143–5.
to at least 50 IU/L, and/ or AFP is ele- 55 Daras M, Kaley TJ. Benign brain tumors
vated, a biopsy is often not n ­ ecessary and tumors associated With phakomato-
and the diagnosis of non germinoma- ses. Continuum: Lifelong Learning in
tous germ cell tumor can be made. Neurol. 2015;21(2):397–414.
Serum and CSF levels of AFP and beta 55 Pajtler KW, Witt H, Sill M, et al. Molecular
HCG should be obtained. If these levels classification of ependymal tumors across
are normal biopsy might be necessary. All CNS Compartments, Histopathologi
Surgical resection is important in cal Grades, and Age Groups. Cancer Cell.
the management of mature teratomas, 2015;27:728–43.
but its role in prolonging survival in ger- 55 Thompson YY, Ramaswamy V, Diamandis P,
minomas and other nongerminomatous Daniels C, Taylor MD. Posterior fossa
germ cell tumors is unclear. ependymoma: current insights. Childs Nerv
Again, serum and CSF levels of AFP Syst. 2015;31(10):1699–706.
and beta HCG should be obtained ini- 55 Korshunov A. Witt H, Hielscher T, Benner
tially. Surgical resection is important Remke M, Ryzhova M, Milde T, Bender S,
in management of mature teratomas, Wittmann A, Schettler A, Kulozik AE,
while others are treated with chemo- Witt 0, Deimling AV, Lichter P, Pfister
therapy and radiation alone. S. Molecular Staging of lntracranial
Serum and CSF levels of AFP and Ependymoma in Children and Adults. J
beta HCG should be obtained; a follow Clin Oncol. 2010;28(19):3182–90.
up MRI is not the best option. 55 Collins P, Jones DTW, Giannini
C. Pilocytic astrocytoma: pathology,
zz Suggested Reading molecular mechanisms, and markers. Acta
55 Dhall G. Medulloblastoma. J Child Neurol. Neuropathol. 2015;129(6):775–88.
2009;24(11):1418–30. 55 Recht LO, van den Bent M, Shih
55 Wells EM, Packer RJ. Pediatric brain H. Management of low grade glioma. Wen
tumors. Continuum: Lifelong Learning PY, Loeffler JS, Eichler AF, editors.
Neurol. 2015;21(2):373–96. Up-to-date 2015.
55 Cohen KJ, Heideman RL, Zhou T, Holmes 55 Quaglietta L, Vitale V, Verrico A,
EJ, Lavey RS, Bouffet E, Pollack Migliorati R, Ozek M, Cinalli G, Maixner
IF. Temozolomi de in the treatment of W, Sainte-­Rose C. Treatment of
children with newly diagnosed diffuse Medulloblastoma: Chemotherapy. In:
intrinsic pontine gliomas: a report from Posterior Fossa Tumors in Children.
the Children’s Oncology Group. Neuro- Cham: Springer International Publishing;
Oncology. 2011;13(4):410–6. 2015. p. 349–61.
55 Fujimaki T. Central nervous system germ 55 Nolan CP, DeAngelis LM. Neurological
cell tumors: classification, clinical features, complications of chemotherapy and
and treatment with a historical overview. J radiation therapy. Wen PY, ed.
Child Neurol. 2009;24(11):1439–45. Continuum: Lifelong Learning in Neurol
55 Frappaz D, Conter CF, Szathmari 2015;21(2).
A. Valsijevic A. Mottolese 55 Legault G, Allen JC. Germ cell tumors and
C. Themanagement of pineal tumors as a other pineal region tumors. In: Packer RJ,
model for a multidisciplinary approach in Schiff D, editors. Neuro-Oncology. 1st ed.
576 Chapter 19 · Neuro-Oncology

Hoboken: Wiley-Blackwell; 2012. up gaze and when he attempted to


p. 95–102. follow your finger upwards, his eyes
55 Alexiou GA, Varela M, Prodromou N. were noted to retract. His MRI shows
Management of pineal region tumours in a heterogeneously enhancing pineal
children. J Solid Tumors. 2012;2(2). region tumor. You ordered alpha
55 Packer, RJ. Embryonal Tumors. In: Packer RJ, fetoprotein (AFP) and beta human
Schiff D, editors. Neuro-Oncology. 1st ed. chorionic gonadotropin (beta HCG) in
Hoboken: Wiley-Blackwell; 2012. p. 157–66. serum and cerebrospinal fluid. AFP is
markedly elevated while beta HCG is
?? 26. A newborn girl with megaencephaly within normal limits. You started him
has a prenatal ultrasound showed on carboplatin and etoposide, alter-
mass growing in both lateral ventricles. nating with ifosfamide and etoposide.
What is the most likely diagnosis? Although AFP levels normalize, tumor
A. Ependymoma growth is noted on imaging.  
B. Craniopharygioma What is the most likely cause?
C. Teratoma A. Regrowth of the patient’s existing
D. Meningioma germinom
B. Regrowth of the patient’s exist-
vv Correct Answer is: C ing nongerminomatous germ cell
Intracranial teratomas account for 50% tumor
of all congenital brain tumors. C. Growing teratoma syndrome
Teratomas are benign tumors and con- D. Growing germinoma syndrome
sist of all 3 germinal layers. Brain terato-
mas are also commonly located in the vv Correct Answer is: C
pineal, supra-cellar and cerebellar This is a presentation of nongermino-
regions. Other less common congenital matous germ cell tumor based on the
tumors include astrocytomas, choroid boys’ initial labs, and imaging findings.
plexus papilloma, primitive neuroecto- Typically, germinomas are associated
dermal tumors, rhabdoid tumors, and with mild elevations in beta HCG. Yolk
medulloblastomas. sac tumors and choriocarcinomas are
Congenital ependymomas are very both nongerminomatous germ cell
rare but do occur in the ventricles. tumors. Yolk sac tumors are associated
Congenital craniopharygiomas are with high levels of AFP while choriocar-
rare and located in near the pituitary cinomas are associated with high levels
stalk. of beta hCG. The boy is most likely hav-
Congenital meningiomas are ing a yolk sac tumor because his ele-
extremely rare and do not develop in vated AFP and normal beta HCG. These
the ventricles. are often treated with a combination of
radiation and chemotherapy. Growing
zz Suggested Reading teratoma syndrome occurs in about
55 Severino M. Congenital tumors of the 21% of intracranial nongerminomatous
central nervous system. Neuroradiology. germ cell tumors, usually during treat-
2010. 52(6):531–48. ment. Often, growth on MRI, accompa-
nied by normalization of serum and CSF
19 ?? 27. You are seeing a 9-year-old boy for his markers is noted. This is typically man-
headache. You examined him and you aged with maximal surgical resection
noted that, he has limited voluntary and radiotherapy.
Neuro-Oncology
577 19
Germinomas are typically associated
with mild elevations in beta HCG, not
marked elevations of AFP.
Normalization of serum and CSF AFP
levels with growth on imaging is consis-
tent with growing teratoma syndrome;
regrowth of the original tumor would be
associated with continued increases in AFP.
The patient’s most probable initial
diagnosis is nongerminomatous germ
cell tumor. Growing germinoma syn-
drome does not exist.

zz Suggested Reading
55 Jozwiak S, Migone N, Ruggieri M. The
tuberous sclerosis complexs. In: Ruggieri M,
Pascual­Castroviejo I, Di Rocco C, editors.
Neurocutaneous disorders Phakomatoses
and Hamartoneoplastic syndromes.
Morlenbach: Springer-Verlag/Wien; 2008.
p. 181–227.
55 Tabori U, Laberge A-M, Ellezam B, Carret
..      Fig. 19.19 Vestibular schwannoma. 7 RadsWiki.
A-S. Cancer predisposition in children with (RadsWiki. Wikipedia)
braintumors. In: Scheinemann K, Bouffet E,
editors. Pediatric Neuro-­Oncology.
New York: Springer; 2015. p. 69–89. What is your most likely diagnosis?  
55 Gorlin, RJ. Nevoid basal cell carcinoma. In: A. Vestibular schwannoma
Ruggieri M, Pascual-Castroviejo I, Di B. Oligodendroglioma
Rocco C, editors. Neurocutaneous disor- C. Cerebellar hemangioblastoma
ders Phakomatoses and Hamartoneoplastic D. Ependymoma
syndromes. Morlenbach: Springer/Wien; E. Metastatic lesion
2008. p. 669–94.
55 Chan TSY, Wang X, Spence T, Taylor MD, vv Correct Answer is: A
Huang A. Embryonal brain cell tumors. In: The MRI shows a vestibular schwan-
Scheinemann K, Bouffet E, editors. noma. Schwannoma occur commonly in
Pediatric Neuro-Oncology. New York: the fourth and fifth decades, and they
Springer; 2015. p. 127–38. are on the vestibular portion of cranial
55 Lee J, Johnston DL. Chemotherapy. In: nerve VIII. They grow in the cerebello-
Scheinemann K, Bouffet E, editors. pontine angle, and may compress the
Pediatric Neuro-Oncology. New York: brain stem and erode into the internal
Springer; 2015. p. 69–89. auditory meatus. They grow slowly and
they are either asymptomatic, or pres-
?? 28. You are seeing an 18-year-old female ent with hearing loss, tinnitus, and in
with a history of gradually progres- some cases cerebellar findings given
sive unsteadiness and hearing loss in that the cerebellar peduncles are com-
the left ear. An MRI is obtained and is pressed. On imaging, they are circum-
shown below.   scribed isointense tumors with contrast
578 Chapter 19 · Neuro-Oncology

enhancement. The location and radio- associated with EBV. In general, PCNSL
logic appearance of the tumor as shown in immunocompromised patients
in the figure, distinguish it from the occurs at earlier ages than in those who
other choices. It is extra-axial (not are immunocompetent. It is not possi-
within the brain parenchyma) and is not ble to differentiate PCNSL in immuno-
within the ventricular system. This is not compromised patients versus
a location typical for metastatic lesions. immunocompetent patients only on the
Given the clinical and radiologic find- basis of imaging, CSF findings, or cytol-
ings, this patient does not have an oli- ogy. There is no clear evidence of differ-
godendroglioma, cerebellar ences in response to steroids in either
hemangioblastoma, ependymoma, or group.
metastatic tumor.
zz Suggested Reading
zz Suggested Reading 55 Comprehensive Review in Clinical
55 Bradley WG, Daroff RB, Fenichel GM, Neurology: A Multiple-Choice Question
et al. Neurology in clinical practice. 5th ed. Book for the Wards and Boards ©2011
Philadelphia: Elsevier; 2008. Wolters Kluwer Health Lippincott
55 Prayson RA, Goldblum JR. Williams & Wilkins. All rights reserved.
Neuropathology. 1st ed. Philadelphia:
Elsevier; 2005. ?? 30. The medulloblastomas are respon-
55 Comprehensive Review in Clinical sible for great percentage of tumors
Neurology: A Multiple Choice Question during childhood.  
Book for the Wards and Boards © 2011 What is the INCORRECT option about
Wolters Kluwer Health Lippincott these tumors?
Williams & Wilkins. All rights reserved. A. Typically, they originate at the cer-
ebellar vermis
?? 29. Primary CNS lymphoma occurs more B. They are pathologically different
frequently in patients with AIDS but from the PNETs (primitive neuro-
may also be diagnosed in immuno- ectodermal tumors)
competent patients. What is the dif- C. Use of ventriculo-­peritonea l
ference between the CNS lymphoma derivation can lead to peritoneal
in the patient populations? metastasis
A. Association with Epstein Barr virus D. There is no difference in survival
B. Findings on cytology rate between partial or total resec-
C. Response to steroids tion, but only if a surgical proce-
D. B-cell origin dure is followed by radiotherapy0
E. M RI findings Radiation of the entire neuro-axis
is normally recommended.
vv Correct Answer is: A
Primary CNS lymphoma (PCNSL) is more vv Correct Answer is: D
frequently diagnosed in immunocom- Studies show that groups of patients
promised patients, especially in patients treated by radical or gross total resec-
with AIDS. Almost all of these cases are tion showed much higher postoperative
associated with Epstein-Barr virus (EBV), survival, so it’s incorrect to affirm only
19 and CSF EBV PCR is helpful in the diag- radiotherapy affects the survival rate.
nosis. In contrast, PCNSL in immuno- All the other options are correctly
competent patients is not commonly describe medulloblastomas.
Neuro-Oncology
579 19
zz Suggested Reading includes temozolomide, which has
55 Raimondi AJ, Tomita T. Medulloblastoma replaced PCV (procarbazine, lomustine,
in childhood: comparative results of and vincristine).
partial and total resection. Childs Brain.
1979;5(3):310–28. zz Suggested Reading
55 Bradley WG, Daroff RB, Fenichel GM,
?? 31. Regarding the prognosis of oligoden- et al. Neurology in clinical practice. 5th ed.
drogliomas, which of the following is Philadelphia: Elsevier; 2008.
correct? 55 Prayson RA, Goldblum JR.
A. Chromosome 1p and 19q dele- Neuropathology. 1st ed. Philadelphia:
tions confer a worse prognosis Elsevier; 2005.
B. Younger patients have better 55 Comprehensive Review in Clinical
survival Neurology: A Multiple-Choice Question
C. p16 gene deletion is associated Book for the Wards and Boards ©2011
with better outcomes Wolters Kluwer Health Lippincott
D. p53 mutations occur in more than Williams & Wilkins. All rights reserved.
50% of the cases
E. The prognosis in oligodendrogliom. ?? 32. Which of the following is incorrect
as is worse than in astrocytomas regarding brain metastases?
A. They are more common than pri-
vv Correct Answer: B mary brain tumors
The prognosis in oligodendrogliomas is B. Hemorrhagic metastasis can be
better than in astrocytomas. Other fac- seen with melanomas, non-small
tors that influence prognosis include cell carcinomas, and renal cell car-
age of the patient (better survival with cinomas
younger patients), pre-surgical perfor- C. The majority are infra-­tentorial
mance status (worse prognosis with D. Multiple metastatic lesions can be
poorer baseline functional and neuro- seen with small cell carcinomas
logic status), bulk of resection (the and melanomas
greater the extent of resection the bet- E. Steroids are used to treat surround-
ter), and genetic alterations. ing edema in brain metastasis
Important genetic alterations are
chromosome 1p and 19q co-deletions, vv Correct Answer is: C
which occur in 50–80% of the cases, and Metastatic lesions are more common
are associated with a better prognosis, than primary brain tumors. 80% of brain
enhanced survival, and better response metastases are supra-tentorial and 20%
to chemotherapy, radiation therapy, infra-tentorial. Infra-cranial metastatic
or both. Mutation of p53 is not typi- lesion can affect the skull and dura and
cally seen in oligodendrogliomas, and brain parenchyma or produce a menin-
its presence suggests the possibility geal carcinomatosis. The most common
of an astrocytic neoplasm. Deletion of Lung is the most common source of
the chromosomal region p16 is more metastasis to the brain, followed by
commonly seen in anaplastic astrocy- breast and then melanoma.
tomas and glioblastomas, and is associ- Gastrointestinal tumors (especially
ated with progression. Treatment of colon and rectum), kidney cancer and
oligodendrogliomas involves surgical tumors originating from the gallbladder,
resection, chemotherapy, and radia- liver, thyroid, testicle, uterus, ovary, and
tion therapy. The chemotherapy used pancreas may produce brain metastasis.
580 Chapter 19 · Neuro-Oncology

Prostate, esophagus, oropharynx, and D. Screening for somatic and germ-


skin (other than melanoma) tumors are line mutations in the gene coding
rare tumors to have parenchymal brain for the protein tuberin
metastases. Colon and pelvic cancers
have a tendency to spread to the poste- vv Correct Answer is: C
rior fossa. Metastatic lesions can be mul- Choroid plexus carcinoma occurs in
tiple or single. Small cell carcinomas and patients with Li Fraumeni syndrome.
melanomas can have multiple metasta- There are two sets of diagnostic criteria
ses. Those that are frequently found as for this disorder.
single metastasis originate from the kid- In the Classic Li Fraumeni Syndrome
ney, breast, thyroid, or adenocarcinoma Diagnostic criteria, a patient must have
of the lung. Hemorrhagic metastases are all of the following:(1) presence of a
seen with melanoma, choriocarcinomas, sarcoma diagnosed prior to age 45 years
non-small cell carcinomas, thyroid carci- (2) presence of a 1st degree relative
nomas, and renal cell carcinomas. with any cancer before the age of 45.
Metastasis in the brain can cause sei- (3) A 1st or 2nd degree relative with any
zure, focal neurologic findings, head- cancer before age 45 years OR with a
aches, and sometimes with increased sarcoma at any time.
intracranial pressure. Symptom onset is The Compete diagnostic criteria for
typically relatively rapid and abrupt Li Fraumeni are slightly different. In this
rather than gradual. The treatment of case, a diagnosis of Li Fraumeni can be
brain metastasis includes brain irradia- made in 1 of 4 circumstances: (1) the
tion, surgical intervention for solitary patient has at least 1 tumor typical of
metastasis in some cases, and chemo- LFS diagnosed prior to age 46 (brain
therapy. Steroids play a significant role tumor, adrenocortical carcinoma, leuke-
in the treatment of surrounding edema. mia, bronco alveolar cancer, soft tissue
sarcoma, osteosarcoma, or premeno-
zz Suggested Reading pausal breast cancer) WITH > 1 1st or
55 Prayson RA, Goldblum JR. 2nd degree relative with a tumor typical
Neuropathology. 1st ed. Philadelphia: for LFS before age 56, or the presence
Elsevier; 2005. of multiple tumors. (2) The patient has
55 Comprehensive Review in Clinical many tumors,>2 within the disease
Neurology: A Multiple-Choice Question spectrum, and > 1 occurring prior to
Book for the Wards and Boards ©2011 age 46. (3) The patient has adrenocorti-
Wolters Kluwer Health Lippincott cal carcinoma or a tumor of the choroid
Williams & Wilkins. All rights reserved. plexus.
It is suggested that screening for
?? 33. Which of the following is recom- mutations in TP53 should be performed
mended for a patient with choroid in (1) individuals meeting criteria for the
plexus carcinoma? syndrome (2) women with breast cancer
A. Screening for somatic and germ- before age 30 with no BRCA1 or BRCA 2
line mutations in the gene coding mutations (3) people with adrenocorti-
for the protein neurofibromin cal carcinoma (4) people with choroid
B. Screening for somatic and germ- plexus carcinoma or (5) people with
19 line mutations in the gene cord- childhood sarcoma.
ing for the protein hamartin Ewings sarcoma is an exception to
C. Screening for somatic and germ- the latter. Thus, a patient with a choroid
line mutations in the gene coding plexus carcinoma should be screened for
for the protein p53 germline mutations in the TP53 gene.
Neuro-Oncology
581 19
Mutations in the NF1 gene lead to Pediatric Neuro-Oncology. New York:
neurofibromatosis type 1. This gene Springer; 2015. p. 127–38.
encodes neurofibromin. This syndrome 55 Lee J, Johnston DL. Chemotherapy. In:
is associated with gliomas including Scheinemann K, Bouffet E, eds. Pediatric
those affecting the optic pathway and Neuro-Oncology. New York: Springer;
brainstem. 2015. p. 69–89.
Mutations in the tuberous sclerosis 55 Glass TD, Cochrane D, Rassekh SR,
complex 1 (TSC1) gene encode for the Goddard K, Hukin J. Growing teratoma
protein hamartin. This is associated with syndrome in intracranial non germino-
tuberous sclerosis complex a syndrome matous germ cell tumors (iNGGCTs): a
associated with sub-ependymal giant risk for secondary malignant transforma-
cell astrocytoma (SEGA) and sub-epen- tion- a report of two cases. Child Nerv
dymal nodules. Syst. 2014;30(5). p. 953–7.
Mutations in the tuberous sclerosis 55 Bartels U, Singhal A. Central nervous
complex 2 (TSC2) gene encode for the system germ cell tumors. In: Scheinemann
protein tuberin. This is also associ- K, Bouffet E, editors. Pediatric Neuro-­
ated with tuberous sclerosis complex. Oncology. New York: Springer; 2015.
Mutations the NF2 gene lead to dys- p. 149–62.
function of the protein merlin, associ- 55 Mach SC, Ramaswamy V, Wang X, Remke
ated with neurofibromatosis type 2. M, Sin-Chan P, Chan WSY, Bertrand KC,
Bilateral vestibular schwannomas are Merino D, Zayne K, Huang A, Taylor MD.
characteristic of this disorder and a Basic science of pediatric brain tumors. In:
diagnosis of neurofibromatosis type Scheinemann K, Bouffet E, editors.
2 can be made in the presence of this Pediatric Neuro-Oncology. New York:
alone. Ependymomas are also character- Springer; 2015. p. 59–67.
istic of this syndrome. 55 Jones DTW, Korshunov A. Pfister SM,
Tayllor MD, Northcott PA.
zz Suggested Reading Medulloblastoma and CNS primitive
55 Jozwiak S, Migone N, Ruggieri M. The neuroectodermal tumors. In: Cagle PT,
tuberous sclerosis complexes. In: Ruggieri Matthias KA, Zagzag D, editors. Molecular
M, Pascual­Castroviejo I, Di Rocco C, pathology of nervous system tumors.
editors. Neurocutaneous disorders New York: Springer; 2015. p. 121–42.
Phakomatoses and Hamartoneoplastic 55 Vanan Ml, Mehta V, Eisenstat DD. Diffuse
syndromes. Morlenbach: Springer-Verlag/ intrinsic pontine glioma. In: Scheinemann K,
Wien; 2008. p. 181–227. Bouffet E, editors. Pediatric Neuro-­Oncology.
55 Tabori U, Laberge A-M, Ellezam B, Carret New York: Springer; 2015. p. 117–26.
A-S.Cancer predisposition in children with 55 Rutkowski S, Bode U, Deinlein F,
brain tumors. In: Scheinemann K, Bouffet E, Ottensmeier H, Warmuth-Metz M,
editors. Pediatric Neuro-Oncology. Soeroenson N,Grade N, Emser A. Pietsch T,
New York: Springer; 2015. p. 69–89. Wolff JE, Kortmann RD, Kuehl J. Treatment
55 Gorlin, RJ. Nevoid basal cell carcinoma. In: of early childhood medulloblastoma by
Ruggieri M, Pascual-Castroviejo I, Di post-­operative chemotherapy alone. N Eng J
Rocco C, editors. Neurocutaneous disor- Med. 2005;352(10):978.
ders Phakomatoses and Hamartoneoplastic 55 Geyer JR, Sposto R, Jennings M, Boyett
syndromes. Morlenbach: Springer-Verlag/ JM, Axtell RA, Brieger D, Broxso E,
Wien; 2008. p. 669–94. Donahue B, Finlay JL, Goldwein JW, Heier
55 Chan TSY, Wang X, Spence T, Taylor MD, LA, Johnson DM, Mazewski C, Miller DC,
Huang A. Embryonal brain cell tumors. In: Packer R, Puccetti D, RadcliffeJ, Tao ML,
Scheinemann K, Bouffet E, editors. Shiminski-Maher T. Multiagent chemo-
582 Chapter 19 · Neuro-Oncology

therapy and deferred radiotherapy in vv Correct Answer is: C


infants with malignant brain tumors: a Slight elevation of beta HCG accompa-
report from the Children’s Cancer Group. nied by normal AFP levels are consistent
J Clin Oncol. 2005;23(30):7621. with germinoma.
55 Wells EM, Packer RJ. Pediatric Brain Yolk sac tumor is a non-germinoma-
Tumors. Continuum: Lifelong Learning tous germ cell tumor typically associated
Neurol. 2015;21(2):373–96. with elevated levels of AFP, although
55 Fleming AJ. Rare tumors of the central beta HCG may also be mildly elevated.
nervous system in children. In: Mature teratomas tend not to
Scheinemann K, Bouffet E, editors. secrete beta HCG or AFP, while chorio-
Pediatric Neuro-Oncology. New York: carcinomas tend to secrete high levels of
Springer; 2015. p. 185–94. beta HCG alone. Embryonal carcinoma
55 Evans DW. Li-Fraumeni syndrome. In: may secrete both beta HCG and AFP.
Up-to-date, Wen PY, Ross ME (Ed), Yolk sac tumors are typically associ-
Up-to-date, Waltham, MA (Accessed on ated with elevated levels of AFP although
November 22 2015). beta HCG can be mildly elevated.
55 Ichimura K, Nishikawa R, Matsutani Mature teratomas are usually not
M. Molecular markers in pediatric associated with elevations in AFP or
neuro-­oncology. Neuro­ Oncology. beta HCG, although this can occur.
2012;14(Suppl 4):iv90–9. doi:7 https://doi. Choriocarcinoma is typically associ-
org/10.1093/neuonc/nos204. ated with more marked elevations of
55 Vora NL. Melanoma and BRAF. eMedi- beta HCG; milder elevations occur in
cine. 7 http://emedicine.medscape.com/ patients with germinoma.
article/2045059-overview (7 http:// Embryonal carcinoma is a non ger-
emedicine.medscape.com/article/2045059- minomatous germ cell tumor subtype
overview). Updated January 14 2014. which is associated with elevations in
Accessed November 23 2015. both AFP and beta HCG.
55 Daras M, Kaley TJ. Benign brain tumors
and tumors associated with phakomatoses. ?? 35. This is a histopathologic specimen
Continuum: Lifelong Learning in Neurol. form an intraventricular mass biop-
2015;21(2):373–96. sied form a 5-year-old.  
Which of the following is incorrect
?? 34. You are evaluating a 6-year-old girl regarding this lesion?
for her unsteady gait. On exam, she Most common location is in the lat-
is noted to have eyelid retraction and eral ventricles
voluntary up-gaze restriction. An MRI A. CSF production may be increase
is performed and an enhancing pineal B. It may present with hydrocepha-
region mass is noted. Serum and CSF lus and features of increased intra-
alpha fetoprotein (AFP) levels are cranial pressure
normal. Serum beta HCG is slightly C. It is a WHO grade II tumor
elevated at a concentration of 20 D. It is more common in children
international units per liter.  
What is the most likely diagnosis? vv Correct Answer is: C
The slide shows a proliferation of epi-
19 A. Yolk sac tumor
B. Mature teratoma thelial cells in a papillary pattern lining
C. Germinoma fibro-­vascular cores, consistent with
D. Choriocarcinoma choroid plexus papilloma, which is a
E. Embryonal carcinoma WHO grade I tumor. Choroid plexus pap-
Neuro-Oncology
583 19

..      Fig. 19.20 Micrograph of a choroid plexus papilloma. H&E stain. (Own work. Nephron. Wikipedia)

illoma is a childhood tumor, the most and considered low grade or WHO grade
common location is the lateral ventri- I. Choroid plexus carcinomas are higher
cles, followed by the fourth ventricle grade (WHO grade Ill), and after surgical
and then the third ventricle. They are resection, radiotherapy and chemother-
either asymptomatic, or present with apy may be needed.
hydrocephalus and manifestations of
increased intracranial pressure, some- zz Suggested Reading
times due to a combination of 55 Prayson RA. Goldblum JR.
obstructed flow and increased CSF pro- Neuropathology. 1st ed. Philadelphia:
duction. Choroid plexus papilloma is cir- Elsevier; 2005.
cumscribed mass composed of 55 Ropper AH, Samuels MA. Adams and
epithelium lining fibro-vascular cores, Victor’s principles of neurology. 9th ed.
resembling normal choroid plexus. In New York: McGraw-Hill; 2009.
­contrast to papilloma, choroid plexus 55 Comprehensive Review in Clinical
carcinomas are invasive, with nuclear Neurology: A Multiple-Choice Question
pleomorphism, demonstrating atypia, Book for the Wards and Boards © 2011
mitosis, and necrosis. Choroid plexus Wolters Kluwer Health Lippincott
papilloma can be surgically resected Williams & Wilkins. All rights reserved.
585 20

Neuro-Ophthalmology

© Springer International Publishing AG, part of Springer Nature 2018


Y. M. Awaad, Absolute Pediatric Neurology, https://doi.org/10.1007/978-3-319-78801-2_20
586 Chapter 20 · Neuro-Ophthalmology

?? 1. A 16-year-old boy is complaining of cigarettes, but does not drink alcohol


increasingly blurry vision for the past or take recreational drugs. He is a high
8 weeks. His right eye was affected school student and is currently taking
first, but was soon bilateral. He has no AP English and AP math. On exami-
past medical history of eye or vision nation, his vision is 20/400 OD and
difficulty before. Family history is pos- 20/200 OS, normal reactive pupils,
itive for a grandfather with macular extraocular motility, and intraocular
degeneration, both paternal grand- pressure. Funduscopic examination is
parents with cataracts, and an uncle shown below:
who has had “poor vision” for most You ordered visual field testing. His
of his life. He occasionally smokes visual field testing is below:

..      Fig. 20.1 Goldmann visual field record showing a ..      Fig. 20.2 Goldmann visual field record showing a
ceentrocaecal scotoma, i.e. a central scotoma involving macular or central scotoma. (Own work. RobertB3009)
the blind spot. (Own work. RobertB3009)

You have never seen a central scotoma The first symptoms of LHON are
quite as large as his. usually blurring and clouding of vision.
His likely diagnosis is: These vision problems can start unilat-
A. Leber’s hereditary optic neuropathy eral or simultaneously in both eyes; if
B. Methanol toxicity vision loss is unilateral, the other eye is
C. Pseudo tumor cerebri usually affected within several weeks or
D. Nicotine-induced visual loss months. Over time, vision in both eyes
E. Neuromyelitis optica worsens with a severe loss of acuity and
color vision. It affects mainly central
vv Correct Answer is: A vision and this is reflected in the very
Leber hereditary optic neuropathy (LHON) large central scotomata seen on the
is an inherited form of vision loss. Usually visual field testing. Vision loss results
it begins in the teens or twenties, but it from the death of cells in the optic nerve,
can appear in early childhood or later in showing as an irregularly shrunken and
adulthood in rare cases. Males are more pale optic nerve head on the funduscopic
20 affected than females. exam.
Neuro-Ophthalmology
587 20
zz Suggested Reading B. OPG associated with NF1 are less
55 Finsterer J. Inherited mitochondrial aggressive
neuropathies. J Neurol Sci. 2011; C. NF-1 associated OPGs rarely
304(1–2):9–16. invade the optic radiations
55 Yen MY, Wang AG, Wei YH. Leber’s D. Bilateral OPG are seen more fre-
hereditary optic neuropathy: a multifacto- quently in the setting of NF1
rial disease. Prog Retin Eye Res.
2006;25(4):381–96. vv Correct Answer is: A
Optic Pathway Gliomas (OPGs) are low-­
?? 2. Concerning ophthalmoplegic grade pilocytic astrocytoma. One third
migraine: of children having an OPG have been
A. The paresis of the oculomotor affected by NF1, with an OPG prevalence
nerve may develop up to 4 days of up to one quarter in the NF population.
after the headache begins In NF1 tumors are less aggressive and
B. The headache can last up to a rarely invade the optic radiations. Bilateral
week OPGs are common with NF1 than with
C. Has been reclassified by the sporadic occurrence. Visual acuity assess-
International Classification of ment is a useful way to identify patients
Headache Disorders II as a cranial with OPGs in either setting.
neuralgia and not a subset of
migraine zz Suggested Reading
D. All of the above 55 Ophthalmological assessment of children
with neurofibromatosis type 1. Catherine
vv Correct Answer is: D Cassiman & Eric Legius & Werner Spileers
The new reclassification of the ophthal- & Ingele Casteels. Eur J Pediatr.
mologic migraine is a cranial neuralgia 2013;172:1327–33. doi 7 https://doi.
and not a complicated migraine. The third org/10.1007/s00431-013-2035-2.
nerve palsy is present and the headache 55 National Institutes of Health Consensus
typically lasts at least a week. Typically the Development Conference Statement:
ophthalmoplegia develops 3–5 days after neurofibromatosis. Bethesda, July 13–15,
development of the headache. Brain MRI 1987. Neurofibromatosis. 1988;1(3):172–8.
can show gadolinium enhancement of the
third nerve. ?? 4. You are evaluating a 5-week-old baby
boy who was admitted for jaundice
zz Suggested Reading and phototherapy. The nurse noticed
55 Roy et al. Childhood steroid responsive dysconjugate gaze. His physical exam
ophthalmoplegic migraine. J Pediatr is normal except he is jaundiced. He
Neurosci. 2011;6(1):69–71. has no trouble feeding, but appears
sluggish. The child displays abnormal
?? 3. Which statement below is incorrect ocular alignment with transitory shifts
regarding the differences between of alignment from convergence to
Optic Pathway Gliomas (OPGs) accom- divergence. His neurological exam is
panied with neurofibromatosis type 1 unremarkable. What do you tell his
(NF1) as compared to sporadic? family?
A. Test of visual acuity is not useful in A. Consult a pediatric ophthalmolo-
OPG screening in NF 1 gist to rule out retinoblastoma
588 Chapter 20 · Neuro-Ophthalmology

B. Non- paralytic strabismus second- nerve. Lesion in the region of the red nucleus
ary to kernicterus may present with signs of third nerve palsy,
C. The child will require corrective not fourth nerve (trochlear) palsy.
surgery before 6 months of age Third nerve palsy presents with
D. This is normal exotropia because of medial rectus
impairment. The third nerve also inner-
vv Correct answer is: D vates the superior rectus, inferior rectus
Poor ocular coordination is common in and inferior oblique.
newborns and infants. By 3–4 weeks of age, Retinal disease does not typically
Ocular alignment occurs and consistent present with ocular misalignment.
alignment by 3 months. However, is not
uncommon for it to take as long as 5 months. zz Suggested Reading
Severe brain injury can cause signs of poor 55 Yang HK, Kim H, Hwang JM. Congenital
feeding, weak or high-pitched cry, lethargy, superior oblique palsy and trochlear nerve
hypotonia and opisthotonic posturing. absence: a clinical and radiological study.
At 5 weeks of age it is not possible to Ophthalmology. 2012;119(1):170–77.
determine treatment or even if treatment
will be required in the future. ?? 6. A 7-year-old girl with moderate to
severe developmental delays was
zz Suggested Reading referred to you for a new onset of
55 Wan MJ, Vanderveen DK. Eye disorders in “problems moving her eye”. On exami-
newborn infants (excluding retinopathy of nation, vision was reduced in the left
prematurity). Archives of Disease in eye with limited mobility in all direc-
Childhood-Fetal and Neonatal Edition, tions. The left pupil is sluggish with a
fetal neonatal-2014. 2014. white component. No abnormalities
were noted in the right eye.
?? 5. A 9-week old infant has a persistent left What is your next step in management?
head tilt and misalignment of the eyes. A. Contact pediatric oncologist for
Where is the location of his neuro- immediate referral
logical deficit? B. Contact ophthalmologist for
A. Retina immediate referral
B. Left oculomotor nerve C. MRI with contrast of the anterior
C. Medial to the red nucleus in the visual pathways
midbrain D. Send her back to her primary care
D. Right trochlear nerve physician

vv Correct answer is: D vv Correct answer is: B


Local fourth cranial nerve trauma at birth Retinoblastoma is the only malignant intra-
can cause congenital superior oblique ocular tumor of childhood. Immediate refer-
palsy, but many cases are idiopathic. ral to an ophthalmologist is crucial because
The infant tilts the head away from the surgical removal is the first treatment step.
affected side in attempt to maintain the A neurological consultation is required
eyes in alignment. The significant ocu- when visual tumor symptomology is
lar manifestation is hypertropia of the accompanied with neurological syndromes,
affected eye in the field of the impaired such as Sturge-Weber, neurofibromatosis
superior oblique muscle. and 13q deletion syndrome. Leukocoria
20 The fibers of the oculomotor nucleus is the most recognized initial feature in
pass medially to the red nucleus before retinoblastoma. The pupil appears white in
exiting the midbrain as the third cranial sunlight or flash photography.
Neuro-Ophthalmology
589 20
Funduscopic examination will estab- Kearns-Sayre syndrome is a mitochon-
lish the diagnosis. High-resolution MRI is drial disorder, it is a triad of symptoms
important for pre-surgical assessment. CT including: progressive external ophthal-
is no longer recommended. Surgery moplegia, onset before age 20 and one of
followed by chemotherapy and radiation the following; short stature, pigmentary
therapy is the treatment of choice. retinopathy, cerebellar ataxia, heart block,
cerebellar ataxia, and elevated cerebrospi-
zz Suggested Reading nal protein.
55 Abramson DH, Frank CM, Susman M,
Whalen MP, Dunkel lJ, Boyd zz Suggested Reading
NW. Presenting signs of retinoblastoma. 55 Weleber RG, Francis PJ, Trzupek KM,
J Pediat. 1998;132(3):505–8. Beattie C.Leber congenital amaurosis. 2013.

?? 7. A 10-month-old boy was referred by ?? 8. 16-year-old female came to your office


his pediatrician because he recently with history of right upper and lower
stopped visual tracking and a nystag- facial weakness for 1 month. Taste aber-
mus was noted. On examination, he ration and right hyperacusis are also
has sluggish pupillary response and reported. A blink reflex is performed
the rest of his neurological exam is with the following results: stimulating
within normal limits. the right side: absent right R1 and R2
What is your diagnosis? potentials, and normal left R2; stimulat-
A. Retinoblastoma ing the left side: normal left R1 and R2
B. Craniopharyngioma potentials, and absent right R2.
C. Kearns-Sayre syndrome Where is the site of the lesion?
D. Leber Congenital amaurosis A. left facial
B. left trigeminal
vv Correct answer is: D C. right facial
Leber congenital amaurosis is a retinal D. right medullary
dystrophy and is the most common genetic E. right trigeminal
cause of congenital visual impairment. The
impairment starts in the first 3–4 months vv Correct Answer is: C
of life followed by nystagmus and sluggish This is a right facial nerve lesion.
pupillary responses. Visual acuity will rarely Stimulating the affected side results in
be better than 20/400 when they get older. absence of ipsilateral R1 and R2 poten-
Mid-facial hypoplasia and mental retarda- tials, but a normal contralateral R2 poten-
tion can be associated in some children. tial; whereas stimulating the unaffected
Retinoblastoma manifestation are side results in a normal ipsilateral R1 and
unilateral progressive visual loss in a R2, but absent contralateral R2 suggesting
young child and strabismus. The first most a unilateral facial lesion.
common sign is leucocoria, in which the
white tumor reflects light and blocks view zz Suggested Reading
of the red colored retina. 55 Misulis KE, Head TC. Essentials of clinical
Craniopharygioma is the most common neurophysiology. 3rd ed. Philadelphia:
non-glial tumor in children and usually Butterworth-­Heinemann; 2003.
occurs between the ages of 6–14. There is a 55 Preston DC, Shapiro BE.
1–2-year history of progressive visual loss, Electromyography and neuromuscular
delayed sexual maturation growth failure disorders. 1st ed. Philadelphia:
and bitemporal hemianopia. Butterworth-Heinemann; 1998.
591 21

­Neurophysiology/
Localization

© Springer International Publishing AG, part of Springer Nature 2018


Y. M. Awaad, Absolute Pediatric Neurology, https://doi.org/10.1007/978-3-319-78801-2_21
592 Chapter 21 · Neurophysiology/Localization

?? 1. What test can be used in your office to suddenly moving that causes him to
21 determine if a patient has conductive suddenly lurch to one side.  
verses a sensorineural hearing loss? Which symptom is indicative of a
A. Rinne test central etiology?
B. Weber test A. Hearing loss
C. Ask her to repeat words that test B. Tinnitus
high, medium and low frequency C. Ataxia with eye closes
sounds D. Hoarseness
D. Tympanometry
vv Correct Answer is: D
vv Correct Answer is: A When other cranial nerve affection is asso-
The Rinne test: activating a tuning fork ciated with vertigo, this indicates central
and holding the base against the mas- pathology. Hoarseness is caused by the
toid process to determine if bone con- ten-­cranial nerve injury. Etiologies of cen-
duction is perceived, then when the tral vertigo include migraine, multiple
patient reports the sound has faded the sclerosis, seizure, stroke/TIA and tumor.
tuning fork is held 1 inch for the ipsilat- Hearing loss is a peripheral eighth
eral ear. Since air conduction is more cranial nerve affection and not central.
sensitive than the bone conduction, the Causes of peripheral vertigo include;
vibratory sound should still be heard benign positional vertigo, vestibu-
twice as long as the bone conduction. lar neuronitis, labyrinthitis, Meiere’s
Impaired air conduction response with disease and vestibular nerve tumor
normal bone conduction indicates a (schwannoma or meningioma).
conductive hearing loss. Tinnitus is a peripheral eighth cra-
The Weber test: compares bone con- nial nerve impairment and not central.
duction in the two ears. The tuning fork Maintaining balance without visual
is activated and placed at the center of input, but normal with eyes open, is
the forehead and the patient is asked if characteristic of a disorder of peripheral
one side is louder than the other. sensory input. Ataxic gait is a sign of
Informal testing of hearing fre- central pathology.
quencies is helpful to determine if
there is a Sensorineural hearing defi- zz Suggested Reading
cit. Tympanometry with pneumatic 55 Thompson T, Amedee R. Vertigo: a review
testing determines if there is abnor- of common peripheral and central
malities of the external ear and normal vestibular disorders. Ochsner J.
movement of the tympanic memory, 2009;9(1):20–6.
and will only determine if immobile
tympanic membrane is contributing to
hearing loss. ?? 3. Which of the following tests would be
most helpful What test will be help-
zz Suggested Reading ful in diagnosing a 6-year-­old male
55 Nunez DA, Qi L. Tests for hearing. Logan presenting with frequent starring
Turner’s Diseases of the Nose, Throat and episode that can be provoked in the
Ear: Head and Neck Surgery. 2015;375. office by having him hyperventilate?
A. MRI of brain
?? 2. A 9-year-old boy is complaining for B. EEG
the last 5 weeks of vertigo symptoms, C. Evoked Potential
including the sensation of the floor D. Positron Emission Tomography
Neurophysiology/Localization
593 21
vv Correct Answer is: B movements (Huntington’s disease)
The diagnostic EEG pattern for child- or delay in initiating movements
hood absence epilepsy is a 3 per-second (Parkinson’s disease).
spike­ and-wave. Cerebellum pathology leads in impair-
No abnormal brain structure on ment in coordination of motor movements
neuroimaging has been recognized in and manifest as ataxia or dysarthria.
children with generalized epilepsies, Frontal lobe lesion has different
including absence. Generalized epilepsy manifestations based on the affected
is considered ion channelopathy. location. Lesions in the motor planning
Evoked potential studies measure and supplementary motor areas of the
the response time (and strength) of a frontal cortex may lead to a deficit in
stimulation of a sensory nerve pathway executing motor actions and control
to reach the brain. This test detects the over voluntary actions.
slowing of electrical conduction most
commonly caused by demyelination zz Suggested Reading
along sensory pathways. 55 Hirano M, Hatzoglou V, Karimi S, Young
PET scans measure local cerebral RJ. Hypertrophic olivary degeneration
glucose metabolism and provides a rep- resulting from posterior fossa masses and
resentative image. This test can be help- their treatments. Clin Imaging.
ful in identifying an epileptic focus in 2015;39(5):787–90.
an individual with focal onset seizures.
Cerebral tissue that has epileptic poten- ?? 5. What is the EEG finding in a 10-year-
tial is often metabolically dysfunctional, old boy that presented with acute
and when inactive, is hypometabolic. onset of fever, headache, lethargy with
subsequent development of hemipa-
zz Suggested Reading resis and has focal seizures, and has
55 Camfield P, Camfield C. Epileptic syn- multiple cold sores on his skin?
dromes in childhood: clinical features, A. General Epileptiform discharges
outcomes, and treatment. Epilepsia. B. Normal EEG
2002;43(Suppl 3):27–32. C. per second spike and wave forma-
tions
?? 4. A 12-year-old female is complaining D. Periodic lateralizing epileptiform
of bilateral rhythmic contractions of discharges
her palate.  
Where do you locate the lesion? vv Correct Answer is: D
A. Central tegmental tract Herpes simplex virus (HSV) is the single
B. Basal ganglia most common cause of non-epidemic
C. Cerebellum cases of encephalitis. A presumptive
D. Frontal lobe diagnosis of HSV encephalitis can be
based on the ­presence of periodic later-
vv Correct Answer is: A alizing epileptiform discharges. MRI
Palatal myoclonus is a segmental myoc- Classically, there is focal abnormalities
lonus. The pathology is almost exclu- in the temporal and inferior frontal
sively located in brainstem lesions of the lobes on the MRI. Only about 20% have
central tegmental tract or the dentate- a history of labial infections.
olivary pathways. Brainstem tumor and About 80% manifest focal neurologi-
stroke are the most common etiologies. cal symptoms that include hemiparesis,
Basal ganglia pathology leads to cranial nerve deficits, visual field loss,
failure in suppressing the unwanted aphasia and focal seizures.
594 Chapter 21 · Neurophysiology/Localization

Primary generalized epilepsies have vv Correct Answer is: D


21 generalized EEG patterns and not in Hirschsprung’s disease is due to con-
encephalitis. genital absence of the myenteric
Essentially all persons with encepha- plexus. The enteric nervous system
litis will have focal EEG findings. consists of the myenteric (or Auer-
High voltage 3 per second spike bach’s) plexus that is located between
and wave complexes are diagnostic for the outer and inner smooth muscle lay-
absence epilepsy. ers of the gastrointestinal tract and the
sub-mucous (or Meissner’s) plexus,
zz Suggested Reading which is located between the circular
55 Cappellari AM, Tardini G, Bona AR, Belli muscle layer and the mucosa. The
M, Triulzi F, Fossali EF. Teaching myenteric plexus is predominantly
Neurolmages: infantile herpes simplex involved in gut motility, whereas the
encephalitis. Neurology. 2014;83(7):e85–6. sub-mucous plexus is involved in secre-
tory functions. Together, the myenteric
?? 6. A 2-week-old infant is seen by his and sub-mucous plexus control the
pediatrician because of distended function of the gastrointestinal tract.
abdomen. By mother’s report, he has Hirschsprung’s disease is due to mal-
had a few bowel movements, but development of the myenteric plexus.
not having the normal bowel move- Most commonly, focal congenital
ments that his siblings has at his age. absence of the myenteric plexus in the
On examination, he has a distended internal anal sphincter or recto sig-
abdomen with hypoactive bowel moid junction occurs, though in rare
sounds. X-ray of the abdomen shows cases, the myenteric plexus may be
dilated loops of bowel with exces- absent throughout the gastrointestinal
sive fecal material. A diagnosis of system. Segments of the colon that
Hirschsprung’s disease is suspected lack myenteric plexus cannot relax,
after additional testing.   leading to fecal retention and disten-
All of the following is correct expect? tion of proximal colonic segments.
A. It consists of the myenteric plexus, Some cases of Hirschsprung’s disease
located between the outer and inner are due to mutations in the RET proto-
smooth muscle layers of the gastro- oncogene.
intestinal tract, and the sub mucous
plexus, located between the circular zz Suggested Reading
muscle layer and the mucosa 55 Comprehensive Review in Clinical
B. The myenteric plexus is predomi- Neurology: A Multiple-Choice Question
nantly involved in gut motility Book for the Wards and Boards © 2011
C. The sub-mucous plexus is pre- Wolters Kluwer Health Lippincott
dominantly involved in secretory Williams & Wilkins. All rights reserved.
functions of the gastrointestinal
tract ?? 7. Which one of the following is derived
D. Hirschsprung’s disease is due to from the neural crest cells?
congenital absence of the submu- A. Cerebellum
cous plexus B. Meninges
E. Hirschsprung’s disease is often C. Autonomic ganglia
focal, though rare cases involve D. Olfactory bulb
the entire colon E. Optic nerve
Neurophysiology/Localization
595 21
vv Correct Answer is: C D. Photic stimulation
The peripheral nervous system including E. Recording during sleep
autonomic ganglia, chromaffin cells in
adrenal medulla and melanocytes are vv Correct Answer is: C
derived from neural crest cells, which are EEG represents the recording along the
derived from neural tube after fusion. scalp of electrical l activity that is pro-
duced by the firing of pyramidal neu-
zz Suggested Reading rons within the cerebral cortex. EEG
55 Mowzoon N, Flemming KD. Neurology recording is based on differential ampli-
board review: an illustrated study guide. fication, in which the difference in volt-
Rochester: Mayo Clinic Scientific; 2007. age between two sites is compared with
Print. each recording channel. This recording
helps in the evaluation and diagnosis of
?? 8. Which stage of sleep is characterized abnormalities in brain electrical activity,
by sleep spindles and K complexes? but is also useful in the evaluation of
A. Wakefulness other neurologic conditions. Patients
B. Stage 1 undergoing an evaluation for “spells”
C. Stage 2 will often have an EEG to determine the
D. Stage3 presence or absence of epileptogenic
activity. During a routine recording, var-
vv Correct Answer is: C ious procedures are utilized to enhance
Wakefulness/Drowsiness produces pri- potentially abnormal electrical activity.
marily alpha waves. Stage 1 is character- These so-called activation procedures
ized by low amplitude waves with sharp include hyperventilation, photic stimu-
waves over the vertex. There are slow lation, sleep deprivation prior to the
eye movements. EEG, and recording during sleep. Nox-
Stage 2 is noted by the presence of K ious stimulation is not part of the EEG
complexes and sleep spindles. activation procedures. Hyperventilation
Stage 3 is characterized by slow may produce generalized slowing or no
waves with large amplitudes (delta effect in normal subjects; however, in
waves). REM sleep is noted by low EMG certain epilepsies, such as absence epi-
tone, rapid eye movements, and saw lepsy, hyperventilation may activate
tooth waves. epileptiform discharges and even sei-
zures.
zz Suggested Reading Photic stimulation is performed
55 Rundell 0H, Jones, RK. Polysomnography with various frequencies, and may be
methods and interpretations. Otolaryngol helpful to induce epileptiform activity
Clin North Am. 1990;23(4):583–92. and seizures, most commonly myo-
55 De Gennaro L, Ferrara M. Sleep spindles: clonic seizures, in individuals with
an overview. Sleep Med Rev. photosensitive epilepsies. Sleep depri-
2003;7(5):423–40. vation may also enhance epileptogenic
activity. Furthermore, seizures may
?? 9. All of the following are an activation occur predominantly (or exclusively)
procedure used during EEG recording during sleep in certain epilepsy syn-
except? dromes, and EEG abnormalities may not
A. Hyperventilation be seen in the awake patient. Therefore,
B. Sleep deprivation prior to the EEG an EEG may be incomplete without a
C. Noxious stimulation sleep recording.
596 Chapter 21 · Neurophysiology/Localization

zz Suggested Reading tics can represent neurologically benign


21 55 Tatum WO, Husain AM, Benbadis SR, behavior in some children who use
et al. Handbook of EEG Interpretation, these behaviors to relieve tension for
Demos Medical Publishing, LLC; 2008. self-soothing. These behaviors can also
55 Comprehensive Review in Clinical be seen in other disorders such as
Neurology: A Multiple-Choice Question autism and OCD. In this case, since these
Book for the Wards and Boards ©20 11 behaviors seem to happen more ran-
Wolters Kluwer Health Lippincott domly, and the child is non-responsive
Williams & Wilkins. All rights reserved. during the events, your suspicion would
be raised to rule out a seizure disorder.
?? 10. A healthy 3-year-old female has Unfortunately, in many cases of com-
been evaluated because her “strange plex partial seizures, the exam, blood
behavior”. The mother reports that work, and even EEG are normal. A sleep-
her daughter sometimes rocks back deprived EEG is more sensitive and
and forth and makes strange faces might be a good next step before refer-
for periods of time ranging from ring the child to a neurologist or refer-
a few seconds to minutes. During ring her for video EEG.
those spells, she completely ignores
everyone. You reviewed a video of ?? 11. Which of the following definitions of
an event, which shows while she is neural tube defects is incorrect?
sitting on the couch rocking forward A. Meningocele-isolated meningeal
and back she is making repetitive protrusion
smiling motions with her mouth for B. Myelomeningocele-­protrusion of
approximately 40 s. There are no par- spinal cord and meninges
ticular events that precede the rock- C. Diastematomyelia-­splitting of the
ing and face behaviors, she seems spinal cord into two portions by a
to do them somewhat randomly at midline septum
various times of day. Her general D. Diplomyelia-­duplication of the
and neurological examinations are spinal cord
normal. Her labs including, serum E. Sacral agenesis-­isolated absence
electrolytes, calcium, and an EEG are of the sacral spinal cord
normal.  
What is your next test to order? vv Correct Answer is: E
A. Sleep-deprived EEG Sacral agenesis is absence of the
B. Reassure the mother this is normal sacrum, rather than absence of the
stimulating/soothing behavior sacral spinal cord, and is frequently
C. Obtain lumbar puncture associated with other malformations.
D. CT scan head The neural tube defects (NTDs) include
E. Brain MRI meningocele, myelomeningocele, dia-
stematomyelia, diplomyelia, and sacral
vv Correct Answer is: A agenesis. Caudal NTDs result from fail-
The clinical presentation could be diffi- ure of fusion of the posterior neuropore
culty to differentiate between repetitive at day 26 of gestation. Meningocele is
soothing/stimulating behaviors and isolated protrusion of the meninges
complex partial seizures. Rocking, head into a bony defect within the vertebral
pounding, nail biting, thumb sucking, column. This protrusion is typically cov-
masturbating, hair pulling, and facial ered by skin. It is not usually associated
Neurophysiology/Localization
597 21
with neurologic deficits, and if neuro- Wolters Kluwer Health Lippincott
logic deficit is found, myelomeningo- Williams & Wilkins. All rights reserved.
cele should be suspected.
Myelomeningocele (also known as spi- ?? 12. Which of the following EEG patterns is
nal dysraphism or rachischisis) is protru- associated with epilepsy?
sion of potentially all layers of A. Temporal rhythmic delta activity
intraspinal contents through a bony B. Stimulus-induced, rhythmic, peri-
defect: spinal cord, nerve roots, and odic ictal discharges (SIRPIDs)
meninges. Either the spinal cord may be C. Wicket spikes
exposed or a thin membrane may cover D. Burst suppression
the protrusion. They most often occur in E. Triphasic waves
the lumbosacral region but can occur at
any level. This is a clinically severe NTD vv Correct Answer is: A
associated with hydrocephalus, motor Temporal rhythmic delta activity
and sensory abnormalities of the legs, (TIRDA) is usually associated with tem-
and bowel/bladder dysfunction. Myelo- poral lobe epilepsy.
meningocele occurs in association with Wicket spikes are usually a normal
Chiari II malformations. Diastematomy- finding and not associated with epi-
elia is splitting of the spinal cord into lepsy. SIRPIDs are usually seen in criti-
two portions by a midline septum. cally ill patients, even those without
Diplomyelia is duplication of the spinal neurological problems.
cord and is distinguished from diaste- Triphasic waves are commonly seen
matomyelia by the presence of two cen- with hepatic or renal dysfunction or
tral canals each surrounded by gray and encephalopathy.
white matter as in a normal spinal cord.
Sacral agenesis, or absence of the whole ?? 13. Which one of the following neurologi-
(or in some cases parts of) the sacrum, cal abnormalities is caused by failure
classically occurs in association with a of closure of posterior neuropore?
variety of other urogenital, gastrointes- A. Anencephaly
tinal, and spinal cord abnormalities. It B. Spina bifida
has been associated with maternal insu- C. Lissencephaly
lin-dependent diabetes. Autosomal D. Dandy-Walker syndrome
dominant forms associated with E. Holoprosencephaly
homeobox gene mutations have been
identified. Clinical manifestations range vv Correct Answer is: B
from mild motor deficits to severe sen- Neural tube closure occurs around 4th
sory and motor deficits and bowel and week of gestation, anterior neuropore
bladder dysfunction. Management of closes at day 25; failure leads to enceph-
NTDs involves surgical approaches and alocele and anencephaly. Posterior neu-
management of complications includ- ropore closes at day 27; failure leads to
ing hydrocephalus and bowel and blad- spina bifida or myelomeningocele.
der dysfunction.
zz Suggested Reading
zz Suggested Reading 55 Mowzoon N, Flemming KD. Neurology
55 Comprehensive Review in Clinical board review: an illustrated study guide.
Neurology: A Multiple-Choice Question Rochester: Mayo Clinic Scientific; 2007.
Book for the Wards and Boards © 2011 Print.
598 Chapter 21 · Neurophysiology/Localization

21

..      Fig. 21.1 EEG of brain and heart action on closing the eyes. (Own work. Otoomuch. Wikipedia)

?? 14. Which of the following is correct The posterior background is the first
regarding the EEG in the figure below? feature usually analyzed when reading
A. There is poor reactivity of the pos- an EEG. There are no spikes or other epi-
terior background. leptiform discharges seen in the figure
B. The patient has an occipital seizure. and therefore no evidence of an occipital
C. The posterior background shows seizure. The posterior background has
Li-frequencies. Alpha-frequencies (in this case around
D. The patient is sleeping 9 Hz) and not Li-­frequencies. This patient
E. An eye closure is recorded during has good reactivity of the background
this EEG page rhythm, suggesting an awake patient.
There are no sleep structures visualized.
vv Correct Answer is: E
There is a normal reactivity of the poste- zz Suggested Reading
rior background after an eye closure. 55 Tatum WO, Husain AM, Benbadis SR,
Alpha rhythm (8 and 13 Hz), is a normal et al. Handbook of EEG Interpretation.
finding seen in the posterior head
regions in normal people when they are ?? 15. A 12-year-old boy came to the office
awake, more prominent with eye clo- after he had a generalized seizure
sure, and attenuating with eye opening. preceded by left side eye and head
Failure of attenuation with eye opening deviation.  
and reactivity with eye closure may be a Where do you locate the seizure focus?
sign of abnormality. A. Hypothalamus
By 3 years of age, alpha rhythm of B. Right frontal lobe
8 Hz is seen in normal children, and it C. Left frontal lobe
increases with age. A voltage asymme- D. Right temporal lobe
try of >50% between sides is abnormal. E. Left temporal lobe
Neurophysiology/Localization
599 21
vv Correct Answer is: B right arm extension at the elbow with
Lateralization signs and symptoms dur- the fist clenched and left arm flexion
ing epileptic seizures are helpful to locate at the elbow. Then he had a general-
the seizure focus side, and this will help ized tonic-clonic (GTC) seizures.  
to classify the epilepsy correctly and also Where is the seizure focus location?
help in the pre-surgical evaluation of A. Right temporal lobe
drug resistant patients. Auras are a focal B. Left temporal lobe
discharge, which activate the area C. Right supplementary motor area
responsible for generation of the D. Left supplementary motor area
patient’s aura at the beginning of the E. Right occipital lobe
clinical (but not necessarily the electro-
graphic) seizure. Thus, auras are good vv Correct Answer is: D
clue for the seizure origin. Some seizures There is asymmetric tonic posturing
do not start with an aura, but other semi- during a seizure, which is characteristic
ological signs may point toward the ori- of seizures arising from the supplemen-
gin of these seizures. Eyes and head tary motor area (SMA). This specific case
version, ­characterized strictly by a forced describes the “figure of 4 sign,” in which
and involuntary movement leading to an the extended arm is contralateral to the
unnatural position of the head toward seizure focus. Therefore, this patient’s
one side, are associated with a seizure seizures originate in the left SMA.
focus in the contralateral hemisphere, Addendum: “the figure of 4” sign is
and more specifically in the frontal region an arm flexed at the elbow and a contra-
in the frontal eye fields and motor areas lateral arm that is extended - the ictal
anterior to the precentral gyrus. The asso- focus is ipsilateral to the flexed arm and
ciation of version with the contralateral contralateral to the extended I dystonic
hemisphere is more robust if the version arm.
occurs immediately prior to the second-
arily generalized tonic-clonic phase. In zz Suggested Reading
this case, it is clear that the seizure is 55 Harvey AS, Freeman JL. Epilepsy in
coming from the right frontal region. hypothalamic hamartoma: clinical and
EEG features. Semin. Pediatr Neurol.
zz Suggested Reading 2007;14(2):60–4.
55 Harvey AS, Freeman J L. Epilepsy in 55 Loddenkemper T, Kotagal P. Lateralizing
hypothalamic hamartoma: clinical and signs during seizures in focal epilepsy.
EEG features. Semin Pediatr Neural. Epilepsy Behav. 2005;7:1–17.
2007;14(2):60–4. 55 Comprehensive Review in Clinical
55 Loddenkemper T, Kotagal P. Lateralizing Neurology: A Multiple-Choice Question
signs during seizures in focal epilepsy. Book for the Wards and Boards ©2011
Epilepsy Behav. 2005;7:1–17. Wolters Kluwer Health Lippincott
55 Comprehensive Review in Clinical Williams & Wilkins. All rights reserved.
Neurology: a Multiple-Choice Question
Book for the Wards and Boards ©2011 ?? 17. Which one of the following is true
Wolters Kluwer Health Lippincott regarding myelination of the central
Williams & Wilkins. All rights reserved. nervous system?
A. Myelination of axons is done by
?? 16. The father of a 16-year-old male Schwann cells
descript his son seizure that begin B. Myelination starts at 6 weeks of
with asymmetric tonic posturing with gestation
600 Chapter 21 · Neurophysiology/Localization

C. Extensor plantar response in by 4–6 months of age. When the spine is


21 infants is an abnormal finding stroked or tapped with the infant lying
D. Bulk of myelination is done in the in a prone position, the hips move
3rd trimester toward the touch.
E. All cranial nerves myelinate The tonic neck reflex, or “fencing
prenatally posture,” is present at birth, and disap-
pears by 5–6 months of age. When the
vv Correct Answer is: D baby’s head is turned to the side, the
Myelination in central nervous system is ipsilateral arm and leg extend and the
done by oligodendrocytes and occurs as contra lateral arm and leg flex.
early as 14 weeks of gestation and con- The Landau reflex does not typi-
tinues to adulthood. Bulk of active cally appear until 5–10 months of age,
myelination begins at third trimester and may persist until 24 months. It is a
and continues to about 2 years postna- postural reflex elicited by holding the
tally. Corticospinal tract myelinates baby in a prone position. When moving
throughout first 2 years of life: extensor the baby’s head downward (flexion), the
plantar response is present at birth and legs will follow, moving downward by
disappears sometime during this devel- flexion at the hip.
opment. Poor myelination of cortical Crossed adduction appears at birth,
pathways is responsible for lack of corti- and resolves by approximately 7 months
cal control over motor function at birth of age. When testing a patellar reflex,
and is responsible for primitive reflexes there is adduction of the contralateral
present at birth, such as grasp reflex. leg.
Optic nerve myelinates postnatally; The Moro reflex appears at birth,
most other cranial nerves myelinate pre- and disappears at 5–6 months of age.
natally. Certain subcortical association With a sudden stimulus, both arms will
fibers (especially of frontal cortex) do extend.
not complete myelination until early
adulthood. ?? 19. Regarding the pharmacokinetics of
phenytoin.which of the following is
zz Suggested Reading correct?
55 Mowzoon N, Flemming KD. Neurology A. It is predominantly renally metab-
board review: an illustrated study guide. olized
Rochester: Mayo Clinic Scientific; 2007. B. It is exclusively metabolized by
Print. the liver
C. It has zero-order (nonlinear)
?? 18. Which is most likely to be the last kinetics
reflex to disappear in an infant with D. It is a hepatic enzyme inhibitor
normal development? E. It has first-order (linear) kinetics
A. Galant reflex
B. Tonic neck reflex vv Correct Answer is: C
C. Landau reflex Phenytoin is used for the treatment of
D. Crossed adduction partial and/or generalized tonic­clonic
E. Moro reflex (GTC) seizures (primary or secondary).
Its primary mechanism of action is inhi-
vv Correct Answer is: C bition of voltage-dependent neuronal
Galant reflex (Lateral incurvature of the sodium channels. It undergoes predom-
trunk) appears at birth and disappears inantly liver metabolism, although there
Neurophysiology/Localization
601 21
is also minimal renal metabolism. calculating a supplementing (or loading)
Patients who are in low-­protein disease IV bolus of phenytoin is as follows: (tar-
states (such as liver failure, etc.) need to get total phenytoin level - current total
be followed with free phenytoin levels phenytoin level) x (kilogram body weight
because of less available protein for x volume of distribution). The therapeutic
binding, making the total levels unreli- range for phenytoin is 10–20 Mg/ml. The
able. It is important to understand that range for volume of distribution for phe-
phenytoin exhibits nonlinear (zero- nytoin is 0.5–1 L/kg, with an average of
order) kinetics, as the metabolic path- 0.8 L/kg often used. If we insert the num-
ways responsible for its metabolism bers from the case into the formula, the
become saturated. This means that calculation will be as follows: (15–10) ×
when the dose of phenytoin is increased (75 × 0.8) = 300 mg. An accurate reassess-
beyond a certain point, its plasma con- ment of new levels can be obtained by
centration at steady state will no longer checking free and total levels approxi-
increase in a proportionate manner, mately 2 h after the IV load.
rather small dose changes may result in
a large/toxic increment in plasma con- zz Suggested Reading
centrations. In general, phenytoin 55 Ahn JE, Cloyd JC, Brundage RC, et al.
approaches zero-order kinetics at total Phenytoin half-life and clearance during
levels >10–15 Mg/ml and small dose maintenance therapy in adults and elderly
increments can potentially cause large patients with epilepsy. Neurology.
increases in the serum level. 2008;71(1):38–43.
Idiosyncratic reactions caused by 55 Ropper AH, Samuels MA. Adams and
phenytoin include aplastic anemia, Victor’s Principles of Neurology, 9th ed.
Stevens-­Johnson syndrome, and hepatic New York: McGraw-Hill; 2009.
failure. Other side effects include throm- 55 Schachter SC. Review of the mechanisms
bocytopenia, lymphadenopathy, gingival of action of antiepileptic drugs. CNS
hyperplasia, acne, coarse facial features Drugs. 1995;4:469–77.
(also called “phenytoin facies,” from 55 Comprehensive Review in Clinical
hypertrophy of subcutaneous facial tis- Neurology: A Multiple-Choice Question.
sue), hirsutism, purple glove syndrome 55 Book for the Wards and Boards ©2011
(with intravenous administration), nys- Wolters Kluwer Health Lippincott
tagmus, ataxia, dysarthria, diplopia, nau- Williams & Wilkins. All rights reserved.
sea, dizziness, and drowsiness. Phenytoin
can also cause folate deficiency and ?? 20. Deficiency of which of the following
increased vitamin D metabolism, result- neurotransmitters has been impli-
ing in premature osteoporosis. cated in the pathogenesis of Narco-
Chronically, its use has been asso- lepsy?
ciated with a usually mild peripheral A. Noradrenaline
neuropathy and with cerebellar, but not B. Acetylcholine
cortical atrophy. Acutely, the IV form can C. Serotonin
cause phlebitis, pain, burning, hypoten- D. Histamine
sion, and cardiac conduction abnormali- E. Hypocretin
ties. Phenytoin is a liver enzyme inducer,
so it can increase metabolism of many vv Correct Answer is: E
other drugs. There are variations of cal- Hypocretin, also known as orexin, is
culating loading and correcting doses of deficient in the cerebrospinal fluid of
phenytoin. A general simple formula for most narcoleptics with cataplexy.
602 Chapter 21 · Neurophysiology/Localization

zz Suggested Reading may occur in individuals who do not


21 55 Blumenfeld H. Neuroanatomy through have known seizures. Also, in patients
clinical cases. 1st ed. Sunderland: Sinauer who do have clinical and EEG findings
Associates; 2002. characteristic of this form of epilepsy,
the EEG abnormalities may persist even
?? 21. Which of the following is most likely after the seizures have stopped. Thus,
to be true of benign focal epilepsy decisions on whether to start or stop
with central mid temporal spikes? anti-epileptic drugs are not based on
A. Lifelong treatment with anti-epi- EEG findings.
leptic medications is often neces-
sary ?? 22. Regarding embryonal nervous system
B. Seizures usually occur during the development, which of the following
daytime while the child is awake statements is correct?
C. Children with this disorder may A. Vertebral bodies arise from ecto-
complain of facial ­numbness or dermal cells of the neural plate
may be observed drooling B. Neural crest cells give rise to the
D. EEG findings are used to deter- peripheral nervous system
mine whether to continue, dis- C. Failure of fusion of the posterior
continue, or start anti-epileptic neuropore leads to disorders such
medications as anencephaly
D. Failure of fusion of the anterior
vv Correct Answer is: C neuropore leads to disorders such
Benign rolandic epilepsy (Benign focal as spina bifida
epilepsy with centrotemporal spikes), E. The notochord gives rise to chro-
accounts for 15% of childhood epilepsy. maffin cells of the adrenal medulla
It begins between 7 and 9 years of age.
Its name is referring to the typical EEG vv Correct Answer is: B
findings. Diphasic or triphasic sharp The main embryonal layer giving rise to
waves tend to occur in the mid temporal the nervous system is the ectoderm. In
and central areas. The clinical presenta- early stages of nervous system develop-
tion includes, speech arrest, hypersali- ment, a structure known as the neural
vation, unilateral facial twitching, and plate forms. The notochord, a layer of
paresthesias inside the mouth. mesodermal cells in contact with the
Treatment with anti-epileptic medica- ectoderm, induces formation of the neu-
tions is often not necessarily. Seizures ral plate from the ectoderm and later
tend to occur at night and most patients signals differentiation of various cell
have under 10 seizures total. Typically, types mediated by inductive signals. The
remission occurs within 2–4 years of notochord later gives rise to the verte-
onset of seizures. Most are in remission bral column. The neural plate forms a
by age 16. Treatment may be necessary in structure known as the neural tube
patients who have more frequent seizures, through a process called neurulation.
or in those with generalized seizures. Neurulation involves proliferation and
75% of seizures occur either during migration of ectodermal cells and invag-
the night, or when the patient wakes. ination, folding, and fusion of the neural
On EEG studies, epileptiform activity is plate in a specific pattern. An important
more prominent during N-REM sleep. step in neurulation includes the forma-
EEG activity typical of benign focal tion of a midline groove along which the
epilepsy with centrotemporal spikes lateral margins of the neural plate fold.
Neurophysiology/Localization
603 21
These lateral margins start to fuse in which occurs at 5–6 weeks of gestation,
the center, so that for a period of time, lead to disorders such as septo-­optic dys-
there are openings at each end, the plasia. As the nervous system becomes
anterior and posterior neuropore. more organized and specific areas more
Fusion then reaches the neuropores, specialized, neuronal migration begins
the anterior one first, then the poste- along specific routes, and abnormalities
rior one, and the neural tube is thus of neuronal proliferation and migration
formed. The ventral and dorsal aspects in turn lead to specific developmental
of the neural tube each give rise to abnormalities. The peripheral nervous
specific cell populations and ultimately system (including the autonomic gan-
specific parts of the nervous system. glia) forms from neural crest cells that
As mentioned, mesodermal cells of the are derived from the neural tube after it
notochord provide signals for differen- fuses. In addition to peripheral nervous
tiation to cells in the ventral aspect of system structures, neural crest cells give
the neural tube. These signals include rise to the chromaffin tissue of the adre-
sonic hedgehog protein and bone mor- nal medulla and melanocytes.
phogenetic proteins, of which several
types have been identified. Neurulation ?? 23. Which of the following anticonvul-
occurs at 3–6 weeks’ gestation, and sants binds to synaptic vesicle protein
failure of crucial processes at any stage SV2A?
leads to a variety of abnormalities, col- A. Levetiracetam
lectively known as neural tube defects. B. Topiramate
As mentioned earlier, the neural plate C. Lamotrigine
undergoes fusion to form the neural D. Ethosuximide
tube in different areas at different times, E. Perampanel
and failure to fuse at each site results
in specific defects. Abnormal rostral vv Correct Answer is: A
fusion at the anterior neuropore leads Levetiracetam binds to synaptic vesicle
to abnormalities such as encephalocele protein SV2A, which decreases calcium
or anencephaly, whereas abnormal cau- influx into the presynaptic terminal. The
dal fusion (at the posterior neuropore) drug also blocks high voltage activated
leads to disorders such as spina bifida. calcium channels and affects gamma­
Following neurulation, the neural tube aminobutyric acid (GABA) function. It is
undergoes segmentation into three unclear, however, how these actions
vesicles, in a process called specifica- relate to its anticonvulsant effect.
tion, whereby different segments begin Topiramate has multiple mechanisms
to acquire cell types and characteristics of action, blocking sodium channels,
specific to the CNSstructure that will calcium channels, kainate receptors,
eventually arise from them. The three and a-amino-3-­hydroxyl-5-methyl-4-i
segments include the prosencephalon, soxazole-proprionate (AMPA) glutamate
mesencephalon, and rhombencepha- receptors, while enhancing GABA activity.
lon. The prosencephalon subsequently Lamotrigine also has multiple mech-
forms the telencephalon, which gives anisms of action, blocking sodium chan-
rise to the cerebral hemispheres, as nels and high voltage activated calcium
well as the diencephalon, which forms channels.
the hypothalamus and thalamus. The Ethosuximide blocks thalamic T-type
rhombencephalon gives rise to the brain calcium channels, and likely inhibits
stem. Abnormalities during specification, sodium currents.
604 Chapter 21 · Neurophysiology/Localization

Perampanel is a new anticonvulsant D. The diencephalon gives rise to the


21 with a novel mechanism of action, act- midbrain
ing as a non-­competitive AMPA gluta- E. The prosencephalon gives rise
mate receptor antagonist. to the telencephalon, which ulti-
mately forms the brain stem
zz Suggested Reading
55 Stafstrom CE. Mechanisms of action of vv Correct Answer is: C
antiepileptic drugs: the search for synergy. The main embryonal layer giving rise to
Curr Opin Neurol. 23:157–63. the nervous system is the ectoderm. In
early stages of nervous system devel-
?? 24. Which of the cell types forms the opment, a structure known as the neu-
blood-brain barrier and help in neuro- ral plate forms. The notochord, a layer
nal migration during development? of mesodermal cells in contact with the
A. Astrocytes ectoderm, induces formation of the
B. Oligodendrocytes neural plate from the ectoderm and
C. Microglia later signals differentiation of various
D. Neurons cell types mediated by inductive sig-
nals. The notochord later gives rise to
vv Correct Answer is: A the vertebral column. The neural plate
Three cellular elements of the brain forms a structure known as the neural
microvasculature compose the BBB: tube through a process called neurula-
endothelial cells, astrocyte end-feet, tion. Neurulation involves proliferation
and pericytes. and migration of ectodermal cells and
Astrocytes are one type of glia cells, invagination, folding, and fusion of the
responsible for the development and neural plate in a specific pattern. An
framework of the neurons. They out- important step in neurulation includes
number neurons 5:1 and contiguously the formation of a midline groove
tile the entire CNS. They also form part along which the lateral margins of the
of the blood-brain barrier. neural plate fold. These lateral margins
start to fuse in the center, so that for a
zz Suggested Reading period of time, there are openings at
55 Ballabh P, Braun A, Nedergaard M. The each end, the anterior and posterior
blood-brain barrier: an overview: struc- neuropore. Fusion then reaches the
ture, regulation, and clinical implications. neuropores, the anterior one first, then
Neurobiol Dis. 2004 Jun;16(1):1–13. the posterior one, and the neural tube
is thus formed. The ventral and dorsal
?? 25. Regarding embryonal nervous system aspects of the neural tube each give
development, which of the following rise to specific cell populations and ulti-
statements is correct? mately specific parts of the nervous
A. The rhombencephalon gives rise system. As mentioned, mesodermal
to the cerebral hemispheres cells of the notochord provide signals
B. The mesencephalon gives rise to for differentiation to cells in the ventral
the hypothalamus and thalamus aspect of the neural tube. These signals
C. The prosencephalon gives rise include sonic hedgehog protein and
to the telencephalon, which ulti- bone morphogenetic proteins, of which
mately forms the cerebral hemi- several types have been identified.
spheres Neurulation occurs at 3–6 weeks’ gesta-
Neurophysiology/Localization
605 21
tion, and failure of crucial processes at ?? 26. In the setting of a fetus with a neural
any stage leads to a variety of abnor- tube defect (NTD), which of the fol-
malities, collectively known as neural lowing statements regarding prenatal
tube defects. As mentioned earlier, the diagnosis is incorrect?
neural plate undergoes fusion to form A. Fetoprotein level is elevated in
the neural tube in different areas at dif- maternal serum
ferent times, and failure to fuse at each B. A-Fetoprotein level is elevated in
site results in specific defects. Abnor- amniotic fluid
mal rostral fusion at the anterior neuro- C. Acetylcholinesterase is elevated
pore leads to abnormalities such as in amniotic fluid and increases
encephalocele or anencephaly, whereas sensitivity and specificity of NTD
abnormal caudal fusion (at the poste- screening
rior neuropore) leads to disorders such D. There is little utility in prenatal
as spina bifida. Following neurulation, ultrasonography in the detection
the neural tube undergoes segmenta- of NTDs
tion into three vesicles, in a process E. When a NTD is detected prena-
called specification, whereby different tally, karyotyping to assess for tri-
segments begin to acquire cell types somies and other genetic defects
and characteristics specific to the CNS may be indicated
structure that will eventually arise from
them. The three segments include the vv Correct Answer is: D
prosencephalon, mesencephalon, and Neural tube defects (NTDs) can be
rhombencephalon. The prosencepha- detected by prenatal ultrasonography.
lon subsequently forms the telencepha- Prenatally, in the setting of most NTDs,
lon, which gives rise to the cerebral serum maternal A-fetoprotein level is
hemispheres, as well as the diencepha- elevated. A-Fetoprotein is a normal com-
lon, which forms the hypothalamus and ponent of fetal CSF, and leakage into the
thalamus. The rhombencephalon gives amniotic fluid from an open neural tube
rise to the brain stem. Abnormalities leads to elevated amniotic A-fetoprotein
during specification, which occurs at level. The extent of elevation correlates
5–6 weeks of gestation, lead to disor- with the severity of the NTD. Elevations
ders such as septa-­optic dysplasia. As in amniotic fluid acetylcholinesterase
the nervous system becomes more levels also occur, and combined with an
organized and specific areas more spe- elevated fetoprotein level, increase sen-
cialized, neuronal migration begins sitivity and specificity in prenatal screen-
along specific routes, and abnormali- ing. Ultrasonography also can be used to
ties of neuronal proliferation and detect NTDs and characterize them.
migration in turn lead to specific devel- Prenatal MRI is also used in some
opmental abnormalities. The peripheral cases to assess the extent of the abnor-
nervous system (including the auto- mality, aiding in prognostication of
nomic ganglia) forms from neural crest neurologic function in life. When an NTD
cells that are derived from the neural is detected with screening, and/or other
tube after it fuses. In addition to abnormalities are detected on ultraso-
peripheral nervous system structures, nography, fetal karyotyping to assess
neural crest cells give rise to the chro- for trisomy 13, 18, and others may be
maffin tissue of the adrenal medulla done to assist in management of the
and melanocytes. pregnancy.
606 Chapter 21 · Neurophysiology/Localization

?? 27. Regarding neural tube defects result- round, protuberant, fluctuant masses
21 ing from failure of fusion of the ante- covered by an opaque membrane or
rior neuropore, which of the following normal skin. They are compatible with
statements is correct? life although they cause multiple com-
A. Failure of fusion of the anterior plications. Associated clinical features
neuropore leads to spina bifida include microcephaly, developmental
B. Failure of fusion of the anterior delay (which is more severe in occipital
neuropore leads to anencephaly as compared with frontal encephalo-
and encephalocele celes), and invariably hydrocephalus.
C. Anterior neuropore defects typi- Chromosomal aberrations commonly
cally occur from insults in the last seen in patients with encephaloceles
trimester of gestation include trisomy 13 and trisomy 18.
D. Anencephaly may be occult and Occipital encephaloceles should be
diagnosed in early childhood or distinguished from cranial meningocele
even adolescence in which only leptomeninges and CSF
E. Encephalocele is incompatible are herniated through a skull defect.
with life
?? 28. Which of the following is not a risk
vv Correct Answer is: B factor for the occurrence of neural
The neural plate fuses to form the neu- tube defects?
ral tube at different locations and at dif- A. Male gender
ferent times, and failure of fusion at B. Folate deficiency
these various locations leads to various C. Exposure to retinoic acid
neural tube defects. Failure of anterior D. Exposure to valproic acid
neuropore fusion rostrally, leads to E. Maternal diabetes
anencephaly and encephalocele. By day
26 of gestation, the anterior neuropore vv Correct Answer is: A
fuses and abnormalities in neurulation Several risk factors for neural tube
leading to these disorders therefore defects (NTDs) have been identified.
likely occur at or prior to this time NTDs are more common in females.
(weeks 1–4 gestation). Folate is involved in various pathways of
Anencephaly is the complete nucleic acid synthesis and DNA methyla-
absence of both cerebral hemispheres. tion reactions, and maternal folate defi-
Because the underlying mesoderm also ciency is a well-established risk factor
fails to properly differentiate, a large for NTD. Therefore, prenatal and perina-
cranial vault defect (in skull, meninges, tal maternal supplementation with
and skin) also occurs. This is most often 0.4 mg folic acid is recommended.
not compatible with life, and most such Teratogens associated with NTDs
infants are still born; in rare cases in include retinoic acid (vitamin A or the
which the infant is born alive, death acidic form, tretinoin, found in acne
occurs soon after birth. medications). Other teratogens associ-
Encephalocele is defined by hernia- ated with NTDs include anti-epileptics,
tion of neural tissues (hamartomatous particularly valproic acid and carbam-
brain tissue, without recognizable archi- azepine, which may lead to NTDs by
tecture) into a midline defect in the skull. affecting folate metabolism. Other risk
Encephaloceles are most often located factors for NTDs include maternal diabe-
in the occipital area and less often in tes and history of a prior pregnancy
frontal areas. Clinically, they appear as resulting in an infant with an NTD.
Neurophysiology/Localization
607 21
?? 29. At 9 months, an infant is typically able ?? 30. Regarding normal cerebral cortex
to: architecture, which of the following
A. Take three steps unaided statements is incorrect?
B. Scribble with a crayon A. Normal cerebra l cortex has six
C. Remove clothing layers
D. Have a pincer grasp B. Pyramidal cells are the most com-
E. Stack two cubes mon type of cortical neurons
C. Granular cells are found in higher
vv Correct Answer is: D numbers in sensory cortex
While the timing of developmental D. Different cortical and subcorti-
stages may vary greatly between chil- cal areas project to and receive
dren, a general timeline of important projections from specific cortical
milestones follows: layers
E. Betz cells function predominantly
zz 3 Months: as interneurons and are found in
55 Puts hands together. secondary association cortices
55 Has a social smile, recognizes parents.
55 Laughs. vv Correct Answer is: E
Betz cells are the upper motor neurons
zz 4 Months: of the nervous system; these are large
55 Rolls from tummy to back. cells found in the primary motor cortex.
The majority of the cerebral cortex
zz 6 Months: (>90%) consists of neocortex (also
55 Sits unsupported. known as isocortex), a six-layered cortex
55 Rolls in both directions. (as opposed to more primitive cortex
55 Begins to crawl. with less numbers of layers, as occurs in
55 Reaches and passes objects from one hand the paleocortex (found in olfactory and
to another. limbic cortices) and archicortex, seen in
55 Babbles with strings of vowel and conso- the hippocampus). The six layers of the
nant sounds. neocortex are molecular layer (layer I),
external granular cell layer (layer II),
zz 9 Months: external pyramidal cell layer (layer Ill),
55 May develop stranger anxiety. internal granular cell layer (layer IV),
55 Stands unassisted. internal pyramidal cell layer (layer V),
55 Has pincer grasp. and multiform layer (layer VI). Cortico-
cortical efferents (projections from one
zz 12 Months: area of cortex to another) arise mainly in
55 Walks. layer Ill and project predominantly to
55 Scribbles. layers II and Ill. Layers I, IV, and VI receive
55 Says “mama” and “dada”. the majority of thalamic efferents. Layer
V gives rise to corticostriate projections
zz 15 Months: (from cortex to striatum), and layer VI
55 Stacks two cubes. gives rise to corticothalamic projections
55 Runs. (from cortex to thalamus). Pyramidal
55 Removes clothing. cells constitute the largest number of
55 Has a 6-word vocabulary cortical neurons and are found in high-
608 Chapter 21 · Neurophysiology/Localization

est numbers in cortical areas that give ?? 32. A 15-year-old boy has a large right
21 rise to efferents; granular (or stellate) hemispheric tumor resection recently.
cells function predominantly as cortical He is admitted to inpatient reha-
interneurons and predominate in bilitation, and his progress is being
regions involved in sensory function or hindered by his lack of awareness of
integration (secondary association corti- the left side of his body. He barely
ces, etc.). Betz cells are the upper motor acknowledges the presence of weak-
neurons of the nervous system; these are ness on the left side of his body; in
large cells found in layer V of the primary fact, he largely does not recognize the
motor cortex. Formation of gyri and sulci left side of his body as being his own.  
normally occurs between weeks 20 and This is consistent with:
36 of gestation. Abnormalities in this A. A lesion in the dominant hemi-
process can lead to a variety of malfor- sphere parietal region involving
mations of cortical development. the primary somatosensory cortex
B. A lesion in the nondominant
?? 31. What is the most useful treatment for hemisphere primary visual cortex
an 8-year-old boy who has continues C. A lesion in the nondominant fron-
paroxysmal EEG discharges throughout tal lobe
sleep and presents with hyperactivity, D. A lesion in the nondominant hemi-
learning disabilities and occasional sphere parietal region involving
severe behavioral out bursts? the primary somatosensory cortex
A. Risperidone E. A lesion in the dominant hemi-
B. Carbamazepin sphere angular gyrus
C. Valproic acid
D. Phenytoin vv Correct Answer is: D
E. ACTH This boy exhibits anosognosia, or a lack
of awareness of an acquired neurologic
vv Correct Answer is: E deficit, and hemispatial neglect syn-
Electrical status epilepticus during slow drome, consistent with a lesion in the
wave sleep (ESES) is characterized by nondominant hemisphere involving the
sleep induced paroxysmal EEG activity primary somatosensory cortex (area SI).
and is associated with neuropsychologi- A lesion in the thalamus can also lead to
cal dysfunction and behavioral disorders. a neglect syndrome.
Most children with isolated epileptiform
EEG activity restricted to sleep do not ?? 33. A 17-year-old football player is
have daytime clinical seizures. Seizure brought by the EMS after he had
types during wakeful include atypical a traumatic brain injury, sustained
absence, myoclonic and akinetic. High when he was tackled to the ground
dose steroids and ACTH are the most by an opposite player. He develops a
useful treatment. Standard anticonvul- fever of 102.2 °F (39 °C), which is unre-
sants are rarely effective and anti-psy- lated to an infection or inflammation.  
chotic medications are not indicated. The fever is most likely due to a lesion
of which of the following?
zz Suggested Reading A. Pre-optic Nucleus
55 Smith S. EEG in the diagnosis, classifica- B. Arcuate nucleus
tion, and management of patients with C. Lateral hypothalamus
epilepsy. J Neural Neurosurg Psychiatry. D. Paraventricular nucleus
2005;76:ii2–7. E. Posterior nucleus
Neurophysiology/Localization
609 21
vv Correct Answer is: A vv Correct Answer is: C
The pre-optic nucleus controls heat loss, The cortex forms inside-out: cells that
and lesions in this area would produce a migrate out first form deeper layers,
fever. The hypothalamus regulates body whereas cells that migrate later form
temperature. Core body temperature, more superficial structures. The cerebral
the temperature of the deep tissues of hemispheres form from an initially sin-
the body, is detected by thermorecep- gle layer of columnar epithelium located
tors located within the anterior hypo- in the sub-ependymal region known as
thalamus. The anterior hypothalamus the ventricular zone (or primary germi-
also contains neurons r­ esponsible for ini- nal zone). Cells in this layer are pluripo-
tiating reflexes, such as vasodilation and tential and frequently divide. Another
sweating, which are designed to reduce layer, the sub-ventricular zone, consists
body temperature. Heat-producing of cells known as radial glia, which send
reflexes, such as shivering, and heat-­ out processes that extend all the way to
maintenance reflexes, such as vasocon- the pial membrane at the cortical sur-
striction, are initiated by neurons located face. Yet another, more superficial layer,
within the posterior hypothalamus. The the marginal zone, forms by the fifth
posterior hypothalamus regulates cate- week of gestation. Cells leave the mar-
cholamine secretion. The paraventricular ginal zone and migrate along the radial
nucleus regulates TSH secretion. The lat- glial processes in two waves, one at
eral hypothalamus regulates thirst, and approximately 6 weeks’ gestation and
the arcuate nucleus exerts neuroendo- the other at 11 weeks’ gestation, peak-
crine control of prolactin secretion. ing at weeks 12–14. Cells destined to
form layers 2–6 of the cortex originate
zz Suggested Reading in this second wave. The cortex devel-
55 Mowzoon N, Flemming KD. Neurology ops inside-out, such that cells in the ear-
board review: an illustrated study guide. liest portion of migration form deeper
Rochester: Mayo Clinic Scientific; 2007. Print. layers, whereas cells that migrate later
form more superficial structures. Several
?? 34. Which of the following statements is factors influence neurogenesis (forma-
incorrect regarding cortical develop- tion of neurons, including pyramidal
ment? neurons, cortical granular or stellate
A. The cerebral hemispheres form neurons, Betz cells, and others) and glio-
from a single layer of columnar genesis (formation of glia, including
epithelium located in the sub- astrocytes, oligodendrocytes, epen-
ependymal region known as the dyma, and microglia), as well as cell
primary germinal zone migration. These include regulatory pro-
B. Cells of the marginal zone proliferate teins, transcription factors, and neu-
and migrate along a scaffold formed rotransmitters, each varying with the
by the processes of radial glia stage of development. Cajal-Retzius
C. The most superficial cortical layers cells are a group of stellate neurons
form from cells that migrate first found in the cortex prior to arrival of the
(the cortex forms outside-­in) first wave of cells. These cells secrete
D. Neuron types include pyramidal GABA and acetylcholine and express
neurons, cortical granular or stel- several products including US1 and Ree-
late neurons, and Betz cells lin, genes necessary for neuronal migra-
E. Glial cell types include astrocytes, tion along radial glia. All six layers of the
oligodendrocytes, ependyma, and cortex are identifiable by 27 weeks’
microglia ­gestation.
611 22

Short Q/A

© Springer International Publishing AG, part of Springer Nature 2018


Y. M. Awaad, Absolute Pediatric Neurology, https://doi.org/10.1007/978-3-319-78801-2_22
612 Chapter 22 · Short Q/A

?? 1. All the following are components of ?? 4. Which of the following statements


or functionally related to the basal concerning ligand-gating of neuronal
ganglia EXCEPT membrane channels is true?
22 A. caudate nucleus A. The normal triggering mechanism
for gating involves non-­specific
B. red nucleus
C. Sub-thalamic nucleus binding by large classes of mol-
D. putamen ecules.
E. substantia nigra B. Channels are opened when a
given molecule selectively binds
vv Correct Answer is: B with the gating molecule
C. Ligand- gating is triggered by
?? 2. All the following statements concern- changes in the electrical potential
ing spina bifida are correct EXCEPT across the membrane
A. spina bifida occurs after failure D. The channels are constructed of a
of closure of the posterior arch of mixture of proteins and lipids
one or more vertebrae E. The gating molecule shows no
B. in spina bifida, the defect occurs conformational change during the
most frequently at sacral and lum- gating process
bar levels of the spinal cord
C. in spina bifida occulta, the menin- vv Correct Answer is: B
geal herniation extends far above
the defect in the verbal arch ?? 5. After the occurrence of an action
D. failure of closure of the caudal potential, there is a repolarization of
neuropore typically results in the membrane.
myelomeningocele The principal explanation for this
E. myelomeningocele is frequently event is that
accompanied by obstruction to A. potassium channels have been
the flow of cerebrospinal fluid opened
B. sodium channels have been
vv Correct Answer is: C opened
C. potassium channels have been
?? 3. All the following events occur during inactivated
the process of retrograde degenera- D. the membrane becomes imper-
tion EXCEPT meable to all ions
A. a displacement of the nucleus E. there has been a sudden influx of
toward the periphery of the cell calcium
B. an initial swelling of the cell body
C. a proliferation of Nissl granules vv Correct Answer is: A
D. degeneration of process along the
axon distal to the lesion ?? 6. Which of the following statements
E. an initial accumulation of mito- concerning sodium channels is true?
chondria in the axoplasm at the A. They are opened when the mem-
nodes of Ranvier brane is hyperpolarized
B. They display a high conductance
vv Correct Answer is: C in the resting membrane
Short Q/A
613 22
C. They open rapidly following depo- D. a positive Babinski sign
larization of the membrane E. a reduction in the size of the inter-
D. They are rapidly activated by nal capsule on the side ipsilateral
tetrodotoxin to the lesion

vv Correct Answer is: C vv Correct Answer is: C

?? 7. Epileptiform activity is believed to ?? 11. Unilateral deafness may result from a


include the activation of lesion of
A. AMPA receptors alone A. The auditory cortex of one side
B. Glutamate receptors alone B. The lateral lemniscus of one side
C. metabotropic receptors alone C. Cranial nerve VIII on one side
D. both AMPA and glutamate receptors D. The medial geniculate
E. both AMPA and metabotropic E. The medial lemniscus
receptors
vv Correct Answer is: C
vv Correct Answer is: D
?? 12. A man is unable to move his eyes
?? 8. All the following are usually classified downward. The lesion is most likely
as excitatory transmitters EXCEPT situated in the
A. glumate A. medulla
B. acetylcholine B. basilar aspect of the pons
C. substance P C. pontine tegmentum
D. leucine D. midbrain
E. Aspartate E. cerebellum

vv Correct Answer is: D vv Correct Answer is: D

?? 9. All of the following are drugs that ?? 13. Spasticity may result from a lesion of
affect central cholinergic synapses A. ventral horn cells
EXCEPT B. corpus callosum
A. atropine C. postcentral gyrus
B. scopolamine D. internal capsule
C. haloperidol E. substantia nigra
D. Physostigmine
E. Hemicholinium vv Correct Answer is: D

vv Correct Answer is: C ?? 14. The Klüver-Bucy syndrome is typically


associated with lesions of the
?? 10. All of the following are common A. septal area
effects of a lesion of the pyramidal B. amygdale
system EXCEPT C. cingulated gyrus
A. loss of volitional movement of the D. medial hypothalamus
limb(s) contralateral to the lesion E. lateral hypothalamus
B. spasticity
C. hypotonia of the limb(s) ipsilateral vv Correct Answer is: B
to the lesion
614 Chapter 22 · Short Q/A

?? 15. All the following events are likely to ?? 17. The cerebellar abnormality shown is most
occur during a seizure or as a conse- consistent with which of the following?
quence of seizure activity EXCEPT A. arachnoid cyst
22 A. synchronization of synaptic activity B. Chiari type II ­malformation
C. cystic hemangioblastoma with
B. a decrease in GABA receptors
C. an increase in NMDA receptors mural nodule
D. an increase in extracellular con- D. Dandy-Walker malformation
centration of calcium ions E. post-surgical changes
E. a change in noradrenergic innerva-
tions of the epileptogenic region

vv Correct Answer is: D

?? 16. Head CT of a 5-years-­old child. What


procedure has this patient most likely
undergone?

..      Fig. 22.2 A T2-weighted sagittal MRI scan, from a


patient with Chiari-like symptomatology, demonstrat-
ing tonsillar herniation less than 3 mm. Dimensions of
the posterior fossa in patients symptomatic for Chiari I
malformation but without cerebellar tonsillar descent.
(Cerebrospinal Fluid Research 2005, 2:11. doi: 7 https://
doi.org/10.1186/1743-8454-2-11. Raymond F Sekula Jr.,
Peter J Jannetta, Kenneth F Casey, Edward M Marchan,
L Kathleen Sekula and Christine S McCrady. Wikipedia)

vv Correct Answer is: B


..      Fig. 22.1 A ventriculoperitoneal shunt as seen on
CT imaging. (Own work. James Heilman, MD. Wikipedia)
?? 18. The proton density MRI shown was
obtained on a five-­year-­old with quadri-
A. Recanalization of superior sagittal paresis. What is the most likely diagnosis?
sinus A. Binswanger’s disease
B. Removal of left parietal tumor B. ceroid lipofuscinosis
C. Removal of right parietal tumor C. herpetic encephalitis
D. Removal of subdural hematoma D. Leigh’s disease
E. Ventriculo-peritoneal shunting E. metachromatic leukodystrophy

vv Correct Answer is: E vv Correct Answer is: E


Short Q/A
615 22

a b c

d e f

g h i

j k l

m n o

..      Fig. 22.3 Cranial MRIs of our patients. a-c repre- white matter on T1-weighted imaging. This pattern
sents patient 1, d-f patient 2, g-i patient 3, j-l patient of dysmyelination resembles the skin of tiger (radial
4, and m-o patient 5. a, d, g, j, m Hypointense radially stripes) and leopard (dots), the so-called tigroid and
oriented stripes and dots seen within the hyperintense leopard pattern of dysmyelination in metachromatic
cerebral white matter (resembling tiger skin) on leukodystrophy. (Liaw, Hsiang-Ru, et al. “Late infantile
T2-weighted axial imaging. b, e, h, k, n Hypointense metachromatic leukodystrophy: Clinical manifesta-
dots resembling leopard skin seen on T2-weighted tions of five Taiwanese patients and Genetic features in
axial imaging at the level of centrum ovale. c, f, i, Asia.” Orphanet Journal of Rare Diseases 10.1 (2015): 1.
l, o Iso to hyperintense dots seen in the cerebral HR Liaw, HF Lee, CS Chi. Wikipedia)
616 Chapter 22 · Short Q/A

?? 19. The MRI showed that of a young man. ?? 20. An18-year-old girl presents with
What is the most likely diagnosis of headache, nausea, and vomiting.
this patient? Physical examination reveals marked
22 A. Chiari type I malformation papilledema. A subsequent MRI scan
shows a four-cm diameter mass in
B. Machado-Joseph disease
C. Olivopontocerebellar atrophy the right parietal lobe. Biopsy is per-
D. Spinocerebellar ataxia type 6 formed. Based on the morphologic
E. Supracerebellar arachnoid cyst features shown in the figure, what is
the most likely diagnosis?
A. Astroblastoma
B. Ependymoma
C. Fibrillary astrocytoma
D. Gemistocytic astrocytoma
E. Oligodendoglioma

..      Fig. 22.6 High magnification micrograph of an


oligodendroglioma. (H&E stain. Own work. Nephron.
Wikipedia)

vv Correct Answer is: E

?? 21. MRI image of a 16-year-old girl who is


..      Figs. 22.4 and 22.5 Brain MRI images of the
having seizures postpartum are shown.
younger patient (II-14) at age 25 years old, showing What is the most likely diagnosis?
cerebellar atrophy. (Nicolaou et al. BMC Medical Genet- A. Adrenoleukodystrophy
ics 2008 9:28 doi: 7 https://doi.org/10.1186/1471- B. Bilateral hemorrhages
2350-9-28. Nicolaou et al. Wikipedia) C. Primary CNS angiitis
D. Reversible posterior leukoenceph-
vv Correct Answer is: D alopathy syndrome
E. Terminal area of non-­myelination
Short Q/A
617 22

..      Fig. 22.8 Hydrocephalus as seen on a CT scan of


the brain. The black areas in the middle of the brain
are abnormally large and filled with fluid. (Lucien Mon-
..      Fig. 22.7 Magnetic resonance image showing fils. Wikipedia)
multiple cortico-subcortical areas of hyperintense
signal involving the occipital and parietal lobes bilater-
vv Correct Answer is: A
ally and pons in a patient with posterior reversible
encephalopathy syndrome. Near fatal posterior revers-
ible encephalopathy syndrome complicating chronic ?? 23. The histopathology of a pineal tumor
liver failure and treated by induced hypothermia and removed from a 16-year-old man is shown.
dialysis: a case report. (J Medical Case Reports 2009, What is the most appropriate diagnosis?
3:6623. doi: 7 https://doi.org/10.1186/1752-1947-3-
A. Astrocytoma
6623. Rashmi Chawla, Daniel Smith and Paul E Marik.
Wikipedia) B. Germinoma
C. Pinealoma
D. Pineoblastoma
vv Correct Answer is: D E. Pineocytoma

?? 22. A 2-year-old boy presents with sei-


zures. Head CT is shown. The findings
are consistent with which of the fol-
lowing?
A. Communicating hydrocephalus
B. Hydrocephalus “ex vacuo”
C. Lobar atrophy
D. Microvascular disease
E. Multiple sclerosis

..      Fig. 22.9 Histology of a Germinoma (HE stain, ×200


magnification). (Own work. Jensflorian. Wikipedia)

vv Correct Answer is: B


618 Chapter 22 · Short Q/A

?? 24. A 13-year-old girl has a seven-year


history of complex partial seizures.
MRI scan reveals a hypo intense lesion
22 in the left mesial temporal lobe. Based
on the image shown, what is the most
appropriate diagnosis?
A. Dysembryoplastic neuroepithelial
tumor
B. Fibrillary astrocytoma
C. Glioblastoma
D. Pilocytic astrocytoma
E. Pleomorphic xanthoastrocytoma

..      Fig. 22.11 TUBEROUS SCLEROSIS. ILAE.org. Epi-


lepsyDiagnosis.org, Diagnostic Manual

vv Correct Answer is: C

..      Fig. 22.10 Dysembryoplastic neuroepithelial ?? 26. An image of the choroid plexus of
tumor (HE stain, histology). (Own work. Jensflorian. the lateral ventricle trigone (glomus
Wikipedia) choroideum) is shown. The cells iden-
tified by the arrow represent the cell
of origin for which of the following
vv Correct Answer is: A
neoplasms?
A. Choroid plexus papilloma
?? 25. What is the most likely diagnosis for
B. Germinoma
the findings shown of an adolescent
C. Glomus tumor
boy with leg numbness and blurry
D. Meningiomz
vision?
E. Teratoma
A. Diffuse axonal injury
B. Metastatic melanoma
C. Multiple sclerosis
D. vasculitis
E. vertebral artery dissection

..      Fig. 22.12 Very high magnification micrograph


of a glomus tumour, also written glomus tumor. H&E
stain. (Own work.Nephron. Wikipedia)

vv Correct Answer is: C


Short Q/A
619 22
?? 27. The catheter arteriogram shown is ?? 28. A 16-year-old woman presents with
that of a 3-month-old infant who has seizure disorder. Her brain MRI is shown.
congestive heart failure and increased What is the most likely diagnosis?
intracranial pressure. What is the most A. Bilateral strokes
likely diagnosis? B. Epidermoid
A. Anterior communicating artery C. Mesial temporal sclerosis
aneurysm D. Middle fossa arachnoid cyst
B. Hemangioblastoma E. Previous viral encephalitis
C. Mycotic aneurysm
D. Posterior communicating aneurysm
E. Vein of Galen aneurysm

..      Fig. 22.14 HSV encephalitis. This coronal


T2-weighted MR image shows high signal in the tem-
..      Fig. 22.13 Maximum-intensity projection from a poral lobes including hippocampal formations and
CT cerebral angiogram showing an aneurysmal deep parahippogampal gyrae, insulae, and right inferior
venous structure which drains to the vein of Galen, frontal gyrus. The right gyrus rectus and the columns
and then to an abnormal falcine sinus. (Radiopaedia. of the fornices were also involved (not shown). There
Dr. Laughlin Dawes. Wikipedia) was no associated hemorrhage or enhancement. There
was moderate mass-effect. (7 http://www.radpod.
org/2007/03/24/herpes-simplex-encephalitis. dr
vv Correct Answer is: E Laughlin Dawes. Wikipedia)

vv Correct Answer is: E


620 Chapter 22 · Short Q/A

?? 29. Axial and sagittal T1-weighted images


of a 5-week-old child are shown. What
is the most likely diagnosis?
22 A. arachnoid cyst
B. Chiari type II malformation
C. Dandy-Walker malformation
D. epidermoid tumor
E. mega cistern magna

..      Fig. 22.16 Haematoxylin&Eosin stained paraffin


section of an ependymoma (WHO Grade II). (Own
work. Sbrandner. Wikipedia)

vv Correct Answer is: A

?? 31. Based on the MR image shown of a


17-year-old boy with seizures, what is
the most likely diagnosis?
A. Abscess
B. Cerebral cavernous angioma
C. Intracranial hemorrhage
D. Pleomorphic xanthoastrocytoma
E. Porencephalic cyst

..      Fig. 22.15 Variant DWS with dysplasia of the pons


and cerebellum in an 8-year old. T2 weighted sagittal
MRI. (Hellerhoff. Wikipedia)

vv Correct Answer is: C

?? 30. A 24-year-old woman presents with


intractable headache and a fourth
ventricular tumor. Based on the
accompanying micrographs shown,
what is the most likely diagnosis? ..      Fig. 22.17 Brain coronal section with an Intracra-
nial hemorrhage (left hemisphere), from a MRI. (Own
A. Ependymoma work. Jlguenego. Wikipedia)
B. Medulloblastoma
C. Meningioma
D. Oligodendroglioma vv Correct Answer is: C
E. Primitive neuroectodermal tumor
Short Q/A
621 22
?? 32. The sagittal T1-­weighted MR image ?? 33. The S1 dorsal root ganglia are most
shown, for an 18-year-old woman frequently located at which of the fol-
with weakness of both upper and lowing locations?
lower extremities, indicates a need for A. distal to the intervertebral fora-
which of the following? men
A. CT scan with intravenous contrast B. proximal to the intervertebral
B. Omnipaque® foramen
C. Radiation therapy C. within the intervertebral foramen
D. Spinal angiography D. within the spinal cord dorsal horn
E. Surgical exploration E. within the spinal cord ventral horn

vv Correct Answer is: B


According to the anatomy and radio-
graphic studies, the S1 dorsal root
ganglia are intra-spinal and proximal
to the intervertebral foramen in 71% of
individuals. This is in contrast to cervical
and thoracic dorsal root ganglia, which
are typically located within the interver-
tebral foramen and lumbar dorsal root
ganglia, which are intra-­spinal 3% of the
time at L3 and L4, and 11% to 38% of
the time at L5.

zz Suggested Reading
55 Levin K. Diseases of the nerve roots.
Continuum Lifelong Learn Neurol
2008;14(3):134–55.

..      Fig. 22.18 Cervical Spine MRI of patient with SCI:


C4 fracture and dislocation, spinal cord compression.
(Own work. Андрей Королев 86. Wikipedia)

vv Correct Answer is: E


623 23

Sleep Disorders

© Springer International Publishing AG, part of Springer Nature 2018


Y. M. Awaad, Absolute Pediatric Neurology, https://doi.org/10.1007/978-3-319-78801-2_23
624 Chapter 23 · Sleep Disorders

?? 1. A 12-year-old boy complaining of relative CNS dopamine deficiency


difficulty falling asleep at night for seems to be the underlying etiology in
last few months, and he is irritable primary RLS.
and inattentive at school. He reports
that his legs tingle when he lies down zz Suggested Reading
to sleep and he has to keep moving 55 Lipton J, Becker RE, Kothare SV. Insomnia
23 them around to get the feeling to go of childhood. Curr Opin Pediatr.
away. The mother has noticed that 2008;20(6):641–9.
he kicks his legs frequently when he
sleeps. Otherwise he has no other ?? 2. A 7-year-old boy wakes up at night
medical problems and takes no medi- screaming for the past 3 months,
cations. His general and neurological which happens several times nightly.
examinations are normal.   An hour after falling asleep he sits up
What laboratory studies should in bed, with open eyes and begins
you order to further evaluate his screaming, is tremulous, sweaty and
condition? appears frightened. The episode stops
A. NCV after a few minutes and he goes back
B. SSEP to sleep. Last night he got out of
C. Liver function tests bed and ran around the house while
D. Ferritin screaming. His family cannot console
E. Fasting glucose him during the fits and he cannot
remember what happened the follow-
vv Correct Answer is: D ing morning. His family reports that
The presentation is typical of restless snores loudly at night and sometimes
legs syndrome, which is estimated to gasps for air. Past medical history is
occur in 2% of children 8–17 years of remarkable for episodes of recurrent
age. It is characterized by an urge to otitis media as a child, for which he had
move the legs, unpleasant sensations, myringotomy tubes placed. His early
relieved by movement, and symptoms development was normal, but he is
are worse at night. Primary RLS is idio- hyperactive and inattentive in second
pathic, while secondary RLS is caused grade. On examination, he is a mouth
by several conditions, including iron breather and has enlarged tonsils.
deficiency, pregnancy, renal disease, What is his best treatment option?
neuropathy, medications (including A. Parental reassurance
neuroleptics and serotonin re-uptake B. Adenotonsillectomy
inhibitors). Genetic factors have an C. Benzodiazepine
important role. The condition inter- D. Oxygen
feres with falling asleep, thereby caus- E. SSRI
ing insomnia. Most patients have
periodic limb movements (stereotyped vv Correct Answer is: B
limb movements lasting 0.5–5 s recur- The clinical presentation is typical of
ring in 20–40 s intervals. In children sleep terrors (aka night terrors, pavor
with RLS, secondary causes should be nocturnus), which are a form of non-REM
sought, the most common of which is parasomnias occurring during slow wave
iron deficiency. Ferritin level below 50 sleep. They affect about 3% of children,
warrants a trial of iron replacement. with peak incidence at 5–7 years of age,
Dopamine agonists (ropinirole, prami- usually resolving by adolescence. The
pexole) are an effective treatment, as manifestations are characterized by
Sleep Disorders
625 23
marked autonomic nervous system acti- because this was not of her character.
vation: tachycardia, tachypnea, tremu- Her physical and neurological exami-
lousness, mydriasis, and sweating. Facial nation is normal. You order routine labs
expression of intense fear is associated and a brain MRI which are all normal.  
with uncontrollable shouting, scream- An EEG during the episode may
ing, gasping, moaning, and agitation. show:
Most children remain in bed during the A. Generalized slowing of back-
events, but sometimes will walk or run ground activity
during them. Their duration typically last B. Slow spike and wave
one to several minutes, sometimes up to C. Electrographic status of slow wave
half an hour. The child cannot be con- sleep
soled and attempts to awaken the child D. Central-temporal spike and wave
make them more intense. Differential discharges
diagnosis includes nightmares (briefer, E. Frontal spikes
second half of the night, consolable,
dream recollection), seizures (1–2-min vv Correct Answer is: A
stereotyped hyper-motor activity), panic Klein-Levin syndrome is a rare disorder
attacks, and cluster headaches. They can featuring recurrent episodes of excessive
be exacerbated by acute stress, sleep sleepiness and prolonged total sleep
deprivation, and some medications time. Usually, the onset is in late adoles-
(stimulants, sedative-hypnotics, neuro- cence, but younger and older children
leptics, antihistamines). Studies have cases have been reported. The excessive
shown that sleep disordered breathing sleepiness and true hypersomnia can
and restless legs syndrome-periodic develop suddenly, but symptoms typi-
limb movement disorder can increase cally have a gradual onset of 1–7 days.
sleep terrors; treatment of these condi- During spells of hypersomnia, patients
tions can significantly improve the ter- rarely leave their beds and sleep contin-
rors. Signs and symptoms of obstructive uously. In addition to excessively long
sleep apnea are present in this boy and sleep episodes, abnormal behaviors,
should be addressed. including but not limited to compulsive
and excessive overeating and sexually
zz Suggested Reading acting out behavior can occur.
55 Mason T, Pack A. Sleep terrors in child- Compulsive overeating and hyper sexu-
hood. J Pediatr. 2005;147(3):388–92. ality are not present with each episode.
Usually, physical examination is normal.
?? 3. A 15-year-old female came to your Also, brain MRI is typically normal.
office complaining of six episodes of Routine EEG obtained during attacks
hypersomnia in the last six months. may show generalized slowing of the
She is an excellent academic stu- background or may be unremarkable.
dent, well adjusted, and enjoys many
extracurricular activities between the zz Suggested Reading
episodes. During the episodes, she is 55 Sheldon et al. Principles of practice of
fatigued over several days, and then pediatric sleep medicine. 2005.
sleeps almost continuously for over
a week but awakens to eat compul- ?? 4. A 10-year-old boy with a history of
sively. On three occasions she sent ADHD came for an evaluation. His
sexually suggestive text messages to family reports that he is constantly
classmates which greatly disturbed her moving, always on the go and cannot
626 Chapter 23 · Sleep Disorders

sit still at bedtime. He told his family stretching, at least as long as the activ-
that he has to move otherwise his ity continues
legs would feel funny. He told you 4. The urge to move or unpleasant sensa-
that he feels like there are bugs crawl- tions are worse in the evening or night
ing on his legs and this sensation than during the day, or only occur dur-
keeps him up at night, and if he walks ing the evening or night
23 or runs his legs feel better.  
One of the criteria for Restless leg Definite childhood restless legs
syndrome is that he has to relate a syndrome (RLS)
description in his own words that is 1. The child meets all four essential adult
consistent with leg discomfort.   criteria, and
All the criteria below for definite 2. The child relates a description in his or
childhood restless leg syndrome are her own words that is consistent with
true except: leg discomfort
A. An urge to move the legs, usu- OR
ally accompanied or caused by 1. The child meets all four essential adult
uncomfortable and unpleasant criteria and has two of three following
sensations in the legs. A. sleep disturbance for age
B. The urge to move or unpleasant B. biological parent or sibling with
sensations begin or worsen dur- definite RLS
ing periods of rest or inactivity C. polysomnographically documented
such as lying down or sitting. periodic limb movement index of
C. The urge to move or unpleasant five or more per hour of sleep
sensations are partially or totally
relieved by movement, such as Probable childhood RLS
walking or stretching, at least as 1. The child meets all four essential adult
long as the activity continues. criteria for RLS except criterion 4 (the urge
D. The urge to move or unpleasant to move or sensations are worse in the
sensations are worse in the eve- evening or at night than during the day),
ning or night than during the day, And
or only occur during the evening 2. The child has a biological parent or
or night. sibling with definite RLS
E. Ferritin levels must be lower than OR
the 5th percentile. 1. The child is observed to have behav-
ioral manifestations of lower extremity
vv Correct Answer is: E discomfort when sitting or
The Criteria for Restless Leg syndrome is Lying, accompanied by motor
below. movement of the affected limbs;
Essential diagnostic criteria (adults) the discomfort has characteristics of
1. An urge to move the legs, usually accom- adult criteria 2, 3, and 4,
panied or caused by uncomfortable and And
unpleasant sensations in the legs 2. The child has a biological parent or
2. The urge to move or unpleasant sensa- sibling with definite RLS
tions begin or worsen during periods Adapted from: Allen RP, Pichhietti D,
of rest or inactivity such as lying down Hening WA, et al. Restless legs syn-
or sitting drome: diagnostic criteria, special con-
3. The urge to move or unpleasant sensa- siderations, and epidemiology.
tions are partially or totally relieved A report from the restless
by movement, such as walking or legs syndrome diagnosis and
Sleep Disorders
627 23
epidemiology workshop at the ?? 6. A 5-year-old boy has been evaluated
National Institutes of Health. Sleep for nocturnal spells, in which he wakes
Medicine 2003; 4:101–119. up and repeatedly screams, “No! Ow!”
His family reports that on awaken-
zz Suggested Reading ing he will have dilated pupils and his
55 Kotagal S, et al. Childhood-­onset restless heart races, but no stereotyped move-
legs syndrome. Ann Neurol. 2004;56:803–7. ments have been noted. Try to comfort
him, makes him more agitated. Those
?? 5. A 17-year-old boy is complaining of spells started 5 months ago and they
excessive daytime sleepiness for the past are infrequent, occurring once every
2 years. He goes to bed at 8:30 p.m. and 2–3 weeks, usually within 2 h of his
falls asleep quickly but still feels tired going to bed. They are about 10 min
upon awakening. He is not a restless in duration, and then he goes back to
sleeper and does not snore. He school sleep. He does not seem to remember
grades are dropping, and his teachers the events in the morning. His family
complained of him falling asleep in class reports that he snores most nights, but
on several occasions. He takes short a polysomnogram did not demonstrate
naps during his lunch period and on the sleep apnea.
bus ride home from school, after which What is the most appropriate
he feels rested for a couple of hours. ­treatment?
Another neurologist has evaluated her A. Clonazepam
2 months ago after having a seizure like B. Reassurance
event versus syncope. While he was jok- C. Clonidine
ing around with his friends, suddenly D. Carbamazepine
he dropped his head to his chest and E. Referral to psychiatry
he was not able to respond for about
15 s. A polysomonogram (PSG) followed vv Correct Answer is: B
by multiple sleep latency test (MSLT) is Parasomnias are “undesirable physical
ordered. The PSG shows an increased events or experiences that occur during
proportion of stage 1 sleep. The MSLT entry into sleep, within sleep, or during
shows a mean sleep latency of 4 min and arousals from sleep”, according to the
presence of REM sleep in 3 of the 5 naps. International Classification of Sleep
What is the most appropriate rec- Disorders (ISCD-2). They are classified as:
ommendation to address this boy’s (1) disorders of arousal from non-rapid
daytime sleepiness? eye movements (NREM) sleep; (2) para-
A. Start melatonin 1 h before bed- somnias usually associated with REM
time sleep; and (3) other parasomnias. While
B. Otolaryngologist referral for a ton- the nightmares are the exception, they
sillectomy are associated with REM sleep, most of
C. Start atomoxetine 80 mg daily the pediatric parasomnias are disorders
D. Start fluoxetine 20 mg daily of arousal from NREM sleep, including
E. Start modafinil 100 mg daily in the sleepwalking, confusional arousals, and
morning sleep terrors. Sleepwalking peaks at
11–12 years old, males and females are
vv Correct Answer is: E equally affected. Most children who
This boy clinical scenario and even the sleep sleepwalk appear calm; however, they are
studies are indicative of Narclepsy. The best at risk for injury. Children, generally
treatment option is a stimulant to help him infants and toddlers, with confusional
out the overcome his sleep disorder. arousals are more agitated. They may
628 Chapter 23 · Sleep Disorders

have their eyes open or closed and may ?? 7. Parents of a 6-year-old boy awakens
cry or call out and thrash around for several times a week on their son’s
5–15 min before returning to restful piercing scream. He sits up in bed,
sleep, often becoming more agitated looking afraid, flushed, sweating and
with attempts to console them. Sleep ter- breathing heavily. He is confused and
rors are more dramatic still, with affected disoriented. He goes back to sleep
23 children appearing intensely fearful, several minutes later and does not
showing evidence of autonomic activa- remember the event the next day. His
tion (mydriasis, diaphoresis, tachycardia, physical and neurological examina-
and hyperventilation). Sleep terrors affect tion is normal.  
approximately 3% of children between What is your most likely diagnosis?
the ages of 4–12 years, with peak preva- A. Temporal lobe epilepsy
lence between 5 and 7 years. Since sleep- B. Central sleep apnea
walking, confusional arousals, and sleep C. Electrical status epilepticus of
terrors all arise out of NREM sleep, they sleep
tend to occur in the first third of the night D. Sleep terrors
when NREM sleep is most prevalent. E. Benign epilepsy with centrotem-
Several studies showed that arousal para- poral spikes
somnias have a genetic predisposition.
Sleep deprivation, a stimulating sleep vv Correct Answer is: D
environment, intrinsic sleep disorders Sleep terrors (night terrors, pavor noc-
(e.g. obstructive sleep apnea, periodic turnus) occur during slow wave sleep,
limb movements in sleep), and certain typically in the first third of the night
medications are known triggers. and last seconds to minutes. Fearful
The differential diagnosis includes behavior and sympathetic hyperactivity
epilepsy, particularly nocturnal frontal are typical. Sleep deprivation can be a
lobe epilepsy. Parasomnias can often be trigger.
diagnosed on clinical grounds, but in
some cases, sleep-­deprived EEG, ambu- ?? 8. A 5-year-old girl has been evaluated
latory continuous EEG, and/or inpatient of possible seizures. She has several
video EEG monitoring may be neces- episodes every month where she
sary to clarify the diagnosis. Overnight awakens screaming, flails her limbs
polysomnography is indicated if there is and cannot be consoled. She has
concern for an intrinsic sleep disorder. “pounding heart” and she sweats
Treatment consists of reassurance, profusely during the episodes. She
caffeine-­free beverages, and minimizing has been having those episodes for
sleep deprivation. Parents should be the past 4 months, though more
instructed not trying to wake the child often over the past month or so.
during an episode and, when appropri- They occur almost about 2 h after
ate, about instituting safety measures. she falls asleep, last up to 20 min,
Scheduled anticipatory awakenings and then she is back asleep. Some-
and/or low-dose clonazepam given at times she has more than 1 spell per
bedtime can be helpful in children with night. No color change or incon-
frequent parasomnias. tinence is associated. She cannot
recall anything about the episodes
zz Suggested Reading the following morning. Her general
55 Mason TBA, Pack AI. Pediatric parasom- and neurologic examination are
nias. In Sleep 2007;30(2):141–51. normal.   
Sleep Disorders
629 23
What is your explanation for these thrashing, sweating, and tachycardia.
episodes? Usually the child appears awake with
A. nightmares eyes open, but he is still asleep and can-
B. nocturnal frontal lobe epilepsy not communicate effectively and cannot
C. psychogenic non-­epileptic events respond consistently to the family.
D. REM behavior disorder Family often confuse the events with
E. sleep terrors nightmares, but unlike nightmares
(which occur in REM sleep), child cannot
vv Correct Answer is: E be aroused and do not report having
Paroxysmal events are very common had a dream. The night terror episode is
referrals to pediatric and adult neurolo- usually several minutes in length, often
gists, mainly to rule out seizure. There up to 15–20 min, after which the child
are many paroxysmal non-epileptic returns back to sleep and wakes up the
events, some of which are unique to the next morning usually well rested (unlike
pediatric age group. A detailed history the parents) without remembering the
of the episodes might be the single most event. Although there have been many
important aspect of the evaluation and theories about the reasons for night ter-
can often allow a specific diagnosis with- rors (including emotional stressors and
out the need for further testing. In fact, occasionally in adults prescription drugs
without an appropriately acquired his- or alcohol use), most often there is no
tory, ancillary testing often leads to an immediate cause identified and a family
incorrect diagnosis as it is recognized history of similar or other parasomnias is
that up to 3% of the population will have common. Treatment is often not indi-
interictal epileptiform features on an cated unless the events are occurring
EEG without having epileptic seizures. multiple times per week and impairing
Video-recording of the event by the fam- the quality of the sleep of the rest of the
ily can be very helpful. Events that occur household. Searching for co-­morbid
during sleep can be quite distressful to sleep disorders including obstructive
the child and the entire family. sleep apnea is important. Medications
Parasomnias are sleep related disorders used with varying degrees of success
occurring during REM and non-REM have included iron supplementation
sleep leading to complete or partial (even if not frankly iron deficient), mela-
arousals. Examples include nightmares, tonin, clonidine and benzodiazepines.
night terrors, sleepwalking and confu- Often, reassurance is all that is needed.
sional arousals. Night terrors in particu- Nocturnal frontal lobe epilepsy, some-
lar are very common in young children, times involving chiefly the supplemen-
occurring in up to 5% of children ages tary motor cortex. Adding to the
3–5 years. They occur in non-­REM slow diagnostic confusion is the potential
wave sleep (stage 3) and as such occur that there may never be any associated
usually 60–120 min after sleep onset events during wakefulness. These are
(and occasionally during subsequent typically short (less than 1–2 min) abrupt
entries into these stages of sleep later at arousals sometime accompanied by
night.) They do not occur at sleep-wake vocalizations, but not the same degree
transitions, Seizures on the other hand of persistent screaming. Hyperkinetic
usually occur just after entering sleep or motor manifestations dominate the epi-
immediately prior to awakening. Sleep sodes, but can also be more subtle.
terrors have a sudden onset, and the Multiple episodes throughout the
child appears terrified, with screaming, night are commonly reported. In most
630 Chapter 23 · Sleep Disorders

cases, an accurate history and/or video vv Correct Answer is: D


recording of the events will allow clear Recording REM on a routine EEG is
differentiation between these events abnormal. This indicates either extreme
and parasomnias. Psychogenic non- sleep deprivation or a potential diagno-
epileptic seizures can be challenging to sis of narcolepsy. Excessive daytime
distinguish from epileptic seizures as sleepiness is the most common present-
23 they can occur at apparent sleep wake ing feature of narcolepsy in children
transitions (though the patient may not (though narcolepsy is certainly not the
have actually fallen asleep prior to the most common reason for excessive day-
event onset). These events are not very time sleepiness in children). The diagno-
common in children of this age, typi- sis is not confirmed until the additional
cally not until near the end of the first core features of the disorder such as cat-
decade of life. REM behavior disorder aplexy, sleep paralysis and hypnagogic
involves the lack of REM atonia and hallucinations develop in the teenage or
leads to acting out of dreams. This is not early adult years. Disturbed nighttime
common in children and in adults can sleep, automatic behavior and short-
be the first presentation of an impend- term memory problems are subtle fea-
ing neurodegenerative disorder like tures of the disorder in children.
Parkinson disease. The patient can be The EEG in absence or benign
typically woken from the spell and will Rolandic epilepsy should show repre-
recall the detail of the dream on most sentative interictal discharges and not
occasions. epochs of REM sleep, but excessively
sleepiness is not typically a feature of
zz Suggested Reading either conditions.
55 Drazkowski JF, Chung SS. Differential Conversion disorder and substance
diagnosis of epilepsy. Continuum Lifelong abuse should always be considered in
Learning Neurol. 2010;16(3):36–56. atypical cases of altered responsiveness
or excessive sleepiness, though onset at
?? 9. A 9-year-old boy with a six months the age of nine would be highly unusual
history of increasing daytime sleepi- for either.
ness and falling asleep in unusual
places. His family reported that he zz Suggested Reading
has weekly episodes of staring and 55 Macleod S, Ferrie C, Zuberi
unresponsiveness at times with facial SM. Symptoms of narcolepsy in children
twitching. His brain MRI is normal. misinterpreted as epilepsy. Epileptic
He has a normal awake and sleep Disord. 2005;7(1):13–7.
EEG. He has been taking ethosuxi-
mide for 2 weeks and has been hav- ?? 10. A 17-year-old senior high school
ing more episodes. The family also student comes to your clinic with
reported that he has become irritable complaints of daytime sleepiness
and is having behavioral problems at which are causing him to fall asleep
school.   in the classroom for the last year,
What is the likely diagnosis? despite having adequate night sleep.
A. Substance abuse He reported that he would likely doze
B. Benign Rolandic epilepsy off pretty quickly if he is sitting in a
C. Conversion disorder chair in a quiet room. He adds that
D. Narcolepsy at times he will find himself nodding
E. Absence epilepsy off in front of the computer, and that
Sleep Disorders
631 23
just before nodding off he will write does not have a TV in his bedroom,
nonsensical sentences which he will and doesn’t work on the computer
discover only once he awakens from or do homework after 10 pm. He is
nodding off. He mentioned, brief naps not snoring. Physical exam including
help. He denies ever falling down or blood pressure shows a fit young ado-
having weakness in the knees from lescent with no apparent deficits. He
laughing or other strong emotion. is not feeling depressed, and says in
He often gets tired while driving at general, he likes school.  
night, he pulls over and exercise, or What is your diagnosis?
roll down the window to stay awake. A. Circadian rhythm disorder,
He has no hallucinations right as he delayed phase type
is going to sleep or as he is waking B. obstructive sleep apnea
up, but reports that at times he does C. shift work disorder
wake up, unable to move for a few D. typical adolescent behavior, with-
seconds, which is very scary. No snor- out pathology
ing has been reported, but he wakes E. occult alcohol abuse
up feeling tired. He sleeps at least 7 h
at night, with more on weekends.   vv Correct Answer is: A
What medicine can be used to treat Delayed sleep phase syndrome can be
this disorder? manifested in teenagers and young
A. Stimulants-modafinil, armodafinil, adults. This presentation is classic.
dextroamphetamine, metham- Obstructive sleep apnea, is not ruled
phetamine and methylphenidate out, but in a young, physically fit indi-
B. Sodium oxybate vidual who does not snore, it is much
C. Tricyclic anti-­depressants less likely. Shift work disorder is an
D. SSRls and SNRls excess of sleepiness experienced when-
ever a person is working when they
vv Correct Answer is: All of the above would ordinarily be sleeping, which
does not fit the above clinical presenta-
?? 11. A 16-year-old boy is complaining of tion. As it is not typical adolescent
having a hard time waking up in the behavior without pathology, and a
morning at 7 a.m. to go to school, and sleepy teenage driver is a potent mix
that he finds it difficult to fall asleep (looking forward a year or two if this
before 2 a.m. in the morning. He behavior is not diagnosed and treated).
sometimes falls asleep in class, and Occult alcohol abuse is much lower on
does not yet drive. After he comes the differential, and while possible,
home back form school he feels tired, hardly the most likely diagnosis, and
and occasionally naps, but says that should manifest with other symptoms
cross country practice has him revved (headache, frequent urination,
up until about 5 or 6 pm. He does not decreased school performance, etc.).
have trouble sleeping on the week-
ends, and you ask him to keep a sleep ?? 12. Which of the following statements
log over the next 4 weeks. His sleep regarding sleep phenomena is not
log showing 6 h of sleep on week- correct?
days, and 10–11 h of sleep on the A. Sleep paralysis is a feature of nar-
weekends. He does not drink caffein- colepsy
ated beverages, and gets daily exer- B. Hypnagogic hallucinations occur
cise with cross country practice. He on falling asleep
632 Chapter 23 · Sleep Disorders

C. Hypnopompic hallucinations Kluwer Health Lippincott Williams &


occur on waking up from sleep Wilkins. All rights reserved.
D. Patients have recollection of
nightmares ?? 13. A mother of a 9 years old boy com-
E. REM sleep behavior disorder (RBD) plaining that her son sleepwalks
is associated with excessive atonia “frequently”, at least 3–4 times per
23 during REM sleep month. She has difficulty to arouse
him during these times, and he has
vv Correct Answer is: E had odd behaviors while sleepwalk-
REM sleep behavior disorder (RBD) is ing, urinating in corners of the house.
characterized by REM without atonia and He does not recall the episodes. He
by the appearance of complex motor has good sleep hygiene, getting
activity during which the patient acts 10 h of sleep at night, with regular
out dreams. Dream content is usually bedtimes. He has no snoring. Family
violent with associated movements (e.g. is history is positive of his father also
Punching, kicking, and running). Sleep was a sleepwalker as a child.  
paralysis is a feature of narcolepsy and is What is the most important consid-
characterized by a transient paralysis eration in this case?
during sleep onset or on awakening; the A. having the child wear diapers
patient is fully conscious during these when sleeping
events. Hypnagogic hallucinations occur B. ensuring the child is not drinking
at sleep onset and are seen in patients before bedtime
with narcolepsy, and hypnopompic hal- C. avoiding afternoon naps
lucinations occur on awakening from D. making sure the house is safe for
sleep. A nightmare is a complicated the child during his sleepwalking
dream that becomes frightening toward episodes
the end. Patients with nightmares usu- E. locking him in the room where he
ally remember their dream in detail, and is sleeping
occurs in the last third of the night, and
there is much less autonomic activity as vv Correct Answer is: D
compared to sleep terrors. When Making sure the house is safe for the
patients are awakened from nightmares, child during his sleepwalking episodes,
they tend to have good intellectual l protecting the child from any harm
function and are not confused. while sleepwalking is of at most impor-
tance. Benzodiazepines are occasionally
zz Suggested Reading used with severe (i.e. nightly) sleepwalk-
55 American Academy of Sleep Medicine. ing for a few months, with subsequent
International classification of sleep tapers. The other measures are either
disorders, 2nd ed. Diagnostic and coding unnecessary, counterproductive, or not
manual, Westchester, IL: American as important of considerations.
Academy of Sleep Medicine; 2005.
55 Bradley WG, Daroff RB, Fenichel GM, zz Suggested Reading
et al. Neurology in clinical practice, 5th ed. 55 Bhargava S. Diagnosis and management of
Philadelphia, PA: Elsevier; 2008. common sleep problems in children.
Comprehensive review in clinical neurol- Pediatr Rev. 32(3);91–9, 201.
ogy: a multiple choice question book for 55 Duysse DJ, Strollo PJ, Black JE, et al.
the wards and boards ©2011 Wolters “Chapter 22: sleep disorders” in the
Sleep Disorders
633 23
American Psychiatric Publishing Textbook ?? 16. Which of the following statements is true
of Psychiatry, 5th Ed. regarding the differentiation of frontal
55 Hales RE, Yudofsky SC, Gabbard GO, lobe seizures from sleep disorders?
editors. American Psychiatric Publishing, A. Seizures tend to occur fewer times
2011. per night than episodes due to
sleep disorders
?? 14. Which of the following is TRUE regard- B. Events occurring within the first
ing sleep? 30 min of sleep are more likely to
A. newborns spent 90% of their be seizures
sleep in REM C. Episodes of dystonic posturing are
B. duration of sleep decreases dur- less likely to be seizure
ing adolescence D. Seizures tend to be of longer
C. stage 3 and 4 sleep peaks during duration than episodes due to
adolescence sleep disorders
D. number of night-time awakenings E. None of the above
decreases as adulthood progresses
E. stage 2 Non-REM and REM sleep vv Correct Answer is: B
increases as adulthood progresses Seizures occur during the first 30 min of
sleep rather than sleep-related disorders.
vv Correct Answer is: C Other different manifestations are:
Adolescence marks the peak of “deep” Frontal lobe seizures are more likely
sleep, adolescents and they do not tend in younger patients, while sleep disor-
to sleep more as newborns, and new- ders are often evident at older adults.
borns spend approximately 50% of their Seizures are brief, less than 2 min dura-
sleep in REM-like sleep. Duration of sleep tion. Episodes related to sleep disorders
actually increases during adolescence. are more likely to be prolonged, lasting
Number of awakenings, as well as the more than 10 min. Episodes that cluster,
amount of “light” stage 1 sleep, increases with 3 events occurring per night more
as you progress in later adulthood. likely to be seizures. Events related to
sleep disorders commonly occur 1–2
?? 15. A 5-year-old girl has frequent episodes times per night. Auras are more common
of inconsolable crying shortly after with seizure activity. Wandering outside
falling asleep. Her family have noted of the bedroom is uncommon with sei-
that her pupils appear large and her zure activity. Directed behaviors (e.g.
heart races. She appears very scared dressing, preparing food) are uncom-
and confused and will not react to her mon with seizure activity. Stereotyped
family’s attempts to comfort her. movements, dystonic posturing, tonic
This girl most likely has: limb extension, and cramping are more
A. Nocturnal seizures common with seizure activity. Recall of
B. Nocturnal panic attacks events may occur with Seizure activity,
C. Sleep terrors and is uncommon with sleep disorders.
D. REM sleep behavior disorder
E. Sleep-onset association disorder zz Suggested Reading
55 Derry et al. Distinguishing sleep disorders
vv Correct Answer is: C from seizures: diagnosing bumps in the
This is a typical presentation of child- night. Arch Neurol. 2006;63(5):705–9.
hood sleep terrors. Erratum in: Arch Neurol. 2006;63(7):1037.
634 Chapter 23 · Sleep Disorders

Sleep disorder Frontal lobe seizure

Age of onset More likely in older patients, More likely in younger J,-patients
55 years of age

Duration More likely if prolonged, >10 min More likely if brief: <2 min

23 Clusters Commonly 1–2 events per night More likely to have multiple
events (2:3 per night)

Timing May occur at any time during the Likely within 30 min of sleep
night Onset

Aura Uncommon Common

Wandering outside of the Common Uncommon


bedroom

Directed behavior Common Uncommon


(i.e. dressing)

Dystonic posturing, tonic Uncommon Common


limb extension, or cramping

Stereotyped behavior Uncommon Common

Recall of events Uncommon May occur

Speech during event More likely to have no speech, If coherent speech present, it is
sounds: only, or single words, if more likely to be recalled
coherent speech is present, there
is often no recall

Epileptiform EEG abnormalities Absent Present, although may be difficult


to detect on scalp recordings

?? 17. A 17-year-old student complains of vv Correct Answer is: C


difficulty staying awake in classroom. Excessive daytime sleepiness and cata-
He reports having always been sleepy plexy are classical features of narco-
during the daytime since his early lepsy. Narcolepsy is a classic tetrad: (1)
teenage years. He gets about 7 h of excessive daytime sleepiness, (2) cata-
sleep nightly. He feels more alert after plexy, (3) hypnagogic or hypnopompic
taking a short nap. Recently, he has hallucinations, (4) sleep paralysis.
developed spells of sudden falling Excessive daytime sleepiness – often the
down when laughing or being angry. first symptom in adolescence or early
He denies loss of consciousness dur- adulthood. Cataplexy – sudden, tran-
ing those episodes.   sient loss of muscle tone with associ-
The patient most likely has: ated hyporeflexia or areflexia, provoked
A. Psychogenic unresponsiveness by an emotional stimulus such as laugh-
B. Syncopal events ter, surprise, anger. Partial or complete
C. Narcolepsy muscle atonia (sparing of respiratory
D. Complex partial seizures and extraocular muscles). Sleep paraly-
E. Idiopathic hypersomnia sis – occur mostly upon awakening but
Sleep Disorders
635 23
also at sleep onset and resolves sponta- vv Correct Answer is: C
neously or with sensory stimuli (touch- REM sleep is characterized by a mixed
ing patient, calling person’s name). voltage EEG, which looks similar to
Hypnogogic and hypnopompic halluci- Stage 1 sleep, with rapid phasic eye
nations – mostly visual hallucinations at movements. Muscle tone should be
transition between sleep and wakeful- absent, except for intermittent phasic
ness; auditory, tactile, vestibular halluci- muscle twitches.
nations also possible.
zz Suggested Reading
zz Suggested Reading 55 Duysse DJ, Strollo PJ, Black JE, et al.
55 Mowzoon N, Flemming KD. Neurology “Chapter 22: sleep disorders” in The
board review: an illustrated study guide. American Psychiatric Publishing Textbook
Rochester, MN: Mayo Clinic Scientific, of Psychiatry, 5th ed. Hales RE, Yudofsky
2007. Print. SC, Gabbard GO, editors. American
Psychiatric Publishing, 2011.
?? 18. Which chromosome is associated with
Narcolepsy? ?? 20. A 15-year-old boy is having episodes
A. chromosome 2 of walking around his house in the
B. chromosome 4 middle of the night. Once, during
C. chromosome 6 such an episode, he took the garbage
D. chromosome 13 out of his house and placed it in front
E. chromosome 18 of his neighbor’s door. His neighbor
tried to talk to him, but he did not
vv Correct Answer is: C respond. When the neighbor shook
A mutation in chromosome 6 is strongly him, the boy suddenly changed his
associated with narcolepsy-cataplexy. behavior, started to scream, and
Although this mutation is seen in 90% became violent and confused. You
to 100% of patients with narcolepsy, it diagnosed him with a sleep disorder.  
also occurs in as many as 50% of people Which of the following is correct?
without narcolepsy. A. He has a REM parasomnia
B. He has confusional arousals
zz Suggested Reading C. He has narcolepsy
55 Chromosomes and their associated D. During these episodes, the EEG
diseases: Kaufman DM. (2007). Clinical shows slow-wave sleep
neurology for psychiatrists, 6th ed. E. He has sleep terrors
Philadelphia: W.B. Saunders.
vv Correct Answer is: D
?? 19. At what time during the night does The boy has sleepwalking which is a
REM sleep occur? non-­REM parasomnia. Parasomnias have
A. right when a person falls asleep been classified into REM parasomnias,
B. in the middle of the cycle arousal disorders, sleep-wake transition
C. in the second half of the night, in disorders, and other parasomnias.
the morning hours Arousal disorders include confusional
D. in the first half of the night, in the arousals, sleepwalking, and sleep ter-
evening hours rors. The latter three are non-­REM para-
E. none of the above somnias arising from slow-wave sleep or
636 Chapter 23 · Sleep Disorders

stage III sleep (previously known as complains of being very sleepy dur-
stage Ill and stage IV sleep; now they ing daytime, having multiple episodes
are combined into stage III sleep). of falling asleep during the day, after
Sleepwalking consists of a group of which she feels refreshed.
complex behaviors resulting in walking What is your the most likely
during sleep. It is more commonly in ­diagnosis?
23 children, but can present in adolescents A. Gelastic seizures
and adults, and a positive family his- B. Narcolepsy with cataplexy
tory has been reported in many cases. C. Pseudo-seizures
Sleepwalking occurs more commonly in D. Kleine-Levin syndrome
the first third of the sleep, and patients E. Idiopathic hypersomnia
have complex motor behaviors, with
amnesia of the episode. These patients vv Correct Answer is: B
are difficult to arouse, and may become This is the clinical presentation of narco-
confused and exhibit violent behavior lepsy with cataplexy. Narcolepsy is char-
when this is attempted. Confusional acterized by excessive sleepiness,
arousals are a non-­REM parasomnia cataplexy, sleep paralysis, and hypnago-
occurring in children, in which the gic and hypnopompic hallucinations.
patient is confused following an arousal Narcolepsy patients complain of sleep
from slow-wave sleep. The clinical sce- attacks, in which they have an irresist-
nario of this boy is not consistent with ible desire to fall asleep during inappro-
narcolepsy, nor with sleep terrors. priate circumstances. The sleep attacks
last 15–30 min, and the patient feels
zz Suggested Reading refreshed afterward. Even though cata-
55 American Academy of Sleep Medicine. plexy is associated with narcolepsy, not
International classification of sleep all patients with narcolepsy have cata-
disorders, 2nd ed. Diagnostic and coding plexy. Cataplexy is an episode of sudden
manual. Westchester, IL: American loss of tone of voluntary muscles, except
Academy of Sleep Medicine; 2005. respiratory and ocular muscles.
55 Bradley WG, Daroff RB, Fenichel GM, During these attacks, the patients
et al. Neurology in clinical practice, 5th ed. may fall and be cannot move, and deep
Philadelphia, PA: Elsevier; 2008. tendon reflexes are decreased or absent.
Comprehensive review in clinical neurol- Patients are fully conscious. Cataplectic
ogy: a multiple choice question book for attacks are triggered by emotional
the Wards and Boards © 2011 Wolters events, such as laughter or anger. This
Kluwer Health Lippincott Williams & patient does not have pseudo-­seizures,
Wilkins. All rights reserved. nor gelastic seizures. This patient does
not have Kleine-Levin syndrome, which
?? 21. A 17-year-old female has been evalu- is a recurrent hypersomnia, character-
ated for possible pseudo-seizures. ized by recurrent episodes of hyper-
During the episodes she falls sud- somnia occur weeks or months apart.
denly to the floor and loses her The episodes can last several days and
muscle tone completely, and she can- sometimes weeks, occurring several
not move for 30–40 s. These events times per year, and these patients sleep
usually come after emotional triggers, for prolonged periods of time, 18–20 h,
frequently laughter. She is completely waking only to eat and void. During
conscious during these events. She these episodes, patients may have
Sleep Disorders
637 23
disrupted behaviors, such as irritabil- vv Correct Answer is: A
ity, aggressiveness, confusion, hyper Loss of hypocretin neurons in the lat-
sexuality, and a voracious appetite. In eral hypothalamus might be associated
between episodes, patients have normal with narcolepsy with cataplexy, and
behavior and sleep pattern. This patient hypocretin CSF levels are low in these
does not have idiopathic hypersomnia, patients. Patient’s loss their muscle
which is characterized by long-term tone during the attack and they have
inability to obtain adequate sleep, in hypo- or areflexia. Usually, they have
which the patients have excessive sleep- sleep paralysis accompany the attack.
iness and the etiology is thought to You can diagnose narcolepsy with on
be abnormal neurologic control of the clinical grounds; however, polysomno-
sleep-wake system. Insomnia patients, gram and mean sleep latency test
they sleep for long periods of time, but (MSLT) can provide information to sup-
they do not feel refresh afterward. The port the diagnosis by detecting a mean
diagnosis is made only when no other sleep latency of less than 8 min and
medical or psychiatric condition can two or more sleep-onset REM periods
explain the hypersomnia. (SOREMPs). R-hydroxybutyrate has
been approved for the treatment of
zz Suggested Reading sleepiness and cataplexy that is associ-
55 American Academy of Sleep Medicine. ated with narcolepsy. However, tricy-
International classification of sleep clic antidepressants or serotonin
disorders, 2nd ed. Diagnostic and coding reuptake inhibitors can be used for
manual, Westchester, IL: American treatment.
Academy of Sleep Medicine; 2005.
55 Bradley WG, Daroff RB, Fenichel GM, zz Suggested Reading
et al. Neurology in clinical practice, 5th ed. 55 American academy of sleep medicine.
Philadelphia, PA: Elsevier; 2008. International classification of sleep
55 Comprehensive review in clinical neurol- disorders, 2nd ed. Diagnostic and
ogy: a multiple-choice question book for coding manual, Westchester, IL:
the Wards and Boards ©201 1 Wolters American Academy of Sleep Medicine;
Kluwer Health Lippincott Williams & 2005.
Wilkins. All rights Reserved. 55 Bradley WG, Daroff RB, Fenichel GM,
et al. Neurology in clinical practice, 5th ed.
?? 22. Regarding narcolepsy with cataplexy, Philadelphia, PA: Elsevier; 2008.
which of the following is incorrect?
A. CSF hypocretin levels are ?? 23. A 14-year-old boy wakes up in the
increased middle of the night with frightening
B. During cataplexy, muscle tone is dreams of a big monster eating his
lost and patients are hyporeflexic little sister. On awaking, he recalls the
or areflexic dream well and does not look con-
C. Sleep paralysis is frequent fused.  
D. Mean sleep latency test (MSLT) What is your diagnosis?
needs to be performed to help A. Nightmares
make the diagnosis B. REM sleep behavior disorder
E. B-hydroxybutyrate is approved for C. Sleep terrors
the treatment of narcolepsy with D. Confusional arousals
cataplexy E. Non-REM parasomnia
638 Chapter 23 · Sleep Disorders

vv Correct Answer is: A C. phase advance


This presentation of a nightmare, in D. consist of sleep similar to a new-
which is a complicated dream that born’s in stages and amount
becomes frightening toward the end. E. none of the above
Nightmares should be differentiated
from sleep terrors. Nightmares can be vv Correct Answer is: C
23 recalled by the patients in detail, it There is a gradual change between mid-
occurs in the last third of the night, and dle and late adulthood. Newborn’s sleep
patients have much less autonomic activ- is profoundly different from an adult in
ity as compared to sleep terrors. When later adulthood. Newborns spend
patients are awakened from nightmares, approximately 50% of their sleep time
they are not confused and tend to have in a stage of sleep resembling REM
good intellectual function. Sleep terrors sleep, which rapidly decreases in the
occur more commonly in children, and first year of life. Delta waves are most
patients wake up screaming or crying, prominent in the sleep of newborns,
with prominent accompanied autonomic whereas most adults late in life have
and behavioral manifestations of intense very little delta wave “deep” sleep.
fear, as well as confusion on awakening.
Sleep terrors occur in the first third of the zz Suggested Reading
night. Confusional arousals are a non- 55 Duysse DJ, Strollo PJ, Black JE, et al.
REM parasomnia occurring in children, in “Chapter 22: sleep disorders” in the
which the patient is confused following American Psychiatric Publishing Textbook
an arousal from slow-wave sleep. The of Psychiatry, 5th Ed.
boy’s clinical presentation is not consis- 55 Hales RE, Yudofsky SC, Gabbard GO,
tent with REM sleep behavior disorder or editors. American Psychiatric Publishing,
a non-REM parasomnia. 2011.

zz Suggested Reading ?? 25. An 18-year-old male complains of


55 American Academy of Sleep Medicine. fatigue, increased daytime somno-
International classification of sleep lence, and periodic sudden loss of
disorders, 3rd ed. Diagnostic and coding muscle tone. Polysomnography con-
manual, Westchester, IL: American firms the diagnosis of narcolepsy.  
Academy of Sleep Medicine; 2014. Narcolepsy is associated with which
55 Bradley WG, Daroff RB, Fenichel GM, of the following?
et al. Neurology in clinical practice, 5th ed. A. Decreased adenosine levels in the
Philadelphia, PA: Elsevier; 2008. reticular formation
Comprehensive review in clinical neurol- B. Increased discharge of noradrener-
ogy: a multiple choice question book for gic neurons in the locus ceruleus
the Wards and Boards ©2011 Wolters C. Hypothalamic dysfunction with
Kluwer Health Lippincott Williams & decreased CSF levels of orexins
Wilkins. All rights reserved. D. Increased discharge of serotoner-
gic neurons in the midbrain Raphe
?? 24. In adolescents the entire sleep period
tends to phase delay; the sleep period vv Correct Answer is: C
in later adulthood tends to: Narcolepsy is associated with low CSF
A. phase delay as well levels of the orexins and a defect in one
B. maintain sleep habits as in middle of the receptors for orexins (hypocre-
adulthood tins) in the hypothalamus. Adenosine
Sleep Disorders
639 23
induces sleep, and serotonin agonists places for the last 3 months. He has
suppress sleep. Fatal familial insomnia is occasional episodes of staring and
a progressive prion disease, character- unresponsiveness at times with facial
ized by worsening insomnia, impaired twitching. Brain MRI is normal. Rou-
autonomic and motor functions, tine EEG is normal with waking and
dementia, and death. REM sleep recorded. He has been tak-
ing ethosuximide for last 4 weeks and
zz Suggested Reading but has been having more episodes.
55 Mowzoon N, Flemming KD. Neurology He has become irritable and is having
board review: an illustrated study guide. behavioral problems at school.  
Rochester, MN: Mayo Clinic Scientific, What is your likely diagnosis?
2007. Print. A. Absence epilepsy
B. Benign rolandic epilepsy
?? 26. Which type of seizure is activated in C. Conversion disorder
REM sleep? D. Narcolepsy
A. Partial seizures E. Substance abuse
B. Tonic-clonic
C. Tonic vv Correct Answer is: D
D. Myoclonic It is unusual and abnormal to record
E. None of the above REM on a routine EEG. This suggests
either extreme sleep deprivation or a
vv Correct Answer is: E potential diagnosis of narcolepsy.
Partial seizures show a complex rela- Excessive daytime sleepiness if by far the
tionship with sleep, and the relationship most common presenting feature of nar-
may be dictated by the location of the colepsy in children (though narcolepsy is
focus. Data indicate a protective effect certainly not the most common reason
of REM sleep. Generalized tonic-clonic for excessive daytime sleepiness in chil-
seizures are associated with NREM dren). The diagnosis is not finalized until
sleep, or as in JME, soon after awaken- the additional core manifestations of the
ing. Tonic seizures are activated by disorder such as cataplexy, sleep paraly-
NREM sleep. Myoclonic seizures are acti- sis and hypnagogic hallucinations
vated soon after awakening. REM sleep develop in the teenage or early adult
appears to be protective against the years. Other subtle features of the disor-
occurrence of seizures. Seizures are der in children may include disturbed
most common in Stage 2 sleep. nighttime sleep, automatic behavior and
short-term memory problems.
zz Suggested Reading Excessively sleepiness is not typi-
55 Ng M, Pavlova M. Why are seizures rare in cally a feature of either absence or
rapid eye movement sleep? Review of the benign Rolandic epilepsy, and the EEG
frequency of seizures in different sleep in these epilepsy syndromes should
stages. show representative inter-ictal dis-
55 Epilepsy Res Treat. 2013; E pub 2013 Jun 18. charges and not epochs of REM sleep.
Derry CP and Duncan S. Sleep and epilepsy. Conversion disorder and substance
Epilepsy Behav. 2013;26(3):394–404. abuse should always be considered in
atypical cases of altered responsiveness
?? 27. You are seeing a 7-year-old boy with or excessive sleepiness, though the
a history of increasing daytime sleepi- onset at age 7 would be highly unusual
ness and falling asleep in unusual for either.
640 Chapter 23 · Sleep Disorders

zz Suggested Reading and are similar to the recent event. He


55 Macleod S, Ferrie C, Zuberi SM. takes Ox carbamazepine, but missed
Symptoms of narcolepsy in children his doses for the past few days. His
misinterpreted as epilepsy. Epileptic family history is remarkable of similar
Disord. 2005;7(1):13–7. seizures in his father, paternal uncle
and his brother.  
23 ?? 28. You are evaluating a 16-year-old female What is the best way to make
for day time sleepiness. She goes to a specific diagnosis?
sleep at 11 PM and waking at 7 AM A. Clinical presentation
and feels refreshed when she awakes. B. CSF lactate level
She does not drink caffeine. She has C. Inter-ictal epileptiform pattern on
difficulty remaining awake during her EEG
classes which has her concerned she D. MRI of the brain
may fail some classes. She has a normal E. Skin biopsy
physical neurologically exams and she
has a BMI of 24.3. She had a previously vv Correct Answer is: A
normal polysomnogram. Autosomal dominant nocturnal frontal
What is the next best step in evalua- lobe epilepsy is a focal epilepsy charac-
tion and management? terized by frequent, violent, brief seizures
A. Overnight oximetry at night, usually beginning in childhood.
B. Uvulopalatopharyngoplasty A gene for ADNFLE is on chromosome 20
C. Mandibular advancement and associated with the nicotinic acethy-
D. Overnight polysomnography choline receptor. The diagnosis is based
with multiple sleep latency test on clinical grounds as well as genetic
E. Continuous positive air pressure testing for the more common variants.
Inter-ictal frontal lobe EEG’s dis-
vv Correct Answer is: B charges are not findings specific to this
Daytime sleepiness is the most common syndrome. CSF studies and skin biopsy
symptom of narcolepsy. Given her previ- are considerations in progressive epilep-
ous normal polysomnogram as well as sies or those associated with prominent
lack of risk factors for obstructive sleep developmental delay or cognitive decline,
apnea, she should have narcolepsy or acute/subacute onset, and would not
excluded. be expected to be helpful in defining an
idiopathic epilepsy syndrome.
zz Suggested Reading
55 Scammell TE. Narcolepsy. N Engl J Med. zz Suggested Reading
2015;373(27):2654–62. 55 Kurahashi H, Hirose S. Autosomal
dominant nocturnal frontal lobe epilepsy.
?? 29. An 18-year-old male is brought to the 2002 May 16 [Updated 2015 Feb 19]. In:
emergency room by his roommate. Pagon RA, Adam MP, Ardinger HH, et al.
He was asleep on the coach, when editors. GeneReviews® [Internet]. Seattle
he suddenly screamed and thrashed (WA): University of Washington, Seattle;
violently about shaking his head. The 1993–2015. Available from: 7 http://www.
spell was 5 s long. His past medical ncbi.nlm.nih.gov/books/NBK1169/.
history is remarkable of epilepsy in 55 Rudzinski LA, Shih JJ. “The classification
the past 10 years of age. His seizures of seizures and epilepsy syndromes.”
have all occurred while he is sleeping Continuum. 2010;16(3):15–35.
Sleep Disorders
641 23
?? 30. You are seeing an 18-year-old female C. children with very frequent cata-
with the diagnosis of narcolepsy for plexy attacks should be tested for
her recent sudden spells of bilateral Niemann-Pick disease type C
skeletal muscle weakness triggered D. loss of consciousness is common
by intense, usually “positive”, emo- in cataplexy
tions such as laughter and excite- E. low or absence of cerebrospinal
ment. She has one of her spells in the fluid hypocretin levels have been
office and she became arefexic. Respi- found in more than 90% of patients
ratory muscles are spared and she did with narcolepsy with cataplexy
not loss her consciousness.
Which of the following statements vv Correct Answer is: D
is FALSE? Loss of consciousness does not happen in
A. 85% to 95% of patients with nar- cataplexy. The other statements are correct.
colepsy with cataplexy are posi-
tive for HLA-DQB1*0602 zz Suggested Reading
B. cataplexy usually develops within 55 Silber MH, editor. Sleep medicine in
a few months or years of the onset clinical practice. London: Taylor &
of sleepiness Francis; 2004. p. 92–3.
643 24

Spinal Cord

© Springer International Publishing AG, part of Springer Nature 2018


Y. M. Awaad, Absolute Pediatric Neurology, https://doi.org/10.1007/978-3-319-78801-2_24
644 Chapter 24 · Spinal Cord

?? 1. A 3-day-old baby has weakness of the and extended at the fingers (the waiter’s
right arm since birth. He was deliv- tip poistion). Total plexus injury is second
ered vaginally but it was complicated most common type and it results in long
by shoulder dystocia. Birthweight was term disability. The majority (70–95%)
4500 gm. The mother had gestational of patients make complete spontane-
diabetes. Immediately after birth his ous recovery. Evaluation using electro-
right arm was weak and areflexic, but myography, nerve conduction studies,
over time he has improved. Now, he somatosensory evoked potentials, and
24 has normal grasp and finger exten- MRI of the cervical spine can be utilized
sion, but is unable to flex at the elbow if recovery at 3 months is not complete,
or abduct at the shoulder and the although the best timing of these stud-
biceps reflex is absent. ies is controversial. The primary goal of
In addition to range of motion exer- management is to maintain passive range
cises, what is your recommendation of motion, maintain joints, and muscle
at this time? strength. Indications for surgical explora-
A. Observation only tion and reconstruction of the brachial
B. MRI of the cervical spine plexus include failure of recovery of elbow
C. Splint to the elbow to maintain flexion and shoulder abduction from the
flexion 3rd to the 6th month of life. It might take
D. Nerve conduction velocity study up to 2 years for complete recovery.
E. Surgical exploration for nerve
grafting zz Suggested Reading
55 Zafeiriou DI, Psychogiou K. Obstetrical
vv Correct Answer is: A brachial plexus palsy. Pediatr Neurol.
Brachial plexus palsy is a flaccid paresis 2008;38:235–42.
of an upper extremity due to traumatic
stretching of the brachial plexus dur- ?? 2. A 10–year-old girl came with a 3 days’
ing birth. The incidence is 0.38–3 per history of weakness in the lower extrem-
1000 live births. The most significant ities and bladder control difficulty. She
risk factor is high birth weight, and it is had an episode of loss of vision in her
3 times more frequently specially with right eye one year ago which resolved
birth weight > 4500 gm. Breech delivery in several weeks. She has a mild afferent
is another risk factor. Maternal fac- pupillary defect and 20/80 vision in her
tors include diabetes mellitus, obesity, right eye. Tendon reflexes in her lower
excessive weight gain, age > 35 years, extremities were increased and she had
maternal pelvic anatomy, and primiparity extensor plantar responses. Sensory
are also risk factors. Shoulder dystocia level was not appreciated.
is a strong predictor for brachial plexus Which of the following antigens
injury. Clavicle and humerous fractures, is useful in the diagnosis of this
facial nerve injury, cephalhematoma, and patient?
torticollis are usually associated injuries. A. Acetylcholine receptor (AChR)
Injuries include neurapraxia, neuroma, B. Aquaporin 4 channel (AQP4)
rupture, and avulsion. Erb’s palsy (Upper C. Voltage gated calcium channel
plexus injury (C5 and C6) is the most com- (VGCC)
mon type. In these patients, the adducted D. Alpha-4-beta integrin (ά4β1)
and internally rotated at the shoulder, E. Anti-Muscle specific kinase anti-
extended at the elbow, flexed at the wrist, body (MuSK)
Spinal Cord
645 24
vv Correct Answer is: B Most likely he has:
The integrin protein is important in cell A. Anterior spinal artery infarction
signaling in glial neuronal interactions. B. Ependymoma
The AChR is the primary ion channel in the C. Epidural abscess
neuromuscular junction and is important D. Syringomyelia
in impulse transmission as is the calcium E. Multiple sclerosis
channel. The MuSK antibody is associated
with a form of Myasthenia. Although the vv Correct Answer is: D
role of the aquaporin channel is incom- Syringomyelia is a spinal cord condition
pletely understood, it is associated with which is secondary to of other congenital
neuromyelitis optica and distinct from abnormalities (e.g., tethered cord or hydro-
multiple sclerosis. cephalus) or trauma. It is a longitudinal
“hole” in the spinal cord. The shape and
zz Suggested Reading size of the “hole” generally determines
55 Roemer SF, et al. Pattern-specific loss of the symptoms and signs that result. A
aquaporin-4 immunoreactivity distin- “suspended sensory loss”, particularly one
guishes neuromyelitis optica from that is unilateral, like the “cape” sensory
multiple sclerosis. Brain 2007;130: changes described in this case, is character-
1194–205. istic of a syrinx. The MRI scan shown dem-
55 Kira J. Neuromyelitis optica and Asian onstrates the 2–3 vertebral level extent of
phenotype of multiple sclerosis. Ann N Y this intrinsic cord lesion.
Acad Sci 2008;1142; 58–71.
zz Suggested Reading
?? 3. A 10-year-old boy is referred to neu- 55 Vandertop WP. Syringomyelia.
rologist for constant tingling over Neuropediatrics. 2014;45(1):3–9.
his right shoulder and the lateral
aspect of his proximal arm. He has ?? 4. An 8-year-old boy presents with
mildly diminished pain and temper- gradually progressive difficulty run-
ature sense in the same distribution ning for the past 2 years. He referred
as the tingling. Spinal MRI is shown the problem to intermittent low back
below: pain, which is worsened by activity
but does not radiate. He has also had
intermittent urinary incontinence for
the last 4 months. On examination, he
has normal bulk but mildly increased
tone in his distal lower extremities
and mild weakness in ankle dorsiflex-
ion bilaterally. He has normal sensory
examination. Deep tendon reflexes
are 2+ in the upper extremities and
3+ in the lower extremities. Plantar
responses are flexor bilaterally. Rec-
tal tone is normal. He has lumbar
levoscoliosis and a sacral heman-
gioma. He elevates his left shoulder
..      Fig. 24.1 Siringomielia. Own work. Luciobgomes. and hip while walking but maintains
Wikipedia a narrow base. Spinal MRI shows a
646 Chapter 24 · Spinal Cord

low-lying conus m­ edullaris at L3 and progression after surgical tethering was


an L4 hemi-vertebra with associated noted in one cohort in long term follow
levoscoliosis. Surgical treatment is up. However, there was improvement in
most likely to lead to improvement muscle strength, gait, and spasticity. The
in the following symptoms except? role of surgery in asymptomatic patients
A. Scoliosis remains less clear.
B. Weakness
C. Bladder function zz Suggested Reading
24 D. Pain 55 Bowman RM, Mohan A, et al. Tethered
E. Spasticity cord release: a long term study in 114
patients. J Neurosurg Pediatr.
vv Correct Answer is: A 2009;3(3):181–7.
Tethered cord syndrome (TCS) is a con-
stellation of motor and sensory findings ?? 5. A 6-year-old girl came to ER after
that are attributable to excessive tension she was struck by a car while she
on the spinal cord. The majority of cases was crossing the street. Paramedics
are associated with spinal dysraphism; found her to be awake, but distraught
however, TCS can also occur when an and complaining of neck pain. They
intradural process (e.g. abnormal thick- immobilized her head and neck in
ened, shortened filum terminale, lipoma neutral position before transferring
of the filum, fibrous adhesions) interferes her too ER. On examination, she
with normal ascension of the conus has normal breathing l, normal vital
with respect to the vertebral column. It signs, and moving all four extremities
can present at any age. Infants usually equally. Radiographs of the cervical
have cutaneous stigmata of spina bifida spine show abnormal widening of the
occulta, such as nevi, lipomas, tufts of interspace at C1-C2.
hair, hemangiomas, and dermal sinuses. Which of the following is the ideal
Other alarming signs include urinary next step in management?
dribbling, anorectal malformations, A. Emergency tracheostomy
orthopedic deformities of the lower B. Continued immobilization of the
extremities, and scoliosis. Children and head and neck, careful observa-
adolescents may be asymptomatic or tion, and neurosurgical consulta-
may present with slowly progressive tion.
motor and sensory deficits. The clinical C. fluoroscopy Lumbar puncture to
picture may also include varying degrees exclude subarachnoid hemor-
of weakness; patchy sensory deficits; rhage
scoliosis; non-dermatomal pain in the D. Ophthalmology consultation for
low back, perineum, and/or legs; bladder dilated fundus examination
or less frequently bowel incontinence; E. Flexion and extension views of the
orthopedic deformities of the lower neck under
extremities; and gait disturbances. MRI
is the study of choice, as it can detect vv Correct Answer is: B
both the level of the conus and the cause Cervical spine (C-spine) injuries in pre-­
of the tethering. Surgical untethering pubertal children are relatively uncom-
should be done in symptomatic patients, mon in comparison to older age groups.
which results in pain relief in almost all In young children, C-spine injuries are
cases and stabilization of neurologic usually occurring at the cranio-cervical
decline in up to 90% of patients. Scoliosis junction or upper vertebral segments
Spinal Cord
647 24
C1-C4. Severe disruption of ligaments What is the most appropriate treat-
or other soft supporting tissues without ment at this time?
obvious abnormalities of the vertebrae A. IVIG 0.5 g/kg I.V. daily × 4 days
on radiographs may occur in children. B. Methylprednisolone 500 mg
However, selected MR sequences are I.V.B.I.D × 5 days
sensitive to such soft tissue injuries, if C. Prednisone 40 mg P.O. daily with a
indicated. Children who are unresponsive, slow taper
intoxicated, have abnormal findings on D. Cyclophosphamide 800 mg/m2
neurological examination, or neck pain I.V. every 4 weeks
should remain immobilized until C-spine E. Interferon-β − 1a 30 μg I.M. every
stability has been determined by the week
appropriate studies. The use of corticoste-
roid as neuroprotection following spinal vv Correct Answer is: B
injury remains contentious. Transverse myelitis (TM) is a heteroge-
neous disorder. It occurs as an idiopathic
zz Suggested Reading or secondary to a wide array of inflam-
55 Platzer P, Jaindl M, Thalhammer G, et al. matory (post-infectious, demyelinat-
Cervical spine injuries in pediatric ing), metabolic, and vascular processes.
patients. J Trauma. 2007;62:389–96. Inflammatory demyelinating lesions,
55 Hall ED, Springer JE. Neuroprotection and as seen in clinically isolated syndrome,
acute spinal cord injury: a reappraisal. multiple sclerosis, and neuromyelitis
NeuroRx. 2004;1:80–100. optica, are the most common cause of
TM. Acute treatment for those demy-
?? 6. A 14-year-old boy came to the elinating conditions is a high-dose,
emergency department with pro- intravenous corticosteroids for 3–5 days.
gressive weakness for 10-h while A subsequent oral steroid taper over
he was seated in his classroom. He 4–6 weeks is recommended for patients
described it as a stinging sensation with acute disseminated encepha-
in his arms that progressed to numb- lomyelitis. Long-term therapy with
ness and weakness to the level of immunosuppressants may decrease the
his upper chest. He has not urinated relapse rate for patients with NMO. Non-­
for several hours and he wakes only demyelinating forms of transverse
with assistance. He has moderate myelitis must be treated according to
hypertonia in his upper extremities the underlying etiology, e.g. paren-
and his lower extremities are flac- teral cyanocobalamin (vitamin B12)
cid. Formal strength testing reveals supplementation for subacute combined
(R/L): biceps 4/4, triceps 3/3, and wrist degeneration.
extension 4/4, with all other muscle
groups tested being grade 0. He has zz Suggested Reading
a sensory level at T2, and deep ten- 55 Brinar VV, Habek M, et al. Current
don reflexes are absent throughout. concepts in the diagnosis of transverse
MRI of the spine shows an ill-defined myelopathies. Clin Neurol Neurosurg.
intramedullary hyperintensity extend- 2008;110:919–27.
ing from C5 to T2 without significant 55 Pohl D. Epidemiology, immuno-­
cord expansion or enhancement. MRI pathogenesis and management of pediatric
of the head is normal. CSF studies are central nervous system inflammatory
normal, and no oligoclonal bands are demyelinating conditions. Curr Opin
detected. Neurol. 2008;21:366–72.
648 Chapter 24 · Spinal Cord

?? 7. A 16 yr. old girl presents with hands for the last 6 months. Her imaging is
tingling after a “deep belly laugh” shown below:

24

2006 2014

..      Figs. 24.2 and 24.3 Imágenes cortesía de la Chiari Sección del Filum Terminale. Own work. Chiari &
& Scoliosis & Syringomyelia Foundation. Las imágenes Scoliosis & Syringomyelia Foundation
obtenidas por MR se tomaron antés y después de la

What is the most likely diagnosis? in the syrinx indicating a congenital etiol-
A. Klippel-Feil syndrome ogy. Chiari 1 malformations is a deviation
B. Transverse myelitis of the cerebellar tonsil down through the
C. Platybasia formen magnum. When associated with
D. Chiari 1 malformation with syrin- a syringomyelia, the Chiari is felt to be
gomyelia symptomatic and surgical intervention
E. Intramedullary cervical spinal is indicated. Other common symptoms
cord tumor could include occipital headaches, early
morning headache, and headaches
vv Correct Answer is: D brought on by coughing or sneezing.
The “deep belly laugh” is a Valsalva Patients may experience difficulty swal-
maneuver which caused paresthesias lowing due to decreased gag and paras-
secondary to the dilated space within the thesias of the shoulders and hands.
cervical cord or syrinx. The MRI image on Klippel-Feil syndrome is a triad of short
the top shows the presence of septations neck, low posterior hair line and limited
Spinal Cord
649 24
neck movement. It involves fusion of vv Correct Answer is: E
cervical vertebrae. Platybasia involves Tethered cord syndrome (TCS) is a spec-
flattening of the base of the skull, fre- trum of congenital anomalies secondary
quently seen in patients with bony to an abnormally low position of the
dysplasias. Transverse myelitis was not conus medullaris that may lead to neu-
shown on imaging which would appear rological, musculoskeletal, urological, or
patchy increased T2 signal in the cord. This gastrointestinal abnormalities. Most com-
image shows the syringomyelia which is monly TCS is associated with spinal dysra-
similar in signal to CSF and not consistent phism. The signs and symptoms correlate
with transverse myelitis. with the radiological findings showing the
conus medullaris is anatomically lower
zz Suggested Reading than the L2 vertebra or below the L 1–2
55 Fernandez A, et al. Malformations of the disc space. However, some patients with
craniocervical junction (chiari type I and signs and symptoms of TCS with a normal
syringomyelia: classification, diagnosis and conus medullaris position. The clinical
treatment). BMC Musculoskelet Disord. presentation is broad and different with
2009;10(Suppl 1). age at presentation as well as underlying
cause. Toddlers and young children have
?? 8. A 12-year-old boy complains of low clumsy gait, regression in motor function
back pain for the past 6 months or bladder control, or arrest in achieve-
and developed recently problems ment of developmental milestones. Foot
with urinary incontinence, often deformities, scoliosis, non-segmental sen-
times occurring when he is walking sory loss and pain in the back and lower
upstairs. He was seen by a urologist, extremities are also common features.
and urodynamic studies showed In the late childhood and teenage years,
detrusor hyperreflexia. Past medi- pain in the lumbosacral region, perineum,
cal history is otherwise unremark- and legs is the predominant symptom.
able. Review of systems is pertinent Progression of scoliosis, constipation
for constipation. His vital signs are and urinary incontinence are common.
normal. General examination shows Urodynamic studies usually demonstrate
mild thoracolumbar scoliosis and a detrusor hyperreflexia. MRI demonstrates
small capillary hemangioma overly- a thickened filum terminale (>2 mm) and
ing the lumbar spine. Mental status, a conus positioned below the L2 level.
cranial nerves, strength and sensa- Supine and prone MRI demonstrate lack of
tion are within normal limits. Deep motion of the spinal cord. Surgical deteth-
tendon reflexes are 2+ in the upper ering of the cord is an effective treatment,
extremities and 3+ in the lower particularly if performed early.
extremities. Plantar responses are
extensor bilaterally. He has a normal zz Suggested Reading
gait but he has trouble with tandem 55 Hertzler DA, DePowell JJ, Stevenson CB,
gait. Mangano FT. Tethered cord syndrome: a
Which of the following is most likely review of the literature from embryology
to be found on spine MRI? to adult presentation. Neurosurg Focus.
A. Cauda equina compression 2010;29:1–9.
B. Diastematomyelia
C. Syringomyelia ?? 9. A 12-year-old girl complained of pain
D. Chiari malformation between her shoulder blades during
E. Tethered spinal cord school. Later that day she had sudden
650 Chapter 24 · Spinal Cord

loss of strength in both arms and decreased pin and temperature


both legs, she could not stand and appreciation below C-4 spinal level
she was carried out of the shower. In with intact vibratory, position, and
the emergency department, a lumbar touch sensation. She has no reflexes
puncture was performed and cere- in the flaccid upper extremities and
brospinal fluid was normal except for increased reflexes in the lower extrem-
24 lymphocytes/mm3. Her condition ities with bilateral Babinski. She has a
remains unchanged and a week later neurogenic bladder and bowel. Her
24 her examination shows the following; spine MRI of the spine are shown.

a b c

..      Fig. 24.4 T2-weighted sagittal a and transverse c and a reduced apparent diffusion coefficient (ADC)
MRI 30 h after symptom onset show a hyperintense 0.49 × 10−5 cm2/s on calculated maps of the ADC e,
lesion with edematous expansion in the central portion confirming the ischemic pathology. (Neuroimages.
of the lumbar cord and the conus medullaris. Diffusion- Neurology June 13, 2000 vol. 54 no. 11 2195. Figures
weighted MRI demonstrates strong hyperintensity on Only Free)
sagittal b and transverse d images (b = 1000 s/mm2)

What is the most likely diagnosis? vv Correct Answer is: C


A. Neuromyelitis optica The classical presentation of anterior spi-
B. Acute transverse myelitis nal artery infarction is:
C. Spinal cord infarction 55 Sudden onset of transient back or neck pain.
D. Multiple sclerosis 55 Rapidly progressive flaccid and arreflexic
E. Viral infection of the spinal cord paraplegia or tetraplegia in acute period.
Spinal Cord
651 24
55 Spasticity below the lesion with extensor
plantar responses will follow.
55 Loss of pain and temperature below
level.
55 Preservation of proprioception, light touch
and vibration sensation.
55 Autonomic dysfunction and bladder
dysfunction.
55 Painful dysesthesias below the level of the
lesion.

The sensory changes are what separate


this from demyelinating disease such as
Devic’s neuromyelitis optica, multiple
sclerosis and acute transverse myelitis.
Viral infections of the spine (poliomyelitis)
usually spare the sensory system but ..      Fig. 24.5 Image of a Type 1 Arnold-Chiari
frequently is associated with pain in the Malformation. The cerebellum has descended 7 mm
affected extremities. and there are herniated cerebellar tonsils into the
foramen magnum. Basket of Puppies
zz Suggested Reading
55 Love BB. Spinal cord vascular syndromes, A. Hemicord syndrome
In: Noseworthy JH, editor. Neurological B. Spinal stenosis
therapeutics principles and practice. 2nd C. Syringomyelia
ed. Informa healthcare; 2006. D. Spinal cord infarction
E. Transverse myelitis
?? 10. A 13-year-old female has been seen
for headaches which began one a vv Correct Answer is: C
year ago. They occur 3–4 days per The patient has a Chiari I malformation.
week, are located in the occipital area, Chiari malformation was first described
and last for several hours unless she by Hans Chiari in 1891. It is defined as cer-
takes Tylenol. Occasionally they are ebellar tonsil extension below the foramen
triggered by coughing or sneezing. magnum. Severity ranges from a morpho-
She has no associated nausea, vomit- logical asymptomatic finding to a clinical
ing, sensitivity to light or sound, and Chiari. There appears to be evolution of
dizziness but says sometimes objects the condition over time, the number of
look like they are jiggling when she symptomatic patients increasing with age.
has the pain. Past medical history is Patients with this malformation have the
otherwise normal and family history risk of developing a dilation of the central
is noncontributory. Examination is canal (syrinx or hydromyelia). Abnormal
normal except for hyperreflexia in the CSF flow at the level of the foramen mag-
lower extremities. num is due to underdevelopment of the
What is the most common second- mesodermal occipital somite, resulting in
ary complications could have hap- a smaller posterior fossa. How this results
pened? in dilation of the central canal is not well
652 Chapter 24 · Spinal Cord

understood? Neurosurgical decompres- vv Correct Answer is: E


sion of the posterior fossa via a suboc- Brachial plexus birth palsy (BPBP) occurs
cipital craniectomy, often combined with in 0.4–4 per 1000 live births. Perinatal risk
laminectomy to the level of the tonsillar factors include macrosomia, multiparous
herniation, with duraplasty or dural scor- pregnancies, and previous deliveries result-
ing is the optimal treatment of the condi- ing in BPBP, prolonged labor, breech deliv-
tion. Surgical complication rate is much ery, and assisted and difficult deliveries.
higher if the subarachnoid membrane is 50% of patients have 1 or more of these risk
24 invaded. factors, which demonstrate that the etiol-
ogy of BPBP is not yet fully known. In chil-
zz Suggested Reading dren, the extent of brachial plexus injury
55 Navarro R, Olavarria G, et al. Surgical differs greatly, and therefore the prognosis
results of posterior fossa decompression does as well. Between 66% and 92% have a
for patients with Chiari I malformation. mild injury and recover completely, usually
Childs Nerv Syst. 2004;20:349–56. within the first 2 months. Those with only
55 Eule JM, Erickson MA, O’Brien MF, C5-C6 involvement have classic Erb’s palsy
Handler M. Chiari I malformation (46%) and have the most favorable prog-
associated with syringomyelia and nosis. The most severely affected babies
scoliosis. A 20-year review of surgical and have a flail extremity, Horner’s syndrome,
nonsurgical treatment in a pediatric and hemi-­diaphragmatic paralysis. Early
population. Spine 2002;27:1451–5. evaluation and intervention is warranted
for babies with a more severe form of BPBP,
?? 11. One week-old baby boy born at full since spontaneous recovery is unlikely. In
term has weakness of the left arm this case, patient has a mild form, so the
since birth. No movements has been best management is observation with
noted at the shoulder and does not gentle range of motion exercises to prevent
flex at the elbow, but he has active contractures.
elbow extension. Wiggle his fingers
and has a grasp reflex in the hand. No zz Suggested Reading
respiratory difficulty was reported. His 55 Hale HB, Bae DS, Waters PM. Current
pupils are symmetric and there is no concepts in the management of brachial
ptosis. He has normal tone and move- plexus birth palsy. J Hand Surg
ments in the other limbs. 2010;35A:322–31.
What is your next step in manage-
ment at this time? ?? 12. An 8-year-old boy came to your office
A. Immobilize the right arm in a with a history of difficulty running
splint that has been gradually progressive
B. MRI of the cervical cord and left over the past 2 years. He attributes this
shoulder problem to intermittent low back pain,
C. Nerve conduction and EMG stud- which is worsened by activity but does
ies of the left arm not radiate. Also, in the past 6 months
D. Micro neurosurgical intervention he had intermittent urinary inconti-
E. Physiotherapy for gentle range of nence. He has normal bulk but mildly
motion exercises increased tone in his distal lower
Spinal Cord
653 24
extremities and mild weakness in The clinical picture may also include vary-
ankle dorsiflexion bilaterally. Sensory ing degrees of weakness; patchy sensory
examination is normal. Deep tendon deficits; scoliosis; non-dermatomal pain
reflexes are 2+ in the upper extremi- in the low back, perineum, and/or legs;
ties and 3+ in the lower extremities. He bladder or less frequently bowel incon-
has bilateral flexor plantar responses. tinence; orthopedic deformities of the
Rectal tone is normal. He has lumbar lower extremities; and gait disturbances.
levoscoliosis and a sacral hemangioma. MRI is the study of choice, as it can detect
He elevates his right shoulder and hip both the level of the conus and the cause
while walking but maintains a narrow of the tethering. Surgical untethering
base. MRI spine shows a low-lying should be done in symptomatic patients,
conus medullaris at L3 and an L4 hemi- which results in pain relief in almost all
vertebra with associated levoscoliosis. cases and stabilization of neurologic
Surgical treatment is most likely to decline in up to 90% of patients. Scoliosis
lead to improvement in which of this progression after surgical tethering was
patient’s symptoms? noted in one cohort in long term follow
A. Pain up. However, there was improvement in
B. Spasticity muscle strength, gait, and spasticity. The
C. Scoliosis role of surgery in asymptomatic patients
D. Bladder function remains less clear.
E. Weakness
zz Suggested Reading
vv Correct Answer is: A 55 Bowman RM, Mohan A, et al. Tethered
Tethered cord syndrome (TCS) is a con- cord release: a long term study in 114
stellation of motor and sensory findings patients. J Neurosurg Pediatr.
that are attributable to excessive tension 2009;3(3):181–7.
on the spinal cord. The majority of cases 55 Lew SM, Kothbauer KF. Tethered cord
are associated with spinal dysraphism; syndrome: an updated review. Pediatr
however, TCS can also occur when an Neurosurg. 2007;43:236–248.
intradural process (e.g. abnormal thick-
ened, shortened filum terminale, lipoma ?? 13. A 12-year-old boy presents with
of the filum, fibrous adhesions) interferes walking difficulty, which has been
with normal ascension of the conus progressive over the last few months.
with respect to the vertebral column. It He has been dragging his feet on the
can present at any age. Infants usually ground and has tripped and fallen
have cutaneous stigmata of spina bifida several times. Recently, he com-
occulta, such as nevi, lipomas, tufts of hair, plained of urinary urgency and has
hemangiomas, and dermal sinuses. Other had occasional incontinence. Workup
alarming signs include urinary dribbling, by his pediatrician was negative
anorectal malformations, orthopedic including routine blood work and
deformities of the lower extremities, and UA. He has increased tone and brisk
scoliosis. Children and adolescents may be deep tendon reflexes in both lower
asymptomatic or may present with slowly extremities with spastic gait. Spine
progressive motor and sensory deficits. MRI shown below.
654 Chapter 24 · Spinal Cord

spinal cord to the sacrum. In adults,


the spinal cord ends around the level of
L1. Tethered cord syndrome occurs in
association with the spectrum of spinal
dysraphisms which include spina bifida
occulta, meningocele, myelomeningocle,
diastematomyelia, intraspinal lipoma,
sacral agenesis and fibrosis of the filum
24 terminale, leading to abnormal fixation
of the lower end of the spinal cord. The
different rate of growth of the vertebral
column and the spinal cord leads to the
stretching of the spinal cord. In milder
cases, symptoms might appear in child-
hood with progressive stretching of the
cord. However if severe it can be symp-
tomatic at birth. Symptoms can include
myelopathy with upper motor neuron
type weakness with increased tone and
brisk reflexes as well as bladder dysfunc-
tion. Neurosurgical de-­tethering proce-
dures can improve the symptoms.

zz Suggested Reading
55 Swaiman KF, Ashwal S, Ferriero
DM. Pediatric neurology principles and
practice, 4th ed. Philadelphia: Mosby
Elsevier; 2006.
..      Fig. 24.6 Tethered Cord. American Academy of
Neurology Institute, produced by permission
55 Gruener G, Biller J. Spinal cord anatomy,
localization and overview of spinal cord
syndromes. Continuum Lifelong Learning
What is the most likely diagnosis? Neurol. 2008;14(3):11–35.
A. Spinal Meningioma
B. Subacute Combined Degenera- ?? 14. A 4-year-old girl has been admit-
tion of the Cord ted to the hospital for evaluation of
C. Tethered Cord weakness 3 days. Initially weakness
D. Spinal AVM involved her left arm but one day
E. Transverse Myelitis later it worsened to involve the right
arm and both legs. She complains of
vv Correct Answer: C pain in the left shoulder. Past medical
The MRI shows tethered cord with low history is remarkable for fever and
lying conus ending much lower than cough 2 week ago while shopping
normal S2-S3 level and also rules out the with her grandmother; symptoms
other options listed above. The cauda resolved within 2 days. No medication
Equine includes the terminal spinal was taken. Her examination is remark-
cord, spinal roots from T12 to S5 and able for 3/5 power in all limbs. It is
the filum terminale, which is a fibrous difficult to elicit dep tendon reflexes
band that anchors the lower tip of the in the upper extremities and are brisk
Spinal Cord
655 24
in the lower extremities. She has Diagnosis of idiopathic ATM is estab-
plantar flexor response and sensation lished according to criteria which include
is intact. MRI of the cervical spine is a clinical picture of bilateral symptoms
obtained and a sagittal T2-weighted and signs attributable to spinal cord
sample is shown. disease, existence of focal cord inflam-
mation by MRI and cerebrospinal fluid
studies, and exclusion of other diseases,
especially cord compression and disease-­
associated ATM, (e.g, Sjögren disease).
Idiopathic ATM patients have motor,
sensory and autonomic dysfunctions
with acute or subacute onset, progress-
ing to nadir in 4 h to 21 days, with full
or partial recovery in most cases. Acute
treatment to stop inflammation with cor-
ticosteroids is required as well as early
appropriate symptomatic therapies.
Recent discovery of novel biomarkers
has set apart specific inflammatory and
autoimmune myelopathies, namely para-
neoplastic myelitis and neuromyelitis
optica spectrum diseases, the diagnosis
of which is important to establish thera-
..      Fig. 24.7 Transverse myelitis, this 53 year old peutic strategies.
woman rapidly developed neurological symptoms,
which resolved over the course of weeks. Followup
MRI three months later demonstrated significant
zz Suggested Reading
reduction is the size of the lesion. It is felt to be 55 Sá MJ. Acute transverse myelitis: a
consistent with transverse myelitis. (7 http:// practical reappraisal. Autoimmun Rev.
radiopaedia.org/cases/transversemyelitis and 2009;9(2):128–31.
7 http://radiopaedia.org/uploads/radio/0001/3690/
TM_01.jpg. Frank Gaillard. Wikipedia)
?? 15. Physical complications of spina bifida
include which of the following?
A. Allergy to latex
Would will be your initial treatment? B. Leg weakness and paralysis
A. acyclovir C. Club foot
B. high-dose methylprednisolone D. (B) and (C)
C. plasmapheresis E. (A) (B) and (C)
D. intravenous immunoglobulin
­infusions vv Correct answer: E
E. surgical drainage of syrinx 68% of children with spina bifida have
latex allergy, ranging from mild to life-­
vv Correct Answer: B threatening. The common use of latex in
The presentation and MRI findings are medical facilities makes this a particularly
consistent with acute transverse myelitis serious concern. Leg paralysis or weakness
(ATM). ATM, a subgroup of different and orthopedic abnormalities such as
conditions that cause transverse spinal club foot, hip dislocation or scoliosis, and
cord syndromes, is an inflammatory and urinary and bowel control problems are
usually idiopathic spinal cord condition. frequently present.
656 Chapter 24 · Spinal Cord

?? 16. A 14-year-old girl presents with pro- lower extremities. She has a sensory
gressive lower extremity numbness level at T2. Sagittal MRI is shown
and weakness bilaterally, walking below.
difficulty for 2 months. She reports Which of the following is the most
worsening over the past 3 days, when likely diagnosis?
she woke up this morning she was A. Spinal cord infarct
unable to move her legs and felt B. Acute transverse myelitis
numb from her upper chest down. C. Epidural hematoma
24 Examination shows mild weakness D. Cord compression from a meta-
and areflexia in both upper extremi- static tumor
ties, and significant weakness with E. Meningioma with spinal cord
spasticity and hyperreflexia in the compression

..      Figs. 24.8 and 24.9 Meningeom im Spinalkanal MRT. Links T1 sagittal mit Kontrastmittel, rechts T2 coronar.
Own work. Hellerhoff. Wikipedia

vv Correct answer: E very gradually and may compensate for the


MRI shows an intradural extramedullary degree of compression because they grow
tumor compressing the spinal cord. The slowly. Sometimes large tumors causing
tumor is located at the C7 to T1 vertebral significant compression without prominent
level and is dural based. Meningiomas manifestations on clinical examination.
are one of the most common intradural The presentation is not acute to subacute
extramedullary tumors. They are non-glial so the patient does not have transverse
neoplasms that are usually benign and can myelitis nor a spinal cord infarct, with MRI
cause neurologic problems due to com- findings showing a parenchymal cord
pression. Neurologic symptoms progress lesion and not an extramedullary mass.
Spinal Cord
657 24
A spinal cord infarct may be sudden in nificant motor and cognitive delay
onset and usually affects the anterior cord even if he has developed normally
(anterior spinal artery), causing primarily up until the age of 3 years
bilateral weakness and pain and tempera- C. This tuft of hair signifies a possible
ture loss rather than vibration and position underlying defect in the posterior
loss (dorsal columns are relatively spared). bony component of the vertebral
Acute transverse myelitis progresses over column
hours or a few days rather than months. A D. This tuft of hair signifies the pres-
metastatic tumor has a subacute presenta- ence of a myelomeningocele
tion with prominent back pain. Because E. On the basis of the finding of this
metastatic tumors develop more rapidly, tuft of hair, it can be concluded
the patient would have more symptoms that abnormalities in the spinal
and neurologic deficits than this patient cord will definitely be seen on
has for the degree of compression seen imaging
on this MRI. An epidural hematoma also
presents rapidly with an acute compres- vv Correct answer is: D
sive myelopathy syndrome manifesting as The patient’s history and examination
spinal shock, usually with no evidence of are consistent with spina bifida occulta.
spasticity or hyperreflexia on presentation. This is a defect in the bony components
along the posterior aspect of the vertebral
zz Suggested Reading column. It can be asymptomatic, but an
55 Comprehensive review in clinical neurol- abnormal conus medullaris and filum
ogy: a multiple choice question book for terminale are possible. The presence of
the wards and boards © 2011 Wolters a hair tuft, implying underlying spina
Kluwer Health Lippincott Williams & bifida occulta, but not necessarily imply
Wilkins. All rights reserved. impending cognitive or motor delay. In
fact, when early neurologic development
?? 17. A 5-month-old baby is brought to his is normal, it will typically continue to
PCP for a well-child visit. PCP found be so. However, associated neurologic
on examination that the child has a deficits may portend future neurologic
small, midline tuft of hair over the impairment. When there is associated neu-
lower lumbar region. The rest of his rologic deficits in a child with a tuft of hair
physical examination is otherwise over the lumbar region but with no other
entirely normal. On annual follow-up evidence of neural tube defect, the dis-
at 3 years of age, the child continues order is named occult spinal dysraphism.
to have normal development and a In this case, a variety of developmental
normal neurologic examination. abnormalities may be seen involving the
What statement regarding this child spinal cord or roots and posterior fossa,
is correct? and associated findings may include
A. On the basis of the finding of this dermoid or epidermoid cysts, intraspinal
tuft of hair, it can be concluded or cutaneous lipomas, and tethered cord.
that he will eventually have sig- Diastematomyelia, or dividing of the spi-
nificant cognitive delay, even if he nal cord, may also be seen. Rarely, a sinus
has developed normally up until tract connects the dura with the surface
the age of 3 years of the skin. In occult spinal dysraphism,
B. On the basis of the finding of this neurologic manifestations vary widely and
tuft of hair, it can be concluded may range from mild motor deficits and
that he will eventually have sig- ankle hyporeflexia to bowel and bladder
658 Chapter 24 · Spinal Cord

dysfunction, sensory loss, and paraparesis this acute presentation. Lumbar puncture
or paraplegia. Although patients may be should follow imaging. Electromyography
initially asymptomatic, these neurologi- does not have a role in the diagnosis of
cal deficits can develop suddenly and be transverse myelitis.
irreversible.
zz Suggested Reading
?? 18. You are evaluating a previously healthy 55 Wingerchuk DM, Weinshenker BG. Acute
17-year-old girl presents to the ED disseminated encephalomyelitis, trans-
24 with difficulty walking for 2 days. She verse myelitis, and neuromyelitis optica.
reports having difficulty moving or Continuum (Minneap Minn). 2013;19(4
feeling her legs. Also she has difficulty Multiple Sclerosis):944–67
urinating. On neurological exam,
she has 3/5 weakness in both legs, ¾
reflexes in the bilateral patella reflexes, ?? 19. The most severe form of spina bifida
and 5 beats of clonus in the right Achil- is which of the following:
les reflex. There is a pinprick sensation A. Spina bifida occulta
loss at the level of the umbilicus. B. Myeloschisis
What is the next best step in man- C. Meningocele
agement? D. Myelomeningocele
A. Electromyography examination E. Arnold-Chiari II malformation
B. MRI brain with and without gado-
linium vv Correct answer: B
C. MRI thoracic spine with and with- In myeloschisis, the affected area is a flat-
out gadolinium tened, plate-like mass of nervous tissue
D. MRI cervical spine with and with- with no overlying membrane.
out gadolinium The exposure of these nerves and
E. Lumbar puncture tissues makes the infant more prone to
life-­threatening infections such as
vv Correct answer is: C meningitis. Spina bifida occulta is the least
The clinical history and exam is suggestive severe form and may be asymptomatic.
of transverse myelitis. Given the sensory Many individuals with spina bifida have an
level at T10, a thoracic localization should associated abnormality of the cerebellum,
be deduced. MRI of the brain and cervical called the Arnold Chiari II malformation in
spine would be useful in evaluating for which, the back portion of the brain is
possible underlying multiple sclerosis, but displaced from the back of the skull down
would not be used for the treatment of into the upper neck.
659 25

Static Encephalopathy/
Cerebral Palsy/
Rehabilitation

© Springer International Publishing AG, part of Springer Nature 2018


Y. M. Awaad, Absolute Pediatric Neurology, https://doi.org/10.1007/978-3-319-78801-2_25
660 Chapter 25 · Static Encephalopathy/Cerebral Palsy/Rehabilitation

?? 1. A 2-year-old premature boy, he was The reason for this abnormality is not clear,
26 weeks of gestation and his birth however the common marked loss of white
weight was 750 grams. He had retro- matter volume would strongly suggest
lental fibroplasia but did not require ischemia occurred and most likely contrib-
laser surgery. At 2 years, he has sever uted. Others suggest parenchymal hemor-
global developmental delay and he is rhage as a main factor but the variety of
unable to walk or talk. On examination, injury observed suggest other possibilities,
he has remarkable hypotonia, athetois mainly ischemia.
and dysmetria. Brain MRI is shown.
zz Suggested Reading
55 Bodensteiner JB, Johnsen SD. MRI
25 findings in children surviving extremely
premature delivery and extremely low
birth weight with cerebral palsy. J Child
Neurology. 2006;21(9):743–7.

?? 2. A 15-month-old girl with decreased


usage of her right hand in the last
4 months of age which was noted by
her family. Her pediatrician diagnosed
her as a right hemiparesis at 1 year of
age. The mother had no complication
..      Fig. 25.1 Head MRI showing deep intracerebral during pregnancy and there was no
hemorrhage due to bleeding within the cerebellum, history of prenatal or perinatal prob-
approximately 30 h old: same patient as TAC craneo lems. Her birth weight was 6 pounds
ECV.jpg, Head MRI stroke.JPG and Head CT stroke.jpg. 5 ounces. She was discharged 1 day
(Own work. Bobjgalindo. Wikipedia)
form the hospital. Her development
was normal. She says a few words and
What is the most likely cause of his understands many. On examination,
condition? her height, weight and head circum-
A. Intraventricular hemorrhage ference just below the 2nd percentile.
B. Neonatal streptococcal meningitis She is bright, alert and attentive.
C. Ischemia Cranial nerves exam was normal.
D. Congenital toxoplasmosis Visual fields are intact to distraction.
E. Neonatal herpes simplex infection She has decreased usage of her right
arm and leg. Right thumb is abducted
vv Correct Answer is: C and grasping with the right hand is
Cerebellum abnormalities on the MRI crude. Tone is increased and tendon
became more recognizable by the clinician reflexes are exaggerated in the right
evaluating children with cerebral palsy limbs and her right plantar response
or developmental delay. This boy’s birth is extensor.
weight was under 1000 grams and gesta- What is your explanation for her
tional age of less than 28 weeks makes the neurological condition?
child extremely vulnerable to cerebellar A. Congenital CMV infection
injury. Cerebellar abnormalities can be B. Lissencephaly
presented in about 60% in such children C. Postnatal cerebral infarction
with cerebral palsy. Those abnormalities D. Hemiparesis cerebral palsy
do not see in the more mature premature. E. Fetal alcohol syndrome
Static Encephalopathy/Cerebral Palsy/Rehabilitation
661 25
vv Correct Answer is: D vv Correct Answer is: C
She has cerebral palsy (hemi paretic The studies referenced below indicate that
type), which statistically is most likely intraventricular hemorrhage may be a
prenatal in origin. Brain MRI is essential radiological marker indicating wider spread
to determine if further evaluation is indi- insult. Thus, in a very low weight premature
cated. Limited further evaluation may be it may indicate a less favorable outcome.
indicated if the child has a schizenceph-
aly. On the other hand, a full hematologic zz Suggested Reading
evaluation of the mother and the child 55 Inder TE. Neurodevelopmental impact of
in the presence of a pre-natal infarction. low-grade intraventricular hemorrhage in
The chances of CMV causing hemiparesis very preterm infants. J Pediatr.
is less likely but would be suggested by 2006;149(2):152–4.
malformation and calcification. Hearing 55 Patra K, Wilson-Costello D, Taylor G,
and urine screening for CMV is of great Mercuri-Minich N, Hack M. Grades I-II
help. Because the child has a benign Intraventricula hemorrhage in extremely
neonatal course, the post-natal injury is low birth weight infants: Effects on
not likely. neurodevelopment. J Pediatr.
2006;149(2):169–73.
zz Suggested Reading
55 Oskoui M, Shevell MI. Profile of pediatric ?? 4. A 30-week premature infant has a
hemiparesis. J Child Neurol. severe intraventricular hemorrhage
2005;20(6):471. and hydrocephalus at 3 weeks of age.
He has been stable and recently came
?? 3. A 2-day old baby boy who is one of an off the respirator. On examination, his
identical twin born at 27-week gesta- anterior fontanelle is full but not firm.
tion. Birth weight was 800 grams. A Sutures are not spread. Head growth
routine head ultrasound testing was has been normal.
done and showed a small intraventric- What is your advice to the family at
ular hemorrhage but no evidence of this time?
ventricular enlargement or parenchy- A. The patient needs an immediate
mal damage. His identical twin sibling ventriculo-peritoneal shunt and
has a normal ultrasound. His family is her prognosis is good.
asking about his prognosis. B. The patient needs an immediate
What statement is the most accurate ventriculo-peritoneal shunt but
one? despite intervention her progno-
A. He is no more likely than his sis is very bad.
sibling to have cerebral palsy or C. The patient will only need medical
mental delay management with acetazolamide
B. He is no more likely than his and/or furosemide but despite
sibling to have mental delay but treatment her prognosis is bad.
more likely to have cerebral palsy. D. The patient should be monitored
C. He is more likely than his sibling to with ultrasound and may need a
have cerebral palsy or mental delay drainage procedure; prognosis is
D. He is more like than his sibling to indeterminant.
have mental delay but not cere- E. The patient will only need medical
bral palsy management with acetazolamide
E. The results at this gestational age and/or furosemide and with this
are indeterminate treatment her prognosis is good
662 Chapter 25 · Static Encephalopathy/Cerebral Palsy/Rehabilitation

vv Correct Answer is: D and birth date, knows his colors, and
Prematurity has detrimental effects the can count to 20. Cranial nerve exam is
brain. The complication of hemorrhage normal. Upper extremities show nor-
and consequent hydrocephalus is cer- mal strength and tone, with some fine
tainly undesirable. The practical issue is motor clumsiness for his given age.
the relative risk of drainage and possible Lower extremities show increased
infection versus watching carefully and tone at the ankles, with some limi-
allowing additional damage to the brain. tation of dorsiflexion, more subtle
increases in tone are noted bilaterally
zz Suggested Reading in the hip adductors and hamstrings.
55 Casey AT, Kimmings EJ, Kleinlugtebeld
25 AD, Taylor WA, Harkness WF, Hayward
Reflexes are 2+ in the upper extremi-
ties and 3+ in the lower extremities.
RD. The long-term outlook for hydro- He has mildly crouched gait at both
cephalus in childhood. A ten-year cohort knees and on his toes (less so in his
study of 155 patients. Pediatr Neurosurg. orthotics). Brain MRI is shown.
1007;27(2):63–70.
55 Brouwer A, Groenendaal F, van Haastert
IL, Rademaker K, Hanlo P, de Vries
L. Neurodevelopmental outcome of
preterm infants with severe intraventricu-
lar hemorrhage and therapy for post-­
hemorrhagic ventricular dilatation. J
Pediatr. 2008;152(5):648–54.
55 Peretta P, Ragazzi P, Carlino CF, Gaglini P,
Cinalli G. The role of Ommaya reservoir
and endoscopic third ventriculostomy in
the management of post-hemorrhagic
hydrocephalus of prematurity. Childs Nerv
Sys. 2007;23(7):765–71.

?? 5. A 5-year-old boy with a history of


delayed development, mainly motor
milestones. He stared walking inde-
..      Fig. 25.2 Cation, PVL. American Academy of
pendently at 2 years of age and con- Neurology Institute, produced by permission
tinues to walk and run on his toes. He
is getting faster with time and there
has been no worsening of his gait pat- What is the most likely diagnosis?
tern. He was 32 weeks gestation, his A. arginase deficiency
mother had preterm labor and gesta- B. bilateral closed lip schizencephaly
tional diabetes. He went home after C. cerebral palsy
2 weeks of life and had some mild dif- D. dopa responsive dystonia
ficulties with feeding and weight gain E. idiopathic toe walking
initially.
His mother reports that he has no vv Correct Answer is: C
seizures and he has normal social and This boy has a delayed motor develop-
cognitively abilities. On your exami- ment, but no regression or daytime
nation, he is an alert and interactive fluctuation, and evidence of mild lower
young boy, who states his name, age, extremity spasticity and brisk reflexes. He
Static Encephalopathy/Cerebral Palsy/Rehabilitation
663 25
has a history of mild prematurity, and MRI difficulty. Spastic forms are the most
findings of mild hyperintense signal on T2 common, and can involve the legs primarily
FLAIR images of the periventricular white (most common type affecting premature
matter, all those findings are most consis- infants) called diplegic CP, involve one-half
tent with a diagnosis of cerebral palsy. of the body (usually from unilateral
Cerebral palsy is one of the neurode- hemorrhagic or ischemic stroke or unilat-
velopmental disability disorders of eral brain malformation) called hemiplegic
childhood. It is not a specific etiologic CP or involve all four extremities (often
diagnosis, rather a descriptive term for a from HIE in a term infant or severe diffuse
disorder of motor control (affecting traumatic injury) called quadriparetic CP.
movement, tone or posture) causing Various degrees of dyskinesia (dystonic and
activity limitation that is non-­progressive choreoathetoid movements) often accom-
in nature, resulting from an injury to or pany the spasticity, especially in cases
malformation of the brain at its early where there is prominent basal ganglia
formative stages. Rarely, loss of motor injury. Ataxic and hypotonic subtype are
function with time, which is typically diagnoses of exclusion after a thorough
secondary to complications of under evaluation for other primary cerebellar or
treated spasticity (such as joint contrac- lower motor neuron disorders are excluded.
tures or scoliosis) or rapid changes in Neuroimaging is useful in the evaluation of
weight or height during puberty rather the child with cerebral palsy if the etiology
than inherent progression of neurologic has not been established from the history.
disease. CP refers only to the motor MRI is superior than CT scan because it
dysfunction seen; it does not confer provides more detail of the white matter
(though is often associated with) cogni- and often shows abnormal white matter
tive impairment, hearing loss, strabismus signal or volume, or in the case of hemipa-
or epilepsy. It happens in 1–3 out of 1000 retic CP evidence of old focal infarction or
live births, prematurity is the most malformation of cortex. If the history and
common risk factor, which raises incidence neuroimaging do not determine a specific
to 4–10% if born with very low birth structural abnormality, or not consistent
weight and up to 20% risk if born at with a static process, metabolic and genetic
24–26 weeks gestational age. testing can be considered.
Nonetheless, full term infants represent In this case, there is no sign of cortical
more the half of all cases of cerebral palsy. malformation present, which argues
The potential etiologies includes brain against polymicrogyria as the etiology.
malformations or genetic disorders, Arginase deficiency presents much
post-natal brain injury at a young age (CNS differently than the other urea cycles
infection, accidental or non-accidental disorders, with onset of progressive spastic
trauma, kernicterus), prenatal factors paraplegia, intellectual deterioration, and
(maternal chorioamnionitis, intrauterine hyperactivity. Episodes of acute encepha-
growth retardation, toxin exposure, lopathy and seizures are unusual. Dopa-
congenital infections, maternal trauma), responsive dystonia is most commonly
perinatal brain injury (hypoxic-­ischemic caused by mutations in GTP cyclohydrolase
encephalopathy, neonatal stroke, traumatic and presents with dystonic spasms,
brain injury, intracranial hemorrhage) and commonly isolated to the legs initially and
brain injury related to prematurity (periven- leading to a gait pattern that may mimic
tricular leukomalacia and intraventricular that seen in cerebral palsy. Two-thirds of
hemorrhage). CP has been classified based cases manifest a classic diurnal variation,
on the location and type of motor control with progressive worsening over the course
664 Chapter 25 · Static Encephalopathy/Cerebral Palsy/Rehabilitation

of the day, which would not be typical with C. Symptoms are identifiable at birth
cerebral palsy. It is important to remember D. It is commonly related to obstetri-
to ask this question in the history, espe- cal procedures
cially if there is a hint that motor symptoms E. None of the above
are worsening. Idiopathic toe-walking
manifests early in childhood (usually in the vv Correct Answer is: A
first 1–2 years of life) and shows a prefer- Cerebral palsy is caused by intrauterine
ence to be on the toes (in standing or with injury (i.e. infarcts), unrelated to complica-
slow walking), that tends to lessen with tions of delivery. Symptoms are usually
attempts to run. No other upper motor identified after 6 months of age, or later in
25 neuron findings should be present on exam
and neuroimaging studies are normal.
patients with the choreoathetotic subtype.
An initial, but often overlooked, feature
is the demonstration of hand dominance
zz Suggested Reading within the first year. Cognition is most
55 Oskoui M, Shevell MI. Cerebral palsy and often normal, with intellectual disability in
the transition from pediatric to adult care. 20–25% of patients. Cerebral palsy may be
Continuum Lifelong Learning Neurol. characterized by the following subtypes:
2009;15(6):64–77. Ashwal S, Russman BS, 55 Paraplegic (44%), caused by injury in the
Blasco PA, et al. Practice parameter: centrum semiovale, typically superior and
diagnostic assessment of the child with lateral to the caudate. Lesions affect the
cerebral palsy. Report of the Quality lower extremity corticospinal tract fibers
Standards Subcommittee of the American more so than upper extremity fibers.
Academy of Neurology and the Practice 55 Hemiplegic (33%), often caused by
Committee of the Child Neurology placental emboli, periventricular hemor-
Society. Neurology. 2004;62(6):851–63. rhagic infarcts, and malformations. In
approximately 70% of cases, the left
?? 6. The Surveillance of Cerebral Palsy in hemisphere is affected due to flow through
Europe (SCPE) reported the following the patent ductus arteriosus entering the
incidence of co-morbidities in chil- left carotid artery. Approximately 50% of
dren with CP: children will have seizures.
A. Active seizures 20% 55 Quadriplegic (6%), caused by malforma-
B. Mental disadvantage (IQ < 50) 70% tions and hypoxic-ischemic injury.
C. Active seizures 50% 55 Choreoathetotic (12%), previously caused
D. Mental disadvantage (IQ < 50) 30% by Rh incompatibility leading to kernic-
E. Both (A) and (D) terus. Presently, most cases are due to
idiopathic injuries of the basal ganglia.
vv Correct Answer is: E 55 Ataxic (4%), most often caused by genetic
The SCPE reported an incidence of 20% disorders.
active seizures and 30% mental disad-
vantage (IQ < 50) in their cerebral palsy zz Suggested Reading
population. 55 Fenichel GM. Clinical Pediatric
Neurology.5th edition. Philadelphia:
?? 7. Which of the following statements Elsevier Saunders; 2005. p. 239–40,
regarding cerebral palsy is true? 269–70.
A. Hemiplegic cerebral palsy is often 55 Nass R. Developmental disabilities. In:
due to placental embolisms Bradley WG, Daroff RB, Fenichel GM,
B. Hemiplegic cerebral palsy involves Jankovic J, editors. Neurology in Clinical
the right hemisphere in >80% of Practice. 4th ed. Elsevier, Inc.; 2004.
cases p. 1791–810.
Static Encephalopathy/Cerebral Palsy/Rehabilitation
665 25
?? 8. An infant with epileptic encephalopa- ketogenic diet should be started as soon as
thy came to your office for evaluation. the diagnosis is suspected, since this treat-
On examination, he is micro cephalic ment option improves seizure control and
and has prominent developmental the abnormal movements; however, it is less
delay. His CSF analysis showed, glu- effective for the psychomotor impairment.
cose level was 30 mg/dl, whereas the
level of serum glucose was 112 mg/dl. zz Suggested Reading
The rest of his CSF tests are normal. 55 ©2011 Wolters Kluwer Health Lippincott
Which statement is correct regard- Williams& Wilkins N2013E. All rights
ing his condition? reserved.
A. It is caused by deficiency in the
glucose transporter type 1 ?? 9. The diagnosis of cerebral palsy (CP) is
B. Brain MRI usually suggests the divided into four major classifications
diagnosis which include which of the following:
C. Ketogenic diet has been shown to A. Hemiparetic, spastic diplegia,
be ineffective for seizure control ataxic, and athetoid
D. Phenobarbital is the treatment of B. Hemiplegic, spastic diplegia,
choice ataxic, and athetoid
E. It is inherited in an X-­linked fashion C. Hemiplegic, quadriplegic, dyski-
netic, and athetoid
vv Correct Answer is: A D. Hemiplegic, diplegic, ataxic, and
This is the clinical presentation of a glucose mixed
transporter type 1 (GLUT-1) deficiency. The E. Spastic, ataxic, athetoid/ dyski-
brain utilizes glucose as its primary source netic and mixed
of energy. In fasting conditions, glycogen
is exhausted within minutes, and since vv Correct Answer is: E
amino acids and fat cannot be used for the Quadriplegia, which is secondary to spi-
production of energy in the brain, ketones nal cord injury or traumatic brain injury,
become the alternative fuel. Glucose should not be confused with spastic quad-
crosses the blood brain barrier facilitated riplegia. Also, tardive dyskinesia not to be
by GLUT-­1, which is a membrane-bound confused with dyskinetic cerebral palsy, or
protein encoded by the SLC2A 1 gene on the condition of (paralytic) “diplegia” with
chromosome 1p35–31.3. It is inherited in spastic diplegia.
an autosomal dominant syndrome and
causes a defect in glucose transport across ?? 10. Which of the following statements
the blood-brain barrier and into brain cells, about CP are true?
manifesting as an epileptic encephalopathy A. The intellectual level among peo-
with infantile-­onset seizures, developmen- ple with CP varies from genius to
tal delay, microcephaly, and complex move- intellectually impaired, as it does
ment disorders. CSF glucose level is low with in the general population.
a normal serum glucose level, and other B. Therapies used to treat tight mus-
CSF studies are normal, excluding other cles due to spasticity may include
causes of hypoglycorrhachia (such as CNS surgical lengthening procedures,
infection). EEG may show 2.5–4 Hz spikes Botox, and orthotics.
and waves and the interictal EEG findings C. Individuals with CP have a short-
may improve with glucose. Neuroimaging ened life expectancy
does not show specific abnormalities. D. CP is a progressive disorder because
Later-onset forms with episodic movement symptoms can become more severe
disorders and ataxia have been described. A due to subdural damage.
666 Chapter 25 · Static Encephalopathy/Cerebral Palsy/Rehabilitation

vv Correct Answer is: (A) and (B) brain tissue or have an effect on the final
In spite of a great number of CP indi- outcome. However, in the last 5 years this
viduals have mental disadvantage, their situation has changed because of the
intellectual level varies from genius to publication of positive results from several
intellectually impaired just it does in the controlled trials of therapeutic moder-
general population. Spasticity may be ate hypothermia for term infants with
treated surgically, pharmacologically or neonatal HIE. This led wider clinical appli-
with the use of appropriate orthotics. cation of therapeutic hypothermia for
Individuals with CP usually have normal neonatal encephalopathy. These advances
life expectancies. CP is a static neurologic in treatment are based on knowledge
25 disorder. gained from clinical observation of babies
with asphyxia and extensive laboratory
?? 11. A 2-hour-old baby is admitted to NICU research with experimental models. Data
after being delivered by emergency from these experiments led to a heuristic
C-section at full term to a 35-year-old model in which hypoxia-ischemia trig-
female. Pregnancy was uncompli- gers a delayed series of events that lead
cated until the day of delivery when to cell death in the brain. This period of
the mother had vaginal bleeding delay, or latent interval, suggested that
secondary of placental abruption. At post-insult interventions could be protec-
delivery, Apgar scores were 2 at 1 min tive if started in time. Studies show that
and 3 at 5 min. Resuscitated was done improves Survival without cerebral palsy
and the baby was placed on mechani- or other disability is about 40% if moder-
cal ventilation. On examination, he ate hypothermia within 6 h of asphyxia
has decreased tone, minimal response has been applied, also death or neurologi-
to stimulation, sluggish pupillary, cal disability will be reduced by nearly
oculo-cephalic, and corneal reflexes. 30%.
What is the best treatment you can
use to minimize the likelihood of zz Suggested Reading
the future development of cerebral 55 Johnston MV, Fatemi A, Wilson MA,
palsy? Northington F. Treatment advances in
A. Indomethacin neonatal neuroprotection and neurointen-
B. Dextromethorphan sive care. In Lancet Neurol 2011;10:372–82.
C. Moderate hypothermia
D. Magnesium sulfate ?? 12. A 5-year-old boy has been evaluated
E. Phenytoin for toe walking. The mother went
into labor at 32 weeks gestation. He
vv Correct Answer is: C was admitted to the neonatal inten-
Damaging the developing brain for any sive care unit for observation, he
reason can cause death or permanent dis- did not require intubation and was
ability such as cerebral palsy. Term infants discharged home at 3 weeks of age.
could suffer hypoxic-ischemic encepha- His early milestones were on target
lopathy (HIE) about 3 per 1000 live-born except he walked at 18 months of
infants in developed countries, higher in age. He has been toeing walking ever
the developing world. Perinatal asphyxia since. His toe walking has improved
can cause a million deaths worldwide. In over time, but he still is not consis-
the past physicians traditionally provided tently able to pedal a bicycle with
supportive care with little expectation training wheels. He has no other
that their interventions would salvage medical complications. He speaks in
Static Encephalopathy/Cerebral Palsy/Rehabilitation
667 25
full sentences, can write his name and static upper motor neuron disorder. Limb-­
enjoys playing video games. Head girdle muscular dystrophy and Charcot-­
circumference is at the 5th percentile. Marie-­Tooth disease type X (CMT-X) are
On examination, speech is mildly disorders of the lower motor neuron and
dysarthric. He has increased tone in would not be expected to lead to spastic-
both legs, with slightly limited passive ity. Although CMT-X can rarely have associ-
leg extension and foot dorsiflexion. ated white matter changes and extensor
Arm strength, bulk and tone are nor- plantar responses, distal weakness and
mal. He has increased deep tendon sensory loss expected with this disorder
reflexes in both lower extremities. are absent. No evidence of gray matter
He is a slow walker, unsteady with a damage (cognition, language or social
mildly crouched appearance and he skills) making pervasive developmental
walks on the balls of his feet. Brain disorder (PDD) and San Filippo disease
MRI axial T2 sample is shown. inappropriate choices. Cerebral palsy is
one of the neurodevelopmental disability
disorders of childhood. It is not a specific
etiologic diagnosis, rather a descriptive
term for a disorder of motor control (affect-
ing movement, tone or posture) causing
activity limitation that is non-progressive
in nature, resulting from an injury to or
malformation of the brain at its early
formative stages. Rarely, loss of motor
function with time, which is typically sec-
ondary to complications of under treated
spasticity (such as joint contractures or
scoliosis) or rapid changes in weight or
height during puberty rather than inher-
ent progression of neurologic disease. CP
refers only to the motor dysfunction seen;
it does not confer (though is often associ-
ated with) cognitive impairment, hearing
loss, strabismus or epilepsy. It happens in
1–3 out of 1000 live births, prematurity is
..      Fig. 25.3 American Academy of Neurology the most common risk factor, which raises
Institute, produced by permission incidence to 4–10% if born with very low
birth weight and up to 20% risk if born at
What is most likely diagnosis? 24–26 weeks gestational age.
A. cerebral palsy Nonetheless, full term infants represent
B. Charcot-Marie-Tooth disease type X more the half of all cases of cerebral palsy.
C. limb-girdle muscular dystrophy The potential etiologies includes brain
D. pervasive developmental disorder malformations or genetic disorders,
E. Sanfilippo disease post-natal brain injury at a young age (CNS
infection, accidental or non-accidental
vv Correct Answer is: A trauma, kernicterus), prenatal factors
The boy has spasticity in his both lower (maternal chorioamnionitis, intrauterine
extremities which is a long standing and growth retardation, toxin exposure,
non-progressive by history. This indicates a congenital infections, maternal trauma),
668 Chapter 25 · Static Encephalopathy/Cerebral Palsy/Rehabilitation

perinatal brain injury (hypoxic-­ischemic 55 Ashwal S, Russman BS, Balsco PA. Practice
encephalopathy, neonatal stroke, trau- parameter: diagnostic assessment of the
matic brain injury, intracranial hemor- child with cerebral palsy: Report of the
rhage) and brain injury related to Quality Standards Subcommittee of the
prematurity (periventricular leukomalacia American Academy of Neurology and the
and intraventricular hemorrhage). CP has Practice Committee of the Child
been classified based on the location and Neurology Society. Neurology.
type of motor control difficulty. Spastic 2004;62(6):851–63.
forms are the most common, and can
involve the legs primarily (most common ?? 13. A 16-month-old girl is being evalu-
25 type affecting premature infants) called
diplegic CP, involve one-half of the body
ated for delayed motor develop-
ment. She stared sitting (10 months)
(usually from unilateral hemorrhagic or and began army-crawling at
ischemic stroke or unilateral brain malfor- 15 months. She cannot pull to stand,
mation) called hemiplegic CP or involve all cruise, or walk. When you held her
four extremities (often from HIE in a term upright, she beards weight, but
infant or severe diffuse traumatic injury) stand on her tip-toes. She was born
called quadriparetic CP. Various degrees of premature at 27 weeks gestation,
dyskinesia (dystonic and choreoathetoid she was on a ventilator for 6 weeks
movements) often accompany the in the NICU. She had bilateral sub-­
spasticity, especially in cases where there is ependymal hemorrhages which
prominent basal ganglia injury. Ataxic and resolved on follow up ultrasound
hypotonic subtype are diagnoses of testing. On examination, she has
exclusion after a thorough evaluation for severe spasticity in both lower
other primary cerebellar or lower motor extremities, 3+ reflexes at the knees
neuron disorders are excluded. and non-­sustained clonus at both
Neuroimaging is useful in the evaluation of ankles. She has a crossed adductor
the child with cerebral palsy if the etiology response and scissors her legs when
has not been established from the history. held upright.
MRI is superior than CT scan because it What might you see on her MRI?
provides more detail of the white matter A. Bilateral subdural effusions
and often shows abnormal white matter B. Communicating hydrocephalus
signal or volume, or in the case of hemipa- C. Diffuse cystic encephalomalacia
retic CP evidence of old focal infarction or D. Noncommunicating hydrocephalus
malformation of cortex. If the history and E. Periventricular white matter injury
neuroimaging do not determine a specific
structural abnormality, or not consistent vv Correct Answer is: E
with a static process, metabolic and genetic Cerebral palsy (CP) can occur secondary
testing can be considered. to a varieties of central nervous system
abnormalities, which can occur pre-,
zz Suggested Reading peri-, or postnatally. By definition, CP
55 Oskoui M, Shevall MI. Cerebral palsy and excludes progressive or degenerative
the transition from pediatric to adult care. disorders. Brain MRI is currently the most
Continuum. 2009;15(6):64–77. commonly radiological used method of
Static Encephalopathy/Cerebral Palsy/Rehabilitation
669 25
investigating for cause of CP. A number of
abnormalities can be identified, includ-
ing periventricular white matter injury
(leukomalacia), diffuse encephalopathy
(including cystic encephalomalacia),
focal ischemic lesions, and malforma-
tions. This is true in 80–90% of patients.
Periventricular leukomalacia is the most
common abnormality seen in patients
with spastic diplegic type of CP, which
is the predominant type of CP seen in
babies born prematurely.

zz Suggested Reading
55 Robinson MN, Peake LJ, et al. Magnetic
resonance imaging findings in a popula-
tion-based cohort of children with cerebral
palsy. Dev Med Child Neurol.
2009;51(1):39–45.
..      Fig. 25.4 Axial CT scan showing schizencephaly in
a 6-year old child. (Hellerhoff. Wikipedia)
?? 14. An 8-year-old girl came to the ER with
her mother for evaluation of head- A. Carbamazepine toxicity
aches. She has right hemiplegia and B. Hydrocephalus
developed headaches a month ago, C. Hyponatremia
which have worsened over time then D. Migraine headaches
became constant. She vomits with E. Subclinical seizures
the severe headache. She never had
headaches before, but does have his- vv Correct Answer is: B
tory of complex partial epilepsy and This child has schizencephaly (open-­
she is on carbamazepine. Her mother lipped type) as the cause of her cerebral
has a history of migraine headaches. palsy. This condition can be complicated
On examination, she is obtunded, by hydrocephalus. The fluid accumulation
appears ill, and has spasticity of her is causing mass effect, with a midline shift
right limbs with a hemiparetic gait. to the right side. It is helpful to compare
Fundus exam was difficult to perform. the head scan with prior scans if possible,
Carbamazepine level is within normal as significant changes in CSF flow dynam-
therapeutic limits. She has normal ics related to the underlying malformation
CBC, electrolytes with sodium level can be subtle, yet cause significant clinical
129 mEq/L. Head CT scan is shown. No problems.
previous images are available for com- Carbamazepine toxicity can be
parison. associated with headaches but also with
Which of the following is the most diplopia and ataxia, which are not present.
likely cause of her headaches and The mild ­hyponatremia may be due to
obtundation? carbamazepine therapy but is not likely
670 Chapter 25 · Static Encephalopathy/Cerebral Palsy/Rehabilitation

low enough to be causing problems. tonia is abnormally increased resistance to


Migraine headaches are a diagnosis of externally imposed movement at a joint.
exclusion. Subclinical seizures are not a Spasticity, dystonia, and rigidity, alone
common cause of headaches. or in combination can cause hypertonia.
Hydrocephalus can complicate It is important to differentiate between
schizencephaly, particularly in the these syndromes because each may
“open-lipped” form. Some patients also require different treatments. Spasticity is
have septo-optic dysplasia, with agenesis the most common etiology of hypertonia
of the septum pellucidum and optic nerve in children with cerebral palsy. It is a
hypoplasia. velocity-dependent resistance of a muscle
25 zz Suggested Reading
to stretch and is defined as having one
or both of the following: (1) resistance to
55 Maeda T, Akaishi M, Shimizu M, et al. The externally imposed movement increases
subclassification of schizencephaly and its with increasing speed of stretch and varies
clinical characterization. Brain Dev. with the direction of joint movement, and/
2009;31:694–701. or (2) resistance to externally imposed
movement rises rapidly above a threshold
?? 15. A 5-year-old boy has cerebral palsy speed or joint angle. On examination,
came for his regular follow up visit. He upper motor signs (hyperreflexia, clonus,
was premature at 32 weeks gestation Babinski response) are present. Children
and had both hypoxic ischemic injury who continue to have problems with
and grade 3 intraventricular hemor- hypertonia despite adequate spasticity
rhage requiring VP shunt. His treat- treatment may have dystonia or rigidity.
ment includes PT 4 days/week and This boy has the typical features of dys-
10 mg baclofen four times a day. He is tonia, in which co-­contraction of agonist
doing his PT and taking his medicine and antagonist muscle groups result in
religiously, but still he has increased twisting and abnormal postures. In the
tone and often appears uncomfort- child whose etiology is undetermined, a
able because of muscle contractions trial of carbidopa-levodopa is given, as
which cause twisting of his limbs and some have “dopa-responsive dystonia”.
lateral torsion of his trunk. On exami- Otherwise, trihexyphenidyl is used, start-
nation, he has increased deep tendon ing at 2.5 mg/day and titrating up slowly
reflexes and increased tone in all to 60–80 mg/day.
limbs, but minimal spasticity.
What is your next step in his man- zz Suggested Reading
agement? 55 Deon LL, Gaebler-Spira D. Assessment
A. Obtain EEG and treatment of movement disorders in
B. Prescribe dantrolene children with cerebral palsy. Orthop Clin
C. Assess for shunt malfunction N Am. 2010;41:507–17.
D. Prescribe diazepam
E. Prescribe trihexyphenidy ?? 16. A 10-year-old boy with a spastic
diplega on daily physiotherapy, he
vv Correct Answer is: E has trouble ambulating without sup-
Cerebral palsy is the most common motor port in addition to painful muscle
disability in childhood. Patients with CP spasms. His family asked you for any
usually have mixed muscle tone, some other treatment options. He is tak-
muscle groups have low tone and others ing oral baclofen 20 mg QID, which
have increased tone. Definition of hyper- has provided modest benefit. On
Static Encephalopathy/Cerebral Palsy/Rehabilitation
671 25
examination, he has significant spas- facilitating mobility and dexterity, and
ticity with marked increase in adduc- improving patient ease of care. A multi-
tor tone. disciplinary panel systematically reviewed
What is your response to the family? relevant literature from 1966 to 2008 and
A. Oral Dantrolene generated practice parameter regarding
B. Oral Diazepam pharmacologic treatment of spasticity in
C. Botulinum toxin type A children and adolescents with cerebral
D. Selective dorsal rhizotomy palsy. For localized or egmental spasticity,
E. Intrathecal baclofen pump botulinum toxin type A is established as
an effective treatment to reduce spasticity
vv Correct Answer is: C in the upper or lower extremities. For gen-
Cerebral palsy (CP) is the most common eralized spasticity, baclofen and diazepam
cause of motor disability in children, with are probably effective, tizanidine is pos-
a prevalence of 3.6 per 1000 in 8-year-old sibly effective, and there was insufficient
children. The majority of children with CP data on the use of dantrolene. Intrathecal
are affected by spasticity, but it is only 1 baclofen therapy use in generalized spas-
component of the multifaceted motor dis- ticity became very popular. In this patient
ability of CP and may not be the main fac- with segmental spasticity, botulinum
tor interfering with function. The decision toxin is the preferred treatment option.
to use anti-­spasticity medications requires
careful assessment of the patient’s other zz Suggested Reading
impairments (e.g. weakness, movement 55 Delgado MR, Hirtz D, et al. Practice
disorders). Reasons to treat spasticity parameter: pharmacologic treatment of
include reducing pain and muscle spasms, spasticity in children and adolescents with
facilitating brace use, improving posture, cerebral palsy (an evidence-based review).
minimizing contractures and deformity, Neurology. 2010;74:336–43.
673 26

Toxic Encephalopathy

© Springer International Publishing AG, part of Springer Nature 2018


Y. M. Awaad, Absolute Pediatric Neurology, https://doi.org/10.1007/978-3-319-78801-2_26
674 Chapter 26 · Toxic Encephalopathy

?? 1. A 16 year old girl came to the ER by injury from kava kava has included sev-
EMS in a coma. Her mother reported eral incomplete or overlapping reports,
that her daughter became overly and causality was rarely well shown.
weight and health conscious for the Nevertheless, there are a small number
past year and she is using her job of cases of severe hepatic injury arising
money to buy vitamins and other during therapy that are convincing.
supplements. The mother is con-
cerned that her daughter eats these zz Suggested Reading
supplements instead of food. The girl 55 Teschke R, Schulze J, Schwarzenboeck A,
was adequately oxygenated and had Eickhoff A, Frenzel C. Herbal hepatotoxic-
a normal heart rate, so the attending ity: suspected cases assessed for alternative
physician ordered a stat MRI scan. causes. Eur J Gastroenterol Hepatol.
There is an increased signal in the 2013;25(9):1093–8.
26 striatum, thalamus, and internal cap- 55 Binesh N, Huda A, Thomas MA, Wyckoff
sule bilaterally on the T1-weighted N, Bugbee M, Han S, Rasgon N, Davanzo
images. The T2-weighted images P, Sayre J, Guze B, Martin P, Fawzy
showed white matter abnormalities F. Hepatic encephalopathy: a neurochemi-
and MR spectroscopy demonstrated cal, neuroanatomical, and neuropsycho-
elevated glutamine and glutamate in logical study. J Appl Clin Med Phys.
the white matter. Blood work shows 2006;7(1):86–96.
highly elevated liver function tests
and a serum ammonia of 360 mcg/dl.   ?? 2. A foster parents brought their 5-year-
The supplements she has taken most old boy for an evaluation of develop-
likely include: mental delay. The boy came to their
A. Kava kava home 2 months ago and they are not
B. Ginko biloba able to provide any medical history.
C. Vitamin B12 He tends to eat toys, paper, wood and
D. Selenium clay.  
E. Chinese red rice What tests should you be ordered?
A. EEG
vv Correct Answer is: A B. Lead level
The clinical presentation indicates C. Liver function tests
hepatic encephalopathy because of the D. Brain MRI
high ammonia level, MRI and MRS scans E. Thyroid function tests
findings. Kava kava is an herbal derived
from roots of the plant Piper methysti- vv Correct Answer is: B
cum, a member of the pepper family Lead is one of the most common envi-
found in the Western and South Pacific. ronmental toxin. There is a clear correla-
It has been proposed to be anxiolytic tion between highly elevated serum
and used in patients with anxiety disor- lead levels, clinical symptoms, and cog-
ders and as treatment for insomnia, pre- nitive deficits. In the past decades blood
menstrual syndrome and stress. levels have ­dramatically fallen (from 15
Recently, the safety of kava products ug/dl to 2.7 in 1991 to 1994), still there
became in question because reports of is big number of children who may have
liver injury. Other groups have argued blood levels equal or greater than 10ug/
that hepatotoxicity is related to the use dl. The current guidelines recommend
of concomitant drugs or herbals. screening of all children with identifi-
Furthermore, the literature on liver able risk factors including children
Toxic Encephalopathy
675 26
l­ iving in housing before 1950, children room air. Currently he is lethargic with
of ethnic or racial minority groups who mildly dilated but reactive pupils,
may be exposed to lead containing food normal tone, but prominent shivering
remedies, children who have emigrated movements. DTRs are brisk in all limbs
(or adopted) from countries where lead with clonus at both knees and ankles.
poisoning is prevalent, children with A urine toxicology screen is normal.
iron deficiency, children exposed to con- Serum comprehensive metabolic
taminated dust or soil, children with profile is normal. LP is performed and
developmental delay whose oral behav- shows 5 WBC/hpf, 0 RBC/ hpf with
ior places them at risk for lead exposure, normal protein and glucose.   
children whose parents are expose to What is most appropriate treatment
lead (vocationally, avocationally, and for this patient?
during home renovation), children of A. IV acyclovir
low income families and victims of B. Oral bromocriptine
abuse and neglect. C. Oral cyproheptadine
D. IV dantrolene
zz Suggested Reading E. IV fosphenytoin
55 Practice parameter: Evaluation of the child
with global developmental delay: report of vv Correct Answer is: C
the Quality Standards subcommittee of the The patient has serotonin syndrome
American Academy of Neurology and the which can be diagnosed by having at
practice committee of the Child Neurology least three of the following clinical find-
Society, Shevell M, Ashwal S Neurology ings: agitation, mental status changes,
2003;60:367–80. myoclonus, hyperreflexia, diaphoresis,
55 Davioli CT, Chisolm JJ. Childhood Lead shivering, tremor, diarrhea, incoordina-
Poisoning. In: Capute AJ, Accardo PJ, tion and fever. The syndrome can occur
editors. Developmental disabilities in soon after the addition of, or increase in
infancy and childhood. Vol. 1. 2nd ed. dose of a serotonergic agent. You have
Baltimore: Brooke’s publishing.1996. 232–3. to exclude other etiologic causes (such
as substance abuse or withdrawal, met-
?? 3. A 13-year-old boy was admitted to abolic derangement or infectious cause)
the emergency department with and mandates that an antipsychotic
abnormal movements for the last drug (with dopaminergic blocking prop-
3 h. He stayed home with his family erties) has not recently been started or
and he was acting normally, without dose increased. Antidepressant medica-
any signs of sickness. After having tions (both serotonin specific reuptake
his dinner with the family, he started inhibitors such as s­ ertraline and tricyclic
“acting strange”, and had twitching antidepressants such as amitriptyline),
movements of his extremities. He stimulants (especially amphetamine-
has a history of mild depression and based), triptans and ergotamine based
migraine headaches, and he takes compound for acute abortive migraine
50 mg of amitriptyline at bedtime. treatment, lithium, and the monamine
One day ago, his neurologist added oxidase inhibitors such as selegiline
sertraline, 50 mg once daily. He took (typically used as an antiparkonsonian
his first dose tonight with dinner. In medication in adults) can lead to sero-
the emergency department, he has tonin excess in pediatric neurology
a temperature of 102 F, pulse of 125, patients. Recreational drugs such as
and oxygen saturations of 98% on “Ecstasy” and cocaine also place a
676 Chapter 26 · Toxic Encephalopathy

patient at risk. Typically, a combination attributed to her constipation and


of two agents with serotonergic proper- recent move to a different house. The
ties is required to produce the syn- family moved into an older home
drome, though recent cases occurring 3 months ago, which they have been
with SSRI’s in isolation have been renovating. She have no fever. Her
reported. white blood cell count is normal but
In this case, there was no neuroleptic she has microcytic anemia. Her optic
use, treatments for neuroleptic malig- disc margins are obscure.  
nant syndrome, such as dantrolene or What is your the most likely diag-
bromocriptine would not be appropri- nosis?
ate. The patient does not have herpes A. Glioblastoma
encephalitis because the acute onset of B. Lead poisoning
symptoms without a viral prodrome and C. Pseudo tumor cerebri
26 a normal CSF profile. The patient’s clini- D. Bacterial meningitis
cal presentation is not suggestive of a E. Migraine headaches
seizure, making fosphenytoin an incor-
rect choice. vv Correct Answer is: B
Physicians have to inform patients This is a clinical presentation of an
who are taking any serotonergic medi- encephalopathy which is insidious in
cations to watch for the symptoms and onset. Lead encephalopathy may pres-
signs noted above, especially if two ent with increased intracranial pressure.
agents are being used together. The There may also be nuchal rigidity sec-
only combination that is clearly contra- ondary to tonsillar herniation. The initial
indicated are SSRI’s and MAO inhibitors. picture maybe confused with meningitis
The risk of prescribing triptans with or a case of increased intracranial pres-
either antidepressants or stimulant sure like space occupying lesions.
medication is controversial, and it has to Rarely, there may be cerebellar ataxia
be on case to case bases. and 6th and 7th cranial nerve palsies.
Most cases of serotonin syndrome Peripheral neuritis from axonal degen-
occur in the first 24–48 h. For milder eration may be present in older children
cases, stopping of the offending or adults. Classically the presentation
agent(s) is adequate treatment. More maybe insidious. The child may be pale,
involved cases can be treated with ben- irritable, and listless with decreased
zodiazipines or cyproheptadine (a 5HT-2 appetite. Vomiting and epigastic pain
antagonist) administered orally or via are common. Radiographic findings are
nasogastric tube. The syndrome itself characteristic. There may be dense, radi-
usually resolves within 24 h of discon- opaque band at the metaphyses of
tinuation of the offending medication. numerous long bones. The patient does
not have bacterial meningitis because
zz Suggested Reading she is afebrile with a normal white
55 Seminars in Pediatric Neurology.10(1); blood cell count.
2003.
zz Suggested Reading
?? 4. A 2–year-old girl was admitted to the 55 Menke J. Toxic and Nutritional disorders.
hospital because of her vomiting and In: Menke J, Sarnat H, Maria BL, editors.
lethargy for 4 days. Her past medical Child Neurology. 7th ed. Philadelphia:
history is negative. 3 months ago she Lippincott Williams and Wilkins. 2006;
began to have irritability, which was pp. 703–5.
Toxic Encephalopathy
677 26
?? 5. A 3-year-old girl was admitted to the zz Suggested Reading
Emergency Department for altered 55 Glatstein M, Finkelstein Y, Scolnick
consciousness. She was found her D. Accidental methadone ingestion in an
in bed room, pale, disoriented, and infant: report and review of the literature.
unable to walk. She has become pro- Pediatr Emerg Care. 2009;25:109–11.
gressively more lethargic, and is barely
arousable at the time of her arrival at ?? 6. A 10-year-old boy has the history of
the ED. No recent illnesses or injuries developmental delayed, attention def-
have been reported by her family and icit hyperactivity disorder and obses-
has had normal development. When sive compulsive disorder brought by
you examined her, she was normoten- his family to the emergency depart-
sive and afebrile, but mildly tachycardic. ment with gradual onset of abnormal
She grimaced and withdrew her hands movements of the neck, shoulders
to deep pain, but she did not open and back described by the family as
her eyes. All four extremities appear “stiffening” and “rolling” movements.
to move equally. She has pinpoint He has the movements during awak-
pupils. Further history with the family ing and disappeared during sleep. His
indicates that the child’s grandmother psychiatrist started him on quetiapine
(also living in the same home) is on 50 mg/day as a “mood stabilizer”,
oxycodone hydrochloride for chronic 3 days later the family noticed those
pain. According to the child’s family, her abnormal movements. He had been
grandmother has a bad habit of leaving taking escitlopram for the past 6
her medication around the house. months without any side-­effects. He
Which of the following medications has no intercurrent illnesses, fever/
should be administered? rash, muscle soreness/weakness/
A. Clonidine fatigue, urinary difficulties or mental
B. Naloxone status changes. He is healthy and
C. Flumazenil interactive but distractible, with mod-
D. Apomorphine erately severe spasmodic ­torticollis
E. Propranolol and generalized dystonia of the trunk.
What is your best treatment option?
vv Correct Answer is: B A. Propranolol
Opiate overdoses are characterized by B. Phenytoin
depressed consciousness, decreased C. Metoclopramide
respiratory drive, and miosis, and can D. Diphenhydramine
lead to hypothermia, bradycardia, and E. Bromocriptine
hypotension if sufficient amounts are
ingested. Respiratory support may be vv Correct Answer is: D
required. Naloxone reverses the phar- This is a clinical presentation of drug-­
macological effects of opiates by bind- induced acute dystonia, induced by
ing to opioid receptors (specifically with quetiapine fumarate (Seroquel®), a psy-
preferential binding to the μ-opioid chotropic agent belonging to a chemical
receptor) as a competitive antagonist. class, the dibenzothiazepine derivatives,
When injected intravenously, it acts indicated for bipolar disorder (including
quickly (1 min), but has a relatively short maintenance treatment), mania, depres-
duration of action (less than 1 h), and so sion, and schizophrenia. Extrapyramidal
additional doses may be required to side-effects are common with use of this
counter-act the opiate effect. medication, including dystonia and
678 Chapter 26 · Toxic Encephalopathy

choreo-athetosis. Serotonin syndrome 55 Weerasuriya A, Mizisin AP. The blood-­


and neuroleptic malignant syndrome nerve barrier: structure and functional
can occur with the combination therapy significance. Methods. Mol Biol.
with selective serotonin re-uptake 2011;686:149–73.
inhibitors (SSRIs). Different medications 55 Kanda T. Biology of the blood-nerve
can be used on an empiric basis to find barrier and its alteration in immune-­
one or a combination that is effective mediated neuropathies. J Neurol
for a specific patient. It should be noted Neurosurg Psychiatry. 2013;84:208–12.
that not all patients will respond simi-
larly to the same medications. ?? 8. A 14-year-old boy complained to his
family that he cannot “feel with his
zz Suggested Reading hands”. The child has been moved
55 Dressler D, Benecke R.Diagnosis and man- recently to the states from an African
26 agement of acute movement disorders. country and lives with his aunt who
J Neurol. 2005;252(11):1299–306. is his legal guardian. His medical and
55 Pierre JM. Extrapyramidal symptoms with family history is not available. He is
atypical antipsychotics: incidence, preven- underweight but muscular young boy.
tion and management. Drug Saf. In spite of he has persistent cough he
2005;28(3):191–208. is not taking any medications since he
55 Rodnitzky RL. Drug-induced movement moved to his aunt’s house. The child
disorders in children and adolescents. recalls taking medication for several
Expert Opin Drug Saf. 2005;4(1):91–102. months b ­ ecause of his cough. On
examination, he has reduced sensa-
?? 7. When the nervous system is poten- tion in a glove­and-stocking distribu-
tially vulnerable to toxic, infectious, tion. He has mild weakness in wrist
and immune disorders, the blood- and ankle flexors and extensors. DTR’s
nerve barrier is most likely not intact are within normal limits with the
at which of the following locations? exception of absent ankle reflexes.
A. cauda equina What is the most likely diagnosis?
B. dorsal root ganglion A. Hereditary sensory and auto-
C. median nerve nomic neuropathy type I
D. spinal motor nerve root B. lsoniazid toxicity
E. spinal sensory nerve root C. Hereditary tyrosinemia
D. Congenital hypomyelinating neu-
vv Correct Answer is: B ropathy
The dorsal root ganglion is where the
blood-nerve barrier (analogous to the vv Correct Answer is: B
blood-­brain barrier) is least intact, Glove-and-stocking neuropathy can be
which may be important in the develop- caused by different medications includ-
ment of, for example, herpes zoster ing isoniazid, nitrofuratin, vincristine
infections involving the nervous system and zidovudine. The child came from
or their reactivation, as well as immune- Africa. Africa has the highest rate of
mediated ganglionopathies. tuberculosis in the world (1382 cases
per 100,000) and has a persistent cough.
zz Suggested Reading His sensory lose is highly suggestive of
55 Levin K. Diseases of the nerve roots. a drug induced pyridoxine deficiency.
Continuum Lifelong Learn Neurol. Pyridoxine taken along with isoniazid
2008;14(3):134–55 can prevent neuropathy but has mini-
Toxic Encephalopathy
679 26
mal effect on rate of recovery after the rin) use for symptomatic relief during
neuropathy is present. viral illness. There is an even higher rate
Hereditary sensory and autonomic when administered in children with
neuropathy type I begin in the second chickenpox. The incidence of Reye syn-
decade of life and presents with lancinat- drome decreased after the reduction
ing pains in the legs and ulcerations of aspirin use in children. The progression
the feet. from symptom onset to hepatic failure,
The major clinical features of heredi- cerebral edema and death can occur
tary tyrosinemia are liver failure and rapidly, and has been reported to occur
Fanconi syndrome associated with in less than 24 h.
peripheral dysesthesias or paralysis. Encephalitis has a less progressive
The clinical features in congenital course and with focal neurological
hypomyelinating neuropathy are similar symptoms. Liver involvement is not
to acute infantile spinal muscular atro- common.
phy and presents in the newborn period. Cerebral hemorrhage is not associ-
ated with varicella infections and most
zz Suggested Reading commonly would presents with focal
55 Mafukidze AT, Calnan M, Furin neurological symptoms. Liver involve-
J. Peripheral neuropathy in persons ment is not common.
tuberculosis and other comorbid disease. Vasculitis is not commonly seen in
J Clin Tuberc Other Mycobact Dis. 2015. varicella infections and would present in
a more gradual onset with focal neuro-
?? 9. A 6-year-old boy was brought to the logical symptoms. Severe liver involve-
emergency department because ment is not commonly associated with
he was found to be agitated with vasculitis.
delirium. He was recovering from
chickenpox under the care of his zz Suggested Reading
grandmother because his parents 55 Glasgow JFT, Middleton B. Reye syn-
were away on a vacation. His grand- drome- insights on causation and progno-
mother was giving him analgesics sis. Arch Dis Child. 2001;85(5):351–53.
for discomfort and a low-grade fever.
Three days later he began vomiting, ?? 10. All the following are known side
became lethargic and confused. The effect of carbamazepine except?
ED attending admitted him to the A. A Dizziness
pediatric ICU where he developed B. Nystagmus
decorticate posturing and he was C. Drowsiness
obtunded. He has hyperammonemia D. Hypernatremia
and elevated liver enzymes.   E. Nausea
What was the cause of this clinical
course? vv Correct Answer is: D
A. Varicella encephalitis Carbamazepine is used for partial or
B. Cerebral hemorrhage secondarily generalized tonic-clonic
C. Cerebral vasculitis (GTC) seizures, although it is important
D. Salicylate administration to remember that it can rarely worsen
some generalized epilepsies (including
vv Correct Answer is: D myoclonic and absence epilepsies), simi-
This is a clinical presentation of Reye lar to phenytoin. Its primary mode of
syndrome. It occurs with salicylate (aspi- action is via blockade of sodium chan-
680 Chapter 26 · Toxic Encephalopathy

nels, which leads to a decrease/preven- carbamazepine does not undergo


tion of repetitive firing in depolarized oxidation to epoxide. Carbamazepine
neurons. Side effects include dizziness, on the other hand is oxidized to 10,
vertigo, fatigue, drowsiness, diplopia, 11- carbamazepine epoxide, which is
nystagmus, headache, nausea, vomit- the principal metabolite of carbamaze-
ing, elevated liver function tests, hypo- pine. It is important to remember that
natremia, and ataxia. Serious the 10, 11-carbamazepine epoxide is
idiosyncratic reactions include Stevens-­ pharmacologically active and respon-
Johnson syndrome, leukopenia, and sible for many of the side effects seen
aplastic anemia. with carbamazepine use. Because of
Carbamazepine undergoes liver these differences, oxcarbazepine has
metabolism with renal excretion of less side effects, overall, as compared
metabolites, so caution is advised with to carbamazepine. Oxcarbazepine has
26 kidney or liver failure. less liver enzyme induction, no auto-
Carbamazepine is also a hepatic induction (and can thus be titrated
enzyme inducer, and undergoes auto- more rapidly), and is used for the same
induction. The dose must be titrated seizure types as carbamazepine, having
up gradually to allow tolerance to the same mechanism of action, meta-
develop to its CNS side effects, but also bolic pathways, and side-effect profile.
to avoid early toxicity as carbamazepine Approximately 30% of patients who
“auto- induces” the hepatic enzymes have a history of a rash with carbam-
responsible for its own metabolism. If azepine will also develop a rash when
carbamazepine is started at too high exposed to oxcarbazepine. Valproic
of a dose, or titrated too fast, the result acid inhibits the metabolism of the
would be elevated carbamazepine levels pharmacologically active 10, 11-car-
with accompanying toxicity early on, bamazepine epoxide (the principal
as the hepatic enzymes responsible for metabolite of carbamazepine). Thus,
carbamazepine’s metabolism have not although the carbamazepine level may
been fully activated (auto-induced) yet. It be normal, the patient may experience
is therefore important to remember that toxicity because of elevated 10, 11-
carbamazepine’s half-life decreases from carbamazepine epoxide levels. The 10,
30 h to 10 to 20 h after the first few days 11-carbamazepine epoxide is not rou-
to weeks of use. tinely measured, but can be ordered
Auto-induction is completed after specifically if there are concerns about
3–5 weeks of a fixed dosing regimen. toxicity.
Plasma concentrations decrease in
the first 1 to 2 months, and during zz Suggested Reading
this time, the dose of carbamaze- 55 Koch MW, Polman SK. Oxcarbazepine
pine should be gradually increased. versus carbamazepine monotherapy for
Therefore, carbamazepine would not partial onset seizures. Cochrane Database
be a good option if quick control of Syst Rev.2009; CD006453.
new-­onset, frequent seizures was 55 Porter RJ. How to initiate and maintain
desired. Of note, oxcarbazepine does carbamazepine therapy in children and
not undergo auto- induction and can adults. Epilepsia. 1987;28(Suppl 3):S59-S63.
be titrated faster. Oxcarbazepine is a 55 Purcell TB, Mc Pheeters RA, Feil M, et al.
structural derivative of carbamazepine, Rapid oral loading of carbamazepine in
and is reduced to 10- monohydroxy­ the emergency department. An Emerg
carbamazepine and unlike Med. 2007;50(2):121–26.
Toxic Encephalopathy
681 26
55 Tudur SM, Marson AG, Clough HE, et al. within the previous month. Paint chips
Carbamazepine versus phenytoin mono- can be demonstrate on X-rays within
therapy for epilepsy. Cochrane Database the GI tract or “lead lines” on skeletal
Syst Rev. 2002;(2):CD001911. films (indicative of bone growth arrest).
55 Comprehensive Review in Clinical Skeletal lead concentrations can be
Neurology: a multiple choice question measured by X-ray fluorescence tech-
Book for the Wards and Boards © 2011 niques. Treatment involves chelation
Wolters Kluwer Health Lippincott therapy with dimercaptosucccinic acid
Williams & Wilkins. All rights reserved. (DMSA), calcium disodium ethylene-
diaminetetraacetic acid (CaNa2ETDA),
?? 11. Chronic lead exposure in children and/or British anti-­lewisite (BAL),
most commonly leads to: depending upon the levels. BAL should
A. Neuropsychological symptoms be given prior to EDTA, however, as
B. Sensory neuropathy EDTA may increase the distribution of
C. Motor neuropathy lead into the brain.
D. Optic neuropathy
E. Ataxia zz Suggested Reading
55 Kathuria P. Lead toxicity.Medscape.
vv Correct Answer is: A Neuropsychological 7 http://emedicine.medscape.com/
symptoms article/1174752-oveNiew. Updated 1/29/14.
Lead poisoning has differing clinical fea- Accessed 4/11/14.
tures based on age presentation. 55 Kumar N. Industrial and environmental
Children are more affected because they toxins. Continuum. 2008;14(5):102–37.
absorb lead more effectively compared
to adults. CNS manifestations are more ?? 12. Which of the following agents is most
common in children, but peripheral likely to cause hearing loss and oto-
symptoms are more common in adults. toxicity.
Neuropsychological effects, including A. Vincristine
irritability, fatigue, and impaired mem- B. Cyclophospham ide
ory, attention, and intelligence are com- C. Methotrexat
mon in children with chronic exposure D. Cisplatin
to lead. On the other hand, neuropathy E. Lomustine
is rare.
Motor or sensorimotor neuropathies vv Correct Answer is: D
with subacute and chronic exposures Cisplatin is known to cause ototoxicity
are more common in adults. Lead pois- and hearing loss. Cisplatin is usually
ing is caused by ingestion of paint chips, used in treatment of medulloblastomas.
as lead-­containing residential paints Tinnitus, ototoxicity with hearing loss,
were used in homes built prior to 1978. kidney damage, and myelosuppression
Lead may also leach into drinking water are common side effects. Peripheral
from old lead-­containing plumbing, neuropathy, seizures, and visual impair-
and can be found in soil contaminated ment are rare side effects.
by vehicle exhaust or industrial emis- Vincristine is a vinca alkaloid which
sions. Certain ceramics, cosmetics, used to treat both low and high grade
jewelry, crystal, herbal medications, and gliomas and medulloblastomas. The
other products may also contain lead. most common side effects are constipa-
Blood levels are useful for detection of tion, diarrhea, and peripheral neuropa-
lead poisoning, and indicate exposure thy; but, syndrome of inappropriate
682 Chapter 26 · Toxic Encephalopathy

antidiuretic hormone (SIADH), paralytic disorders Phakomatoses and hamartoneo-


ileus, jaw pain, and seizures occur are plastic syndromes. Morlenbach: Springer-
less side effects. Verlag/Wien; 2008. p. 669–694.
Cyclophosphamide is a nitrogen 55 Chan TSY, Wang X, Spence T, Taylor MD,
mustard, it inhibits DNA replication. Huang A. Embryonal brain cell tumors. In:
Sometimes it has been used to treat Scheinemann K, Bouffet E, editors.
medulloblastoma. The most side effects Pediatric neuro-oncology. New York:
are hemorrhagic cystitis, myelosup- Springer; 2015. p. 127–138.
pression, nausea, vomiting, mucositis, 55 Lee J, Johnston DL. Chemotherapy. In:
or nasal congestion. Hepatotoxicity, Scheinemann K, Bouffet E, editors.
SIADH, and sterility may occur as a rare Pediatric neuro-oncology.
side effects. New York:Springer; 2015. p. 69–89.
Methotrexate works on dihydrofo- 55 Glass TD, Cochrane D, Rassekh SR,
26 late reductase causing DNA and RNA Goddard K, Hukin J. Growing teratoma
synthesis inhibition. It has been used syndrome in intracranial non germinoma-
in the management of high grade glio- tous germ cell tumors (iNGGCTs): a risk
mas and diffuse intrinsic pontine glio- for secondary malignant transformation- a
mas in addition to medulloblastoma. report of two cases. Child Nerv Syst.
Methotrexate can cause myelosuppres- 2014;30(5):953–7.
sion, mucositis, and, in high doses, or 55 Bartels U, Singhal A. Central nervous system
when given intrathecally, it may cause germ cell tumors. In: Scheinemann K,
neurotoxicity. Skin reactions are rare Bouffet E, editors. Pediatric neuro-­oncology.
side effects. New York: Springer; 2015:149–62.
Lomustine is a nitrosurea which ca 55 Mach SC, Ramaswamy V, Wang X, Remke
be used for medulloblastoma. This has M, Sin-Chan P, Chan WSY, Bertrand KC,
also been used for low grade gliomas Merino D, Zayne K. Huang A. Taylor
and, in adults, has been used with pro- MD. Basic science of pediatric brain
carbazine and vincristine in the treat- tumors. In: Scheinemann K, Bouffet E,
ment of grade Ill gliomas. This does not editors. Pediatric neuro-oncology.
tend to cause hearing loss or ototoxicity. New York: Springer; 2015. p. 59–67.
55 Jones DTW, Korshunov A, Pfister SM,
zz Suggested Reading Tayllor MD, Northcott
55 Jozwiak S, Migone N, Ruggieri M. The PA. Medulloblastoma and CNS primitive
tuberous sclerosis complex. In: Ruggieri neuroectodermal tumors. In: Cagle PT,
M, Pascual­Castroviejo I, Di Rocco C, Matthias KA, Zagzag D, editors. Molecular
editors. Neurocutaneous disorders pathology of nervous system tumors.
Phakomatoses and Hamartoneoplastic New York: Springer; 2015:121–42.
syndromes. Morlenbach: Springer-Verlag/ 55 Vanan Ml, Mehta V, Eisenstat DD. Diffuse
Wien;2008. p. 181–227. intrinsic pontine glioma. In: Scheinemann K,
55 Tabori U, Laberge A-M, Ellezam B, Carret Bouffet E, editors. Pediatric neuro-­oncology.
A-S. Cancer predisposition in children New York: Springer; 2015. p. 117–26.
with braintumors. In: Scheinemann K, 55 Rutkowski S, Bode U, Deinlein F,
Bouffet E, editors. Pediatric Neuro-­ Ottensmeier H, Warmuth-Metz M,
Oncology. New York: Springer; 2015. Soeroenson N, Grade N, Emser A, Pietsch
p. 69–89. T, Wolff JE, Kortmann RD, Kuehl
55 Gorlin, RJ. Nevoid basal cell carcinoma. J. Treatment of early childhood medullo-
In: Ruggieri M, Pascual-Castroviejo I, Di blastoma by post-­operative chemotherapy
Rocco C, editors. Neurocutaneous alone. N Eng J Med. 2005;352(10):978.
Toxic Encephalopathy
683 26
55 Geyer JR, Sposto R, Jennings M, Boyett vv Correct Answer is: A
JM, Axtell RA. Brieger D, Broxso E, Oxcarbazepine is a structural derivative
Donahue B, Finlay JL, Goldwein JW, of carbamazepine, and is reduced to
Heier LA, Johnson DM, Mazewski C, 10-monohydroxy-carbamazepine and
Miller DC, Packer R, Puccetti D, Radcliffe unlike carbamazepine does not undergo
J, Tao ML, Shiminski-Maher oxidation to epoxide. Carbamazepine
T. Multiagent chemotherapy and deferred on the other hand is oxidized to 10,
radiotherapy in infants with malignant 11-carbamazepi ne epoxide, which is
brain tumors: a report from the Children’s the principal metabolite of carbamaze-
Cancer Group. J Clin Oneel. pine. 10, 11-carbamazepine epoxide is
2005;23(30):7621. pharmacologically active and responsi-
55 Wells EM, Packer RJ. Pediatric Brain ble for many of the side effects seen
Tumors. Continuum: Lifelong Learning in with carbamazepine use.
Neurology 2015; 21(2):373–96. Oxcarbazepine has less side effects,
55 Fleming AJ. Rare tumors of the central overall, as compared to carbamazepine.
nervous system in children. In: It has less liver enzyme induction, no
Scheinemann K, Bouffet E, editors. auto induction (and can thus be titrated
Pediatric Neuro-Oncology. New York: more rapidly), and is used for the same
Springer; 2015. p. 185–94. seizure types as carbamazepine, having
55 Evans DW. Li-Fraumeni syndrome. In: the same mechanism of action, metabolic
Up-to-date, Wen PY, Ross ME, editors. pathways, and side-effect profile. There
Up-to-date, Waltham. Accessed on 22 Nov is a cross reaction between the two drugs
2015. in 30% of patients who have a history of
55 lchimura K, Nishikawa R, Matsutani a rash with carbamazepine when they
M. Molecular markers in pediatric have been exposed to oxcarbazepine.
neuro-­oncology. Neuro­ Oncology. Valproic acid inhibits the metabolism
2012;14(Suppl 4):iv90–iv99. 7 https://doi. of the pharmacologically active 10,
org/10.1093/neuonc/nos204. 11-­carbamazepine epoxide. Thus,
55 17. Vora NL. Melanoma and BRAF. eMedi- although the carbamazepine level may
cine. 7 http://emedicine.medscape.com/ be normal, the patient may experience
article/2045059-overview). Updated toxicity because of elevated 10, 11- carba-
January 14 2014. Accessed 23 Nov 2015. mazepine epoxide levels. The 10, 11-car-
55 Daras M, Kaley TJ. Benign brain tumors bamazepine epoxide is not routinely
and tumors associated with phakomatoses. measured, but can be ordered specifically
Continuum Lifelong Learn Neurology if there are concerns about toxicity.
2015;21(2):373–96.
zz Suggested Reading
?? 13. Oxcarbazepine is a structural deriva- 55 Koch MW, Polman SK. Oxcarbazepine
tive of carbamazepine. Which of the versus carbamazepine monotherapy for
following is true about oxcarbazepine partial onset seizures. Cochrane
in comparison to carbamazepine? Database Syst Rev. 2009;CD006453.
A. Is not metabolized to an epoxide 55 Porter RJ. How to initiate and maintain
B. Is not a hepatic enzyme inducer carbamazepine therapy in children and
C. Indicated in both partial and gen- adults. Epilepsia. 1987;28(Suppl 3):559–63.
eralized epilepsy 55 Purcell TB, Mc Pheeters RA, Feil M, et al.
D. No risk of hyponatremia Rapid oral loading of carbamazepine in
E. No risk of rash as seen with carba- the emergency department. Ann Emerg
mazepine Med. 2007;50(2):121–26.
684 Chapter 26 · Toxic Encephalopathy

55 Tudur SM, Marson AG, Clough HE, et al. vv Correct Answer is: D
Carbamazepine versus phenytoin mono- Carbamazepine is used for partial or
therapy for epilepsy. Cochrane Database secondarily generalized tonic-clonic
Syst Rev.2002;(2):CD001911. (GTC) seizures, it worsen some general-
55 Comprehensive Review in Clinical ized epilepsies (including myoclonic and
Neurology: a multiple choice question absence epilepsies), like phenytoin. It
Book for the Wards and Boards ©2011 blocks sodium channels, which leads to
Wolters Kluwer Health Lippincott a decrease/prevention of repetitive fir-
Williams & Wilkins. All rights reserved. ing in depolarized neurons. Side effects
include dizziness, vertigo, fatigue,
?? 14. Of the medications listed below, drowsiness, diplopia, nystagmus, head-
which of the following would not ache, nausea, vomiting, elevated liver
lower the seizure threshold? function tests, hyponatremia, and
26 A. Amoxapine ataxia. Stevens-­Johnson syndrome, leu-
B. Venlafaxine kopenia, and aplastic anemia are some
C. Mirtazapine serious idiosyncratic ­reactions.
D. Buspirone Carbamazepine undergoes liver
E. Diazepam metabolism with renal excretion of
metabolites, dose adjustment has to be
vv Correct Answer is: E done in kidney or liver failure.
Buspirone, bupropion, the tricyclics, Carbamazepine is also a hepatic
amoxapine, and clomipramine should enzyme inducer, and undergoes auto-
all be avoided. Amoxapine, which is a induction. The dose must be titrated
TCA inhibiting NE and SHT reuptake, up gradually to allow tolerance to
does lower the seizure threshold. develop to its CNS side effects, but also
Venlafaxine is an SNRI, and has also to avoid early toxicity as carbamazepine
been shown to lower the seizure thresh- “autoinducer” the hepatic enzymes
old. Diazepam is a benzodiazepine and responsible for its own metabolism. If
may be used to treat seizures acutely, carbamazepine is started at too high
and would therefore not lower the sei- of a dose, or titrated too fast, the result
zure threshold. would be elevated carbamazepine lev-
els with accompanying toxicity early on,
zz Suggested Reading as the hepatic enzymes responsible for
55 Lee, HW, Khoshbin S. Seizure Disorders carbamazepine’s metabolism have not
Chapter 76, page 1050, Mass General been fully activated (autoinducer) yet.
Comprehensive Clinical Psychiatry. It is therefore important to remember
that carbamazepine’s half-life decreases
?? 15. Which of the following is true regard- from 30 h to 10 to 20 h after the first few
ing carbamazepine? days to weeks of use.
A. It inhibits its own metabolism Autoinduction is completed after
B. For new-onset, frequent seizures, 3–5 weeks of a fixed dosing regimen.
carbamazepine is a good option Plasma concentrations decrease in
as initial antiepileptic therapy the first 1 to 2 months, and during this
C. It has no effect on its own metab- time, the dose of carbamazepine should
olism be gradually increased. Therefore,
D. It induces its own metabolism carbamazepine would not be a good
E. It has no hepatic metabolism option if quick control of new-­onset,
Toxic Encephalopathy
685 26
frequent seizures was desired. Of note, 55 Purcell TB, Mc Pheeters RA, Feil M, et al.
oxcarbazepine does not undergo auto- Rapid oral loading of carbamazepine in
induction and can be titrated faster. the emergency department. An Emerg
Oxcarbazepine is a structural derivative Med. 2007;50(2):121–6.
of carbamazepine, and is reduced to 55 Tudur SM, Marson AG, Clough HE, et al.
10-­monohydroxy-carbamazepine and Carbamazepine versus phenytoin mono-
unlike carbamazepine does not undergo therapy for epilepsy. Cochrane Database
oxidation to epoxide. Carbamazepine Syst Rev. 2002;(2):CD001911.
on the other hand is oxidized to 10,
11-carbamazepi ne epoxide, which is ?? 16. An 18-year-old male was brought to
the principal metabolite of carbamaze- the ED by his college peers because
pine. 10, 11-carbamazepine epoxide is of his bizarre behavior, specifically
pharmacologically active and respon- being seen wandering around cam-
sible for many of the side effects seen pus. On examination, blood pressure is
with carbamazepine use. 180/90 mm Hg and pulse is 110 beats/
Oxcarbazepine has less side effects, min. He is diaphoretic, agitated, but
overall, as compared to carbamazepine. responding to internal visual stimuli.
It has less liver enzyme induction, no His pupils are dilated, both arms
auto induction (and can thus be titrated tremor, and there is diffuse myoclonus.
more rapidly), and is used for the same What is the most likely cause of his
seizure types as carbamazepine, having symptoms?
the same mechanism of action, meta- A. Cocaine intoxication
bolic pathways, and side-effect profile. B. Ethanol withdrawal
There is a cross reaction between the C. N-hexane inhalation
two drugs in 30% of patients who have D. MDMA (ecstasy) intoxication
a history of a rash with carbamazepine E. Opiate withdrawal
when they have been exposed to oxcar-
bazepine. Valproic acid inhibits the vv Correct Answer is: D
metabolism of the pharmacologically The boy is presenting with a hyperactive
active 10, 11-­carbamazepine epoxide. encephalopathy and systemic signs of
Thus, although the carbamazepine level both hyper-adrenergic activity (tachy-
may be normal, the patient may experi- cardia, hypertension) and excessive
ence toxicity because of elevated 10, serotonin activity (hallucinations,
11- carbamazepine epoxide levels. The tremor). Of the options listed above,
10, 11-carbamazepine epoxide is not MDMA intoxication is most likely to
routinely measured, but can be ordered cause this constellation of symptoms
specifically if there are concerns about (increased catecholamine and serotonin
toxicity. activity).

zz Suggested Reading zz Suggested Reading


55 Koch MW, Polman SK. Oxcarbazepine 55 Brust JC. Abused agents: acute effects,
versus carbamazepine monotherapy for withdrawal, and treatment. Continuum.
partial onset seizures. Cochrane Database 2004;10:14–47.
Syst Rev. 2009;CD006453.
55 Porter RJ. How to initiate and maintain ?? 17. You are evaluating a 17-year-old male
carbamazepine therapy in children and with a history of progressive sensory
adults. Epilepsia. 1987;28(Suppl 3):S59-S63. loss and paresthesia affecting his
686 Chapter 26 · Toxic Encephalopathy

hands and feet and gait unsteadiness position sense is commonly seen with
for 3 months. You examined him and inhaling n-hexane or related com-
you demonstrated that, he has loss of pounds (also known as “huffing”). Those
pinprick and temperature sensation syndrome could be rapid and progress
up to the knee bilaterally; decreased over weeks, sometimes resembling
vibratory sense at the great toe bilat- Guillain-­Barre syndrome. While ethanol
erally; trace reflexes throughout; and abuse can be associated with neuropa-
a positive Romberg test.   thy, this is more of a chronic process
Abuse of which of the following than the subacute pattern described
agents is the reason for his symp- here. Nitrous oxide abuse can also be
toms? associated with distal paresthesia and
A. N-hexane sensory loss though examination find-
B. Ethanol ings typically show more involvement
26 C. Heroin of vibration and joint position sense
D. MDMA (ecstasy) (due to posterior column dysfunction)
E. Nitrous oxide as opposed to the findings described
here.
vv Correct Answer is: A
Distal symmetric polyneuropathy pri- zz Suggested Reading
marily affecting small fibers, resulting in 55 Weimer LH. Spinal and neuromuscular
sensory loss that involves pain and tem- complications of substance abuse.
perature more so than vibration or joint Continuum. 2004;10:115–43.
687 27

Traumatic Brain Injury

© Springer International Publishing AG, part of Springer Nature 2018


Y. M. Awaad, Absolute Pediatric Neurology, https://doi.org/10.1007/978-3-319-78801-2_27
688 Chapter 27 · Traumatic Brain Injury

?? 1. A 1 month old baby has been colicky noid blood, and locations of
after he was discharged from the reg- intraparenchymal blood indicates a
ular nursery. He has a normal eating traumatic brain injury, and the social
pattern, is growing, fixes and follows story is suspicious for but never diag-
his mother’s face, startles in response nostic of non-accidental trauma. You
to loud noise. He has attacks of crying cannot assume that shaking, if it
vigorously during which he pulls his occurred, was the only traumatic event
legs up and appears to be uncomfort- or that the circumstances that led to
able. His mother cannot console him the baby’s current state constituted an
during those attacks, but eventually, isolated incident. Skeletal survey is
he falls asleep and wakes up fine. His typically performed in children in
primary care physician assured the whom non-accidental trauma is
mother that he will outgrow these ­suspected.
attacks.  
His parents decide to go out zz Suggested Reading
27 for dinner. They are grateful to the 55 Preer G, Sorrentino D, Newton AW. Child
17 year old girl who lives next door, as abuse pediatrics: prevention, evaluation,
she has agreed to stay with the baby and treatment. Curr Opin Pediatr.
to give them a well-deserved respite. 2012;24(2):266–73.
When they come home, they pay the
girl and thank her. As soon as she left, ?? 2. A 4-month-old girl was seen by para-
they go into the baby’s room and medics because she vomited and
notice the baby looks very pale and then stopped breathing at home. The
is breathing irregularly. They turn on mother’s boyfriend had been babysit-
the light and try to arouse the baby, ting for the evening while the mother
but he does not wake up. The mother worked. He left the house suddenly
screams as the father calls 9-1-1.   when the mother returned. The
He is rushed by ambulance to the baby had no prior history for signifi-
hospital. His exam reveals a coma- cant health problems. However, her
tose infant with stable vital signs and mother reported that she currently
retinal hemorrhages. His skin, chest, has an upper respiratory infection
abdomen, and limbs appear normal. and has not been sleeping well. In the
A head CT scan shows left occipital, Emergency Room, the child is breath-
right temporal, and bifrontal intrapa- ing regularly, but is unresponsive. A
renchymal hemorrhages and a mod- head CT reveals a small amount of
erate amount of subarachnoid blood.   acute subdural bleeding in the inter-
You ordered a: hemispheric fissure.  
A. Lumbar puncture What is you’re the most appropriate
B. Stat head MRI next step in ­evaluation?
C. Spine MRI A. Funduscopic examination
D. Serum S100b B. Wrist X-ray for bone age determi-
E. Skeletal survey nation
C. Serum copper and ceruloplasmin
vv Correct Answer is: E levels
The most likely diagnosis is Shaken D. Cortisol serum level
baby syndrome. The clinical presenta- E. DNA mutation analysis for osteo-
tion of retinal hemorrhages, subarach- genesis imperfecta
Traumatic Brain Injury
689 27
vv Correct Answer is: A C. This boy should be evaluated by
Shaken baby syndrome and non-­ his coach, and sent home with his
accidental trauma, are common, and older brother.
accounts for a large proportion of trau- D. Concussion occurs as a result of
matic brain injury (TBI) cases in children penetrating forces applied to the
younger than 2 years of age. Usually, the brain.
history at presentation does not match E. After the first concussion, the risks
the pattern of physical injury. Symptoms associated with a second concus-
are non-specific, including vomiting, sion are lower.
apnea, seizures, and depressed con-
sciousness. Subdural hemorrhage, vv Correct Answer is: B
mainly in the interhemispheric fissure, is Concussion (or mild traumatic brain
the single most common feature with injury) associated with official sports is
brain imaging, but cerebral contusions, common in children and teenagers. Ice
stroke, and hypoxic-ischemic injury may hockey and American football appear to
also occur. Other problems may include have the highest incidence rates. It has
retinal hemorrhage, injury to the cervi- been established that athletes (both
cal spine or abdomen, and fractures. pediatric and adult) under-report the
occurrence of symptoms resulting from
zz Suggested Reading concussion. More time to recover to
55 Chiesa A, Duhaime AC. Abusive head baseline is needed for younger athletes,
trauma. Pediatr Clin N Am. 2009;56:317– and more conservative return-to-­play
31. decisions are appropriate for the pediat-
ric age group. Children with one concus-
?? 3. A 12-year-old boy experiences head sion are at higher risk for a second
to head trauma with another boy concussion. Neuropsychological testing
while playing basketball. The boy lay may reveal deficits even in the absence
motionless on the ground, while the of reported symptoms.
second boy walked away without dif-
ficulty. Referees and coaches found zz Suggested Reading
him able to open his eyes to shout, 55 Meehan WP III, Bachur RG. Sport-related
but unable to verbally respond for at concussion. Pediatrics. 2009;123:114–23.
least 2 min. He vomited on the field,
but then was able to stand and walk ?? 4. A 13 year old football player came to
to the sidelines with his coaches’ the office 14 days after sustaining a
assistance.   concussion while playing football. No
Which one of the following is cor- loss of consciousness was reported but
rect? he was stunned after getting hit head
A. Basketball is the sport most likely to head. He has not played for 10 days
to result in concussion (mild trau- and would like to return to play. His
matic brain injury) in the US, as physical examination was unremark-
defined as an incidence rate for all able on examination. However he has
children participating in the sport. difficulty with serial 7’s and recalling
B. Neuropsychological tests 1 week 3/3 words. He has delay in processing
after the incident may show a information. At baseline he is an above
decline compared to pre-injury average student. His family would like
baseline values. to know the risk factors for concussion.
690 Chapter 27 · Traumatic Brain Injury

What is your response? ?? 5. A 9 year old was climbing a tree when


A. Female sex is a risk factor for con- she missed a branch and fell 18 feet
cussion onto the grass below onto her back.
B. Prior concussions are a risk factor She had a brief loss of consciousness.
for concussion She was transferred to the emergency
C. The rate of concussion is higher in room wherein route she had a GCS of
practice than in games 12. In the Emergency Department she
D. The football helmet decreases the vomited several times. She has head-
rate of concussions ache. Vital signs were normal. She was
E. Mouth guards decrease the risk of agitated. She did not have neck point
concussion tenderness. On neurological examina-
tion she had a normal funduscopic
vv Correct Answer is: B examination and pupillary response.
Concussions counts for 8.9% of high However she did not fix and follow and
school athletic injuries and 5.8% of col- did not blink to threat. Opticokinetic
27 legiate athletic injuries, with an inci- nystagmus was absent. The rest of the
dence of 0.23 per 1000 and 0.43 per neurological examination was non-
1000 athlete exposures, respectively. focal. Head CT was negative for bleed or
Female sex confers an increased risk of fracture. She was evaluated by ophthal-
concussion. While overall rates of con- mology and his examination did not
cussion at the high school and colle- reveal any abnormalities. Three hours
giate levels are higher in male athletes, later she was able to track a pen light.
when sports are examined where the The most likely reason for the visual
men’s and women’s games are similar disturbance is:
(including soccer and basketball), A. Ocular migraine
female athletes have a higher risk of B. Post-traumatic Transient Cortical
concussion. Multiple prospective stud- Blindness
ies have identified a history of prior con- C. Conversion reaction
cussion as a risk factor for subsequent D. Retinal detachment
concussion. In high school athletes, a E. Posterior reversible encephalopa-
greater-than-twofold increase in con- thy syndrome
cussion rate was seen with history of
concussion, even when adjusting for vv Correct Answer is: B
sport contact level, grade, and body Post-traumatic transient cortical blindness
mass index. Helmets reduce the acceler- following a head injury is a distinct phe-
ation of the head from collisions and nomenon first described by Bodian. It usu-
decrease severe head injuries, but the ally occurs in children after a mild head
rate and severity of concussion are not injury with occipital trauma being the
affected by different helmets. Mouth most common cause. Visual loss can be
guards in multiple sports and face complete (blindness) or partial (able to
shields in ice hockey decrease dental see light). The examination can be normal.
and orofacial injuries but have no effect The majority of cases resolve within hours.
on concussion risk.
zz Suggested Reading
zz Suggested Reading 55 Gleeson et al. Post traumatic transient
55 Scopaz KA. Risk modifiers for concussion cortical blindness in children: a report of
and prolonged recovery. Sports Health. four cases and a review of the literature.
2013;5(6):537–41. J Accid Emerg Med. 1994;11:250–52.
Traumatic Brain Injury
691 27
?? 6. A three-week-old boy came to age. Retinal hemorrhages are seen in
the Emergency Department with 11–40% of full-term newborn infants
decreased oral intake. He was born after vaginal delivery and are seen more
full term. The mother has gestational commonly after complicated or trau-
diabetes so he was large for gesta- matic births such as a vacuum extrac-
tional age. The delivery was compli- tion.
cated requiring vacuum extraction
and forceps delivery. He was admitted zz Suggested Reading
to the newborn nursery for 2 days and 55 Swaiman. Pediatric Neurology Principles
then discharged home. Over the past and Practice. 1996.
several days he has not been breast
feeding as vigorously and has had a ?? 7. A 3-month-old boy was seen in the
decreased amount of diapers. He was emergency department 30 min after
lethargic but was arousable. Oph- he stopped breathing and had a
thalmological examination revealed seizure at her daycare. His stepfather
retinal hemorrhages. The rest of the says that he rolled off of the couch
examination was unremarkable.   onto the ground, cried briefly, then
Which of the following statements stopped breathing, became cyanotic
is true? and stiff. This lasted 3 min and then
A. Retinal hemorrhages can be seen he became limp and unarousable. In
in full-term newborn infants after the emergency department, his vital
vaginal delivery signs are stable. There is no external
B. Subdural hematomas cannot be evidence of trauma. He does not
an indication to a difficult delivery arouse with vigorous physical stimu-
C. Extracranial injuries such as caput lation. Anterior fontanelle is bulging
succedaneum and cephal-hema- and tight. Pupils are 4 mm and slug-
tomas are uncommon as birth gish, oculocephalic reflexes are intact.
related cranial injury Fundus examination shows bilateral
D. Complicated skull fractures are retinal hemorrhages. Muscle tone
not associated with serious intra- and tendon reflexes are decreased.
cranial pathology Head CT is obtained, showing extra
E. Epidural hematomas are not seen axial effusions and a subdural hem-
in traumatic births orrhage in the left frontotemporal
area. Shaken infant syndrome (Non-
vv Correct Answer is: A accidental trauma) is suspected.  
Many different injuries can happened Which of the following metabolic
during the birth process. Extracranial conditions must be excluded as an
injuries are common including caput alternative explanation?
succedaneum, cephal-hematomas and A. Glutaric acidemia
sub-galeal hematomas. In general skull B. Menkes disease
fractures are either linear or depressed C. Leigh’s disease
and complicated fractures indicate seri- D. Maple syrup urine disease
ous intracranial pathology. Epidural and E. Neonatal adrenoleukodystrophy
subdural hematomas, subarachnoid
hemorrhage, and intracerebral hemor- vv Correct Answer is: A
rhage have all been reported. Subdural The clinical presentation supported by
hematomas are common with a diffi- the history, exam, and CT findings sug-
culty delivery or a large for gestational gestive of shaken baby syndrome (SBS),
692 Chapter 27 · Traumatic Brain Injury

a cause of non-accidental head injury ischemic stroke. The boy was partici-
(NAHI). It is estimated that 25% to 50% pating in skateboarding competitions.
of all children under 2 years of age He always attempting risky jumps
admitted to the hospital with a history that are avoided by other skateboard-
of head trauma are abuse victims. It is ers. He has had multiple falls in the
difficult to categorize head trauma as past, but has never been lost con-
non-­accidental without a perpetrator’s sciousness. He is otherwise healthy.
confession, NAHI is often a diagnosis of Toxicology screen was negative and
exclusion. Shaken baby syndrome is one no history for substance abuse, an
of the most difficult causes of NAHI to angiogram shows an aneurysm of the
diagnose. External bruising is not seen supraclinoid portion of the left inter-
in a significant minority (21%) of fatal nal carotid artery, perhaps secondary
NAHI cases and a history of previous to earlier traumatic injury.  
maltreatment can be absent in up to In order to prevent secondary com-
40% of all cases. SBS refers to a constel- plications from vasospasm in this
27 lation of inflicted injuries mostly seen in patient, which of the following med-
young children (mean age about ications should be administered?
4 months), caused by violent shaking, A. Heparin
and is typically characterized by subdu- B. Nimodipine
ral hematomas and retinal hemor- C. Clonidine
rhages. On the other hand, retinal D. Phenytoin
hemorrhages can be seen in other con- E. Propranolol
ditions, including osteogenesis imper-
fect, homozygous protein C deficiency, vv Correct Answer is: B
Terson syndrome (intraocular bleeding Ruptured intracranial aneurysms are rel-
associated with acute intracranial hem- atively rare in the patients less than
orrhage), Hermansky-Pudlak syndrome 18 years of age. In contrast to adult
(an autosomal recessive condition char- patients, in whom sporadic saccular
acterized by albinism and platelet aneurysms predominate, aneurysms in
abnormalities), hemorrhagic disease of children are more diverse, including
the newborn, and Glutaric Acidemia giant, fusiform, dissecting or infectious
type 1. aneurysms. Posterior cerebral circula-
tion is usually more involved. Familial
zz Suggested Reading predisposition is 10%. The prognosis in
55 Togioka BM, Arnold MA, et al. Retinal children may be more favorable relative
hemorrhages and shaken baby syndrome: to that seen in adults even with severe
an evidence based review. J Emerg Med. subarachnoid hemorrhage (SAH). Many
2009;37:98–106. complications following SAH are possi-
ble, including hydrocephalus, seizures,
?? 8. A 15-year-old boy was seen at the syndrome of inappropriate anti-diuretic
hospital with severe headache lead- hormone (SIADH) or vasospasm leading
ing to unconsciousness. CT scan of to ischemic infarction. Nimodipine is
the head in the Emergency Depart- approved by FDA for prevention of
ment showed subarachnoid hemor- vasospasm in this condition.
rhage, with a significant amount of
blood in the cisterns at the base of zz Suggested Reading
the brain. There was no evidence for 55 Lasjaunias P, Wuppalapati S, Alvarez H,
intra-parenchymal hemorrhage or Rodesch G, Ozanne A. Intracranial
Traumatic Brain Injury
693 27
aneurysms in children aged under D. A loading dose of Fosphenytoin I.V.
15 years: review of 59 consecutive children E. ICU admission, with hyperventila-
with 75 aneurysms. Childs Nerv Syst. tion to pCO2 25
2005;21:437–50.
vv Correct Answer is: C
?? 9. A 16-year-old boy was drinking The peak incidence of epidural hema-
alcohol with his cousin, at a picnic toma (EDH) is in the second decade. EDH
in a public park. A fight between the are relatively not ommon in the elderly.
patient and his cousin ensued, and Non-­operative management with care-
the patient was struck in the head ful ICU observation and serial brain
with a baseball bat, with immediate imaging is an option for those patients
loss of consciousness. He was taken to who are awake and responsive, without
the Emergency Department, and on focal deficits, and with a maximal EDH
examination he had an enlarged and thickness of less than 10–15 mm. Others
non-reactive right pupil, and no motor should be considered for surgical treat-
response on the left side of the body. ment as soon as possible.
An emergency head CT scan is shown.
zz Suggested Reading
55 Bullock MR, Chesnut R, Ghajar J, et al.
Surgical management of acute epidural
hematomas. Neurosurgery.
2006;58(Suppl):S2–7.

?? 10. An 8-year-old girl was transferred to the


Emergency Department after she was
struck by a car while running across
the street. At the scene, EMS found
her to be awake, but distraught and
complaining of neck pain. She was
transported with her head and neck
immobilized in neutral position. Upon
examination, she is breathing well, has
stable vital signs, and appears to be
moving all four extremities equally. Cer-
vical spine radiology shows abnormal
..      Fig. 27.1 Non-contrast CT scan of an acute widening of the interspace at C1-C2.  
epidual hematoma, right fronto-temporal area, due to What is your next step in manage-
trauma. (Own work. Jpogi. Wikipedia)
ment?
A. Lumbar puncture to exclude sub-
Which of the following is the best arachnoid hemorrhage
management approach? B. Urgent tracheostomy
A. ICU admission, careful observa- C. Continued immobilization of the
tion, and serial head CT scans. head and neck, careful observation,
B. ICU admission with osmotic and neurosurgical consultation.
therapy for increased intracranial D. Flexion and extension views of the
pressure. neck under fluoroscopy
C. Surgical evacuation of the hema- E. Ophthalmology consultation for
toma as soon as possible. dilated fundus examination
694 Chapter 27 · Traumatic Brain Injury

vv Correct Answer is: C and is normal. Past medical history is


Cervical spine (C-spine) injuries in pre-­ remarkable for migraine headaches.  
pubertal children are relatively not com- What is your diagnoses?
mon in comparison to older age groups. A. moderate traumatic brain injury
In young children, C-spine injuries are B. confusional migraine
most likely to occur at the cranio-cervi- C. grade 1 concussion
cal junction or upper vertebral seg- D. grade 2 concussion
ments C1-C4. Children may have severe E. grade 3 concussion
Disruption of ligaments or other soft
supporting tissues may occur in chil- vv Correct Answer is: E
dren without obvious abnormalities of The most likely diagnosis in this case is
the vertebrae on radiographs. Such soft grade 3 concussion. According to AAN
tissue injuries can be detected by Practice Parameters, concussion is a
selected MR sequences, if indicated. trauma induced alteration in mental sta-
C-spine stability has to be determined tus that may or may not involve loss of
27 by the appropriate studies if the chil- consciousness. The practice parameter
dren are unresponsive, intoxicated, have using a scale to help differentiate the
abnormal findings on neurological severity of the concussion. Scales were
examination, or if they have a neck pain, reviewed and a grading scale adopted
and they should remain immobilized. that was dependent on the duration of
The role of corticosteroid administration the symptoms and the absence or pres-
for neuroprotection following spinal ence of loss of consciousness. Grade 1
injury remains contentious. concussion has transient symptoms
without loss of consciousness and
zz Suggested Reading symptoms lasting less than 15 min. A
55 Platzer P, Jaindl M, Thalhammer G, et al. grade 2 concussion has symptoms last-
Cervical spine injuries in pediatric ing more than 15 min without loss of
patients. J Trauma. 2007;62:389–96. consciousness. A grade 3 concussion is
55 Hall ED, Springer JE. Neuroprotection and loss of consciousness of any duration.
acute spinal cord injury: a reappraisal. While he has a history of migraines, the
NeuroRx. 2004;1:80–100. fact that he experienced loss of con-
sciousness would make this diagnosis
?? 11. A 16-year-old boy who is a goalie on incorrect. Moderate TBI occurs when a
a field hockey team is knocked to the patient has a Glasgow Coma Scale of
ground and strikes his head. He lost 9–12 with prolonged loss of conscious-
consciousness and doesn’t respond ness and/or neurologic deficit.
immediately. He became conscious
after a few seconds. His coach helped zz Suggested Reading
him to get up and walked him to the 55 Practice Parameter: the management of
sideline. Upon evaluation he was con- concussion in sport. Neurology.
fused and he did not recall the injury. 1997;48(3):581–85.
He felt dizzy and has a headache. Half 55 Geocadin RG. Traumatic brain injury. In:
an hour later, a repeat examination Bhardwaj A, Mirski MA, Ulatowski JA,
is performed. He still does not recall editors. Handbook of neurocritical care.
the event, but is no longer dizzy or
unsteady on his feet but still having ?? 12. A 5-month old baby was seen by a
headache. Family took him to the ER pediatric neurologist for evaluation.
where a head CT scan is performed His mother noticed that he is not
Traumatic Brain Injury
695 27
grasping with his left hand as much as with Erb’s palsy present with weakness
he is with his right hand. Neurological of shoulder abduction, elbow flexion
exam showed a healthy child without and arm supination and babies hold
dysmorphic features. He has normal their arms in an adducted and internally
truncal tone and he makes eye con- rotated position (“waiter’s tip position”).
tact with objects presented to him. On the other hand, a Klumpke palsy can
He has a strong grasp on the right but result from hyper-­abduction of the
cannot grasp objects on the left. He affected arm, often in association with a
can lift his arm but does not use his challenging arm presentation during
hand muscles. He has normal sym- delivery. Infants present with intact
metric tone proximally in the arms, proximal arm strength but weakness of
and decreased distally in the left arm. the hand if C8/T1 roots got affected. A
Biceps reflexes are present bilaterally. congenital Horner syndrome may also
The left pupil is also smaller than the result from the second order neurons
right, and does not dilate well upon for sympathetic supply to the pupil dila-
turning the light off.   tor muscle arising at the C8/T1 level of
Which of the following features from the spinal cord.
the birth history would be most
anticipated? zz Suggested Reading
A. Traction on the infant’s neck as a 55 Ferrante MA. Brachial Plexopathies.
complications from shoulder dys- CONTINUUM: Lifelong Learning in
tocia. Neurol. 2014;20(5):1323–42.
B. Right middle cerebral artery
infarction in the infant. ?? 13. A 15-year-old male lost unconscious-
C. Perinatal asphyxia to the infant. ness for about 20 s after being hit in
D. Left arm presentation during a dif- the head by another player during
ficult vaginal birth. a hockey game. He was dazed for
E. Traumatic right subdural hema- another 15 min after regaining con-
toma during vaginal delivery sciousness.  
What is the correct statement regard-
vv Correct Answer is: D ing neuropsychological testing for
The baby has asymmetric left arm weak- concussion per AAN’s guidelines?
ness. Localization to an upper or lower A. Standardized Assessment of
motor neuron cause has to be deter- Concussion (SAC) has the highest
mined. Cortical injury, including trauma, sensitivity in detecting subtle cog-
arterial stroke or diffuse hypoxic-isch- nitive deficits.
emic insult can easily be excluded B. Commonly used cognitive evalu-
because there are no features of upper ation tools include the “stick drop”
motor neuron disease on exam. The test and the King-Devick Test of
remaining choices represent mecha- visual tracking.
nisms for the two most common obstet- C. Cognitive restructuring is the
rically related brachial plexopathies, standard method of neuropsycho-
affecting up to 2 per 1000 live births, logical assessment immediately
Erb’s palsy, the most common of these, after a concussive head injury.
arises from injury to the upper plexus D. The most appropriate use of pre-
(C5/6) due to traction of the head away injury clinical tools is as a source of
from the shoulder, stretching the upper data to be included as part of a com-
trunk of the brachial plexus. Children prehensive neurologic evaluation.
696 Chapter 27 · Traumatic Brain Injury

E. Standard neuropsychological ment of concussion in Sports. Report of


tests can be administered and the Guideline Development Subcommittee
interpreted by a trained non-­ of the American Academy of Neurology.
neuropsychologist. Neurol. 2013;80:2250.
55 Scorza K, Raleigh M, O’Connor F. Current
vv Correct Answer is: D Concepts in Concussion: evaluation and
Stick drop test “the clinical reaction Management. Am Fam. Physician.
time” and the King-Devick Test of visual 2012;85(2):123.
tracking along with balance tests such 55 Kutcher J, Giza C. Sports Concussion
as the Balance Error Scoring System Diagnosis and Management. Continuum
(BESS) are used for physical evaluation Lifelong Learning Neurol.
and not cognitive evaluation. Common 2014;20(6):1552.
preinjury cognitive tests include com-
puterized tools such as the Axon Sports ?? 14. A 14 year old boy had a sudden sharp,
Computerized Cognitive Assessment throbbing pain in his left upper arm
27 Tool (CCAT) and the Immediate Post-­ 1 week ago and the pain did not dis-
Concussion Assessment and Cognitive appeared. He was not able to play on
Testing (ImPACT) assessment tool, and the basketball team since that time.
paper-and-pencil tools such as the Pain does not disappear by rest, and
Standardized Assessment of Concussion activity aggravates his discomfort. He
(SAC). In general, none of the preinjury has some improvement in his pain in
evaluation tool has been shown to be the past 3 days. On examination there
superior over another. Preinjury cogni- is weakness in shoulder muscles, tri-
tive assessment is useful both in assess- ceps and his wrist extensors. He has
ing the severity of effects of concussion good strength in wrist flexion.  
and recovery of symptoms as compared What is his diagnosis and where is
to baseline. However it is not specific the lesion?
and should be used as part of a compre- A. Upper brachia! Plexus – Brachial
hensive neurologic evaluation. Plexitis
Although neuropsychological test- B. Upper and lower brachia! Plexus
ing may be administered by a trained C. Spinal cord compression – C4-C6
non-­neuropsychologist, only a neuro- D. Spinal cord compression C8-T1
psychologist should interpret them.
Cognitive restructuring is not a vv Correct Answer is: A
test for evaluation but a form of brief Sudden onset of acute pain without
psychological counseling that consists trauma at the time, is an indication of a
of education, reassurance, and reattri- brachial neuritis. The clinical presenta-
bution of symptoms. Although are no tion indicate that the axillary and radian
specific studies using cognitive restruc- nerves (via roots C4-C6; upper plexus)
turing specifically in sports concussions, are affected based on the muscle
it has been shown to be effective in impairment distribution. Even though
decreasing the development of chronic the boy plays basketball, there was no
post-­concussion syndrome in patients impact near the time of the pain onset.
with mild traumatic brain injury. Median and ulnar nerves that inner-
vate the wrist flexors contribute to the
zz Suggested Reading Lower brachial Plexus roots.
55 Giza C, et al. Summary of evidence-based Although C4-C6 nerves are involved,
guideline update: evaluation and manage- the clinical course of spinal cord
Traumatic Brain Injury
697 27
compression in most cases would be zz Suggested Reading
more gradual. 55 Jeffery AR, Ellis FJ, Repka MX, Buncic
C8-T1 contribute to the lower bra- JR. Pediatric Horner syndrome. J AAPOS.
chia! Plexus branches only and there- 1998;2(3):159–67.
fore would not be consistent with the
symptoms in this case. ?? 16. A 9 year-old boy was evaluated by
you after he received multiple organ
zz Suggested Reading trauma in an automobile accident.
55 Tharin BD, Jonathan AK, York GE, Ritter Brain CAT scan shows sign of cerebral
JL. Brachia! Plexopathy: a Review of edema. Caloric testing was performed
Traumatic and Non-­traumatic Causes. Am by instilling 5 cc’s of cool water into
J Neonatology. 2014;202(1):W67–75. the right external auditory canal. 10 s
later both eyes deviated to the right
side, but was followed by a fast com-
?? 15. A child that develops right-sided ipsi- ponent back to the left.  
lateral ptosis, miosis, and facial anhi- What is your interpretation of this
drosis after a fall from a ladder.   test result?
Most likely has injury to which area? A. This is an indication of cerebral
A. Right maxillary bone trauma
B. Right temporal cortex B. There is involvement of the brain-
C. Left foramen ovale stem injury
D. Right brachial plexus C. There is a pending brain hernia-
tion
vv Correct Answer is: D D. This is a normal finding
Falling from a high places with an out-
stretched arm to attempt to break the vv Correct Answer is: D
fall, can cause neuropraxis (stretching) Irrigation of the external auditory canal
of the brachial plexus. Horner syndrome with cold-water produces a slow compo-
results from the sympathetic denerva- nent movement toward the ipsilateral
tion associated with the trauma to the side of the stimulated ear, and then a
brachial plexus and nerve roots. At fast nystagmus (corrective movement)
birth, brachial plexus injury is second- in the opposite direction. This requires
ary to neuroblastoma tumor invasion an intact vestibular nerve, brainstem
can also cause Horner syndrome. vestibular nuclei, medial longitudinal
Right maxillary bone fracture may fasciculus abducens nuclei, trochlear
damage the maxillary branch of the fifth nerve and oculomotor nuclei. Severe
cranial nerve causing sensory deficits in cerebral trauma without significantly
the area. increased intracranial pressure typically
Right temporal bone impact or frac- does not result in brainstem nuclei dys-
ture can be result in concussion or intra- function. Brainstem injury in the
cranial hemorrhage. The patient has no medulla and/or pontine regions will
altered mental status or contralateral result in abnormal ocular response. The
sensory motor symptoms. test results are normal in this clinical
The mandibular branch of the fifth presentation. Brain herniation causing
cranial nerve passed through the fora- pressure on the brainstem and lead to
men ovale and compression would dysfunction of the nuclei and/or exiting
result in sensory disturbances in that cranial nerves resulting in reduction or
region. absent function of vestibular, abducens,
698 Chapter 27 · Traumatic Brain Injury

trochlear and/or oculomotor nuclei. If middle ear infections can result in ossifi-
we have no response to caloric testing, cation of the cartilaginous connections
this would suggest severe brainstem between the bones, limits the ossicles
damage/injury and may contribute to a movement and results in hearing loss.
diagnosis of brain death.
zz Suggested Reading
zz Suggested Reading 55 Nemholt SS, Schmidt JH, Wedderkopp N,
55 Buttner-Ennever J. A review of otolith Baguley DM. Prevalence of tinnitus and/or
pathways to brainstem and cerebellum. hyperacusis in children and adolescents:
Ann NY Acad Sci. 1999;871:51–64. study protocol for a systematic review.
BMJ open. 2015;5(1):e006649.
?? 17. A 4-year-old girl has been evaluated
for super sensitive hearing. The child ?? 18. A large infant (10.5 pounds) was deliv-
covers her ears or runs to another ered to a mother with a history of
room when the vacuum is started or gestational diabetes. The delivery was
27 the television is turned loud. When prolonged and difficult.  
a sudden loud noise occurs, she The baby sustained an Erb-Duch-
appears to be in physical pain for few enne paralysis, resulting from trau-
seconds. She has a normal develop- matic evulsion of:
ment.   A. C4-CS
Which of the following conditions B. CS-C6
may have predisposed him to this C. C6-C7
problem? D. C7-C8
A. Pervasive Developmental disorder E. C8-T1
B. Bell’s Palsy
C. Ototoxicity form medication vv Correct Answer is: B
D. Frequent middle ear infections Erb-Duchenne paralysis is due to trau-
matic avulsion of C5-C6 nerve roots,
vv Correct Answer is: B typically occurred at birth due to trac-
Chorda tympani branch of the seventh tion on the head during delivery of the
cranial nerve can be affected in Bell’s shoulder. The injury results in loss of
palsy. It innervates the stapedius muscle shoulder abduction and elbow flexion,
and can limit the movement of the bony and loss of the biceps and brachioradia-
ossicles when loud noises are present. lis reflexes. The arm is held in an inter-
Inability of the chorda tympani to inner- nally rotated, extended position, with
vate the muscle results in greater move- pronation of the forearm – often
ment of the ossicles and a greater referred to as the “waiter’s tip” position.
amplitude is transmitted to the inner
ear causing hyperacusis. ?? 19. A 14-year-old boy falls while skate-
Children with pervasive develop- boarding and hits the side of his head
mental disorders can have hearing or against the side curb. He did not
sound sensitivities, but it is not a pri- have his helmet on. He had a minor
mary behavioral disturbance. The child scalp laceration that stops bleeding
in this case has normal language devel- within 5 min. Initially, he is alert, but
opment. he suddenly he lost his consciousness
Sensorineural hearing deficits, 20 min later. A bystander sees him
not hyperacusis can be the result of collapse and calls 911. Head CT scan
Ototoxicity form medications. Frequent is shown below.  
Traumatic Brain Injury
699 27
What blood vessel most likely has Trauma also does not damage the
been damaged? Inferior cerebellar artery which may
A. Great vein of Galen cause a subarachnoid hemorrhage.
B. Middle meningeal artery Subdural hemorrhages can be the
C. Inferior cerebellar artery result of the damage to bridging veins,
D. Bridging veins where bleeding occurs beneath the dura.
Bleeding from the veins of the cav-
ernous sinus would be slow and lead to
progression of symptoms over a longer
period of time.

?? 20. A 15-year-boy was involved in a


motor vehicle accident 6 months ago
and suffered sustained head trauma,
multiple craniofacial surgeries were
done. His past medical history is
remarkable for chronic sinusitis. He
now presents with fever and change
in mental status. B rain MRI is shown.  
Which of the following is correct
regarding this condition?

..      Fig. 27.2 Epidurales Hämatom bei Fraktur (Pfeil)


der Schädelkalotte. (Own work. Hellerhoff. Wikipedia)

vv Correct Answer is: B


This is the classic presentation of an epi-
dural hematoma presenting with the
“lucid period” and lens-shaped area on
the CT scan. Hematoma can compress
intracranial structures and impinge on
CN Ill, lead to a fixed and dilated pupil on
the side of the injury; the eye will also be
directed down and out because of unop-
posed CN IV and CN VI innervation. The
most serious complication is uncal herni-
ation, leading to respiratory arrest as the
medullary structures are compressed in
the foramen magnum. The patient sus- ..      Fig. 27.3 MRI (T1 + contrast): showing a small
tained damage to his left temple area, ring-enhancing lesion with mild surrounding edema
which is where the middle meningeal adjacent to the ventricular catheter and ventricular
artery runs along the temporal bone. dilatation. (Jamjoom et al. Cases Journal 2009 2:110.
7 https://doi.org/10.1186/1757-1626-2-110. Brain
Trauma in general does affect the
abscess formation as a CSF shunt complication: a case
great vein of Galen, however it may be report. Aimun AB Jamjoom, Abrar R Waliuddin and
damaged in cases of thrombosis. Abdulhakim B Jamjoom. Wikipedia)
700 Chapter 27 · Traumatic Brain Injury

A. It can be polymicrobial Wolters Kluwer Health Lippincott


B. Highly active antiretroviral ther- Williams & Wilkins. All rights reserved.
apy is the treatment of choice
C. CSF shows atypical lymphocytes ?? 21. A 10-year-old boy falls off and frac-
with monoclonal B cells tures the proximal humerus. After the
D. He should be treated with pyri- removal of the cast, he is noted to
methamine and sulfadiazine have weakness of the arm, and he was
E. It is caused by mycobacteria referred for neurologic consultation.
On examination, he is able to abduct
vv Correct Answer is: A the arm up to around 30°, but not
There is a small ring-enhancing lesion beyond that. Sensory examination
with mild surrounding edema adjacent reveals a small area of hypoesthesia
to the ventricular catheter and ventricu- on the lateral aspect of the upper
lar dilatation on his MRI. The history and arm, but is otherwise normal. Arm
the clinical findings are consistent with adduction, forearm pronation, and
27 brain abscesses. This lesion originate forearm, wrist, and finger extension,
from invasion of brain parenchyma by are of normal strength.  
bacterial organisms, commonly polymi- What is the most likely diagnosis in
crobial, with a combination of strepto- this patient?
cocci, staphylococci, enterobacteria, A. Axillary neuropathy
and anaerobes. The bacterial agents B. Radial neuropathy
spread via hematogenous routes or C. Posterior cord lesion
from a contiguous infected site, such as D. C8 radiculopathy
the sinuses, ears, or teeth. They also can E. Middle trunk lesion
be encountered after open trauma or
neurosurgical interventions. Patients vv Correct Answer is: A
may present with fever, headaches, neck This is the classic presentation of axil-
stiffness, focal neurologic findings, and lary neuropathy based on the patient’s
altered mental status. The diagnosis is history and examination. Axillary neu-
made on the basis of clinical suspicion ropathies occur secondary to a fractures
and the presence of risk factors, along at the surgical neck of the humerus and
with clinical and radiologic findings, with anterior shoulder dislocations. The
with the MRI showing ring enhancing axillary nerve is a continuation of the
lesions with surrounding edema. posterior cord, and carries predomi-
Lumbar puncture may be contraindi- nantly CS and C6 fibers. The axillary
cated in the presence of mass effect; nerve innervates the deltoid muscle,
however, if a CSF sample is obtained, which is the main arm abductor, particu-
there may be increased WBC and pro- larly between 30° and 90° (supraspina-
tein levels, with normal to decreased tus significantly contributes to arm
glucose level. Atypical lymphocytes and abduction in the first 30° of abduction
monoclonal B cells are seen in lymphop- and the trapezius contributes to greater
roliferative disorders and not in brain than 90°). The deltoid has three heads:
abscesses. (1) the anterior head, which is involved
arm flexion (in front of the body),
zz Suggested Reading assisted by serratus anterior, (2) the lat-
55 Comprehensive Review in Clinical eral head, which along with the anterior
Neurology: a Multiple Choice Question head is mainly involved in arm abduc-
Book for the Wards and Boards © 2011 tion to the side and slightly anteriorly,
Traumatic Brain Injury
701 27
and (3) the posterior head, which is the second most cause of abducens
involved in posterior movement of the nerve palsies in children, it occurs in
abducted arm. The axillary nerve also 13% of cases. Primary brainstem glio-
innervates teres minor, which externally mas are a common etiology. Isolated
rotates the arm along with infraspina- abducens weakness may be the initial
tus. The axillary nerve provides sensory sign of illness. Foville syndrome is an
innervation to the upper lateral arm abducens and facial nerve palsies with
through the upper lateral brachial cuta- contralateral hemiplegia, also most
neous nerve. Intact arm abduction in commonly caused by basilar artery
the first 30° suggests the supraspinatus infarcts. The lesion is in the caudal teg-
is of normal strength. In addition, there mental pons, potentially affecting the
is intact sensation over the posterolat- abducens nucleus or para-­pontine retic-
eral arm, territory innervated by the C8 ular formation (PPRF), facial cranial
nerve root through the lateral and pos- nerve or nucleus, and corticospinal
terior cutaneous nerves to the arm tract. A normal motor examination in
(radial nerve branches). These findings this patient makes this syndrome
make a C8 radiculopathy unlikely. Intact unlikely. Millard-­Gubler syndrome con-
strength of radial nerve-innervated sists of a facial nerve palsy with contra-
muscles excludes a posterior cord lesion lateral hemiparesis, classically caused
and a radial neuropathy intact strength by basilar artery infarcts of the caudal
of predominantly C7-­innervated mus- ventral pons involving the cranial nerve
cles excludes a middle trunk lesion. nucleus or fascicle and corticospinal
tract. Later descriptions of the syn-
zz Suggested Reading drome included abducens nerve palsies,
55 Russell SM. Examination of Peripheral as well. The isolated abducens weakness
Nerve Injuries. New York: Thieme; 2006. in this patient makes this syndrome
55 Comprehensive Review in Clinical unlikely. Gradenigo’s syndrome is the
Neurology: A Multiple Choice Question paralysis of the abducens nerve due to
Book for the Wards and Boards ©2011 osteitis of the petrous pyramid, second-
Wolters Kluwer Health Lippincott ary to middle ear infections or mastoid-
Williams & Wilkins. All rights reserved. itis. The classic triad of symptoms
includes diplopia, periorbital unilateral
?? 22. A 9-year-old boy presents with pain due to trigeminal nerve involve-
impaired abduction of the left eye. ment, and persistent otorrhea. This syn-
The rest of his neurologic examina- drome is rare in the current age of
tion was normal.   antibiotic use.
The most likely cause is:
A. Trauma zz Suggested Reading
B. Malignancy 55 Pedroso JL, et al. Gradenigo’s syndrome:
C. Foville syndrome beyond the classical trial of diplopia, facial
D. Milard-Gubler syndrome pain and otorrhea. Case Rep Neurol.
E. Gradenigo’s syndrome 2011;3(1):45–47.
55 Sweeney PJ, Hanson MR. Cranial neuropa-
vv Correct Answer is: A thies. In: Bradley WG, Daroff RB, Fenichel
Trauma is the most frequent cause of GM, Ankovic J, editors. Neurology in
childhood-onset abducens palsies, Clinical Practice. 4rth ed. Elsevier, Inc.;
occurring in 40% of cases. Malignancy is 2004. p. 2107–23.
702 Chapter 27 · Traumatic Brain Injury

?? 23. Late complications from head injury the axilla, sub-conjunctival area, and
include all of the following except: palate. Multiple petechial hemorrhages
A. Hormone disturbances can be seen in the gray and white mat-
B. Seizures ter of the brain on autopsy. This clinical
C. Postconcussion syndrome presentation of petechial hemorrhages
D. Posttraumatic hydrocephalus after multiple fractures is not consistent
E. Cluster headaches with the other options.

vv Correct Answer is: E zz Suggested Reading


Cluster headaches are not typically a 55 Posner JB, Saper CB, Schiff ND, Plum
complication of head injury. F. Plum and Postern’s Diagnosis of Stupor
and Coma, 4rth ed. New York, NY: Oxford
?? 24. A 17-year-old girl is involved in a University Press; 2007.
motorcycle accident while riding with 55 Comprehensive Review in Clinical
her boyfriend. She did not have her Neurology: A Multiple-Choice Question
27 helmet on. Her boyfriend was wearing Book for the Wards and Boards ©2011
a helmet. However, he suffered lung Wolters Kluwer Health Lippincott
contusions, left sided pneumothorax, Williams & Wilkins. All rights reserved.
multiple fractures, and splenic lacera-
tion. The boyfriend remained coma- ?? 25. A 16-year-old girl is complaining of
tose, and his examination a few days chronic headaches, problems falling
after admission demonstrated multi- asleep, scary dreams, and poor school
ple petechial hemorrhages all over his performance for the last few months.
skin, particularly in the axillary region. She lost interest in school activities, in
The patient died a few days later. On socializing with friends, and has been
autopsy, the brain showed multiple increasingly irritable, moody and
diffuse petechial hemorrhages.   angry. She told you that her father
How do you explain the neuro- had been killed in a car accident
pathologic findings? 6 months ago, also she broke up with
A. Fat embolization her boyfriend after numerous fights
B. Diffuse axonal injury 3 months ago. She has been afraid to
C. Meningococcal meningitis drive in a car and has had recurring
D. Coup and countercoup injury nightmares about her father’s death.
E. SAH Her headaches are not migraine in
nature, and the rest of her neurologi-
vv Correct Answer is: A cal examination is normal.  
Fat embolism, is most frequently What is her diagnosis?
occurred after fractures of long bones A. Depression
such as the femur which results from fat B. Post-traumatic stress disorder
droplets entering the circulation usually C. Bipolar disorder
in the setting of surgery or trauma. Fat D. Insomnia
micro-particles from the bone marrow E. Learning disability
travel in the venous system to the lungs
and spread systemically. The typical vv Correct Answer is: B
clinical presentation is agitation, delir- Stressful events that lead to both emo-
ium, coma, respiratory distress, anemia, tional and physical consequences can
thrombocytopenia, and generalized be experienced by all children and ado-
petechial rash, often concentrated in lescents. Usually the reactions to
Traumatic Brain Injury
703 27
s­ tressful events are generally brief and lary reactivity, CT findings, and presence
associated with full recovery, the expo- of major extracranial injury. Bilateral
sure to catastrophic or repeated trau- dilated unreactive pupils predict poor
matic events can lead to unconscious outcome and, when present, is the
mental dissociation, an emotional strongest predictor.
numbing to deaden or block the pain
and trauma. It is important to realize zz Suggested Reading
that since younger children have limited 55 Mangat HS. Severe traumatic brain injury.
verbal skills and non-specific ways of Continuum (Minneap Minn),
reacting to stress, they may not always 2012;18:532–46.
fully meet the DSM-IV-­TR criteria
(designed for adults) for post-­traumatic ?? 27. You are seeing a football player who
stress disorder. Behavioral lability is has an episode of convulsion with loss
more common in children than adults, of consciousness for 60 s after collid-
and fewer criteria may be necessary to ing with another player. Imaging is
establish a diagnosis. negative. He was admitted for obser-
vation. The following morning, he is
zz Suggested Reading back to his baseline, but complaining
55 American Academy of Child and of mild headache, but with an entirely
Adolescent Psychiatry Work Group on intact mental status.  
Quality Issues. Practice parameters for the What is the appropriate statement
assessment and treatment of children and regarding management of his con-
adolescents with posttraumatic stress vulsion?
disorder. J Am Acad Child Adolesc A. He should be started on an anti-
Psychiatry. 1998;37(10 Suppl):4S–26S. epileptic drug and continued on it
55 American Psychiatric Association. for 30 days.
Diagnostic and statistical manual of B. He does not need medication for
mental disorders DSM-IV-TR. 4rth ed. seizure prophylaxis but should be
Washington, D.C.: American Psychiatric monitored under concussion pro-
Association; 2000. tocol.
C. He would benefit from antiepi-
?? 26. A 16-year-old boy presents with a leptic drug prophylaxis for 7 days
closed traumatic head injury.   even though his imaging is normal.
Which of the following is the stron- D. He should undergo continuous
gest predictor of poor outcome? EEG monitoring to detect any
A. Presence of convexity subarach- underlying epileptiform discharges
noid hemorrhage on head CT E. Seizure prophylaxis for 7 days
B. Bilateral dilated unreactive pupils post-injury will substantially
C. Glasgow coma scale of 10 after reduce the risk of him developing
full resuscitation late-­onset posttraumatic epilepsy
D. Midline shift on head CT
E. Unilateral compression of ambient vv Correct Answer is: B
cistern on head CT Patient should be in the concussion pro-
tocol as recommended by the American
vv Correct Answer is: B Academy of Neurology guidelines for
The major predictors of outcome after evaluation and management of concus-
severe traumatic brain injury are GCS sion in sports because he suffered con-
score after full resuscitation, age, pupil- cussive convulsion. The risk of early
704 Chapter 27 · Traumatic Brain Injury

posttraumatic seizures ranges between traumatic epilepsy (usually defined as


10% and 20%. Patient’s imaging does seizures more than 1 week removed
not suggest any penetrative injury or from the traumatic insult.)
intracranial contusions, which would
otherwise place him at high risk for zz Suggested Reading
recurrent early post-traumatic seizures 55 Mangat HS. Continuum (Minneap Minn).
and therefore benefit from short-term 2012;18(3):542–6.
seizure prophylaxis for 7 days. 55 Giza CC, Kutcher JS, Ashwal S MH, et al.
Continuous EEG would not be necessary Summary of evidence-based guideline
either given no obvious injuries that put update: evaluation and management of
him at risk for seizures. There is no evi- concussion in sports: report of the
dence to suggest that early institution Guideline Development Subcommittee of
of anti-epileptic medication helps pre- the American Academy of Neurology.
vent the occurrence of late-onset post- Neurol. 2013;80(24):2250–57.

27
705 28

Vascular

© Springer International Publishing AG, part of Springer Nature 2018


Y. M. Awaad, Absolute Pediatric Neurology, https://doi.org/10.1007/978-3-319-78801-2_28
706 Chapter 28 · Vascular

?? 1. You are evaluating a 3-year-old boy vv Correct Answer is: D


in the ED for his lethargy. His family There is an absence of signal in the infe-
reported that he has been irritable, rior sagittal sinus on the CT Venogram.
sleepy most of the day and he cannot The incidence of cerebral venous sinus
get up and walk one day before his thrombosis (CVST) is approximately 67
admission to the ER. His past medical per 100,000 children per year. The dif-
history is unremarkable and his devel- ferential diagnosis of cerebral venous
opment is on target. On examination, sinus thrombosis includes malignancy,
he is lethargic, and difficult to arouse. nephrotic syndrome, congenital heart
He has drooling, intact gag, some disease, rheumatologic disorders such as
inward deviation of his right eye, and no Lupus, infection, hypercoagulable disor-
papilledema. He moves all four extremi- ders, medications and genetic disorders,
ties equally, but overall has a paucity head injury, and recent surgery. Clinical
of movement. DTRs are decreased, but manifestations include lethargy, irritabil-
toes are up-going bilaterally. An urgent ity, seizures, and symptoms of increased
CT head is ordered, followed by a CT intracranial pressure, mutism, vomiting,
Venogram. headache, coma and death.
In CNS vasculitis, MRI findings
28 typically include multifocal cortical and
­subcortical white and grey matter lesions.
The Globus pallidus is preferentially
involved in carbon monoxide poisoning,
not the bilateral thalami. Mitochondrial
disorders typically involve the basal
ganglia on MRI as well.

zz Suggested Reading
55 Rodríguez-Fanjul J, et al. Deep vein
thrombosis: rare cases of diagnoses in a
pediatric emergency department. Pediatr
Emerg Care. 2011;27(5):417–9.
55 Kanakkande A, et al. Deep vein thrombosis
in children. Pediatr Rep. 2013;5(2):48–9.
55 Se ́bire B, et al. Cerebral venous sinus
thrombosis in children: risk factors,
presentation, diagnosis and outcome.
..      Fig. 28.1 CT Venogram of the brain showing a Brain. 2005;128:477–89.
filling defect in the Sagital Sinus. Most likely a
55 deVeber G, et al. Canadian pediatric
Thrombus. self-made. (Heather. Wikipedia)
ischemic stroke study group. Cerebral
sinovenous thrombosis in children. N Engl
What is the most likely diagnosis? J Med. 2001;345(6):417–23.
A. CNS vasculitis
B. Carbon monoxide poisoning ?? 2. You were asked to see a 7-month-old
C. Mitochondrial disorder baby for her developmental delay and
D. Inferior sagittal sinus thrombosis seizure-like spells. She is a full- term baby,
E. None of the above and was born without complications.
Vascular
707 28
She never has been able to roll over or vv Correct Answer is: E
sit, does not consistently visually regard Hydranencephaly is characterized by
or track objects, and does not reach for severe dysgenesis of the cerebral hemi-
objects with her hands, but does put her spheres, with relative preservation of the
hands in her mouth. Head circumfer- thalamus, cerebellum and brainstem.
ence is at 5th percentile, but there are The affected areas are usually localized
no dysmorphic features. Her pupils are within the territories of the middle cere-
reactive to light but she does not track. bral arteries. Vasculopathy with carotid
She has symmetric facial movements artery occlusion has been suggested
upon crying. She has decreased muscle as the reason of hydranencephaly. This
tone in her neck and shoulder girdle, but has been supported by the distribution
increased in the hands and ankles. Ten- of the marked hemispheric destruction
don reflexes are brisk, with bilateral ankle observed in those patients. Experiments
clonus. Her side picture is shown below. entailing ligation of both the common
carotid arteries and the jugular veins in
monkeys during early gestation resulted
in subsequent cerebral dysgenesis and
focal destruction, with the regions sup-
plied by the anterior and middle cerebral
arteries being reduced to thin transparent
membranes. The tissues supplied by the
posterior cerebral arteries, which receive
collateral blood flow from the vertebro-
basilar system through the posterior com-
municating arteries, were usually mainly
preserved. These findings support the
proposed pathologic process in hydran-
encephaly.

zz Suggested Reading
55 Tsai JD, Kuo HT, Chou IC.
Hydranencephaly in Neonates. Pediatr
Neonatol. 2008;49(4):154−7.
..      Fig. 28.2 Head illumination showing Hydraneceph-
aly. 7 http://neuropathology-web.org/chapter3/
?? 3. You are seeing an 8-year-old female in
images3/3-8l.jpg. Dimitri Agamanolis, M.D. Department the ED who has complex partial sta-
of Pathology, Akron Children’s Hospital, One Perkins tus epilepticus, fluctuating confusion
Square, Akron, Ohio, 44,308, USA. Wikipedia. and intermittent clonic movements of
her right arm and leg. She has normal
What is the most likely mechanism to birth history and early development.
account for her condition? At age 5 years, she started having gen-
A. Congenital infection eralized tonic-clonic seizures, easy fati-
B. Inborn error of metabolism gability, generalized muscle weakness,
C. Migrational brain anomaly and intermittent headaches. She has
D. Perinatal hemorrhage mild, bilateral sensorineural hearing
E. Prenatal ischemia loss, and she failed a school hearing
708 Chapter 28 · Vascular

screen. Her family history is remark- Those patients have normal MR angiog-
able of her mother has migraine raphy, but MR spectroscopy may show a
headaches and hypertrophic cardio- decrease in the N-acetyl aspartate signal
myopathy of unknown etiology. On and a high lactate peak. Muscle biopsy
examination, her height and weight reveals scattered, vacuolated muscle
are both <3rd percentile. She has a left fibers with a clear surrounding rim on
homonymous hemi-anopsia, proximal hematoxylin-eosin staining; basophilic
muscle weakness, and hypoactive inclusions on Gomori trichrome staining;
reflexes. Her EEG shows intermittent and scattered ragged-red fibers on succi-
theta slowing in the right posterior nate dehydrogenase staining. Molecular
head region. Brain MR shows lacunar testing can be performed on blood leu-
infarcts of both putamen, as well as an kocytes, skin fibroblasts, urine sediment
infarction of the right occipital cortex. cells, and buccal mucosa, with the latter
Her serum creatinine kinase is 540 IU/L 2 sources carrying the highest mutation
(elevated), and her serum lactate is 4.2 loads. Treatment consists of symptomatic
(elevated). management and supplementation with
What other complications are a “mitochondrial cocktail,” consisting of
expected in this patient? some combination of creatine, coenzyme
28 A. Arthritis Q10, alpha-lipoic acid, riboflavin, thia-
B. Asthma mine, vitamin C, vitamin E, and biotin.
C. Diabetes mellitus There are no curative treatments for
D. Heliotropic rash MELAS.
E. Precocious puberty
zz Suggested Reading
vv Correct Answer is: C 55 Sproule DM, Kaufmann P. Mitochondrial
This is a clinical presentation of MELAS, encephalopathy, lactic acidosis, and stroke-
which is a mitochondrial disorder like episodes: basic concepts, clinical
characterized by the triad of encepha- phenotype, and therapeutic management
lopathy (seizures, dementia, or both), of MELAS syndrome. Ann NY Acad Sci.
lactic acidosis, and stroke-like episodes. 2008;1142:133–58.
Short stature, cognitive impairment,
sensorineural hearing loss, migraines, ?? 4. You are seeing a 10-year-old boy in
depression, cardiomyopathy, cardiac the PICU who has sudden headache,
conduction defects, and diabetes mel- followed by rapid depressed level of
litus are secondary manifestations. Age consciousness and right-side body
of onset ranges from 2 to 60 years, but weakness. He has no recent history
almost 70% of patients present between of fever, trauma, or systemic illness.
the ages of 2–20 years. The disorder is He is not taking any medications. On
typically caused by an A-to-G transition examination, you were able to arouse
(m.3243 A > G) in the gene that encodes him with deep pain, but he had a gaze
mitochondrial tRNA (Leu); hence, it is preference to his right and lack of
inherited via the mothers. Brain MRI usu- motor response to the right arm or leg.
ally demonstrates asymmetric lesions Urgent brain CT showed a large intra-
of the occipital and parietal lobes that parenchymal bleeding within the left
mimic ischemia, except that they are parieto-­occipital region with evidence
usually restricted to the cortex without of herniation of brain tissue from left
respect for particular vascular territories. to right under the falx. A neurosurgeon
Vascular
709 28
was called and an emergency crani- ­ orbidity and mortality with hemorrhage
m
otomy was performed to evacuate from AVMs in children is high, and a pro-
the hemorrhage, followed by an angi- active approach to treatment is usually
ography showed an arteriovenous recommended when they are discovered.
malformation (AVM) in the left parietal Surgical treatment should be individual-
midline region fed predominately by ized to the circumstances of each case,
the left posterior cerebral artery. but can include craniotomy with surgi-
What symptom would be present in cal excision, endovascular embolization,
this child before the AVM rupture? or stereotactic radiosurgery (or some
A. Episodes of left sided weakness ­combination of these).
B. Learning disability
C. Migraine with aura zz Suggested Reading
D. No symptoms prior to the hemor- 55 Niazi TN, Klimo P Jr., Anderson RC,
rhage Raffel C. Diagnosis and management of
E. Simple partial sensory seizures arteriovenous malformations in children.
Neurosurg Clin N Am. 2010;21:443–56.
vv Correct Answer is: D
AVM is the most common cause of non-­ ?? 5. You are evaluating a 1-month-old boy
traumatic intra-parenchymal brain hem- for his remarkable developmental delay
orrhage in children after the newborn and rapidly enlarging head size. He has
period. There are no symptoms prior to unremarkable prenatal and perinatal
the hemorrhage itself in 75–80% of chil- histories. He has a full and enlarged
dren and adolescents. There is a higher fontanelle. Head circumference is 45 cm.
likelihood of hemorrhage into the pos- He does not fixate or follow. He has
terior fossa or deep cerebral structures, increased tone, hyperactive reflexes and
and the bleeding itself is often more no purposeful motor activity. Head MRI
massive from AVM bleeding in adults. The scan is shown.

a b c

..      Fig. 28.3 (a–c) A 31-day-old infant born preterm cysts. b Axial fluid-attenuated inversion recovery
(31 weeks) shows mild-to-moderate hypoxic ischemic (FLAIR) and T2WI at the level of lateral ventricles show
injury. The infant also had refractory hypoglycemia. frontal and posterior periventricular white matter cysts
a Axial T1WI at the level of lateral ventricles shows c with gliosis (arrows). (Indian Journal of Radiology
frontal and posterior periventricular white matter and Imaging. Year: 2016;26(3):316–27)
710 Chapter 28 · Vascular

What is your most likely explanation This condition puts this baby at high-
for his condition? est risk for which of the following neu-
A. Migration brain disturbance rological conditions?
B. X-linked aqueduct stenosis A. Deafness
C. Prenatal vascular brain injury B. Cerebral palsy
D. Brain injury from herpes C. Cortical visual impairment
­encephalitis D. Epilepsy
E. Congenital virus infection E. Autism

vv Correct Answer is: C vv Correct Answer is: B


Patients with this clinical picture are The ultra sound shows grade 3 IVH (ger-
always a challenge to the child neurolo- minal matrix hemorrhage with IVH filling
gist. Therapeutic futility rules. Recognizing and distending lateral ventricles). This
the underlying vascular insult leading to hemorrhage is secondary to immaturity
the severe cortical insult may not ­obviate of the germinal matrix vessels which are
a shunt for a rapidly expanding head but thought to rupture due to high central
it puts the objectives and limitation of any venous pressures transmitted to the
intervention in proper perspective. region during or following birth during
28 vaginal delivery, need for high pressure
zz Suggested Reading ventilation or other interventions. Both
55 Jung JH, Graham JM Jr., Schultz N, grade 3 and 4 IVH have approximately
Smith DW. Congenital hydranencephaly/ 50% chance of producing post-hem-
porencephaly due to vascular disruption orrhagic hydrocephalus that requires
in monozygotic twins. Pediatrics. treatment in the neonatal period. In addi-
1984;73(4):467–9. tion, grade 3 IVH is associated with 50%
chance of cerebral palsy and develop-
?? 6. A 29- week gestation infant has been mental delay.
doing well since birth. She is on CPAP
with no need for inotropic medications zz Suggested Reading
or antibiotics, but it was very. A routine 55 Allan WC, Vohr BR, Makuch RW, Katz KH,
ultrasound performed on her second and Ment LR. Antecedents of cerebral
day of life is shown. palsy in a multicenter trial of indometha-
cin for intraventricular hemorrhage. Arch
Pediatr Adolesc Med. 1997;151:580–5.

?? 7. You are seeing a 14-year-old boy in the


ER for his sudden onset of headache
and left face and extremities weakness.
When he woke up in the morning, he
noticed the symptoms which improved
after 1 h, but worsened again 4 h later.
He has unremarkable past medical his-
tory and takes no medications. He is
afebrile, pulse 85/min, BP 95/65, respi-
rations 18/min. He has mild weakness
of the lower left face, and more dense
..      Fig. 28.4 Ultra Sound showing grade 3 IVH. With
permission from med-ed.virginia.edu. Free for paralysis of his left upper and lower
residents and students extremities. He lost sensation in his left
Vascular
711 28
face and limbs. DTRs are brisk on the left An MRA demonstrates narrowing and
side, with extensor plantar response. A beading of the right middle and ante-
bolus of IV fluids was given and his BP rior cerebral arteries. He is started on
rises to 130/80. A brain MRI is shown. low-dose aspirin.

FLAIR DWI ADC

..      Fig. 28.5 MRI with FLAIR, DWI, and ADC showing an established infarction in the right frontal lobe matching
the CTP-delineated ischemic core. (Permission by Hindawi. Free access)

What is your next step in manage- Headache and acute focal motor or sensory
ment? deficits are typical presentations. Brain and
A. Hypothermia leptomeningeal biopsy are not typically
B. Prednisone needed if the classic findings of inflamma-
C. Warfarin tory vasculitis have been demonstrated by
D. Verapamil the angiogram. Patients with neurocogni-
E. Phenylephrine tive dysfunction at presentation, multifo-
cal T2 hyper-intense lesions on MRI, and
vv Correct Answer is: B evidence of distal stenosis on angiography
This is a case of acute ischemic stroke in are at higher risk of progressive disease
the right anterior and middle cerebral and should be treated aggressively (induc-
arteries territory. The findings of stenosis tion with monthly IV cyclophosphamide
and beading are typical of inflammatory and high-­dose steroids for 6 months fol-
vasculitis. The vasculitis can be a primary lowed by oral mycophenalate mofetil or
CNS disease or secondary to a variety of oral azathioprine for 18 months. Patients
inflammatory (lupus, Behcet’s, Sjogren, who do not have the high-risk profile are
dermatomyositis, scleroderma, polyar- often treated with an antiplatelet agent
teritis nodosa, Kawasaki disease, HSP, combined with high-dose steroids for
Wegener’s, Churg-Strauss, and inflamma- 3 months.
tory bowel disease) or infectious (bacte-
rial, viral, fungal, spiorochete) conditions. zz Suggested Reading
Primary CNS vasculitis is divided into large- 55 Elbers J, Benseler SM. Central nervous
medium vessel and small vessel types. The system vasculitis in children. Curr Opin
boy has the ­large-medium vessel variety. Rheumatol. 2008;20(1); 47–54.
712 Chapter 28 · Vascular

?? 8. You are seeing a 12-year-old trisomy 21 by a baseball at school. She had a history
girl in the ED for her transient weakness of infantile spasms when she was 1 year
episode of her right hand and face along old, which resolved after treatment. She
with difficulty speaking, all of which has had migraine headaches for several
resolved spontaneously after an hour. years. She has the typical features of tri-
The event occurred when she was upset somy 21, but no focal deficits. Her brain
and crying after being struck in the head MRI and MRA are shown.

..      Fig. 28.6 T1-weighted MR image of moyamoya


disease. Flow void in the basal ganglia is indicated
by the arrow. Own work. (Takuma-sa. Wikipedia)

28

..      Fig. 28.7 (Left) MIP reconstructed MR angiography by commons:User:MBq, (Right image) 7 https://
of a 11 year old girl with en:Moya moya disease. (Right) commons.wikimedia.org/wiki/File:Mra-mip.jpg.
healthy patient, for comparison. (Left image) own work commons:User:MBq, commons:User:SBarnes. Wikipedia
Vascular
713 28
What is the most likely diagnosis? ?? 9. You are seeing a 7-year-old black boy,
A. Atlantoaxial dislocation with sickle cell disease in the ED after
B. Partial seizure developing right-sided weakness and
C. Moya Moya syndrome numbness. MRI shows a small stroke
D. Breath-holding spell in the left fronto-parietal region. MR
E. Traumatic arterial dissection angiogram shows narrowing of the left
middle cerebral artery, and peak systolic
vv Correct Answer is: C velocities of the left middle cerebral
Moya Moya syndrome is a cerebrovas- artery on transcranial Doppler are
cular condition which lead to stroke in 230 cm/s.
affected patients secondary to progres- What is the most appropriate step for
sive stenosis of the intracranial internal secondary stroke prevention?
carotid arteries and their proximal ves- A. anticoagulation with warfarin,
sels. Development of compensatory col- goal international normalized
lateral small blood vessels near the apex ratio (INR) of 2–3
of the carotid arteries occur secondary B. dual antiplatelet therapy with
to the stenosis. Patients at increased risk aspirin and clopidogrel
include, people with history of radio- C. intracranial angioplasty and
therapy to the head or neck, trisomy 21, ­stenting
neurofibromatosis type 1, and sickle cell D. permissive hypertension with
disease. The most presenting manifes- goal systolic BP greater than
tations are, migraine-like headaches, 160 mmHg
ischemic symptoms such as hemiparesis, E. transfusion with goal HbS fraction
aphasia, and cognitive impairment. Less of less than 30%
frequent presenting symptoms include
visual deficits, syncope, seizures, and vv Correct Answer is: E
cerebral hemorrhage. Ischemic symptoms Sickle cell disease patients are at high risk
are often precipitated by hyperventilation for stroke due to narrowing of the distal
from crying, dehydration and exertion. internal carotid artery and major intracra-
Choreiform movements can develop sec- nial vessels. This is a non-atherosclerotic
ondary to the dilated collateral vessels in vasculopathy, possibly related to trauma
the basal ganglia. The vascular stenosis from sickled red blood cells.
is due to a combination of hyperplasia of The treatment of choice for stroke
smooth muscle cells and luminal throm- prevention in sickle cell disease patients
bosis. If no treatment has been provided, is transfusion to maintain an HbS frac-
most patients will progress over time tion below 30%. Transfusions are typi-
and are therefore at high risk of stroke. cally begun when MCA velocities on
Medical treatment options such as anti- transcranial Doppler exceed 200 cm/s.
platelet agents, anticoagulation, and The use of antiplatelet, or antico-
calcium channel blockers seem to be less agulant therapy, in sickle cell disease is
effective than surgical revascularization complex, since patients often develop a
procedures, either direct or indirect (e.g. Moya Moya like pattern, where ischemic
encephalo-duroarterio-­synangiosis). strokes are more common in younger
patients and hemorrhagic stroke risk
zz Suggested Reading increases with age and disease severity.
55 Scott RM, Smith ER. Moya Moya disease The roles of intracranial angioplasty
and Moya Moya syndrome. NEJM. and stenting, as well as blood pressure
2009;360(12);1226–37. goals, are unclear in this setting.
714 Chapter 28 · Vascular

zz Suggested Reading ?? 10. An 18-year-old African-­American boy


55 Roach ES, Golomb MR, Adams R, et al. is seen in the ED for acute onset of
Management of stroke in infants and right-sided weakness and numbness.
children. Stroke. 2008;39(9):2644–91. There is mild expressive aphasia and
55 Adams RJ. Big strokes in small persons. right hemiparesis on his examination.
Arch Neurol. 2007;64:1567–74. MR Angiogram is shown.

28

..      Fig. 28.8 (Left) MIP reconstructed MR angiography by commons:User:MBq, (Right image) 7 https://
of a 11 year old girl with en:Moya moya disease. (Right) commons.wikimedia.org/wiki/File:Mra-mip.jpg.
healthy patient, for comparison. (Left image) own work commons:User:MBq, commons:User:SBarnes. Wikipedia

What is most likely diagnoses? sickle cell disease. The visualized portions
A. bilateral carotid artery dissection of the carotid arteries appear normal, so
B. fibromuscular dysplasia fibromuscular dysplasia or bilateral carotid
C. moya Moya syndrome artery dissections would be less likely. Both
D. paradoxical embolism to the left distal carotid arteries are abnormal, so an
middle cerebral artery embolic process to the left middle cerebral
E. reversible cerebral vasoconstric- artery would not explain the imaging find-
tion syndrome ings. Reversible cerebral vasoconstriction
­syndrome is an idiopathic condition affect-
vv Correct Answer is: C ing more distal cerebral arteries.
The abrupt abnormality in the distal inter-
nal carotid arteries is highly suggestive of zz Suggested Reading
Moya Moya syndrome, a progressive non-­ 55 Roach ES, Golomb MR, Adams R, et al.
atherosclerotic vasculopathy. Although idio- Management of stroke in infants and
pathic Moya Moya disease is seen almost children. Stroke. 2008;39(9):2644–91.
exclusively in Asian populations, secondary 55 Scott RM, Smith ER. Moyamoya disease
forms can be seen in several systemic con- and moyamoya syndrome. N Engl J Med.
ditions, including African-Americans with 2009;360:1226–37.
Vascular
715 28
?? 11. What is the estimated annual risk twitching and tingling of his right
of hemorrhage in a 13-year-old boy hand, followed by 12 h of weakness
who is admitted to the hospital for and sensory loss in the hand? His MRI
evaluation of a seizure which involved and MRA are shown.

..      Fig. 28.10 Arteriovenöse Malformation im Gehirn:


MRT TOF gedrehte MIP-Projektion. Own work.
(Hellerhoff. Wikipedia)

..      Fig. 28.9 Arteriovenöse Malformation des Gehirns


in der MRT T1 KM axial. Own work. (Hellerhoff.
Wikipedia)

A. 2–4% 55 Al-Shahi R, Warlow C. A systematic


B. 5–10% review of the frequency and prognosis of
C. 15–20% arteriovenous malformations of the brain
D. 30% in adults. Brain. 2001;124:1900–26.
E. 50%
?? 12. An 18-year-old female has an acute
vv Correct Answer is: A onset of weakness of the right arm and
There is a lack of high quality data how- expressive aphasia. Her MRI shows acute
ever, a systematic review of the epide- ischemia in a segment of the left MCA.
miology and natural history of AVM’s Of the following infectious agents,
showed an annual risk of first hemor- which is most likely to be responsible
rhage of about 2%. for her condition?
A. cytomegalovirus
zz Suggested Reading B. echovirus
55 Wilkins RH. Natural history of intracra- C. mycoplasma
nial vascular malformations: a review. D. parvovirus
Neurosurgery. 1985;16(3):421–30. E. varicella-zoster virus
716 Chapter 28 · Vascular

vv Correct Answer is: E B. cerebral contusion in the left fron-


Varicella-zoster virus (VZV) has been asso- tal region
ciated with stroke at almost any age and C. leptomeningeal vascular malfor-
not all patients have a preceding rash. In mation in the left posterior region
one study of 30 patients, rash occurred in D. multiple cerebral cavernous mal-
19 (63%); CSF pleocytosis in 20 (67%); and formations
imaging abnormalities in 29 (97%). Angi- E. subdural hematoma in the left
ography in 23 patients revealed abnor- frontotemporal region
malities in 16 (70%). Large and small
arteries were involved in 15 (50%), small vv Correct Answer is: C
arteries in 11 (37%), and large arteries in The boy has Sturge-Weber syndrome
only 4 (13%) of 30 patients. Average time (SWS), which is a congenital capillary-­
from rash to neurologic symptoms and venous vascular malformation involving
signs was 4.1 months and from neuro- the brain (leptomeningeal angioma), skin
logic symptoms and signs to CSF virology (capillary malformation) and eye. Seizure
analysis was 4.2 months. in infancy is a very common presentation
CSF of 9 (30%) patients contained in patients with brain involvement. Neuro-
VZV DNA while 28 (93%) had anti-VZV logic deficits can be acquired slowly over
28 IgG antibody in CSF; in each of these time, or as the result of stroke-like episodes
patients reduced serum/CSF ratio of VZV associated with seizures and/or migraines.
IgG confirmed intrathecal synthesis. VZV Toddlers and young children are
vasculopathy is caused by a productive particularly prone to stroke-like epi-
viral infection in arteries, as evidenced by sodes triggered by falls with minor head
the presence of multinucleated giant cells, injury. SWS brain involvement should be
Cowdry A inclusion bodies and herpes suspected in any newborn with a port-
virus particles. Postmortem virology analy- wine birthmark in the V1 distribution
sis reveals VZV antigen in cerebral vessels. (forehead to 1 side and/or upper eyelid).
A PWB of any size in that location has
zz Suggested Reading approximately a 10% to 20% risk of
55 Nagel MA, Cohrs RJ, Mahalingam R, et al. SWS brain involvement. However, the
The varicella zoster virus vasculopathies: risk increases with the size, extent, and
clinical, CSF, imaging, and virologic bilaterally of the birthmark. Approxi-
features. Neurology. 2008;70(11):853–60. mately 10% of those with typical SWS
brain involvement will have no birth-
?? 13. You are seeing a 14-month-old boy mark or eye involvement.
in the ED who has a left-sided focal SWS brain involvement is diag-
motor seizure 2 h ago. He has been nosed by visualization of the typical
sleepy and has not moved his left leptomeningeal vascular malformation
limbs since then. He fell 1 foot off of a on contrast enhanced T1-weighted
couch onto a carpeted floor an hour MRI imaging. Associated dilation and
prior to the seizure, but he did not lost enhancement of the choroid plexus
his consciousness. On examination, he (glomus) on the involved side is also
has a port-wine birthmark involving often present in older children and
the right upper eyelid and forehead. adults; dilated deep draining venous
What are you going to see on his MRI vessels underlying the effected cortical
scan? region may also be seen. Cortical and
A. arteriovenous malformation in the subcortical calcification which develop
left frontal region over time are best seen on head CT.
Vascular
717 28
zz Suggested Reading What is the most likely etiology of the
55 Comi AM. Presentation, diagnosis, imaging findings shown?
pathophysiology, and treatment of the A. arteriovenous malformation
­neurological features of Sturge-­Weber B. basilar artery ­thrombosis
syndrome. Neurologist. 2011;17(4): C. posterior reversible encephalopa-
179–84. thy syndrome
D. reversible cerebral vasoconstric-
?? 14. You are seeing a 7-day postpartum tion syndrome
18-Year-old girl, after having a full- E. venous sinus thrombosis
term baby. She complains of severe
headache and blurred vision. On vv Correct Answer is: C
arrival, her BP is 160/95; heart rate 90; Posterior reversible encephalopathy syn-
temperature 37.5 °C. She has a gener- drome (PRES) is very common in a young
alized tonic-clonic seizure while she in women with pre-eclampsia or eclampsia.
the ER. Brain MRI is shown. This is secondary to impaired cerebral
autoregulation and increased cerebral
blood flow volume in the pregnant state.
The symmetric hyper-intensities seen
in FLAIR in the posterior regions of the
cerebrum are the most common and
characteristic imaging finding in this
syndrome.

zz Suggested Reading
55 Oehm E, Hetzel A, Els T, et al. Cerebral
hemodynamics and autoregulation in
reversible posterior leukoencephalopathy
syndrome caused by pre-/eclampsia.
Cerebrovasc Dis. 2006;22(2–3):204–8.

?? 15. Which of the following clinical


­manifestations is more prevalent in
children with moyamoya disease?
A. dystonia
B. intracerebral hemorrhage
C. ischemic cerebrovascular events
..      Fig. 28.11 Magnetic resonance image showing D. optic disk abnormalities
multiple cortico-subcortical areas of hyperintense
E. subarachnoid hemorrhage
signal involving the occipital and parietal lobes
bilaterally and pons in a patient with posterior
reversible encephalopathy syndrome. Near fatal vv Correct Answer is: C
posterior reversible encephalopathy syndrome compli- Seventy to eighty percent of Moya
cating chronic liver failure and treated by induced Moya disease children can present with
hypothermia and dialysis: a case report. (J Medical
ischemic episodes in the form of tran-
Case Reports 2009, 3:6623. 10.1186/1752-­1947-­3-6623.
Rashmi Chawla, Daniel Smith and Paul E Marik. sient ischemic attacks or strokes that
Wikipedia) are ­frequently recurrent, involving the
718 Chapter 28 · Vascular

anterior brain circulation because of a


­predilection of the involvement of the
terminal portions of the internal carotid
arteries. Posterior circulation events can
occur through the involvement of the
vertebra-basilar system, but these events
are much less common. Signs are often
provoked by periods of hyperventilation,
e.g., after crying, coughing, straining, or
playing on a wind instrument.
(The reduced partial pressure
of carbon dioxide [Paco2] results in
reduced cerebral blood flow because
of vasoconstriction. The fall in cerebral
blood flow compromises the already
reduced cerebral perfusion to the
point of causing clinical manifesta-
tions, which can include dysarthria,
28 aphasia, hemiparesis, or seizures.)
..      Fig. 28.12 CT-scan of the brain with a RIGHT MCA
Atypical presenting signs include syn-
infarct. Own work. (Lucien Monfils. Wikipedia)
cope, para-paresis, visual disturbance,
and involuntary movements. Children
with frontal lobe involvement may vv Correct Answer is: C
also develop long-term intellectual The CT scan shows a right -side infarct in
­impairment. the distribution of the MCA; therefore, the
preferred response is left homonymous
zz Suggested Reading hemianopia.
55 Currie S, Raghavan A, Batty R, et al.
Childhood moyamoya disease and zz Suggested Reading
moyamoya syndrome: a pictorial 55 NIH Stroke Scale. 7 www.ninds.nih.gov.
review. Pediatr Neurol. 2011;44(6):
401–13. ?? 17. Which of the following is true regard-
ing septic cavernous sinus thrombo-
?? 16. You are evaluating an 18-year-old phlebitis?
female for her sudden onset of head- A. CSF usually shows a lymphocytic
aches. You ordered a non-­contrast CT pleocytosis
scan, which is shown. Her NIH Stroke B. CSF usually shows a polymorpho-
Scale is reported to be 2. nucleocyte pleocytosis
On neurologic examination, which C. engorgement of ophthalmic veins
of the following findings is most likely? leads to proptosis and chemosis
A. complete right homonymous D. infection is usually from extension
hemianopia from the mastoid air cells
B. forced gaze deviation to the left E. vision loss due to optic neuropa-
C. left hemiparesis with some effort thy is uncommon
against gravity
D. severe aphasia vv Correct Answer is: C
E. severe sensory loss involving the Vision loss is common and rarely recov-
right face, arm, and leg ers completely. CSF is typically normal
Vascular
719 28
unless there is an associated meningitis What is the best treatment option?
or subdural empyema. Venous congestion A. anticoagulation using enoxaparin
of ophthalmic and retinal veins leads to B. continued medical management
proptosis, chemosis, orbital edema, and with surgical intervention planned
retinal hemorrhage. Involvement of mas- for 5 to 6 months of age
toid air cells is more common with septic C. endovascular embolization per-
lateral sinus thrombosis. formed urgently
D. placement of a ventriculo-­
zz Suggested Reading peritoneal shunt
55 Ropper AH, Samuels MA. Infections of E. surgical removal of the
the nervous system (bacterial, fungal, ­abnormality
­spirochetal, parasitic) and sarcoidosis.
In: Ropper AH, Samuels MA, editors. vv Correct Answer is: C
Adams and Victor’s principles of This is a presentation of a vein of Galen
­neurology. 9th ed. New York: malformation (VGM). High-output cardiac
McGraw-­Hill; 2009. failure and neurologic symptoms second-
ary to cerebral venous congestion and
?? 18. You are evaluating a 3-day-old infant abnormal CSF flow are the most patho-
who has a congestive heart failure physiological consequences of VGM. The
which has been refractory to medical severity of the symptoms is variable and
management. He has impaired renal dependent on the underlying angioarchi-
function, with hydrops fetalis. You tecture of the VGM as well as the age of
ordered a head ultrasound, showed the child. Neonates typically present with
an abnormal vascular structure. Brain congestive heart failure; but infants pres-
CTA is obtained and shown here. ent with hydrocephalus; and the older
children or adults present with headache
and, frequently, with signs and symp-
toms of subarachnoid hemorrhage. The
development of endovascular therapy,
together with a comprehensive multidis-
ciplinary approach in the intensive care
units, has significantly improved the poor
prognosis of patients with VGM.
Despite advances in micro-neurosur-
gery, complete removal of the lesion in
newborns is rarely achieved or advised
because of the hemodynamic instability
and location of the lesion, poor myelina-
tion of the brain parenchyma and cere-
bral venous hypertension. Thus, surgical
treatment is now reserved for the evacu-
ation of intracranial hematomas and the
treatment of hydrocephalus, or in cases
of embolization failure.
Medical treatment with diuretics,
inotropes, and other cardiovascular
..      Fig. 28.13 Axial image from CTA showing arteriove-
nous communication in vein of Galen malformation. agents is used only to relieve symp-
Radiopaedia. (Dr. Laughlin Dawes. Wikipedia) toms of cardiovascular instability and
720 Chapter 28 · Vascular

renal insufficiency until the patient can the most reliable method for differen-
undergo embolization. tiating between a traumatic lumbar
Most of the newborns with VGM puncture and an SAH, its presence is not
need immediate treatment. The main pathognomonic for SAH. Recent studies
goal in the newborn with high-output have demonstrated that xanthochromia
cardiac failure is to decrease the flow begins to develop immediately after mix-
in the VGM and therefore reduce the ing of blood and CSF as seen in traumatic
cardiac output failure, improve the coro- lumbar puncture. The degree of xantho-
nary perfusion, and increase the blood chromia correlates with the amount of
flow to the renal arteries. bleeding induced by trauma. Clinically,
recent studies have established that in
zz Suggested Reading the presence of a traumatic lumbar punc-
55 Hoang S, Choudhri O, Edwards M, ture and an RSC concentration of 10,000
Guzman R. Vein of Galen malformation. RSC per L, xanthochromia cannot be reli-
Neurosurg Focus. 2009;27(5): E8. ably used to confirm SAH. Conversely,
xanthochromia in the setting of RSC
?? 19. Which of the following CSF findings is concentrations 5000 RSC per L is a reliable
most reliable for differentiating SAH indicator of SAH. The method of compar-
28 from a traumatic lumbar puncture? ing RSC counts in the first and fourth
A. A relative CSF leukocytosis will be tubes has been shown to be an unreliable
present in cases of SAH. means of differentiating these two enti-
B. The presence of xanthochromia ties. None of the other methods listed has
indicates SAH. been shown to be reliable.
C. A twofold or greater decrease in
the number of RBCs from tube 1 zz Suggested Reading
to tube 4 is always due to a trau- 55 1200 Questions to Help You Pass the
matic tap. Emergency Medicine Boards © 2012
D. A positive CSF clotting assay is Wolters Kluwer Health Lippincott
consistent with a traumatic tap. Williams & Wilkins. All rights reseNed.
E. All of the above is equally useful.
?? 20. You are evaluating a 7- year-old female
vv Correct Answer is: B because sub-­acutely she is not able to
Xanthochromia is the presence of a yel- move her left eye. While you are exam-
lowish coloration to the supernatant ining her, her right eye follows your
of centrifuged CSF samples. It second- hand movements, but left eye moves
ary to the breakdown of hemoglobin only minimally. The pupillary response is
first to oxyhemoglobin and then to absent in the left eye but it is normal in
bilirubin. Methemoglobin may also be the right eye. The rest of cranial nerves
produced, but like bilirubin, it occurs are normal. She has non-focal neuro-
after oxyhemoglobin is generated. These logical exam. Her left eye is swollen and
latter molecules have a yellowish tint erythematous. She had fever 2 days ago,
and characteristic spectrophotometric and seemed excessively tired today and
absorption curves. Typically, oxyhemo- complained of a headache.
globin will appear within 2 h of SAH and What is your most likely diagnosis?
reach a peak concentration between 24 A. Encephalitis
and 46 h. Bilirubin does not appear until B. Cavernous sinus thrombosis
approximately 10 h after SAH. Although C. Retinal vein occlusion
the presence of xanthochromia remains D. Eye trauma
Vascular
721 28
vv Correct Answer is: B vv Correct Answer is: D
The typical history is, development of The characteristic “puff of smoke” can
fever, malaise, and frontal headache be imaged by MRA or CT angiogram but
following dental procedures. However, a definitive diagnosis requires arterio-
infection can also originate from the face, graphic demonstration of bilateral steno-
nose or paranasal sinuses as well. Propto- sis in the distal internal carotid arteries
sis, orbital congestion, ptosis, external with collateral development in the basal
ophthalmoplegia, pupillary paralysis, and ganglia and meninges. It is important to
blindness may follow the initial symp- rule out an underlying vasculopathy and
toms. The infection begins in one cavern- coagulopathy.
ous sinus and may spread to the other. If MRI in this condition would identify
untreated, it may extend to the meninges. cerebral regions affected by ischemia
Even with vigorous antibiotic treatment, but does not image arteries.
the mortality rate is 15%. PET scan are not typically utilized
Encephalitis presents with altered to evaluate cause of stroke or vascular
mental status and seizures. Retinal vein disorders.
occlusion is rare in children and young MRA is an adequate non-invasive
adults. The presenting symptoms are arterial imaging mode, but the detec-
rapid onset blurred vision or loss of tion and resolution quality is not suffi-
vision. There was no evidence described cient for a definitive diagnosis.
of trauma to head or eye. Trauma would
not be associated with fever and would zz Suggested Reading
not typically present with intra-­ocular 55 Hackenberg A, Plecko B, Khan N. OP26-
muscle paralysis. 2867: Recurrent stroke in toddlers and
hemiplegic or atypical migraine in school
zz Suggested Reading aged children: Think of Moya Moya. Europ
55 Press CA, Lindsay A, Stence NV, J Pediatr Neurol. 2015;19:59.
Fenton LZ, Bernard TJ, Mirsky DM.
Cavernous sinus thrombosis in children ?? 22. A 6 -year-old boy is brought to the ED
imaging characteristics and clinical after a recent right ischemic middle
outcomes. Stroke. 2015;46(9):2657–60. cerebral artery stroke. He has no his-
tory of fever or any sickness. He has
?? 21. You are evaluating a 3-year-old girl normal cognitive and motor develop-
in the hospital with an acute onset ment. Family history is remarkable for
hemiparesis. She has a remarkable deep vein thrombosis in his father.
past medical history of four tran- What laboratory test is most likely to
sient ischemic attacks. Her previous aid in identifying the etiology of this
workups for hematological etiolo- child’s stroke?
gies were normal and there were no A. Serum triglyceride
signs of inflammatory conditions. B. Serum cholesterol
The diagnosis of Moya Moya is C. Plasma Protein C and S
­suspected. D. Muscle biopsy
What study could make a definitive
diagnosis in her condition? vv Correct Answer is: C
A. MRI of the base of the brain A history of deep vein thrombosis or
B. Positron Emission Tomography a positive family history for deep vein
C. Magnetic resonance angiography thrombosis are the risk factors for a
D. Angiography prothrombotic state in a child. Protein C
722 Chapter 28 · Vascular

or Protein S deficiency, antithrombin Ill would still allow the other side to carry
deficiency, factor V Leiden mutation and bilateral auditory information to higher
antiphospholipid antibody syndrome are processing centers.
the most common hypercoagulable states Visual information travels through
in children. the optic nerve to the lateral geniculate
Serious atherosclerosis formation body and then to the occipital lobe.
causing stroke in children as a conse- Auditory information is transmitted
quence of hypertriglyceridemia is rare. with bilateral representation before the
Certain mitochondrial disorders lateral lemniscus pathways.
are associated with stroke in children,
but are accompanied with preexisting zz Suggested Reading
cognitive and motor deficits. Mitochon- 55 Celesia GG. Hearing disorders in brain-
drial conditions include mitochondrial stem lesions. Handbook Clin Neurol.
encephalopathy, Lactic acidosis and 2014;129:509–36.
stroke (MELAS).
?? 24. What do you expect as a diagnosis in
zz Suggested Reading a 5 -year-old female with a history of
55 Doan D, Barreto A, Gildersleeve K, frequent ear infections and tonsillitis
28 Sarraj A, Butler I. Comparison of that presented with fever, right neck
Hypercoagulable Evaluation in Perinatal tenderness and subsequent rapid
and Pediatric lschemic Stroke (P2. 261). onset of a right-sided hemiplegia?
Neurology. 2015;84(14 Suppl):P2–261. A. Moya Moya disease
B. Protein C deficiency
?? 23. An 8 -year-old boy suffered a minor C. Cervical arteritis
ischemic stroke, during an episode of D. Sickle cell anemia
vasculitis, involving the right lateral
lemniscus. vv Correct Answer is: C
What sensory disturbance will she Children with chronic tonsillitis and otitis
likely experience once she recovers? media with cervical lymphadenopathy
A. Left visual field deficient can have an occlusion of cervical portions
B. Right visual field deficit of the internal carotid arteries. Reduction
C. Leaf ear hearing loss in lumen diameter and turbulence in flow,
D. Right ear hearing loss which increases the risk of thromboem-
E. No auditory of visual deficit bolism generation can occur secondary
to cervical arteritis which is a local inflam-
vv Correct Answer is: E matory vasculitis.
The auditory portion of the eighth cranial Moya Moya is a slowly progressive
nerve transmits information to the ipsilat- non-­inflammatory vasculopathy that
eral cochlear nuclei in the medulla. Each most commonly results in the occlu-
cochlear nucleus sends information to sion of the internal carotid arteries.
both superior olivary nuclei and therefore Rapid onset of a hemiplegia can occur
bilateral representation of hearing occurs but not associated with fever and neck
through the remainder of the ascending ­tenderness.
pathways. Both lateral lemniscus carries Protein C deficiency is a hereditary
information from both superior olivary condition that causes hyper-coagula-
nuclei to the inferior colliculi. Even com- bility and increases risk of stroke. Acti-
plete destruction of one lateral lemniscus vated protein C inactivates coagulation
Vascular
723 28
factors Va and VIIIa and normal levels However, arterial ischemia in the form of
aids in the coagulation hemodynamic venous infarction is seen, as are paradoxi-
balance. A deficit in protein C produc- cal emboli to the cerebral vasculature
tion results in a high risk of vascular from the venous system through an intra-­
thromboembolism formation. cardiac right­to-left shunt. Other causes
Jaundice, pallor, weakness and fati- of both venous and arterial thrombosis
gability from chronic anemia are the include malignancy, heparin-induced
most common presentations of sickle thrombocytopenia, systemic vasculitis
cell anemia. Forty percent of patients and myeloproliferative diseases (such
can have strokes at some time in their as essential thrombocythemia, polycy-
lives affected individuals. Dehydration themia rubra vera, and others). APLS is a
is the most commonly known cause of a disorder of thrombosis resulting from the
sickle cell crisis that can cause cerebral presence of circulating antibodies against
hypo perfusion or stroke. phospholipid­bound proteins such as
anti-cardiolipin antibodies and lupus
zz Suggested Reading anticoagulant antibodies. In addition to
55 Weiss PF. Pediatric vasculitis. Pediatr Clin venous and less commonly arterial throm-
North Am. 2012;59(2):407–23. bosis, other neurologic manifestations
seen in APLS include chorea, headaches,
?? 25. Which of the following has not been and seizures.
associated with increased risk of
venous thrombosis that may involve zz Suggested Reading
the CNS? 55 Comprehensive review in clinical neurol-
A. Anti-phospholipid antibody syn- ogy: a multiple-choice question book for
drome the wards and boards © 2011 Wolters
B. Acquired factor VIII deficiency Kluwer Health Lippincott Williams &
C. G2021O A prothrombin polymor- Wilkins. All rights reserved.
phism
D. Antithrombin Ill deficiency ?? 26. A patient has a large intracranial hem-
E. Factor V Leiden orrhage. On examination, he has a left
fixed dilated pupil and he has hemi-
vv Correct Answer is: B paresis on the right side.
Acquired hemophilia (Acquired factor Which type of herniation does this
VIII deficiency) leads to increased risk patient have?
of hemorrhage rather than thrombosis. A. Uncal herniation
Anti-­phospholipid antibody syndrome B. Subfalcine herniation
(APLS) and most of the so-called heredi- C. Tonsillar herniation
tary thrombophilias, including G20210 A D. Central transtentorial herniation
prothrombin polymorphism, mutation in E. Transcalvarial herniation
the methylene tetrahydrofolate reductase
gene (leading to hyper-homocystein- vv Correct Answer is: A
emia), and factor V Leiden, as well as oth- Uncal herniation produces mass effect
ers including anti-­thrombin Ill, protein C, and pressure over the ipsilateral mid-
and protein S deficiency, lead to venous brain, affecting the ipsilateral cranial
thrombosis. Therefore, venous sinus nerve Ill nucleus, and the corticospi-
thrombosis is the most common neuro- nal fibers. The mass effect compresses
logic manifestation of these disorders. parasympathetic fibers that mediate
724 Chapter 28 · Vascular

miosis, resulting in mydriasis. Since the D. Carotid artery, through the carotid
corticospinal tract has not decussated at canal
the level of the midbrain, patients have E. Middle meningeal artery, through
a contralateral hemiparesis. Occasion- the foramen spinosum
ally, the uncal herniation will lead to
displacement of the midbrain against the vv Correct Answer is: E
contralateral Kernohan’s notch, result- Epidural hematoma is most commonly
ing in a contralateral compression of caused by head trauma, leading to rup-
the corticospinal tract, and therefore an ture of the middle meningeal artery,
ipsilateral hemiparesis. The other types of which passes through the foramen spino-
herniation syndromes do not include an sum. Rupture of this artery results in accu-
ipsilateral dilated pupil with contralateral mulation of blood in the epidural space.
­hemiparesis. The appearance on CT is lenticular shaped
or biconvex. Clinically, patients may pres-
zz Suggested Reading ent with a brief loss of consciousness fol-
55 Posner JB, Saper CB, Schiff ND, Plum F. lowed by a lucid interval and subsequent
Plum and Postern’s diagnosis of stupor and deterioration over hours.
coma, 4th ed. New York: Oxford The jugular vein passes through the
28 University Press; 2007. jugular foramen; however, injury to it is
55 Comprehensive review in clinical neurol- not the cause of epidural hematomas.
ogy: a multiple-choice question book for The carotid artery passes through
the wards and boards ©2011 Wolters the carotid canal and runs in the fora-
Kluwer Health Lippincott Williams & men lacerum; however, injury to this
Wilkins. All rights reserved. structure is not the cause of epidural
hematomas.
?? 27. An 18-year-old man hit his head
on a rock while he was rafting with zz Suggested Reading
his father. He had no helmet on. 55 Blumenfeld H. Neuroanatomy through
His father noticed that the boy was Clinical Cases, 1st ed. Sunderland:
slightly confused immediately after Sinauer; 2002. Suarez JI.
the head trauma, but he came back 55 Critical care neurology and neurosurgery,
to normal after few minutes. Few 1st ed. Totowa: Humana Press; 2004.
hours later he became lethargic and 55 Comprehensive review in clinical neurol-
his right side was not moving prop- ogy: a multiple-choice question book for
erly. He was taken to the emergency the wards and boards ©2011 Wolters
room of a local hospital. The CT scan Kluwer Health Lippincott Williams &
showed an epidural hematoma. Wilkins. All rights reserved.
What is the vessel ruptured and
through which foramen does it ?? 28. Which of the following is correct
enter the skull? regarding thrombosis of the venous
A. Jugular vein, through the jugular sinuses?
foramen A. Diplopia with a sixth cranial nerve
B. Middle meningeal artery, through palsy is specific for cavernous
the foramen lacerum sinus thrombosis
C. Jugular vein, through the foramen B. Headache is present in less than
spinosum 50% of cases
Vascular
725 28
C. Increased intracranial pressure is ?? 29. You are seeing a newborn baby in the
uncommon NICU, who has a high-output cardiac
D. Superior sagittal sinus thrombosis failure at birth. His head circumfer-
may produce bilateral thalamic ence has been increased in size and a
venous infarcts cranial ultrasound is pending.
E. Seizures are more common in What is your treatment of choice in
venous infarcts as compared to this condition?
arterial infarcts A. Ventroperitoneal shunt placement
B. Aggressive treatment of cardiac
vv Correct Answer is: E disease
Venous sinus thrombosis is associated C. Embolization of cerebral vein
with seizures in 40% of patients, it is a D. Close observation and monitoring
higher percentage than that in patients
with arterial strokes. In patients with vv Correct Answer is: C
venous sinus thrombosis, Headache is Vein of Galen malformation is actually
the most common symptom in adults a median prosencephalic arteriovenous
(about 90%). Given the occlusion of fistula (ANF) that dilates, it is associated
venous drainage along with hemorrhagic with hydrocephalus secondary to venous
infarct and edema, patients may develop hypertension or compression of the aque-
increased intracranial pressure. Diplopia duct of Sylivis. This is the most common
caused by a sixth cranial nerve palsy antenatal diagnosed intracranial mal-
has non-localizing value and may be a formation. The median prosencephalic
manifestation of increased intracranial vein is a precursor to the development of
pressure. In patients with venous infarcts, the vein of Galen and typically regresses
focal neurologic findings will be present before 40 weeks gestation. On ausculta-
depending on the area affected along the tion of the skull, a cranial bruit can be
thrombosed venous sinus. Thrombosis heard. Cardiac symptoms are present due
of the deep venous system may lead to to the large left-to-right shunting through
deep venous infarcts, including bilateral the cerebral arteriovenous fistula.
thalamic infarcts. This is not seen with Embolization of the AV fistula resolves
superior sagittal sinus thrombosis, which the cardiac failure and will gradually allow
instead can lead to infarcts in the para- cerebrospinal fluid to flows more nor-
sagittal cortex bilaterally along the sinus. mally as the venous pressure subsides.
A ventro-peritoneal shunt does not
zz Suggested Reading address the etiology of the increased ven-
55 Ropper AH, Samuels MA. Adams and tricular pressure. In fact, reducing ven-
Victor’s principles of neurology, 9th ed. tricular pressure may allow more rapid
New York: McGraw­Hill; 2009. expansion of the arteriovenous fistula.
55 Stam J. Thrombosis of the cerebral veins Since the high-out cardiac cycle is
and sinuses. N Engl J Med. 2005;352: compensatory and survival depends
1791–8. on it, aggressive treatment to resolve it
55 Comprehensive review in clinical neurol- would be harmful.
ogy: a multiple-choice question book for This condition does not typically
the wards and boards ©2011 Wolters resolve spontaneously. The infant
Kluwer Health Lippincott Williams & would have a cardiac failure without
Wilkins. All rights reserved. ­intervention.
726 Chapter 28 · Vascular

zz Suggested Reading zz Suggested Reading


55 Sheth RD, Tiznado-­Garcia E. 2011. Vein of 55 Brazis PW, et al. Localization in clinical
Galen malformation. neurology. 6th ed. Chapter 8: the localiza-
tion of lesions affecting the ocular motor
?? 30. You are evaluating a 16 -year-old system. 2011. p. 184–206.
male for his progressive double vision
and left eye pain for 2 days. He has ?? 31. A healthy 10- year- old female had
incomplete left eye ptosis and oph- been crying after skinning her knee
thalmoplegia, but his visual acuity is on the playground. She devolved sud-
preserved. Sensation in the face on den onset of left face, arm, hand, and
the left is diminished in the V1 and V2 leg weakness and numbness, which
distribution. The rest of the neurologi- resolved completely in 30 min. She
cal examination is normal. has a normal MRI, and angiography
Where is lesion causing this clinical was ordered.
presentation?
A. Cerebral peduncle
B. Superior orbital fissure
C. Cavernous sinus
28 D. Orbital apex
E. Central pons

vv Correct Answer is: C


This is a clinical picture of a cavernous
sinus thrombosis. Infection is the main
cause of the cavernous sinus throm-
bosis, and causes ipsilateral ptosis and
ophthalmoplgia by involving CN Ill, IV,
and VI. Ipsilateral loss of sensation is
due to involvement of V1 and V2 of the
trigeminal nerve which traverse through
the cavernous sinus as well. Visual acuity
is usually preserved, but can be affected ..      Fig. 28.14 Right ICA DSA in patient with moya
late in the course with progressing infec- moya disease. (Dr. Frank Gaillard. Wikipedia)
tion. Antibiotics based on the underlying
infection is the treatment of choice. The most likely diagnosis is:
Superior orbital fissure syndrome A. Carotid dissection
is similar to cavernous sinus syndrome, B. Moyamoya disease
however, the V2 portion of the trigemi- C. Vasculitis
nal nerve exits the skull via the foramen D. Fibromuscular dysplasia
rotundum after traversing through the E. Non-vascular cause of symptoms
cavernous sinus, and thus does not
cross the superior orbital fissure. vv Correct Answer is: B
Orbital apex syndrome is also similar There is a stenosis of the supra-clinoid
to cavernous sinus syndrome, but has Right ICA with the development of col-
early involvement of the optic nerve, lateral circulation on the angiogram,
thus resulting in loss of visual acuity. consistent with Moya Moya disease. Moya
Lesions of the cerebral peduncle of Moya disease consists of progressive
the midbrain and the central pons would non-atherosclerotic bilateral stenosis of
affect motor tracts and result in weakness. the distal internal carotid arteries and
Vascular
727 28
trunks of the anterior and middle cerebral vv Correct Answer is: A
arteries. Collaterals develop at the base Substitution of amino acid at position
of the brain, with a characteristic “puff number 6 of the b-polypeptide chain of
of smoke” appearance on angiography. hemoglobin, with valine in place of glu-
Data suggest, however, that the classic tamic acid is the etiology for sickle cell
“puff of smoke” may only be evident in anemia. It is an autosomal recessive dis-
approximately 30% of cases. Intra-cranial order, and it is more common African or
aneurysms may also be present. The dis- African­American patients. Hemoglobin S,
ease is most common in Japanese girls, is an abnormal hemoglobin which is less
and may be of autosomal recessive inheri- soluble, more viscous, and forms poly-
tance. The same angiographic pattern can mers when deoxygenated, which leads
also be seen with a number of conditions, to a characteristic sickle shape of the red
however, including sickle cell anemia, cell and membrane rigidity. The abnor-
basilar meningitis, tuberous sclerosis, and mal cells adhere to vessel walls causing
many others. Hence, Moya Moya may be vaso-occlusive crises with severe pain,
considered as a descriptive term for the often in the back, chest, and extremities.
vascular process rather than a specific Strokes usually occur in early childhood.
disease. Symptoms in children tend to be In children, typically it is an ischemic
ischemic, while manifestations in adults stroke, while initial strokes in adults are
tend to be hemorrhagic. Moya Moya can most often hemorrhagic. Proximal arterial
cause TIA, infarcts, headaches, seizures, stenosis with a Moya Moya pattern may
movement disorders, cognitive decline, be evident. The risk of ischemic stroke
and hemorrhages. TIAs may be precipi- is increased with a prior TIA, elevated
tated by crying or hyperventilation. systolic blood pressure, low hemoglobin
levels, and frequent or recent acute chest
zz Suggested Reading syndromes. Risk ­factors for hemorrhagic
55 Biller J, Love BB. Vascular diseases of the stroke include low hemoglobin levels and
nervous system. In: Bradley WG, Daroff elevated white blood cell counts. Only
RB, Fenichel GM, Jankovic J, editors. chronic transfusion therapy has been
Neurology in clinical practice. 4th ed. shown to dramatically reduce the
Elsevier, Inc.; 2004. p. 1197–250. risk of initial strokes in high-risk c­ hildren.
55 Simon RP, et al. Clinical neurology. 4th ed.
Stamford: Appleton & Lange. p. 282–3. ?? 33. A 4-year-old girl came with acute
55 Images from: Shoukat S, et al. Moyamoya onset of flaccid weakness of the left
disease: a clinical spectrum, literature side of her body, and inability to talk.
review and case series from a tertiary care She is lethargic, on stimulation she
hospital in Pakistan. BMC Neurol. awakens and shows visual regard. Her
2009;9:15. past medical history is significant for
complex congenital heart disease,
?? 32. Which of the following is standard of including transposition of the great
care for primary stroke prevention in vessels, and underwent an arterial
children with sickle cell. switch procedure at 15 months of age,
A. Chronic transfusion therapy with no complications. She has no
B. Aspirin history of recent illnesses or injuries.
C. Warfarin Head CT shows a hypodense region
D. Hydroxyurea in the distribution of the right middle
E. None of the above; no preventa- cerebral artery (MCA), and CT angiog-
tive treatment has been shown to raphy shows a sharp cutoff of blood
be effective flow distal to the right MCA origin.
728 Chapter 28 · Vascular

What test is most likely to identify the for predicting the outcome of patients
etiology of this stroke? with intra-parenchymal hemorrhage in
A. Echocardiogram 30 days after the event. The score ranges
B. Factor V Leiden mutation analysis from 0 (least severe with low expected
C. Erythrocyte sedimentation rate mortality) to 6 (the worst possible score
D. Holter monitor with death likely). The total ICH Score
E. Protein C level is the sum of the points of the 5 various
variables which are considered as predic-
vv Correct Answer is: A tors of outcome in ICH. These variables
Acute ischemic stroke in children has are Glasgow Coma Scale (GCS), age, loca-
many causes, but congenital heart disease tion of hematoma, volume of hematoma
is the most common. Many different car- and presence or absence of intraventricu-
diac conditions may be responsible, for lar hemorrhage. Serum glucose level was
example right-to-left intra-cardiac shunt- not shown to be a predictive of 30-day
ing of blood flow, increases the risk of outcome.
emboli entering the cerebral circulation.
GCS
Other causes include hemoglobinopa-
  3–4 = 2 pts
28 thies (such as sickle cell disease), acquired
vasculopathies (such as Moya-Moya syn-
  5–12 = 1 pt.
  13–15 = 0 pts
drome), systemic illness (such as varicella),
and multiple hypercoagulable states. ICH Volume
  >30 cm3 = 1 pt.
zz Suggested Reading   <30 cm3 = 0 pts
55 Roach ES. Etiology of stroke in children. IVH
Semin Pediatr Neurol. 2000;7:244–60.   Yes = 1 pt.
  No = 0 pts
?? 34. Which of the f­ ollowing is not con-
sidered as a predictive variable of Location
outcome in patients with intra-paren-   Infratentorial = 1 pt.
chymal hemorrhage?   Supratentorial = 0 pts
A. Serum glucose level Age
B. Age   >80 yrs = 1 pt.
C. Volume of the hematoma   <80 yrs = 0 pts
D. Location of the hematoma
E. Glasgow Coma Scale (GCS) zz Suggested Reading
55 Hemphill JC 3rd, Bonovich DC,
vv Correct Answer is: A Besmertis L, et al. The ICH score: a simple,
The intracerebral hemorrhage (ICH) reliable grading scale for intracerebral
score is a well-validated scoring system hemorrhage. Stroke. 2001;32:891–7.
729

Supplementary
­Information
Index – 731

© Springer International Publishing AG, part of Springer Nature 2018


Y. M. Awaad, Absolute Pediatric Neurology, https://doi.org/10.1007/978-3-319-78801-2
731 A

Index

A
Adrenomyeloneuropathy 90, 96, Astatic seizures 245
132, 134 Astrocytoma 574
AACG, see Acute angle closure Adults with viral encephalitis 312 Ataxia telangiectasia (AT) 47, 115,
glaucoma (AACG) Advanced neurological imaging 144, 566
Abetalipoproteinemia 87, 140 techniques 35 Atenolol 369
–– clinical presentation 426 AED prophylaxis 212, 254, 281 Athetosis 460
Absence epilepsy 211 AHC, see Alternating hemiplegia of Atomexetine 27
Absence seizures 200, 308 childhood Attention-deficit hyperactivity
Absolute neutrophil count (ANC) 400 Aicardi syndrome 73, 308 disorder (ADHD) 18, 22, 24, 25, 44
Acetominophen 354 AIP, see Acute intermittent porphyria Auditory seizures 304
Acetylcholine receptor (AChR) 645 Alexander disease 68, 142 Autism 25
–– antibody-positive disease 512 Alice in Wonderland Autism Diagnostic Interview-Revised
Achilles tendons 515, 517 syndrome 366, 375 (AID-R) 9
Acid alpha-glucosidase 42 Almotriptan (Axert) 370 Autism Diagnostic Observation
Acquired developmental skills Alpha-dystroglycanopathies 45 Schedule (ADDOS) 9
(neuro-regression) 419 Alpha L iduronidase deficiency 425 Autism spectrum disorder
Acquired hemophilia (Acquired factor Alpha-N-acetylgalactosaminidase (ASD) 22, 65
VIII deficiency) 723 deficiency 60 –– catatonia-like episodes 11
Acquired strabismus/nystagmus 568 Alternating hemiplegia of childhood –– child’s language delay 6, 7
Activité moyenne 277 (AHC) 250, 260 –– clinical presentation 9
Acute angle closure glaucoma American Academy of Pediatrics 336 –– 16p11.2 deletion/duplication 2
(AACG) 240 Amitryptiline 285, 349, 369 –– developmental delay evaluation 2
Acute demyelinating encephalomy- Amoxapine 684 –– diagnosis 2
elopathy, see Acute disseminated Andersen-Tawil syndrome 532 –– emotionless 8
encephalomyelitis (ADEM) Anencephaly 606 –– environmental factors 2
Acute disseminated encephalomyeli- Angelman syndrome (AS) 42, 176, –– with epilepsy in children and
tis (ADEM) 147, 150, 387 196, 263 adolescents 4
Acute dystonic reaction 353 Anosognosia 608 –– genetic/environmental risk
Acute hepatic failure 493 Antidepressant medications 675 factors 3, 4
Acute inflammatory demyelinating Anti-endomysial antibodies 494–496 –– hearing loss 2
polyneuropathy (AIDP) 146, 517 Antiepileptic drug (AED) 151, –– identification of 9
Acute intermittent porphyria 212, 254 –– language acquisition delay 14, 15
(AIP) 110, 256, 428, 429 Anti-microbial therapy 399 –– management 10
Acute ischemic stroke 75 Anti-phospholipid antibody –– odd behavior 6
–– in children 728 syndrome (APLS) 723 –– older paternal age 4
Acute liver failure 27 Antisocial behavior 21 –– restrictive and repetitive
Acute optic neuritis 153 Apgar scores 666 behaviors 6
Acute pain 696 Applied behavioral analysis –– social interaction 6
Acute postinfectious cerebellitis 405 (ABA therapy) 7, 10 –– symptoms 6
Acute rheumatic fever (ARF), Arginase deficiency 430, 663 –– thalidomide 5
Sydenham’s chorea 447 Argininosuccinic aciduria 423 –– topiramate 5
Acute serous labyrinthitis 390, 391 Aripiprazole 175 –– treatment interventions 10
Acute transverse myelitis Arm flapping 463 –– TSC1 mutations 13
(ATM) 655, 657 Arnold Chiari II malformation 658 –– tuberous sclerosis 11
Adenoma sebaceum 477 Arrhinencephaly 95, 114 –– valproic acid 5
ADHD, see Attention-deficit/ Arteriovenous malformation Autistic behaviors 242
hyperactivity disorder (AVM) 709 Autistic spectrum disorder (ASD) 58
ADNFLE, see Autosomal dominant ARX mutations 38 Autoimmune thyroiditis 412
nocturnal frontal lobe epilepsy Asceptic meningitis 397, 398 Autonomic status epilepticus 286
Adrenocorticotropic hormone ASD, see Autism spectrum disorder Autosomal dominant familial
(ACTH) 216 Ash leaf spot 58 paroxysmal choreoathetosis 239
Adrenoleukodystrophy (ALD) 90, 96, Asperger’s syndrome 3, 6, 9, 10 Autosomal dominant GTP cyclohydro-
134, 421 Assent 338 lase deficiency 448
732 Index

Autosomal dominant nocturnal Benign Rolandic Epilepsy 193, 602 –– oxcarbazepine 680, 683, 685
frontal lobe epilepsy Benzodiazepines 273, 371, 676 –– toxicity 669
(ADNFLE) 244, 251, Beta-hexosaminidase A activity 56 Carbamoyl phosphate synthetase
270, 272, 276, 281, 640 BFNC, see Benign familial neonatal deficiency 423
Autosomal mosaicism for aneu- convulsions Cardiac conduction deficits 507
ploidy 483 BHS, see Breath-holding spells Cardiac musculature 534
Axillary neuropathy 700 Bilateral auditory information 722 Cardiac rhabdomyoma 11, 469–470
Axon Sports Computerized Cognitive Bilateral optic neuritis 155 Cardiomyopathy 94
Assessment Tool 696 Bilateral rhythmic contractions, Carditis 452
central tegmental tract 593 Carnitine palmitoyltransferase 1A

B
Bilateral stenosis 721 (liver) (CPT1A) 426
Biotinidase deficiency 439 Cavernous sinus thrombosis 407, 726
–– biotin levels 433 CDKL5 mutations 179, 243
Bacterial meningitis 338, 386,
Blood-brain barrier, cell types 604 Celiac disease 463, 495
387, 400
Blood-nerve barrier 678 Central cholinergic synapses,
–– Bacterial Meningitis Score 400
Blood test 60 haloperidol 613
–– in children 398
BNFS, see Benign familial neonatal Central core myopathy 541
–– with predominance of neutro-
seizures Central nervous system, myelina-
phils 402, 403
Bone marrow transplantation 421, tion 599, 600
Balance Error Scoring System
440 Centrocaecal scotoma 586
(BESS) 696
Borrelia burgdorferi 385 Centro-nuclear/myotubular
Ballismus 460
Botulinum toxin type A 671 myopathy 540, 548
Bardet-Biedle syndrome 71
Botulism immune globulin 509 Centrotemporal spikes 264, 273
Bartonella infections 381
Brachial plexus birth palsy Cerebellar abnormalities 660
Basal ganglia 612
(BPBP) 644, 652 Cerebellar astrocytoma 370
–– pathology 593
Brain herniation 697 Cerebellar ataxia, acquired
Basilar meningitis, neurologic and
Brain metastases 579, 580 causes 462, 463
radiographic findings 379
Brainstem injury 697 Cerebellar atrophy 616
Basilar migraine 343, 347
Brain teratomas 576 Cerebellum pathology 593
Bassen-Kornzweig syndrome 140
Brain tumors, corticosteroids 573 Cerebral autosomal dominant
–– Abetalipoproteinemia See also
Breath-holding spells (BHS) 255 arteriopathy with subcortical
B12 coenzyme synthesis and
Breech delivery 644 infarcts and leukoencephalopathy
transport 432
British anti-lewisite (BAL) 681 (CADASIL) 376
Becker disease 518, 536
Broad spectrum antiepileptic Cerebral cortex architecture 607, 608
Becker muscular dystrophy 142, 533
medications 296 Cerebral folate deficiency 280
Becker phenotype 523
Bulimia nervosa 186 Cerebral hemorrhage 679
Becker type myotonia 528
Burst suppression 277 Cerebral palsy (CP) 135, 661,
Behavioral therapy 24
663, 664, 666
Bell’s palsy 698

C
–– brain MRI 668
Benign childhood epilepsy with
–– characterization 664
centrotemporal spikes
–– diagnosis of 665
(BECTS) 188, 195, 211, 222, 230 CADASIL, see Cerebral autosomal
–– diplegic 663, 668
Benign familial neonatal convulsions dominant arteriopathy with
–– dystonia 670
(BFNC) 248, 258, 461 subcortical infarcts and leukoen-
–– etiologies 663, 667
Benign familial neonatal seizures cephalopathy
–– hemiparetic 668
(BNFS) 244 Calcium channel 366
–– hemiparetic CP 663
Benign febrile convulsions 257 Canavan disease 52, 58, 119, 431
–– hemiplegic 663, 668
Benign focal epilepsy 316 –– diagnosis 127
–– hypertonia 670
–– central mid temporal spikes 602 CANS, see Childhood Acute
–– incidence of 663
Benign hereditary chorea 445 Neuropsychiatric Syndrome
–– neuroimaging 663
Benign idiopathic neonatal Carbamazepine 212, 272, 313,
–– quadriparetic 663, 668
seizures 305 679, 684
–– schizencephaly 669
Benign intracranial hypertension 374 –– auto-induction 680, 684
–– SCPR report 664
Benign neonatal seizures 311 –– 10, 11-carbamazepine
–– spasticity 670, 671
Benign neonatal sleep epoxide 680
–– symptoms 664
­myoclonus 254 –– hepatic enzyme inducer 680, 684
Cerebral venous sinus thrombosis
Benign paroxysmal torticollis 444 –– liver metabolism with renal
(CVST) 706
Benign paroxysmal vertigo 261, 342 excretion of metabolites 680, 684
Index
733 A–D
Cerebrospinal fluid analysis, ampicillin Cisplatin 681 Corpus callosum 112
and gentamicin 403, 404 Citrullinemia type I 424 –– partial agenesis 433, 434
Ceroid lipofuscinosis, neuronal 1 Classic lissencephaly 37, 101 Cortical development, superficial
(CLN1) 89 Classic tuberous sclerosis 58 cortical layers 609
Ceroid lipofuscinosis, neuronal 4 Clinically isolated syndrome (CIS) 152 Cortical injury 695
(CLN4) 89 Clonazepam 324 Cortical spreading depression 368
Ceroid lipofuscinosis, neuronal 10 Clostridium botulinum 393, 394, 548 Corticosteroids 515, 647, 694
(CLN10) 87 Cluster headaches 351 –– therapy 517
Cervical lymphadenopathy, CMAP, see Compound muscle action Cough headache 363
mycobacterial infections 382 potentials (CMAPs) Cranial nerve injury 592
Cervical spine (C-spine) CMT1X neuropathy 537 Cranial neuralgia, ophthalmologic
injuries 646, 694 CNS vasculitis 706 migraine 587
Charcot-Marie-Tooth (CMT) Cobb syndrome 481 Craniopharyngioma 105, 564, 565,
disease 524, 526, 545 Cobblestone lissencephaly 91, 99, 571, 589
–– CMT1 530 102, 143, 534 Cri du chat syndrome 79
–– CMT1B 525 Cocaine 675 Crossed adduction 600
–– CMT1A 523, 524, 546 Cognitive involvement 172, 178 Crouzon syndrome 57
–– CMT2 group 525, 530, 547 Communicating hydrocephalus 617 Cutaneomeningospinal
–– CMT2D 545 Communication, family members 335 ­angiomatosis 481
–– CMT3 525, 547 Complex regional pain syndrome 362 Cyanotic breath-holding 285
–– CMT4 524, 530 Compound muscle action potentials Cyclic vomiting syndrome
–– CMTX 525, 547, 667 (CMAPs) 517, 521, 539 (CVS) 342, 347
Checklist for autism in toddlers Comprehensive Behavioral Interven- Cyclophosphamide 682
(CHAT) 2 tion for Tics (CBIT) 444 Cyproheptadine 285, 348, 676
Cherry-red spot 85, 104, 111 Concussions 689, 690, 694 Cystathionine 13-synthase
Chiari malformation 651 Confusional arousals 627, 636 ­deficiency 432
–– type I 74, 106, 346, 648 Congenital AIDS 405 Cytochrome c oxidase (COX) 41
–– type II 78, 614 Congenital bilateral perisylvian Cytomegalovirus (CMV) infection 383
Child neurologists 336, 359 syndrome 7 –– urine from infant positive 387, 388
Childhood absence epilepsy Congenital craniopharygiomas 576
(CAE) 189, 217, 245, 317 Congenital cytomegalovirus 183
–– diagnostic EEG pattern 593
Childhood acute neuropsychiatric
Congenital disorders of glycosylation
(CDG) 137
D
syndrome (CANS) 22 Congenital ependymomas 576 Dancing eye and dancing feet
Childhood epileptic encephalopathy Congenital heart disease 183, 259 syndrome 555
syndromes 20 Congenital Horner syndrome 695 Dandy-Walker Syndrome (DWS) 32,
Childhood leukemia Congenital hypomyelinating 50, 51, 82, 620
–– complications 496 ­neuropathy 679 Dantrolene 671
–– cranial irradiation 497 Congenital meningiomas 576 Daytime sleepiness 627, 630, 634,
–– treatment 496 Congenital muscular dystrophy 45 639, 640
Chorda tympani branch 698 Congenital myasthenic syndrome Deafness 31
Chorea 460 (CMS) 520 Deep belly laugh 648
Choreaform movements 459 Congenital myotonic Deep intracerebral hemorrhage 660
Choriocarcinoma (nongerminoma- dystrophy 506, 519 Deep tendon reflexes 645, 649, 653
tous germ cell tumor) 574, 582 Congenital toxoplasmosis 7 Dehydrocholesterol 123
Choroid plexus carcinoma 580, 581 (CT) 378, 379 Dejerine-Sottas syndrome 525, 530
Choroid plexus papilloma 582, 583 –– symptoms 403 Delayed sleep phase syndrome 631
Chromosomal microarray analysis 9 Congestive heart failure Delta brushes 293
Chronic daily headache syn- –– and increased intracranial Denervation disease 529
drome 353, 360, 367 pressure 619 Denervation neuropathies 531
Chronic lead exposure in children 681 –– vein of Galen 619 Dermatomyositis 527, 529
Chronic medical conditions 284 Contemporary medical ethics 335 Developmental delay 138
Chronic paroxysmal hemicrania Conversion disorder and substance –– definitive etiology 419
(CPH) 351 abuse 630, 639 –– evaluation, lead level 413
Chronic prophylaxis 314 Convulsions 298 –– white matter (leukodystrophy) vs.
Chronic recurrent headaches 357 Cori/Forbes disease (glycogen storage grey matter (poliodystrophy)
Circadian rhythm disorder 631 disease, type III) 422 disorder 419
734 Index

Diastematomyelia 657 ELISA (enzyme linked Extracranial injuries 691


Diazepam 684 ­immunoelectrotransfer) 401 Eye closure, alpha rhythm 598
Diffuse intrinsic brainstem Embryonal carcinoma 573, 582 Ezogabine 270, 324
­gliomas 558 Embryonal nervous system

F
Diffuse intrinsic pontine development 602–605
gliomas 562, 563 Emery-Dreifuss muscular
DiGeorge syndrome 259 ­dystrophy 507, 524, 541
Fabry disease 75, 90, 92
Dihydroergotamine (DHE) 375 Encephalitis 679, 721
Facial angiofibromas 477
Diplegic CP 663, 668 Encephalocele 606
Facioscapulohumeral dystrophy 72
Disorders of cerebral folate Encephalopathy 382
Fahr disease (idiopathic basal ganglia
­metabolism 11 Encephalotrigeminal
calcification) 458
Distal symmetric polyneuropathy 686 ­angiomatosis 473
Familial hemiplegic migraine
Divalproex sodium 212 Enteric nervous system 64
(FHM) 345, 461
Doose syndrome 263, 264 Enzymatic analysis 93
Fat embolism 702
DOPA decarboxylase deficiency 431 Enzyme replacement therapy
Fazio-Londe disease 31
Dopa-responsive dystonia (DRD) 421, (ERT) 43
Febrile hemi-clonic status
452–454, 460, 663, 670 Enzyme linked immunoelectrotransfer
­epilepticus 262
Down syndrome (trisomy 21) 138, (ELISA) 401
Febrile seizures 247, 265, 287, 297,
388, 412 Enzyme-linked immunoelectrotrans-
320, 330
Dravet syndrome 11, 120, 244, 252, fer blot assay (EITB) 395
Felbamate 191
262, 265, 274, 284, 301, 328 Ependymoma 555, 581, 620
Fencing posture 600
Drug-induced acute dystonia 677 –– in adults 574
Fetal alcohol spectrum (FAS)
DSM V Criteria for Autism Spectrum –– CNS tumor in children 574
disorder 21, 414
Disorder –– intracranial neoplasms 566, 567
FISH 176
–– diagnostic criteria 12, 184 –– in spinal cord 574
FLNa gene 78
–– social communication impairments Epidermal nevus syndrome
Focal cortical dysplasia 113
severity 12, 13 (ENS) 468
Focal epilepsy 300
Duchenne muscular dystrophy Epidural hematoma (EDH) 657,
Food-borne botulism 509
(DMD) 142, 177, 508, 511, 515, 693, 724
Foville syndrome 701
516, 520, 530, 533, 535, 536, 551 Epilepsy syndrome 79, 178, 266, 474
Fragile X syndrome 173, 414, 419
–– early signs of 543 –– EEG patterns 597
Fragile X tremor ataxia syndrome
–– symptoms of 549, 550 –– febrile seizures 320
(FXTAS) 410, 414, 415
Duchenne muscular weakness 540 –– of infancy 319
Friedreich ataxia (FA) 70, 85, 141
Dysembryoplastic neuroepithelial –– management 290
Frontal lobe lesion 593
tumor (DNET) 571, 572, 618 –– risk for 238
Frontal lobe seizures 289, 294,
Dysferlin deficiency 541 –– sports activities 269
301, 330
Dysmorphic features 65 Epileptic encephalopathies 7, 115,
Fronto-polar leads 204
Dystonia musulorum deformans 445 310, 665
Frovatriptan 370
Dystrophic myotonia type 1 (DM1) Epileptic seizures, lateralization signs
Fukuyama congenital muscular
myotonic dystrophy 542 and symptoms 598, 599
dystrophy 506
Dystrophin analysis 142, 526, 535 Epileptiform activity, AMPA and
Functional behavior analysis
Dystrophinopathies 511 glutamate receptors 613
(FBA) 175
Epinephrine 518

E Episodic ataxia 1 (EA1) 461


Episodic ataxia type 2 (EA2) 455, 465
G
Early post-traumatic seizures 704 Epstein-Barr virus (EBV) 578
Eating disorder 186 Erb-Duchenne paralysis 698 Galactosemia 133
Echocardiogram 65 Erb’s palsy 644, 652, 695 Galactose-1-phosphate
Ecstasy 675 Erythromelalgia 362 ­uridyltransferase deficiency 436
Edwards’s syndrome 79 Eslicarbazepine 323 Galactosidase deficiency 112
Ehlers-Danlos syndrome 92 Ethical and policy issues, genetic Galant reflex 600
Electrical status epilepticus of slow testing and screening of Gaucher disease 53, 122
wave sleep (ESES) 7, 325, 608 children 336 –– clinical presentation 440
Electrocardiogram 77, 227 Ethics consultation 286 –– diagnosis 125, 440
Electrodes 292 Ethosuximide 313, 323 –– enzyme replacement with
Electroencephalogram 120 Ewings sarcoma 580 imiglucerase 440
Electromyography (EMG) 338, Exaggerated startle disease 299, 329 –– lysosomal glucocerebrosides 440
526, 527 Excitatory transmitters 613 –– types 427
Index
735 D–H
Gelastic seizures 230 Headache –– HSAN3 532
Generalized epilepsy with febrile –– from cerebral vasculitis 370 –– HSAN4 532
seizures plus (GEFS+) 283, –– Chiari I malformations 346 Hereditary tyrosinemia 679
306, 321 –– in children 359 Herpes simplex encephalitis
Generalized hypotonia 509, –– classification 356 (HSE) 378
512, 538 –– cluster 351 Herpes simplex virus (HSV) 593
Generalized tonic clonic (GTC) –– icepick 354 Heterotopia 78
seizures 301, 309, 327, 599 –– lumbar puncture 357 Hexosaminidase A deficiency 60, 74
–– phenytoin 600, 601 –– mediation-overuse 376 High birth weight 644
Genetic disorders 338 –– of pulsating quality 354 Highly active anti-retroviral therapy
Genetic testing 176 –– topiramate 351 (HAART) 381
Germ cell tumor 574 –– with vomiting 371 Hirschsprung’s disease 43, 64
Germinoma 572, 574, 617 –– in young children 372 –– submucous plexus 594
–– beta HCG, AFP levels 582 Health care provider 339 Holoprosencephaly (HPE) 50, 207
Giant cell neuropathy 133 Hearing loss and ototoxicity 681 Homocysteinuria 60, 432
Gilliam Autism rating scale 9 Hemangioblastomas 557 –– AR disorder 435
Glasgow Coma Scale (GCS) score 694, Hematologic disorders 432 –– collagen metabolism
703, 728 Hematoma 699 ­abnormality 435
Glaucoma 233 Hemiclonic activity 309 –– developmental delay 435
Glial cysts 361 Hemiconvulsive seizures 265 –– intellectual disability 435
Globoid cell leukodystrophy 81, 83 Hemicrania continua 347 Horizontal diplopia 502
–– See also Krabbe disease Hemimegaloencephaly Horner syndrome 697
Globus pallidus 706 (HMC) 40, 131 HPE, see Holoprosencephaly
Glomus tumour 618 Hemiparetic CP 663, 668 HSV encephalitis 397, 619
Glossopharyngeal neuralgia 344 Hemiplegic CP 355, 663, 668 Huffing 686
Glove-and-stocking neuropathy 678 Hemiplegic migraine 345, 360 Hunter syndrome 136, 138, 425, 437
Glucose transporter type I deficiency Hemoglobin S 727 Huntington’s disease 593
syndrome (GLUT1-DS) 11, 115, Hemolytic uremic syndrome 492, Hurler syndrome 425, 438
239, 274, 280, 310, 328, 665 493, 497 Hydranencephaly 35, 707
Glutaric acidemia type 1 449, Hepatic encephalopathy 674 Hydrocephalus 670
450, 458 –– causes 498 Hydromyelia 106
Glycine 306 –– liver transplantation 498 Hyperactive encephalopathy 685
Glycine encephalopathy (non-ketotic –– lorazepam 498 Hyperekplexia 299, 306, 329
hyperglycinemia) 434 –– manganese deposition, basal Hyper-intense regions 66
Glycoproteinoses 128 ganglia 498 Hyperkalemic periodic paralysis 366,
GM1 gangliosidosis 81 –– mannitol/hypertonic saline 528, 532
GM2 gangliosidosis 110, 112, 133 administration 498 Hypermobility 540, 541
Gomori trichrome staining 538 –– pathophysiology 498 Hypertrophic neuropathy of
Gorlin syndrome 470, 475 Hepatolenticular degeneration infancy 525
Gradenigo syndrome 406, 407, 701 (Wilson disease) Hyperventilation (HV) activation 206,
Gray matter diseases 312 –– copper metabolism 427 292, 294, 297
Group A β-hemolytic streptococcal –– lead intoxication in children 427 Hypnogogic and hypnopompic
(GABHS) infection 447 –– neurological symptoms in hallucinations 635
Growing teratoma syndrome 576, children 427 Hypocalcemia 312
577 Hereditary dopa-responsive dystonia Hypoglycemia 292
GTP cyclohydrolase 1-deficient –– clinical presentation 450 Hypokalemia 312
dopa-responsive dystonia –– management 450 Hypokalemic periodic
(GTPCH1) 445 Hereditary hemorrhagic paralysis 528, 532
GTP cyclohydrolase 1 (GCH1) ­telangiectasia 92 Hypomagnesemia 312
mutation 424 Hereditary motor and sensory Hypomelanosis 73
Guillain Barre Syndrome 146, 152, neuropathy type1 (HMSN1), Hypomelanosis of Ito (HI) 483
382, 389, 517, 529, 686 see Charcot-Marie-Tooth 1 (CMT1) Hyponatremia 312, 501
Hereditary neuropathy with liability to Hypoprebetalipoproteinemia 74

H pressure palsies (HNPP) 522


Hereditary sensory and autonomic
Hypothalamic hamartoma 199, 230,
265, 571
Hallervorden-Spatz syndrome 420 neuropathy (HSAN) 679 Hypothalamic-pituitary
Hartnup disease 431 –– HSAN I 532 ­dysfunction 35
Hashimoto encephalopathy (HE) 494 –– HSAN2 532 Hypothermia 293
736 Index

Hypoxanthine-guanine-phosphoribo- Intractable epilepsy 265 Lambert-Eaton myasthenia syndrome


syltransferase (HGPRT) 418 Intractable infantile-onset (LEMS) 502, 521, 538, 539
Hypoxic-ischemic encephalopathy ­epilepsy 280 Laminin a2 (merosin) deficiency 541
(HIE) 666, 668 Intractable seizures 101 Lamotrigine 25, 200, 266, 299, 313
Hypsarrhythmia 197, 204, 216, Intradural extramedullary tumor 656 Landau-Kleffner syndrome (LKS) 20,
226, 327 Intrathecal baclofen therapy 671 209, 242, 269, 411
Intraventricular hemorrhage 661, 710 Landau reflex 600

I
Involuntary movements 458 Language disorder 9
Isotretinoin 178 L-arginine 109
Late-onset spinal muscular atrophy
Iatrogenic botulism 509

J
type 3 or 4 516
Ibuprofen 354
Latex allergy 655
Icute ischemic stroke 711
Lead encephalopathy 676
Idiopathic epilepsy syndrome 640 Jones criteria for rheumatic heart
Lead exposure 674, 675, 681
Idiopathic febrile status epilepti- disease 452
Lead poisoning 681
cus 289 Joubert syndrome 97
Leber congenital amaurosis (LCA) 71,
Idiopathic generalized epilepsy (IGE) Juvenile absence epilepsy (JAE) 211,
104, 589
syndrome 301, 324, 330 214, 323
Leber’s hereditary optic neuropathy
Idiopathic increased intracranial Juvenile Batten disease 55
(LHON) 586
pressure 344 Juvenile dermatomyositis 518
Left hemispheric hemimegalenceph-
Idiopathic intracranial hypertension Juvenile myoclonic epilepsy
aly 84, 130
(llH) 374 (JME) 120, 190, 201, 218, 244, 250,
Left homonymous hemianopia 718
Idiopathic photosensitive occipital 282, 290, 295, 296, 298, 309,
Leigh syndrome 41, 88, 109
lobe epilepsies 241 332, 335
Lennox Gastaut syndrome (LGS) 191,
Immediate Post-Concussion Juvenile neuronal ceroid
193, 209, 215, 253, 263, 282, 295,
Assessment and Cognitive Testing ­lipofuscinosis 120
296, 302
(ImPACT) assessment tool 696 Juvenile pilocytic astrocytoma 561
Leptomeningeal capillary-venous
Infantile botulism 504, 509, 513, 521 Juvenile progressive bulbar palsy 72
malformation 488
Infantile GM1 gangliosidosis 77 Juvenile Tay-Sach’s disease 121
Lesch-Nyhan syndrome (LNS) 135, 421
Infantile myotubular myopathy 548
–– clinical presentation 418
K
Infantile Refsum disease 72
–– in males 457
Infantile spasms (IS) 223, 233, 256,
Levetiracetam 201
314, 322, 456
Kava kava 674 Limb-girdle muscular
Infants, developmental stages 607
Kayser-Fleischer rings 441, 442, 456 dystrophy 132, 667
Inflammatory demyelinating
KCNQ2 277 Lipodystrophy 137
lesions 647
Kearns-Sayre syndrome (KSS) 68, 71, Lissencephaly 37, 65, 91, 101, 549
Informal testing of hearing frequen-
77, 83, 100, 105, 589 –– type I 99
cies 592
Kernicterus 38 Lithium 369
Integrin protein 645
Ketogenic diet 266, 267, 665 Liver enzymes 511
Intellectual disability 183
King-Devick Test 696 Liver function tests 54
Intermittent low back pain 645
Kinky hair syndrome 472 Liver transplantation 493
Intermittent photic stimulation
Klein-Levin syndrome 625 LKS, see Landau-Kleffner syndrome
(IPS) 203
Klippel-Feil syndrome 648 LMNA-related CMD (L-CMD) 506
International Classification of Sleep
Klumpke palsy 695 Lomustine 682
Disorders (ISCD-2) 627
Klüver-Bucy syndrome, amygdale 613 Lorazepam 251
International Headache Society
Krabbe disease 39, 52, 76, 81, 83, 105, Lower brachial Plexus roots 696
criteria 351
121, 124, 132 Lumbar puncture 357, 391, 392,
International League Against Epilepsy
–– See also Globoid cell 658, 700
(ILAE) 251
­leukodystrophy Lyme disease
Intracerebral hemorrhage (ICH)
Krebs cycle 430 –– clinical manifestations 385
score 728
Kugelberg-Welander –– clinical scenario 387
Intracranial hemorrhage 350, 620
syndrome 516 –– erythematous rash 385
Intracranial hypertension,
–– facial nerves involvement 389
­corticosteroids 573
Intracranial teratomas, congenital
brain tumors 576
L –– neurologic presentations 402, 403
Lyme meningitis 390
Lactic acidosis 365 Lymphocytic choriomeningitis
Intractable Childhood Epilepsy with
Lafora body disease 120, 304, 307 virus 379
Generalized Tonic Clonic seizures
Lambda waves 234, 293 Lysosomal storage diseases 303
(ICEGTC) 247
Index
737 H–N

M Methylphenidate/amphetamine 24
Mexiletine 529, 536
Moyamoya disease 471, 712–714,
717, 722, 726
Macrocephaly 84, 100, 131 Microcephaly 25, 84, 100, 130 MPS, see Mucopolysaccharidoses
Macular/central scotoma 586 Migraine MPSI, see Migrating partial seizures of
Maffucci syndrome 478 –– acute treatment 358 infancy
Magnetoencephalogram 202 –– in adults 363 Mucopolysaccharidoses (MPS) 46,
Maple syrup urine disease 312, 427 –– basilar 347 125, 136, 425, 437, 438
–– enzyme branched-chain keto acid –– family history 343 Multiple sclerosis 618
dehydrogenase 434 –– headaches 363, 366, 375, 670, 694 Mu rhythm 291
Marfan syndrome, FBN1 gene –– hemiplegic 345 Muscle biopsy 365
encoding fibrillin 422, 423 –– pathophysiology 368 Muscle eye brain (MEB) disease 505,
Mastoiditis 349 Migraine equivalent syndromes 342 506, 534, 550
Maternal alcohol intake 183 Migrating partial seizures of infancy Muscle fiber necrosis 551
Maternal chromosome 1Sq-q13 (MPSI) 245, 246 Muscle tension 372
deletion 69 Mild hyponatremia 669 Muscular dystrophy 549
Maternal diabetes 606 Mild thoracolumbar scoliosis 649 MuSK antibody 645
Maternal uniparental disomy 414 Mild-to-moderate hypoxic ischemic Mutations of ALD gene 89
McArdle’s disease 530, 531, 550 injury 709 Myasthenia gravis 539, 645
McDonald Criteria 159 Mild traumatic brain injury 689 Mycobacterium tuberculosis
Mcleod syndrome 74 Millard-Gubler syndrome 701 PCR 379, 380
Medication overuse headache 345 Miller-Dieker syndrome Myelin basic protein 526
Medulloblastoma 554–556, 578 (MDS) 55, 91, 101 Myelomeningocele (spinal dysra-
–– posterior fossa 570–571 Miller-Fisher variant of Guillain phism/rachischisis) 597
Megalencephaly 131 Barre 463 Myeloschisis 658
MELAS, see Mitochondrial encepha- Minocycline 360 Myenteric plexus 64
lopathy, lactic acidosis and strokes Mitochondrial disorders 60, 173, 434 Myoadenylate deaminase
(MELAS) Mitochondrial DNA, cytochrome b ­deficiency 530
Meningeom im Spinalkanal MRT 656 deficiency 426 Myoclonic astatic epilepsy, see Doose
Meningiomas 656 Mitochondrial syndrome
Meningitis, enterovirus 385, 386 ­encephalomyopathy 94 Myoclonic-atonic seizures 266
Menkes disease 7, 421, 472 Mitochondrial encephalopathy 302 Myoclonic epilepsy with ragged red
Mental retardation 180 Mitochondrial encephalopathy, fibers (MERFF) 104, 439
–– cerebellar ataxia 410 lactic acidosis and strokes Myoclonic jerks 287
–– learning difficulties 410 (MELAS) 108, 722 Myoclonic seizures 312
–– pre-mutation carriers 410 Mitochondrial myopathies 518, 529 Myoclonus 254, 459
Merosin positive congenital muscular Mitochondrial phenotypes 174 Myoclonus-dystonia (M-D) 445
dystrophy 506 Mixed flexor extensor spasms 322 Myophosphorylase deficiency
Merosin-deficient congenital Mixed germ cell tumors 573, 574 (McArdle disease) 422, 531
muscular dystrophy MLD, see Metachromatic Myotonia congenita 518, 528, 536
(MDCMD) 511, 512 ­leukodystrophy Myotonic disorders 515
MERRF, see Myoclonic epilepsy with Moderate mental retardation 65 Myotonic dystrophy 514
ragged red fibers Modified Checklist for Autism in –– type 1 524, 542, 543
Mesial temporal lobe epilepsy 294 Toddlers 2, 8 –– type 2 524, 542
Metachromatic leukodystrophy Moebius syndrome 31 Myotubular myopathy, see Centro-
(MLD) 49, 86, 98, 128, 132, Molar tooth sign 30, 82, 97 nuclear myopathy
614, 615 5p Monosomy 79

N
Methemoglobin 720 Moro reflex 600
Methotrexate 682 Mother’s headaches 349
Methylation analysis 414 Motion sickness 367
N-acetyl aspartic acid (NAA) 58, 119
Methylenetetrahydrofolate reductase Motor neuron disease 404
Naloxone 677
(MTHFR) deficiency 432 Motor/sensorimotor
Narcolepsy 630, 634, 635, 638,
Methylmalonic acidemia 430 ­neuropathies 681
639, 641
–– adenosylcobalamin 430 Motor tics 459, 460
–– with cataplexy 636, 637
–– diagnosis 431 Movement disorders
–– hypocretin/orexin 601
–– extended newborn screening 431 –– CBIT 444
Nemaline myopathy 538
–– D-methylmalonyl-CoA mutase –– focal motor seizures 458
Neonatal encephalopathy 666
deficiency 430 –– frontal/temporal lobe focal motor
Neonatal herpes simplex
–– treatment 431 seizures 458
­encephalitis 384
738 Index

Neonatal hypocalcemia 259 Neuromyelitis optica (NMO) spectrum Optic nerve glioma 567, 568
Nerve conduction studies 149 diseases 152, 160, 655 Optic nerve hypoplasia 35
Nervus intermedius neuralgia 344 Neuronal ceroid lipofuscinosis Optic nerve pallor 153
Neural crest cells, autonomic (NCL) 89, 113, 118 Optic neuritis 147, 156
ganglia 594, 595 Neuronal membrane channels, Optic Neuritis Treatment Trial
Neural tube closure, neurological ligand-gating 612 (ONTT) 147
abnormalities 597 Neuronal migration, astrocytes 604 Optic pathway gliomas
Neural tube defects (NTD) 596, 597 Neuronal proliferation disorder 113 (OPGs) 587
–– anencephaly and Neuro-ophthalmology –– incidence 563
­encephalocele 606 –– cranial nerve trauma at birth 588 Oral contraceptives 266
–– prenatal ultrasonography 605 –– third nerve palsy 588 Orbital apex syndrome 726
–– risk factors 606 Neuropathy, ataxia and retinitis Orbital sarcoidosis 568
Neuro-cutaneous syndromes pigmentosa (NARP) 68 Ornithine transcarbamylase (OTC)
(phakomatoses) 66, 474 Nevoid basal cell carcinoma 470 deficiency 126, 423, 429, 430, 435,
Neuro-cysticercosis (NCC) 383, 395 Newborn’s sleep 638 436, 441
–– in children 400, 401 Niemann-Pick type A 85 Osler-Weber-Rendu syndrome, see
Neurodegeneration with brain iron Niemann-Pick Type C 99 Hereditary hemorrhagic
accumulation (NBIA) 127 Nightmares 637, 638 telangiectasia
Neurodevelopmental Effects of Night terrors 249, 624, 628 Ototoxicity, cisplatin chemother-
Antiepileptic Drugs (NEAD) NIH Stroke Scale 718 apy 499, 500
study 182 Nimodipine 692 Oxcarbamazepine 200
Neurofibromatosis type 1 NMDAR antibodies 157 Oxcarbazepine 680, 683, 685
(NF1) 468–470, 587 NMDAR encephalitis 157

P
–– age-related manifestations 486 NMO, see Neuromyelitis optica
–– Cafe au lait spots 483, 486 Nocturnal seizures 270
–– cutaneous lesions 474 Non-accidental head injury
PAFAH1B1 55
–– ependymomas 484 (NAHI) 689, 692
Pain 696
–– epidermal nevus syndrome 483 Non-contract CT 368
Palatal myoclonus, segmental
–– genetics 484 Nongerminomatous germ cell
­myoclonus 593
–– hyperpigmented lesions 482 tumor 576
Pallid breath-holding spells 285
–– Lisch nodules 486 Non-rapid eye movements (NREM)
Palmitoyl protein thioesterase 1
–– macrocephaly 471 sleep 627
(PPT1) 113
–– molecular biology 484 Non-REM parasomnia 635, 636
Panayiotopoulos syndrome 195, 196,
–– neurofibromin 485 Non-verbal learning disorder
231, 242, 251, 258, 282, 285, 309
–– nonspecific subcortical hyper- (NVLD) 23
Pantothenate kinase-associated
intensities 484 Noxious stimulation, EEG recording 595
neurodegeneration (PKAN) 64,
–– optic nerve gliomas 484 Nutritional deficiencies 491
127, 420, 445
–– pheochromocytoma 471
Papilledema 490

O
–– schwannomas 484
Paradoxical myotonia 514
–– scoliosis 471
Paramyotonia congenita 367,
–– sphenoid wing dysplasia 471
Occipital encephaloceles 606 514, 528
Neurofibromatosis type 2
Occult spinal dysraphism 657 Paraneoplastic myelitis 655
(NF2) 568–570
Occupational therapy 10 Parasomnias 627, 629
–– bilateral schwannomas of cranial
Ocular alignment 587, 588 Parenchymal cord lesion 656
nerve VIII 474
Ohtahara syndrome 198 Parietal sharp waves 293
–– clinical manifestations 474
Oligodendrogliomas 579, 616 Parkinson’s disease 593
–– CNS tumors 474
Opiates 355, 677 Paroxysmal events 239
–– diagnostic criteria 474
Opioids 373 Paroxysmal exertional
–– merlin 475
Oppositional defiant disorder 18, 21 ­dyskinesias 462
–– pathophysiology 474
Opsoclonus-myoclonus-ataxia (OMA) Paroxysmal hypnogenic
–– unilateral CN VIII schwannoma 474
syndrome ­dyskinesias 455, 462
–– vestibular schwannomas 487
–– clinical presentation 555, 556 Paroxysmal kinesigenic dyskinesias
Neuroleptic malignant syn-
–– neuroblastoma 557 (PKOs) 455, 462
drome 676, 678
–– paraneoplastic syndrome 557 –– anticonvulsants 462
Neuroleptics side effects 23
Opsoclonus myoclonus syndrome –– clinical presentation 462
Neurological dysfunction 35,
(OMS) 565, 566 –– familial/sporadic 462
126, 152
–– clinical presentation 492 –– sudden movement 462
Index
739 N–R
Paroxysmal movement disorder 239 Pineoblastoma 574 Pseudo tumor cerebri 369, 374, 568
Paroxysmal non-epileptic spells 279 Pituitary apoplexy 571 Psychiatric evaluation 343
Paroxysmal non-kinesigenic PKAN, see Pantothenate kinase- Psychogenic headaches 372
dyskinesia (PNKD) 455, 462 associated neurodegeneration Psychogenic movement disorders
Parry-Romberg syndrome 478 PKU, see Phenylketonuria –– psychological intervention 464
Partial monosomy 13q 79 Placenta 365 –– treatment 464
Passive range of motion 644 Plasmapheresis 493 Psychogenic seizures 26, 325
Patau syndrome 79, 138 Plexiform neurofibromas 469 Psychomotor retardation 81
Paternal chromosome 1Sq11–13 PMP22 64, 523, 525 PTEN 39
deletion 69 Poliomyelitis 404, 545, 651 Pyramidal system, limb
Pavor nocturnus 624, 628 Polymicrogyria 36, 73, 97, 98 hypotonia 613
PCR 392, 393 Pompe’s disease 42, 504, 514 Pyridoxine 678
PDD Assessment scale/ screening Ponto-bulbar Palsy with Deafness 72 Pyridoxine-dependent epilepsy 261,
questionnaire 2 Porencephaly 97 276
Pediatric genetic epilepsy 243 Posterior dominant alpha rhythm 291 Pyruvate dehydrogenase (PDH)
Pediatric neurologists 278 Posterior fossa tumors 556 deficiency 140
Pelizaeus-Merzbacher disease 82 Posterior reversible encephalopathy 6-Pyruvoyl tetrahydropterin synthase
Perampanel 273 syndrome (PRES) 500, 501, 717 (PTS) 424
Perinatal asphyxia 295, 666 Postictal paralysis 313

Q
Periodic hyperkalemic paralysis 537 Post-immunization encephalopa-
Periodic lateralizing epileptiform thy 303
discharges (PLEDS) 391, 593, 594 Posttraumatic stress disorder 289
Quadriparetic CP 663, 668
Periodic migrainous neuralgia 344 Post-traumatic transient cortical
Quadriplegia 665
Peripheral myelin protein 22 (PMP22) blindness 690
Quetiapine fumarate (Seroquel®) 677
gene 525, 547 Postural dystonia 454
Quinoid dihydropteridine reductase
Peripheral nerve hypertrophy 525 Prader Willi Syndrome (PWS) 46, 414
(QDPR) gene mutations 424
Peripheral neuritis 382, 676 Pregnancy
Peripheral neuropathy 510 –– chorioamnionitis 396
Periventricular leukomalacia 669
Periventricular nodular
–– GBS bacteriuria 396
–– premature rupture,
R
­heterotopia 78, 113 membranes 396 Ramsay-Hunt syndrome 407
Peroxisomal biogenesis disorders Preinjury cognitive assessment 696 Rapid Anticonvulsant Medication
(PBD) 56, 124 Prematurity 662 Prior to Arrival Trial (RAMPART)
Peroxisomes 103, 118, 124 Prenatal exposure to alcohol 414 study 268
Pervasive developmental disorder Prenatal ischemia 35 Rasmussen’s encephalitis 406
(PDD) 667 Pre-optic nucleus 608, 609 Rebound headaches 357
Petechial hemorrhages 702 Primary brainstem gliomas 701 Reflex sympathetic dystrophy 362
Phelan-Mc Dermid Syndrome-22q13 Primary CNS lymphoma (PCNSL) 578 Refractory epilepsy 271
deletion syndrome 45 Primary CNS vasculitis 711 Refractory infantile spasms 13
Phenobarbital works 324 Primary generalized epilepsy REM atonia 630
Phenylketonuria 53, 131, 180, (PGE) 315 REM sleep 635, 639
431, 434 Prion diseases 312 REM sleep behavior disorder
–– phenylalanine hydroxylase Progressive cerebellar atrophy 454, (RBD) 632
gene 424 455, 464, 465 Renal artery stenosis due to renal
Phenytoin 178, 327 –– on neuroimaging 455 artery dysplasia 471
Phenytoin approaches zero-order Progressive cognitive and motor Repetitive soothing/stimulating
kinetics 601 impairment, phenylketonuria 457 behaviors 596
Phophofructokinase deficiency 535 Progressive HIV encephalopathy 380 Research Society of Alcoholism Fetal
Phosphoglycerate mutase –– anti-retroviral therapy 381 Alcohol study 21
­deficiency 534 Progressive multifocal leukoencepha- Restless leg syndrome (RLS) 624
Photic stimulation (PS) 296, 595 lopathy (PML) 381 –– in childhood 626
Photomyogenic response 296 Progressive subacute ataxia 554 –– diagnosis and epidemiology
Pilocytic astrocytoma 555 Prophylactic therapy 372 626, 627
Pineal parenchymal tumor 574 Propionic acidemia 432 Retigabine 270
Pineal region tumors Protein C deficiency 722 Retinal disease 588
–– histopathology 617 Pseudohypertrophic muscular Retinal hemorrhages 692
–– prognosis 572 dystrophy 519 Retinal vein occlusion 721
740 Index

Retinitis pigmentosa 426 Selective serotonin re-uptake Sodium channel (SCN)


Retinoblastoma 104, 588, 589 inhibitors (SSRIs) 676, 678 subunits 306, 612
Retrograde degeneration 612 Selegiline 675 Sonic hedgehog (SHH) protein 207
Rett syndrome 6, 81, 139, 412, 413, Sensorineural hearing deficits Sotos syndrome 19, 181
419 (deafness) 72, 592, 698 Spasmus nutans 456, 457
Reversible cerebral vasoconstriction Sensory distortions 249 Spastic diparesis 380
syndrome 714 Septic cavernous sinus Spasticity, internal capsule 613
Reversible posterior leukoencepha- thrombophlebitis 718 Speech/language therapy 10, 181
lopathy syndrome 616, 617 Septo-optic dysplasia 35, 95, 102, Sphingomyelinase deficiency 110
Reye syndrome 679 114, 603 Spina bifida 597, 612
Rhabdomyosarcoma 568 Serious atherosclerosis formation 722 –– severe form of 658
Rhythmic myoclonus 459 Serotonin syndrome 675, 676, 678 –– physical complications 95, 655
Right facial nerve lesion 589 Serum prolactin levels 315 Spina bifida occulta 80, 657, 658
Right maxillary bone fracture 697 Serum toxicology 280 Spinal dysraphism 80
Right temporal bone impact or Severe myoclonic epilepsy of infancy Spinal muscular atrophy
fracture 697 (SMEI) 179, 243, 274, 301, 328 (SMA) 544, 545
Rinne test, bone conduction 592 –– See also Dravet syndrome –– PMP2 471
Rizatriptan 373 Sexual abuse 293 –– SMN1 471, 544
Rolandic epilepsy 273 Shaken baby syndrome (SBS) 688, –– SMN3 527, 533
Roussy-Levy syndrome 547 689, 691 Spindle delta brushes 293
Routine neuroimaging 346 Shaken infant syndrome (Non- Spinocerebellar ataxia type
Rubinstein-Taybi syndrome 475 accidental trauma) 691 6\”diagnosis 616
Rufinamide 269 SHANK3 45 Standardized Assessment of
Ruptured intracranial aneurysms 692 Shoulder dystocia 644 Concussion (SAC) 696
Sialidosis 128, 439, 440 Staphylococcus aureus 401, 402

S
–– type 1 A 74, 438 Statins 505
Sickle cell anemia 499, 713, 723 Status epilepticus 268, 288
Sickle hemoglobin 499 Status migrainosus 375
Sacral agenesis 596
Simple febrile seizures (FS) 313, Stimulants 24
Sagital Sinus 706
318, 327 Stressful events 27
Sandhoff disease 110
SIRPIDs 597 Sturge-Weber syndrome (SWS) 143,
Sandifer syndrome 253
SLC2A1 277 233, 481, 483, 488, 496, 716
Sanfilippo syndrome 47, 125, 425,
Sleep deprivation 595 –– clinical findings 473, 479, 481
667
–– EEG 596 –– facial port wine stains 486
Sarcoglycan deficiency 541
Sleep disorders 629 –– neurologic manifestations 481
Sarcoidosis 389
–– adenotonsillectomy 624, 625 –– porencephalic cyst 473
Scheie syndrome 425, 437
–– differential diagnosis 628 –– port-wine stain 473
Schindler disease 73
–– ferritin 624 –– spinal dural angioma 481
–– alpha-N-acetylgalactosaminidase
–– frontal lobe seizure Subacute encephalopathy 490, 491
deficiency 438
differentiation 633 Subacute necrotizing encephalomy-
Schizencephaly 32–34, 67, 97, 98, 661
–– stage 3 and 4 sleep peaks during elopathy 133
Schwannomatosis 475
adolescence 633 Subacute neuropsychiatric syn-
SCN1A 274
–– treatment 627, 628 drome 22
Scoliosis progression 646, 653
Sleepiness 271 Subacute regression 242
Scurvy 177
Sleep pattern 229 Subarachnoid hemorrhage
S1 dorsal root ganglia 621
Sleep phenomena 631 (SAH) 692, 720
Segawa disease 448
Sleep, stages 595 Subcortical band heterotopia 99
Seizure 26, 30, 31, 33, 36, 102, 324,
Sleep terrors 627–629, 633 Subdural hematomas 691
541, 619
Sleepwalking 627, 632, 636 Subdural hemorrhages 699
–– absence 308
SLOS, see Smith-Lemli-Opitz Subependymal giant cell astrocytoma
–– activity 614
syndrome (SEGA) 66, 486, 559
–– cessation of 305
Small capillary hemangioma 649 –– diagnosis 480
–– disorder 619
Small cell lung cancer (SCLC) 539 –– MRI 480
–– febrile 247
Small ring-enhancing lesion 699, 700 Sudden discontinuation of neurolep-
–– semiology 281
Smith-Lemli-Opitz syndrome tics 450
–– threshold 684
(SLOS) 123 Sudden unexplained death in
–– types 241
epilepsy (SUDEP) 275
Index
741 R–W
SUDEP, see Sudden unexplained death Transient neonatal myasthenia gravis Upper plexus injury 644
in epilepsy (TNMG) 512 Urea cycle disorders (UCDs) 51, 126,
Sudomotor axon reflex test 352 Transient tic disorder 445, 446, 429, 430
Suicidal ideation 27 451, 452 Uridine diphosphate glucuronosyl-
Sumatriptan 373 Transient visual loss 371 transferase (UGT) 266
Superior orbital fissure syndrome 726 Transitory myasthenic syndrome 545 Urine CMV culture 383, 384
Supportive care 313 Transverse myelitis (TM) 647, Usher syndrome 68
Surveillance of Cerebral Palsy in 655, 658 Uvulopalatopharyngoplasty 640
Europe (SCPE) 664 Traumatic brain injury 608
Survival motor neuron (SMN) Treacher Collins Syndrome 173
region 516, 526
Suspended sensory loss 645
Treximet 370
Trigeminal dysesthesia 344
V
Sydenham chorea 446, 452 Trigeminal neuralgia 344 Vagal nerve stimulation (VNS) 283
–– in childhood 458 Tripeptidyl peptidase 1 (TTP1) 89 Valproate (VPA) 182, 201, 332
–– etiology 458 Triphasic waves 597 Valproic acid 121, 296, 313, 369, 680
–– group A beta Hemolytic 453 Trisomy 13 79, 138 Varicella-zoster virus (VZV) 716
–– Jones criteria for rheumatic Trisomy 18 79, 138 Vascular stenosis 713
fever 453 Trisomy 21 79 Vasculitis 679
Synaptic vesicle protein Tuberculomas 380 Vein of Galen malformation
(SV2A) 315, 603 Tuberous sclerosis (TS) 62, 67, 388, (VGM) 719, 720, 725
Syringomyelia 106, 645 559, 618 Velocardiofacial syndrome 69
–– ash-leaf hypo-pigmented Venlafaxine 684
Venous sinus thrombosis 724, 725
T
macules 485
–– cerebral cortical tubers 473 Ventriculo-peritoneal shunting 614
–– Confetti macules 485 Very long chain fatty acid
Tangier disease 107 (VLCFA) 134
–– retinal astrocytoma 488
Tardive dyskinesia, dopamine Vestibular neuritis 407
–– sub-ependymal nodules 473
receptor 457 Vestibular schwannomas 560, 561,
Tuberous sclerosis complex (TSC) 62,
Tay-Sachs disease 57, 77, 81, 110, 439 577, 578
469, 471, 472, 475, 559, 570
TEACCH program 10 Vigabatrin (VGB) 190, 216, 255,
–– angiokeratomas 476
Temporal lobe epilepsy 265, 304 268, 322
–– ash leaf spots 477
Temporal rhythmic delta activity Vincristine 681
–– flexor/extensor myoclonus 477
(TIRDA) 597 Viral encephalitis 387
–– genetics 480
Temporal sharp waves 293 Viral meningitis 394
–– hamartomas 477
Tendon reflexes 644 Vitamin B12-responsive methylmalo-
–– hypo-pigmented streaks or
Tethered cord syndrome (TCS) 646, nic academia (MA) 59
patches 478
649, 653, 654 Vitamin D deficiency 122
–– periungual fibroma 476
Thomsen’s disease 518, 536 Voltage-gated calcium channels
–– subungual fibromas 476
Thyroid hormone test 490 (VGCCs) 539
–– TSC1 gene 581
Thyroid ophthalmopathy 501, 502 von Hippel Lindau disease 479, 557
–– TSC2 gene 581
Tinnitus 592
Tympanometry with pneumatic
Tissue plasminogen activator, in
testing 592
children 499
Tizanidine 671
Type 1 Arnold-Chiari
malformation 651
W
Todd paralysis 313, 479 Walker-Warburg syndrome 45, 534,
Type 1 Chairi malformation 32
Tonic neck reflex 600 550
Type I sialidosis 130
Tonic seizure 262 Weber test, bone conduction 592
Topiramate 191, 315, 369, 374 Welander myopathy 534
Torsion dystonia (DYT1 dystonia) 445
Total plexus injury 644
U Werdnig-Hoffmann disease 516
–– See also Spinal muscular atrophy
Tourette syndrome (TS) 6, 446, UCD, see Urea cycle disorders (SMA) type 1
447, 451 Ullrich’s congenital muscular West Nile virus infection 390
–– diagnosis 448 dystrophy 541, 550 West syndrome 179, 243, 264, 267,
–– pharmacotherapy 448 Uncal herniation 723 312, 317
–– pimozide 453 Unilateral deafness, cranial nerve White matter function disorder 421
Trace discontinu 277 VIII 613 Wicket spikes 597
Transient motor tics of childhood 459 Unilateral headaches 356 William’s syndrome 69, 410
742 Index

Wilson’s disease 61, 303, 429, 438, 456


Wolfram syndrome 71
Xeroderma pigmentosum (XP) 93
X-linked adrenoleukodystrophy Z
Wonderland syndrome 348 (X-ALD) 44 Zellweger syndrome 43, 56, 103, 118,
Wyburn-Mason syndrome 92 X-linked lissencephaly 91 124
Zolmitripan (Zomig) 370

X Y Zonisamide 296, 322

Xanthochromia 720 Yolk sac tumors 573, 582

You might also like